Anda di halaman 1dari 1072

ALBANO BAR REVIEW CENTER

PRIVATE CORPORATIONS,
SECURITIES REGULATION,
BANKING AND RELATED LAWS

by: ATTY. MARIA LULU G. REYES

THE CORPORATION CODE OF THE PHILIPPINES


(BP Blg. 68)
I. FORMATION and ORGANIZATION of CORPORATIONS
A. General Principles
Attributes Of A Corporation
Petron v. NCBA
APT v. CA
Mambulao Lumber v. PNB
Hanil v. CA
Bache and Co. V. Ruiz
Sulo ng bayan v. Araneta
B. Classification of Corporations
Private v. Public Corporation
Boy Scout of the Phils. v. COA
Liban v. Gordon
Baluyot v. Holganza
Veterans Federation of the Phils. v. Reyes
Special Charter Corporations
MIA v. CA
C. Stages in the Formation/ Organization of a corporation
Promotion
March II Marketing vs. Joson
Cagayan Fishing v. Sandiko
Caram v. CA
Pioneer Insurance v. CA
Rizal Light v. Municipality of Morong
D. Articles of Incorporation
Lanuza v. CA
Corporate Name
Alonso v. Cebu
Industrial Refractories v. CA
Ang Mga Kaanib sa Iglesia ng Dios v. Iglesia
Universal Mills v. Universal Textile Mills
Lyceum of the Phils. v. CA
Indiana Aerospace University v. CHED
Philips Export BV v. CA
Primary Purpose
Gala v. Ellice
Heirs of Pael v. CA
Uy Sulong v. Director

Page 2 of 1072

Asuncion v. De Yriarte
Principal Office/ Domicile
Davao Light and Power Co. v. CA
Clavecilla Radio System v. Antillon
Sy v. Tyson Enterprises
Young Auto Supply v. CA
Term
Alhambra Cigar and Cigarette Mfg. v. SEC
Paid up Capital Stock
MISCI-NACUSIP Local Chapter v. NWPC
Classification of shares
San Miguel Corp. v. Sandiganbayan
Amendment and/ or rejection of Articles of Incorporation
Republic Planters Bank v. CA
E. Doctrine of Corporate entity vs. Piercing the Veil of corporate
Fiction
Ramirez vs. Mar Fishing, Inc.
Sarona vs. NLRC
Gold Line Tours vs. Heirs of Lacsa
Hacienda Luisita vs. PARC
Pantranco Employees v. NLRC
Cagayan valley Drug v. CIR
Heirs of Pajarillo v. CA
Petron v. NLRC
China Banking v. Dyne-Sem
Executive Sec. v. CA
Re: transport v. Latag
Heirs of De Leon v. CA
Velarde v. Lopez
PNB v. Ritratto
Booc v. Bantuas
Marubeni v. Lirag
Francisco v. Mejia
Landbank v. CA
PNB v. Andrada Electric
AZCOR v. NLRC
Claparols v. CIR
CIR v. Norton and Harrison
Concept Builders v. NLRC
Complex Electronics v. NLRC
Cordon v. Balicanta

Page 3 of 1072

Delpher Trades v. IAC


Del Rosario v. NLRC
First International Bank v. CA
Francisco Motors v. CA
Laguio v. NLRC
Li m v. CA
Malayang Samahan v. Ramos
Matuguina Integrated v. CA
Manila Hotel Corp. v. NLRC
Norton and Harrison v. Collector
Reynoso v. CA
San Juan Structural v. CA
Santos v. NLRC
Tan Boon Bee v. Jarencio
TRB V. CA
Telephone Engg v. WCC
Umali v. CA
Vlason enterprises v. CA
Villa Rey Transit v. Ferrer
Yu v. NLRC
F. De Facto Corporation
Hall v. Piccio
G. Corporation by estoppels
International Express v. CA
Lim Tong v. PFGI, Inc.
Albert v. University Publishing
H. Non- User of Charter vs. Continuous Inoperation
Loyola Grand Villas v. CA
II. BOARD OF DIRECTORS
A. Requirements
Qualifications/ Qualifying Share
Villafuerte V. Moreno
Baguio v. CA
Detective and Protective v. Cloribel
Grace Christian HS v. CA
Lee v. CA
Disqualifications
Brias V. Hord
B. Election
Voting
Aurbach v. Sanitary Wares

Page 4 of 1072

Bataan Shipyard v. PCGG


C. Report on Election
Premium Marble v. CA
D. Term of Office/Holdover
Seneres v. COMELEC
E. How Removed
Lambert v. Fox
F. How Vacancy Filled
Valle Verde Country Club v. Africa
G. How Compensated
Singson v. COA
Western Institute v. Salas
Central Coop. Exchange v. tibe
Lingayen Gulf v. Baltazar
H. Authority of the Board of Directors
La Bugaal v. Ramos
Shipside v. CA
ABS-CBN v. CA
Asset Privatization Trust v. CA
BA Savings Bank v. Sia
Montelibano v. Bacolod Murcia
Powers v. Marshall
Premium Marble v. CA
Ramirez v. Orientalist
I. Delegation of Authority to Corporate Officers
Corporate officers/ Meaning of Office vis-a-vis
Employment
Real v. Sangu
Matling v. Coros
Manila Metal v. PNB
Ongkiko v. NLRC
Lao v. CA
De Tavera v. Phil. Tuberculosis Society
Minimum Set of Officers; Qualifications and
Disqualifications; Authority and Liabilities
Matling v. Coros
Okol vs. Slimmers
Gomez vs. PNOC
E.B. Villarosa and Partners v. Benito
SSPC v. Bardaje
Cagayan Valley Drug v. CIR
Banate, et.al. v. Philippine Countryside Rural bank

Page 5 of 1072

Millennium industrial v. Tan


Pabon v. NLRC
Vlason Enterprises v. CA
Prime White Cement v. IAC
Louis Vuitton SA v. Villanueva
Doctrine of Apparent Authority
Sargasso v. PPA
Acuna v. Batac province
Board of Liquidators v. Kalaw
Francisco v. GSIS
New durawood v. CA
Peoples Aircargo v. CA
San Juan Structural v. CA
Rural Bank v. Oclemia
J. Three-fold Duties of Directors and Officers: Diligence, Loyalty
and Obedience
Personal Liability of directors and other Corporate
officers
Ever Electrical vs. Samahang Manggagawa
Cojuangco v. republic
Ty v. NBI
Queensland-Tokyo Commodities v. George
Wensha Spa center v. Yung
Cebu Mactan v. Masahiro
David v. National Federation of Labor unions
Soriano v. People, BSP and PDIC
Cebu Country Club v. Elizaque
Caltex Inc. V. NLRC
Atrium Management v, CA
ARB construction v. CA
Lim v. CA
Francisco v. Mejia
DBP v. CA
AHS Phils. v. CA
Complex Electronics v. NLRC
Crisologo -Jose v. CA
FCY Construction v. CA
Llamado v. CA
MAM realty Development v. NLRC
Naguiat v. NLRC

Page 6 of 1072

Progress Homes v. NLRC


REAHS Corp. v. NLRC
Santos v. NLRC
Sia v. People
Tramat Mercantile v. CA
Disloyalty
Gokongwei Jr. V. SEC
Strong v. Repide
Watered Stocks
Lirag Textile Mills v. SSS
Nava v. Peers Marketing
Self- Dealing Director/ Officer
Mead v. Mc Cullough
Prime White Cement v. IAC
Contracts between Corporations with Interlocking
Directors
Palting v. San Jose Petroleum
DBP v. CA
Derivative Suit: Remedies to Enforce Personal Liability
Legazpi Towers vs. Muer Et al.
Lisam Enterprise vs. BDO
STRADEC v. Radstock & PNCC
Yu v. Yukayguan
Gochan v. Young
Tam Wing v. Makasira
Lim v. Lim- Yu
Western Institute v. Salas
First International Bank v. Salas
Commart Phils. v. SEC
Chase v. Buencamino
San Miguel Corp. v. Kahn
Everett v. Asia Banking
Gamboa v. Victoriano
Reyes v. Tan
Pascual v. Orozco
III. POWERS OF CORPORATION
A. In General
Theory of Special Capacities v. Theory of General
Capacities
Acebedo Optical v. CA
Express, implied and Incidental Powers
Pilipinas v. SEC

Page 7 of 1072

Luneta Motors v. Santos


Teresa Electric v. PSC
Powers v. Marshall
Power to Have/ Use Corporate Name and Seal
Laureano Investment v. CA
Power to Sue and be Sued
Tam v. Hon. Makasiar
Bitong v. CA
Special Services Corp. v. Centro La paz
R Transport Corp. v. CA
Power to Acquire, Dispose, Encumber Property
Director of Lands v. CA
Power to Make donations
Pirovano v. Dela Rama
B. To Increase or Decrease Capital Stock
Madrigal v. Zamora
Philtrust v. Rivera
C. To Deny Pre-emptive Rights
Datu Benito v. SEC
Dee v. SEC
PCGG v. SEC
Republic v. Sandiganbayan
D. To Sell or Otherwise Dispose of All or Substantially all of
Corporate Assets
PNB v. Andrada Electric
Islamic Directorate v. CA
Edward Nell Co. v. Pacific Farms
Esguerra v. CA
Lopez Realty v. Fontecha
E. To Invest Corporate Funds in Another Corporation of Business
Gokongwei v. SEC
Dela Rama v. Ma-ao Sugar
F. To Acquire Own Shares
Boman Environmental v. CA
Steinberg v. Velasco
G. To Declare Dividends
Kinds: cash, stock, property, scrip
Cojuangco v. republic
Declaration, Payment and record Date
Cojuangco, et.al. v. Sandiganbayan
Limitation on retention of Surplus Profits
Steinberg v. Velasco

Page 8 of 1072

Nielson v. Lepanto
CIR v. Manning
Madrigal v. Zamora
Republic Planters v. Agana
Bitong v. CA
CIR v. CA
H. To Enter Into Management Contract
Aurbach v. Sanitary Wares
PNB v. Producers Warehouse
Nielson and Co. v. Lepanto Mining
Tuason v. Bolanos
I. Ultra Vires Acts
Heirs of Pael v. CA
Pilipinas Loan v. SEC
Crisologo v. CA
Carlos v. Mindoro Sugar
Pirovano v. Dela Rama Steamship
Republic v. Acoje Mining
Republic v. Security Credit
IV. BY-LAWS
A. Function
Nakpil v. IBC
PMI Colleges v. NLRC
Loyola Grand Villas v. CA
Citibank v. Chua
B. When to Adopt and File
Loyola Grand Villas v. CA
C. Contents
Authority to Elect Additional by-laws Officers
Fleischer v. Botica Nolasco
Gokongwei v.SEC
Government v. El Hogar Filipino
D. Amendment and/ or rejection of By- laws
Salafranca v. Philam Life
V. MEETINGS OF STOCKHOLDERS AND THE BOARD OF DIRECTORS
A. Kinds
Pena v. CA
B. Notice Required
Board of Liquidators v. Tan
C. Quorum Required

Page 9 of 1072

Javellana v. Tayo
D. Who Could Attend and Vote
Sales v. SEC
Ponce v. Encarnacion
Lopez v. Ericta
VI.VOTING
A. Who may Exercise
Gamboa v. Teves
COCOFED v. Republic
Republic v. COCOFED
Lee v. CA
Republic v. Sandiganbayan
B. Voting Trust Agreement
Cordon v. Balicanta
NIDC v. Aquino
Lambert v. Fox
VII. CAPITAL STRUCTURE: STOCKS AND STOCKHOLDERS
A. Capital Stock, meaning
Distinguished from Capital
Gamboa v. Teves
Legal or Stated Capital
PLDT v. NTC
Control Test v. Grandfather Rule
Gamboa v. Teves
Agan v. PIATCO
B. Classification of Shares
Voting v. Non- Voting
Gamboa v. Teves
Castillo v. Balinghasay
Sales v. SEC
Redeemable Preferred
Republic Planters Bank v. Agana
Treasury
CIR v, Manning
San Miguel Corp. v. Sandiganbayan
C. Trust Fund Doctrine
National Telecom. V. SEC
Ong v. Tiu
D. What is a Subscription
Ong v. Tiu
Bayla v. Silang Traffic

Page 10 of 1072

Salmon, Dexter and Co. v. Unson


Sunset View Condominium v. Campos
Velasco v. Poizat
E. Acquisition and Ownership of Shares in a Corporation; Extent
of Proprietary Right
Cojuangco v. Republic
Crisostomo v. SEC
Garcia v. Lim
Magsaysay- Labrador v. CA
Nicolas v. CA
Ramos v. CA
Saw v. CA
F. Consideration for Stocks
Apodaca v. NLRC
Fua Cun v. Summers
National Exchange v. Dexter
Nielson and Co. Lepanto Mining
Trillana v. Quezon
G. Collection of Unpaid Subscription
Garcia v. Suarez
PNB v. Bitulok Sawmill
Velasco v. Poizat
Lumanlan v. Cura
Edward Keller v. COB group Marketing
Effect of Delinquency
Valley Golf v. Caram
C alatagan Golf Club v. Clemente Jr.
H. Issuance of the Certificates of Stock
Fua Cun v. Summers
Baltazar v. Lingayen
Tan v. SEC
Embassy Farms v. CA
I. Rights to Transfer of Shares/ validity of restrictions on Right
Fleischer v. Botika Nolasco
Padgett and Bobcock v. Templeton
Rural bank of Salinas v. CA
Thompson v. CA
Yuchengco v. CA
Lim Tay v. CA
J. Transfer of Shares of Stock and Registration
Cojuangco v.Sandiganbayan
Republic v. Sandiganbayan

Page 11 of 1072

Rural Bank of Lipa v. CA


BLTB v. Bitanga
Abejo v. Dela Cruz
Batongbuhay Gold Mines v. CA
Chemphil Export v. CA
Chua Guan v. Samahang Magsasaka
CIR v. Anglo-California Bank
Delos Santos v. Republic
De erquiga v. CA
Garcia v. Jomouad
Lopez v. CA
Monseratt v. Ceron
Puyat De Guzman
Razon v. IAC
Rivera v. Florendo
Santamaria v. Hongkong and Shanghai Bank
Torres v. CA
Won v. Wack-Wack Golf and Country Club
K. Lost or Destroyed Certificates
Philex Mining v. Reyes
VIII. CORPORATE BOOKS AND RECORDS
A. Books to be Kept
Bitong v. CA
B. Inspection of Corporate Books and Records
Sy, et.al. v. Sy
Africa v. PCGG
RP v. Sandiganbayan
Gokongwei v. SEC
Gonzales v. PNB
Pardo v. Hercules lumber
Philpotts v. Philippine Mnufacturing Co.
Republic v. Sandiganbayan
IX. MERGER AND CONSOLIDATION
A. Effects of Merger or Consolidation
BPI v. BPI Employees Union
PNB v. Andrada electric
Babst v. CA
Associated Bank v. CA
Alger Electric v. CA
CIR v. Norton and Harrison

Page 12 of 1072

X. NON- STOCK
A. Purposes

B. Voting

CIR v. Rufino
Solid v. Bio Hong
CORPORATIONS
Chinese YMCA v. Ching
CIR v. Club Filipino
Litonjua v. CA
PPSTA v. Apostol

XI. CLOSE CORPORATION


A. Requirements for Formation
Dulay Enterprises v. CA
San Juan Structural Steel v. CA
Naguiat v. NLRC
XII. SPECIAL CORPORATIONS
A. Religious Corporations
RP v. IAC
Director of Lands v. CA
B. Corporation Aggregate
Iglesia Evangelica v. Bishop Lazaro
XIII. DISSOLUTION OF CORPORATIONS
A. Methods
Vesagas v. CA
Avon dale Garments v. NLRC
Daguhoy Enterprises v. Ponce
PNB v. CF
B. Liquidation
Metropolitan Bank v. Board of Trustees of Riverside
Yam v. CA
Alhambra Cigar v. SEC
Chung Ka Bio v. IAC
Republic v. Marsman
Tan Tiong Bio v. CIR
Reynolds v. CA
Mambulao v. PNB
Catmon Sales v. Liquidator
Knech v. Unites Cigarettes

Page 13 of 1072

Chua v. NLRC
Clemente v. CA
Gelano v. CA
Reburiano v. CA
Republic Planters v. CA

XIV. FOREIGN CORPORATIONS


A. Definition And Rights
Avon v. CA
San Jose Petroleum v. CA
B. Requirements for the Establishment of a Branch/ License to do
Business in the Philippines
GMBH v. Isnani
New York Marine v. CA
C. Amendment of License
Aetna Casualty v. Pacific Star
Bulakhidas v. Navarro
Schmid and Oberly v. RJL
D. Doing Business with or without a License: Suits by or
Against Foreign Corporation
Schwani v. In and Out burger
Lorenzo Shipping v. Chubb
MR Holdings v. Bajar
Commissioner of Customs v. KMK Gani
Communications and Materials Designs v. CA
Columbia Pictures v. CA
Eriks PTE v. CA
Far East international v. Nankai Kogyo
Facilities Management v. dela Osa
French Oil Mill v. CA
HB Zachray v. CA
Home Insurance Co. v. eastern Shipping
Hutchison Ports v. SBMA
La Chemiste Lacoste v. Fernandez
Marubeni nederlands v. Tensuan
Phil. Columbia v. Lantin
Philip Morris v. Fortune Tobacco
Puma v. IAC
SBMA v. Universal International
Time v. Teves
Universal Rubber v. CA

Page 14 of 1072

Van Zuiden v. GTVL industries

SECURITIES AND EXCHANGE COMMISSION LAW


( PD 902-A, as amended by RA No.8799 or the Securities Regulation
Code)
I. POWERS AND FUNCTIONS OF THE SEC
SEC v. PFEC
Aranza v. BF Homes
Quasha v. SEC
Traders royal bank v. CA
VICMAR Devt v. CA
II. ORIGINAL AND EXCLUSIVE JURISDICTION OF THE REGIONAL TRIAL
COURT
Orendain v. BF Homes
Pascual v. CA
A. Devices or Schemes Amounting to fraud or Misrepresentation
Fabia v. CA
A&A Continental v. SEC
Alleje v. CA
Banez v. Dimensional Construction
Sesbreno v. CA
B. Controversies Arising Out of Intra-Corporate or Partnership
Relations
Go Lim vs. Distinction Properties
Strategic Alliance vs. Star Infrastracture
GD Express v. CA
Iglesia v. Juane
Intestate Estate of Ty v. CA
Fabia v. CA
Vesagas v. CA
Abejo v. Dela Cruz
Aguinaldo v. SEC
Pereyra v. IAC
Mainland Construction v. Movilla
SEC v. CA
Sunsetview Condominium v. Campos
Western institute v. Salas
C. Controversies in the Election or Appointment/ Dismissal of
Corporate Officers
Real v. Sangu Phil.

Page 15 of 1072

Matling v. Coros
Garcia v. Eastern telecom
De rossi v. NLRC
Espino v. NLRC
Estrada v. NLRC
Islamic Directorate v. CA
Ongkiko v. NLRC
Paguio v. NLRC
Pearson and George v. NLRC
Apodaca v. NLRC
PSBA v. Leano
Tabang v.NLRC
Union Motors v. NLRC
D. Petitions for Declaration in the State of Suspension of
Payments
Advent Capital vs. Alcantara
Siochi Fishery vs. BPI
Panlilio v. RTC
Castillo v. uniwide Warehouse
Pacific Wide v. Puerto Azul
PNB and EPCIB v. CA
Pryce Corp. v. CA
Uniwide v. Jandecs
BPI v. SEC
PAL v. Heirs of Zamora
Alemars v. Elbinias
Barotac Sugar Mills v. CA
BF Homes v. CA
BPI v. CA
Ching v. LandBank
PCIB v. CA
Radiola- Toshiba v. IAC
RCBC v. IAC
Rubberworld v. NLRC
Union Bank v. CA
SECURITIES REGULATION CODE
(RA NO. 8799)
I. REGISTRATION OF SECURITIES
A. Elements of an Investment contract
SEC v. W.J. Howey Co.
SEC vs. Prosperity Com, Inc.

Page 16 of 1072

Power Homes unlimited v. SEC


SFC vs. Performance
Suzuki v. De Guzman
Bavierra v. Paglinawan
B. What securities are Required to be Registered
Timeshare Realty v. Lao
Makati Stock exchange v. SEC
La Orden v. Stiver and Philtrust
Philippine Stock Exchange v. SEC
C. Exempt Transactions
Timeshare Realty v. Lao
Nestle Phils. v. CA
II. TRADING SECURITIES
A. Margin Requirements
Carolina Industries v. CMS Stock Brokerage
B. Fraudulent Transactions
Phil. Asso. Of Stock Transfer v. CA
SEC v. CA
Onapal v. CA
C. Insiders Duty to disclose When Trading
Strong v. Repide
Union Bank of the Philippines v. SEC
III. PROTECTION OF SHAREHOLDERS INTEREST
A. Tender Offers
CEMCO v. National Life

Page 17 of 1072

TOPIC: ATTRIBUTES OF A CORPORATION


PETRON CORPORATION
VS.
NATIONAL COLLEGE OFBUSINESS AND ARTS
G.R. NO. 155683 February 16, 2007
516 SCRA 168
FACTS:
In 1969, the V. Mapa properties, then owned by Felipe and Enrique
Monserrat, Jr., were mortgaged to the Development Bank of the Philippines
(DBP) as part of the security for the P5.2 million loan of Manila Yellow Taxicab
Co., Inc. (MYTC) and Monserrat Enterprises Co. MYTC, for its part, mortgaged
four parcels of land.
Felipes undivided interest in the V. Mapa properties was levied upon
in execution of a money judgment. DBP challenged the levy through a thirdparty claim asserting that the V. Mapa properties were mortgaged to it and
were, for that reason, exempt from levy or attachment. The RTC quashed it.
MYTC and the Monserrats got DBP to accept a dacion en pago arrangement
whereby MYTC conveyed to the bank the four mortgaged Quiapo properties
as full settlement of their loan obligation. But despite this agreement, DBP
did not release the V. Mapa properties from the mortgage.
Felipe, acting for himself and as Enriques attorney-in-fact, sold the V.
Mapa properties to respondent NCBA. Part of the agreement was that Felipe
and Enrique would secure the release of the titles to the properties free of all
liens and encumbrances including DBPs mortgage lien and Filoils levy on or
before July 31, 1982.
But the Monserrats failed to comply with this
undertaking. Thus, on February 3, 1983, NCBA caused the annotation of an
affidavit of adverse claim on the TCTs covering the V. Mapa properties.
NCBA filed a complaint against Felipe and Enrique for specific
performance. NCBA impleaded DBP as an additional defendant in order to
compel it to release the V. Mapa properties from mortgage.

Page 18 of 1072

During the pendency of Civil Case No. 83-16617, Enriques


undivided interest in the V. Mapa properties was levied on in execution of a
judgment holding him liable to Petron (then known as Petrophil Corporation)
on a 1972 promissory note. The final deeds of sale of Enriques and Felipes
shares in the V. Mapa properties were awarded to Petron in 1986. Sometime
later, the Monserrats TCTs were cancelled and new ones were issued to
Petron. Thus it was that, towards the end of 1987, Petron intervened in
NCBAs suit against Felipe, Enrique and DBP (Civil Case No. 83-16617) to
assert its right to the V. Mapa properties.
The RTC rendered judgment, it ruled among other things, that Petron
never acquired valid title to the V. Mapa properties as the levy and sale
thereof were void and that NCBA was now the lawful owner of the properties.
Moreover, the RTC held Petron, DBP, Felipe and Enrique jointly and severally
liable to NCBA for exemplary damages and attorneys fees
On appeal, the CA affirmed in toto the decision of the RTC.
ISSUE/S:
Whether petitioner Petron Corporation (Petron) should be held liable to
pay attorneys fees and exemplary damages to respondent National
College of Business and Arts (NCBA).
RULING:
No.
Article 2208 lays down the rule that in the absence of stipulation,
attorneys fees cannot be recovered except in the following instances:
xxxxxx xxx
(5)
Where the defendant acted in gross and evident bad faith
in refusing to satisfy the plaintiffs plainly valid, just and
demandable claim;
xxx xxx xxx
Here, the RTC held Petron liable to NCBA for attorneys fees under
Article 2208(5), which allows such an award where the defendant acted in
gross and evident bad faith in refusing to satisfy the plaintiffs plainly valid,
just, and demandable claim. However, the only justification given for this
verdict was that Petron had no reason to claim the V. Mapa properties
because, in the RTCs opinion, the levy and sale thereof were void. This was
sorely inadequate and it was erroneous for the CA to have upheld that ruling
built on such a flimsy foundation.
Article 2208(5) contemplates a situation where one refuses
unjustifiably and in evident bad faith to satisfy anothers plainly valid, just

Page 19 of 1072

and demandable claim, compelling the latter needlessly to seek redress from
the courts. In such a case, the law allows recovery of money the plaintiff had
to spend for a lawyers assistance in suing the defendant expenses the
plaintiff would not have incurred if not for the defendants refusal to comply
with the most basic rules of fair dealing. It does not mean, however, that the
losing party should be made to pay attorneys fees merely because the court
finds his legal position to be erroneous and upholds that of the other party,
for that would be an intolerable transgression of the policy that no one
should be penalized for exercising the right to have contending claims
settled by a court of law. No gross and evident bad faith could be imputed to
Petron merely for intervening in NCBAs suit against DBP and the Monserrats
in order to assert what it believed (and had good reason to believe) were its
rights and to have the disputed ownership of the V. Mapa properties settled
decisively in a single lawsuit.
With respect to the award of exemplary damages, the rule in this
jurisdiction is that the plaintiff must show that he is entitled to moral,
temperate or compensatory damages before the court may even consider
the question of whether exemplary damages should be awarded. In other
words, no exemplary damages may be awarded without the plaintiffs right
to moral, temperate, liquidated or compensatory damages having first been
established. Therefore, in view of our ruling that Petron cannot be made
liable to NCBA for compensatory damages (i.e., attorneys fees), Petron
cannot be held liable for exemplary damages either.

Page 20 of 1072

TOPIC: ATTRIBUTES OF A CORPORATION


ASSET PRIVATIZATION TRUST, petitioner,
vs.
COURT OF APPEALS, JESUS S. CABARRUS, SR., JESUS S. CABARRUS,
JR., JAIME T. CABARRUS, JOSE MIGUEL CABARRUS, ALEJANDRO S.
PASTOR, JR., ANTONIO U. MIRANDA, and MIGUEL M. ANTONIO, as
Minority Stock-Holders of Marinduque Mining and Industrial
Corporation, respondents.
G.R. NO. 121171 December 29, 1998,
FACTS:
Republic Act No. 1828 authorized the development, exploration and
utilization of the mineral deposits in the Surigao Mineral Reservation. By
virtue of which, a Memorandum of Agreement was drawn, whereby the
Republic thru the Surigao Mineral Reservation Board, granted MMIC the
exclusive right to explore, develop and exploit nickel, cobalt and other
minerals in the Surigao mineral reservation. MMIC is a domestic corporation
engaged in mining with respondents Jesus S. Cabarrus, Sr. as President and
among its original stockholders.
The Government undertook to support the financing of MMIC by
purchase of MMIC debenture and extension of guarantees. MMIC, PNB and
DBP executed a Mortgage Trust Agreement whereby MMIC, as mortgagor,
agreed to constitute a mortgage in favor of PNB and DBP as mortgagees,
over all MMICs assets, subject of real estate and chattel mortgage executed
by the mortgagor, and additional assets described and identified, which the
mortgagor may acquire whether in substitution of, in replenishment, or in
addition thereto.
Article IV of the Mortgage Trust Agreement provides for Events of
Default, which expressly includes the event that the MORTGAGOR shall fail to
pay any amount secured by this Mortgage Trust Agreement when due. Article
V of the Mortgage Trust Agreement prescribes in detail, and in addition to the
enumerated events of defaults, circumstances by which the mortgagor may

Page 21 of 1072

be declared in default, the procedure therefor, waiver of period to foreclose,


authority of Trustee before, during and after foreclosure, including taking
possession of the mortgaged properties.
In various request for advances/remittances of loans of huge
amounts, , MMIC invariably committed to pay either on demand or under
certain terms the loans and accommodations secured from or guaranteed by
both DBP and PNB.
By 1984, DBP and PNBs financial exposure both in loans and in equity
in MMIC had reached tremendous proportions, and MMIC was having a
difficult time meeting its financial obligations. DBP and PNB as mortgagees of
MMIC assets, decided to exercise their right to extrajudicially foreclose the
mortgages in accordance with the Mortgage Trust Agreement. The foreclosed
assets were sold to PNB as the lone bidder and were assigned to three newly
formed corporations, namely, Nonoc Mining Corporation, Maricalum Mining
and Industrial Corporation, and Island Cement Corporation. In 1986, these
assets were transferred to the Asset Privatization Trust (APT).
Jesus S. Cabarrus, Sr., together with the other stockholders of MMIC,
filed a derivative suit against DBP and PNB. In the course of the trial, private
respondents and petitioner APT, as successor of the DBP and PNBs interest
in MMIC, mutually agreed to submit the case to arbitration by entering into a
Compromise and Arbitration Agreement.
ISSUE/S:
Whether or not the filing of the derivative suit was proper and the
award of damages was with legal basis.
RULING:
The civil case being a derivative suit, MMIC should have been
impleaded as a party. It was not joined as a party plaintiff or party defendant
at any stage of the proceedings. As it is, the award of damages to MMIC,
which was not a party before the Arbitration Committee, is a complete nullity.
Settled is the doctrine that in a derivative suit, the corporation is the
real party in interest while the stockholder filing suit for the corporations
behalf is only nominal party. The corporation should be included as a party in
the suit.
An individual stockholder is permitted to institute a derivative suit on
behalf of the corporation wherein he holds stock in order to protect or
vindicate corporate rights, whenever the officials of the corporation refuse to
sue, or are the ones to be sued or hold the control of the corporation. In
such actions, the suing stockholder is regarded as a nominal party, with the
corporation as the real party in interest.

Page 22 of 1072

It is a condition sine qua non that the corporation be impleaded as a


party because- Not only is the corporation an indispensible party, but it is
also the present rule that it must be served with process. The reason given
is that the judgment must be made binding upon the corporation and in
order that the corporation may get the benefit of the suit and may not bring
a subsequent suit against the same defendants for the same cause of action.
The arbiters, likewise, exceeded their authority in awarding moral
damages to Jesus Cabarrus, Sr. Cabarrus cause of action for the seizure of
the assets belonging to IEI, of which he is the majority stockholder, having
been ventilated in a complaint he previously filed with the RTC, from which
he obtained actual damages, he was barred res judicata from filing a similar
case in another court, this time asking for moral damages which he failed to
get from the earlier case.
It is a basic postulate that s corporation has a personality separate and
distinct from its stockholders. The properties foreclosed belonged to MMIC,
not to its stockholders. Hence, if wrong was committed in the foreclosure, it
was done against the corporation. Another reason is that Jesus S. Cabarrus,
Sr. cannot directly claim those damages for himself that would result in the
appropriation by, and the distribution to, him part of the corporations assets
before the dissolution of the corporation and the liquidation of its debts and
liabilities.

Page 23 of 1072

TOPIC: ATTRIBUTES OF A CORPORATION


MAMBULAO LUMBER COMPANY, plaintiff-appellant,
vs.
PHILIPPINE NATIONAL BANK and ANACLETO HERALDO Deputy
Provincial Sheriff of Camarines Norte, defendants-appellees
G.R, NO. L-22973 January 30, 1968
FACTS:
Plaintiff applied for an industrial loan of P155,000 with the Naga Branch
of defendant PNB and the former offered real estate, machinery, logging and
transportation equipments as collaterals. The application, however, was
approved for a loan of P100,000 only. To secure the payment of the loan, the
plaintiff mortgaged to defendant PNB a parcel of land.
PNB released from the approved loan the sum of P27,500, for which
the plaintiff signed a promissory note wherein it promised to pay to the PNB
the said sum in five equal yearly. The plaintiff failed to pay the amortizations
on the amounts released to and received by it. It was found that the plaintiff
had already stopped operation about the end of 1957 or early part of 1958.
Plaintiff sent separate letters protesting against the foreclosure of the
real estate and chattel mortgages on the grounds that they could not be
effected unless a Court's order was issued against it (plaintiff) for said
purpose and that the foreclosure proceedings, according to the terms of the
mortgage contracts, should be made in Manila.
Several employees of the PNB arrived in the compound and they
informed Luis Salgado, Chief Security Guard of the premises, that the
properties therein had been auctioned and bought by the PNB, which in turn
sold them to Mariano Bundok. On the following day, two trucks and men of
Mariano Bundok arrived but Salgado did not permit them to take out any
equipment from inside the compound of the plaintiff. Thru the intervention,
however, of the local police and PC soldiers, the trucks of Mariano Bundok

Page 24 of 1072

were able finally to haul the properties originally mortgaged by the plaintiff
to the PNB, which were bought by it at the foreclosure sale and subsequently
sold to Mariano Bundok.
ISSUE/S:
Whether or not moral damages may be awarded to a corporation.
RULING:
Herein appellant's claim for moral damages seems to have no legal or
factual basis. Obviously, an artificial person like herein Appellant Corporation
cannot experience physical sufferings, mental anguish, fright, serious
anxiety, wounded feelings, moral shock or social humiliation which are the
basis of moral damages. A corporation may have a good reputation which, if
besmirched, may also be a ground for the award of moral damages. The
same cannot be considered under the facts of this case, however, not only
because it is admitted that herein appellant had already ceased in its
business operation at the time of the foreclosure sale of the chattels, but
also for the reason that whatever adverse effect the foreclosure sale of the
chattels could have upon its reputation or business standing would
undoubtedly be the same whether the sale was conducted at Jose
Panganiban. Camarines Norte, or in Manila which is the place agreed upon by
the parties in the mortgage contract.
But for the wrongful acts of herein appellee bank and the deputy
sheriff of Camarines Norte in proceeding with the sale in utter disregard of
the agreement to have the chattels sold in Manila as provided for in the
mortgage contract, to which their attentions were timely called by herein
appellant, and in disposing of the chattels in gross for the miserable amount
of P4,200.00, herein appellant should be awarded exemplary damages in the
sum of P10,000.00. The circumstances of the case also warrant the award of
P3,000.00 as attorney's fees for herein appellant.

Page 25 of 1072

TOPIC: ATTRIBUTES OF A CORPORATION


HANIL DEVELOPMENT CO., LTD., petitioner,
vs.
COURT OF APPEALS AND M.R. ESCOBAR EXPLOSIVE ENGINEERS,
INC., respondent.
G.R. No. 113176. July 30, 2001
FACTS:
In the early seventies, the Ministry of Public Works and Highways
(MPWH for brevity) awarded petitioner Hanil Development Co., Ltd. the
contract to construct the 200-kilometer Iligan-Cagayan de Oro-Butuan
Highway Project. On November 14, 1976, Hanil sub-let the rock-blasting work
portion of the contract to private respondent M.R. Escobar Explosive
Engineers, Inc.. By express stipulation of the parties, Escobar will be
compensated.
Escobar commenced its blasting works. It continued its services until
terminated by Hanil on December 15, 1978. It It claimed, however, that
Hanil still partially owes it one million three hundred forty one thousand
seven hundred twenty-seven and 40/100 (P1,341,727.40) pesos for blastings
done in the B-2, B-3 and C-1 areas. The claim was predicated on the theory
that the rocks it caused to explode in the contested areas were solid in
nature, and therefore the volume should be computed using the crosssection approach pursuant to the above-quoted paragraph 9(a). It appears
that all the payments it received were fixed based on the joint survey
method under paragraph 9(b). Escobar stressed that Hanil was always paid
by the MPWH using the cross-section system.
Consequently, Escobar instituted Civil Case No. 35966 for recovery of a
sum of money with damages against Hanil . The CFI handed down a Decision
ordering Hanil to pay for the value of rocks blasted by Escobar.
ISSUE/S:

Page 26 of 1072

Whether or not the award of moral damages and other damages is


proper.
RULING:
No.
Hanil failed to prove the actual value of pecuniary injury which it
sustained as a consequence of Escobars institution of an unfounded civil
suit. The testimony of one of its witnesses, presented in the CFI, to the effect
that; the filing of the complaint affected Hanils reputation and that it
affected the management and engineers working in the site, is not enough
proof. The institution of the suit, unfounded though it may be, does not
always lead to pecuniary loss as to warrant an award of actual or temperate
damages. So, too, must its demand for payment of moral damages fail. The
rule is that moral damages can not be granted in favor of a corporation.
Being an artificial person and having existence only in legal contemplation, a
corporation has no feelings, no emotions, no senses. It cannot, therefore,
experience physical suffering, mental anguish, fright, serious anxiety,
wounded feelings or moral shock or social humiliation, which can be suffered
only by one having a nervous system.
Hanils prayer for exemplary damages must likewise be denied. It
must be remembered that this kind of damages cannot be recovered as a
matter of right. Its allowance rests in the sound discretion of the court, and
only upon a showing of its legal foundation. Under the Civil Code, the
claimant must first establish that he is entitled to moral, temperate,
compensatory or liquidated damages before it may be imposed in his favor.
Hanil failed to do so, hence, it cannot claim exemplary damages.

Page 27 of 1072

TOPIC: ATTRIBUTES OF A CORPORATION


BACHE & CO. (PHIL.), INC. AND FREDERICK E. SEGGERMAN
VS.
HON. JUDGE VIVENCIO M. RUIZ, MISAEL P. VERA
G.R. NO. L-32409 February 27, 1971
FACTS:
Respondent Misael P. Vera, Commissioner of Internal Revenue, wrote a
letter addressed to respondent Judge Vivencio M. Ruiz requesting the
issuance of a search warrant against petitioners for violation of Section 46(a)
of the National Internal Revenue Code and authorizing Revenue Examiner
Rodolfo de Leon, one of herein respondents, to make and file the application
for search warrant which was attached to the letter.
The following day, respondent De Leon and his witness went to the
court and brought with them the following papers:
respondent Vera's aforesaid letter-request;
an application for search warrant already filled up but still
unsigned by respondent De Leon;
an affidavit of respondent Logronio subscribed before
respondent De Leon;
a deposition in printed form of respondent Logronio already
accomplished and signed by him but not yet subscribed;
and
a search warrant already accomplished but still unsigned
by respondent Judge.
Three days later, the BIR agents served the search warrant petitioners
at the offices of petitioner corporation. Petitioners' lawyers protested the
search on the ground that no formal complaint or transcript of testimony was
attached to the warrant.

Page 28 of 1072

Petitioners filed a petition that the search warrant be quashed,


dissolved or recalled. After hearing, the court, presided over by respondent
Judge, issued an order dismissing the petition for dissolution of the search
warrant. In the meantime, the Bureau of Internal Revenue made tax
assessments on petitioner corporation in the total sum of P2,594,729.97,
partly, if not entirely, based on the documents thus seized.
ISSUE/S:
Whether or not the issuance of the search warrant was proper.
HELD:
The right of the people to be secure in their persons, houses, papers
and effects against unreasonable searches and seizures shall not be violated,
and no warrants shall issue but upon probable cause, to be determined by
the judge after examination under oath or affirmation of the complainant and
the witnesses he may produce, and particularly describing the place to be
searched, and the persons or things to be seized."
Personal examination by the judge of the complainant and his
witnesses is necessary to enable him to determine the existence or nonexistence of a probable cause, pursuant to Art. III, Sec. 1, par. 3, of the
Constitution, and Sec. 3, Rule 126 of the Revised Rules of Court, both of
which prohibit the issuance of warrants except "upon probable cause." The
determination of whether or not a probable cause exists calls for the exercise
of judgment after a judicial appraisal of facts and should not be allowed to be
delegated in the absence of any rule to the contrary.
In the case at bar, no personal examination at all was conducted by
respondent Judge of the complainant and his witness. While it is true that the
complainant's application for search warrant and the witness' printed-form
deposition were subscribed and sworn to before respondent Judge, the latter
did not ask either of the two any question the answer to which could possibly
be the basis for determining whether or not there was probable cause
against herein petitioners. The search warrant in question was issued for at
least four distinct offenses under the Tax Code. The first is the violation of
Sec. 46(a), Sec. 72 and Sec. 73 (the filing of income tax returns), which are
interrelated. The second is the violation of Sec. 53 (withholding of income
taxes at source). The third is the violation of Sec. 208 (unlawful pursuit of
business or occupation); and the fourth is the violation of Sec. 209 (failure to
make a return of receipts, sales, business or gross value of output actually
removed or to pay the tax due thereon).
The documents, papers and effects sought to be seized are described
in Search Warrant No. 2-M-70 in this manner:

Page 29 of 1072

"Unregistered and private books of account; records of bank


deposits and withdrawals; and records of foreign remittances,
covering the years 1966 to 1970."
The description does not meet the requirement in Art III, Sec. 1, of the
Constitution, and of Sec. 3, Rule 126 of the Revised Rules of Court, that the
warrant should particularly describe the things to be seized.
A corporation is, after all, but an association of individuals under an
assumed name and with a distinct legal entity. In organizing itself as a
collective body it waives no constitutional immunities appropriate to such
body. Its property cannot be taken without compensation. It can only be
proceeded against by due process of law, and is protected, under the 14th
Amendment, against unlawful discrimination . . ."

TOPIC: ATTRIBUTES OF A CORPORATION


SULO NG BAYAN, INC.,
VS.
GREGORIO ARANETA, INC., PARADISE FARMS, INC., NATIONAL
WATERWORKS & SEWERAGE AUTHORITY, HACIENDA CARETAS, INC.
AND REGISTER OF DEEDS OF BULACAN
G.R. NO. L-31061 August 17, 1976
FACTS:
Plaintiff Sulo ng Bayan, Inc. filed an accion de reivindicacion with the
court to recover the ownership and possession of a large tract of land. The
complaint specifically alleged that plaintiff is a corporation organized and
existing under the laws of the Philippines, with its principal office and place
of business at San Jose del Monte, Bulacan; that its membership is composed
of natural persons residing at San Jose del Monte, Bulacan; that the members
of the plaintiff corporation, through themselves and their predecessors-ininterest, had pioneered in the clearing of the afore-mentioned tract of land,
cultivated the same since the Spanish regime and continuously possessed
the said property openly and publicly under concept of ownership adverse
against the whole world; that defendant-appellee Gregorio Araneta, Inc.,
sometime in the year 1958, through force and intimidation, ejected the
members of the plaintiff corporation from their possession of the
aforementioned vast tract of land; that upon investigation conducted by the
members and officers of plaintiff corporation, they found out for the first time
in the year 1961 that the land in question "had been either fraudulently or
erroneously included, by direct or constructive fraud, in Original Certificate of
Title No. 466 of the Land Records of the province of Bulacan", issued on May
11, 1916, which title is fictitious, non-existent and devoid of legal efficacy
due to the fact that "no original survey nor plan whatsoever" appears to

Page 30 of 1072

have been submitted as a basis thereof and that the court which issued the
decree of registration did not acquire jurisdiction over the land registration
case because no notice of such proceedings was given to the members of
the plaintiff corporation who were then in actual possession of said
properties; that as a consequence of the nullity of the original title, all
subsequent titles derived therefrom, Transfer Certificate of Title No. 4986
issued in the name of Hacienda Caretas, Inc., and another transfer certificate
of title in the name of Paradise Farms, Inc., are therefore void.
ISSUE/S:
Whether or not Plaintiff Corporation (non- stock) may institute an
action in behalf of its individual members for the recovery of certain
parcels of land allegedly owned by said members.
HELD:
NO.
It is a doctrine well-established and obtains both at law and in equity
that a corporation is a distinct legal entity to be considered as separate and
apart from the individual stockholders or members who compose it, and is
not affected by the personal rights, obligations and transactions of its
stockholders or members. The property of the corporation is its property and
not that of the stockholders, as owners, although they have equities in it.
Conversely, a corporation ordinarily has no interest in the individual property
of its stockholders unless transferred to the corporation, "even in the case of
a one-man corporation".
It must be noted, however, that the juridical personality of the
corporation, as separate and distinct from the persons composing it, is but a
legal fiction introduced for the purpose of convenience and to sub serve the
ends of justice. This separate personality of the corporation may be
disregarded, or the veil of corporate fiction pierced, in cases where it is used
as a cloak or cover for fraud or illegality, or to work an injustice, or where
necessary to achieve equity.
It has not been claimed that the members have assigned or transferred
whatever rights they may have on the land in question to the plaintiff
corporation. Absent any showing of interest, therefore, a corporation, like
plaintiff-appellant herein, has no personality to bring an action for and in
behalf of its stockholders or members for the purpose of recovering property
which belongs to said stockholders or members in their personal capacities.
It is fundamental that there cannot be a cause of action without an
antecedent primary legal right conferred by law upon a person. Evidently,
there can be no wrong without a corresponding right, and no breach of duty
by one person without a corresponding right belonging to some other person.

Page 31 of 1072

Thus, the essential elements of a cause of action are legal right of the
plaintiff, correlative obligation of the defendant, an act or omission of the
defendant in violation of the aforesaid legal right. Clearly, no right of action
exists in favor of plaintiff corporation, for as shown heretofore it does not
have any interest in the subject matter of the case which is material and
direct so as to entitle it to file the suit as a real party in interest.
In order that a class suit may prosper, the following requisites must be
present:
1. that the subject matter of the controversy is one of
common or general interest to many persons; and
2. that the parties are so numerous that it is
impracticable to bring them all before the court.
Here, there is only one party plaintiff, and the plaintiff corporation does
not even have an interest in the subject matter of the controversy, and
cannot, therefore, represent its members or stockholders who claim to own
in their individual capacities ownership of the said property. Moreover, as
correctly stated by the appellees, a class suit does not lie in actions for the
recovery of property where several persons claim ownership of their
respective portions of the property, as each one could allege and prove his
respective right in a different way for each portion of the land, so that they
cannot all be held to have identical title through acquisitive prescription.

Page 32 of 1072

TOPIC:CLASSIFICATIONS OF CORPORATION
BOY SCOUTS OF THE PHILIPPINES,
vs.
COMMISSION ON AUDIT
G.R. No. 177131 June 7, 2011
FACTS:
This case arose when the COA issued Resolution No. 99-011on August
19, 1999 with the subject "Defining the Commissions policy with respect to
the audit of the Boy Scouts of the Philippines." In its whereas clauses, the
COA Resolution stated that the BSP was created as a public corporation
under Commonwealth Act No. 111, as amended by Presidential Decree No.
460. and Republic Act No. 7278; that in Boy Scouts of the Philippines v.
National Labor Relations Commission, the Supreme Court ruled that the BSP,
as constituted under its charter, was a "government-controlled corporation
within the meaning of Article IX (B) (2) (1) of the Constitution"; and that "the
BSP is appropriately regarded as a government instrumentality under the
1987 Administrative Code. The COA Resolution also cited its constitutional
mandate under Section 2(1), Article IX (D).
The BSP sought reconsideration of the COA Resolution. The BSP
believes that the cited case has been superseded by RA 7278. The 1987
Administrative Code itself, of which the BSP vs. NLRC relied on for some
terms, defines government-owned and controlled corporations as agencies
organized as stock or non-stock corporations which the BSP, under its
present charter, is not.
ISSUE/S:

Page 33 of 1072

Whether or not Boy Scouts of the Philippines is a government-owned or


controlled corporation or instrumentality, agency, or subdivision of the
government.
RULING:
After looking at the legislative history of its amended charter and
carefully studying the applicable laws and the arguments of both parties, we
find that the BSP is a public corporation and its funds are subject to the
COAs audit jurisdiction.
The BSP Charter (Commonwealth Act No. 111, approved on October
31, 1936), entitled "An Act to Create a Public Corporation to be known as the
Boy Scouts of the Philippines and to define its Powers and Purposes" created
the BSP as a "public corporation".
There are three classes of juridical persons under Article 44 of the Civil
Code and the BSP, as presently constituted under Republic Act No. 7278, falls
under the second classification.
Evidently, the BSP, which was created by a special law to serve a
public purpose in pursuit of a constitutional mandate, comes within the class
of "public corporations" defined by paragraph 2, Article 44 of the Civil Code
and governed by the law which creates it, pursuant to Article 45 of the same
Code.
The BSP is a public corporation or a government agency or
instrumentality with juridical personality, which does not fall within the
constitutional prohibition in Article XII, Section 16, notwithstanding the
amendments to its charter. Not all corporations, which are not government
owned or controlled, are ipso facto to be considered private corporations as
there exists another distinct class of corporations or chartered institutions
which are otherwise known as "public corporations." These corporations are
treated by law as agencies or instrumentalities of the government which are
not subject to the tests of ownership or control and economic viability but to
different criteria relating to their public purposes/interests or constitutional
policies and objectives and their administrative relationship to the
government or any of its Departments or Offices.
Further Section 16, Article XII should not be construed so as to prohibit
Congress from creating public corporations. In fact, Congress has enacted
numerous laws creating public corporations or government agencies or
instrumentalities vested with corporate powers. Moreover, Section 16, Article
XII, which relates to National Economy and Patrimony, could not have tied
the hands of Congress in creating public corporations to serve any of the
constitutional policies or objectives.

Page 34 of 1072

The Court is fortified that the Administrative Code of 1987 designates


the BSP as one of the attached agencies of the Department of Education,
Culture and Sports ("DECS"). An "agency of the Government" is defined as
referring to any of the various units of the Government including a
department, bureau, office, instrumentality, government-owned or
-controlled corporation, or local government or distinct unit therein.
The Court believes that the BSP is appropriately regarded as "a
government instrumentality" under the 1987 Administrative Code. It thus
appears that the BSP may be regarded as both a "government controlled
corporation with an original charter" and as an "instrumentality" of the
Government within the meaning of Article IX (B) (2) (1) of the Constitution.

TOPIC: CLASSIFICATIONS OF CORPORATION


DANTE V. LIBAN, REYNALDO M. BERNARDO, and SALVADOR M. VIARI,
vs.
RICHARD J. GORDON
G.R. No. 175352 July 15, 2009& January 18, 2011
FACTS:
Petitioners are officers of the Board of Directors of the Quezon City Red
Cross Chapter while respondent is Chairman of the Philippine National Red
Cross (PNRC) Board of Governors.
During respondents incumbency as a member of the Senate of the
Philippines,he was elected Chairman of the PNRC during the 23 February
2006 meeting of the PNRC Board of Governors. Petitioners allege that by
accepting the chairmanship of the PNRC Board of Governors, respondent has
ceased to be a member of the Senate as provided in Section 13, Article VI of
the Constitution.
Petitioners cite Camporedondo v. NLRC which held that the PNRC is a
government-owned or controlled corporation. Petitioners claim that in
accepting and holding the position of Chairman of the PNRC Board of
Governors, respondent has automatically forfeited his seat in the Senate,
pursuant to Flores v. Drilon.
Respondent further insists that the PNRC is not a government-owned or
controlled corporation and that the prohibition under Section 13, Article VI of
the Constitution does not apply in the present case since volunteer service to
the PNRC is neither an office nor an employment.
In their Reply, petitioners claim that their petition is neither an action
for quo warranto nor an action for declaratory relief. Petitioners maintain that

Page 35 of 1072

the present petition is a taxpayers suit questioning the unlawful


disbursement of funds, considering that respondent has been drawing his
salaries and other compensation as a Senator even if he is no longer entitled
to his office. Petitioners point out that this Court has jurisdiction over this
petition since it involves a legal or constitutional issue which is of
transcendental importance.
ISSUE/S:
Whether the office of the PNRC Chairman is a government office or an
office in a government-owned or controlled corporation for purposes of
the prohibition in Section 13, Article VI of the Constitution.

RULING:
July 15, 2009 Decision
The PNRC is not government-owned but privately owned. The vast
majority of the thousands of PNRC members are private individuals, including
students. Under the PNRC Charter, those who contribute to the annual fund
campaign of the PNRC are entitled to membership in the PNRC for one year.
Thus, any one between 6 and 65 years of age can be a PNRC member for
one year upon contributing P35, P100, P300, P500 or P1,000 for the year.20
Even foreigners, whether residents or not, can be members of the PNRC.
Thus, the PNRC is a privately owned, privately funded, and privately
run charitable organization. The PNRC is not a government-owned or
controlled corporation.
The Constitution recognizes two classes of corporations. The first refers
to private corporations created under a general law. The second refers to
government-owned or controlled corporations created by special charters.
The Constitution emphatically prohibits the creation of private
corporations except by general law applicable to all citizens. The purpose of
this constitutional provision is to ban private corporations created by special
charters, which historically gave certain individuals, families or groups
special privileges denied to other citizens.
In short, Congress cannot enact a law creating a private corporation
with a special charter. Such legislation would be unconstitutional. Private
corporations may exist only under a general law. If the corporation is private,
it must necessarily exist under a general law. Stated differently, only
corporations created under a general law can qualify as private corporations.
Under existing laws, the general law is the Corporation Code, except that the
Cooperative Code governs the incorporation of cooperatives.

Page 36 of 1072

Just like the Local Water Districts, the PNRC was created through a
special charter. However, unlike the Local Water Districts, the elements of
government ownership and control are clearly lacking in the PNRC. Thus,
although the PNRC is created by a special charter, it cannot be considered a
government-owned or controlled corporation in the absence of the essential
elements of ownership and control by the government.
In sum, we hold that the office of the PNRC Chairman is not a
government office or an office in a government-owned or controlled
corporation for purposes of the prohibition in Section 13, Article VI of the
1987 Constitution. However, since the PNRC Charter is void insofar as it
creates the PNRC as a private corporation, the PNRC should incorporate
under the Corporation Code and register with the Securities and Exchange
Commission if it wants to be a private corporation.
January 18, 2011 Decision
The PNRC, as a National Society of the International Red Cross and Red
Crescent Movement, can neither be classified as an instrumentality of the
State, so as not to lose its character of neutrality as well as its
independence, nor strictly as a private corporation since it is regulated by
international humanitarian law and is treated as an auxiliary of the State.
Based on the above, the sui generis status of the PNRC is now
sufficiently established. Although it is neither a subdivision, agency, or
instrumentality of the government, nor a government-owned or -controlled
corporation or a subsidiary thereof, as succinctly explained in the Decision of
July 15, 2009, so much so that respondent, under the Decision, was correctly
allowed to hold his position as Chairman thereof concurrently while he
served as a Senator, such a conclusion does notipso facto imply that the
PNRC is a private corporation within the contemplation of the provision of
the Constitution, that must be organized under the Corporation Code. As
correctly mentioned by Justice Roberto A. Abad, the sui generis character of
PNRC requires us to approach controversies involving the PNRC on a case-tocase basis.
In sum, the PNRC enjoys a special status as an important ally and
auxiliary of the government in the humanitarian field in accordance with its
commitments under international law. This Court cannot all of a sudden
refuse to recognize its existence, especially since the issue of the
constitutionality of the PNRC Charter was never raised by the parties. It bears
emphasizing that the PNRC has responded to almost all national disasters
since 1947, and is widely known to provide a substantial portion of the
countrys blood requirements. Its humanitarian work is unparalleled. The
Court should not shake its existence to the core in an untimely and drastic
manner that would not only have negative consequences to those who

Page 37 of 1072

depend on it in times of disaster and armed hostilities but also have adverse
effects on the image of the Philippines in the international community. The
sections of the PNRC Charter that were declared void must
therefore stay.

TOPIC: CLASSIFICATIONS OF CORPORATION


FRANCISCA S. BALUYOT
vs.
PAUL E. HOLGANZA and the OFFICE OF THE OMBUDSMAN (VISAYAS)
represented by its Deputy Ombudsman for the Visayas ARTURO C.
MOJICA, Director VIRGINIA PALANCA-SANTIAGO, and Graft
Investigation Officer I ANNA MARIE P. MILITANTE
G.R. No. 136374 February 9, 2000
FACTS:
During a spot audit conducted on March 21, 1977 by a team of auditors
from the Philippine National Red Cross (PNRC) headquarters, a cash shortage
of P154, 350.13 was discovered in the funds of its Bohol chapter. Petitioner
Francisca S. Baluyot was held accountable for the shortage. Thereafter, on
January 8, 1998, private respondent Paul E. Holganza, in his capacity as a
member of the board of directors of the Bohol chapter, filed an affidavitcomplaint charging petitioner of malversation under Article 217 of the
Revised Penal Code.
On February 6, 1998, public respondent issued an Order requiring
petitioner to file her counter-affidavit to the charges of malversation and
dishonesty within ten days from notice, with a warning that her failure to
comply would be construed as a waiver on her part to refute the charges,
and that the case would be resolved based on the evidence on record. On
March 14, 1998, petitioner filed her counter-affidavit;raising principally the
defense that public respondent had no jurisdiction over the controversy. She
argued that the Ombudsman had authority only over government-owned or
controlled corporations, which the PNRC was not, or so she claimed.

Page 38 of 1072

ISSUE/S:
Whether or not PNRC is a private voluntary organization.
RULING:
Resolving the issue set out in the opening paragraph of this opinion,
we rule that the Philippine National Red Cross (PNRC) is a government owned
and controlled corporation, with an original charter under Republic Act No.
95, as amended.
The test to determine whether a corporation is government owned or
controlled or private in nature is simple.
Is it created by its own charter for the exercise of a public
function, or by incorporation under the general corporation law?
Those with special charters are government corporations subject to its
provisions, and its employees are under the jurisdiction of the Civil Service
Commission, and are compulsory members of the Government Service
Insurance System. The PNRC was not "impliedly converted to a private
corporation" simply because its charter was amended to vest in it the
authority to secure loans, be exempted from payment of all duties, taxes,
fees and other charges of all kinds on all importations and purchases for its
exclusive use, on donations for its disaster relief work and other services and
in its benefits and fund raising drives, and be allotted one lottery draw a year
by the Philippine Charity Sweepstakes Office for the support of its disaster
relief operation in addition to its existing lottery draws for blood program.
Clearly then, public respondent has jurisdiction over the matter,
pursuant to Section 13, of Republic Act No. 6770, otherwise known as "The
Ombudsman Act of 1989", to wit:
Sec. 13.Mandate. The Ombudsman and his Deputies, as
protectors of the people, shall act promptly on complaints filed in
any form or manner against officers or employees of the
Government, or of any subdivision, agency or instrumentality
thereof, including government-owned or controlled corporations,
and enforce their administrative, civil and criminal liability in
ever case where the evidence warrants in order to promote
efficient service by the Government to the people.

Page 39 of 1072

TOPIC:CLASSIFICATIONS OF CORPORATION
THE VETERANS FEDERATION OF THE PHILIPPINES represented by
Esmeraldo R. Acorda,
vs.
Hon. ANGELO T. REYES in his capacity as Secretary of National
Defense; and Hon. EDGARDO E. BATENGA in his capacity as
Undersecretary for Civil Relations and Administration of the
Department of National Defense
G. R. No. 155027.February 28, 2006
FACTS:
Petitioner VFP was created under Rep. Act No. 2640, a statute
approved on 18 June 1960.
On 15 April 2002, petitioners incumbent president received a letter
which tended to show that there is an organizational and management
relationship between Veterans Federation of the Philippines and the
Philippine Veterans Bank which for many years have been inadvertently
overlooked. On 10 June 2002, respondent DND Secretary issued the assailed
DND Department Circular No. 04.
In a letter addressed to the President of petitioner, respondent DND
Secretary reiterated his instructions in his earlier letter of 13 April 2002.
Thereafter, petitioners President received a letter dated 23 August 2002
from respondent Undersecretary, informing him that Department Order No.
129 dated 23 August 2002 directed "the conduct of a Management Audit of
the Veterans Federation of the Philippines." The letter went on to state that
respondent DND Secretary "believes that the mandate given by said law can
be meaningfully exercised if this department can better appreciate the
functions, responsibilities and situation on the ground and this can be done
by undertaking a thorough study of the

Page 40 of 1072

Subsequently, the Secretary General of the VFP sent an undated letter


to respondent DND Secretary, with notice to respondent Undersecretary for
Civil Relations and Administration, complaining about the alleged broadness
of the scope of the management audit and requesting the suspension thereof
until such time that specific areas of the audit shall have been agreed upon.
The request was, however, denied by the Undersecretary. Petitioner
thus filed Certiorari with Prohibition under Rule 65 of the 1997 Rules of Civil
Procedure.
ISSUE/S:
Is the VFP a Private non-government Corporation
RULING:
The Supreme Court is constrained to rule that petitioner is in fact a
public corporation. Before responding to petitioners allegations one by one,
here are the more evident reasons why the VFP is a public corporation:
1) Rep. Act No. 2640 is entitled "An Act to Create a Public
Corporation to be Known as the Veterans Federation of the
Philippines, Defining its Powers, and for Other Purposes."
2) Any action or decision of the Federation or of the Supreme
Council shall be subject to the approval of the Secretary of
Defense.
3) The VFP is required to submit annual reports of its
proceedings for the past year, including a full, complete
and itemized report of receipts and expenditures of
whatever kind, to the President of the Philippines or to the
Secretary of National Defense.
4) Under Executive Order No. 37 dated 2 December 1992, the
VFP was listed as among the government-owned and
controlled corporations that will not be privatized.
5) In Ang Bagong Bayani OFW Labor Party v. COMELEC this
Court held in a minute resolution that the "VFP [Veterans
Federation Party] is an adjunct of the government, as it is
merely an incarnation of the Veterans Federation of the
Philippines.

Page 41 of 1072

TOPIC:CLASSIFICATIONS OF CORPORATION
MANILA INTERNATIONAL AIRPORT AUTHORITY,
vs.
COURT OF APPEALS, CITY OF PARAAQUE, CITY MAYOR OF
PARAAQUE, SANGGUNIANG PANGLUNGSOD NG PARAAQUE, CITY
ASSESSOR OF PARAAQUE, and CITY TREASURER OF PARAAQUE,
G.R. No. 155650. July 20, 2006
FACTS:
Petitioner Manila International Airport Authority operates the Ninoy
Aquino International Airport (NAIA) Complex in Paraaque City under
Executive Order No. 903, otherwise known as the Revised Charter of the
Manila International Airport Authority (MIAA Charter). Executive Order No.
903 was issued on 21 July 1983 by then President Ferdinand E. Marcos.
Subsequently, Executive Order Nos. 909 and 298 amended the MIAA
Charter.
As operator of the international airport, MIAA administers the land,
improvements and equipment within the NAIA Complex. The MIAA Charter
transferred to MIAA approximately 600 hectares of land, including the
runways and buildings then under the Bureau of Air Transportation. The
MIAA Charter further provides that no portion of the land transferred to MIAA
shall be disposed of through sale or any other mode unless specifically
approved by the President of the Philippines.
The Office of the Government Corporate Counsel (OGCC) issued
Opinion No. 061. The OGCC opined that the Local Government Code of 1991
withdrew the exemption from real estate tax granted to MIAA under Section
21 of the MIAA Charter. Thus, MIAA negotiated with respondent City of
Paraaque to pay the real estate tax imposed by the City. MIAA then paid
some of the real estate tax already due.

Page 42 of 1072

On 28 June 2001, MIAA received Final Notices of Real Estate Tax


Delinquency from the City of Paraaque for the taxable years 1992 to 2001.
On 17 July 2001, the City of Paraaque, through its City Treasurer,
issued notices of levy and warrants of levy on the Airport Lands and
Buildings. The Mayor of the City of Paraaque threatened to sell at public
auction the Airport Lands and Buildings should MIAA fail to pay the real
estate tax delinquency. MIAA thus sought a clarification of OGCC Opinion No.
061.
ISSUE:
Whether MIAA is a government-owned or controlled corporation.
RULING:
The Supreme Court ruled that MIAAs Airport Lands and Buildings are
exempt from real estate tax imposed by local governments.
First, MIAA is not a government-owned or controlled corporation
but an instrumentality of the National Government and thus
exempt from local taxation. Second, the real properties of MIAA
are owned by the Republic of the Philippines and thus exempt
from real estate tax.
MIAA is Not a Government-Owned or Controlled Corporation
Respondents argue that MIAA, being a government-owned or
controlled corporation, is not exempt from real estate tax. Respondents claim
that the deletion of the phrase any government-owned or controlled so
exempt by its charter in Section 234(e) of the Local Government Code
withdrew the real estate tax exemption of government-owned or controlled
corporations. The deleted phrase appeared in Section 40(a) of the 1974 Real
Property Tax Code enumerating the entities exempt from real estate tax.
There is no dispute that a government-owned or controlled corporation
is not exempt from real estate tax. However, MIAA is not a governmentowned or controlled corporation.
MIAA is a government instrumentality vested with corporate powers to
perform efficiently its governmental functions. MIAA is like any other
government instrumentality, the only difference is that MIAA is vested with
corporate powers.
When the law vests in a government instrumentality corporate powers,
the instrumentality does not become a corporation. Unless the government

Page 43 of 1072

instrumentality is organized as a stock or non-stock corporation, it remains a


government instrumentality exercising not only governmental but also
corporate powers. Thus, MIAA exercises the governmental powers of eminent
domain, police authority and the levying of fees and charges. At the same
time, MIAA exercises all the powers of a corporation under the Corporation
Law, insofar as these powers are not inconsistent with the provisions of this
Executive Order.
MIAA is a Mere Trustee of the Republic
MIAA is merely holding title to the Airport Lands and Buildings in trust
for the Republic. Section 48, Chapter 12, Book I of the Administrative Code
allows instrumentalities like MIAA to hold title to real properties owned by the
Republic.
Under Section 2(10) and (13) of the Introductory Provisions of the
Administrative Code, which governs the legal relation and status of
government units, agencies and offices within the entire government
machinery, MIAA is a government instrumentality and not a governmentowned or controlled corporation. Under Section 133(o) of the Local
Government Code, MIAA as a government instrumentality is not a taxable
person because it is not subject to taxes, fees or charges of any kind by
local governments. The only exception is when MIAA leases its real property
to a taxable person as provided in Section 234(a) of the Local Government
Code, in which case the specific real property leased becomes subject to real
estate tax. Thus, only portions of the Airport Lands and Buildings leased to
taxable persons like private parties are subject to real estate tax by the City
of Paraaque.

Page 44 of 1072

TOPIC: STAGES IN THE FORMATION/ORGANIZATION OF A CORPORATION


MARC II MARKETING, INC. AND LUCILA
v.
JOSON
G.R. No. 171993 ; December 12, 2011
FACTS:
THE board of directors of petitioner Marc II Marketing, Inc. conducted a
meeting on Aug. 29, 1994 where respondent Alfredo M. Joson was appointed
as one of its corporate officers, with the designation or title of general
manager. He was supposed to function as a managing director with other
duties and responsibilities that the board may provide and authorize.
On June 30, 1997, petitioner corporation decided to stop its operations due
to poor sales collection, aggravated by the inefficient management of its
affairs.
Consequently, the services of respondent as general manager were
terminated.
Aggrieved, respondent filed a complaint for reinstatement and money claims
against petitioner.
ISSUE:
Is the controversy intra-corporate?
RULING:
No., That respondent was also a director and a stockholder of
petitioner corporation will not automatically make the case fall within the
ambit of intra-corporate controversy and be subjected to the Regional Trial
Courts jurisdiction. Not all conflicts between the stockholders and the

Page 45 of 1072

corporation are classified as intra-corporate. Other factors such as the status


or relationship of the parties and the nature of the question that is the
subject of the controversy must be considered in determining whether the
dispute involves corporate matters so as to regard them as intra-corporate
controversies.
As previously discussed, respondent was not a corporate officer of petitioner
corporation but a mere employee thereof so there was no intra-corporate
relationship between them. With regard to the subject of the controversy or
issue involved herein, i.e., respondents dismissal as petitioner corporations
General Manager, the same did not present or relate to an intra-corporate
dispute.

TOPIC: STAGES IN THE FORMATION/ORGANIZATION OF A CORPORATION


CAGAYAN FISHING DEVELOPMENT CO., INC., plaintiff-appellant,
vs.
TEODORO SANDIKO, defendant-appellee.
G.R. No. L-43350 December 23, 1937
FACTS:
Manuel Tabora owns four parcels of land which he mortgages in three
separate occasions. The first two mortgages were to PNB and the third to
Severina Buzon. All three mortgages were registered and annotated at the
back of the title. The mortgages were made in order for Tabora to have
enough funds to form a fishery businesses. However, he experienced
financial reverses. He formed a corporation (Cagayan Fishing) composed of
himself, his wife, and a few others. From the articles of incorporation, it
appears that out of the P48,700, amount of capital stock subscribed, P45,000
was subscribed by Manuel Tabora himself and P500 by his wife. Both Tabora
and His wife were directors and the latter was treasurer as well.
In MAY 31, 1930, Tabora executed a public document entitled Escritura
de Transpaso de Propiedad Inmueble (Deed of Transfer of Real Property),
selling such parcels of land to Cagayan Fishing Development Co., Inc. The
sale was concluded with a consideration of P1 and subject to the mortgages
mentioned. The agreement was that the title to the lands will not be
transferred unless Cagayan Fishing will pay Taboras indebtedness.

Page 46 of 1072

Cagayan Fishing filed its articles of incorporation with the Bureau of


Commerce and Industry on OCTOBER 22, 1930. In October of 1931, the
board of directors of the company authorized its president Jose Ventura) to
sell such parcels of lands to Teodoro Sandiko. Sandiko obligated himself to
pay shoulder the three mortgages earlier described and issued a promissory
note in favour of the company. However, Sandiko failed to pay for the
amount stipulated upon, causing Cagayan Fishing to file a case for collection
of sum of money against him.
ISSUE/S:
Whether or not the sale between Cagayan Fishing and Sandiko was
valid.
RULING:
The initial transfer of the land (from Tabora to Cagayan Fishing)
occurred in May of 1930 while the company was incorporated only in October
of the same year. Hence, the transfer occurred five months before the
company was incorporated. While a DULY ORGANIZED corporation can
purchase and hold property for the purposes of its business, the same cannot
be said for a company that has not yet been incorporated. The contract
between Tabora and Cagayan Fishing referred to the company as "una
sociedad en vias de incorporacion" (a society in the process of
incorporation). It was not a de facto corporation at the time it entered into
the contracts. It had not legal personality to do so.
Corporations are creatures of the law, and can only come into
existence in the manner prescribed by law. A corporation, until organized,
has no being, franchises or faculties. Nor do those engaged in bringing it into
being have any power to bind it by contract, unless so authorized by the
charter there is not a corporation nor does it possess franchise or faculties
for it or others to exercise, until it acquires a complete existence. Since
Cagayan Fishing did not have any legal personality when the lands were
transferred to it, it also does not have the right to transfer the same to
Sandiko.
However, the court held that the contract was entered into not
between Manuel Tabora and a non-existent corporation but between the
Manuel Tabora as owner of the four parcels of lands on the one hand and the
same Manuel Tabora, his wife and others, as mere promoters of a
corporations on the other hand. These promoters could not have acted as
agent for a projected corporation since that which no legal existence could
have no agent. A corporation, until organized, has no life and therefore no
faculties. It is, as it were, a child in ventre sa mere.

Page 47 of 1072

TOPIC: STAGES IN THE FORMATION/ORGANIZATION OF A CORPORATION


FERMIN Z. CARAM, JR. and ROSA O. DE CARAM, petitioners
vs.
THE HONORABLE COURT OF APPEALS and ALBERTO V. ARELLANO,
respondents.
G.R. No. L-48627 June 30, 1987
FACTS:
Barretto and Garcia requested Arellano to conduct a project study
regarding putting up an airline business. Arellano was also tasked to perform
pre-organizational services and was promised P50,000 as compensation. The
project study was revised before being presented to prospective investors.
Such investors are petitioners in this case. Garcia, Caram and Barreto
became members of the board of the resultant corporation which is Filipinas
Orient Airways.
The lower court held that the petitioners (who are subsequent
investors of the company) are jointly and severally liable with the corporation
for the compensation of Arellano. The petitioners state that they merely
invested in the corporation after the project study was made and it was
Barreto who was the moving spirit in the creation of the company.

Page 48 of 1072

ISSUE/S:
Who should be liable for the compensation of the promoter
RULING:
Petitioners were not involved in the initial stages of the organization of
the airline, which were being directed by Barretto as the main promoter. The
petitioners were merely among the financiers whose interest was to be
invited and who were in fact persuaded, on the strength of the project study,
to invest in the proposed airline.
Also, there was no showing that the Filipinas Orient Airways was a fictitious
corporation and did not have a separate juridical personality, to justify
making the petitioners, as principal stockholders thereof, responsible for its
obligations. As a bona fide corporation, the Filipinas Orient Airways should
alone be liable for its corporate acts as duly authorized by its officers and
directors.
Hence, the petitioners cannot be held personally liable for the
compensation claimed by the private respondent for the services performed
by him in the organization of the corporation. Te petitioners did not contract
such services. The most that can be said is that they benefited from such
services, but that surely is no justification to hold them personally liable
therefore. Otherwise, all the other stockholders of the corporation, including
those who came in later, and regardless of the amount of their share
holdings, would be equally and personally liable also with the petitioners for
the claims of the private respondent.

Page 49 of 1072

TOPIC: STAGES IN THE FORMATION/ORGANIZATION OF A CORPORATION


PIONEER INSURANCE & SURETY CORPORATION, petitioner,
vs.
THE HON. COURT OF APPEALS, BORDER MACHINERY & HEAVY
EQUIPMENT, INC., (BORMAHECO), CONSTANCIO M. MAGLANA and
JACOB S. LIM, respondents.
G.R. No. 84197 July 28, 1989
FACTS:
Jacob Lim owned and operated Southern Air Lines (SAL) as a single
proprietorship. He sought to expand his business, hence, he offered certain
people the chance to join him in making his business a corporation. Due to
such representations, Border Machinery and Heavy Equipment Company, Inc.
(Bormaheco), Francisco and Modesto Cervantes (Cervanteses) and
Constancio Maglana (Maglana) contributed some funds.
The funds were used to purchase two aircrafts and certain parts from
Japan Domestic Airlines (JDA). Lim had the transaction secured by a surety
bond issued by Pioneer Insurance and Surety Corporation.
Lim also executed chattel mortgages over the aircrafts and equipment.
Lim defaulted on his payments, prompting JDA to seek payment from
Pioneer. Pioneer paid the balance of the purchase price and now seeks to
foreclose the mortgage over the aircrafts. The contributors (Bormaheco,

Page 50 of 1072

Cervanteses and Maglana) intervened, claiming that they were part owners
of such aircrafts and that Lim acted in his own name and not in the name of
the supposed corporation. It should be noted that despite Lims
representations, he did not form incorporate his businesses and did not
include the contributors in his plans.
ISSUE/S:
Should the contributors be held liable for the amount owed to Pioneer
RULING:
While it has been held that as between themselves the rights of the
stockholders in a defectively incorporated association should be governed by
the supposed charter and the laws of the state relating thereto and not by
the rules governing partners, it is ordinarily held that persons who attempt,
but fail, to form a corporation and who carry on business under the corporate
name occupy the position of partners inter se. Thus, where persons associate
themselves together under articles to purchase property to carry on a
business, and their organization is so defective as to come short of creating a
corporation within the statute, they become in legal effect partners inter se,
and their rights as members of the company to the property acquired by the
company will be recognized. However, such a relation does not necessarily
exist, for ordinarily persons cannot be made to assume the relation of
partners, as between themselves, when their purpose is that no partnership
shall exist, and it should be implied only when necessary to do justice
between the parties; thus, one who takes no part except to subscribe for
stock in a proposed corporation which is never legally formed does not
become a partner with other subscribers who engage in business under the
name of the pretended corporation, so as to be liable as such in an action for
settlement of the alleged partnership and contribution. A partnership relation
between certain stockholders and other stockholders, who were also
directors, will not be implied in the absence of an agreement, so as to make
the former liable to contribute for payment of debts illegally contracted by
the latter.
The petitioner never had the intention to form a corporation with the
respondents despite his representations to them. This gives credence to the
cross-claims of the respondents to the effect that they were induced and
lured by the petitioner to make contributions to a proposed corporation
which was never formed because the petitioner reneged on their agreement.
No de facto partnership was created among the parties which would
entitle the petitioner to a reimbursement of the supposed losses of the
proposed corporation. The record shows that the petitioner was acting on his
own and not in behalf of his other would-be incorporators in transacting the
sale of the airplanes and spare parts.

Page 51 of 1072

TOPIC: STAGES IN THE FORMATION/ORGANIZATION OF A CORPORATION


RIZAL LIGHT & ICE CO., INC., petitioner,
vs.
THE MUNICIPALITY OF MORONG, RIZAL and THE PUBLIC SERVICE
COMMISSION, respondents.
G.R. No. L-20993
September 28, 1968
FACTS:
Rizal Light and Ice Co., Inc. is a duly organized corporation that was
granted a certificate of public convenience (CPC) for the installation,
operation and maintenance of an electric light, heat and power service in the
municipality of Morong, Rizal. However, it was found that it failed to fulfil its
duties as stated in its certificate and in the regulations of the Public Service
Commission. Hence, its CPC was cancelled. While Rizal Co was contesting the
cancellation of its CPC, the Commission granted Morong Electric Co a CPC to
install, operate and maintain an electric heat, light and power service in said
municipality. Rizal opposed such act, stating that Morong Electric Co did not
have a corporate personality when the franchise was granted to it since it
was incorporated in October 1962 while the CPC was given in May of the
same year.
ISSUE/S:

Page 52 of 1072

Whether or not the Morong Electric Co was incapacitated to apply for


and accept a CPC.
RULING:
The juridical personality and legal existence of Morong Electric began
only on October 17, 1962 when its certificate of incorporation was issued by
the SEC. Before that date, or pending the issuance of said certificate of
incorporation, the incorporators cannot be considered as de facto
corporation. But the fact that Morong Electric had no corporate existence on
the day the franchise was granted in its name does not render the franchise
invalid, because later Morong Electric obtained its certificate of incorporation
and then accepted the franchise in accordance with the terms and conditions
thereof.
The fact that a company is not completely incorporated at the time the
grant is made to it by a municipality to use the streets does not, in most
jurisdictions, affect the validity of the grant. But such grant cannot take
effect until the corporation is organized. The reason is that a privilege of this
character is a mere license to the corporation until it accepts the grant and
complies with its terms and conditions.
The incorporation of Morong Electric on October 17, 1962 and its
acceptance of the franchise as shown by its action in prosecuting the
application filed with the Commission for the approval of said franchise, not
only perfected a contract between the respondent municipality and Morong
Electric but also cured the deficiency pointed out by the petitioner in the
application of Morong Electric.
The efficacy of the franchise, however, arose only upon its approval by
the Commission on March 13, 1963. The reason is that
Under Act No. 667, as amended by Act No. 1022, a municipal
council has the power to grant electric franchises, subject to the
approval of the provincial board and the President. However,
under Section 16(b) of Commonwealth Act No. 146, as amended,
the Public Service Commission is empowered "to approve,
subject to constitutional limitations any franchise or privilege
granted under the provisions of Act No. 667, as amended by Act
No. 1022, by any political subdivision of the Philippines when, in
the judgment of the Commission, such franchise or privilege will
properly conserve the public interests and the Commission shall
in so approving impose such conditions as to construction,
equipment, maintenance, service, or operation as the public
interests and convenience may reasonably require, and to issue
certificates of public convenience and necessity when such is
required or provided by any law or franchise." Thus, the efficacy

Page 53 of 1072

of a municipal electric franchise arises, therefore, only after the


approval of the Public Service Commission.

TOPIC: CONTENTS OF ARTICLES OF INCORPORATION


JESUS V. LANUZA, MAGADYA REYES, BAYANI REYES and ARIEL REYES,
Petitioner,
vs.
COURT OF APPEALS, SECURITIES AND EXCHANGE COMMISSION,
DOLORES ONRUBIA, ELENITA NOLASCO, JUAN O. NOLASCO III, ESTATE
OF FAUSTINA M. ONRUBIA, PHILIPPINE MERCHANT MARINE SCHOOL,
INC., Respondents.
G.R. No. 131394
March 28, 2005
FACTS:
In 1952, the Philippine Merchant Marine School, Inc. (PMMSI) was
incorporated, with seven hundred (700) founders shares and seventy-six
(76) common shares as its initial capital stock subscription reflected in the
articles of incorporation. Sometime in 1979, a special stockholders meeting
was called and held on the basis of what was considered as a quorum of
twenty-seven (27) common shares, representing more than two-thirds (2/3)
of the common shares issued and outstanding.
On 06 May 192, a special stockholders meeting was held to elect a
new set of directors. Private respondents thereafter filed a petition with the

Page 54 of 1072

SEC questioning the validity of the 06 May 1992 stockholders meeting


alleging that the quorum for the said meeting should not be based on the
165 issued and outstanding shares as per the stock and transfer book, but on
the initial subscribed capital stock of seven hundred seventy-six (776)
shares, as reflected in the 1952 Articles of Incorporation
ISSUE/S:
Whether it is the companys stock and transfer book, or its 1952
Articles of Incorporation, which determines stockholders shareholding
and provides the basis for computing the quorum.
RULING:
The Articles Of Incorporation has been described as one of that defines
the carter of the corporation and the contractual relationships between the
State and the corporation, the stockholders and the State, and between the
corporation and its stockholders.
When PMMSI was incorporated the
prevailing law was Act No. 1459, otherwise known as The Corporation Law,
review of PMMSIs articles of incorporation shows that the corporation
complied with the requirements list down by Act. No, 1459, otherwise known
as "The Corporation Law." Section 6 thereof states:
Sec. 6. Five or more persons, not exceeding fifteen, a majority of
whom are residents of the Philippines, may form a private
corporation for any lawful purpose or purposes by filing with the
Securities and Exchange Commission articles of incorporation
duly executed and acknowledged before a notary public, setting
forth:
....
(7) If it be a stock corporation, the amount of its capital stock, in
lawful money of the Philippines, and the number of shares into
which it is divided, and if such stock be in whole or in part
without par value then such fact shall be stated; Provided,
however, That as to stock without par value the articles of
incorporation need only state the number of shares into which
said capital stock is divided.
(8) If it be a stock corporation, the amount of capital stock or
number of shares of no-par stock actually subscribed the amount
or number of shares of no-par stock subscribed by each and the
sum paid by each on his subscription. . . .
A review of PMMSIs articles of incorporationshows that the corporation
complied with the requirements laid down by Act No. 1459.

Page 55 of 1072

TOPIC: CORPORATE NAME


FRANCISCO M. ALONSO, substituted by his heirs, petitioners,
vs.
CEBU COUNTRY CLUB, INC., respondent.
G.R. No. 130876, January 31, 2002
FACTS:
Petitioner Francisco M. Alonso, who died pendente lite and substituted
by his legal heirs, a lawyer by profession, the only son and sole heir of the
late Tomas N. Alonso and Asuncion Medalle, who died on June 16, 1962 and
August 18, 1963, respectively. Cebu Country Club, Inc. is a non-stock, nonprofit corporation duly organized and existing under Philippine Laws the
purpose of which is to cater to the recreation and leisure of its members.
Sometime in 1992, petitioner discovered documents and records
showing that his father acquired Lot No. 727 of the Banilad Friar Lands Estate
from the Government of the Philippine Islands in or about the year 1911 in
accordance with the Friar Lands Act (Act No. 1120). It appears, however, that
the deed was not registered with the Register of Deeds because of lack of
technical requirements, among them the approval of the deed of sale by the
Secretary of Agriculture and Natural Resources, as required by law. Upon

Page 56 of 1072

investigation of the status of the land, petitioner found out from the office of
the Registrar of Deeds of Cebu City that title to Lot No. 727 of the Banilad
Friar Lands Estate had been "administratively reconstituted from the owners
duplicate" on July 26, 1948 under Transfer Certificate of Title (TCT) No. RT1310 (T-11351) in the name of United Service Country Club, Inc., predecessor
of Cebu Country Club, Inc. On March 8, 1960, upon order of the Court of First
Instance, the name of the registered owner in TCT No. RT-1310 (T-11531) was
changed to Cebu Country Club, Inc.
In the firm belief that petitioners father is still the rightful owner of Lot
No. 727 of the Banilad Friar Lands Estate since there are no records showing
that he ever sold or conveyed the disputed property to anyone, on July 7,
1992, petitioner made a formal demand upon Cebu Country Club, Inc. to
restore to him the ownership and possession of said lot within fifteen (15)
days from receipt thereof.
Left with no other recourse, on September 25, 1992, petitioner filed
with the Regional Trial Court, Cebu City,a complaint for declaration of nullity
and non existence of deed/title, cancellation of certificates of title and
recovery of property against defendant Cebu Country Club, Inc. November 5,
1992, Cebu Country Club, Inc. filed with the trial court its answer with
counterclaim.
On May 7, 1993, the trial court rendered a decision in favor of the
defendant and against the plaintiff, declaring the contested property or Lot
727 as legally belonging to the defendant. On March 31, 1997, the Court of
Appeals promulgated a decision affirming the lower courts decision. On
October 24, 2000, we required the Solicitor General to file comment on the
issue of the validity of the re-constituted title in dispute.On November 8,
2000, the Solicitor General submitted a comment stating that on the basis of
information received from the Land Registration Authority (LRA) and the Land
Management Bureau (LMB), the Cebu Country Club, Inc. had been occupying
the disputed property even before the Second World War and developed it
into a golf course and must have acquired the property in a proper and valid
manner

ISSUE/S:
Whether or not the Court of Appeals lawfully adjudged the validity of
the administrative reconstitution of the title of Cebu Country Club, Inc.
over the OCT of the Government of the Philippine Islands and Sales
Patent No. 14353 on Lot No. 727 in the name of Tomas N. Alonso.
RULING:

Page 57 of 1072

The issue is factual, which, as aforesaid, cannot be reviewed in this


appeal. Nevertheless, petitioners assail the validity of the administrative
reconstitution of Cebu Country Club, Inc.s title No. RT-1310 (T-11351) on
three (3) grounds: Its source title bears the same number as another title
which refers to another parcel of land; There is no recorded transaction of
the land from Tomas Alonso in favor of Cebu Country Club, Inc.; and The
technical description was not transcribed in the title within two (2) years
from the date of its reconstitution. None of the grounds has any basis or
merit.
On the question that TCT No. RT-1310 (T-11351) bears the same
number as another title to another land, we agree with the Court of Appeals
that there is nothing fraudulent with the fact that Cebu Country Club, Inc.s
reconstituted title bears the same number as the title of another parcel of
land.
Petitioners next argue that the reconstituted title of Cebu Country Club,
Inc. had no lawful source to speak of; it was reconstituted through extrinsic
and intrinsic fraud in the absence of a deed of conveyance in its favor. In
truth, however, reconstitution was based on the owners duplicate of the
title, hence, there was no need for the covering deed of sale or other modes
of conveyance. Cebu Country Club, Inc. was admittedly in possession of the
land since long before the Second World War, or since 1931. Petitioner failed
to adduce evidence of fraud. In an action for re-conveyance based on fraud,
he who charges fraud must prove such fraud in obtaining a title. Worse, the
imputation of fraud was so tardily brought, some forty-four (44) years or
sixty-one (61) years after its supposed occurrence, that is, from the
administrative reconstitution of title on July 26, 1948, or from the issuance of
the original title on November 19, 1931, that verification is rendered
extremely difficult, if not impossible, especially due to the supervening event
of the second world war during which practically all public records were lost
or destroyed, or no longer available.
Petitioners next question the lack of technical description inscribed in
the reconstituted title in Cebu Country Club, Inc.s name. This is not a bar to
reconstitution of the title nor will it affect the validity of the reconstituted
title. A registered owner is given two (2) years to file a plan of such land with
the Chief of the General Land Registration Office. The two-year period is
directory, not jurisdictional. In other words, the failure to submit the technical
description within two (2) years would not invalidate the title. At most, the
failure to file such technical description within the two-year period would bar
a transfer of the title to a third party in a voluntary transaction.

Page 58 of 1072

TOPIC:CORPORATE NAME
INDUSTRIAL REFRACTORIES CORPORATION OF THE PHILIPPINES,
petitioner, vs.
COURT OF APPEALS, SECURITIES AND EXCHANGE COMMISSION and
REFRACTORIES CORPORATION OF THE PHILIPPINES, respondents.
G.R. No. 122174.October 3, 2002
FACTS:
Respondent Refractories Corporation of the Philippines (RCP) is a
corporation duly organized on October 13, 1976 for the purpose of engaging
in the business of manufacturing, producing, selling, exporting and otherwise
dealing in any and all refractory bricks, its by-products and derivatives. On
June 22, 1977, it registered its corporate and business name with the Bureau
of Domestic Trade.
Petitioner IRCP on the other hand, was incorporated on August 23,
1979 originally under the name "Synclaire Manufacturing Corporation". It
amended its Articles of Incorporation on August 23, 1985 to change its
corporate name to "Industrial Refractories Corp. of the Philippines". It is

Page 59 of 1072

engaged in the business of manufacturing all kinds of ceramics and other


products, except paints and zincs.
Both companies are the only local suppliers of monolithic gunning
mix.Discovering that petitioner was using such corporate name, respondent
RCP filed on April 14, 1988 with the Securities and Exchange Commission
(SEC) a petition to compel petitioner to change its corporate name on the
ground that its corporate name is confusingly similar with that of petitioners
such that the public may be confused or deceived into believing that they
are one and the same corporation.
The SEC decided in favor the petitioner and against the respondent
declaring the latters corporate name Industrial Refractories Corporation of
the Philippines as deceptively and confusingly similar to that of petitioners
corporate name Refractories Corporation of the Philippines. the SEC En
Banc modified the appealed decision in that petitioner was ordered to delete
or drop from its corporate name only the word "Refractories". Petitioner IRCP
elevated the decision of the SEC En Banc through a petition for review on
certiorari to the Court of Appeals which then rendered the herein assailed
decision. The appellate court upheld the jurisdiction of the SEC over the case
and ruled that the corporate names of petitioner IRCP and respondent RCP
are confusingly or deceptively similar, and that respondent RCP has
established its prior right to use the word "Refractories" as its corporate
name. The appellate court also found that the petition was filed beyond the
reglementary period.

ISSUE/S:
Whether or not there is confusing or deceptive similarity between
petitioner and respondent RCPs corporate names.
RULING:
Pursuant thereto, the Revised Guidelines in the Approval of Corporate
and Partnership Names specifically requires that:
1) a corporate name shall not be identical, misleading or
confusingly similar to one already registered by another
corporation with the Commission; and
2) if the proposed name is similar to the name of a registered
firm, the proposed name must contain at least one distinctive
word different from the name of the company already
registered.
In this case, respondent RCP was incorporated on October 13, 1976
and since then has been using the corporate name "Refractories Corp. of the
Philippines". Meanwhile, petitioner was incorporated on August 23, 1979
originally under the name "Synclaire Manufacturing Corporation". It only

Page 60 of 1072

started using the name "Industrial Refractories Corp. of the Philippines" when
it amended its Articles of Incorporation on August 23, 1985, or nine (9) years
after respondent RCP started using its name. Thus, being the prior registrant,
respondent RCP has acquired the right to use the word "Refractories" as part
of its corporate name.
Anent the second requisite, in determining the existence of confusing
similarity in corporate names, the test is whether the similarity is such as to
mislead a person using ordinary care and discrimination and the Court must
look to the record as well as the names themselves. Petitioners corporate
name is "Industrial Refractories Corp. of the Phils.", while respondents is
"Refractories Corp. of the Phils." Obviously, both names contain the identical
words "Refractories", "Corporation" and "Philippines". The only word that
distinguishes petitioner from respondent RCP is the word "Industrial" which
merely identifies a corporations general field of activities or operations. We
need not linger on these two corporate names to conclude that they are
patently similar that even with reasonable care and observation, confusion
might arise. It must be noted that both cater to the same clientele, i.e. the
steel industry. In fact, the SEC found that there were instances when different
steel companies were actually confused between the two, especially since
they also have similar product packaging. Such findings are accorded not
only great respect but even finality, and are binding upon this Court, unless it
is shown that it had arbitrarily disregarded or misapprehended evidence
before it to such an extent as to compel a contrary conclusion had such
evidence been properly appreciated. And even without such proof of actual
confusion between the two corporate names, it suffices that confusion is
probable or likely to occur.
TOPIC: CORPORATE NAME
ANG MGA KAANIB SA IGLESIA NG DIOS KAY KRISTO HESUS, H.S.K. SA
BANSANG PILIPINAS, INC., petitioner,
vs.
IGLESIA NG DIOS KAY CRISTO JESUS, HALIGI AT SUHAY NG
KATOTOHANAN, respondent.
G.R. No. 137592.December 12, 2001
FACTS:
Respondent Iglesia ng Dios Kay Cristo Jesus, Haligi at Suhay ng
Katotohanan, is a non-stock religious society or corporation registered in
1936. In 1976, one Eliseo Soriano and several other members of respondent
corporation disassociated themselves from the latter and succeeded in
registering on March 30, 1977 a new non-stock religious society or
corporation, named Iglesia ng Dios Kay Kristo Hesus, Haligi at Saligan ng
Katotohanan.

Page 61 of 1072

On July 16, 1979, respondent corporation filed with the SEC a petition
to compel the Iglesia ng Dios Kay Kristo Hesus, Haligi at Saligan ng
Katotohanan to change its corporate name, which petition was docketed as
SEC Case No. 1774. It appears that during the pendency of SEC Case No.
1774, Soriano, et al., caused the registration on April 25, 1980 of petitioner
corporation, Ang Mga Kaanib sa Iglesia ng Dios Kay Kristo Hesus, H.S.K, sa
Bansang Pilipinas. The acronym "H.S.K." stands for Haligi at Saligan ng
Katotohanan. On March 2, 1994, respondent corporation filed before the SEC
a petition, praying that petitioner be compelled to change its corporate name
and be barred from using the same or similar name on the ground that the
same causes confusion among their members as well as the public.
Petitioner filed a petition for review with the Court of Appeals. On October 7,
1997, the Court of Appeals rendered the assailed decision affirming the
decision of the SEC En Banc. Petitioner's motion for reconsideration was
denied by the Court of Appeals on February 16, 1992.
ISSUE/S:
Whether or not the Court of Appeals failed to consider and properly
apply the exceptions established by jurisprudence in the application of
section 18 of the corporation code to the instant case.
RULING:
It is the duty of the SEC to prevent confusion in the use of corporate
names not only for the protection of the corporations involved but more so
for the protection of the public. Parties organizing a corporation must choose
a name at their peril; and the use of a name similar to one adopted by
another corporation, whether a business or a nonprofit organization, if
misleading or likely to injure in the exercise of its corporate functions,
regardless of intent, may be prevented by the corporation having a prior
right, by a suit for injunction against the new corporation to prevent the use
of the name.
The additional words "Ang Mga Kaanib" and "Sa Bansang Pilipinas, Inc."
in petitioner's name are, as correctly observed by the SEC, merely
descriptive of and also referring to the members, or kaanib, of respondent
who are likewise residing in the Philippines. These words can hardly serve as
an effective differentiating medium necessary to avoid confusion or difficulty
in distinguishing petitioner from respondent. This is especially so, since both
petitioner and respondent corporations are using the same acronym H.S.K.;
not to mention the fact that both are espousing religious beliefs and
operating in the same place. Parenthetically, it is well to mention that the
acronym H.S.K. used by petitioner stands for "Haligi at Saligan ng
Katotohanan."

Page 62 of 1072

TOPIC: CORPORATE NAME


UNIVERSAL MILLS CORPORATION, petitioner,
vs.
UNIVERSAL TEXTILE MILLS, INC., respondent.
G.R. No.L-28351. July 28, 1977
FACTS:
The Universal Textile Mills, Inc. was organ on December 29, 1953, as a
textile manufacturing firm for which it was issued a certificate of registration
on January 8, 1954. The Universal Mills Corporation, on the other hand, was
registered in this Commission on October 27, 1954, under its original name,
Universal Hosiery Mills Corporation, having as its primary purpose the
"manufacture and production of hosieries and wearing apparel of all kinds."

Page 63 of 1072

On May 24, 1963, it filed an amendment to its articles of incorporation


changing its name to Universal Mills Corporation, its present name, for which
this Commission issued the certificate of approval on June 10, 1963.
The immediate cause of this present complaint, however, was the
occurrence of a fire which gutted respondent's spinning mills in Pasig, Rizal.
Petitioner alleged that as a result of this fire and because of the similarity of
respondent's name to that of herein complainant, the news items appearing
in the various metropolitan newspapers carrying reports on the fire created
uncertainty and confusion among its bankers, friends, stockholders and
customers prompting petitioner to make announcements, clarifying the real
Identity of the corporation whose property was burned. Petitioner presented
documentary and testimonial evidence in support of this allegation.
ISSUE/S:
Whether or not the order of the Commission enjoining petitioner to its
corporate name constitutes, in the light of the circumstances found by
the Commission, a grave abuse of discretion.
RULING:
The Supreme Court believesthat it is not. Indeed, it cannot be said that
the impugned order is arbitrary and capricious. The corporate names in
question are not Identical, but they are indisputably so similar that even
under the test of "reasonable care and observation as the public generally
are capable of using and may be expected to exercise" invoked by appellant,
We are apprehensive confusion will usually arise, considering that under the
second amendment of its articles of incorporation on August 14, 1964,
appellant included among its primary purposes the "manufacturing, dyeing,
finishing and selling of fabrics of all kinds" in which respondent had been
engaged for more than a decade ahead of petitioner. Factually, the
Commission found existence of such confusion, and there is evidence to
support its conclusion. Since respondent is not claiming damages in this
proceeding, it is, of course, immaterial whether or not appellant has acted in
good faith, but We cannot perceive why of all names, it had to choose a
name already being used by another firm engaged in practically the same
business for more than a decade enjoying well earned patronage and
goodwill, when there are so many other appropriate names it could possibly
adopt without arousing any suspicion as to its motive and, more importantly,
any degree of confusion in the mind of the public which could mislead even
its own customers, existing or prospective. Premises considered, there is no
warrant for our interference.

Page 64 of 1072

TOPIC: CORPORATE NAME


LYCEUM OF THE PHILIPPINES, INC., petitioner,
vs.
COURT OF APPEALS, LYCEUM OF APARRI, LYCEUM OF CABAGAN,
LYCEUM OF CAMALANIUGAN, INC., LYCEUM OF LALLO, INC., LYCEUM
OF TUAO, INC., BUHI LYCEUM, CENTRAL LYCEUM OF CATANDUANES,
LYCEUM OF SOUTHERN PHILIPPINES, LYCEUM OF EASTERN
MINDANAO, INC. and WESTERN PANGASINAN LYCEUM, INC.,
respondents.
G.R. No. 101897. March 5, 1993.
FACTS:
Petitioner is an educational institution duly registered with the
Securities and Exchange Commission. When it first registered with the SEC

Page 65 of 1072

on 21 September 1950, it used the corporate name Lyceum of the


Philippines, Inc. and has used that name ever since.
On 24 February 1984, petitioner instituted proceedings before the SEC
to compel the private respondents, which are also educational institutions, to
delete the word "Lyceum" from their corporate names and permanently to
enjoin them from using "Lyceum" as part of their respective names.
Petitioner had sometime before commenced in the SEC a proceeding against
the Lyceum of Baguio, Inc. to require it to change its corporate name and to
adopt another name not "similar to or identical" with that of petitioner. In an
Order dated 20 April 1977, Associate Commissioner Julio Sulit held that the
corporate name of petitioner and that of the Lyceum of Baguio, Inc. were
substantially identical because of the presence of a "dominant" word,
"Lyceum," the name of the geographical location of the campus being the
only word which distinguished one from the other corporate name.
ISSUE/S:
Whether or not the Court of Appeals erred in holding that respondent
Western Pangasinan Lyceum, Inc. was incorporated earlier than
petitioner and that Lyceum as a generic word cannot be appropriated
by the petitioner to the exclusion of others.
RULING:
The Supreme Court does not consider that the corporate names of
private respondent institutions are "identical with, or deceptively or
confusingly similar" to that of the petitioner institution. True enough, the
corporate names of private respondent entities all carry the word "Lyceum"
but confusion and deception are effectively precluded by the appending of
geographic names to the word "Lyceum." Thus, we do not believe that the
"Lyceum of Aparri" can be mistaken by the general public for the Lyceum of
the Philippines, or that the "Lyceum of Camalaniugan" would be confused
with the Lyceum of the Philippines.
The Court concludes and so hold that petitioner institution is not
entitled to a legally enforceable exclusive right to use the word "Lyceum" in
its corporate name and that other institutions may use "Lyceum" as part of
their corporate names. To determine whether a given corporate name is
"identical" or "confusingly or deceptively similar" with another entity's
corporate name, it is not enough to ascertain the presence of "Lyceum" or
"Liceo" in both names. One must evaluate corporate names in their entirety
and when the name of petitioner is juxtaposed with the names of private
respondents, they are not reasonably regarded as "identical" or "confusingly
or deceptively similar" with each other.

Page 66 of 1072

TOPIC: CORPORATE NAME


INDIANA AEROSPACE UNIVERSITY, petitioner,
vs.
COMMISSION ON HIGHER EDUCATION (CHED), respondent.
G.R. No. 139371. April 4, 2001
FACTS:
Dr. Reynaldo Vera, Chairman, Technical Panel for Engineering,
Architecture, and Maritime Education (TPRAM) of CHED, received a letter
from Douglas R. Macias, Chairman, Board of Aeronautical Engineering of PRC
and Chairman, Technical Committee for Aeronautical Engineering inquiring
whether herein petitioner had already acquired its UNIVERSITY status in view
of the latters advertisement in the Manila Bulletin.
In response to said letter, Dr. Vera requested the concerned Regional
Office under Chairman Alcala to conduct an investigation on the alleged

Page 67 of 1072

misrepresentation. The investigation was then made by the Regional Director


of Cebu. The report made states that the Director met with petitioners
principal to advised them not to use University unless the school had
complied with the basic requirement of being a University as prescribed in
CHED Memorandum Order No. 48, s. 1996.
Subsequently, respondent directed petitioner to desist from using the
term university, including the use of the same in any of its branches. Further,
respondent found out that petitioner had filed a proposal to amend its
corporate name from Indiana School of Aeronautics to Indiana Aerospace
University, which was supposedly favorably recommended by the
Department of Education, Culture and Sports (DECS) per its Endorsement
dated 17 July 1995, and on that basis, SEC issued to petitioner Certificate of
Registration No. AS-083-002689 dated August 7, 1995. However, SEC
Chairman Perfecto Yasay, Jr. informed respondent in his letter to Chairman
Alcala that petitioner has not filed any amended Articles of Incorporation that
changed its corporate name.
Thereafter, petitioner filed an appeal for reconsideration of
respondents order, promising to follow the provisions of CMO No. 48.
However, respondent rejected said appeal and ordered petitioner to cease
and desist from using the word University.
Consequently petitioner filed a complaint for damages with prayer for
Writ of Preliminary injunction and TRO before the RTC. Said court issued the
Writ of Preliminary Injunction which the CA dissolved.

ISSUE/S:
Whether or not CA erred in dissolving the Writ of Preliminary
Injunction issued by the RTC.
RULING:
NO.
The Supreme Court concurs with the CA that the trial court acted with
grave abuse of discretion in issuing the Writ of Preliminary Injunction against
respondent. Petitioner failed to establish a clear right to continue
representing itself to the public as a university. Indeed, it has no vested right
to misrepresent itself. Before an injunction can be issued, it is essential that:
(1) there must be a right in esse to be protected, and
(2) the act against which the injunction is to be directed must
have violated such right.

Page 68 of 1072

The establishment and the operation of schools are subject to prior


authorization from the government. No school may claim to be a
university unless it has first complied with the prerequisites
provided in Section 34 of the Manual of Regulations for Private
Schools. Section 3, Rule 58 of the Rules of Court, limits the grant of
preliminary injunction to cases in which the plaintiff is clearly entitled to the
relief prayed for.
The Court also agrees with the finding of the CA that the act sought to
be enjoined by petitioner is not violative of the latter's rights. Respondent's
Cease and Desist Order of July 30, 1997 merely restrained petitioner from
using the term "university" in its name. It was not ordered to close, but
merely to revert to its authorized name; hence, its proprietary rights were
not violated.

TOPIC:CORPORATE NAME
PHILIPS EXPORT B.V.,PHILIPS ELECTRICAL LAMPS,INC. and PHILIPS
INDUSTRIAL DEVELOPMENT, INC., petitioners,
vs.
COURT OF APPEALS, SECURITIES & EXCHANGE COMMISSION and
STANDARD PHILIPS CORPORATION, respondents.
G.R. No. 96161. February 21, 1992
FACTS:
Petitioners belong to the PHILIPS Group of Companies.
Petitioner Philip Export B.V. (PEBV), a foreign corporation organized
under the laws of the Netherlands and not engaged in business in the
Philippines is the owner of the trademarks PHILIPS andPHILIPS SHIELD
EMBLEM as registered with the Philippine Patent Office. The two other
petitioners were the authorized users of the said trademarks.

Page 69 of 1072

On the other hand, respondent Standard Philips Corporation was issued


a Certificate of registration by respondent SEC. Thus, petitioners filed a letter
complaint with the SEC for the cancellation of the word PHILIPS from
private respondents corporate name.
For private respondents refusal to amend its Articles of Incorporation,
petitioners filed with the SEC a petition for the issuance of a Writ of
Preliminary Injunction on the ground thatPrivate Respondent's use of the
word PHILIPS amounts to an infringement and clear violation of Petitioners'
exclusive right to use the same considering that both parties engage in the
same business. However, the SEC ruled against the issuance of said writ and
thereafter dismissed the petition for lack of merit.
On appeal, the SEC en bancaffirmed the dismissal declaring that the
corporate names of Petitioners and Private Respondent hardly breed
confusion inasmuch as each contains at least two different words and,
therefore, rules out any possibility of confusing one for the other.
The CA dismissed the petition and the Motion for Reconsideration filed
by petitioners.
ISSUE/S:
Whether or not, petitioners have the exclusive right to use the
word PHILIPS which must be free from any infringement by
similarity.
RULING:
Yes.
A corporation acquires its name by choice and need not select a name
identical with or similar to one already appropriated by a senior corporation
while an individual's name is thrust upon him (See Standard Oil Co. of New
Mexico, Inc. v. Standard Oil Co. of California, 56 F 2d 973, 977). A corporation
can no more use a corporate name in violation of the rights of others than an
individual can use his name legally acquired so as to mislead the public and
injure another (Armington vs. Palmer, 21 RI 109. 42 A 308).
Our own Corporation Code, in its Section 18, expressly provides that:
No corporate name may be allowed by the Securities and
Exchange Commission if the proposed name is identical or
deceptively or confusingly similar to that of any existing
corporation or to any other name already protected by law
or is patently deceptive, confusing or contrary to existing
law. Where a change in a corporate name is approved, the
commission shall issue an amended certificate of
incorporation under the amended name.

Page 70 of 1072

The statutory prohibition cannot be any clearer. To come within its


scope, two requisites must be proven, namely:
(1) That the complainant corporation acquired a prior right over
the use of such corporate name; and
(2) The proposed name is either:
a. Identical; or
b. Deceptively or confusingly similarto that of any
existing corporation or to any other name
already protected by law; or
c. Patently deceptive, confusing or contrary to
existing law.
The right to the exclusive use of a corporate name with freedom from
infringement by similarity is determined by priority of adoption (1 Thompson,
p. 80 citing Munn v. Americana Co., 82 N. Eq. 63, 88 Atl. 30; San Francisco
Oyster House v. Mihich, 75 Wash. 274, 134 Pac. 921). In this regard, there is
no doubt with respect to Petitioners' prior adoption of' the name ''PHILIPS" as
part of its corporate name. Petitioners Philips Electrical and Philips Industrial
were incorporated on 29 August 1956 and 25 May 1956, respectively, while
Respondent Standard Philips was issued a Certificate of Registration on 12
April 1982, twenty-six (26) years later (Rollo, p. 16). Petitioner PEBV has also
used the trademark "PHILIPS" on electrical lamps of all types and their
accessories since 30 September 1922, as evidenced by Certificate of
Registration No. 1651.
The second requisite no less exists in this case. In determining the
existence of confusing similarity in corporate names, the test is whether the
similarity is such as to mislead a person, using ordinary care and
discrimination. In so doing, the Court must look to the record as well as the
names themselves (Ohio Nat. Life Ins. Co. v. Ohio Life Ins. Co., 210 NE 2d
298). While the corporate names of Petitioners and Private Respondent are
not identical, a reading of Petitioner's corporate names, to wit: PHILIPS
EXPORT B.V., PHILIPS ELECTRICAL LAMPS, INC. and PHILIPS INDUSTRIAL
DEVELOPMENT, INC., inevitably leads one to conclude that "PHILIPS" is,
indeed, the dominant word in that all the companies affiliated or associated
with the principal corporation, PEBV, are known in the Philippines and abroad
as the PHILIPS Group of Companies.
Moreover, PHILIPS is a trademark or trade name which was registered
as far back as 1922. Petitioners, therefore, have the exclusive right to its use
which must be free from any infringement by similarity. A corporation has an
exclusive right to the use of its name, which may be protected by injunction
upon a principle similar to that upon which persons are protected in the use
of trademarks and trade names (18 C.J.S. 574). Such principle proceeds upon
the theory that it is a fraud on the corporation which has acquired a right to
that name and perhaps carried on its business there under, that another
should attempt to use the same name, or the same name with a slight
variation in such a way as to induce persons to deal with it in the belief that

Page 71 of 1072

they are dealing with the corporation which has given a reputation to the
name (6 Fletcher [Perm Ed], pp. 39-40, citing Borden Ice Cream Co. v.
Borden's Condensed Milk Co., 210 F 510). Notably, too, Private Respondent's
name actually contains only a single word, that is, "STANDARD", different
from that of Petitioners inasmuch as the inclusion of the term "Corporation"
or "Corp." merely serves the Purpose of distinguishing the corporation from
partnerships and other business organizations.
The fact that there are other companies engaged in other lines of
business using the word "PHILIPS" as part of their corporate names is no
defense and does not warrant the use by Private Respondent of such word
which constitutes an essential feature of Petitioners' corporate name
previously adopted and registered and-having acquired the status of a wellknown mark in the Philippines and internationally as well (Bureau of Patents
Decision No. 88-35 [TM], June 17, 1988, SEC Records

TOPIC:PRIMARY PURPOSE
ALICIA E. GALA, GUIA G. DOMINGO and RITA G. BENSON, petitioners,
vs.
ELLICE AGRO-INDUSTRIAL CORPORATION, MARGO MANAGEMENT
AND DEVELOPMENT CORPORATION, RAUL E. GALA, VITALIANO N.
AGUIRRE II, ADNAN V. ALONTO, ELIAS N. CRESENCIO, MOISES S.
MANIEGO, RODOLFO B. REYNO, RENATO S. GONZALES, VICENTE C.
NOLAN, NESTOR N. BATICULON, respondents.
G.R. No. 156819. December 11, 2003
FACTS:
On March 28, 1979, the Ellice Agro-Industrial Corporation was formed
and organized. The total subscribed capital stock of the corporation was
P3.5 Million with 35,000 shares. Additional shares were acquired and
subscribed from said corporation. Subsequently, on September 16, 1982, the
Margo Management and Development Corporation (Margo) was
incorporated. The total subscribed capital stock of Margo was 20,000 shares

Page 72 of 1072

at P200,000.00. Several transfers of shares of Ellice to Margo were made by


the stockholders and some payments of subscription were made by
transferring parcels of land by the Gala Spouses.
In essence, petitioners want this Court to disregard the separate
juridical personalities of Ellice and Margo for the purpose of treating all
property purportedly owned by said corporations as property solely owned
by the Gala spouses. The petitioners contention in support of this theory is
that the purposes for which Ellice and Margo were organized should be
declared as illegal and contrary to public policy. They claim that the
respondents never pursued exemption from land reform coverage in good
faith and instead merely used the corporations as tools to circumvent land
reform laws and to avoid estate taxes. Specifically, they point out that
respondents have not shown that the transfers of the land in favor of Ellice
were executed in compliance with the requirements of Section 13 of R.A.
3844. Furthermore, they alleged that respondent corporations were run
without any of the conventional corporate formalities.
ISSUE/S:
Whether Or Not The Lower Court Erred In Not Declaring As Illegal
And Contrary To Public Policy The Purposes And Manner In Which
Respondent Corporations Were Organized.
RULING:
NO.
At the outset, the Court holds that petitioners contentions impugning
the legality of the purposes for which Ellice and Margo were organized,
amount to collateral attacks which are prohibited in this jurisdiction.
The best proof of the purpose of a corporation is its articles of
incorporation and by-laws. The articles of incorporation must state the
primary and secondary purposes of the corporation, while the by-laws outline
the administrative organization of the corporation, which, in turn, is
supposed to insure or facilitate the accomplishment of said purpose.
In the case at bar, a perusal of the Articles of Incorporation of Ellice
and Margo shows no sign of the allegedly illegal purposes that petitioners are
complaining of. It is well to note that, if a corporations purpose, as stated in
the Articles of Incorporation, is lawful, then the SEC has no authority to
inquire whether the corporation has purposes other than those stated, and
mandamus will lie to compel it to issue the certificate of incorporation.
Assuming there was even a grain of truth to the petitioners claims
regarding the legality of what are alleged to be the corporations true
purposes, we are still precluded from granting them relief. We cannot

Page 73 of 1072

address here their concerns regarding circumvention of land reform laws, for
the doctrine of primary jurisdiction precludes a court from arrogating unto
itself the authority to resolve a controversy the jurisdiction over which is
initially lodged with an administrative body of special competence. Since
primary jurisdiction over any violation of Section 13 of Republic Act No. 3844
that may have been committed is vested in the Department of Agrarian
Reform Adjudication Board (DARAB), then it is with said administrative
agency that the petitioners must first plead their case. With regard to their
claim that Ellice and Margo were meant to be used as mere tools for the
avoidance of estate taxes, suffice it say that the legal right of a taxpayer to
reduce the amount of what otherwise could be his taxes or altogether avoid
them, by means which the law permits, cannot be doubted.
The petitioners allegation that Ellice and Margo were run without any
of the typical corporate formalities, even if true, would not merit the grant of
any of the relief set forth in their prayer. We cannot disregard the corporate
entities of Ellice and Margo on this ground. At most, such allegations, if
proven to be true, should be addressed in an administrative case before the
SEC.
Thus, even if Ellice and Margo were organized for the purpose of
exempting the properties of the Gala spouses from the coverage of land
reform legislation and avoiding estate taxes, we cannot disregard their
separate juridical personalities.

TOPIC:PRIMARY PURPOSE
UY SIULIONG, MARIANO LIMJAP, GACU UNG JIENG, EDILBERTO
CALIXTO and UY CHO YEE, petitioners,
vs.
THE DIRECTOR OF COMMERCE AND INDUSTRY, respondent.
G.R. No. L-15429. December 1, 1919
FACTS:
Petitioners had been associated together as partners in a partnership
known as Mercantil Regular Colectiva, under the style and firm Siulong y
Cia. Said partnership was to be dissolved in order to form a corporation to
be known as "Siulong y Compaia, Incorporada."
The proposed Articles of Incorporation of the said proposed corporation
states the following purposes:
(a) The purchase and sale, importation and exportation, of
the products of the country as well as of foreign countries;
(b) To discount promissory notes, bills of exchange, and
other negotiable instruments;

Page 74 of 1072

(c) The purchase and sale of bills of exchange, bonds,


stocks, or "participaciones de sociedades mercantiles e
industriales [joint account of mercantile and industrial
associations]," and of all classes of mercantile documents;
"comisiones
[commissions];"
"consignaciones
[consignments];"
(d) To act as agents for life, marine and fire insurance
companies;
(e) To purchase and sell boats of all classes "y fletamento
de los mismos [and charterage of same];" and
(f) To purchase and sell industrial and mercantile
establishments.
Respondent in his argument states the following:
(a) that the proposed articles of incorporation presented for
file and registry permitted the petitioners to engage in a
business which had for its end more than one purpose;
(b) that it permitted the petitioners to engage in the
banking business, and
(c) to deal in real estate, in violation of the Act of Congress
of July 1, 1902.
ISSUE/S:
Whether or not the proposed articles of incorporation of "Siuliong
y Cia., Inc.," permits it to engage in a business with more than
one purpose.
RULING:
The Supreme Court states:
1. That a corporation may be organized under the laws of the
Philippine Islands for mercantile purposes, and to engage in such
incidental business as may be necessary and advisable to give
effect to, and aid in, the successful operation and conduct of the
principal business.
2. While we have arrived at the conclusion that the proposed
articles of incorporation do not authorize the petitioners to
engage in a business with more than one purpose, we do not
mean to be understood as having decided that corporations
under the laws of the Philippine Islands may not engage in a
business with more than one purpose. Such an interpretation
might work a great injustice to corporations organized under the
Philippine laws. Such an interpretation would give foreign
corporations, which are permitted to be registered under the
laws here and which may be organized for more than one
purpose, a great advantage over domestic corporations. We do
not believe that it was the intention of the legislature to give

Page 75 of 1072

foreign corporations such an advantage over domestic


corporations.
Considering the particular purposes and objects of the
proposed articles of incorporation which are specially
enumerated above, we are of the opinion that it contains nothing
which violates in the slightest degree any of the provisions of the
laws of the Philippine Islands, and the petitioners are, therefore,
entitled to have such articles of incorporation filed and
registered as prayed for by them and to have issued to them a
certificate under the seal of the office of the respondent, setting
forth that such articles of incorporation have been duly filed in
his office. (Sec. 11, Act No. 1459.)

TOPIC:PRIMARY PURPOSE
NORBERTO ASUNCION, ET AL., petitioners-appellants,
vs.
MANUEL DE YRIARTE, respondent-appellee.
G.R. No. 9321. September 24, 1914
FACTS:
Respondent, the Chief of the Division of Archives of the Executive
Bureau, refused to file a certain Articles of Incorporation on the ground that
the object of the corporation, as stated in the articles was not lawful and
that, in pursuance of Sec. 6 of Act No. 1459, they were not registerable.
Consequently, the proposed Incorporators filed a complaint to compel
said Chief to receive and register said Articles of Incorporation.

Page 76 of 1072

The CFI found in favor of herein respondent and refused to order the
registration of the said Articles. It holds that respondent, under the
Corporation Law, had authority to determine both the sufficiency of the
Articles and the legality of the object of the proposed corporation.
Hence, this appeal.
ISSUE/S:
Whether Or Not The Purposes Of The Corporation As Stated In
The Articles Of Incorporation Are Lawful Within The Meaning Of
The Corporation Law.
RULING:
The purpose of the incorporation as stated in the articles is:
That the object of the corporation is
(a) to organize and regulate the management, disposition,
administration and control which the barrio of Pulo or San
Miguel or its inhabitants or residents have over the
common property of said residents or inhabitants or
property belonging to the whole barrio as such; and
(b) to use the natural products of the said property for
institutions, foundations, and charitable works of common
utility and advantage to the barrio or its inhabitants.
The municipality of Pasig as recognized by law contains within its limits
several barrios or small settlements, like Pulo or San Miguel, which have no
local government of their own but are governed by the municipality of Pasig
through its municipal president and council. The president and members of
the municipal council are elected by a general vote of the municipality, the
qualified electors of all the barrios having the right to participate.
The municipality of Pasig is a municipal corporation organized by law. It
has the control of all property of the municipality. The various barrios of the
municipality have no right to own or hold property, they not being
recognized as legal entities by any law. The residents of the barrios
participate in the advantages which accrue to the municipality from public
property and receive all the benefits incident to residence in a municipality
organized by law. If there is any public property situated in the barrio of Pulo
or San Miguel not belonging to the general government or the province, it
belongs to the municipality of Pasig and the sole authority to manage and
administer the same resides in that municipality. Until the present laws upon
the subject are charged no other entity can be the owner of such property or
control or administer it.
The object of the proposed corporation, as appears from the articles
offered for registration, is to make of the barrio of Pulo or San Miguel a

Page 77 of 1072

corporation which will become the owner of and have the right to control and
administer any property belonging to the municipality of Pasig found within
the limits of that barrio. This clearly cannot be permitted. Otherwise
municipalities as now established by law could be deprived of the property
which they now own and administer. Each barrio of the municipality would
become under the scheme proposed, a separate corporation, would take
over the ownership, administration, and control of that portion of the
municipal territory within its limits. This would disrupt, in a sense, the
municipalities of the Islands by dividing them into a series of smaller
municipalities entirely independent of the original municipality.
What the law does not permit cannot be obtained by indirection. The
object of the proposed corporation is clearly repugnant to the provisions of
the Municipal Code and the governments of municipalities as they have been
organized thereunder.

TOPIC: PRINCIPAL OFFICE/DOMICILE


DAVAO LIGHT & POWER CO., INC., petitioner,
vs.
THE HON. COURT OF APPEALS, HON. RODOLFO M. BELLAFLOR,
Presiding Judge of Branch 11, RTC-Cebu and FRANCISCO
TESORERO, respondents.
G.R. N.O. 111685, JANUARY 20, 2001
FACTS:
Petitioner Davao Light & Power Co., Inc. filed a complaint for
damages against private respondent Francisco Tesorero before the Regional
Trial Court of Cebu City. Private respondent filed a motion to dismiss claiming
among others that venue was improperly laid.
It is private respondent's contention that the proper venue is Davao
City, and not Cebu City where petitioner filed the case. Private respondent

Page 78 of 1072

argues that petitioner is estopped from claiming that its residence is in Cebu
City, in view of contradictory statements made by petitioner prior to the filing
of the action for damages. Private respondent adverts to several contracts it
entered into by petitioner with the National Power Corporation (NAPOCOR)
where in the description of personal circumstances, the former states that its
principal office is P. Reyes St., Davao City." According to private respondent
the petitioner's address in Davao City, as given in the contracts, is an
admission which should bind petitioner.
ISSUE/S:
Whether or not venue has been properly laid.
RULING:
It cannot be disputed that petitioner's principal office is in Cebu City,
per its amended articles of incorporation and by-laws. An action for
damages being a personal action, venue is determined pursuant to Rule 4,
section 2 of the Rules of Court, to wit:
Venue of personal actions.
All other actions may be commenced and tied where the
plaintiff or any of the principal plaintiffs resides, or where the
defendant or any of the principal defendants resides, or in the case of
a non-resident defendant where he may be found, at the election of
the plaintiff.
Thus, the case was properly filed at Cebu City where petitioner has its
residence.
TOPIC:PRINCIPAL OFFICE/DOMICILE
CLAVECILLIA RADIO SYSTEM, petitioner-appellant,
vs.
HON. AGUSTIN ANTILLON, as City Judge of the Municipal Court of
Cagayan de Oro City and NEW CAGAYAN GROCERY, respondentsappellees.
G.R. No. L-22238, FEBRUARY 18, 1967
19 SCRA 379
FACTS:
It appears that on June 22, 1963, the New Cagayan Grocery filed a
complaint against the Clavecilla Radio System alleging, in effect, that on
March 12, 1963, the following message, addressed to the former, was filed at
the latter's Bacolod Branch Office for transmittal thru its branch office at
Cagayan de Oro: NECAGRO CAGAYAN DE ORO (CLAVECILLA): REURTEL

Page 79 of 1072

WASHED NOT AVAILABLE REFINED TWENTY FIFTY IF AGREEABLE SHALL SHIP


LATER REPLY POHANG
The Cagayan de Oro branch office having received the said message
omitted, in delivering the same to the New Cagayan Grocery, the word "NOT"
between the words "WASHED" and "AVAILABLE," thus changing entirely the
contents and purport of the same and causing the said addressee to suffer
damages. After service of summons, the Clavecilla Radio System filed a
motion to dismiss the complaint on the grounds that it states no cause of
action and that the venue is improperly laid. The New Cagayan Grocery
interposed an opposition to which the Clavecilla Radio System filed its
rejoinder. Thereafter, the City Judge, on September 18, 1963, denied the
motion to dismiss for lack of merit and set the case for hearing.
Hence, the Clavecilla Radio System filed a petition for prohibition with
preliminary injunction with the Court of First Instance praying that the City
Judge, Honorable Agustin Antillon, be enjoined from further proceeding with
the case on the ground of improper venue. The respondents filed a motion
to dismiss the petition but this was opposed by the petitioner. Later, the
motion was submitted for resolution on the pleadings.
ISSUE/S:
Whether or not the venue has been properly laid.
RULING:
No.
Settled is the principle in corporation law that the residence of a
corporation is the place where its principal office is established, which in this
case is in Manila, thus suit must be instituted in the City of Manila and not in
the corporations branch office in Cagayan de Oro.
The Court has held in Cohen v. Benguet Comm. Cp., Ltd., 34 Phil. 526
that the same may be served with summons under Sec. 1, Rule 4 of the RRC
does not apply when the defendant resides in the Philippines for in such
case, he may be sued only in his residence, regardless of the place where he
may be found and served with summons. To allow action to be instituted in
nay place when there are branch offices would create confusion ad work
untold inconvenience t the corporation.

Page 80 of 1072

TOPIC:PRINCIPAL OFFICE/DOMICILE
JOHN SY and UNIVERSAL PARTS SUPPLY CORPORATION, petitioners,
vs.
TYSON ENTERPRISES, INC., JUDGE GREGORIO G. PINEDA of the Court
of First Instance of Rizal, Pasig Branch XXI and COURT OF
APPEALS, respondents.
G.R. NO. L-56763, DECEMBER 15, 1982
119 SCRA 367
FACTS:
On August 29, 1979, Tyson Enterprises, Inc. filed against John Sy and
Universal Parts Supply Corporation, residents of Bacolod, a complaint for the
collection of money in Pasig, Rizal. However, there is no allegation in the
complaint as to the office or place of business of plaintiff Tyson Enterprises,
Inc., which is located in Manila. What is alleged is the postal address or

Page 81 of 1072

residence of Dominador Ti, the president and general manager of plaintiff


firm, which is in San Juan, Rizal.
Defendant Sy and Universal Parts Supply Corporation filed a motion to
dismiss on the ground of improper venue. The plaintiff opposed the motion
to dismiss which the trial court denied. On appeal, the Appellate Court
dismissed the petition. It ruled that the parties did not intend Manila as the
exclusive venue of the actions arising under their transactions and that since
the action was filed in Pasig, which is near Manila, no useful purpose would
be served by dismissing the same and ordering that it be filed in Manila.
ISSUE:
Whether or not venue is properly laid.
RULING:
The Court ruled in the affirmative.
For purposes of venue, it is the place of business of the corporation
rather than the residence of its president that is considered. The residence of
its president is not the residence of the corporation because a corporation
ahs a personality separate and distinct from that of its officers and
stockholders.
The collection should have been filed in Manila where the plaintiff
corporation has its residence and the pace designated in its invoice, not in
Bacolod City.

TOPIC:PRINCIPAL OFFICE/DOMICILE
YOUNG AUTO SUPPLY CO. AND NEMESIO GARCIA, petitioners,
vs.
THE HONORABLE COURT OF APPEALS (THIRTEENTH DIVISION) AND
GEORGE CHIONG ROXAS, respondents.
G.R. NO. 104175, JUNE 25, 1993
223 SCRA 670
FACTS:
Defendant sought the dismissal of an action filed by the plaintiff, a
corporation, before the Regional Trial Court of Cebu City, on the ground of
improper venue. Accordingly, venue was improperly laid since the address of
the plaintiff was supposedly in Pasay City, as evidenced by a contract of sale,
letters and several commercial documents sent by the plaintiff to the

Page 82 of 1072

defendant, even though the plaintiff's articles of incorporation stated that its
principal office was in Cebu City.
The complaint was dismissed on the ground of improper venue.
ISSUE/S:
Whether or not venue was properly laid.
RULING:
No.
In the Regional Trial Courts, all personal actions are commenced and
tried in the province or city where the defendant or any of the defendants
resides or may be found, or where the plaintiff or any of the plaintiffs resides,
at the election of the plaintiff. There are two plaintiffs in the case at bench: a
natural person and a domestic corporation. Both plaintiffs aver in their
complaint that they are residents of Cebu City, thus:
The Article of Incorporation of YASCO (SEC Reg. No. 22083) states:
"THIRD. That the place where the principal office of the
corporation is to be established or located is at Cebu City,
Philippines. If it was Roxas who sued YASCO in Pasay City and the
latter questioned the venue on the ground that its principal place
of business was in Cebu City, Roxas could argue that YASCO was
in estoppel because it misled Roxas to believe that Pasay City
was its principal place of business. But this is not the case before
us.
With the finding that the residence of YASCO for purposes of venue is
in Cebu City, where its principal place of business is located, it becomes
unnecessary to decide whether Garcia is also a resident of Cebu City and
whether Roxas was in estoppel from questioning the choice of Cebu City as
the venue.

Page 83 of 1072

TOPIC: TERM OF A CORPORATION


ALHAMBRA CIGAR & CIGARETTE MANUFACTURING COMPANY,
INC., petitioner,
vs.
SECURITIES & EXCHANGE COMMISSION, respondent
G.R. No. L-23606, July 29, 1968
24 SCRA 269
FACTS:
Petitioner Alhambra Cigar and Cigarette Manufacturing Company, Inc.
was duly incorporated under Philippine laws on January 15, 1912. By its
corporate articles it was to exist for fifty (50) years from incorporation. Its
term of existence expired on January 15, 1962. On that date, it ceased
transacting business, entered into a state of liquidation.

Page 84 of 1072

Thereafter, a new corporation. Alhambra Industries, Inc. was


formed to carry on the business of Alhambra and the stockholders named
Angel S. Gamboa trustee to take charge of its liquidation.
On June 20, 1963 within Alhambra's three-year statutory period for
liquidation - Republic Act 3531 was enacted into law. It amended Section 18
of the Corporation Law; it empowered domestic private corporations to
extend their corporate life beyond the period fixed by the articles of
incorporation for a term not to exceed fifty years in any one instance.
Previous to Republic Act 3531, the maximum non-extendible term of such
corporations was fifty years. Alhambra's board of directors resolved to
amend its articles of incorporation to extend its corporate life for an
additional fifty years, or a total of 100 years from its incorporation for which
when submitted to the SEC was denied on ground that its term already
expired.
ISSUE/S:
Whether or not the Alhambra may extend its corporate termby
amendment of its articles of incorporation effected during the
three-year statutory period for liquidation when its original term
of existence had already expired.
RULING:
NO.
The continuance of a "dissolved" corporation as a body corporate for
three years has for its purpose the final closure of its affairs, and no
other; the corporation is specifically enjoined from "continuing the business
for which it was established". The liquidation of the corporation's affairs set
forth in Section 77 became necessary precisely because its life had ended.
For this reason alone, the corporate existence and juridical personality of the
corporation to do business may no longer be extended. The moment a
corporation's right to exist as an "artificial person" ceases, its corporate
powers are terminated "just as the powers of a natural person to take part in
mundane affairs cease to exist upon his death". There is nothing left but to
conduct, as it were, the settlement of the estate of a deceased juridical
person.

Page 85 of 1072

TOPIC: PAID UP CAPITAL STOCK


MSCI-NACUSIP Local Chapter, petitioner,
vs.
NATIONAL WAGES AND PRODUCTIVITY COMMISSION and MONOMER
SUGAR CENTRAL, INC., respondents.
G.R. No. 125198. March 3, 1997
269 SCRA 173
FACTS:
Asturias Sugar Central, Inc. (ASCI, for brevity), executed a
Memorandum of Agreement with Monomer Trading Industries, Inc. (MTII, for
brevity), whereby MTII shall acquire the assets of ASCI by way of a Deed of
Assignment provided that an entirely new organization in place of MTII shall
be organized, which new corporation shall be the assignee of the assets of
ASCI. By virtue of this Agreement, a new corporation was organized and

Page 86 of 1072

incorporated under the corporate name Monomer Sugar Central, Inc. or


MSCI, the private respondent herein.
MSCI applied for exemption from the coverage of Wage Order No. RO
VI-01 issued by the Board on the ground that it is a distressed employer. In
support thereto, MSCI submitted its audited financial statements and income
tax returns duly stamped "received" by the Bureau of Internal Revenue (BIR)
and the Securities and Exchange Commission (SEC) for the period beginning
February 15, 1990 and ending. August 31, 1990, including the quarterly
financial statements and income tax returns for the two quarters ending
November 30, 1990 and February 28, 1991.
The petitioner herein MSCI-NACUSIP Local Chapter (Union, for brevity),
in opposition, maintained that MSCI is not distressed; that respondent
applicant has not complied with the requirements for exemption; and that
the financial statements submitted by MSCI do not reflect the true and valid
financial status of the company, and that the paid-up capital would have
been higher than P5 million and thus impairment would have been lower
than 25% had the pre-organization agreement between ASCI and MTII been
complied with.
ISSUE/S:
What is the correct paid-up capital of MSCI for the pertinent
period covered by the application for exemption P5 million or
P64, 688,528.00?
RULING:
The Board held that the paid-up capital of MSCI on the aforesaid dates
was actually P64,688,528.00 and not P5 million as claimed by MSCI in its
application for exemption and, thus, the established losses amounting to
P3,400,738.00 constitute an impairment of only 5.25% of the true paid-up
capital of P64 million plus,which losses are not enough to meet the required
25% impairment requirement. This conclusion is anchored on the belief of
the Board that the value of the assets of ASCI, party to the Memorandum of
Agreement, transferred to MSCI on March 28, 1990 should be taken into
consideration in computing the paid-up capital of MSCI to reflect its true
financial structure. Moreover, the loans or advances extended by MTII, the
other party to the Agreement, to MSCI should allegedly be treated as
additional investments to MSCI, and must therefore be included in computing
respondent's paid-up capital.
Public respondent Commission thought otherwise. In reversing the
Board and granting the exemption, the Commission held that the Board
exceeded its authority in computing and giving new valuation to what should
be the paid-up capital of MSCI. It stressed that RA No. 6727, or the Wage

Page 87 of 1072

Rationalization Act, and its implementing guidelines have not conferred upon
the Board the authority to change the paid-up capital of a corporation.
The foregoing asseveration of the parties considered, we find no grave
abuse of discretion on the part of the Commission in setting aside the
findings of the Board and granting full exemption to MSCI from Wage Order
No. RO VI-01.
NWPC Guidelines No. 01, Series of 1992 as well as the new NWPC
Guidelines No. 01, Series of 1996, define Capital as referring to paid-up
capital at the end of the last full accounting period, in the case of
corporations or total invested capital at the beginning of the period under
review, in the case of partnerships and single proprietorships. To have a clear
understanding of what paid-up capital is, however, a referral to Sections 12
and 13 of BP Blg. 68 or the Corporation Code.
By express provision of Section 13, paid-up capital is that portion of the
authorized capital stock which has been both subscribed and paid. To
illustrate, where the authorized capital stock of a corporation is worth P 1
million and the total subscription amounts to P250,000.00, at least 25% of
this amount, namely, P62,500.00 must be paid up per Section 13. The latter,
P62,500.00, is the paid-up capital or what should more accurately be termed
as "paid-up capital
stock."
In the case under consideration, there is no dispute, and the Board
even mentioned in its August 17, 1993 Decision, that MSCI was organized
and incorporated on February 15, 1990 with an authorized capital stock of
P60 million, P20 million of which was subscribed. Of the P20 million
subscribed capital stock, P5 million was paid-up. This fact is only too glaring
for the Board to have been misled into believing that MSCI'S paid-up capital
stock was P64 million plus and not P5 million.

TOPIC:CLASSIFICATION OF SHARES
SAN MIGUEL CORPORATION, petitioner,
vs.
SANDIGANBAYAN, respondents
G.R. Nos. 104637-38. September 14, 2000
340 SCRA 289-331
FACTS:
Coconut Industry Investment Fund Holding Companies (CIIF),
composed of 14 companies, sold 33,133,266 shares of the outstanding

Page 88 of 1072

capital stock of San Miguel Corporation to Andres Soriano (Andres) of the


SMC Group payable in 4 installments. Andres paid the initial P500 million to
the UCPB as administrator of the CIIF.
Later, the PCGG sequestered the shares of stock subject of the sale.
Due to the sequestration, the SMC Group suspended payment of the balance
of the purchase price of the subject stocks. In retaliation, the UCPB Group
rescinded the sale. However, the parties were able to thresh out their dispute
to the extent that that they filed with the Sandiganbayan a Joint Petition for
Approval of the Compromise Agreement.
The Republic, through the OSG, opposed the Compromise Agreement.
It contended that the involved coco-levy funds, whether in the form of
earnings or dividends therefrom, or in the form of the value of liquidated
corporate assets represented by all sequestered shares, or in the form of
cash, or in the form of "proceeds" of sale or of "payments" of certain alleged
obligations are public funds (they are the subject of the civil case regarding
ill-gotten wealth of Marcos). As public funds, the coco-levy funds, in any form
or transformation, are beyond or "outside the commerce."
The Sandiganbayan issued an order requiring SMC Group to deliver the
certificates of stock representing the subject matter of the Compromise
Agreement to the PCGG.
ISSUE/S:
Whether it is proper for the Sandiganbayan to order SMC Group
to deliver the treasury shares to PCGG and pay their
corresponding dividends.
RULING:
The case at bar does not merely involve a compromise agreement
dealing with private interest but involves sequestered shares of stock now
worth more than nine (9) billions of pesos. Their ownership is still under
litigation. It is not yet known whether the shares are part of the alleged illgotten wealth of former President Marcos and his "cronies." Any Compromise
Agreement concerning these sequestered shares falls within the
unquestionable jurisdiction of and has to be approved by the Sandiganbayan.
Moreover, SMC Group's primary justification for failing it to turn over
the certificates of stock for the 25.45 million sequestered shares as well as
the cash dividends already accrued thereon that the shares of stock have
allegedly now become Treasury Shares is unmeritorious. Under the
Corporation Code 'Treasury shares are shares of stock which have been
issued and fully paid for, but subsequently reacquired by, the issuing
corporation by purchase, redemption, donation or through some lawful

Page 89 of 1072

means . . .' (Sec. 9, B.P. Blg. 68, Corporation Code). These 26.45 million
shares of stock or any portion thereof can, therefore, become Treasury
Shares, i.e., property of the San Miguel Corporation, only if the sale between
the UCPB Group and the SMC Group is allowed; otherwise these shares
cannot even begin to be deemed to have been 're-acquired by the issuing
corporation,' i.e., the San Miguel Corporation.
But even if, indeed, these shares are treasury shares, they remain
sequestered so that any movement of these shares cannot be of any
permanent character that will alter their being sequestered shares and,
therefore, in 'custodia legis,' that is to say, under the control and disposition
of the Court.
It must finally be said that the conversion of the 26.45 (or 25.45)
million shares by the SMC Group into Treasury Shares is of the SMC Group's
own making and the SMC Group cannot perform acts that will, by its own
say-so, take property away from 'custodia legis.'

TOPIC: AMENDMENT AND/OR REJECTION OF ARTICLES OF INCORPORATION


REPUBLIC PLANTERS BANK, petitioner,
vs.
COURT OF APPEALS and FERMIN CANLAS, respondents.
G.R. No. 93073. December 21, 1992
216SCRA 738
FACTS:
Defendant Shozo Yamaguchi and private respondent Fermin Canlas
were President/Chief Operating Officer and Treasurer respectively, of

Page 90 of 1072

Worldwide Garment Manufacturing, Inc.. By virtue of Board Resolution No.1


dated August 1, 1979, defendant Shozo Yamaguchi and private respondent
Fermin Canlas were authorized to apply for credit facilities with the petitioner
Republic Planters Bank in the forms of export advances and letters of
credit/trust receipts accommodations. Petitioner bank issued nine promissory
notes. In the promissory notes marked as Exhibits C, D and F, the name
Worldwide Garment Manufacturing, Inc. was apparently rubber stamped
above the signatures of defendant and private respondent.
On December 20, 1982, Worldwide Garment Manufacturing, Inc. noted
to change its corporate name to Pinch Manufacturing Corporation.
On February 5, 1982, petitioner bank filed a complaint for the recovery
of sums of money covered among others, by the nine promissory notes with
interest thereon, plus attorney's fees and penalty charges. The complainant
was originally brought against Worldwide Garment Manufacturing, Inc. inter
alia, but it was later amended to drop Worldwide Manufacturing, Inc. as
defendant and substitute Pinch Manufacturing Corporation it its place.
Defendants Pinch Manufacturing Corporation and Shozo Yamaguchi did not
file an Amended Answer and failed to appear at the scheduled pre-trial
conference despite due notice. Only private respondent Fermin Canlas filed
an Amended Answer wherein he, denied having issued the promissory notes
in question since according to him, he was not an officer of Pinch
Manufacturing
Corporation,
but
instead
of
Worldwide
Garment
Manufacturing, Inc., and that when he issued said promissory notes in behalf
of Worldwide Garment Manufacturing, Inc., the same were in blank, the
typewritten entries not appearing therein prior to the time he affixed his
signature.
ISSUE/S:
Whether an amendment in a corporation's Articles of
Incorporation effecting a change of corporate name from
Worldwide Garment manufacturing Inc to Pinch Manufacturing
Corporation extinguished the personality of the original
corporation.
RULING:
The respondent Court made a grave error in holding that an
amendment in a corporation's Articles of Incorporation effecting a change of
corporate name, in this case from Worldwide Garment manufacturing Inc to
Pinch Manufacturing Corporation extinguished the personality of the original
corporation.

Page 91 of 1072

The corporation, upon such change in its name, is in no sense a new


corporation, nor the successor of the original corporation. It is the same
corporation with a different name, and its character is in no respect changed.
A change in the corporate name does not make a new corporation, and
whether effected by special act or under a general law, has no affect on the
identity of the corporation, or on its property, rights, or liabilities. The
corporation continues, as before, responsible in its new name for all debts or
other liabilities which it had previously contracted or incurred.

TOPIC: DOCTRINE OF
CORPORATE FICTION

CORPORATE

ENTITY/

PIERCING

THE

VEIL

OF

RAMIREZ
V.
MAR FISHING, INC
JUNE 13, 2012
FACTS:
On 28 June 2001, respondent Mar Fishing Co., Inc. (Mar Fishing),
engaged in the business of fishing and canning of tuna, sold its principal

Page 92 of 1072

assets to co-respondent Miramar Fishing Co., Inc. (Miramar) through public


bidding.The proceeds of the sale were paid to the Trade and Investment
Corporation of the Philippines (TIDCORP) to cover Mar Fishings outstanding
obligation in the amount of 897,560,041.26. In view of that transfer, Mar
Fishing issued a Memorandum dated 23 October 2001 informing all its
workers that the company would cease to operate by the end of the month.
On 29 October 2001 or merely two days prior to the months end, it notified
the Department of Labor and Employment (DOLE) of the closure of its
business operations.
Thereafter, Mar Fishings labor union, Mar Fishing Workers Union NFL and
Miramar entered into a Memorandum of Agreement. The Agreement
provided that the acquiring company, Miramar, shall absorb Mar Fishings
regular rank and file employees whose performance was satisfactory,
without loss of seniority rights and privileges previously enjoyed.
Unfortunately, petitioners, who worked as rank and file employees, were not
hired or given separation pay by Miramar. Thus, petitioners filed Complaints
for illegal dismissal with money claims before the Arbitration Branch of the
National Labor Relations Commission (NLRC).
ISSUE:
Whether or not Mar Fishing were liable to pay the claims of the
employees?
HELD:
The Labor Arbiter (LA) found that Mar Fishing had necessarily closed its
operations, considering that Miramar had already bought the tuna canning
plant. By reason of the closure, petitioners were legally dismissed for
authorized cause. In addition, even if Mar Fishing reneged on notifying the
DOLE within 30 days prior to its closure, that failure did not make the
dismissals void. Consequently, the LA ordered Mar Fishing to give separation
pay to its workers.

TOPIC: DOCTRINE OF
CORPORATE FICTION

CORPORATE

ENTITY/

PIERCING

THE

VEIL

OF

SARONA, petitioners,
vs.
NATIONAL LABOR RELATIONS COMMISSION and NATIONAL STEEL
CORPORATION (NSC), respondents.
G.R. No. 109902 August 2, 1994

Page 93 of 1072

FACTS:
On 5 July 1990, petitioners filed separate complaints for unfair labor
practice, regularization and monetary benefits with the NLRC, Sub-Regional
Arbitration Branch XII, Iligan City.
The complaints were consolidated and after hearing, the Labor Arbiter
in a Decision dated 7 June 1991, declared petitioners "regular project
employees who shall continue their employment as such for as long as such
[project] activity exists," but entitled to the salary of a regular employee
pursuant to the provisions in the collective bargaining agreement. It also
ordered payment of salary differentials.
Both parties appealed to the NLRC from that decision. Petitioners
argued that they were regular, not project, employees. Private respondent,
on the other hand, claimed that petitioners are project employees as they
were employed to undertake a specific project NSC's Five Year Expansion
Program (FAYEP I & II).
The NLRC in its questioned resolutions modified the Labor Arbiter's
decision. It affirmed the Labor Arbiter's holding that petitioners were project
employees since they were hired to perform work in a specific undertaking
the Five Years Expansion Program, the completion of which had been
determined at the time of their engagement and which operation was not
directly related to the business of steel manufacturing. The NLRC, however,
set aside the award to petitioners of the same benefits enjoyed by regular
employees for lack of legal and factual basis.
Deliberating on the present Petition for Certiorari, the Court considers that
petitioners have failed to show any grave abuse of discretion or any act
without or in excess of jurisdiction on the part of the NLRC in rendering its
questioned resolutions of 8 January 1993 and 15 February 1993.
ISSUE:
Whether or not National Steel Corporation is liable to its employees?
HELD:
In the case of Mercado, Sr. vs. National Labor Relations Commission,
this Court ruled that the proviso in the second paragraph of Article 280
relates only to casual employees and is not applicable to those who fall
within the definition of said Article's first paragraph, i.e., project employees.
The familiar grammatical rule is that a proviso is to be construed with
reference to the immediately preceding part of the provision to which it is
attached, and not to other sections thereof, unless the clear legislative intent
is to restrict or qualify not only the phrase immediately preceding the proviso
but also earlier provisions of the statute or even the statute itself as a whole.
No such intent is observable in Article 280 of the Labor Code, which has been
quoted earlier.
ACCORDINGLY, in view of the foregoing, the Petition for Certiorari is hereby
DISMISSED for lack of merit. The Resolutions of the NLRC dated 8 January

Page 94 of 1072

1993 and 15 February 1993 are hereby AFFIRMED. No pronouncement as to


costs.

TOPIC: DOCTRINE OF
CORPORATE FICTION

CORPORATE

ENTITY/

PIERCING

THE

VEIL

OF

GOLD LINE TOURS


v.
HEIRS OF LACSA
June 18, 2012
FACTS:

Page 95 of 1072

On August 2, 1993, Ma. Concepcion Lacsa (Concepcion) and her sister,


Miriam Lacsa (Miriam), boarded a Goldline passenger bus with Plate No.
NXM-105 owned and operated by Travel &Tours Advisers, Inc. They were
enroute from Sorsogon to Cubao, Quezon City. At the time, Concepcion,
having just obtained her degree of Bachelor of Science in Nursing at the Ago
Medical and Educational Center, was proceeding to Manila to take the
nursing licensure board examination. Upon reaching the highway at
Barangay San Agustin in Pili, Camarines Sur, the Goldline bus, driven by
Rene Abania (Abania), collided with a passenger jeepney with Plate No. EAV313 coming from the opposite direction and driven by Alejandro Belbis. As a
result, a metal part of the jeepney was detached and struck Concepcion in
the chest, causing her instant death.
On August 23, 1993, Concepcions heirs, represented by Teodoro
Lacsa, instituted in the RTC a suit against Travel & Tours Advisers Inc. and
Abania to recover damages arising from breach of contract of carriage. The
complaint, docketed as Civil Case No. 93-5917 and entitled Heirs of
Concepcion Lacsa, represented by Teodoro Lacsa v. Travel & Tours Advisers,
Inc. (Goldline) and Rene Abania, alleged that the collision was due to the
reckless and imprudent manner by which Abania had driven the Goldline
bus.
In support of the complaint, Miriam testified that Abania had been
occasionally looking up at the video monitor installed in the front portion of
the Goldline bus despite driving his bus at a fast speed; that in Barangay San
Agustin, the Goldline bus had collided with a service jeepney coming from
the opposite direction while in the process of overtaking another bus; that
the impact had caused the angle bar of the jeepney to detach and to go
through the windshield of the bus directly into the chest of Concepcion who
had then been seated behind the drivers seat; that concerned bystanders
had hailed another bus to rush Concepcion to the Ago Foundation Hospital in
Naga City because the Goldline bus employees and her co-passengers had
ignored Miriams cries for help; and that Concepcion was pronounced dead
upon arrival at the hospital.
To refute the plaintiffs allegations, the defendants presented SPO1
Pedro Corporal of the Philippine National Police Station in Pili, Camarines Sur,
and William Cheng, the operator of the Goldline bus. SPO1 Corporal opined
that based on his investigation report, the driver of the jeepney had been at
fault for failing to observe precautionary measures to avoid the collision; and
suggested that criminal and civil charges should be brought against the
operator and driver of the jeepney. On his part, Cheng attested that he had
exercised the required diligence in the selection and supervision of his
employees; and that he had been engaged in the transportation business
since 1980 with the use of a total of 60 units of Goldline buses, employing
about 100 employees (including drivers, conductors, maintenance personnel,

Page 96 of 1072

and mechanics); that as a condition for regular employment, applicant


drivers had undergone a one-month training period and a six-month
probationary period during which they had gotten acquainted with Goldlines
driving practices and demeanor; that the employees had come under
constant supervision, rendering improbable the claim that Abania, who was a
regular employee, had been glancing at the video monitor while driving the
bus; that the incident causing Concepcions death was the first serious
incident his (Cheng) transportation business had encountered, because the
rest had been only minor traffic accidents; and that immediately upon being
informed of the accident, he had instructed his personnel to contact the
family of Concepcion.
The defendants blamed the death of Concepcion to the recklessness of
Bilbes as the driver of the jeepney, and of its operator, Salvador Romano;
and that they had consequently brought a third-party complaint against the
latter.
ISSUE:
Did the CA rightly find and conclude that the RTC did not gravely abuse
its discretion in denying petitioners verified third-party claim?
HELD:
The main contention of Third Party Claimant is that it is the owner of
the Bus and therefore, it should not be seized by the sheriff because the
same does not belong to the defendant Travel & Tours Advises, Inc.
(GOLDLINE) as the third party claimant and defendant are two separate
corporation with separate juridical personalities. Upon the other hand, this
Court had scrutinized the documents submitted by the Third party Claimant
and found out that William Ching who claimed to be the operator of the
Travel & Tours Advisers, Inc. (GOLDLINE) is also the President/Manager and
incorporator of the Third Party Claimant Goldline Tours Inc. and he is joined
by his co-incorporators who are Ching and Dy thereby this Court could
only say that these two corporations are one and the same corporations.
This is of judicial knowledge that since Travel & Tours Advisers, Inc. came to
Sorsogon it has been known as GOLDLINE.
This Court is not persuaded by the proposition of the third party claimant
that a corporation has an existence separate and/or distinct from its
members insofar as this case at bar is concerned, for the reason that
whenever necessary for the interest of the public or for the protection of
enforcement of their rights, the notion of legal entity should not and is not to
be used to defeat public convenience, justify wrong, protect fraud or defend
crime.

Page 97 of 1072

TOPIC: DOCTRINE OF
CORPORATE FICTION

CORPORATE

ENTITY/

PIERCING

THE

VEIL

OF

HACIENDA LUISITA INC. (HLI)


V.
PRESIDENTIAL AGRARIAN REFORM COUNCIL (PARC), ET AL.,
G.R. NO. 171101, NOVEMBER 22, 2011
I.

THE FACTS

On July 5, 2011, the Supreme Court en banc voted unanimously (11-0)


to DISMISS/DENY the petition filed by HLI and AFFIRM with MODIFICATIONS
the resolutions of the PARC revoking HLIs Stock Distribution Plan (SDP) and
placing the subject lands in Hacienda Luisita under compulsory coverage of
the Comprehensive Agrarian Reform Program (CARP) of the government.
The Court however did not order outright land distribution. Voting 6-5,
the Court noted that there are operative facts that occurred in the interim
and which the Court cannot validly ignore. Thus, the Court declared that the
revocation of the SDP must, by application of the operative fact principle,
give way to the right of the original 6,296 qualified farmworkers-beneficiaries
(FWBs) to choose whether they want to remain as HLI stockholders or
[choose actual land distribution]. It thus ordered the Department of Agrarian
Reform (DAR) to immediately schedule meetings with the said 6,296 FWBs
and explain to them the effects, consequences and legal or practical
implications of their choice, after which the FWBs will be asked to manifest,
in secret voting, their choices in the ballot, signing their signatures or placing
their thumbmarks, as the case may be, over their printed names.
The parties thereafter filed their respective motions for reconsideration
of the Court decision.
II.

THE ISSUES

(1) Is the operative fact doctrine available in this case?


(2) Is Sec. 31 of RA 6657 unconstitutional?
(3) Cant the Court order that DARs compulsory acquisition of Hacienda
Lusita cover the full 6,443 hectares allegedly covered by RA 6657 and
previously held by Tarlac Development Corporation (Tadeco), and not just the
4,915.75 hectares covered by HLIs SDP?
(4) Is the date of the taking (for purposes of determining the just
compensation payable to HLI) November 21, 1989, when PARC approved
HLIs SDP?
(5) Has the 10-year period prohibition on the transfer of awarded lands
under RA 6657 lapsed on May 10, 1999 (since Hacienda Luisita were placed

Page 98 of 1072

under CARP coverage through the SDOA scheme on May 11, 1989), and thus
the qualified FWBs should now be allowed to sell their land interests in
Hacienda Luisita to third parties, whether they have fully paid for the lands
or not?
(6) THE CRUCIAL ISSUE: Should the ruling in the July 5, 2011 Decision that
the qualified FWBs be given an option to remain as stockholders of HLI be
reconsidered?
III. THE RULING
[The Court PARTIALLY GRANTED the motions for reconsideration of
respondents PARC, et al. with respect to the option granted to the original
farmworkers-beneficiaries (FWBs) of Hacienda Luisita to remain with
petitioner HLI, which option the Court thereby RECALLED and SET ASIDE. It
reconsidered its earlier decision that the qualified FWBs should be given an
option to remain as stockholders of HLI, and UNANIMOUSLY directed
immediate land distribution to the qualified FWBs.]
1.

YES, the operative fact doctrine is applicable in this case.

[The Court maintained its stance that the operative fact doctrine is
applicable in this case since, contrary to the suggestion of the minority, the
doctrine is not limited only to invalid or unconstitutional laws but also applies
to decisions made by the President or the administrative agencies that have
the force and effect of laws. Prior to the nullification or recall of said
decisions, they may have produced acts and consequences that must be
respected. It is on this score that the operative fact doctrine should be
applied to acts and consequences that resulted from the implementation of
the PARC Resolution approving the SDP of HLI. The majority stressed that the
application of the operative fact doctrine by the Court in its July 5, 2011
decision was in fact favorable to the FWBs because not only were they
allowed to retain the benefits and homelots they received under the stock
distribution scheme, they were also given the option to choose for
themselves whether they want to remain as stockholders of HLI or not.]
2.

NO, Sec. 31 of RA 6657 NOT unconstitutional.

[The Court maintained that the Court is NOT compelled to rule on the
constitutionality of Sec. 31 of RA 6657, reiterating that it was not raised at
the earliest opportunity and that the resolution thereof is not the lis mota of
the case. Moreover, the issue has been rendered moot and academic since
SDO is no longer one of the modes of acquisition under RA 9700. The
majority clarified that in its July 5, 2011 decision, it made no ruling in favor of
the constitutionality of Sec. 31 of RA 6657, but found nonetheless that there
was no apparent grave violation of the Constitution that may justify the
resolution of the issue of constitutionality.]

Page 99 of 1072

3.
NO, the Court CANNOT order that DARs compulsory acquisition of
Hacienda Lusita cover the full 6,443 hectares and not just the 4,915.75
hectares covered by HLIs SDP.
[Since what is put in issue before the Court is the propriety of the revocation
of the SDP, which only involves 4,915.75 has. of agricultural land and not
6,443 has., then the Court is constrained to rule only as regards the 4,915.75
has. of agricultural land.Nonetheless, this should not prevent the DAR, under
its mandate under the agrarian reform law, from subsequently subjecting to
agrarian reform other agricultural lands originally held by Tadeco that were
allegedly not transferred to HLI but were supposedly covered by RA 6657.
However since the area to be awarded to each FWB in the July 5, 2011
Decision appears too restrictive considering that there are roads, irrigation
canals, and other portions of the land that are considered commonly-owned
by farmworkers, and these may necessarily result in the decrease of the area
size that may be awarded per FWB the Court reconsiders its Decision and
resolves to give the DAR leeway in adjusting the area that may be awarded
per FWB in case the number of actual qualified FWBs decreases. In order to
ensure the proper distribution of the agricultural lands of Hacienda Luisita
per qualified FWB, and considering that matters involving strictly the
administrative implementation and enforcement of agrarian reform laws are
within the jurisdiction of the DAR, it is the latter which shall determine the
area with which each qualified FWB will be awarded.
On the other hand, the majority likewise reiterated its holding that the 500hectare portion of Hacienda Luisita that have been validly converted to
industrial use and have been acquired by intervenors Rizal Commercial
Banking Corporation (RCBC) and Luisita Industrial Park Corporation (LIPCO),
as well as the separate 80.51-hectare SCTEX lot acquired by the
government, should be excluded from the coverage of the assailed PARC
resolution. The Court however ordered that the unused balance of the
proceeds of the sale of the 500-hectare converted land and of the 80.51hectare land used for the SCTEX be distributed to the FWBs.]
4.
YES, the date of taking is November 21, 1989, when PARC approved
HLIs SDP.
[For the purpose of determining just compensation, the date of taking is
November 21, 1989 (the date when PARC approved HLIs SDP) since this is
the time that the FWBs were considered to own and possess the agricultural
lands in Hacienda Luisita. To be precise, these lands became subject of the
agrarian reform coverage through the stock distribution scheme only upon
the approval of the SDP, that is, on November 21, 1989. Such approval is
akin to a notice of coverage ordinarily issued under compulsory acquisition.

Page 100 of 1072

On the contention of the minority (Justice Sereno) that the date of the notice
of coverage [after PARCs revocation of the SDP], that is, January 2, 2006, is
determinative of the just compensation that HLI is entitled to receive, the
Court majority noted that none of the cases cited to justify this position
involved the stock distribution scheme. Thus, said cases do not squarely
apply to the instant case. The foregoing notwithstanding, it bears stressing
that the DAR's land valuation is only preliminary and is not, by any means,
final and conclusive upon the landowner. The landowner can file an original
action with the RTC acting as a special agrarian court to determine just
compensation. The court has the right to review with finality the
determination in the exercise of what is admittedly a judicial function.]
5.
NO, the 10-year period prohibition on the transfer of awarded lands
under RA 6657 has NOT lapsed on May 10, 1999; thus, the qualified FWBs
should NOT yet be allowed to sell their land interests in Hacienda Luisita to
third parties.
[Under RA 6657 and DAO 1, the awarded lands may only be transferred or
conveyed after 10 years from the issuance and registration of the
emancipation patent (EP) or certificate of land ownership award (CLOA).
Considering that the EPs or CLOAs have not yet been issued to the qualified
FWBs in the instant case, the 10-year prohibitive period has not even started.
Significantly, the reckoning point is the issuance of the EP or CLOA, and not
the placing of the agricultural lands under CARP coverage. Moreover, should
the FWBs be immediately allowed the option to sell or convey their interest
in the subject lands, then all efforts at agrarian reform would be rendered
nugatory, since, at the end of the day, these lands will just be transferred to
persons not entitled to land distribution under CARP.]
6.
YES, the ruling in the July 5, 2011 Decision that the qualified FWBs be
given an option to remain as stockholders of HLI should be reconsidered.
[The Court reconsidered its earlier decision that the qualified FWBs should be
given an option to remain as stockholders of HLI, inasmuch as these qualified
FWBs will never gain control [over the subject lands] given the present
proportion of shareholdings in HLI. The Court noted that the share of the
FWBs in the HLI capital stock is [just] 33.296%. Thus, even if all the holders
of this 33.296% unanimously vote to remain as HLI stockholders, which is
unlikely, control will never be in the hands of the FWBs. Control means the
majority of [sic] 50% plus at least one share of the common shares and other
voting shares. Applying the formula to the HLI stockholdings, the number of
shares that will constitute the majority is 295,112,101 shares (590,554,220
total HLI capital shares divided by 2 plus one [1] HLI share).
The
118,391,976.85 shares subject to the SDP approved by PARC substantially
fall short of the 295,112,101 shares needed by the FWBs to acquire control
over HLI.]

Page 101 of 1072

TOPIC: DOCTRINE OF
CORPORATE FICTION

CORPORATE

ENTITY/

PIERCING

THE

VEIL

OF

PANTRANCO EMPLOYEES ASSOCIATION (PEA-PTGWO) and


PANTRANCO RETRENCHED EMPLOYEES ASSOCIATION (PANREA),
Petitioners,
vs.
NATIONAL LABOR RELATIONS COMMISSION (NLRC), PANTRANCO
NORTH EXPRESS, INC. (PNEI), PHILIPPINE NATIONAL BANK (PNB),
PHILIPPINE NATIONAL BANK-MANAGEMENT AND DEVELOPMENT
CORPORATION (PNB-MADECOR), and MEGA PRIME REALTY AND
HOLDINGS CORPORATION (MEGA PRIME), Respondents.
G.R. No. 170689. March 17, 2009
PHILIPPINE NATIONAL BANK, Petitioner,
vs.
PANTRANCO EMPLOYEES ASSOCIATION, INC. (PEA-PTGWO),
PANTRANCO RETRENCHED EMPLOYEES ASSOCIATION (PANREA) AND
PANTRANCO ASSOCIATION OF CONCERNED EMPLOYEES (PACE), ET
AL., PHILIPPINE NATIONAL BANK-MANAGEMENT DEVELOPMENT
CORPORATION (PNB-MADECOR), and MEGA PRIME REALTY
HOLDINGS, INC., Respondents.
G.R. No. 170705. March 17, 2009
FACTS:
The Gonzales family owned two corporations, namely, the PNEI and
Macris Realty Corporation (Macris). PNEI provided transportation services to
the public, and had its bus terminal at the corner of Quezon and Roosevelt
Avenues in Quezon City. The terminal stood on four valuable pieces of real
estate (known as Pantranco properties) registered under the name of Macris.
The Gonzales family later incurred huge financial losses despite attempts of
rehabilitation and loan infusion. In March 1975, their creditors took over the
management of PNEI and Macris. By 1978, full ownership was transferred to
one of their creditors, the National Investment Development Corporation
(NIDC), a subsidiary of the PNB.
Macris was later renamed as the National Realty Development
Corporation (Naredeco) and eventually merged with the National
Warehousing Corporation (Nawaco) to form the new PNB subsidiary, the PNBMadecor.
In 1985, NIDC sold PNEI to North Express Transport, Inc. (NETI), a
company owned by Gregorio Araneta III. In 1986, PNEI was among the

Page 102 of 1072

several companies placed under sequestration by the Presidential


Commission on Good Government (PCGG) shortly after the historic events in
EDSA. In January 1988, PCGG lifted the sequestration order to pave the way
for the sale of PNEI back to the private sector through the Asset Privatization
Trust (APT). APT thus took over the management of PNEI.
In 1992, PNEI applied with the Securities and Exchange Commission
(SEC) for suspension of payments. A management committee was thereafter
created which recommended to the SEC the sale of the company through
privatization. As a cost-saving measure, the committee likewise suggested
the retrenchment of several PNEI employees. Eventually, PNEI ceased its
operation. Along with the cessation of business came the various labor
claims commenced by the former employees of PNEI where the latter
obtained favorable decisions.
ISSUE/S:
Whether there isa valid ground that may exist to warrant the
piercing of the corporate veil.
RULING:
Under the doctrine of "piercing the veil of corporate fiction," the court
looks at the corporation as a mere collection of individuals or an aggregation
of persons undertaking business as a group, disregarding the separate
juridical personality of the corporation unifying the group.
Another
formulation of this doctrine is that when two business enterprises are owned,
conducted and controlled by the same parties, both law and equity will,
when necessary to protect the rights of third parties, disregard the legal
fiction that two corporations are distinct entities and treat them as identical
or as one and the same.
Clearly, what can be inferred from the earlier cases is that the doctrine
of piercing the corporate veil applies only in three (3) basic areas, namely:
1. defeat of public convenience as when the corporate
fiction is used as a vehicle for the evasion of an
existing obligation;
2. fraud cases or when the corporate entity is used to
justify a wrong, protect fraud, or defend a crime; or
3. alter ego cases, where a corporation is merely a
farce since it is a mere alter ego or business conduit
of a person, or where the corporation is so organized
and controlled and its affairs are so conducted as to
make it merely an instrumentality, agency, conduit
or adjunct of another corporation.
In the absence of malice, bad faith, or a specific provision of law
making a corporate officer liable, such corporate officer cannot be made
personally liable for corporate liabilities.

Page 103 of 1072

Applying the foregoing doctrine to the instant case, we quote with approval
the CA disposition in this wise: It would not be enough, then, for the
petitioners in this case, the PNEI employees, to rest on their laurels with
evidence that PNB was the owner of PNEI. Apart from proving ownership, it is
necessary to show facts that will justify us to pierce the veil of corporate
fiction and hold PNB liable for the debts of PNEI. The burden undoubtedly
falls on the petitioners to prove their affirmative allegations. In line with the
basic jurisprudential principles we have explored, they must show that PNB
was using PNEI as a mere adjunct or instrumentality or has exploited or
misused the corporate privilege of PNEI.
The Court do not see how the burden has been met. Lacking proof of a
nexus apart from mere ownership, the petitioners have not provided us with
the legal basis to reach the assets of corporations separate and distinct from
PNEI.

Page 104 of 1072

TOPIC: DOCTRINE OF
CORPORATE FICTION

CORPORATE

ENTITY/

PIERCING

THE

VEIL

OF

CAGAYAN VALLEY DRUG CORPORATION, petitioner,


vs.
COMMISSIONER OF INTERNAL REVENUE, respondent.
G.R. No. 151413. February 13, 2008
545 SCRA 10
FACTS:
Petitioner, a corporation duly organized and existing under Philippine
laws, is a duly licensed retailer of medicine and other pharmaceutical
products. It operates two drugstores, one in Tuguegarao, Cagayan, and the
other in Roxas, Isabela, under the name and style of Mercury Drug.
Petitioner alleged that in 1995, it granted 20% sales discounts to
qualified senior citizens on purchases of medicine pursuant to Republic Act
No. (RA) 74323 and its implementing rules and regulations. In compliance
with Revenue Regulation No. (RR) 2-94, petitioner treated the 20% sales
discounts granted to qualified senior citizens in 1995 as deductions from the
gross sales in order to arrive at the net sales, instead of treating them as tax
credit as provided by Section 4 of RA 7432.
On December 27, 1996, however, petitioner filed with the Bureau of
Internal Revenue (BIR) a claim for tax refund/tax credit of the full amount of
the 20% sales discount it granted to senior citizens for the year 1995,
allegedly ]abelled] to PhP 123,083 in accordance with Sec. 4 of RA 7432.
The BIRs inaction on petitioners claim for refund/tax credit compelled
petitioner to file on March 18, 1998 a petition for review before the CTA
docketed as C.T.A. Case No. 5581 in order to forestall the two-year
prescriptive period provided under Sec. 2304 of the 1977 Tax Code, as
amended. Thereafter, on March 31, 2000, petitioner amended its petition for
review.
ISSUE/S:
Whether petitioners president can sign the subject verification
and certification sans the approval of its Board of Directors.
RULING:
It must be borne in mind that Sec. 23, in relation to Sec. 25 of the
Corporation Code, clearly enunciates that all corporate powers are exercised,

Page 105 of 1072

all business conducted, and all properties controlled by the board of


directors. A corporation has a separate and distinct personality from its
directors and officers and can only exercise its corporate powers through the
board of directors. Thus, it is clear that an individual corporate officer cannot
solely exercise any corporate power pertaining to the corporation without
authority from the board of directors. This has been our constant holding in
cases instituted by a corporation.
In a slew of cases, however, we have recognized the authority of some
corporate officers to sign the verification and certification against forum
shopping. In Mactan-Cebu International Airport Authority v. CA, we
recognized the authority of a general manager or acting general manager to
sign the verification and certificate against forum shopping; in Pfizer v.
Galan, we upheld the validity of a verification signed by an employment
specialist who had not even presented any proof of her authority to
represent the company, in Novelty Philippines, Inc., v. CA, we ruled that a
personnel officer who signed the petition but did not attach the authority
from the company is authorized to sign the verification and non-forum
shopping certificate;and in Lepanto Consolidated Mining Company v. WMC
Resources International Pty. Ltd. (Lepanto), we ruled that the Chairperson of
the Board and President of the Company can sign the verification and
certificate against non-forum shopping even without the submission of the
boards authorization.
In sum, we have held that the following officials or employees of the
company can sign the verification and certification without need of a board
resolution:
(1) the Chairperson of the Board of Directors,
(2) the President of a corporation,
(3) the General Manager or Acting General Manager,
(4) Personnel Officer, and
(5) an Employment Specialist in a labor case.
While the above cases do not provide a complete listing of authorized
signatories to the verification and certification required by the rules, the
determination of the sufficiency of the authority was done on a case to case
basis. The rationale applied in the foregoing cases is to justify the authority
of corporate officers or representatives of the corporation to sign the
verification or certificate against forum shopping, being in a position to
verify the truthfulness and correctness of the allegations in the petition.

Page 106 of 1072

TOPIC:DOCTRINE OF CORRPORATE ENTITY V. PIERCING THE VEIL


THE HEIRS OF THE LATE PANFILO V. PAJARILLO
VS.
THE HON. COURT OF APPEALS, ET, AL.
G.R. NO. 155056-57 OCTOBER 19, 2007
FACTS:
Panfilo V. Pajarillo was the owner and operator of several buses. He
used the name "PVP Liner" in his buses. Private respondents were employed
as drivers, conductors and conductresses by Panfilo.
Private respondents and several co-employees formed a union called
"SAMAHAN NG MGA MANGGAGAWA NG PANFILO V. PAJARILLO. DOLE issued a
Certificate of Registration in favor of the respondent union.
Upon learning of the formation of union, Panfilo and his children
ordered some of the private respondents to sign a document affirming their
trust and confidence in Panfilo and denying any irregularities on his part.
Other private respondents were directed to sign a blank document which
turned out to be a resignation letter. Private respondents refused to sign the
said documents, hence, they were barred from working or were dismissed
without hearing and notice. Panfilo and his children and relatives also formed
a company union where they acted as its directors and officers.
Respondent union and several employees filed a Complaint for unfair
labor practice and illegal deduction and illegal dismissal before the Labor
Arbiter. Labor Arbiter dismissed the complaint for lack of merit. NLRC
reversed the decision of Arbiter Asuncion and ordered the reinstatement of,
and payment of backwages, ECOLA, 13th month pay, legal holiday pay and
service incentive leave pay to, private respondents. The Court of Appeals
rendered a decision granting the respondent unions petition and nullifying
the Orders of the NLRC. It also reinstated the Decision of the NLRC.
ISSUE:
Whether or not the Court of Appeals seriously erred in piercing
the veil of corporate entity of PVP Pajarillo Liner Inc.

Page 107 of 1072

RULING:
It is a fundamental principle of corporation law that a corporation is an
entity separate and distinct from its stockholders and from other
corporations to which it may be connected. However, this separate and
distinct personality of a corporation is merely a fiction created by law for
convenience and to promote justice. Hence, when the notion of separate
juridical personality is used to defeat public convenience, justify wrong,
protect fraud or defend crime, or is used as a device to defeat labor laws,
this separate personality of the corporation may be disregarded or the veil of
the corporate fiction pierced. This is true likewise when the corporation is
merely an adjunct, a business conduit or an alter ego of another corporation.
The corporate mask may be lifted and the corporate veil may be pierced
when a corporation is but the alter ego of a person or another corporation.
It is apparent that Panfilo started his transportation business as the
sole owner and operator of passenger buses utilizing the name PVP Liner for
his buses. After being charged by respondent union of unfair labor practice,
illegal deductions, illegal dismissal and violation of labor standard laws,
Panfilo transformed his transportation business into a family corporation,
namely, P.V. Pajarillo Liner Inc. He and petitioners were the incorporators,
stockholders and officers therein. P.V. Pajarillo Inc. and the sole proprietorship
of Panfilo have the same business address. P.V. Pajarillo Inc. also uses the
name "PVP Liner" in its buses. Further, the license to operate or franchise of
the sole proprietorship was merely transferred to P.V. Pajarillo Liner Inc.
It is clear from the foregoing that P.V. Pajarillo Liner Inc. was a mere
continuation and successor of the sole proprietorship of Panfilo. It is also
quite obvious that Panfilo transformed his sole proprietorship into a family
corporation in a surreptitious attempt to evade the charges of respondent
union. Given these considerations, Panfilo and P.V. Pajarillo Liner Inc. should
be treated as one and the same person for purposes of liability.

Page 108 of 1072

TOPIC:DOCTRINE OF CORRPORATE ENTITY V. PIERCING THE VEIL


PETRON CORPORATION AND PETER C. MALIGRO
VS.
NATIONAL LABOR RELATIONS COMMISSION AND CHITO S. MANTOS
G.R. No. 154532, October 27, 2006
FACTS:
Petron is engaged in the refining, sale and distribution of petroleum
and other related products, while Peter C. Maligro was the former Visayas
Operations Assistant Manager of Petron's Visayas-Mindanao District Office at
Lahug, Cebu City.
Petron hired Chito S. Mantos, an Industrial Engineer, as a managerial,
professional and technical employee with initial designation as a Bulk Plant
Engineering Trainee. It was while assigned at Petron's Cebu District Office
with Peter Maligro as his immediate superior, when Mantos, thru a Notice of
Disciplinary Action, was suspended for 30 days for violating company rules
and regulations regarding AWOL, not having reported for work during the
period August 5 to 27, 1996.
Subsequently, in a notice Termination of Services, Mantos' services
were altogether terminated effective December 1, 1996, by reason of his
continued absences from August 28, 1996 onwards, as well as for
Insubordination/Discourtesy for making false accusations against his
superior.
Mantos filed with the NLRC, a complaint for illegal dismissal and other
monetary claims against Petron and/or Peter C. Maligro. The Labor Arbiter
declared Mantos to have been constructively dismissed but ruled that only
Petron could be held liable to him for separation pay in lieu of reinstatement
and the cash equivalent of his certificate of stocks, less his personal
accountabilities. The NLRC reversed the findings of the Labor Arbiter
regarding Mantos' constructive dismissal considered him to have been
illegally dismissed only on December 1, 1996 and the NLRC adjudged
Maligro solidarily liable with Petron The CA outrightly dismissed the petition
for being defective in form because only Petron signed the verification and
certification on non-forum shopping without its co-petitioner Peter Maligro
likewise signing the same.
ISSUE/S:

Page 109 of 1072

Whether or not the corporate officers can be held solidary liable


with the corporation in case of illegal dismissal.

RULING:
Settled is the rule in this jurisdiction that a corporation is invested by
law with a legal personality separate and distinct from those acting for and in
its behalf and, in general, from the people comprising it. Thus, obligations
incurred by corporate officers acting as corporate agents are not theirs but
the direct accountabilities of the corporation they represent. True, solidary
liabilities may at times be incurred by corporate officers, but only when
exceptional circumstances so warrant. For instance, in labor cases, corporate
directors and officers may be held solidarily liable with the corporation for
the termination of employment if done with malice or in bad faith.
In the present case, the apparent basis for the NLRC in holding
petitioner Maligro solidarily liable with Petron were its findings that
(1) the Investigation Committee was created a day after the
summons in NLRC RAB-VII Case No. 11-1439-96 was received,
with Maligro no less being the chairman thereof; and
(2) the basis for the charge of insubordination was the private
respondent's alleged making of false accusations against Maligro.
Those findings, however, cannot justify a finding of personal liability on
the part of Maligro inasmuch as said findings do not point to Maligro's
extreme personal hatred and animosity with the respondent. It cannot,
therefore, be said that Maligro was motivated by malice and bad faith in
connection with private respondent's dismissal from the service.

Page 110 of 1072

TOPIC:DOCTRINE OF CORRPORATE ENTITY V. PIERCING THE VEIL


CHINA BANKING CORPORATION
VS.
DYNE-SEM ELECTRONICS CORPORATION
G.R. No. 149237, June 11, 2006
FACTS:
Dynetics and Elpidio O. Lim borrowed a total of P8,939,000 from China
Banking Corporation. The loan was evidenced by six promissory notes. The
borrowers failed to pay when the obligations became due. Petitioner
consequently instituted a complaint for sum of money on June 25, 1987
against them. The complaint sought payment of the unpaid promissory notes
plus interest and penalties.
An amended complaint was filed by petitioner impleading Dyne-Sem
and its stockholders Vicente Chuidian, Antonio Garcia and Jacob Ratinoff.
According to petitioner, respondent was formed and organized to be
Dynetics alter ego as established by the following circumstances: that
Dynetics, Inc. and respondent are both engaged in the same line of business
of manufacturing, producing, assembling, processing, importing, exporting,
buying, distributing, marketing and testing integrated circuits and
semiconductor devices; that the principal office and factory site of Dynetics,
Inc. located at Avocado Road, FTI Complex, Taguig, Metro Manila, were used
by respondent as its principal office and factory site; that respondent
acquired some of the machineries and equipment of Dynetics, Inc. from
banks which acquired the same through foreclosure; that respondent
retained some of the officers of Dynetics, Inc.
The trial court ruled that Dyne-Sem Electronics Corporation is not
an alter ego of Dynetics, Inc. Thus, Dyne-Sem Electronics Corporation is not
liable under the promissory notes. The Court of Appeals dismissed the appeal
and affirmed the trial courts decision.
ISSUE/S:
Whether or not the trial court have ruled in accordance with law
and/or applicable jurisprudence to the extent that the doctrine of
piercing the veil of corporate fiction is not applicable in the case
at bar?

Page 111 of 1072

RULING:
The general rule is that a corporation has a personality separate and
distinct from that of its stockholders and other corporations to which it may
be connected. This is a fiction created by law for convenience and to prevent
injustice.
Nevertheless, being a mere fiction of law, peculiar situations or valid
grounds may exist to warrant the disregard of its independent being and the
piercing of the corporate veil. In Martinez v. Court of Appeals, we held: The
veil of separate corporate personality may be lifted when such personality is
used to defeat public convenience, justify wrong, protect fraud or defend
crime; or used as a shield to confuse the legitimate issues; or when the
corporation is merely an adjunct, a business conduit or an alter ego of
another corporation or where the corporation is so organized and controlled
and its affairs are so conducted as to make it merely an instrumentality,
agency, conduit or adjunct of another corporation; or when the corporation is
used as a cloak or cover for fraud or illegality, or to work injustice, or where
necessary to achieve equity or for the protection of the creditors. In such
cases, the corporation will be considered as a mere association of persons.
The liability will directly attach to the stockholders or to the other
corporation. To disregard the separate juridical personality of a corporation,
the wrongdoing must be proven clearly and convincingly.
In this case, petitioner failed to prove that Dyne-Sem was organized
and controlled, and its affairs conducted, in a manner that made it merely an
instrumentality, agency, conduit or adjunct of Dynetics, or that it was
established to defraud Dynetics creditors, including petitioner. The similarity
of business of the two corporations did not warrant a conclusion that
respondent was but a conduit of Dynetics. As we held in Umali v. Court of
Appeals, "the mere fact that the businesses of two or more corporations are
interrelated is not a justification for disregarding their separate personalities,
absent sufficient showing that the corporate entity was purposely used as a
shield to defraud creditors and third persons of their rights." Likewise,
respondents acquisition of some of the machineries and equipment of
Dynetics was not proof that respondent was formed to defraud petitioner. As
the Court of Appeals found, no mergertook place between Dynetics and
respondent Dyne-Sem. What took place was a sale of the assets of the
former to the latter. Merger is legally distinct from a sale of assets. Thus,
where one corporation sells or otherwise transfers all its assets to another
corporation for value, the latter is not, by that fact alone, liable for the debts
and liabilities of the transferor. Petitioner itself admits that respondent
acquired the machineries and equipment not directly from Dynetics but from
the various corporations which successfully bidded for them in an auction
sale. The contracts of sale executed between the winning bidders and
respondent showed that the assets were sold for considerable amounts. The

Page 112 of 1072

Court of Appeals thus correctly ruled that the assets were not "diverted" to
respondent as an alter ego of Dynetics. The machineries and equipment
were transferred and disposed of by the winning bidders in their capacity as
owners. The sales were therefore valid and the transfers of the properties to
respondent legal and not in any way in contravention of petitioners rights as
Dynetics creditor. Finally, it may be true that respondent later hired
Dynetics former Vice-President Luvinia Maglaya and Assistant Corporate
Counsel Virgilio Gesmundo. From this, however, we cannot conclude that
respondent was an alter ego of Dynetics. In fact, even the overlapping of
incorporators and stockholders of two or more corporations will not
necessarily lead to such inference and justify the piercing of the veil of
corporate fiction.

Page 113 of 1072

TOPIC:DOCTRINE OF CORRPORATE ENTITY V. PIERCING THE VEIL


THE EXECUTIVE SECRETARY, THE SECRETARY OF JUSTICE, THE
SECRETARY OF LABOR AND EMPLOYMENT, AND THE SECRETARY OF
FOREIGN AFFAIRS, OWWA PUNO, ADMINISTRATOR, AND POEA
ADMINISTRATOR,
VS.
THE HON. COURT OF APPEALS AND ASIAN RECRUITMENT COUNCIL
PHILIPPINE CHAPTER (ARCO-PHIL.), INC., REPRESENTING ITS
MEMBERS: WORLDCARE SERVICES INTERNATIONALE, INC.,
STEADFAST INTERNATIONAL RECRUITMENT CORPORATION, DRAGON
INTERNATIONAL MANPOWER SERVICES CORPORATION, VERDANT
MANPOWER MOBILIZATION CORPORATION, BRENT OVERSEAS
PERSONNEL, INC., ARL MANPOWER SERVICES, INC., DAHLZHEN
INTERNATIONAL SERVICES, INC., INTERWORLD PLACEMENT CENTER,
INC., LAKAS TAO CONTRACT SERVICES, LTD. CO., AND SSC
MULTISERVICES,
G.R. NO. 131719, MAY 25, 2004
FACTS:
The Omnibus Rules and Regulations Implementing the Migrant Workers
and Overseas Filipino Act of 1995 was, thereafter, published in the April 7,
1996 issue of the Manila Bulletin. However, even before the law took effect
ARCO-Phil. filed, o petition for declaratory relief under Rule 63 of the Rules of
Court with the Regional Trial Court of Quezon City to declare as
unconstitutional Section 2, paragraph (g), Section 6, paragraphs (a) to (j), (l)
and (m), Section 7, paragraphs (a) and (b), and Sections 9 and 10 of the law,
with a plea for the issuance of a temporary restraining order and/or writ of
preliminary injunction enjoining the respondents therein from enforcing the
assailed provisions of the law.
It prayed that the court issue a temporary restraining order to enjoin
the enforcement of Section 6, paragraphs (a) to (m) on illegal recruitment,
Section 7 on penalties for illegal recruitment, and Section 9 on venue of
criminal actions for illegal recruitments. The respondent alleged that Section
6, subsections (a) to (m) is unconstitutional because licensed and authorized
recruitment agencies are placed on equal footing with illegal recruiters. It
contended that while the Labor Code distinguished between recruiters who
are holders of licenses and non-holders thereof in the imposition of penalties,

Page 114 of 1072

Rep. Act No. 8042 does not make any distinction. The penalties in Section
7(a) and (b) being based on an invalid classification are, therefore, repugnant
to the equal protection clause, besides being excessive; hence, such
penalties are violative of Section 19(1), Article III of the Constitution. 9 It was
also pointed out that the penalty for officers/officials/employees of
recruitment agencies who are found guilty of economic sabotage or largescale illegal recruitment under Rep. Act No. 8042 is life imprisonment. Since
recruitment agencies usually operate with a manpower of more than three
persons, such agencies are forced to shut down, lest their officers and/or
employees be charged with large scale illegal recruitment or economic
sabotage and sentenced to life imprisonment. Thus, the penalty imposed by
law, being disproportionate to the prohibited acts, discourages the business
of licensed and registered recruitment agencies.
The respondent also posited that Section 6(m) and paragraphs (15)
and (16), Sections 8, 9 and 10, paragraph 2 of the law violate Section 22,
Article III of the Constitution10 prohibiting ex-post facto laws and bills of
attainder. This is because the provisions presume that a licensed and
registered recruitment agency is guilty of illegal recruitment involving
economic sabotage, upon a finding that it committed any of the prohibited
acts under the law. Furthermore, officials, employees and their relatives are
presumed guilty of illegal recruitment involving economic sabotage upon
such finding that they committed any of the said prohibited acts.
The appellate court dismissed the petition and affirming the assailed
order and writ of preliminary injunction issued by the trial court.
ISSUE/S:
Whether or not the trial court committed grave abuse of its
discretion amounting to excess or lack of jurisdiction in issuing
the assailed order and the writ of preliminary injunction on a
bond of only P50,000
RULING:
The validity of Section 6 of R.A. No. 8042 which provides that
employees of recruitment agencies may be criminally liable for illegal
recruitment has been upheld in People v. Chowdury:
As stated in the first sentence of Section 6 of RA 8042, the
persons who may be held liable for illegal recruitment are the
principals, accomplices and accessories. An employee of a
company or corporation engaged in illegal recruitment may be
held liable as principal, together with his employer, if it is shown
that he actively and consciously participated in illegal
recruitment. It has been held that the existence of the corporate
entity does not shield from prosecution the corporate agent who

Page 115 of 1072

knowingly and intentionally causes the corporation to commit a


crime. The corporation obviously acts, and can act, only by and
through its human agents, and it is their conduct which the law
must deter. The employee or agent of a corporation engaged in
unlawful business naturally aids and abets in the carrying on of
such business and will be prosecuted as principal if, with
knowledge of the business, its purpose and effect, he consciously
contributes his efforts to its conduct and promotion, however
slight his contribution may be.
By its rulings, the Court thereby affirmed the validity of the assailed
penal and procedural provisions of Rep. Act No. 8042, including the
imposable penalties therefor. Until the Court, by final judgment, declares that
the said provisions are unconstitutional, the enforcement of the said
provisions cannot be enjoined.

Page 116 of 1072

TOPIC:DOCTRINE OF CORRPORATE ENTITY V. PIERCING THE VEIL


R & E TRANSPORT, INC., AND HONORIO ENRIQUEZ
VS.
AVELINA P. LATAG, REPRESENTING HER DECEASED HUSBAND, PEDRO
M. LATAG
G.R. NO. 155214, FEBRUARY 13, 2004
FACTS:
Pedro Latag was a regular employee of La Mallorca Taxi since March 1,
1961. When La Mallorca ceased from business operations, Latag transferred
to R & E Transport, Inc. Latag got sick in January 1995 and was forced to
apply for partial disability with the SSS, which was granted. When he
recovered, he reported for work in September 1998 but was no longer
allowed to continue working on account of his old age.
Latag thus asked the administrative officer of petitioners, for his
retirement pay pursuant to Republic Act 7641 but he was ignored. Thus
Latag filed a case for payment of his retirement pay before the NLRC.
The Labor Arbiter rendered a decision in favor of Latag. On January 21,
2000, Avelina Latag was invited to the office of petitioners counsel and was
offered the amount of P38,500.00, which she accepted and asked to sign an
already prepared quitclaim and release and a joint motion to dismiss the
case. After a day or two, respondent, received a copy of the January 10, 2000
Decision of the Labor Arbiter. Petitioners interposed an appeal before the
NLRC.
NLRC dismissed the appeal for failure to post a cash or surety bond, as
mandated by law. The CA ruled that the labor arbiters January 10, 2000
Decision and May 23, 2000 Order had already become final and executory.
ISSUE:
Whether or not the facts of the case warrants the application of
doctrine of piercing the veil of corporate fiction?

Page 117 of 1072

RULING:
The question of whether a corporation is a mere alter ego is one of
fact.
Piercing the veil of corporate fiction may be allowed only if the
following elements concur:
1. control -- not mere stock control, but complete domination
-- not only of finances, but of policy and business practice
in respect to the transaction attacked, must have been
such that the corporate entity as to this transaction had at
the time no separate mind, will or existence of its own;
2. such control must have been used by the defendant to
commit a fraud or a wrong to perpetuate the violation of a
statutory or other positive legal duty, or a dishonest and an
unjust act in contravention of plaintiffs legal right; and
3. the said control and breach of duty must have proximately
caused the injury or unjust loss complained of."
Respondent has not shown by competent evidence that one taxi
company had stock control and complete domination over the other or vice
versa. In fact, no evidence was presented to show the alleged renaming of
"La Mallorca Taxi" to "R & E Transport, Inc." The seven-year gap between the
time the former closed shop and the date when the latter came into being
also casts doubt on any alleged intention of petitioners to commit a wrong or
to violate a statutory duty. This lacuna in the evidence compels us to reverse
the Decision of the CA affirming the labor arbiters finding of fact that the
basis for computing Pedros retirement pay should be 37 years, instead of
only 14 years.

Page 118 of 1072

TOPIC:DOCTRINE OF CORPORATE ENTITY VS. PIERCING THE VEIL OF


CORPORATE
FICTION
HEIRS OF TRINIDAD DE LEON VDA. DE ROXAS
VS.
COURT OF APPEALS AND MAGUESUN MANAGEMENT AND
DEVELOPMENT CORPORATION
G.R. NO. 138660, FEBRUARY 5, 2004
422 SCRA 101
FACTS:
In a suit filed by the Roxas heirs to set aside the decree of registration
over two unregistered parcels of land in Tagaytay City granted to Maguesun
before the RTC but has since reached the Supreme Court, Meycauayan
Central Realty filed a Petition for Intervention alleging that it purchased three
parcels of land from Maguesun which form part of the property awarded to
the Roxas heirs. The Supreme Court denied the said Petition and
subsequently, the Motion for Reconsideration of Meycauayan. Thus, the
decision became final on August 21, 1997.
However, despite the finality of the decision in favour of the Roxas
heirs, Meycauayan filed a Complaint for reconveyance, damages and
quieting of title with the trial court entitled Meycauayan Central Realty Corp.
v. Heirs of Manual A. Roxas and Trinidad de Leon vda. De Roxas, Maguesun
Management and Development Corp., Register of Deeds of Tagaytay City,
City Assessor of Tagaytay City and Land Registration Authority, which is an
almost exact reproduction of the Petition for Intervention filed by
Meycauayan before the Supreme Court.
The trial court dismissed the said Complaint for lack of merit. The court
further held that Meycauayan is guilty of forum shopping.
Meanwhile, the Roxas heirs filed on June 2, 1999 before the Supreme
Court a Petition to cite for indirect contempt the officers of Meycauayan.
ISSUE/S:
Whether or not the corporation and its officers may be held liable
for contempt.

Page 119 of 1072

RULING:
The general rule is that a corporation and its officers and agents may
be held liable for contempt. A corporation and those who are officially
responsible for the conduct of its affairs may be punished for contempt in
disobeying judgments, decrees, or orders of a court made in a case within its
jurisdiction.
Under Section 1 of Rule 71 of the Rules of Court, direct contempt is
punishable by a fine not exceeding two thousand pesos (P2,000) or
imprisonment not exceeding ten (10) days, or both, if committed against a
Regional Trial Court or a court of equivalent or higher rank. Hence,
Meycauayan and its Executive Vice President Juan M. Lamson, Jr. are each
fined P2,000 for direct contempt of court for forum shopping.
WHEREFORE, we find Meycauayan Central Realty Corporations
Executive Vice President Juan M. Lamson, Jr. GUILTY of INDIRECT CONTEMPT
and FINE him TEN THOUSAND PESOS (P10,000). Furthermore, we find
Meycauayan Central Realty Corporation and its Executive Vice President Juan
M. Lamson, Jr. GUILTY of DIRECT CONTEMPT for forum shopping and FINE
them TWO THOUSAND PESOS (P2,000) each. The Court warns them that a
repetition of the same or similar offense shall merit a more severe penalty.

Page 120 of 1072

TOPIC:DOCTRINE OF CORPORATE ENTITY VS. PIERCING THE VEIL OF


CORPORATEFICTION
MEL V. VELARDE, petitioner,
vs.
LOPEZ, INC., respondent.
G.R. NO. 153886, JANUARY 14, 2004
419 SCRA 422
FACTS:
A Notarized loan agreement covering the amount of 10 million pesos
was executed between, Eugenio Lopez Jr., then President of respondent
Lopez, Inc., as LENDER, and petitioner Mel Velarde, then General Manager of
Sky Vision Corporation, a subsidiary of respondent, as BORROWER.
Petitioner, however, defaulted in payment of the agreed instalments for the
aforesaid loan. The respondent, in response to the letter of petitioner,
advised him by letter that he could use his retirement benefits in Sky Vision
in partial settlement of his loan after he settles his accountabilities to the
latter and gives his written instructions to it. Petitioner however, refused,
asserting that the imputed unliquidated advances from Sky Vision had
already been properly liquidated.
Thus, respondent filed an action for collection of sum of money with
damages before the Pasig City RTC. Herein petitioner filed his answer which
included a counterclaim against respondent asserting that he is entitled to
retirement benefits from Sky Vision, unpaid salaries, unpaid incentives,
unpaid share from the net income of Lopez, Inc., equity in his service vehicle,
reasonable return on the stock ownership plan for services rendered as
General Manager, and moral damages and attorneys fees. Respondent
moved to dismiss the said counterclaim for lack of jurisdiction which drew
petitioner to assert in his comment and opposition thereto that the veil of
corporate fiction must be pierced to hold respondent liable for his
counterclaims. The RTC denied respondents Motion to Dismiss holding
among others that there exists an identity of interest between Sky Vision and
Lopez, Inc. to warrant the piercing of the veil of corporate fiction.
ISSUE:
Whether or not the existence of a parent-subsidiary relationship
between corporations is sufficient ground to pierce the veil of
corporate fiction.

Page 121 of 1072

RULING:
It cannot be gainsaid that a subsidiary has an independent and
separate juridical personality, distinct from that of its parent company,
hence, any claim or suit against the latter does not bind the former and vice
versa.
Petitioner argues nevertheless that jurisdiction over the subsidiary is
justified by piercing the veil of corporate fiction. Piercing the veil of corporate
fiction is warranted, however, only in cases when the separate legal entity is
used to defeat public convenience, justify wrong, protect fraud, or defend
crime, such that in the case of two corporations, the law will regard the
corporations as merged into one. The rationale behind piercing a
corporations identity is to remove the barrier between the corporation from
the persons comprising it to thwart the fraudulent and illegal schemes of
those who use the corporate personality as a shield for undertaking certain
proscribed activities.
In applying the doctrine of piercing the veil of corporate fiction, the
following requisites must be established:
1. control, not merely majority or complete stock
control;
2. such control must have been used by the defendant
to commit fraud or wrong, to perpetuate the violation
of a statutory or other positive legal duty, or
dishonest acts in contravention of plaintiffs legal
rights; and
3. the aforesaid control and breach of duty must
proximately cause the injury or unjust loss
complained of.
Nowhere, however, in the pleadings and other records of the case can
it be gathered that respondent has complete control over Sky Vision, not only
of finances but of policy and business practice in respect to the transaction
attacked, so that Sky Vision had at the time of the transaction no separate
mind, will or existence of its own. The existence of interlocking directors,
corporate officers and shareholders is not enough justification to pierce the
veil of corporate fiction in the absence of fraud or other public policy
considerations.
This Court is thus not convinced that the real party-in-interest with
regard to the counterclaim for damages arising from the alleged tortuous
manner by which petitioner was forced to retire as General Manager of Sky
Vision is respondent.
WHEREFORE, the instant petition for review on certiorari is hereby
DENIED.

Page 122 of 1072

TOPIC:DOCTRINE OF CORPORATE ENTITY VS. PIERCING THE VEIL OF


CORPORATE FICTION
PHILIPPINE NATIONAL BANK, petitioner,
vs.
RITRATTO GROUP INC., RIATTO INTERNATIONAL, INC., and DADASAN
GENERAL MERCHANDISE, respondents.
G.R. NO. 142616, JULY 31, 2001
362 SCRA 216
FACTS:
On May 29, 1996, PNB International Finance Ltd. (PNB-IFL) a subsidiary
company of PNB, organized and doing business in Hong Kong, extended a
letter of credit in favor of the respondents in the amount of US$300,000.00
secured by real estate mortgages constituted over 4 parcels of land in Makati
City. Respondents made repayments of the loan incurred by remitting those
amounts to their loan account with PNB-IFL in Hong Kong.
However, they defaulted in payment. Thus, PNB-IFL, through its
attorney-in-fact PNB, notified the respondents of the foreclosure of all the
real estate mortgages and that the properties subject thereof were to be sold
at a public auction.
However, herein respondents filed before the RTC of Makati a complaint for
injunction with prayer for the issuance of a writ of preliminary injunction
and/or temporary restraining order to enjoin PNB from foreclosing the said
properties. Petitioner, on the other hand, assailed the said petition through a
Motion to Dismiss on the ground that it does not state a cause of action and
there is no privity of contract between PNB and the respondents, and that
PNB is merely an agent of PNB-IFL.
The RTC granted the said injunction on the ground that since PNB-IFL,
is a wholly owned subsidiary of defendant Philippine National Bank, the suit
against the defendant PNB is a suit against PNB-IF.
ISSUE/S:
Whether or not the RTC erred in granting the said injunction
against PNB.

Page 123 of 1072

RULING:
The general rule is that as a legal entity, a corporation has a
personality distinct and separate from its individual stockholders or
members, and is not affected by the personal rights, obligations and
transactions of the latter. The mere fact that a corporation owns all of the
stocks of another corporation, taken alone is not sufficient to justify their
being treated as one entity. If used to perform legitimate functions, a
subsidiary's separate existence may be respected, and the liability of the
parent corporation as well as the subsidiary will be confined to those arising
in their respective business. The courts may in the exercise of judicial
discretion step in to prevent the abuses of separate entity privilege and
pierce the veil of corporate entity.
It is manifestly impossible to catalogue the infinite variations of fact
that can arise but there are certain common circumstances which are
important and which, if present in the proper combination, are controlling.
These are as follows:
a. The parent corporation owns all or most of the capital
stock of the subsidiary.
b. The parent and subsidiary corporations have
common directors or officers.
c. The parent corporation finances the subsidiary.
d. The parent corporation subscribes to all the capital
stock of the subsidiary or otherwise causes its
incorporation.
e. The subsidiary has grossly inadequate capital.
f. The parent corporation pays the salaries and other
expenses or losses of the subsidiary.
g. The subsidiary has substantially no business except
with the parent corporation or no assets except those
conveyed to or by the parent corporation.
h. In the papers of the parent corporation or in the
statements of its officers, the subsidiary is described
as a department or division of the parent
corporation, or its business or financial responsibility
is referred to as the parent corporation's own.
i. The parent corporation uses the property of the
subsidiary as its own.
j. The directors or executives of the subsidiary do not
act independently in the interest of the subsidiary
but take their orders from the parent corporation.
k. The formal legal requirements of the subsidiary are
not observed.
The Supreme Court have held that the doctrine of piercing the
corporate veil is an equitable doctrine developed to address situations where

Page 124 of 1072

the separate corporate personality of a corporation is abused or used for


wrongful purposes. The doctrine applies when the corporate fiction is used to
defeat public convenience, justify wrong, protect fraud or defend crime, or
when it is made as a shield to confuse the legitimate issues, or where a
corporation is the mere alter ego or business conduit of a person, or where
the corporation is so organized and controlled and its affairs are so
conducted as to make it merely an instrumentality, agency, conduit or
adjunct of another corporation.
In Concept Builders, Inc. v. NLRC, the Supreme Court have laid the test
in determining the applicability of the doctrine of piercing the veil of
corporate fiction, to wit:
1. Control, not mere majority or complete control, but complete
domination, not only of finances but of policy and business
practice in respect to the transaction attacked so that the
corporate entity as to this transaction had at the time no
separate mind, will or existence of its own.
2. Such control must have been used by the defendant to commit
fraud or wrong, to perpetuate the violation of a statutory or other
positive legal duty, or dishonest and, unjust act in contravention
of plaintiffs legal rights; and,
3. The aforesaid control and breach of duty must proximately cause
the injury or unjust loss complained of.
The absence of any
corporate veil." In applying
courts are concerned with
operated and the individual

one of these elements prevents "piercing the


the "instrumentality" or "alter ego" doctrine, the
reality and not form, with how the corporation
defendant's relationship to the operation.

Aside from the fact that PNB-IFL is a wholly owned subsidiary of


petitioner PNB, there is no showing of the indicative factors that the former
corporation is a mere instrumentality of the latter are present. Neither is
there a demonstration that any of the evils sought to be prevented by the
doctrine of piercing the corporate veil exists. Inescapably, therefore, the
doctrine of piercing the corporate veil based on the alter ego or
instrumentality doctrine finds no application in the case at bar.
IN VIEW OF THE FOREGOING, the petition is hereby GRANTED. The assailed
decision of the Court of Appeals is hereby REVERSED. The Orders dated June
30, 1999 and October 4, 1999 of the Regional Trial Court of Makati, Branch
147 in Civil Case No. 99-1037 are hereby ANNULLED and SET ASIDE and the
complaint in said case DISMISSED.

Page 125 of 1072

TOPIC:DOCTRINE OF CORPORATE ENTITY VS. PIERCING THE VEIL OF


CORPORATE FICTION
SALVADOR O. BOOC, complainant,
vs.
MALAYO B. BANTUAS, SHERIFF IV, RTC, BRANCH 3, ILIGAN CITY,
respondent.
A.M. No. P-01-1464, March 13, 2001
354 SCRA 279
FACTS:
Herein respondent Sheriff Malayo B. Bantuas, pursuant to a Writ of
Execution issued in Civil Case No. 1718 filed a Notice of Levy with the
Register of Deeds, Iligan City over a parcel of land covered by TCT No. T19209 and owned by Five Star Marketing Corporation. The corporation
through the complainant reiterated to respondent sheriff that it was the
owner of the property and Rufino Booc had no share or interest in the
corporation. Respondent sheriff, however, did not heed the corporations
demand. Thus, after due notice, he scheduled the public auction on August
31, 1999. Consequently, the corporation, to protect its rights and interests,
filed an action for Quieting of Title with the RTC, Branch 4 of Iligan City.
Bantuas, in his answer to the complaint filed against him before the
OCA, said that he filed a Notice of Levy with the Register of Deeds of Iligan
City on the share, rights, interest and participation of Rufino Booc in the
parcel of land owned by Five Star Marketing Corporation. Respondent sheriff
claimed that Rufino Booc is the owner of around 200 shares of stock in said
corporation according to a document issued by the Securities and Exchange
Commission. Moreover, Bantuas averred that the corporation is merely a
dummy of Rufino Booc and his brother Sheikding Booc based on an affidavit
executed by the latter.
ISSUE/S:
Whether or not the sheriff erred in levying the property belonging
to the corporation on the ground that said corporation is merely
a dummy of petitioner.

Page 126 of 1072

RULING:
Respondent sheriff, however, overstepped his authority when he
disregarded the distinct and separate personality of the corporation from
that of Rufino Booc as stockholder of the corporation by levying on the
property of the corporation. Respondent sheriff should not have made the
levy based on mere conjecture that since Rufino Booc is a stockholder and
officer of the corporation, then he might have an interest or share in the
subject property.
It is settled that a corporation is clothed with a personality separate
and distinct from that of its stockholders. It may not be held liable for the
personal indebtedness of its stockholders. In the case of Del Rosario vs.
Bascar, Jr., we imposed the fine of P5,000.00 on respondent sheriff Bascar for
allocating unto himself the power of the court to pierce the veil of corporate
entity and improvidently assuming that since complainant Esperanza del
Rosario is the treasurer of Miradel Development Corporation, they are one
and the same. In the said case we reiterated the principle that the mere fact
that one is a president of the corporation does not render the property he
owns or possesses the property of the corporation since the president, as an
individual, and the corporation are separate entities.
WHEREFORE , respondent Malayo B. Bantuas, Sheriff IV of the RTC of
Iligan City , Branch 3, is hereby FINED in the sum of Five Thousand Pesos
(P5,000.00) with the STERN WARNING that a repetition of the same or similar
acts in the future will be dealt with more severely.

Page 127 of 1072

TOPIC:DOCTRINE OF CORPORATE ENTITY VS. PIERCING THE VEIL OF


CORPORATE FICTION
MARUBENI CORPORATION, RYOICHI TANAKA, RYOHEI KIMURA and
SHOICHI ONE, petitioners,
vs.
FELIX LIRAG, respondent.
G.R. NO. 130998, August 10, 2001
FACTS:
Respondent Felix Lirag filed with the RTC Makati a complaint for
specific performance and damages claiming that petitioners owed him the
sum of P6,000,000.00 representing commission pursuant to an oral
consultancy agreement with Marubeni. Lirag claimed that on February 2,
1987, petitioner Ryohei Kimura hired his consultancy group for the purpose
of obtaining government contracts of various projects. Petitioners allegedly
promised to pay him 6% consultancy fee based on the total costs of the
projects obtained. Lirag claims that one of the projects handled he handled
was the Bureau of Post project, amounting to P100,000,000.00 which was
awarded to the "Marubeni-Sanritsu tandem." He further claims that Marubeni
and Sanritsu are sister corporations, and that it is really Marubeni who is the
supplier and real contractor of the project and subcontracted the same to
Sanritsu, thereby implying the need to pierce the veil of corporate fiction.
Lirag alleged that despite his repeated formal verbal demands for
payment of the agreed consultancy fee, petitioners did not pay.
In their answer, petitioners denied the consultancy agreement on the
ground that petitioner Ryohei Kimura did not have the authority to enter into
such agreement in behalf of Marubeni and that Marubeni did not participate
in the bidding for the Bureau of Post project, nor benefited from the
supposed project.
Both the RTC and the CA ruled in favour of Lirag finding that there exists an
oral consultancy agreement between his consultancy group and Marubeni.
ISSUE/S:

Page 128 of 1072

Whether or not piercing the veil of corporate fiction between


Marubeni and Sanritsu is justified.
RULING:
Not because two foreign companies came from the same country and
closely worked together on certain projects would the conclusion arise that
one was the conduit of the other, thus piercing the veil of corporate fiction.
To disregard the separate juridical personality of a corporation, the
wrongdoing must be clearly and convincingly established. It cannot be
presumed. The separate personality of the corporation may be disregarded
only when the corporation is used as a cloak or cover for fraud or illegality, or
to work injustice, or where necessary for the protection of creditors. We could
not just rely on respondent's testimony regarding the existence of the
"Marubeni-Sanritsu tandem" to justify his claim for payment of commission.
This conclusion is too conjectural to be believed.
Aside from the self-serving testimony of respondent regarding the
existence of a close working relationship between Marubeni and Sanritsu,
there was nothing that would support the conclusion that Sanritsu was an
agent of Marubeni.

Page 129 of 1072

TOPIC: DOCTRINE OF CORPORATE ENTITY VS. PIERCING THE VEIL OF


CORPORATE FICTION
ADALIA B. FRANCISCO and MERRYLAND DEVELOPMENT
CORPORATION, petitioners,
vs.
RITA C. MEJIA, as Executrix of Testate Estate of ANDREA CORDOVA
VDA. DE GUTERREZ, respondent.
G.R. No. 141617
August 14, 2001
FACTS:
Andrea Cordova Vda. de Gutierrez (Gutierrez) was the registered owner
of a parcel of land in Camarin, Caloocan City known as Lot 861 of the Tala
Estate. The property was later subdivided into five lots with an area of five
hectares each and pursuant thereto, TCT No. 5779 was cancelled and five
new transfer certificates of title were issued in the name of Gutierrez.
On 21 December 1964, Gutierrez and Cardale Financing and Realty
Corporation (Cardale) executed a Deed of Sale with Mortgage relating to the
lots covered by TCT Nos. 7124, 7125, 7126 and 7127, for the consideration
of P800, 000.00. Cardale paid Gutierrez P171,000.00. It was agreed that the
balance of P629,000.00 would be paid in several installments .Thereafter, the
titles of Gutierrez were cancelled and in lieu thereof TCT Nos. 7531 to 7534
were issued in favor of Cardale.
To secure payment of the balance of the purchase price, Cardale
constituted a mortgage on three of the four parcels. The encumbrance was
annotated upon the certificates of title and the owner's duplicate certificates.
The owner's duplicates were retained by Gutierrez.
On 26 August 1968, owing to Cardale's failure to settle its mortgage
obligation, Gutierrez filed a complaint for rescission of the contract.On 20
October 1969, during the pendency of the rescission case, Gutierrez died and
was substituted by her executrix, respondent Rita C. Mejia (Mejia). However,
Cardale, which was represented by petitioner Adalia B. Francisco (Francisco)
in her capacity as Vice-President and Treasurer of Cardale, lost interest in
proceeding with the presentation of its evidence and the case lapsed into
inactive status for a period of about fourteen years.

Page 130 of 1072

The mortgaged parcels of land became delinquent in paying real estate


taxes, thus it was levied in an auction sale on 1 and 12 September 1983. .
The highest bidder for the three parcels of land was petitioner Merryland
Development Corporation (Merryland), whose President and majority
stockholder is Francisco.
The title was later transferred in the name of Merryland. Mejia, in her
capacity as executrix of the Estate of Gutierrez, filed with the RTC of Quezon
City a complaint for damages with prayer for preliminary attachment against
Francisco, Merryland and the Register of Deeds of Caloocan City.
ISSUE/S:
Whether or not Cardale Financing and Realty Corporation and
Merryland Development Corporation are one and the same.
RULING:
No, they are not one and the same.
A corporation is a juridical person with a separate and distinct
personality from mat of the stockholders or members who compose it .
However, when the legal fiction of the separate corporate personality is
abused, such as when the same is used for fraudulent or wrongful ends, the
courts have not hesitated to pierce the corporate veil. When the notion of
legal entity is used to defeat public convenience, justify wrong, protect fraud,
or defend crime, the law will regard the corporation as an association of
persons.
With specific regard to corporate officers, the general rule is that the
officer cannot be held personally liable with the corporation, whether civilly
or otherwise, for the consequences of his acts, if he acted for and in behalf of
the corporation, within the scope of his authority and in good faith. In such
cases, the officer's acts are properly attributed to the corporation. However,
if it is proven that the officer has used the corporate fiction to defraud a third
party, or that he has acted negligently, maliciously or in bad faith, then the
corporate veil shall be lifted and he shall be held personally liable for the
particular corporate obligation involved.
The Court, after an assiduous study of this case, is convinced that the
totality of the circumstances appertaining conduce to the inevitable
conclusion that petitioner Francisco acted in bad faith. The events leading up
to the loss by the Gutierrez estate of its mortgage security attest to this. It
has been established that Cardale failed to comply with its obligation to pay
the balance of the purchase price for the four parcels of land it bought from
Gutierrez covered by TCT Nos. 7531 to 7534, which obligation was secured
by a mortgage upon the lands covered by TCT Nos. 7531, 7532 and 7533.
This prompted Gutierrez to file an action for rescission of the Deed of Sale
with Mortgage but the case dragged on for about fourteen years when
Cardale, as represented by Francisco, who was Vice-President and Treasurer

Page 131 of 1072

of the same, lost interest in completing its presentation of evidence.


Francisco knew that the properties subject of the mortgage had become tax
delinquent. In fact, as treasurer of Cardale, Francisco herself was the officer
charged with the responsibility of paying the realty taxes on the
corporation's properties. In addition, notices dated 9 July 1982 from the City
Treasurer of Caloocan demanding payment of the tax arrears on the subject
properties were sent directly to Francisco's address in White Plains, Quezon
City. Thus, as early as 1982, Francisco could have informed the Gutierrez
estate or the trial court in Civil Case No. Q-12366 of the tax arrears and of
the notice from the City Treasurer so that the estate could have taken the
necessary steps to prevent the auction sale and to protect its interests in the
mortgaged properties, but she did no such thing. Finally, in 1983, the
properties were levied upon and sold at public auction wherein Merryland
a corporation where Francisco is a stockholderand concurrently acts as
President and director was the highest bidder.
That Merryland acquired the property at the public auction only serves
to shed more light upon Francisco's fraudulent purposes. Based on the
findings of the Court of Appeals, Francisco is the controlling stockholder and
President of Merryland.Thus, aside from the instrumental role she played as
an officer of Cardale, in evading that corporation's legitimate obligations to
Gutierrez, it appears that Francisco's actions were also oriented towards
securing advantages for another corporation in which she had a substantial
interest.
The only act imputable to Merryland in relation to the mortgaged
properties is that it purchased the same and this by itself is not a fraudulent
or wrongful act. No evidence has been adduced to establish that Merryland
was a mere alter ego or business conduit of Francisco. Time and again it has
been reiterated that mere ownership by a single stockholder or by another
corporation of all or nearly all of the capital stock of a corporation is not of
itself sufficient ground for disregarding the separate corporate personality
.Neither has it been alleged or proven that Merryland is so organized and
controlled and its affairs are so conducted as to make it merely an
instrumentality, agency, conduit or adjunct of Cardale. Even assuming that
the businesses of Cardale and Merryland are interrelated, this alone is not
justification for disregarding their separate personalities, absent any showing
that Merryland was purposely used as a shield to defraud creditors and third
persons of their rights. Thus, Merryland's separate juridical personality must
be upheld.

Page 132 of 1072

TOPIC: DOCTRINE OF CORPORATE ENTITY VS. PIERCING THE VEIL OF


CORPORATE FICTION
LAND BANK OF THE PHILIPPINES, petitioner,
vs.
THE COURT OF APPEALS, ECO MANAGEMENT CORPORATION and
EMMANUEL C. OATE, respondents.
G.R. No. 127181
September 4, 2001
FACTS:
On various dates in September, October, and November, 1980,
appellant Land Bank of the Philippines (LBP) extended a series of credit
accommodations to appellee ECO, using the trust funds of the Philippine
Virginia Tobacco Administration (PVTA) in the aggregate amount of
P26,109,000.00. The proceeds of the credit accommodations were received
on behalf of ECO by appellee Oate.
On the respective maturity dates of the loans, ECO failed to pay the
same. ECO claims that the company was in financial difficulty for it was
unable to collect its investments with companies which were affected by the
financial crisis brought about by the Dewey Dee scandal. On October 20,
1981, ECO proposed and submitted to LBP a "Plan of Payment" whereby the
former would set up a financing company which would absorb the loan
obligations. It was proposed that LBP would participate in the scheme
through the conversion of P9,000,000.00 which was part of the total loan,
into equity.How ever such plan was rejected by the LBP and asked ECO for
the revision of the plan. ECO in turn delete the LBPs participation in the Plan
of Payment.
LBP then sent a letter to the PVTA for the latters comments. The letter
stated that if LBP did not hear from PVTA within five (5) days from the latters
receipt of the letter, such silence would be construed to be an approval of
LBPs intention to file suit against ECO and its corporate officers. PVTA did
not respond to the letter.
On June 28, 1982, Landbank filed a complaint for Collection of Sum of
Money against ECO and Emmanuel C. Oate.

Page 133 of 1072

ISSUE/S:
Whether or not the corporate veil of ECO Management Corporation
should be pierced.
Whether or not Emmanuel C. Oate should be held jointly and
severally liable with ECO Management Corporation for the loans
incurred from Land Bank.
RULING:
NO.
A corporation, upon coming into existence, is invested by law with a
personality separate and distinct from those persons composing it as well as
from any other legal entity to which it may be related. By this attribute, a
stockholder may not, generally, be made to answer for acts or liabilities of
the said corporation, and vice versa. This separate and distinct personality is,
however, merely a fiction created by law for convenience and to promote the
ends of justice. For this reason, it may not be used or invoked for ends
subversive to the policy and purpose behind its creationor which could not
have been intended by law to which it owes its being. This is particularly true
when the fiction is used to defeat public convenience, justify wrong, protect
fraud, defend crime,confuse legitimate legal or judicial issues,perpetrate
deception or otherwise circumvent the law.This is likewise true where the
corporate entity is being used as an alter ego, adjunct, or business conduit
for the sole benefit of the stockholders or of another corporate entity. In all
these cases, the notion of corporate entity will be pierced or disregarded with
reference to the particular transaction involved.
The burden is on petitioner to prove that the corporation and its stockholders
are, in fact, using the personality of the corporation as a means to perpetrate
fraud and/or escape a liability and responsibility demanded by law. In order
to disregard the separate juridical personality of a corporation, the
wrongdoing must be clearly and convincingly established.In the absence of
any malice or bad faith, a stockholder or an officer of a corporation cannot be
made personally liable for corporate liabilities.
The mere fact that Oate owned the majority of the shares of ECO is
not a ground to conclude that Oate and ECO are one and the same. Mere
ownership by a single stockholder of all or nearly all of the capital stock of a
corporation is not by itself sufficient reason for disregarding the fiction of
separate corporate personalities.Neither is the fact that the name "ECO"
represents the first three letters of Onates name sufficient reason to pierce
the veil. Even if it did, it does not mean that the said corporation is merely a
dummy of Oate. A corporation may assume any name provided it is lawful.
There is nothing illegal in a corporation acquiring the name or as in this case,
the initials of one of its shareholders.
That respondent corporation in this case was being used as a mere
alter ego of Oate to obtain the loans had not been shown. Bad faith or fraud

Page 134 of 1072

on the part of ECO and Oate was not also shown. As the Court of Appeals
observed, if shareholders of ECO meant to defraud petitioner, then they
could have just easily absconded instead of going out of their way to propose
"Plans of Payment."Likewise, Oate volunteered to pay a portion of the
corporations debt.This offer demonstrated good faith on his part to ease the
debt of the corporation of which he was a part. It is understandable that a
shareholder would want to help his corporation and in the process, assure
that his stakes in the said corporation are secured. In this case, it was
established that the P1 Million did not come solely from Oate. It was taken
from a trust account which was owned by Oate and other investors. It was
likewise proved that the P1 Million was a loan granted by Oate and his codepositors to alleviate the plight of ECO. This circumstance should not be
construed as an admission that he was really the debtor and not ECO.
As to the second issue, the evidence presented by the petitioner does
not suffice to hold respondent Oate personally liable for the debt of corespondent ECO.

Page 135 of 1072

TOPIC: DOCTRINE OF CORPORATE ENTITY VS. PIERCING THE VEIL OF


CORPORATE FICTION
PHILIPPINE NATIONAL BANK & NATIONAL SUGAR DEVELOPMENT
CORPORATION, petitioners,
vs.
ANDRADA ELECTRIC & ENGINEERING COMPANY, respondent.
G.R. No. 142936. April 17, 2002
381 SCRA 244
FACTS:
Andrada Electric and Engineering Company (AEEC) is a partnership
duly organized, existing and operating under the laws of the Philippines.
PNB acquired the assets of Pampanga Sugar Mills Inc. (PASUMIL) that
were earlier foreclosed by the Development Bank of the Philippines under LOI
No. 13. That PNB organized the NASUDECO in September 1975, to take
ownership and possession of assets and ultimately nationalize and
consolidate its interest in other PNB controlled sugar mills.
Prior to October 29, 1971, PASUMIL engaged the service of AEEC for
electrical rewinding and repair most of which were partially paid by the
defendant PASUMIL, leaving several unpaid accounts with the AEEC, that
finally on October 29, 1971 AEEC and PASUMIL entered into a contract to be
performed by AEEC.
Total obligation was P777, 263.80, PASUMIL paid only P250, 000.00
leaving a balance of P527, 263.80 and another payment of P14, 000.00, thus
total balance of P513, 263.80. The defendant PNB, PASUMIL and now
NASUDECO failed and refused to pay; that the President of the NASUDECO is
also the Vice-President of the PNB, and this official holds office at the 10 th
Floor of the PNB, Escolta, Manila, and plaintiff besought this official to pay
the outstanding obligation of the defendant PASUMIL, inasmuch as the
defendant PNB and NASUDECO now owned and possessed the assets of the
defendant PASUMIL, and these defendants all benefited from the works, and
the electrical, as well as the engineering and repairs, performed by the
plaintiff; that because of the failure and refusal of the defendants to pay their

Page 136 of 1072

just, valid, and demandable obligations, plaintiff suffered actual damages in


the total amount of P513,263.80; and that in order to recover these sums,
the plaintiff was compelled to engage the professional services of counsel, to
whom the plaintiff agreed to pay a sum equivalent to 25% of the amount of
the obligation due by way of attorneys fees. . Accordingly, the plaintiff
prayed that judgment be rendered against the defendants PNB, NASUDECO,
and PASUMIL to be jointly and severally liable.
PNB and NASUDECO filed a joint motion to dismiss on the ground that
they were not part to the contract, but the motion was denied and decision
was in favor of AEEC. The decision of the lower court was affirmed by the
Court of Appeals.
ISSUE/S:
Whether or not PNB and NASUDECO are liable for the obligations of
PASUMIL?
RULING:
General Rule states that, a corporation that purchases the assets of
another will not be liable for the debts of the selling corporation; provided
the former acted in good faith and paid adequate consideration. Except
under the following circumstances:
a) purchaser expressly or impliedly agrees to assume
the debts;
b) transaction amounts to consolidation or merger;
c) purchasing company is a merely continuation of the
selling company;
d) transaction is fraudulently entered into in order to
escape liability.
In the case at bar, we hold that there is no merger or consolidation
with respect to PASUMIL and PNB. The procedure prescribed under Title IX of
the Corporation Code was not followed.
In fact, PASUMILs corporate existence, as correctly found by the CA,
had not been legally extinguished or terminated. Further, prior to PNBs
acquisition of the foreclosed assets, PASUMIL had previously made partial
payments to respondent for the formers obligation in the amount of
P777,263.80.
As of June 27, 1973, PASUMIL had paid P250,000 to
respondent and, from January 5, 1974 to May 23, 1974, another P14,000.
Neither did petitioner expressly or impliedly agree to assume the debt
of PASUMIL to respondent. LOI No. 11 explicitly provides that PNB shall study
and submit recommendations on the claims of PASUMILs creditors. Clearly,

Page 137 of 1072

the corporate separateness between PASUMIL and PNB remains, despite


respondents insistence to the contrary.

TOPIC: DOCTRINE OF CORPORATE ENTITY VS. PIERCING THE VEIL OF


CORPORATE FICTION
AZCOR MANUFACTURING INC., FILIPINAS PASO and/or ARTURO
ZULUAGA/Owner, petitioners,
vs.
NATIONAL LABOR RELATIONS COMMISSION (NLRC) AND CANDIDO
CAPULSO, respondents.
G.R. No. 117963. February 11, 1999
303 SCRA 26
FACTS:
Capulso worked for Azcor as ceramics worker for more than two years.
However, an amount of P50.00 was deducted from his salary without
informing him of the reason therefore.
Capulso verbally requested to go on sick leave due to bronchial
asthma. He went back to petitioner to resume his work but was not allowed
informing him that only the owner, Zuluaga could allow him to continue in his
job.
He filed complaint for illegal dismissal.
Petitioner alleged that Capulso resigned from work and joined Filipinas
Paso but there was no vacancy.
The Labor Arbiter dismissed the complaint but the NLRC declared there
was illegal dismissal.
ISSUE/S:
Whether or not petitioner could be held jointly and severally liable to
Capulso for back wages since Azcor and Fili[inas Paso are separate and
distinct corporation with different personalities.

Page 138 of 1072

RULING:
Capulso had no knowledge that he was already working under Filipinas
Paso since he continued to retain his Azcor i.d; his pay slips contained the
name of Azcor giving the impression that Azcor was paying his salary. He was
paid the same salary and performing the same kind of job in the same work
area, location, using the same tools and under the same supervisor.
His employment contract with Filipinas Paso was signed by Azcor
personnel officer, which showed that Capulso was being hired from 1 March
1990 to 31 August 1990 by AZCOR to do jobs for Filipinas Paso.
It is evident from the foregoing discussion that Capulso was led into
believing that while he was working with Filipinas Paso, his real employer was
AZCOR. Petitioners never dealt with him openly and in good faith, nor was
he informed of the developments within the company, i.e., his alleged
transfer to Filipinas Paso and the closure of AZCORs manufacturing
operations beginning 1 March 1990. Understandably, he sued AZCOR alone
and was constrained to implead Filipinas Paso as additional respondent only
when it became apparent that the latter also appeared to be his employer.
The totality of the evidence was a veil attempt by petitioners to
deprive Capulso of what he had earned through hard labor by taking
advantage of his low level of education and confusing him as to who really
was his true employer - such a callous and despicable treatment of a worker
who had rendered faithful service to their company thus petitioners AZCOR
MANUFACTURING, INC., FILIPINAS PASO and ARTURO ZULUAGA are ORDERED
to pay, jointly and solidarily to the heirs of Capulso.

Page 139 of 1072

TOPIC: DOCTRINE OF CORPORATE ENTITY VS. PEIRCING THE VEIL


EDUARDO CLAPAROLS, ROMULO AGSAM and/or CLAPAROLS STEEL
AND NAIL PLANT, petitioners,
vs.
COURT OF INDUSTRIAL RELATIONS, ALLIED WORKERS' ASSOCIATION
and/or DEMETRIO GARLITOS, ALFREDO ONGSUCO, JORGE
SEMILLANO, SALVADOR DOROTEO, ROSENDO ESPINOSA, LUDOVICO
BALOPENOS, ASER AMANCIO, MAXIMO QUIOYO, GAUDENCIO QUIOYO,
and IGNACIO QUIOYO, respondents.
G.R. No. L-30822 July 31, 1975
65 SCRA 613
FACTS:
On August 6, 1957, unfair labor practice was filed against Claparol
Steel and Nail Plant. Claparol was ordered to reinstate the employees but
they were refused by the employer on several attempts on the ground that
there was no order from the corporation owner.
Claparol Steel and Nail Plant ceased operation on June 30, 1957 and
succeed by Claparol Steel Corporation on July 1, 1957.
Claparol Steel Corporation claimed that it ceased to operate on
December 7, 1962, thus re-employment of respondent workers cannot go
beyond December 7, 1962.
ISSUE/S:
Whether or not Claparol Steel Corporation is liable for backwages and
reinstatement being an alter ego of Claparol Steel and Nail
Corporation.
RULING:
YES.
Claparol Steel and Nail Plant, which ceased operation of June 30, 1957,
was SUCCEEDED by the Claparol Steel Corporation effective the next day,
July 1, 1957 up to December 7, 1962, when the latter finally ceased to
operate, were not disputed by petitioners.

Page 140 of 1072

It is very clear that the latter corporation was a continuation and


successor of the first entity, and its emergence was skillfully timed to avoid
the financial liability that already attached to its predecessor, the Claparol
Steel and Nail Plant. Both predecessors and successor were owned and
controlled by the petitioner Eduardo Claparol and there was no break in the
succession and continuity of the same business. This "avoiding-the-liability"
scheme is very patent, considering that 90% of the subscribed shares of
stocks of the Claparol Steel Corporation (the second corporation) was owned
by respondent (herein petitioner) Claparol himself, and all the assets of the
dissolved Claparol Steel and Nail Plant were turned over to the emerging
Claparol Steel Corporation.

Page 141 of 1072

TOPIC: DOCTRINE OF CORPORATE ENTITY VS. PIERCING THE VEIL OF


CORPORATE FICTION
COMMISSIONER OF INTERNAL REVENUE
vs.
NORTON and HARRISON COMPANY
G.R. No. L-17618
August 31, 1964
11 SCRA 714
FACTS:
Norton and Harrison is a corporation organized in 1911. Jackbilt is,
likewise, a corporation organized primarily for the purpose of making,
producing and manufacturing concrete blocks. Norton and Jackbilt entered
into an agreement whereby Norton was made the sole and exclusive
distributor of concrete blocks manufactured by Jackbilt. Pursuant to this
agreement, whenever an order for concrete blocks was received by the
Norton & Harrison Co. from a customer, the order was transmitted to Jackbilt
which delivered the merchandise direct to the customer. Payment for the
goods is, however, made to Norton, which in turn pays Jackbilt the amount
charged the customer less a certain amount, as its compensation or profit.
During the existence of the distribution or agency agreement, Norton &
Harrison acquired by purchase all the outstanding shares of stock of Jackbilt.
Apparently, due to this transaction, the Commissioner of Internal Revenue,
after conducting an investigation, assessed the respondent Norton &
Harrison for deficiency sales tax and surcharges. In other words, the
Commissioner considered the sale of Norton to the public as the original
sale and not the transaction from Jackbilt. As Norton and Harrison did not
conform with the assessment, the matter was brought to the Court of Tax
Appeals.
The Commissioner of Internal Revenue contends that since Jackbilt was
owned and controlled by Norton & Harrison, the corporate personality of the
former Jackbilt should be disregarded for sales tax purposes, and the sale of
Jackbilt blocks by petitioner to the public must be considered as the original
sales from which the sales tax should be computed. The Commissioner of
Internal Revenue appealed the issue to the SC.
ISSUE/S:
Whether the acquisition of all the stocks of the Jackbilt by the
Norton & Harrison Co., merged the two corporations into a single
corporation

Page 142 of 1072

RULING:
Yes.
The decision appealed from should be as it is hereby reversed and
another entered making the appellee Norton & Harrison liable for the
deficiency sales taxes assessed against it by the appellant Commissioner of
Internal Revenue, plus 25% surcharge thereon. If the income of Norton
should be considered separate from the income of Jackbilt, then each would
declare such earning separately for income tax purposes and thus pay lesser
income tax. The combined taxable Norton-Jackbilt income would subject
Norton to a higher tax. Based upon the 1954-1955 income tax return of
Norton and Jackbilt, and assuming that both of them are operating on the
same fiscal basis and their returns are accurate, we would have the following
result: Jackbilt declared a taxable net income of P161,202.31 in which the
income tax due was computed at P37,137.00. Whereas Norton declared as
taxable, a net income of P120, 101.59, on which the income tax due was
computed at P25, 628.00. The total of these liabilities is P50, 764.84. On the
other hand, if the net taxable earnings of both corporations are combined,
during the same taxable year, the tax due on their total which is P281,
303.90 would be P70, 764.00. So that, even on the question of income tax
alone, it would be to the advantages of Norton that the corporations should
be regarded as separate entities.

Page 143 of 1072

TOPIC: DOCTRINE OF CORPORATE ENTITY VS. PIERCING THE VEIL OF


CORPORATE FICTION
CONCEPT BUILDERS, INC.,
vs.
THE NATIONAL LABOR RELATIONS COMMISSION, (First Division); and
Norberto Marabe, Rodolfo Raquel, Cristobal Riego, Manuel Gillego,
Palcronio Giducos, Pedro Aboigar, Norberto Comendador, Rogello
Salut, Emilio Garcia, Jr., Mariano Rio, Paulina Basea, Alfredo Albera,
Paquito Salut, Domingo Guarino, Romeo Galve, Dominador Sabina,
Felipe Radiana, Gavino Sualibio, Moreno Escares, Ferdinand Torres,
Felipe Basilan, and Ruben Robalos
GR 108734
29 May 1996
257 SCRA 149
FACTS:
Concept Builders, Inc., (CBI) a domestic corporation, is engaged in the
construction business while. On November 1981, Marabe, et. al., were served
individual written notices of termination of employment by CBI, effective on
30 November 1981. It was stated in the individual notices that their
contracts of employment had expired and the project in which they were
hired had been completed.
The National Labor Relations Commission (NLRC) found it to be, the
fact, however, that at the time of the termination of Marabe, et.al.'s
employment, the project in which they were hired had not yet been finished
and completed. CBI had to engage the services of sub-contractors whose
workers performed the functions of Marabe, et. al.
Aggrieved, Marabe, et. al. filed a complaint for illegal dismissal. On 19
December 1984, the Labor Arbiter rendered judgment ordering CBI to
reinstate Marabe et. al. and to pay them back wages. On 27 November 1985,
the NLRC dismissed the motion for reconsideration. Labor Arbiter issued a
writ of execution directing the sheriff to execute the Decision. The writ was
partially satisfied through garnishment of sums from CBI's debtor.
On 1 February 1989, an Alias Writ of Execution was issued by the Labor
Arbiter directing the sheriff to collect from CBI the sum of P117, 414.76,
representing the balance of the judgment award, and to reinstate Marabe, et.
al. to their former positions. On 26 September 1986, upon motion of Marabe,
et. al., the Labor Arbiter issued a second alias writ of execution.

Page 144 of 1072

On 6 November 1989, a certain Dennis Cuyegkeng filed a third-party


claim with the Labor Arbiter alleging that the properties sought to be levied
upon by the sheriff were owned by HPPI, of which he is the Vice-President.
On 23 November 1989, Marabe, et. al. filed a Motion for Issuance of a
Break-Open Order," alleging that HPPI and CBI were owned by the same
incorporator/stockholders. Thereafter, it directed the sheriff to proceed with
the auction sale of the properties already levied upon. It dismissed the thirdparty claim for lack of merit. CBI moved for reconsideration but the motion
was denied by the NLRC in a Resolution, dated 3December 1992.
Hence, the petition.
ISSUE/S:
Whether the NLRC was correct in issuing the break-open order to
levy the HPPI properties.
RULING:
Yes.
It is a fundamental principle of corporation law that a corporation is an
entity separate and distinct from its stock holders and from other
corporations to which it may be connected. But, this separate and distinct
personality of a corporation is merely a fiction created by law for
convenience and to promote justice. So, when the notion of separate juridical
personality is used to defeat public convenience, justify wrong, protect fraud
or defend crime, or is used as a device to defeat the labor laws, this separate
personality of the corporation may be disregarded or the veil of corporate
fiction pierced. This is true likewise when the corporation is merely an
adjunct, a business conduit or an alter ego of another corporation. The
conditions under which the juridical entity may be disregarded vary
according to the peculiar facts and circumstances of each case.

Page 145 of 1072

TOPIC: DOCTRINE OF CORPORATE ENTITY VS. PIERCING THE VEIL OF


CORPORATE FICTION

COMPLEX ELECTRONICS CORPORATION


vs.
NATIONAL LABOR RELATIONS COMMISSION
G.R. No. 121315

July 19, 1999

310 SCRA 403

FACTS:
Complex Electronics Corporation (Complex) was engaged in the
manufacture of electronic products. The rank and file workers of Complex
were organized into a union known as the Complex Electronics Employees
Association, herein referred to as the Union. On March 4, 1992, Complex
received a facsimile message from Lite-On Philippines Electronics Co.,
requiring it to lower its price by 10%. Consequently, Complex informed its
Lite-On personnel that such request of lowering their selling price by 10%
were not feasible as they were already incurring losses at the present prices
of their products. Under such circumstances, Complex regretfully informed
the employees that it was left with no alternative but to close down the
operations of the Lite-On Line. The company, however, promised the
following:
1) Complex will follow the law by giving the people to be
retrenched the necessary 1 month notice. Hence,
retrenchment will not take place until after 1 month from
March 09, 1992.
2) The Company will try to prolong the work for as many
people as possible for as long as it can by looking for job
slots for them in another line if workload so allows and if
their skills are compatible with the line requirement.

Page 146 of 1072

3) The company will give the employees to be retrenched a


retrenchment pay as provided for by law i.e. half a month
for every year of service in accordance with Article 283 of
the Labor Code of Philippines.

The Union, on the other hand, pushed for a retrenchment pay


equivalent to one (1) month salary for every year of service, which Complex
refused. The Union filed a notice of strike with the National Conciliation and
Mediation Board (NCMB). A complaint was, thereafter, filed with the Labor
Arbitration Branch of the NLRC for unfair labor practice.
Ionics was
impleaded as a party defendant because the officers and management
personnel of Complex were also holding office at Ionics with Lawrence Qua
as the President of both companies.

Ionics contended that it was an entity separate and distinct from


Complex and had been in existence since July 5, 1984 or eight (8) years
before the labor dispute arose at Complex. Like Complex, it was also
engaged in the semi-conductor business where the machinery, equipment
and materials were consigned to them by their customers. On April 30,
1993, the Labor Arbiter rendered a decision in favor of the employees.

Separate appeals were filed by Complex, Ionics and Lawrence Qua


before the respondent NLRC who also ruled in favor of the employees.

Complex, Ionics and the Union filed their motions for reconsideration
which were denied.
Hence these petitions.

ISSUE/S:

Page 147 of 1072

Whether or not Ionics may be held liable together with complex on


the ground that both companies have the set of board of directors.
RULING:
No.

In the case at bar, petitioner seeks to pierce the veil of corporate


entity, alleging that the creation of the corporation is a devise to evade the
application of the CBA between petitioner Union and private respondent
company. Ionics may be engaged in the same business as that of Complex,
but this fact alone is not enough reason to pierce the veil of corporate fiction
of the corporation. Well-settled is the rule that a corporation has a
personality separate and distinct from that of its officers and
stockholders. This fiction of corporate entity can only be disregarded in
certain cases such as when it is used to defeat public convenience, justify
wrong, protect fraud, or defend crime. To disregard said separate juridical
personality of a corporation, the wrongdoing must be clearly and
convincingly established.

TOPIC: DOCTRINE OF CORPORATE ENTITY VS. PIERCING THE VEIL OF


CORPORATE FICTION

Page 148 of 1072

ROSAURA P. CORDON
vs.
JESUS BALICANTA
A.C. No. 2797

October 4, 2002

390 SCRA 299

FACTS:
When her husband Felixberto C. Jaldon died, Rosaura Cordon and her
daughter Rosemarie inherited the properties.
respondent enticed
complainant and her daughter to organize a corporation that would develop
the said real properties into a high-scale commercial complex with a
beautiful penthouse for complainant. Relying on these apparently sincere
proposals, complainant and her daughter assigned 19 parcels of land to
Rosaura Enterprises, Incorporated, a newly-formed and duly registered
corporation in which they assumed majority ownership. The subject parcels
of land were then registered in the name of the corporation. Thereafter,
respondent single-handedly ran the affairs of the corporation in his capacity
as Chairman of the Board, President, General Manager and Treasurer.

Complainant and her daughter made several demands on respondent


for the delivery of the real properties they allegedly assigned to the
corporation, for an accounting of the proceeds of the LBP loan and as well as
the properties sold, and for the rentals earned by BCC. But the demands
remained unheeded. Hence, complainant and her daughter, terminated the
services of respondent as their lawyer and repeated their demands for
accounting and turn-over of the corporate funds, and the return of the 19
titles that respondent transferred to the corporation.

Page 149 of 1072

For his defense, respondent, in his comment and position paper,


denied employing deceit and machination in convincing complainant and her
daughter to assign their real properties to the corporation; that they freely
and voluntary executed the deeds of assignment and the voting trust
agreement that they signed

ISSUE/S:
Can the accused raise the separate personality of the corporation as
a defense?

RULING:
No.
Respondent Attorney Jesus T. Balicanta is disbarred. for commission
of acts of misconduct and disloyalty by taking undue and unfair advantage of
his legal knowledge as a lawyer to gain material benefit for himself at the
expense of complainant Rosaura P. Jaldon-Cordon and caused serious
damage to the complainant.

The fraudulent acts he carried out against his client followed a well
thought of plan to misappropriate the corporate properties and funds
entrusted to him. At the very outset, he embarked on his devious scheme by
making himself the President, Chairman of the Board, Director and Treasurer
of the corporation; although he knew he was prohibited from assuming the
position of President and Treasurer at the same time. As Treasurer, he
accepted in behalf of the corporation the 19 titles that complainant and her
daughter co-owned. The other treasurer appointed, Farnacio Bucoy, did not
appear to be a stockholder or director in the corporate records. The minutes
of the meetings supposedly electing him and Bucoy as officers of the
corporation actually bore the signatures of respondent and the secretary

Page 150 of 1072

only, contrary to his claim that they were signed by the directors and
stockholders.

TOPIC: DOCTRINE OF CORPORATE ENTITY VS. PIERCING THE VEIL OF


CORPORATE FICTION

DELPHER TRADES CORPORATION, and DELPHIN PACHECO


vs.
INTERMEDIATE APPELLATE COURT and HYDRO PIPES PHILIPPINES,
INC.
G.R. No. L-69259
January 26, 1988
157 SCRA 349
FACTS:
Delfin Pacheco and sister Pelagia were the owners of a parcel of land in
Polo (now Valenzuela). On April 3, 1974, they leased to Construction

Page 151 of 1072

Components International Inc. The property and providing for a right of first
refusal should it decide to buy the said property. Construction Components
International, Inc. assigned its rights and obligations under the contract of
lease in favor of Hydro Pipes Philippines, Inc. with the signed conformity and
consent of Delfin and Pelagia. In 1976, a deed of exchange was executed
between lessors Delfin and Pelagia Pacheco and defendant Delpher Trades
Corporation whereby the Pachecos conveyed to the latter the leased
property together with another parcel of land also located in Malinta Estate,
Valenzuela for 2,500 shares of stock of defendant corporation.
On the ground that it was not given the first option to buy the leased
property pursuant to the proviso in the lease agreement, respondent Hydro
Pipes Philippines, Inc., filed an amended complaint for reconveyance of the
lot.
ISSUE/S:
Whether the Deed of Exchange of the properties executed by the
Pachecos and the Delpher Trades Corporation on the other was
meant to be a contract of sale which, in effect, prejudiced the Hydro
Phils right of first refusal over the leased property included in the
deed of exchange
RULING:
No.
By their ownership of the 2,500 no par shares of stock, the Pachecos
have control of the corporation. Their equity capital is 55% as against 45% of
the other stockholders, who also belong to the same family group. In effect,
the Delpher Trades Corporation is a business conduit of the Pachecos. What
they really did was to invest their properties and change the nature of their
ownership from unincorporated to incorporated form by organizing Delpher
Trades Corporation to take control of their properties and at the same time
save on inheritance taxes. The Deed of Exchange of property between the
Pachecos and Delpher Trades Corporation cannot be considered a contract
of sale. There was no transfer of actual ownership interests by the Pachecos
to a third party. The Pacheco family merely changed their ownership from
one form to another. The ownership remained in the same hands. Hence, the
private respondent has no basis for its claim of a light of first refusal under
the lease contract.

Page 152 of 1072

TOPIC:DOCTRINE OF CORPORATE ENTITY VS. PIERCING THE VEIL OF


CORPORATE FICTION
FRANCISCO V. DEL ROSARIO,
vs.
NATIONAL LABOR RELATIONS COMMISSION and LEONARDO V.
ATIENZA
G.R. No. 85416. July 24, 1990
309 S 73
FACTS:

Page 153 of 1072

In POEA Case No. 85-06-0394, the Philippine Overseas Employment


Administration (POEA) promulgated a decision on February 4, 1986
dismissing the complaint for money claims for lack of merit. The decision was
appealed to the National Labor Relations Commission (NLRC), which on April
30, 1987 reversed the POEA decision and ordered Philsa Construction and
Trading Co., Inc. (the recruiter) and Arieb Enterprises (the foreign employer)
to jointly and severally pay private respondent the peso equivalent of
$16,039.00, as salary differentials, and $2,420.03, as vacation leave
benefits. The case was elevated to the Supreme Court, but the petition was
dismissed on August 31, 1987 and entry of judgment was made on
September 24, 1987.
A writ of execution was issued by the POEA but it was returned
unsatisfied as Philsa was no longer operating and was financially incapable of
satisfying the judgment. Private respondent moved for the issuance of an
alias writ against the officers of Philsa. This motion was opposed by the
officers, led by petitioner, the president and general manager of the
corporation.
On February 12, 1988, the POEA issued a resolution, the dispositive
portion of which read:
WHEREFORE, premises considered, let an alias writ of Execution
be issued and the handling sheriff is ordered to execute against
the properties of Mr. Francisco V. del -Rosario and if insufficient,
against the cash and/or surety bond of Bonding Company
concerned for the full satisfaction of the judgment awarded.
Petitioner appealed to the NLRC. On September 23, 1988, the NLRC
dismissed the appeal.
On October 21, 1988, petitioner's motion for reconsideration was
denied.
ISSUE/S:
Whether or not Philsa International Placement and Services
Corp.should be liable.
RULING:
The Court finds grave abuse of discretion on the part of the NLRC.
Under the law a corporation is bestowed juridical personality, separate
and distinct from its stockholders. But when the juridical personality of the
corporation is used to defeat public convenience, justify wrong, protect fraud
or defend crime, the corporation shall be considered as a mere association of
persons, and its responsible officers and/or stockholders shall be held

Page 154 of 1072

individually liable. For the same reasons, a corporation shall be liable for the
obligations of a stockholder, or a corporation and its successor-in-interest
shall be considered as one and the liability of the former shall attach to the
latter.
But for the separate juridical personality of a corporation to be
disregarded, the wrongdoing must be clearly and convincingly established. It
cannot be presumed.
In this regard we find the NLRC's decision wanting. The conclusion that
Philsa allowed its license to expire so as to evade payment of private
respondent's claim is not supported by the facts. Philsa's corporate
personality therefore remains inviolable.
Thus, at the time Philsa allowed its license to lapse in 1985 and even at
the time it was delisted in 1986, there was yet no judgment in favor of
private respondent. An intent to evade payment of his claims cannot
therefore be implied from the expiration of Philsa's license and its delisting.
Neither will the organization of Philsa International Placement and
Services Corp. and its registration with the POEA as a private employment
agency imply fraud since it was organized and registered in 1981, several
years before private respondent filed his complaint with the POEA in 1985.
The creation of the second corporation could not therefore have been in
anticipation of private respondent's money claims and the consequent
adverse judgment against Philsa.
Likewise, substantial identity of the
corporations does not necessarily imply fraud.

incorporators

of

the

two

TOPIC:DOCTRINE OF CORPORATE ENTITY VS. PIERCING THE VEIL OF


CORPORATE FICTION
FIRST PHILIPPINE INTERNATIONAL BANK (Formerly Producers Bank
of the Philippines) and MERCURIO RIVERA,
vs.
COURT OF APPEALS, CARLOS EJERCITO, in substitution of DEMETRIO
DEMETRIA, and JOSE JANOLO
G.R. No. 115849. January 24, 1996

Page 155 of 1072

252 S 259
FACTS:
In the course of its banking operations, the defendant Producer Bank of
the Philippines acquired six parcels of land with a total area of 101 hectares.
The property used to be owned by BYME Investment and Development
Corporation which had them mortgaged with the bank as collateral for a
loan. The original plaintiffs, Demetrio Demetria and Jose O. Janolo, wanted to
purchase the property and thus initiated negotiations for that purpose.
In the early part of August 1987 said plaintiffs, upon the suggestion of
BYME investment's legal counsel, Jose Fajardo, met with defendant Mercurio
Rivera, Manager of the Property Management Department of the defendant
bank. The meeting was held pursuant to plaintiffs' plan to buy the property.
After the meeting, plaintiff Janolo, following the advice of defendant Rivera,
made a formal purchase offer to the bank for (P3, 500,000.00) PESOS, in
cash.
On September 1, 1987, defendant Rivera made on behalf of the bank a
formal reply by letter stating among others that the bank's counter-offer is at
P5.5 million for more than 101 hectares on lot basis.
Plaintiffs thru a letter stating that they would like to amend my
previous offer and I now propose to buy the said lot at P4.250 million in
CASH.
There was no reply to Janolo's foregoing letter of September 17, 1987.
What took place was a meeting on September 28, 1987 between the
plaintiffs and Luis Co, the Senior Vice-President of defendant bank. Rivera as
well as Fajardo, the BYME lawyer, attended the meeting. Two days later, or on
September 30, 1987, plaintiff Janolo sent to the bank, through Rivera, stating
that they are accepting his offer to purchase the property at Sta. Rosa,
Laguna, formerly owned by Byme Investment, for a total price of PESOS: FIVE
MILLION FIVE HUNDRED THOUSAND (P5,500,000.00).
On October 12, 1987, the conservator of the bank was replaced by an
Acting Conservator in the person of defendant Leonida T. Encarnacion. On
November 4, 1987, defendant Rivera wrote plaintiff saying that your proposal
to buy the properties the bank foreclosed from Byme investment Corp.
located at Sta. Rosa, Laguna is under study yet as of this time by the newly
created committee for submission to the newly designated Acting
Conservator of the bank.
What thereafter transpired was a series of demands by the plaintiffs for
compliance by the bank with what plaintiff considered as a perfected
contract of sale

Page 156 of 1072

On May 16, 1988, plaintiffs filed a suit for specific performance with
damages against the bank, its Manager Rivers and Acting Conservator
Encarnacion.

ISSUE/S:
Did the bank conservator have the unilateral power to repudiate the
authority of the bank officers and/or to revoke the said contract?
RULING:
It is not disputed that the petitioner Bank was under a conservator
placed by the Central Bank of the Philippines during the time that the
negotiation and perfection of the contract of sale took place.
The issue of the Conservator's alleged authority to revoke or repudiate
the perfected contract of sale was raised for the first time in this Petition
as this was not litigated in the trial court or Court of Appeals. As already
stated earlier, issues not raised and/or ventilated in the trial court, let alone
in the Court of Appeals, "cannot be raised for the first time on appeal as it
would be offensive to the basic rules of fair play, justice and due process."
In the second place, there is absolutely no evidence that the
Conservator, at the time the contract was perfected, actually repudiated or
overruled said contract of sale. The Bank's acting conservator at the time,
Rodolfo Romey, never objected to the sale of the property to Demetria and
Janolo. What petitioners are really referring to is the letter of Conservator
Encarnacion, who took over from Romey after the sale was perfected on
September 30, 1987 which unilaterally repudiated not the contract but
the authority of Rivera to make a binding offer and which unarguably
came months after the perfection of the contract. Said letter dated May 12,
1988 is reproduced hereunder:
In the third place, while admittedly, the Central Bank law gives
vast and far-reaching powers to the conservator of a bank, it
must be pointed out that such powers must be related to the
"preservation of the assets of the bank, the reorganization of the
management thereof and the restoration of its viability." Such
powers, enormous and extensive as they are, cannot extend to
the post-facto repudiation of perfected transactions, otherwise
they would infringe against the non-impairment clause of the
Constitution.
Hence, the conservator merely takes the place of a bank's board of
directors. What the said board cannot do such as repudiating a contract
validly entered into under the doctrine of implied authority the
conservator cannot do either. Ineluctably, his power is not unilateral and he

Page 157 of 1072

cannot simply repudiate valid obligations of the Bank. His authority would be
only to bring court actions to assail such contracts as he has already done
so in the instant case. A contrary understanding of the law would simply not
be permitted by the Constitution. Neither by common sense. To rule
otherwise would be to enable a failing bank to become solvent, at the
expense of third parties, by simply getting the conservator to unilaterally
revoke all previous dealings which had one way or another or come to be
considered unfavorable to the Bank, yielding nothing to perfected
contractual rights nor vested interests of the third parties who had dealt with
the Bank.

TOPIC:DOCTRINE OF CORPORATE ENTITY VS. PIERCING THE VEIL OF


CORPORATE FICTION
FRANCISCO MOTORS CORPORATION
vs.
COURT OF APPEALS and SPOUSES GREGORIO and LIBRADA MANUEL
G.R. No. 100812. June 25, 1999

Page 158 of 1072

FACTS:
On January 23, 1985, petitioner filed a complaintagainst private
respondents to recover three thousand four hundred twelve and six centavos
(P3,412.06), representing the balance of the jeep body purchased by the
Manuels from petitioner; an additional sum of twenty thousand four hundred
fifty-four and eighty centavos (P20,454.80) representing the unpaid balance
on the cost of repair of the vehicle; and six thousand pesos (P6,000.00) for
cost of suit and attorney's fees.To the original balance on the price of jeep
body were added the costs of repair.In their answer, private respondents
interposed a counterclaim for unpaid legal services by Gregorio Manuel in
the amount of fifty thousand pesos (P50,000) which was not paid by the
incorporators, directors and officers of the petitioner.
The trial court decided the case on June 26, 1985, in favor of petitioner
in regard to the petitioner's claim for money, but also allowed the counterclaim of private respondents. Both parties appealed. On April 15, 1991, the
Court of Appeals sustained the trial court's decision.
ISSUE/S:
Whether or not the Court of Appeals erred in applying the Doctrine
of Piercing the veil of Corporate Entity
RULING:
Basic in corporation law is the principle that a corporation has a
separate personality distinct from its stockholders and from other
corporations to which it may be connected.
In our view, however, given the facts and circumstances of this case,
the doctrine of piercing the corporate veil has no relevant application here.
Respondent court erred in permitting the trial court's resort to this doctrine.
The rationale behind piercing a corporation's identity in a given case is to
remove the barrier between the corporation from the persons comprising it
to thwart the fraudulent and illegal schemes of those who use the corporate
personality as a shield for undertaking certain proscribed activities. However,
in the case at bar, instead of holding certain individuals or persons
responsible for an alleged corporate act, the situation has been reversed. It
is the petitioner as a corporation which is being ordered to answer for the
personal liability of certain individual directors, officers and incorporators
concerned. Hence, it appears to us that the doctrine has been turned upside
down because of its erroneous invocation.
Furthermore, considering the nature of the legal services involved,
whatever obligation said incorporators, directors and officers of the

Page 159 of 1072

corporation had incurred, it was incurred in their personal capacity. When


directors and officers of a corporation are unable to compensate a party for a
personal obligation, it is far-fetched to allege that the corporation is
perpetuating fraud or promoting injustice, and be thereby held liable therefor
by piercing its corporate veil. While there are no hard and fast rules on
disregarding separate corporate identity, we must always be mindful of its
function and purpose. A court should be careful in assessing the milieu where
the doctrine of piercing the corporate veil may be applied. Otherwise an
injustice, although unintended, may result from its erroneous application.
The personality of the corporation and those of its incorporators,
directors and officers in their personal capacities ought to be kept separate
in this case. The claim for legal fees against the concerned individual
incorporators, officers and directors could not be properly directed against
the corporation without violating basic principles governing corporations.
Moreover, every action including a counterclaim must be prosecuted or
defended in the name of the real party in interest. It is plainly an error to lay
the claim for legal fees of private respondent Gregorio Manuel at the door of
petitioner (FMC) rather than individual members of the Francisco family.

TOPIC:DOCTRINE OF CORPORATE ENTITY VS. PIERCING THE VEIL OF


CORPORATE FICTION
SOL LAGUIO, RENE LAOLAO, ANNALIZA ENSANDO, EDELIZA ASAS,
LILIA MARAY, EVELYN UNTALAN,* ROSARIO CHICO, REYNALDO
GARCIA, MERLITA DE LOS SANTOS,* JOSEPHINE DERONG,* GEMMA
TIBALAO BANTOLO, LUCY ALMONTE,* CRISPINA VANQUARDIA,
NARCISA VENZON, NORMA ELEGANTE,* AMELITA MORENO,* ABNER

Page 160 of 1072

PETILOS, NARCISO HILAPO, DOLORES OLAES, MELINDA LLADOC,


ERNA AZARCON, and APRIL TOY, INC. WORKERS UNION ALAB ,
vs.
NATIONAL LABOR RELATIONS COMMISSION, WELL WORLD TOYS,
INC., APRIL TOYS, INC., YU SHENG LING, JENN L. WANG, EUCLIFF
CHENG, CHI SHENG LIN, NENITA C. AGUIRRE, MA. THERESA R.
CADIENTE and GLICERIA R. AGUIRRE
G.R. No. 108936. October 4, 1996
262 SCRA 709
FACTS:
Private respondent April Toy, Inc. is a domestic corporation
incorporated on January 6, 1989, for the purpose of "manufacturing,
importing, exporting, buying , selling, sub-contracting or otherwise dealing
in, at wholesale and retail," stuffed toys, with principal place of business at
Paraaque, Manila. On December 20, 1989, or after almost a year of
operation, April posted a memorandum within its premises and circulated a
copy of the same among its employees informing them of its dire financial
condition. To avert further business reverses, April decided to shorten its
corporate term "up to February 28, 1990," submitted a notice of dissolution
to the Securities and Exchange Commission and published the same in a
newspaper of general circulation. April also notified its employees, the
Department of Labor and Employment, the Social Security System, the Board
of Investments, the Bureau of Internal Revenue, and the Municipality of
Paraaque of its dissolution.
In view of April's cessation of operations, petitioners who initially
composed of seventy-seven employees below filed a complaint for "illegal
shutdown/retrenchment/dismissal and unfair labor practice." On June 21,
1990, petitioners amended their complaint to implead private respondent
Well World Toys, Inc. (Well World for brevity), a corporation also engaged in
the manufacture of stuffed toys for export with principal office located at Las
Pias, Manila.
To bolster their claim that April and Well World are one and the same
corporation, petitioners argue that both corporations have the same set of
incorporators.
ISSUE/S:
Whether or not April Toy and Well-World Toy are one and the same
RULING:
We can not fully subscribe to the above contention of the
complainants. We do not believe that the circumstances related by the
complainants are sufficient indicia that the two corporations are one and the
same corporation although it appears that two of the original incorporators
and stockholders of April Toy, Inc. were incorporators and minority

Page 161 of 1072

stockholders of Well-World Toy, Inc. Hence it does not mean that the two (2)
corporations are adjunct and conduit. There is not express provision under
the Corporation Law prohibiting stockholders or incorporators of a
corporation to be a stockholder or incorporator of another corporation.
The fiction that a corporation was a distinct and separate personality
shall not be used as a subterfuge to commit injustice and circumvent the law
does not apply in the present case. There is no conclusive evidence to
convince us that respondent April Toy, Inc. was established and later on
closed to defeat the rights of the workers of Well-World Toy, Inc. which would
otherwise support the charge of unfair labor practice. Hence, we find that the
two (2) corporations are separate and distinct entities.
Anent the question of whether or not April Toy and Well-World Toy are
one and the same, with the facts and circumstances showing that the owners
of April Toy are different from those of Well-World, the management of one
being different from the other, and the office of April Toy is situated more
than ten kilometers away from Well-World, plus the fact that the closure of
April Toy was for valid reasons, the Labor Arbiter likewise correctly opined
that the two corporations are separate and distinct from each other, and that
there is no basis for piercing the veil of corporate fiction.

TOPIC:DOCTRINE OF CORPORATE ENTITY VS. PIERCING THE VEIL OF


CORPORATE FICTION
RUFINA LUY LIM
vs.

Page 162 of 1072

COURT OF APPEALS, AUTO TRUCK TBA CORPORATION, SPEED


DISTRIBUTING, INC., ACTIVE DISTRIBUTORS, ALLIANCE MARKETING
CORPORATION, ACTION COMPANY, INC.
G.R. No. 124715.January 24, 2000
323 SCRA 102
FACTS:
Private respondents Auto Truck Corporation, Alliance Marketing
Corporation, Speed Distributing, Inc., Active Distributing, Inc. and Action
Company are corporations formed, organized and existing under Philippine
laws and which owned real properties covered under the Torrens system.
On 11 June 1994, Pastor Y. Lim died intestate. Herein petitioner, as
surviving spouse and duly represented by her nephew George Luy, fried on
17 March 1995, a joint petition for the administration of the estate of Pastor
Y. Lim before the Regional Trial Court of Quezon City.
Private respondent corporations, whose properties were included in the
inventory of the estate of Pastor Y. Lim, then filed a motionfor the lifting of
lispendens and motionfor exclusion of certain properties from the estate of
the decedent.
In an orderdated 08 June 1995, the Regional Trial Court of Quezon City,
Branch 93, sitting as a probate court, granted the private respondents' twin
motions.
On 04 July 1995, the Regional Trial Court acting on petitioner's motion
issued an orderreinstating the annotation of lispendens.
On 04 September 1995, the probate court appointed Rufina Lim as
special administrator and Miguel Lim and Lawyer Donald Lee, as co-special
administrators of the estate of Pastor Y. Lim, after which letters of
administration were accordingly issued.
In an order12 dated 12 September 1995, the probate court denied anew
private respondents' motion for exclusion.
ISSUE/S:
Whether the corporations are
instrumentalities of Pastor Lim

the

mere

alter

egos

or

RULING:
It is settled that a corporation is clothed with personality separate and
distinct from that of the persons composing it. It may not generally be held
liable for that of the persons composing it. It may not be held liable for the

Page 163 of 1072

personal indebtedness of its stockholders or those of the entities connected


with it.
Rudimentary is the rule that a corporation is invested by law with a
personality distinct and separate from its stockholders or members. In the
same vein, a corporation by legal fiction and convenience is an entity
shielded by a protective mantle and imbued by law with a character alien to
the persons comprising it.
Nonetheless, the shield is not at all times invincible.
Piercing the veil of corporate entity requires the court to see through
the protective shroud which exempts its stockholders from liabilities that
ordinarily, they could be subject to, or distinguishes one corporation from a
seemingly separate one, were it not for the existing corporate fiction.
The corporate mask may be lifted and the corporate veil may be
pierced when a corporation is just but the alter ego of a person or of another
corporation. Where badges of fraud exist, where public convenience is
defeated; where a wrong is sought to be justified thereby, the corporate
fiction or the notion of legal entity should come to naught.
Mere ownership by a single stockholder or by another corporation of all or
nearly all of the capital stock of a corporation is not of itself a sufficient
reason for disregarding the fiction of separate corporate personalities.
Moreover, to disregard the separate juridical personality of a
corporation, the wrong-doing must be clearly and convincingly established. It
cannot be presumed.
Granting arguendo that the Regional Trial Court in this case was not merely
acting in a limited capacity as a probate court, petitioner nonetheless failed
to adduce competent evidence that would have justified the court to impale
the veil of corporate fiction.

TOPIC: DOCTRINE OF CORPORATE ENTITY VS. PIERCING THE VEIL OF


CORPORATE FICTION
MALAYANG SAMAHAN NG MGA MANGGAGAWA SA M. GREENFIELD
VS.
HON. CRESENCIO J. RAMOS

Page 164 of 1072

G.R. No. 113907. February 28, 2000


FACTS:
Herein petitioner (local Union) is an affiliate of the private respondent
United Lumber and General Workers of the Philippines. On September 12,
1986, a local union election was held under the auspices of the ULGWP
where the petitioner Beda Magdalena Villanueva and the other union officers
were proclaimed as winners. However, a petition for impeachment was filed
by the defeated candidates.
In lieu of this, the local union held a general membership meeting
however, several of its members was unable to attend the same prompting
the Executive Board to conduct an investigation regarding such absence.
After due investigation, through a memorandum, the national federation
approved the imposition of a 50.00 fine. This imposition of the fine became
the subject of a bitter disagreement between the local union and the
federation, culminating in the formers declaration of general autonomy from
the latter.
The Officials of ULGWP, called a special Board meeting where they
came up with a resolution to place MSMG under trusteeship and appointing
Cesar Clarete as administrator. Thereafter, said administrator designated one
Alfredo Kalingking as local union president and disauthorizing the incumbent
officers from representing the employees. Nevertheless, the union officers
were expelled from the ULGWP. Upon the other hand, the corporation, under
pressure of a threatened strike terminated the 30 union officers from
employment.
ISSUE/S:
Whether or not the complainants have recourse against the
federation and not against the corporation and its officers.
RULING:
The Supreme Court ruled that said contention that the issue of
expulsion of the union officers is purely and intra-union matter is untenable.
While it is true that the issue of expulsion is originally between the
local union and the federation, the issue was later converted into a
termination dispute when the company dismissed the petitioners from work
without the benefit of a separate notice and hearing. Notwithstanding the
fact that the dismissal was at the instance of the federation and that it
undertook to hold the company free from any liability resulting from such
dismissal, the company may still be held liable if it was remiss from its duty
to accord the would be dismissed employees their right to be heard on the
matter.

Page 165 of 1072

It is likewise the opinion of the Court and so holds, the respondent


company officials cannot be held personally liable for damages on account of
the employees dismissal because the employer corporation has a
personality separate and distinct from its officer who merely acted as its
agents.

TOPIC: DOCTRINE OF CORPORATE ENTITY VS. PIERCING THE VEIL OF


CORPORATE FICTION
MATAGUINA INTEGRATED WOOD PRODUCTS,INC.
VS.
COURT OF APPEALS

Page 166 of 1072

263 SCRA 490

OCTOBER 24, 1996

FACTS:
On June 28, 1973, the Acting Director of the Bureau of Forest
Development issued PTL No. 30 covering an area of 5,400 hectares to Mrs.
Milagros Mataguina who was doing business under the name MLE which is a
sole proprietorship venture. Thereafter, petitioner MIWPI was incorporated
having an authorized capital stock of ten million pesos. Milagros Mataguina
became the majority stockholder when the board approved the transfer of
the stocks held by Henry Wee to the latter.
In a letter addressed to the Director of BFD, Matguina requested for a
change of name and transfer of management of PTL No. 30 from a single
proprietorship under her name to that of MIWPI. Mataguina and MIWPI then
executed a deed of transfer involving all the rights and interest of Mataguina
over PTL No. 30 in consideration of 148,000 shares of stocks in MIWPI.
Pending approval of the request, DAVENCOR complained to the District
Forester that Mataguina encroached on their concession area. During the
pendency of the case, Mataguina disposed her shares in MIWPI, thereby
ceasing to be a shareholder of the petitioner. When the decision of the
Minister of Natural Resources became final an executory, it directed the
issuance of a writ of execution not only against MLE but also against MIWPI.
Thus, the filing of the instant complaint for prohibition, damages and
injunction, with the RTC of Davao.
The trial court ruled in favor of MIWPI but upon appeal, the appellate
court reversed said decision, hence this petition.
ISSUE/S:
Whether or not it is possible to pierce MIWPI veil of corporate
existence, thus making it a mere conduit of MLE.
RULING:
Generally accepted is the principle that no man shall be affected by
any proceeding which he is a stranger, and strangers to the case are not
bound by the judgment rendered by the Court. In the same manner, an
execution can be issued only against a party and not against one who did not
have his day in court. Thus, the court found that there is no basis for the
issuance of the order of execution against the petitioner. The same was
issued without giving the petitioner an opportunity to defend itself and
oppose the request of DAVENCOR for the issuance of the writ against it. It
does not appear that petitioner was at all furnished with a copy of

Page 167 of 1072

DAVENCORs letter requesting for the execution of the Secretarys decision


against it.
The Court also ruled that the evidence presented at trial does not
warrant the piercing the veil of MIWPIs corporate existence. It is settled that
a corporation is clothed with a personality separate and distinct from the
persons composing it. It may not generally be held liable for that of the
persons composing it. It may not be held liable for the personal indebtedness
of its stockholders. But when the juridical personality of the corporation is
used to defeat public convenience, justify wrong protect fraud or defend
crime, the corporation shall be considered as a mere association of persons
and its responsible officers and/or stockholders shall be individually liable.
The Court reiterated the observation of the appellate court to in stating that
the alleged control of Plaintiff Corporation was not evident in any particular
corporate acts of plaintiff corporation where Milagros Mataguina Logging
Enterprises using plaintiff corporation, executed acts or power directly
involving plaintiff. Neither was there evidence of defendants that MLE, using
the facilities and resources of MIWPI, involved itself on transaction using both
single proprietorship and plaintiff corporation in such particular line of
business undertaking.

TOPIC: DOCTRINE OF CORPORATE ENTITY VS. PIERCING THE VEIL OF


CORPORATE FICTION

Page 168 of 1072

THE MANILA HOTEL CORP. AND MANILA HOTEL INTL. LTD.,


petitioners,
vs.
NATIONAL LABOR RELATIONS COMMISSION, ARBITER CEFERINA J.
DIOSANA AND MARCELO G. SANTOS, respondents.
GR NO. 120077 OCTOBER 13, 2000
FACTS:
Herein respondent was an overseas worker employed as a printer at
the Mazoon Printing Press, Sultanate of Oman. Subsequently he was directly
hired by the Palace Hotel, Beijing and later terminated due to retrenchment.
MHC at the time of the filing of the case was still a government owned and
controlled corporation. MHICL is a corporation duly organized and existing
under the laws of Hong Kong. MHC is an incorporator of MHICL owning 50%
of its capital stock. By virtue of a management agreement, MHICL trained the
personnel and staff of the Palace Hotel in Beijing.
As said, respondent Santos was hired by Palace Hotel as printer but
with higher monthly salary and increased benefits. The contract with Palace
Hotel was for a period of two years. A year after, the executive secretary
suggested in a handwritten note that Santos be given one month notice of
his release from employment. A month after, Palace Hotel informed Santos
that his employment would be terminated die to business reverses brought
about by the political upheaval in China. Upon his return in the Philippines,
respondent filed a complaint for illegal dismissal with the NLRC. The
complaint named MHC and MHICL, Palace Hotel and Mr. Shmidt as
respondents.
ISSUE/S:
Whether or not MHC is liable for Santos retrenchment.
RULING:
Even if we assume two things:
(1) that the NLRC had jurisdiction over the case, and
(2) that MHICL was liable for Santos' retrenchment, still MHC, as
a separate and distinct juridical entity cannot be held liable.
True, MHC is an incorporator of MHICL and owns fifty percent (50%) of
its capital stock. However, this is not enough to pierce the veil of corporate
fiction between MHICL and MHC.
Piercing the veil of corporate entity is an equitable remedy. It is
resorted to when the corporate fiction is used to defeat public convenience,
justify wrong, protect fraud or defend a crime. 41 It is done only when a
corporation is a mere alter ego or business conduit of a person or another
corporation.

Page 169 of 1072

In Traders Royal Bank v. Court of Appeals, we held that "the mere


ownership by a single stockholder or by another corporation of all or nearly
all of the capital stock of a corporation is not of itself a sufficient reason for
disregarding the fiction of separate corporate personalities."
The tests in determining whether the corporate veil may be pierced
are:
1. the defendant must have control or complete domination
of the other corporation's finances, policy and business
practices with regard to the transaction attacked. There
must be proof that the other corporation had no separate
mind, will or existence with respect the act complained of.
2. control must be used by the defendant to commit fraud
or wrong.
3. the aforesaid control or breach of duty must be the
proximate cause of the injury or loss complained of. The
absence of any of the elements prevents the piercing of
the corporate veil.
It is basic that a corporation has a personality separate and distinct
from those composing it as well as from that of any other legal entity to
which it may be related. Clear and convincing evidence is needed to pierce
the veil of corporate fiction. In this case, we find no evidence to show that
MHICL and MHC are one and the same entity.

Page 170 of 1072

TOPIC: DOCTRINE OF CORPORATE ENTITY VS. PIERCING THE VEIL OF


CORPORATE FICTION
COMMISSIONER OF INTERNAL REVENUE, petitioner,
vs.
NORTON and HARRISON COMPANY, respondent
G.R. No. L-17618, August 31, 1964
FACTS:
Norton and Harrison is a corporation organized in 1911,
1. to buy and sell at wholesale and retail, all kinds of goods,
wares, and merchandise;
2. to act as agents of manufacturers in the United States and
foreign countries; and
3. to carry on and conduct a general wholesale and retail
mercantile establishment in the Philippines.
Jackbilt is, likewise, a corporation organized on February 16, 1948
primarily for the purpose of making, producing and manufacturing concrete
blocks.
On July 27, 1948, Norton and Jackbilt entered into an agreement
whereby Norton was made the sole and exclusive distributor of concrete
blocks manufactured by Jackbilt. Thus, whenever an order for concrete
blocks was received by the Norton from a customer, the order was
transmitted to Jackbilt which delivered the merchandise direct to the
customer. Payment for the good is, however, made to Norton, which in turn
pays Jackbilt the amount charged the customer less a certain amount, as its
compensation or profit.
On May 1, 1953 the agency agreement was terminated and a
management agreement between the parties was entered into wherein it
provided that Norton would sell concrete blocks for Jackbilt for a fixed
monthly management fee of P2,000.00, which was later increased to
P5,000.00. During the existence of the distribution or agency agreement, or
on June 10, 1949, Norton & Harrison acquired by purchase all the
outstanding shares of stock of Jackbilt.
The CIR considered the sale of Norton to the public as the original sale
and not the transaction from Jackbilt.
The CIR contended and as
subsequently ruled by the CTA that since Jackbilt was owned and controlled
by Norton, the corporate personality of the former (Jackbilt) should be
disregarded for sales tax purposes, and the sale of Jackbilt blocks by
petitioner to the public must be considered as the original sales from which
the sales tax should be computed.

Page 171 of 1072

ISSUE/S:
Whether the acquisition of all the stocks of the Jackbilt by the
Norton & Harrison Co., merged the two corporations into a single
corporation
RULING:
The court found sufficient ground to support the theory that the
separate identities of the two companies should be disregarded. Among
these circumstances, which were successfully refuted by appellee Norton
are:
a. Norton and Harrison owned all the outstanding stocks of the
Jackbilt; of the 15,000 authorized shares of Jackbilt on March
31, 1958, 14, 998 shares belonged to Norton and Harrison and
one each to seven others,
b. Norton constituted Jackbilt's board of directors in such a way
as to enable it to actually direct and manage the other's
affairs by making the same officers of the board for both
companies
c. Norton financed the operations of the Jackbilt,
d. Norton treats Jackbilt employees as its own. Evidence show
that Norton paid the salaries of Jackbilt employees and gave
the same privileges as Norton employees
e. Compensation given to board members of Jackbilt, who are
also board members and/or employees of Norton, indicate
that Jackbilt is merely a department of Norton.
All these lead to the conclusion that the Jackbilt is merely an adjunct,
business conduit or alter ego, of Norton and that the fiction of corporate
entities, separate and distinct from each, should be disregarded. The
corporate fiction raised by petitioner was only used to avoid payment of
taxes.

Page 172 of 1072

TOPIC: DOCTRINE OF CORPORATE ENTITY VS. PIERCING THE VEIL OF


CORPORATE FICTION
BIBIANO O. REYNOSO IV
VS.
HON. COURT OF APPEALS AND GENERAL CREDIT CORPORATION
GR NOS. 116124-35 NOVEMBER 22, 2000
FACTS:
Commercial Credit Corporation a financing investment firm decided to
organize franchise companies in different parts of the country wherein it shall
hold 30% equity. Herein petitioner was designated as the resident manager
of the franchise company in Quezon City known as CCC-QC.
CCC-Qc thereafter entered into an exclusive management contract
with CCC whereby the latter was granted the management and full control of
the business activities of the former. But pursuant to the DOSRI Rule,
prohibiting lending of funds by corporations to its directors, CCC decided to
form CCC Equity Corporation a wholly owned subsidiary, to which CCC
transferred its 305 equity in CCC-QC. Under this set up, several Officials of
CCC including Reynoso, became employees of CCC-Equity. Thus, petitioner in
order to boost the business activities of CCC-QC deposited his personal funds
in the company and in turn, the CCC-QC issued him its interest bearing
promissory notes.
Thereafter, a complaint for collection of sum of money with preliminary
attachment was instituted by CCC-QC against petitioner, who had in the
meantime been dismissed from his employment. The Complaint alleged that
petitioner embezzled the funds. Out of the funds missing, at least 630,000
were used for the purchase of a house. In the meantime, CCC became known
as General Credit Corporation. It then filed a special appearance and
opposition alleging that it was not a party to the case and therefore
petitioner must direct its claim against CCC-QC.
ISSUE/S:
Whether or not General Credit Corporation is a stranger to the civil
case and thus judgment cannot be enforced against it.
RULING:
As established, a corporation is an artificial being invested by law with
a personality separate and distinct from those of the persons composing it as
well as from that of any other legal entity to which it may be related. It has

Page 173 of 1072

the advantage of non-dependence on the lives of those who compose it even


as it enjoys certain rights and conducts activities of natural persons.
Any piercing of the corporate veil has to be done with caution. But
where the corporate fiction is sued as an unfair device to achieve an
inequitable result, defraud creditors, evade contracts and obligations, or to
shield it from the effects of a court decision, the Court will not hesitate to
apply its supervisory and adjudicative matters. The doctrine of the piercing
the corporate fiction applies only when such fiction is used to defeat public
convenience, justify wrong, protect fraud or defend crime.

Page 174 of 1072

TOPIC:DOCTRINE OF CORPORATE ENTITY VS. PIERCING THE VEIL OF


CORPORATE FICTION
SAN JUAN STRUCTURAL AND STEEL FABRICATORS, INC.,
petitioner,vs.COURT OF APPEALS, MOTORICH SALES CORPORATION,
NENITA LEE GRUENBERG, ACL DEVELOPMENT CORP. and JNM REALTY
AND DEVELOPMENT CORP., respondents.
G.R. No. 129459. September 29, 1998
FACTS:
On 14 February 1989, plaintiff-appellant entered into an agreement
with defendant-appellee Motorich Sales Corporation for the transfer to it of a
parcel of land identified as Lot 30, Block 1 of the Acropolis Greens
Subdivision located in the District of Murphy, Quezon City Metro Manila,
containing an area of 414 square meters, covered by TCT No. (362909) 2876,
that as stipulated in the Agreement of 14 February 1989, plaintiff-appellant
paid the down payment in the sum of P100,000.00, the balance to be paid on
or before March 2, 1989.
That defendant-appellee Motorich Sales Corporation despite repeated
demands and in utter disregard of its commitments had refused to execute
the Transfer of Rights/Deed of Assignment which is necessary to transfer the
certificate of title. Defendant ACL Development Corp. is impleaded as a
necessary party since TCT No. (362909) 2876 is still in the name of said
defendant; while defendant JNM Realty & Development Corp. is likewise
impleaded as a necessary party in view of the fact that it is the transferor of
right in favor of defendant-appellee Motorich Sales Corporation. That on April
6, 1989, defendant ACL Development Corporation and Motorich Sales
Corporation entered into a Deed of Absolute Sale whereby the former
transferred to the latter the subject property.
ISSUE/S:
Whether or not the doctrine of piercing the veil of corporate fiction
is applicable in the instant case.
RULING:
Thus, the Court has consistently ruled that when the fiction is used as
a means of perpetrating a fraud or an illegal act or as vehicle for the evasion
of an existing obligation, the circumvention of statutes, the achievement or
perfection of a monopoly or generally the perpetration of knavery or crime,
the veil with which the law covers and isolates the corporation from the
members or stockholders who compose it will be lifted to allow for its
consideration merely as an aggregation of individuals.

Page 175 of 1072

We stress that the corporate fiction should be set aside when it


becomes a shield against liability for fraud, illegality or inequity committed
on third persons. The question of piercing the veil of corporate fiction is
essentially, then, a matter of proof. In the present case, however, the Court
finds no reason to pierce the corporate veil of Respondent Motorich.
Petitioner utterly failed to establish that said corporation was formed, or that
it is operated, for the purpose of shielding any alleged fraudulent or illegal
activities of its officers or stockholders; or that the said veil was used to
conceal fraud, illegality or inequity at the expense of third persons like
petitioner.

Page 176 of 1072

TOPIC:DOCTRINE OF CORPORATE ENTITY VS. PIERCING THE VEIL OF


CORPORATE FICTION
BENJAMIN A. SANTOS, petitioner,
vs.
NATIONAL LABOR RELATIONS COMMISSION, HON. LABOR ARBITER
FRUCTUOSO T. AURELLANO and MELVIN D. MILLENA, respondents.
G.R. No. 101699. March 13, 1996
FACTS:
Private respondent, on 01 October 1985, was hired to be the project
accountant for MMDC's mining operations in Gatbo, Bacon, Sorsogon. On 12
August 1986, private respondent sent to Mr. Gil Abao, the MMDC corporate
treasurer, a memorandum calling the latter's attention to the failure of the
company to comply with the withholding tax requirements of, and to make
the corresponding monthly remittances to, the Bureau of Internal Revenue
("BIR") on account of delayed payments of accrued salaries to the company's
laborers and employees. Private respondent expressed "shock" over the
termination of his employment. He complained that he would not have
resigned from the Sycip, Gorres & Velayo accounting firm, where he was
already a senior staff auditor, had it not been for the assurance of a
"continuous job" by MMDC's Engr. Rodillano E. Velasquez. Private respondent
requested that he be reimbursed the "advances" he had made for the
company and be paid his "accrued salaries/claims.
The claim was not heeded, on 20 October 1986, private respondent
filed with the NLRC a complaint for illegal dismissal. On 27 July 1988, Labor
Arbiter, finding no valid cause for terminating complainant's employment. In
a resolution, dated 04 September 1989, the NLRC affirmed the decision of
the Labor Arbiter. On 16 August 1991, the NLRC dismissed the motion for
reconsideration. In holding petitioner personally liable for private
respondent's claim, the responsible officer of an employer corporation
(could) be held personally, not to say even criminally, liable for non-payment
of back wages," and that where the employer corporation was no longer
existing and unable to satisfy the judgment in favor of the employee, the
officer should be liable for acting on behalf of the corporation.
ISSUE/S:
Whether or not the NLRC have gravely abused their discretion "in
finding petitioner solidarily liable with MMDC even in the absence of
bad faith and malice on his part."

Page 177 of 1072

RULING:
A corporation is a juridical entity with legal personality separate and
distinct from those acting for and in its behalf and, in general, from the
people comprising it. The rule is that obligations incurred by the corporation,
acting through its directors, officers and employees, are its sole liabilities.
Nevertheless, being a mere fiction of law, peculiar situations or valid grounds
can exist to warrant, albeit done sparingly, the disregard of its independent
being and the lifting of the corporate veil.
As a rule, this situation might arise when a corporation is used to
evade a just and due obligation or to justify a wrong, to shield or perpetrate
fraud, to carry out similar other unjustifiable aims or intentions, or as a
subterfuge to commit injustice and so circumvent the law. It is well-settled
instances when, without necessarily piercing the veil of corporate fiction,
personal civil liability can also be said to lawfully attach to a corporate
director, trustee or officer; to wit: When
1. He assents
a. to a patently unlawful act of the corporation, or
b. for bad faith or gross negligence in directing its affairs,
or
c. for conflict of interest, resulting in damages to the
corporation, its stockholders or other persons;
2. He consents to the issuance of watered stocks or who, having
knowledge thereof, does not forthwith file with the corporate
secretary his written objection thereto;
3. He agrees to hold himself personally and solidarily liable with
the corporation; or
4. He is made, by a specific provision of law, to personally
answer for his corporate action.
The case of petitioner is way off these exceptional instances. It is not
even shown that petitioner has had a direct hand in the dismissal of private
respondent enough to attribute to him a patently unlawful act while acting
for the corporation. Neither can Article 289 of the Labor Code be applied
since this law specifically refers only to the imposition of penalties under the
Code. It is undisputed that the termination of petitioner's employment has,
instead, been due, collectively, to the need for a further mitigation of losses,
the onset of the rainy season, the insurgency problem in Sorsogon and the
lack of funds to further support the mining operation in Gatbo.

Page 178 of 1072

TOPIC:DOCTRINE OF CORPORATE ENTITY VS. PIERCING THE VEIL OF


CORPORATE FICTION
TAN BOON BEE & CO., INC.
vs.
THE HONORABLE HILARION U. JARENCIO, PRESIDING JUDGE OF
BRANCH XVIII of the Court of First Instance of Manila, GRAPHIC
PUBLISHING, INC., and PHILIPPINE AMERICAN CAN DRUG COMPANY;
G.R. No. L-41337; June 30, 1988
FACTS:
Petitioner herein, doing business under the name and style of Anchor
Supply Co., sold on credit to herein private respondent Graphic Publishing,
Inc. (GRAPHIC ) paper products as evidenced by a promissory note. When
Graphic failed to fulfill its obligation, petitioner filed a collection suit. After
trial, the court ruled in favor of petitioner, thus, issuing a writ of execution
over a printing machine which was levied by the sheriff.
However, respondent Philippine American Drug Company (PADCO) had
informed the sheriff that the printing machine is its property and not that of
GRAPHIC but the sheriff proceeded with the auction sale. Thus, respondent
judge issued an order declaring the sale to be null and void and ordered the
return of the machine to PADCO.
ISSUE/S:
Whether or not respondent judge committed grave abuse of
discretion in not piercing the veil of corporate fiction
RULING:
It is true that a corporation, upon coming into being, is invested by law
with a personality separate and distinct from that of the persons composing
it as well as from any other legal entity to which it may be related. However,
such separate personality of the corporation may be disregarded, or the veil
of corporate fiction pierced, in cases where it is used as a cloak or cover for
fraud or illegality, or to work an injustice, or where necessary to achieve
equity or when necessary for the protection of creditors. Corporations are
composed of natural persons and the legal fiction of a separate corporate
personality is not a shield for the commission of injustice and inequity.
Likewise, this is true when the corporation is merely an adjunct, business
conduit or alter ego of another corporation. In such case, the fiction of
separate and distinct corporation entities should be disregarded.
In the instant case, petitioner's evidence established that PADCO was
never engaged in the printing business; that the board of directors and the
officers of GRAPHIC and PADCO were the same; and that PADCO holds 50%
share of stock of GRAPHIC. Petitioner likewise stressed that PADCO's own

Page 179 of 1072

evidence shows that the printing machine in question had been in the
premises of GRAPHIC since May, 1965, long before PADCO even acquired its
alleged title on July 11, 1966 from Capitol Publishing. That the said machine
was allegedly leased by PADCO to GRAPHIC on January 24, 1966, even before
PADCO purchased it from Capital Publishing on July 11, 1966, only serves to
show that PADCO's claim of ownership over the printing machine is not only
farce and sham but also unbelievable.
Considering the aforestated principles and the circumstances
established in this case, respondent judge should have pierced PADCO's veil
of corporate Identity.

TOPIC:DOCTRINE OF CORPORATE ENTITY VS. PIERCING THE VEIL OF


CORPORATE FICTION

Page 180 of 1072

TRADERS ROYAL BANK, petitioner,


vs.
COURT OF APPEALS, FILRITERS GUARANTY ASSURANCE
CORPORATION and CENTRAL BANK of the PHILIPPINES, respondents.
G.R. No. 93397. March 3, 1997
FACTS:
Filriters is the registered owner of CBCI No. D891. Under a deed of
assignment Filriters transferred CBCI No.D891 to Philippine Underwriters
Finance Corporation (Philfinance). Subsequently, Philfinance transferred CBCI
No. D891, which was still registered in the name of Filriters, to appellant
Traders Royal Bank (TRB). Armed with the deed of assignment, TRB sought
the transfer and registration of CBCI No. D891 in its name before the Central
Bank. The Central Bank, however, refused to effect the transfer and
registration in view of an adverse claim filed by defendant Filriters.
The lower court ruled against TRB. The CA found that the assignment
of the certificate from Filriters to Philfinance was fictitious, having made
without consideration. Thus, TRB's claimed interest has no basis, what
happened was Philfinance merely borrowed CBCI No. D891 from Filriters, a
sister corporation, to guarantee its financing operations. Thus, this petition.
Petitioner now argues that the transfer of the subject CBCI to TRB must
upheld, as the respondent Filriters and Philfinance, though separate
corporate entities on paper, have used their corporate fiction to defraud TRB
into purchasing the subject CBCI, which purchase now is refused registration
by the Central Bank.
ISSUE:
Whether or not TRB properly invoked the defense of piercing the
veil of corporate fiction
RULING:
Piercing the veil of corporate entity requires the court to see through
the protective shroud which exempts its stockholders from liabilities that
ordinarily, they could be subject to, or distinguished one corporation from a
seemingly separate one, were it not for the existing corporate fiction. But to
do this, the court must be sure that the corporate fiction was misused, to
such an extent that injustice, fraud, or crime was committed upon another,
disregarding, thus, his, her, or its rights. It is the protection of the interests of
innocent third persons dealing with the corporate entity which the law aims
to protect by this doctrine.
The corporate separateness between Filriters and Philfinance remains,
despite the petitioners insistence on the contrary. For one, other than the

Page 181 of 1072

allegation that Filriters is 90% owned by Philfinance, and the identity of one
shall be maintained as to the other, there is nothing else which could lead
the court under circumstance to disregard their corporate personalities.
In the case at bar, there is sufficient showing that the petitioner was
not defrauded at all when it acquired the subject certificate of indebtedness
from Philfinance. On its face the subject certificates states that it is
registered in the name of Filriters. This should have put the petitioner on
notice, and prompted it to inquire from Filriters as to Philfinance's title over
the same or its authority to assign the certificate. As it is, there is no showing
to the effect that petitioner had any dealings whatsoever with Filriters, nor
did it make inquiries as to the ownership of the certificate.

TOPIC:DOCTRINE OF CORPORATE ENTITY VS. PIERCING THE VEIL OF


CORPORATE FICTION
TELEPHONE ENGINEERING & SERVICE COMPANY, INC., petitioner,

Page 182 of 1072

vs.
WORKMEN'S COMPENSATION COMMISSION, PROVINCIAL SHERIFF OF
RIZAL and LEONILA SANTOS GATUS, for herself and in behalf of her
minor children, Teresita, Antonina and Reynaldo, all surnamed
GATUS, respondents.
G.R. No. L-28694. May 13, 1981
FACTS:
TESCO is engaged in the business of manufacturing telephone
equipment with offices at Sheridan Street, Mandaluyong, Rizal. Its Executive
Vice-President and General Manager is Jose Luis Santiago. It has a sister
company, the Utilities Management Corporation (UMACOR), with offices in
the same location. UMACOR is also under the management of Jose Luis
Santiago. On September 8, 1964, UMACOR employed the late Pacifico L.
Gatus as Purchasing Agent. On May 16, 1965, Pacifico L. Gatus was detailed
with petitioner company. He reported back to UMACOR on August 1, 1965.
On January 13, 1967, he contracted illness and although he returned to work
on May 10, 1967, he died nevertheless on July 14, 1967 of "liver cirrhosis
with malignant degeneration."
Pacificos widow filed a claim for compensation Workmen's
Compensation Commision (WCC), alleging therein that her deceased
husband was an employee of TESCO and that he died of liver cirrhosis. WCC
required TESCO to submit an Employer's Report of Accident or Sickness. The
report was thus submitted with UMACOR indicated as the employer of the
deceased and was signed by Jose Luis Santiago. The employer stated that it
would not controvert the claim for compensation, and admitted that the
deceased employee contracted illness "in regular occupation." On the basis
of this Report, the Acting Referee awarded death benefits plus burial
expenses in favor of the heirs of Pacifico against TESCO.
TESCO filed with SC a petition seeking to annul the award and to enjoin
the Sheriff from levying and selling its properties at public auction. TESCO
takes the position that WCC has no jurisdiction to render a valid award in this
suit as there was no employer-employee relationship between them, the
deceased having been an employee of UMACOR and not of TESCO.
ISSUE:
Whether TESCO and UMACOR are one and the same entity so that if
in the affirmative TESCO can be considered as the employer of
Pacifico and the award against it is proper.
RULING:
It is only in this Petition that TESCO denied, for the first time, the
employer-employee relationship. In fact, TESCO represented and defended
itself as the employer of the deceased. Nowhere in said documents did it

Page 183 of 1072

allege that it was not the employer. There was even an admission by TESCO
itself that TESCO and UMACOR are sister companies operating under one
single management and housed in the same building. Although respect for
the corporate personality as such, is the general rule, there are exceptions.
In appropriate cases, the veil of corporate fiction may be pierced as when the
same is made as a shield to confuse the legitimate issues.

TOPIC:DOCTRINE OF CORPORATE ENTITY VS. PIERCING THE VEIL OF


CORPORATE FICTION

Page 184 of 1072

BUENAFLOR C. UMALI, MAURICIA M. VDA. DE CASTILLO, VICTORIA M.


CASTILLO, BERTILLA C. RADA, MARIETTA C. ABAEZ, LEOVINA C.
JALBUENA and SANTIAGO M. RIVERA, petitioners,
vs.
COURT OF APPEALS, BORMAHECO, INC. and PHILIPPINE MACHINERY
PARTS MANUFACTURING CO., INC., respondents.
G.R. No. 89561. September 13, 1990
FACTS:
Santiago Rivera is the nephew of plaintiff Mauricia Meer Vda. de
Castillo. The Castillo family are the owners of a parcel of land located in
Lucena City which was given as security for a loan from the Development
Bank of the Philippines. For their failure to pay the amortization, foreclosure
of the said property was about to be initiated. This problem was made known
to Santiago Rivera, who proposed to them the conversion into subdivision of
the four (4) parcels of land adjacent to the mortgaged property to raise the
necessary fund. The Idea was accepted by the Castillo family and to carry
out the project, a Memorandum of Agreement was executed by and between
Slobec Realty and Development, Inc., represented by its President Santiago
Rivera and the Castillo family. In this agreement, Santiago Rivera obliged
himself to pay the Castillo family the sum of P70,000.00 immediately after
the execution of the agreement and to pay the additional amount of
P400,000.00 after the property has been converted into a subdivision.
Rivera, armed with the agreementapproached Mr. Modesto Cervantes,
President of defendant Bormaheco, and proposed to purchase from
Bormaheco two (2) tractors Model D-7 and D-8 Subsequently, a Sales
Agreement
Bormaheco, Inc. and Slobec Realty and Development, Inc., represented
by its President, Santiago Rivera, executed a Sales Agreement over one unit
of Caterpillar Tractor D-7 over the said equipment as security for the
payment of the aforesaid balance of P180,000.00. As further security of the
aforementioned unpaid balance, Slobec obtained from Insurance Corporation
of the Phil. a Surety Bond, with ICP (Insurance Corporation of the Phil.) as
surety and Slobec as principal, in favor of Bormaheco, as borne out by
Exhibit '8' (p. 111, Record). The aforesaid surety bond was in turn secured by
an Agreement of Counter-Guaranty with Real Estate Mortgage executed by
Rivera as president of Slobec and Mauricia Meer Vda. de Castillo, Buenaflor
Castillo Umali, Bertilla Castillo-Rada, Victoria Castillo, Marietta Castillo and
Leovina Castillo Jalbuena, as mortgagors and Insurance Corporation of the
Philippines (ICP) as mortgagee. In this agreement, ICP guaranteed the
obligation of Slobec with Bormaheco in the amount of P180,000.00. In giving
the bond, ICP required that the Castillos mortgage to them the properties in
question, namely, four parcels of land covered by TCTs in the name of the
mortgagors.
ISSUE/S:

Page 185 of 1072

Whether or not piercing the veil of corporate entity is the proper


remedy in order that the foreclosure proceeding may be declared a
nullity
RULING:
No.
Under the doctrine of piercing the veil of corporate entity, when valid
grounds therefore exist, the legal fiction that a corporation is an entity with a
juridical personality separate and distinct from its members or stockholders
may be disregarded. In such cases, the corporation will be considered as a
mere association of persons. The members or stockholders of the corporation
will be considered as the corporation, that is, liability will attach directly to
the officers and stockholders. The doctrine applies when the corporate fiction
is used to:
defeat public convenience,
justify wrong, protect fraud, or
defend crime, or
when it is made as a shield to confuse the legitimate issues or
where a corporation is the mere alter ego or business conduit of
a person, or

where the corporation is so organized and controlled and its


affairs are so conducted as to make it merely an instrumentality,
agency, conduit or adjunct of another corporation.
In the case at bar, petitioners seek to pierce the V621 Of corporate
entity of Bormaheco, ICP and PM Parts, alleging that these corporations
employed fraud in causing the foreclosure and subsequent sale of the real
properties belonging to petitioners. While we do not discount the possibility
of the existence of fraud in the foreclosure proceeding, neither are we
inclined to apply the doctrine invoked by petitioners in granting the relief
sought.
Piercing the veil of corporate entity is not the proper remedy in order
that the foreclosure proceeding may be declared a nullity under the
circumstances obtaining in the legal case at bar.
In the first place, the legal corporate entity is disregarded only if it is
sought to hold the officers and stockholders directly liable for a corporate
debt or obligation. In the instant case, petitioners do not seek to impose a
claim against the individual members of the three corporations involved; on
the contrary, it is these corporations which desire to enforce an alleged right
against petitioners. Assuming that petitioners were indeed defrauded by
private respondents in the foreclosure of the mortgaged properties, this fact
alone is not, under the circumstances, sufficient to justify the piercing of the
corporate fiction, since petitioners do not intend to hold the officers and/or

Page 186 of 1072

members of respondent corporations personally liable therefor. Petitioners


are merely seeking the declaration of the nullity of the foreclosure sale,
which relief may be obtained without having to disregard the aforesaid
corporate fiction attaching to respondent corporations. Secondly, petitioners
failed to establish by clear and convincing evidence that private respondents
were purposely formed and operated, and thereafter transacted with
petitioners, with the sole intention of defrauding the latter.
The mere fact, therefore, that the businesses of two or more
corporations are interrelated is not a justification for disregarding their
separate personalities, absent sufficient showing that the corporate entity
was purposely used as a shield to defraud creditors and third persons of their
rights.

Page 187 of 1072

TOPIC:DOCTRINE OF CORPORATE ENTITY VS. PIERCING THE VEIL OF


CORPORATE FICTION
VLASON ENTERPRISES CORPORATION, petitioner,
vs.
COURT OF APPEALS and DURAPROOF SERVICES, represented by its
General Manager, Cesar Urbino Sr., respondents.
G.R. Nos. 121662-64. July 6, 1999
FACTS:
Poro Point Shipping Services, then acting as the local agent of Omega
Sea Transport Company of Honduras & Panama, a Panamanian company,
(Omega), requested permission for its vessel M/V Star Ace, which had engine
trouble, to unload its cargo and to store it at the Philippine Ports Authority
(PPA) compound in San Fernando, La Union while awaiting transhipment to
Hongkong. The request was approved by the Bureau of Customs.
Despite the approval, the customs personnel boarded the vessel when
it docked on January 7, 1989, on suspicion that it was the hijacked M/V Silver
Med owned by Med Line Philippines Co., and that its cargo would be
smuggled into the country. The district customs collector seized said vessel
and its cargo. A notice of hearing of SFLU Seizure Identification No. 3-89 was
served on its consignee, Singkong Trading Co. of Hongkong, and its shipper,
Dusit International Co., Ltd. of Thailand.
While seizure proceedings were ongoing, La Union was hit by three
typhoons, and the vessel ran aground and was abandoned. On June 8, 1989,
its authorized representative, Frank Cadacio, entered into a salvage
agreement with private respondent to secure and repair the vessel at the
agreed consideration of $1 million and fifty percent (50%) of the cargo after
all expenses, cost and taxes. The District Collector of Customs, Aurelio M.
Quiray, lifted the warrant of seizure on July 16, 1989. However, in a Second
Indorsement dated November 11, 1989, then Customs Commissioner
Salvador M. Mison declined to issue a clearance for Quirays Decision;
instead, he forfeited the vessel and its cargo in accordance with Section
2530 of the Tariff and Customs Code. Accordingly, acting District Collector of
Customs John S. Sy issued a Decision decreeing the forfeiture and the sale of
the cargo in favor of the government.
To enforce its preferred salvors lien, herein Private Respondent
Duraproof Services filed with the RTC of Manila a Petition for Certiorari,
Prohibition and Mandamus assailing the actions of Commissioner Mison and
District Collector Sy.
Also impleaded as respondents were PPA
Representative Silverio Mangaoang and Med Line Philippines, Inc. the
complaint was amended to include former District Collector Quiray; PPA Port
Manager Adolfo Ll. Amor Jr; Petitioner Vlason Enterprises as represented by

Page 188 of 1072

its president, Vicente Angliongto; Singkong Trading Company as represented


by Atty. Eddie Tamondong; Banco Du Brasil; Dusit International Co., Inc.;
Thai-Nan Enterprises Ltd. and Thai-United Trading Co., Ltd. In both Petitions,
private respondent plainly failed to include any allegation pertaining to
petitioner, or any prayer for relief against it. Summonses for the amended
Petition were served on Atty. Joseph Capuyan for Med Line Philippines:
Angliongto (through his secretary, Betty Bebero), Atty. Tamondong and
Commissioner Mison. Upon motion of the private respondent, the trial court
allowed summons by publication to be served upon the alien defendants who
were not residents and had no direct representatives in the country.
Thereafter, there was a motion to declare defendants in default but such
motion was denied as it was not acted upon. After trial, the court a quo
adjudged the private respondent liable. Respondent later on appealed
contending Anglioto, who is served with summons through his secretary
should also be made liable.

ISSUE/S:
Can piercing the veil of corporate fiction be resorted to when
serving summons?
RULING:
No.
A corporation may be served summons through its agents or officers
who under the Rules are designated to accept service of process. A summons
addressed to a corporation and served on the secretary of its president binds
that corporation. This is based on the rationale that service must be made
on a representative so integrated with the corporation sued, that it is safe to
assume that said representative had sufficient responsibility and discretion
to realize the importance of the legal papers served and to relay the same to
the president or other responsible officer of the corporation being sued. The
secretary of the president satisfies this criterion. This rule requires, however,
that the secretary should be an employee of the corporation sought to be
summoned. Only in this manner can there be an assurance that the
secretary will "bring home to the corporation the notice of the filing of the
action" against it.
In the present case, Bebero was the secretary of Angliongto, who was
president of both VSI and petitioner, but she was an employee of VSI, not of
petitioner. The piercing of the corporate veil cannot be resorted to when
serving summons.Doctrinally, a corporation is a legal entity distinct and
separate from the members and stockholders who compose it. However,
when the corporate fiction is used as a means of perpetrating a fraud,
evading an existing obligation, circumventing a statute, achieving or
perfecting a monopoly or, in generally perpetrating a crime, the veil will be

Page 189 of 1072

lifted to expose the individuals composing it. None of the foregoing


exceptions has been shown to exist in the present case. Quite the contrary,
the piercing of the corporate veil in this case will result in manifest injustice.
This we cannot allow.
Hence, the corporate fiction remains.

Page 190 of 1072

TOPIC:DOCTRINE OF CORPORATE ENTITY VS. PIERCING THE VEIL OF


CORPORATE FICTION
VILLA REY TRANSIT, INC., plaintiff-appellant,
vs.
EUSEBIO E. FERRER, PANGASINAN TRANSPORTATION CO., INC. and
PUBLIC SERVICE COMMISSION, defendants.
G.R. No. L-23893. October 29, 1968
FACTS:
On January 8, 1959, Jose M. Villarama sold two certificates of public
convenience (CPC) to Pantranco, with the condition that the seller shall not
for a period of 10 years from the date of this sale, apply for any TPU service
identical or competing with the buyer." On March 6, 1959, a corporation
called Villa Rey Transit, Inc. was organized with the wife of Jose M. Villarama
as one of the incorporators, and at the same time the treasurer of the
corporation.
On April 7, 1959, the Corporation executed with one Valentin Fernando
a contract of sale over five CPC. The Sheriff of Manila, on July 7, 1959, levied
on two of the five CPC, by virtue of a writ of execution issued by the CFI
Pangasinan, in favor of Eusebio Ferrer, judgment creditor, against Valentin
Fernando, judgment debtor. A public sale was conducted by the Sheriff of
the said two CPC. Ferrer was the highest bidder, and a certificate of sale was
issued in his name. Thereafter, Ferrer sold the two CPC to Pantranco.
Thus, two sets of applications for approval of sale were filed before the
PSC, by Fernando and the Corporation, and that of Ferrer and Pantranco. In
the meantime, the Court decreed that until the issue on the ownership of the
disputed certificates shall have been finally settled by the proper court, the
Corporation should be the one to operate the lines provisionally.
On November 4, 1959, the Corporation filed in the CFI-Manila, a
complaint for the annulment of the sheriff's sale of the aforesaid two
certificates of public convenience in favor of Ferrer, and the subsequent sale
thereof by the latter to Pantranco. Pantranco, on its part, filed a third-party
complaint against Jose M. Villarama, alleging that Villarama and the
Corporation, are one and the same; that Villarama and/or the Corporation
was disqualified from operating the two certificates in question by virtue of
the contract non-competition stipulation.
ISSUE/S:
Whether or not Villa Rey Transit, Inc. is an alter ego of Jose M.
Villarama.

Page 191 of 1072

RULING:
Yes.
Taking account of the foregoing evidence, together with Celso Rivera's
testimony, it would appear that: Villarama supplied the organization
expenses and the assets of the Corporation, such as trucks and equipment;
there was no actual payment by the original subscribers of the amounts of
P95,000.00 and P100,000.00 as appearing in the books;Villarama made use
of the money of the Corporation and deposited them to his private accounts;
and the Corporation paid his personal accounts.
Villarama himself admitted that he mingled the corporate funds with
his own money.He also admitted that gasoline purchases of the Corporation
were made in his name because "he had existing account with Stanvac
which was properly secured and he wanted the Corporation to benefit from
the rebates that he received."
The foregoing circumstances are strong persuasive evidence showing
that Villarama has been too much involved in the affairs of the Corporation to
altogether negative the claim that he was only a part-time general manager.
They show beyond doubt that the Corporation is his alter ego.
The doctrine that a corporation is a legal entity distinct and separate
from the members and stockholders who compose it is recognized and
respected in all cases which are within reason and the law. When the fiction
is urged as a means of perpetrating a fraud or an illegal act or as a vehicle
for the evasion of an existing obligation, the circumvention of statutes, the
achievement or perfection of a monopoly or generally the perpetration of
knavery or crime,the veil with which the law covers and isolates the
corporation from the members or stockholders who compose it will be lifted
to allow for its consideration merely as an aggregation of individuals.
Upon the foregoing considerations, We are of the opinion, and so hold,
that the preponderance of evidence have shown that the Villa Rey Transit,
Inc. is an alter ego of Jose M. Villarama, and that the restrictive clause in the
contract entered into by the latter and Pantranco is also enforceable and
binding against the said Corporation. For the rule is that a seller or promisor
may not make use of a corporate entity as a means of evading the obligation
of his covenant.31 Where the Corporation is substantially the alter ego of the
covenantor to the restrictive agreement, it can be enjoined from competing
with the covenantee.

Page 192 of 1072

TOPIC:DOCTRINE OF CORPORATE ENTITY VS. PIERCING THE VEIL OF


CORPORATE FICTION
JAMES YU and WILSON YOUNG, petitioners,
vs.
THE NATIONAL LABOR RELATIONS COMMISSION, LABOR ARBITER
DANIEL C. CUETO, TANDUAY DISTILLERY INC., FERNANDO DURAN,
EDUARDO PALIWAN, ROQUE ESTOCE AND RODRIGO SANTOS,
respondents.
G.R. Nos. 111810-11. June 16, 1995
FACTS:
Private respondents were former employees of Tanduay Distillery Inc
(TDI).
On Mar 29, 1988, the employees of TDI, including private respondents
received a memorandum from TDI that their services were to be terminated
within 30 days from receipt thereof for reasons of retrenchment.
Then, on June 1, 1988, Twin Ace Holdings, buyer of TDI's assets, took
over the business and assumed to use the business name Tanduay Distillers.
Thereafter, the retrenched employees filed a motion to implead James Yu and
Wilson Young as party in the illegal dismissal case. Petitioners herein
objected said motion claiming , asserting that they are representatives of
Tanduay Distillers an entity distinct and separate from TDI, the previous
owner, and that there is no employer-employee relationship between
Tanduay Distillers and private respondents.
The LA granted the motion for the speedy disposition of justice. It also
ruled that there was illegal dismissal and ordered TDI to reinstate the
employees.
The NLRC affirmed the decision. The employees then filed a motion for
execution against Young and Yu which the petitioners opposed claiming that
there is no basis for the execution since Tanduay Distillers is different from
TDI, the entity against which the judgment was rendered. Subsequently TDI
filed a manifestation which was affirmed by the LA indicating the payment of
the monetary claims. The LA executing the order ruled that TDI, Young and
Yu should reinstate the employees. The petitioners filed a petition for
certiorari to the NLRC which denied the petition.
ISSUE/S:
Whether respondent NLRC committed grave abuse of discretion in
holding petitioners Yu and Young liable.

Page 193 of 1072

RULING:
The Supreme Court holds that petitioners, for a number of reasons,
may not be held answerable and liable under the final judgment of Labor
Arbiter Cauton-Barcelona.
It is basic that a corporation is invested by law with a personality
separate and distinct from those of the persons composing it as well as from
that of any other legal entity to which it may be related (Palay, Inc. et al. vs.
Clave, et al., 124 SCRA 641 [1983]).
The genuine nature of the sale to Twin Ace is evidenced by the fact
that Twin Ace was only a subsequent interested buyer. At the time when
termination notices were sent to its employees, TDI was negotiating with the
First Pacific Metro Corporation for the sale of its assets. Only after First Pacific
gave up its efforts to acquire the assets did Twin Ace or Tanduay Distillers
come into the picture. Respondents-employees have not presented any proof
as to communality of ownership and management to support their
contention that the two companies are one firm or closely related. The
doctrine of piercing the veil of corporate entity applies when the corporate
fiction is used to defeat public convenience, justify wrong, protect fraud, or
defend crime or where a corporation is the mere alter ego or business
conduit of a person (Indophil Textile Mill Workers Union vs. Calica, 205 SCRA
697, 703 (1992]). To disregard the separate juridical personality of a
corporation, the wrong-doing must be clearly and convincingly established. It
cannot be presumed (Del Rosario vs. NLRC, 187 SCRA 777, 7809 [1990]).
The complaint for unfair labor practice, illegal lay off, and separation
benefits was filed against TDI. Only later when the manufacture and sale of
Tanduay products was taken over by Twin Ace or Tanduay Distillers were
James Yu and Wilson Young impleaded.
The corporation itself Twin Ace or Tanduay Distillers was never
made a party to the case.
Another factor to consider is that TDI as a corporation or its shares of
stock were not purchased by Twin Ace. The buyer limited itself to purchasing
most of the assets, equipment, and machinery of TDI. Thus, Twin Ace or
Tanduay Distillers did not take over the corporate personality of DTI although
they manufacture the same product at the same plant with the same
equipment and machinery. Obviously, the trade name "Tanduay" went with
the sale because the new firm does business as Tanduay Distillers and its
main product of rum is sold as Tanduay Rum. There is no showing, however,
that TDI itself was absorbed by Twin Ace or that it ceased to exist as a

Page 194 of 1072

separate corporation, In point of fact TDI is now herein a party respondent


represented by its own counsel.
Significantly, TDI in the petition at hand has taken the side of its former
employees and argues against Tanduay Distillers. In its memorandum filed
on January 9, 1995, TDI argues that it was not alone its liability which arbiter
recognized "but also of James Yu and Wilson Young representatives of Twin
Ace and/or the Allied Bank Group doing business under the name "TANDUAY
DISTILLERS," to whom the business and assets of TDI were sold." If TDI and
Tanduay, Distillers are one and the same group or one is a continuation of
the other, the two would not be fighting each other in this case. TDI would
not argue strongly "that the petition for certiorari filed by James Yu and
Wilson Young be dismissed for lack of merit." It is obvious that the second
corporation, Twin Ace or Tanduay Distillers, is an entity separate and distinct,
from the first corporation, TDI. The circumstances of this case are different
from the earlier decisions of the Court in labor cases where the veil of
corporate fiction was pierced.
In Claparols vs. Court of Industrial Relations (65 SCRA 613 (1975]), the
Claparols Steel and Nail Plant, which was ordered to pay its workers
backwages, ceased operations on June 30, 1956 and was succeeded on the
very next day, July 1, 1957, by the Claparols Steel Corporation. Both
corporations were substantially owned and controlled by the same person
and there was no break or cessation in operations. Moreover, all the assets of
the steel and nail plant were transferred to the new corporation.
In fine, the fiction of separate and distinct corporate entities cannot, in
the instant case, be disregarded and brushed aside, there being not the least
indication that the second corporation is a dummy or serves as a client of the
first corporate entity.
In the case at bench, since TDI and Twin Ace or Tanduay Distillers are
two separate and distinct entities, the order for Tanduay Distillers (and
petitioners) to reinstate respondents-employees is obviously without legal
and factual basis.

Page 195 of 1072

TOPIC:DE FACTO CORPORATION


C. ARNOLD HALL and BRADLEY P. HALL, petitioners,
vs.
EDMUNDO S. PICCIO, Judge of the Court of First Instance of Leyte,
FRED BROWN, EMMA BROWN, HIPOLITA CAPUCIONG, in his capacity
as receiver of the Far Eastern Lumber and Commercial Co., Inc.,
respondents.
G.R. No. L-2598. June 29, 1950
FACTS:
On May 28, 1947, the petitioners C. Arnold Hall and Bradley P. Hall, and
the respondents Fred Brown, Emma Brown, Hipolita D. Chapman and
Ceferino S. Abella, signed and acknowledged in Leyte, the articles of
incorporation of the Far Eastern Lumber and Commercial Co., Inc., organized
to engage in a general lumber business to carry on as general contractors,
operators and managers, etc. Attached to the articles was an affidavit of the
treasurer stating that 23,428 shares of stock had been subscribed and fully
paid with certain properties transferred to the corporation described in a list
appended thereto.
Immediately after the execution of said articles of incorporation, the
corporation proceeded to do business with the adoption of by-laws and the
election of its officers.
On December 2, 1947, the said articles of incorporation were filed in
the office of the SEC Commissioner, for the issuance of the corresponding
certificate of incorporation.
On March 22, 1948, pending action on the articles of incorporation by
the aforesaid governmental office, the respondents filed the civil case
alleging among other things that the Far Eastern Lumber and Commercial Co.
was an unregistered partnership; that they wished to have it dissolved
because of bitter dissension among the members, mismanagement and
fraud by the managers and heavy financial losses. The defendants in the suit
filed a motion to dismiss, contesting the court's jurisdiction and the
sufficiency of the cause of action.
After hearing the parties, the dissolution of the company was ordered;
and at the request of plaintiffs, appointed the respondent Pedro A. Capuciong
as receiver of the properties thereof. The petitioners offered to file a counterbond for the discharge of the receiver, but the respondent judge refused to
accept the offer and to discharge the receiver.

Page 196 of 1072

Thus, the present special civil action was instituted in this court.
ISSUE/S:
Whether or not court had jurisdiction to decree the dissolution of the
company, because it being a de facto corporation, dissolution
thereof may only be ordered in a quo warranto proceeding instituted
in accordance with section 19 of the Corporation Law.
RULING:
Section 19 reads as follows:
. . . The due incorporation of any corporations claiming in good
faith to be a corporation under this Act and its right to exercise
corporate powers shall not be inquired into collaterally in any
private suit to which the corporation may be a party, but such
inquiry may be had at the suit of the Insular Government on
information of the Attorney-General.
There are least two reasons why this section does not govern the
situation. Not having obtained the certificate of incorporation, the Far Eastern
Lumber and Commercial Co. even its stockholders may not probably
claim "in good faith" to be a corporation.
Under our statue it is to be noted (Corporation Law, sec. 11) that it is
the issuance of a certificate of incorporation by the Director of the Bureau of
Commerce and Industry which calls a corporation into being. The immunity if
collateral attack is granted to corporations "claiming in good faith to be a
corporation under this act." Such a claim is compatible with the existence of
errors and irregularities; but not with a total or substantial disregard of the
law. Unless there has been an evident attempt to comply with the law the
claim to be a corporation "under this act" could not be made "in good faith."
(Fisher on the Philippine Law of Stock Corporations, p. 75. See also
Humphreys vs. Drew, 59 Fla., 295; 52 So., 362.)
Second, this is not a suit in which the corporation is a party. This is a
litigation between stockholders of the alleged corporation, for the purpose of
obtaining its dissolution. Even the existence of a de jure corporation may be
terminated in a private suit for its dissolution between stockholders, without
the intervention of the state.
There might be room for argument on the right of minority
stockholders to sue for dissolution;1 but that question does not affect the
court's jurisdiction, and is a matter for decision by the judge, subject to
review on appeal. Whkch brings us to one principal reason why this petition

Page 197 of 1072

may not prosper, namely: the petitioners have their remedy by appealing the
order of dissolution at the proper time.
There is a secondary issue in connection with the appointment of a
receiver. But it must be admitted that receivership is proper in proceedings
for dissolution of a company or corporation, and it was no error to reject the
counter-bond, the court having declared the dissolution. As to the amount of
the bond to be demanded of the receiver, much depends upon the discretion
of the trial court, which in this instance we do not believe has been clearly
abused.

Page 198 of 1072

TOPIC: CORPORATION BY ESTOPPEL


INTERNATIONAL EXPRESS TRAVEL & TOUR SERVICES, INC., petitioner
vs.
COURT OF APPEALS, respondent
G.R. NO. 119002, OCTOBER 19, 2000
FACTS:
Petitioner IETTS wrote a letter to the Philippine Football Federation
through its president Henri Khan offering its services as a travel agency to
the latter. Such offer was accepted. For partial payment, Khan issued a
personal check.however, no other payments were made.Thus, a suit was
filed against Henri Khan in his personal capacity as the president of the
Football Federation.
It was claimed that the Federation was not a corporation with valid
corporate personality. An entity may be recognize as a national sports
association when recognized by the accrediting association the Philippine
Amateur Athletic Federation under R.A. 3135 and P.D. 604 which Henri Khan
failed to substantiate.
ISSUE/S:
Whether or not the doctrine of estoppels is applicable in this case.
RULING:
NO.
The doctrine of estoppel applies to a third party only when he tries to
escape liabilities on a contract from which he has benefited on the irrelevant
ground of defective incorporation. Herein IETTSI is not trying to escape
liability from the contract but rather is the one claiming from the contract.
Henri Khan should be held liable for the unpaid obligations of the
unincorporated Federation. It is settled that any person acting or purporting
to act on behalf of a corporation which has no valid existence assumes such
privileges and becomes personally liable.

Page 199 of 1072

TOPIC:CORPORATION BY ESTOPPEL
LIM TONG LIM, petitioner
vs.
PHIL. FISHING GEAR INDUSTRIES, INC., respondent
G.R. NO. 136448, NOVEMBER 3, 1999
317 SCRA 728
FACTS:
On behalf of Ocean Quest Fishing Corp., Antonio Chua & Peter Yao
entered into a contract with respondents for the purchase of fishing nets of
various sizes and floats. They claimed to be engaged in a business venture
with petitioner who however was not made a signatory of the agreement.
Chua, Yao and petitioner failed to pay for the fishing nets and the
floats, hence respondent filed for a collection suit and writ of preliminary
attachment in their capacities as general partners on the allegation that the
corporation they represent was a non-existent corporation as certified by the
SEC.
ISSUE/S:
Whether or not petitioner is estopped from assailing the existence
of partnership on ground that he was not a signatory of the
contract.
RULING:
Yes.
The evidence clearly showed that the partnership existed among
petitioner, Chua & Yao to engage in a fishing business by buying boats
finance by a loan secured from petitioners brother.
Under Sec. 21 of the Corporation Code, corporation by estoppel applies
to an alleged corporation and to a third party. The instant case is an
unincorporated corporation where it is estopped from denying its corporate
capacity in a suit against it by a third person who relied in good faith on such
representation. It cannot allege lack of personality to be sued to shy away
from its responsibility for a contract it entered into and by virtue of which it
received advantages and benefits.

Page 200 of 1072

In this case, though petitioner s name does not appear on the


contract, petitioner benefited from the use of the nets found inside F/B
Lourdes, the boat which was proven to be the asset of the partnership placed
under his name to assure payment of the debt he and his partners owed.
TOPIC:CORPORATION BY ESTOPPEL
MARIANO ALBERT, plaintiff-appellant
vs.
UNIVERSITY PUBLISHING CO., INC., defendant-appellee
G.R. NO. L -19118, JANUARY 30, 1965
FACTS:
Petitioner entered into contract with respondent through its president,
Mr. Jose Aruego for the exclusive right to publish his Commentaries on the
RPC for an amount of P30, 000.00 to be paid in instalments. However,
respondent failed to comply, thus A collection suit was filed against the
corporation.
Upon the issuance of a writ of execution, it was found out that
respondent corporation is non-existing as certified by the SEC. Thus,
petitioner sought to enforce the writ against Mr. Aruego who signed as its
president and who represented the corporation in the proceedings.
ISSUE/S:
Whether or not respondent is estopped from assailing its corporate
personality.
HELD:
The Court ruled that the Corporation-By-Estoppel Doctrine is not
applicable in this case as the purported corporation is in reality non existent.
There was a misrepresentation by Mr. Aruego signing as a president that the
corporation is duly organized and existing under the laws of the Philippines
and misled petitioner into believing the same.
The evidence is clear that Mr. Aruego acting as the representative of a
non-existent principal was the real party in the contract and that he was the
one who reaped the benefits resulting from it, so much so that partial
payments of the consideration were made by him. Perforce, responsibility
under judgment falls on him.

Page 201 of 1072

TOPIC:CORPORATION BY ESTOPPEL
LOYOLA GRAND VILLAS HOMEOWNERS (SOUTH) ASSOCIATION,
INC., petitioner,
vs.
HON. COURT OF APPEALS, HOME INSURANCE AND GUARANTY
CORPORATION, EMDEN ENCARNACION and HORATIO
AYCARDO, respondents.
G.R. NO. 117188, AUGUST 7, 1997
FACTS:
Loyola Grand Villas Homeowners Association (LGVHA) is the sole
homeowners' association in Loyola Grand Villas, duly registered subdivision
revoked the certificates of registration issued to Loyola Grand Villas
homeowners (North) Association Incorporated (the North Association for
brevity) and Loyola Grand Villas Homeowners (South) Association
Incorporated (the South Association) for failure to file its corporate by-laws.
These developments prompted the officers of the LGVHAI to lodge a
complaint with the HIGC. They questioned the revocation of LGVHAI's
certificate of registration without due notice and hearing and concomitantly
prayed for the cancellation of the certificates of registration of the North and
South Associations by reason of the earlier issuance of a certificate of
registration in favor of LGVHAI.
ISSUE:
Whether or not the doctrine of estoppels is applicable in this case.
HELD:
No.
The failure to file its by - laws presupposes that the corporation is
already incorporated and has the effect only of suspension or revocation
pursuant to PD 902 A after proper notice and hearing.
It necessarily follows that failure to file the by-laws within that period
does not imply the "demise" of the corporation. By-laws may be necessary
for the "government" of the corporation but these are subordinate to the
articles of incorporation as well as to the Corporation Code and related
statutes. In the absence of charter or statutory provisions to the contrary,

Page 202 of 1072

by-laws are not necessary either to the existence of a corporation or to the


valid exercise of the powers conferred upon it, certainly in all cases where
the charter sufficiently provides for the government of the body; and even
where the governing statute in express terms confers upon the corporation
the power to adopt by-laws, the failure to exercise the power will be ascribed
to mere nonaction which will not render void any acts of the corporation
which would otherwise be valid.
The mere fact, however, of the existence of power in the corporation to
adopt by-laws does not ordinarily and of necessity make the exercise of such
power essential to its corporate life, or to the validity of any of its acts.

Page 203 of 1072

TOPIC: QUALIFICATIONS/QUALIFYING SHARE


REP. LUIS R. VILLAFUERTE, PROSPERO A. PICHAY, CHRISTIAN TAN,
WILSON YOUNG, TERESITA ABUNDO, TONY FABICO, BONIFACIO
ALENTAJAN, RIZALITO DELMORO, GODOFREDO E. GALLEGA, MANNY
A. GATCHALIAN, MA. CARMEN S. PADOR, CELESTINO S. MARTINEZ,
ANTONIO TAN ITURALDE, ALEXANDER WANG, YUL C. BENOSA,
ELBERT CATAMPUNGAN ATILLANO, SR., LORENZO CO SY, EDWARD YU
CHUA and LEONCIO CHUA, Petitioners,
vs.
GOV. OSCAR S. MORENO, MANUEL V. PANGILINAN, MARIEVIC G.
RAMOS-AONUEVO, JOSE A. CAPISTRANO, JR., PEDRO C. ALFARO,
JR., BERNARDO GABRIEL L. ATIENZA, JOSE EMMANUEL M. EALA,
FERNANDO G. LOZANO, FR. PAUL M. DE VERA OSB, NICANOR
FORTICH JORGE, DANIEL DANILO V. SORIA and NATHANIEL P.
PADILLA, Respondents.
G.R. No. 186566. October 2, 2009
FACTS:
On 28 August 2006, at the sideline of the 18 th FIBA World Congress
held at Tokyo, Japan, a Joint Communique (Tokyo Communique) was
entered into by the feuding Basketball Association of the Philippines (BAP)
and the newly formed Pilipinas Basketbol (PB), through their then
incumbent Presidents, Jose D. Lina, Jr. And Bernardo Gabriel L. Atienza,
respectively, and as witnessed not only by their other representatives but
also by the representative of the Philippine Olympic Committee (POC) and
the FIBA Secretary General Patrick Baumann. The main objectives of the
Tokyo Communique are (1) to unify said rival basketball associations and (2)
to facilitate the lifting of the suspension imposed by the Federation
Internationale de Basketball (FIBA), which prevented the country from
participating in any international basketball competitions.
Specifically, the Tokyo Communique provides for the merger of the BAP
and the PB resulting to a single united basketball organization that will seek
membership with the POC and will eventually take over the membership of
BAP in the FIBA, subject to the appropriate FIBA regulations on membership.
Pursuant to the provisions of the Tokyo Communique relative to the creation
of a three-man panel, petitioner Manuel V. Pangilinan (Petitioner Pangilinan)
was named as its third member and was even chosen as its Chairman. Also,
the BAP and PB submitted to FIBA their respective lists of membersassociations in compliance with the provisions thereof.
In keeping with the merger and unification efforts as embodied in the
Tokyo Communique, the Samahang Basketbol ng Pilipinas, Inc. (SBP) was

Page 204 of 1072

established and its constitutive documents consisting of the Articles of


Incorporation were signed by the five (5) incorporators, which include
petitioner Pangilinan. On the same day, the incorporators likewise passed
and signed its by-laws. The three-man panel met in Bangkok, Thailand where
it forged and executed a Memorandum of Agreement (Bangkok
Agreement) integrating therein the final terms and conditions of the unity
and merger of BAP and PB. In said agreement, the BAP and PB amended the
corporate name of SBP from Samahang Basketbol ng Pilipinas, Inc. To
BAP-Samahang Basketbol ng Pilipinas, Inc. (BAP-SBP).
Petitioners filed before the Regional Trial Court of Manila a petitionfor
declaration of nullity of the election of respondents as members of the Board
of Trustees and Officers of BAP-SBP. The case was docketed as Civil Case No.
08-119546. Petitioners alleged that the June 12, 2008 election was a sham,
illegal, and void. They also claimed to be the rightful and legally elected
trustees and officers of the BAP-SBP and thus prayed that the corporate reins
of BAP-SBP be turned over to them.
ISSUE/S:
Which members of the BAP-SBP are entitled to vote and be voted
upon as trustees and officers of said organization based on the
terms and conditions of the Tokyo Communique, the Bangkok
Agreement and the Articles of Incorporation and By-Laws of the
organization.
RULING:
The Supreme Court finds that the Court of Appeals correctly held that
Clause 3 of the Bangkok Agreement merely intended to recognize the
associations affiliated with BAP and PB as members as against being
]abelled as just probationary members of the BAP-SBP. However, said
recognition does not dispense with the need to classify said members in
accordance with the provisions of BAP-SBPs Articles of Incorporation and ByLaws, and the Tokyo Communique. Had the intention been otherwise, the
parties would have expressed this by means of the appropriate provisions
repealing or amending the contradictory provisions in said documents as
what they did to a provision in the Bangkok Agreement with respect to the
removal of officers.
Moreover, Clause 3 of the Bangkok Agreement must be read not in
isolation but in conjunction with the Tokyo Communique and the BAP-SBPs
Article of Incorporation and By-Laws. The Court of Appeals historical account
as to how all subject documents came into being is enlightening, thus:
Pertinently, the Tokyo Communique purposely created a threeman panel to review, verify, and validate the list of members as

Page 205 of 1072

submitted by PB and BAP to the FIBA Central Board Special


Commission created to hear the Philippine case based on agreed
set of criteria for membership formulated by three-man panel.
Pursuant to the stipulations of the Tokyo Communique, the SBP
was created leading to the execution and adoption of its Articles
of Incorporation and by-laws, which laid down, among others, the
criteria for membership of the SBP. Subsequent thereto, the
three-man panel again convened and executed the said Bangkok
Agreement, in which the admission of all the bona fide members
of BAP and PB as appearing in the lists submitted to FIBA as
members instead of probationary members of SBP was agreed
upon.
To reiterate, the Tokyo Communiques directive to the three-man panel
is for it to review, verify, and validate the list of members as submitted by PB
and BAP to the FIBA Central Board Special Commission created to hear the
Philippine Case based on an agreed set of criteria for membership as
formulated by said three-man panel. In other words, there is a given process
for validation of membership rather than the automatic grant of voting or
active membership status being insisted upon by petitioners. Besides, had it
intended all bona fide members to be admitted as accredited members or
first members or active members, the three-man panel would have
specifically used such term since its members were all aware that the SBPs
Articles of Incorporation and by-laws were already in existence at the time
and also provided for three classes or categories of members.

Page 206 of 1072

TOPIC: QUALIFICATIONS/QUALIFYING SHARE


CONSTANCIO T. BAGUIO, petitioner,
vs.
COURT OF APPEALS (Fourteenth Division), LAS PALMAS
INTERNATIONAL MANPOWER CORPORATION, SPOUSES DONALDO
PALMA AND CONSUELO P. PALMA and CYNTHIA C. CALAPRE,
respondents.
G.R. No. 93417. September 14, 1993
226 SCRA 366
FACTS:
Respondent Donaldo Palma was the president of respondent Las
Palmas; his wife, respondent Consuelo Palma, was the vice-president and
treasurer; and respondent Cynthia G. Calapre was the corporate secretary.
Respondents Palmas were also officers of Masters & Mates Association of the
Philippines, a sublessee of a portion of the office space leased by petitioner.
In the early part of April 1982, petitioner saw respondent Donaldo
Palma to collect his 25% share in the profits earned by respondents Palmas
when they sent 21 workers to Saudi Arabia. Instead of paying petitioner,
respondents Palmas offered to sell him 600 shares of stock or respondent Las
Palmas for P60,000.00 and to make him a director and vice-president of the
corporation.
As to the subsequent events, the trial court accepted as true the
version of petitioner. According to the trial court, respondents Palmas went to
the office of petitioner on July 8, 1982 at 7:00 p.m., where petitioner handed
the amount of P60,000.00 in P100.00-bills to them in the presence of Jose
Baldeo, Jr., a security guard of the building. Respondents Palmas, in turn
delivered to petitioner a copy of the Board Resolution No. 001, series of 1982
of respondent corporation (Exh. A) and the secretary's certificate (Exh. B).
When the petitioner asked for a receipt, respondents Palmas assured him
that the board resolution and the secretary's certificate were better evidence
of payment than an ordinary receipt. He was likewise told that the stock
certificate would be issued in December 1982, after the board meeting.
Respondents Palmas used the money to pay their employees, whose salaries
had not been paid for several months.

Page 207 of 1072

As December 1982 came and no certificate of stock was issued to him,


petitioner became suspicious of respondents Palmas. Sometime in May 1983,
petitioner inquired from the Securities and Exchange Commission about the
legal personality of Respondent Corporation. He discovered that the board
resolution (Exh. A) and secretary's certificate (Exh. B) Were not recorded with
said office. Moreover, the corporate secretary listed in the SEC records was a
certain Anabelle Acapulco and not respondent Calapre.
Petitioner, having lost his patience, ejected Master and Mates
Association of the Philippines, the agency owned by respondents Palmas.
ISSUE/S:
Whether petitioner can claim that being a member of the board of
directors and occupying the position of Vice-President-International
necessarily imply that he must have owned duly-paid shares of
stock.
RULING:
Under Section 63 of the Corporation Code, no transfer of shares of
stock shall be valid, except as between the parties, until the transfer is
recorded in the books of the corporation showing the names of the parties to
the transfer, the date of the transfer and the number of the certificates and
shares transferred. Petitioner has not shown compliance with this law.
Petitioner further cites that if it were true that respondents Palmas
failed to receive his payment they should have passed another board
resolution, specifically cancelling the offer contained in board resolution
(Exh. A). There was no need to issue another resolution cancelling the board
resolution, Exhibit A, because no certificate of stock was issued and no
transfer of shares was recorded in the books of the corporation pursuant
thereto. More so, if we consider the transaction is not between the
corporation and petitioner but between private respondents qua
stockholders and petitioner.
Petitioner cannot claim that being a member of the board of directors
and occupying the position of Vice-President-International necessarily imply
that he must have owned duly-paid shares of stock.
The election of a person to the board of directors of a corporation does
not necessarily mean that he has paid for the shares recorded in his name. In
most cases, nominee directors do not pay for the qualifying shares assigned
to them. Likewise, the Corporation Code does not require that one elected or
appointed as vice-president of a corporation should be the owner of shares of
stock of the corporation.

Page 208 of 1072

TOPIC: QUALIFICATIONS/QUALIFYING SHARE


DETECTIVE & PROTECTIVE BUREAU, INC., petitioner,
vs.
THE HONORABLE GAUDENCIO CLORIBEL, in his capacity as Presiding
Judge of Branch VI, Court of First Instance of Manila, and FAUSTINO
S. ALBERTO, respondents.
G.R. No. L-23428. November 29, 1968
26 SCRA 255
FACTS:
Plaintiff was a corporation duly organized and existing under the laws
of the Philippines; that defendant was managing director of plaintiff
corporation from 1952 until January 14, 1964; that in June, 1963, defendant
illegally seized and took control of all the assets as well as the books,
records, vouchers and receipts of the corporation from the accountantcashier, concealed them illegally and refused to allow any member of the
corporation to see and examine the same; that on January 14, 1964, the
stockholders, in a meeting, removed defendant as managing director and
elected Jose de la Rosa in his stead; that defendant not only had refused to
vacate his office and to deliver the assets and books to Jose de la Rosa, but
also continued to perform unauthorized acts for and in behalf of plaintiff
corporation; that defendant had been required to submit a financial
statement and to render an accounting of his administration from 1952 but
defendant has failed to do so; that defendant, contrary to a resolution
adopted by the Board of Directors on November 24, 1963, had been illegally
disposing of corporate funds; that defendant, unless immediately restrained
ex-parte, would continue discharging the functions of managing director; and
that it was necessary to appoint a receiver to take charge of the assets and
receive the income of the corporation.
Plaintiff prayed that a preliminary injunction ex-parte be issued
restraining defendant from exercising the functions of managing director and
from disbursing and disposing of its funds; that Jose M. Barredo be appointed
receiver; that, after judgment, the injunction be made permanent and
defendant be ordered to render an accounting.
ISSUE/S:

Page 209 of 1072

Whether Jose de la Rosa could not be elected managing director


because he did not own any stock in the corporation.
RULING:
There is in the record no showing that Jose de la Rosa owned a share of
stock in the corporation. If he did not own any share of stock, certainly he
could not be a director pursuant to the mandatory provision of Section 30 of
the Corporation Law, which in part provides:
Sec. 30. Every director must own in his own right at least one
share of the capital stock of the stock corporation of which he is
a director, which stock shall stand in his name on the books of
the corporations....
If he could not be a director, he could also not be a managing director
of the corporation, pursuant to Article V, Section 3 of the By-Laws of the
Corporation which provides that:
The manager shall be elected by the Board of Directors from
among its members.... (Record, p. 48)
If the managing director-elect was not qualified to become
managing director, respondent Fausto Alberto could not be
compelled to vacate his office and cede the same to the
managing director-elect because the by-laws of the corporation
provides in Article IV, Section 1 that Directors shall serve until
the election and qualification of their duly qualified successor.

Page 210 of 1072

TOPIC: QUALIFICATIONS/QUALIFYING SHARE


GRACE CHRISTIAN HIGH SCHOOL, petitioner,
vs.
THE COURT OF APPEALS, GRACE VILLAGE ASSOCIATION, INC.,
ALEJANDRO G. BELTRAN, and ERNESTO L. GO, respondents.
G.R. No. 108905. October 23, 1997
281 SCRA 133
FACTS:
Petitioner Grace Christian High School is an educational institution
offering preparatory, kindergarten and secondary courses at the Grace
Village in Quezon City. Private respondent Grace Village Association, Inc., on
the other hand, is an organization of lot and/or building owners, lessees and
residents at Grace Village, while private respondents Alejandro G. Beltran
and Ernesto L. Go were its president and chairman of the committee on
election, respectively, in 1990, when this suit was brought.
As adopted in 1968, the by-laws of the association. Subsequently, on
December 20, 1975, a committee of the board of directors prepared a draft
of an amendment to the by-laws. The draft was never presented to the
general membership for approval. Nevertheless, from 1975, after it was
presumably submitted to the board, up to 1990, petitioner was given a
permanent seat in the board of directors of the association. On February 13,
1990, the associations committee on election in a letter informed James Tan,
principal of the school, that it was the sentiment that all directors should be
elected by members of the association because to make a person or entity
a permanent Director would deprive the right of voters to vote for fifteen (15)
members of the Board, and it is undemocratic for a person or entity to hold
office in perpetuity. For this reason, Tan was told that the proposal to make
the Grace Christian High School representative as a permanent director of
the association, although previously tolerated in the past elections should be
reexamined. Following this advice, notices were sent to the members of the
association that the provision on election of directors of the 1968 by-laws of
the association would be observed.
Petitioner requested the chairman of the election committee to change
the notice of election by following the procedure in previous elections,
claiming that the notice issued for the 1990 elections ran counter to the

Page 211 of 1072

practice in previous years and was in violation of the by-laws (of 1975)
and unlawfully deprive[d] Grace Christian High School of its vested right [to]
a permanent seat in the board.
As the association denied its request, the school brought suit for
mandamus in the Home Insurance and Guaranty Corporation to compel the
board of directors of the association to recognize its right to a permanent
seat in the board.
ISSUE/S:
Whether petitioner has already acquired a vested right to a
permanent seat in the Board of Directors of Grace Village
Association.
RULING:
The board of directors of corporations must be elected from among the
stockholders or members. There may be corporations in which there are
unelected members in the board but it is clear that in the examples cited by
petitioner the unelected members sit as ex officio members, i.e., by virtue of
and for as long as they hold a particular office. But in the case of petitioner,
there is no reason at all for its representative to be given a seat in the board.
Nor does petitioner claim a right to such seat by virtue of an office held. In
fact it was not given such seat in the beginning. It was only in 1975 that a
proposed amendment to the by-laws sought to give it one.
Since the provision in question is contrary to law, the fact that for
fifteen years it has not been questioned or challenged but, on the contrary,
appears to have been implemented by the members of the association
cannot forestall a later challenge to its validity. Neither can it attain validity
through acquiescence because, if it is contrary to law, it is beyond the power
of the members of the association to waive its invalidity. For that matter the
members of the association may have formally adopted the provision in
question, but their action would be of no avail because no provision of the
by-laws can be adopted if it is contrary to law.
It is probable that, in allowing petitioners representative to sit on the
board, the members of the association were not aware that this was contrary
to law. It should be noted that they did not actually implement the provision
in question except perhaps insofar as it increased the number of directors
from 11 to 15, but certainly not the allowance of petitioners representative
as an unelected member of the board of directors. It is more accurate to say
that the members merely tolerated petitioners representative and tolerance
cannot be considered ratification.
Nor can petitioner claim a vested right to sit in the board on the basis
of practice. Practice, no matter how long continued, cannot give rise to any

Page 212 of 1072

vested right if it is contrary to law. Even less tenable is petitioners claim that
its right is ]oterminous with the existence of the association.

TOPIC: QUALIFICATIONS/QUALIFYING SHARE


RAMON C. LEE and ANTONIO DM. LACDAO, petitioners,
vs.
THE HON. COURT OF APPEALS, SACOBA MANUFACTURING CORP.,
PABLO GONZALES, JR. and THOMAS GONZALES, respondents.
G.R. No. 93695. February 4, 1992
205 SCRA 752
FACTS:
On November 15, 1985, a complaint for a sum of money was filed by
the International Corporate Bank, Inc. Against the private respondents who,
in turn, filed a third party complaint against ALFA and the petitioners on
March 17, 1986.
On September 17, 1987, the petitioners filed a motion to dismiss the
third party complaint which the Regional Trial Court of Makati, Branch 58
denied in an Order dated June 27, 1988.
On July 18, 1988, the petitioners filed their answer to the third party
complaint. Meanwhile, on July 12, 1988, the trial court issued an order
requiring the issuance of an alias summons upon ALFA through the DBP as a
consequence of the petitioners letter informing the court that the summons
for ALFA was erroneously served upon them considering that the
management of ALFA had been transferred to the DBP.
In a manifestation dated July 22, 1988, the DBP claimed that it was not
authorized to receive summons on behalf of ALFA since the DBP had not
taken over the company which has a separate and distinct corporate
personality and existence.
ISSUE:
Whether the execution of the voting trust agreement between
petitioners and the other stockholders of ALFA, as one party, and
the DBP, as the other party, the former assigned and transferred all
their shares in ALFA to DBP, as trustee.

Page 213 of 1072

RULING:
The facts of this case show that the petitioners, by virtue of the voting
trust agreement executed in 1981 disposed of all their shares through
assignment and delivery in ]avour of the DBP, as trustee. Consequently, the
petitioners ceased to own at least one share standing in their names on the
books of ALFA as required under Section 23 of the new Corporation Code.
They also ceased to have anything to do with the management of the
enterprise. The petitioners ceased to be directors. Hence, the transfer of the
petitioners shares to the DBP created vacancies in their respective positions
as directors of ALFA. The transfer of shares from the stockholder of ALFA to
the DBP is the essence of the subject voting trust agreement as evident from
the following stipulations:
1. The TRUSTORS hereby assign and deliver to the TRUSTEE
the certificate of the shares of the stocks owned by them
respectively and shall do all things necessary for the
transfer of their respective shares to the TRUSTEE on the
books of ALFA.
2. The TRUSTEE shall issue to each of the TRUSTORS a trust
certificate for the number of shares transferred, which shall
be transferrable in the same manner and with the same
effect as certificates of stock subject to the provisions of
this agreement;
3. The TRUSTEE shall vote upon the shares of stock at all
meetings of ALFA, annual or special, upon any resolution,
matter or business that may be submitted to any such
meeting, and shall possess in that respect the same
powers as owners of the equitable as well as the legal title
to the stock;
4. The TRUSTEE may cause to be transferred to any person
one share of stock for the purpose of qualifying such
person as director of ALFA, and cause a certificate of stock
evidencing the share so transferred to be issued in the
name of such person;
xxx xxx xxx
9. Any stockholder not entering into this agreement may
transfer his shares
to the same trustees without the need of revising this
agreement, and this agreement shall have the same force
and effect upon that said stockholder. (CA Rollo, pp. 137138; Emphasis supplied)
Considering that the voting trust agreement between ALFA and the
DBP transferred legal ownership of the stock covered by the agreement to
the DBP as trustee, the latter became the stockholder of record with respect

Page 214 of 1072

to the said shares of stocks. In the absence of a showing that the DBP had
caused to be transferred in their names one share of stock for the purpose of
qualifying as directors of ALFA, the petitioners can no longer be deemed to
have retained their status as officers of ALFA which was the case before the
execution of the subject voting trust agreement. There appears to be no
dispute from the records that DBP has taken over full control and
management of the firm.

TOPIC: BOARD OF DIRECTORS - DISQUALIFICATIONS


ENRIQUE P. BRIAS Y ROXAS, petitioner,
vs.
JOHN S. HORD, ET AL., respondents.
G.R. No. L-8387, February 5, 1913
24 PHIL. 286
FACTS:
Respondent, John S. Hord, has been the duly elected, qualified, and the
acting president of the Bank of the Philippine Islands while petitioner was
duly elected and appointed as a member of the committee of credits of said
board of directors. He made application to respondent for authority and
opportunity to examine and inspect the books of account of said corporation
then and there in the possession and under the immediate control of said
respondent which was denied repeatedly.
It is claimed by the respondents that the petitioner did, on the 26th of
September, 1912, resign, voluntarily, unequivocally, and absolutely, as a
member of said board of directors which petitioner refuted.
ISSUE/S:
Whether or not petitioner is disqualified to be a member of the
board of directors of the respondent bank having resigned from his
post.
RULING:
No.
It is not disputed that a resignation per verba is just as effective and
binding as a resignation per scripta. In this case, it will be noted that no

Page 215 of 1072

words are here attributed to the petitioner which indicate that he then and
there absolutely and unequivocally resigned. The most that can be said is
that he "ceased to attend its meetings." If the petitioner had resigned, at the
time and in the manner alleged, then he had forfeited his right to act in any
relation with the board. His resignation per verba was sufficient. No formal
acceptance of his resignation was necessary; neither was it necessary to
make an entry thereof in the minutes of the board. While this is true, it must,
however, appear that he positively and affirmatively stated or indicated that
it was his intention to resign then and there.

TOPIC:ELECTION, VOTING
WOLRGANG AURBACH, JOHN GRIFFIN, DAVID P. WHITTINGHAM AND
CHARLES CHAMSAY
VS.
SANITARY WARES MANUFACTURING CORPORATOIN, ERNESTO V.
LAGDAMEO, ERNESTO R. LAGDAMEO, JR., ENRIQUE R. LAGDAMEO,
GEORGE F. LEE, RAUL A. BONCAN, BALDWIN YOUNG AND AVELINO V.
CRUZ
G.R. NO. 75875, DECEMBER 15, 1989
180 SCRA 131
FACTS:
Saniwares is a domestic corporation incorporated for the primary
purpose of manufacturing and marketing sanitary wares. It was composed of
Filipino investors and ASI, an American corporation as stockholders. In the
election of its Board of Directors, it was agreed that as long as AmericanStandard shall own at least 30% of the outstanding stock of the Corporation,
three of the nine directors shall be designated by American-Standard, and
the others six: shall be designated by the Filipino stockholders of the
Corporation.
In the election of its Board of Directors, the Secretary then certified for
the election of the following ---- Wolfgang Aurbach, John Griffin, David
Whittingham, Ernesto Lagdameo, Sr., Ernesto Lagdameo, Jr., Enrique
Lagdameo, George F. Lee, Raul A. Boncan, Baldwin Young. Because of
disagreement, the ASI Group conducted a second election where Wolfgang
Aurbach, John Griffin, David Whittingham and Charles Chamsay were
nominated, Luciano E. Salazar voted for himself, thus the said five directors
were certified as elected directors by the Acting Secretary, Andres
Gatmaitan.

Page 216 of 1072

ISSUE/S:
Who are the elected officers of the business?
RULING:
Equally important as the consideration of the contractual intent of the
parties is the consideration as regards the possible domination by the foreign
investors of the enterprise in violation of the nationalization requirements
enshrined in the Constitution and circumvention of the Anti-Dummy Act. In
this regard, petitioner Salazar's position is that the Anti-Dummy Act allows
the ASI group to elect board directors in proportion to their share in the
capital of the entity. It is to be noted, however, that the same law also limits
the election of aliens as members of the board of directors in proportion to
their allowance participation of said entity. In the instant case, the foreign
Group ASI was limited to designate three directors. This is the allowable
participation of the ASI Group. Hence, in future dealings, this limitation of six
to three board seats should always be maintained as long as the joint
venture agreement exists considering that in limiting 3 board seats in the 9man board of directors there are provisions already agreed upon and
embodied in the parties' Agreement to protect the interests arising from the
minority status of the foreign investors.
With these findings, we the decisions of the SEC Hearing Officer and
SEC which were impliedly affirmed by the appellate court declaring Messrs.
Wolfgang Aurbach, John Griffin, David P Whittingham, Emesto V. Lagdameo,
Baldwin young, Raul A. Boncan, Emesto V. Lagdameo, Jr., Enrique Lagdameo,
and George F. Lee as the duly elected directors of Saniwares at the March
8,1983 annual stockholders' meeting.

Page 217 of 1072

TOPIC:ELECTION, VOTING
BATAAN SHIPYARD & ENGINEERING CO., INC. (BASECO)
Vs.
PRESIDENTIAL COMMISSION ON GOOD GOVERNMENT, CHAIRMAN
JOVITO SALONGA, COMMISSIONER MARY CONCEPCION BAUTISTA,
COMMISSIONER RAMON DIAZ, COMMISSIONER RAUL R. DAZA,
COMMISSIONER QUINTIN S. DOROMAL, CAPT. JORGE B. SIACUNCO
G.R. No. 75885 May 27, 1987
FACTS:
After the Marcos Regime, and under the Aquino Administration, the
President, Corazon C. Aquino, promulgated Executive Orders number 1 and 2
on February 28, 1986 and March 12, 1986 ordering the sequestration,
provisional take over freezing and recovery of al the ill-gotten property
amassed by the leaders and supporters of the previous regime.
Under said E.O., the PCGG undertook to sequester and provisionally
take over Bataan shipyard and Engineering Co.. Inc., a corporation known to
be an asset of the Marcos. The PCGG ordered the said corporation to
produce certain documents. It likewise took over the execution and carrying
out of the business, as well as making executive decisions in behalf of the
corporation including the pulling out of certain business transactions entered
into by the corporation.
In a special civil action filed by BASECO, the latter assailed E.O.s 1
and 2 contending that both are unconstitutional. It further contended that
the acts of PCGG in sequestering and provisionally taking over the
corporation are not valid and is in fact illegal on the following grounds,

Page 218 of 1072

1. no notice and hearing was accorded to it before its


properties and business prerogatives were taken over by
the PCGG,
2. PCGG is not a court, it is a mere investigative agency, thus,
it cannot competently act as a prosecutor and a judge in
the same cause,
3. PCGG illegally interfered with BASECOs right of dominion
and management of its business affairs.
ISSUE/S:
Whether or not the PCGG may properly exercise the prerogative to
vote sequestered stock of corporations

RULING:
It is within the parameters of these conditions and circumstances that
the PCGG may properly exercise the prerogative to vote sequestered stock of
corporations, granted to it by the President of the Philippines through a
Memorandum dated June 26, 1986. That Memorandum authorizes the PCGG,
"pending the outcome of proceedings to determine the ownership of * *
(sequestered) shares of stock," "to vote such shares of stock as it may have
sequestered in corporations at all stockholders' meetings called for the
election of directors, declaration of dividends, amendment of the Articles of
Incorporation, etc." The Memorandum should be construed in such a manner
as to be consistent with, and not contradictory of the Executive Orders
earlier promulgated on the same matter. There should be no exercise of the
right to vote simply because the right exists, or because the stocks
sequestered constitute the controlling or a substantial part of the corporate
voting power. The stock is not to be voted to replace directors, or revise the
articles or by-laws, or otherwise bring about substantial changes in policy,
program or practice of the corporation except for demonstrably weighty and
defensible grounds, and always in the context of the stated purposes of
sequestration or provisional takeover, i.e., to prevent the dispersion or undue
disposal of the corporate assets. Directors are not to be voted out simply
because the power to do so exists. Substitution of directors is not to be done
without reason or rhyme, should indeed be shunned if at an possible, and
undertaken only when essential to prevent disappearance or wastage of
corporate property, and always under such circumstances as assure that the
replacements are truly possessed of competence, experience and probity.
In the case at bar, there was adequate justification to vote the
incumbent directors out of office and elect others in their stead because the
evidence showed prima facie that the former were just tools of President
Marcos and were no longer owners of any stock in the firm, if they ever were
at all. This is why, in its Resolution of October 28, 1986; this Court declared

Page 219 of 1072

that Petitioner has failed to make out a case of grave abuse or excess of
jurisdiction in respondents' calling and holding of a stockholders' meeting for
the election of directors as authorized by the Memorandum of the President *
* (to the PCGG) dated June 26, 1986, particularly, where as in this case, the
government can, through its designated directors, properly exercise control
and management over what appear to be properties and assets owned and
belonging to the government itself and over which the persons who appear
in this case on behalf of BASECO have failed to show any right or even any
shareholding in said corporation.

TOPIC:REPORT ON ELECTION
PREMIUM MARBLE RESOURCES, INC.
VS.
THE COURT OF APPEALS AND INTERNATIONAL CORPORATE
BANK, PRINTLINE CORPORATION V. THE COURT OF APPEALS AND
INTERNATIONAL CORPORATE BANK
G.R. NO. 96551 NOVEMBER 4, 1996
FACTS:
On Aug. to Oct. 1982, Ayala Investment and Development Corporation
issued 3 checks in the total amount of P31,663.88 payable to Premium.
Former officers of the Premium headed by Saturnino Belen Jr. without any
authority from the Premium deposited the checks to his conduit corporation
Intervest Merchant Finance. Even though the checks were payable to
premium, International Corporate Bank cleared the checked in Intervests
favor and allowed the latter to use the funds. Thus Premium filed an action
for damages assisted by Atty. Dumadag.
Subsequently after, Premium represented by the Siguion Reyna Law
Firm filed a motion to dismiss the complaint claiming that it was filed without
the authority of the BOD of Premium. Atty. Dumadag claimed that the MOD
was signed by Belen, Jr., Nograles and Reyes who are not directors of the
corporation but were former officers dismissed for various irregularities and
fraudulent acts. The Sigion Reyna law firm claimed that it should be the
general information sheet filed with the SEC that is the best evidence of who
are the stockholders and not the AI. The LC that since no officers have yet
been elected and qualified the officers of Premium are Nograles, Belen and
Reyes and therefore those represented by Atty. Dumadag have yet no legal
capacity to file the case. The CA affirmed the decision.

Page 220 of 1072

ISSUE/S:
Whether or not the filing of the case for damages against private
respondent was authorized by a duly constituted Board of Directors
of the petitioner corporation.
RULING:
By the express mandate of the Corporation Code (Section 26), all
corporations duly organized pursuant thereto are required to submit within
the period therein stated (30 days) to the Securities and Exchange
Commission the names, nationalities and residences of the directors,
trustees and officers elected. Sec. 26 of the Corporation Code provides, thus:
Sec. 26. Report of election of directors, trustees and officers. Within thirty
(30) days after the election of the directors, trustees and officers of the
corporation, the secretary, or any other officer of the corporation, shall
submit to the Securities and Exchange Commission, the names, nationalities
and residences of the directors, trustees and officers elected. . . .
Evidently, the objective sought to be achieved by Section 26 is to give
the public information, under sanction of oath of responsible officers, of the
nature of business, financial condition and operational status of the company
together with information on its key officers or managers so that those
dealing with it and those who intend to do business with it may know or have
the means of knowing facts concerning the corporation's financial resources
and business responsibility.
The claim, therefore, of petitioners as represented by Atty. Dumadag,
that Zaballa, et al., are the incumbent officers of Premium has not been fully
substantiated. In the absence of an authority from the board of directors, no
person, not even the officers of the corporation, can validly bind the
corporation.

Page 221 of 1072

TOPIC:TERM OF OFFICE/HOLD OVER


DR. HANS CHRISTIAN M. SEERES
VS.
COMMISSION ON ELECTIONSAND MELQUIADES A. ROBLES
G.R. NO. 178678, APRIL 16, 2009
FACTS:
Robles was elected president and chairperson of Buhay, a party-list
group duly registered with COMELEC. The constitution of BUHAY provides for
a three-year term for all its party officers, without re-election. BUHAY
participated in the 2001 and 2004 elections, with Robles as its president. All
the required Manifestations of Desire to Participate in the said electoral
exercises, including the Certificates of Nomination of representatives, carried
the signature of Robles as president of BUHAY.
On March 29, 2007, Robles signed and filed a Certificate of Nomination
of BUHAYs nominees for the 2007 elections containing the following names:
(i) Rene M. Velarde, (ii) Ma. Carissa Coscolluela, (iii) William Irwin C. Tieng,
(iv) Melchor R. Monsod, and (v) Teresita B. Villarama. Earlier, however, or on
March 27, 2007, petitioner Hans Christian Seeres, holding himself up as
acting president and secretary-general of BUHAY, also filed a Certificate of
Nomination with the COMELEC, nominating: (i) himself, (ii) Hermenegildo C.
Dumlao, (iii) Antonio R. Bautista, (iv) Victor Pablo C. Trinidad, and (v) Eduardo
C. Solangon, Jr.
Seeres filed with the COMELEC a Petition to Deny Due Course to
Certificates of Nomination, alleged that he was the acting president and
secretary-general of BUHAY, having assumed that position since August 17,

Page 222 of 1072

2004 when Robles vacated the position. Pushing the point, Seeres would
claim that the nominations made by Robles were, for lack of authority, null
and void owing to the expiration of the latters term as party president.
Furthermore, Seeres asserted that Robles was, under the Constitution,
disqualified from being an officer of any political party, the latter being the
Acting Administrator of the Light Railway Transport Authority (LRTA), a
government-controlled corporation. Robles, so Seeres would charge, was
into a partisan political activity which civil service members, like the former,
were enjoined from engaging in.
ISSUE/S:
Whether or not Robles has the right to assume the presidency in
hold over capacity?

RULING:
As a general rule, officers and directors of a corporation hold over after
the expiration of their terms until such time as their successors are elected
or appointed. Sec. 23 of the Corporation Code contains a provision to this
effect, thus: Section 23. The board of directors or trustees.Unless otherwise
provided in this Code, the corporate powers of all corporations formed under
this Code shall be exercised, all business conducted and all property of such
corporations controlled and held by the board of directors or trustees to be
elected from among the holders of stocks, or where there is no stock, from
among the members of the corporation, who shall hold office for one (1) year
until their successors are elected and qualified.
The holdover doctrine has, to be sure, a purpose which is at once legal
as it is practical. It accords validity to what would otherwise be deemed as
dubious corporate acts and gives continuity to a corporate enterprise in its
relation to outsiders. This is the analogical situation obtaining in the present
case. The voting members of BUHAY duly elected Robles as party President
in October 1999. And although his regular term as such President expired in
October 2002, no election was held to replace him and the other original set
of officers. Further, the constitution and by-laws of BUHAY do not expressly or
impliedly prohibit a hold-over situation. As such, since no successor was ever
elected or qualified, Robles remained the President of BUHAY in a "hold-over"
capacity.

Page 223 of 1072

TOPIC:HOW REMOVED
LEON J. LAMBERT, plaintiff-appellant,
vs.
T. J. FOX, defendant-appellee.
G.R. NO. L-7991, JANUARY 29, 1914
FACTS:
John R. Edgar & Co., engaged in the retail book and stationery
business, found itself in such condition financially that its creditors agreed to
take over the business, incorporate it and accept stock therein in payment of
their respective credits. This was done, the plaintiff and the defendant
becoming the two largest stockholders in the new corporation called John R.
Edgar & Co., Incorporated. A few days after the incorporation was completed
plaintiff and defendant entered into an agreement whereby the shockholders
mutually and reciprocally agree not to sell, transfer, or otherwise dispose of
any part of their present holdings of stock in said John R. Edgar & Co. Inc., till
after 1 year from the date hereof and that Either party violating this
agreement shall pay P1000.00 as liquidated damages, unless previous
consent in writing to such sale, transfer, or other disposition be obtained.
Notwithstanding this contract, Fox sold his stock in the said corporation
to E. C. McCullough of the firm of E. C. McCullough & Co. of Manila, a strong
competitor of the said John R. Edgar & Co., Inc. This sale was made by the
defendant against the protest of the plaintiff and with the warning that he
would be held liable under the contract hereinabove set forth and in
accordance with its terms. In fact, the defendant Foz offered to sell his shares

Page 224 of 1072

of stock to the plaintiff for the same sum that McCullough was paying them
less P1,000, the penalty specified in the contract.
ISSUE/S:
Whether or not the suspension of the power to sell the stock is valid
and legal.
RULING:
In this jurisdiction penalties provided in contracts of this character are
enforced . It is the rule that parties who are competent to contract may make
such agreements within the limitations of the law and public policy as they
desire, and that the courts will enforce them according to their terms. (Civil
Code, articles 1152, 1153, 1154, and 1155; Fornow vs. Hoffmeister, 6 Phil.
Rep., 33; Palacios vs. Municipality of Cavite, 12 Phil. Rep., 140; Gsell vs.
Koch, 16 Phil. Rep., 1.) The only case recognized by the Civil Code in which
the court is authorized to intervene for the purpose of reducing a penalty
stipulated in the contract is when the principal obligation has been partly or
irregularly fulfilled and the court can see that the person demanding the
penalty has received the benefit of such or irregular performance. In such
case the court is authorized to reduce the penalty to the extent of the
benefits received by the party enforcing the penalty.
In this jurisdiction, there is no difference between a penalty and
liquidated damages, so far as legal results are concerned. Whatever
differences exists between them as a matter of language, they are treated
the same legally. In either case the party to whom payment is to be made is
entitled to recover the sum stipulated without the necessity of proving
damages. Indeed one of the primary purposes in fixing a penalty or in
liquidating damages, is to avoid such necessity.
It is also urged by the appelle in this case that the stipulation in the
contract suspending the power to sell the stock referred to therein is an
illegal stipulation, is in restraint of trade and, therefore, offends public policy.
We do not so regard it. The suspension of the power to sell has a beneficial
purpose, results in the protection of the corporation as well as of the
individual parties to the contract, and is reasonable as to the length of time
of the suspension. We do not here undertake to discuss the limitations to the
power to suspend the right of alienation of stock, limiting ourselves to the
statement that the suspension in this particular case is legal and valid.
The judgment is reversed, the case remanded with instructions to
enter a judgment in favor of the plaintiff and against the defendant for
P1,000, with interest; without costs in this instance.

Page 225 of 1072

TOPIC: BOARD OF DIRECTORS HOW VACANCY IS FILLED


VALLE VERDE COUNTRY CLUB, INC., ERNESTO VILLALUNA, RAY
GAMBOA, AMADO M. SANTIAGO, JR., FORTUNATO DEE, AUGUSTO
SUNICO, VICTOR SALTA, FRANCISCO ORTIGAS III, ERIC ROXAS, in
their capacities as members of the Board of Directors of Valle Verde
Country Club, Inc., and JOSE RAMIREZ, Petitioners,
vs.
VICTOR AFRICA, Respondent.
G.R. NO. 151969, SEPTEMBER 4, 2009
FACTS:
On February 27, 1996, during the Annual Stockholders Meeting of
petitioner Valle Verde Country Club, Inc. (VVCC), the following were elected
as members of the VVCC Board of Directors: Ernesto Villaluna, Jaime C.
Dinglasan, Eduardo Makalintal, Francisco Ortigas III, Victor Salta, Amado M.
Santiago, Jr., Fortunato Dee, Augusto Sunico, and Ray Gamboa. In the years
1997, 1998, 1999, 2000, and 2001, however, the requisite quorum for the
holding of the stockholders meeting could not be obtained. Consequently,
the above-named directors continued to serve in the VVCC Board in a holdover capacity.
On September 1, 1998, Dinglasan resigned as member of the VVCC
Board. In a meeting held on October 6, 1998, the remaining directors, still
constituting a quorum of VVCCs nine-member board, elected Eric Roxas to
fill in the vacancy created by the Dinglasans resignation.

Page 226 of 1072

A year later, Makalintal also resigned as member of the VVCC Board.


He was replaced by Jose Ramirez, who was elected by the remaining
members of the VVCC Board on March 6, 2001.
Respondent Africa, a member of VVCC, questioned the election of
Roxas and Ramirez as members of the VVCC Board with the SEC and the RTC,
respectively on the grounds that the election of Roxas was contrary to
Section 29 of the Corporation Code and that Makalintals term as well as
those of the other members of the WCC board has already expired. Thus,
according to Africa, the resulting vacancy should have been filled by the
stockholders in a regular or special meeting called for that purpose, and not
by the remaining members of the VVCC Board, as was done in this case.
ISSUE/S:
Whether or not the election of Roxas and Makalintal to the VVCC
Board is valid.
RULING:
While the Court in El Hogar approved of the practice of the directors to
fill vacancies in the directorate, we point out that this ruling was made
before the present Corporation Code was enacted and before its Section 29
limited the instances when the remaining directors can fill in vacancies in the
board, i.e., when the remaining directors still constitute a quorum and when
the vacancy is caused for reasons other than by removal by the stockholders
or by expiration of the term.
It also bears noting that the vacancy referred to in Section 29
contemplates a vacancy occurring within the directors term of office. When
a vacancy is created by the expiration of a term, logically, there is no more
unexpired term to speak of. Hence, Section 29 declares that it shall be the
corporations stockholders who shall possess the authority to fill in a vacancy
caused by the expiration of a members term.
As correctly pointed out by the RTC, when remaining members of the
VVCC Board elected Ramirez to replace Makalintal, there was no more
unexpired term to speak of, as Makalintals one-year term had already
expired. Pursuant to law, the authority to fill in the vacancy caused by
Makalintals leaving lies with the VVCCs stockholders, not the remaining
members of its board of directors.

Page 227 of 1072

TOPIC: BOARD OF DIRECTORS HOW COMPENSATED


GABRIEL C. SINGSON, ANDRE NAVATO, EDGARDO P. ZIALCITA,
ARACELI E. VILLANUEVA, TYRONE M. REYES, JOSE CLEMENTE, JR.,
FEDERICO PASCUAL, ALEJANDRA C. CLEMENTE, ALBERT P. FENIX, JR.,
and MELPIN A. GONZAGA, Petitioners,
vs.
COMMISSION ON AUDIT, Respondent.
G.R. No. 159355, August 9, 2010
FACTS:
The Philippine International Convention Center, Inc. (PICCI) is a
government corporation whose sole stockholder is the Bangko Sentral ng
Pilipinas (BSP). Petitioner Araceli E. Villanueva was then a member of the
PICCI Board of Directors and Officer-in-Charge (OIC) of PICCI, while her copetitioners were then members of the PICCI Board of Directors and officials of
the BSP. By virtue of the PICCI By-Laws, petitioners were authorized to
receive P1,000.00 per diem each for every meeting attended. Pursuant to its
Monetary Board (MB) Resolution No. 15 dated January 5, 1994, as amended,
the BSP MB granted additional monthly RATA, in the amount of P1,500.00, to
each of the petitioners, as members of the Board of Directors of PICCI.
On June 7, 1999, then the PICCI Corporate Auditor issued a Notice of
Disallowance to Villanueva, disallowing in audit the payment of petitioners
RATA in the total amount of P1,565,000.00, and directing them to settle
immediately the said disallowances, on the ground of double compensation

Page 228 of 1072

due to the fact that they are already receiving compensation as officers of
the BSP which is contrary to law.
The petitioners filed a Motion for Reconsideration before the Corporate
Auditor but the same was denied. The said disallowance was subsequently
affirmed by COA.
ISSUE/S:
Whether or not respondent COA committed grave abuse of
discretion in finding that the petitioners violated its by-laws when
section 30 of the corporation code authorizes the stockholders to
grant compensation to its directors.
RULING:
Section 30 of the Corporation Code, which authorizes the stockholders
to grant compensation to its directors, states:
Sec. 30.Compensation of Directors. In the absence of any
provision in the by-laws fixing their compensation, the directors
shall not receive any compensation, as such directors, except for
reasonable per diems; Provided, however, that any such
compensation (other than per diems) may be granted to
directors by the vote of the stockholders representing at least a
majority of the outstanding capital stock at a regular or special
stockholders meeting.
In no case shall the total yearly
compensation of directors, as such directors, exceed ten (10%)
percent of the net income before income tax of the corporation
during the preceding year.
In construing the said provision, it bears stressing that the directors of
a corporation shall not receive any compensation for being members of the
board of directors, except for reasonable per diems. The two instances
where the directors are to be entitled to compensation shall be when it is
fixed by the corporations by-laws or when the stockholders, representing at
least a majority of the outstanding capital stock, vote to grant the same at a
regular or special stockholders meeting, subject to the qualification that, in
any of the two situations, the total yearly compensation of directors, as such
directors, shall in no case exceed ten (10%) percent of the net income before
income tax of the corporation during the preceding year.
Section 8 of the Amended By-Laws of PICCI, in consonance with Section
30 of the Corporation Code, restricted the scope of petitioners compensation
by fixing their per diem at P1,000.00.

Page 229 of 1072

MB Resolution No. 15, dated January 5, 1994, as amended by MB


Resolution No. 34, dated January 12, 1994, are valid corporate acts of
petitioners that became the bases for granting them additional monthly RATA
of P1,500.00, as members of the Board of Directors of PICCI. The RATA is
distinct from salary (as a form of compensation). Unlike salary which is paid
for services rendered, the RATA is a form of allowance intended to defray
expenses deemed unavoidable in the discharge of office. Hence, the RATA is
paid only to certain officials who, by the nature of their offices, incur
representation and transportation expenses. Indeed, aside from the RATA
that they have been receiving from the BSP, the grant of P1,500.00 RATA to
each of the petitioners for every board meeting they attended, in their
capacity as members of the Board of Directors of PICCI, in addition to their
P1,000.00 per diem, does not run afoul the constitutional proscription
against double compensation.TOPIC: BOARD OF DIRECTORS HOW
COMPENSATED
WESTERN INSTITUTE OF TECHNOLOGY, INC., HOMERO L. VILLASIS,
DIMAS ENRIQUEZ, PRESTON F. VILLASIS & REGINALD F. VILLASIS,
petitioner,
vs.
RICARDO T. SALAS, SALVADOR T. SALAS, SOLEDAD SALAS-TUBILLEJA,
ANTONIO S. SALAS, RICHARD S. SALAS & HON. JUDGE PORFIRIO
PARIAN, respondents.
G.R. No. 113032. August 21, 1997
278 SCRA 216
FACTS:
Private respondents, belonging to the same family, are the majority
and controlling members of the Board of Trustees of Western Institute of
Technology, Inc. In a board meeting, the Board of Trustees passed Resolution
No. 48, s. 1986, granting monthly compensation to the private respondents
as corporate officers.
On March 13, 1991, petitioners Homero Villasis, Prestod Villasis,
Reginald Villasis and Dimas Enriquez filed an affidavit-complaint against
private respondents before the Office of the City Prosecutor of Iloilo, as a
result of which two (2) separate criminal informations, one for falsification of
a public document and the other for estafa. The charge for falsification of
public document was anchored on the private respondents' submission of
WIT's income statement for the fiscal year 1985-1986 with the Securities and
Exchange Commission (SEC) reflecting therein the disbursement of corporate
funds for the compensation of private respondents based on Resolution No.
4, series of 1986, making it appear that the same was passed by the board
on March 30, 1986, when in truth, the same was actually passed on June 1,
1986, a date not covered by the corporation's fiscal year 1985-1986
(beginning May 1, 1985 and ending April 30, 1986). The respondents were

Page 230 of 1072

acquitted of both charges, without the imposition of any civil liability against
them. Thus, Petitioners filed a Motion for Reconsideration of the civil aspect
of the case, but was denied.
ISSUE/S:
Whether or not the said resolution granting monthly compensation
to the private respondents as corporate officers is valid.
RULING:
The pertinent section of the Corporation Code provides:
Sec. 30. Compensation of directors.--- In the absence of any
provision in the by-laws fixing their compensation, the directors
shall not receive any compensation, assuch directors, except for
reasonable per diems: Provided, however, That any such
compensation (other than per diems) may be granted to
directors by the vote of the stockholders representing at least a
majority of the outstanding capital stock at a regular or special
stockholders meeting.
In no case shall the total yearly
compensation of directors, as such directors, exceed ten(10%)
percent of the net income before income tax of the corporation
during the preceding year.
There is no argument that directors or trustees, as the case may be,
are not entitled to salary or other compensation when they perform nothing
more than the usual and ordinary duties of their office. This rule is founded
upon a presumption that directors /trustees render service gratuitously and
that the return upon their shares adequately furnishes the motives for
service, without compensation Under the foregoing section, there are only
two (2) ways by which members of the board can be granted compensation
apart from reasonable per diems:
1. when there is a provision in the by-laws fixing their
compensation; and
2. when the stockholders representing a majority of the
outstanding capital stock at a regular or special
stockholders meeting agree to give it to them.
This proscription, however, against granting compensation to
directors/trustees of a corporation is not a sweeping rule. Worthy of note is
the clear phraseology of Section 30 which states: xxx The directors shall not
receive any compensation, as such directors, xxx. The phrase as such
directors is not without significance for it delimits the scope of the prohibition
to compensation given to them for services performed purely in their
capacity as directors or trustees. The unambiguous implication is that
members of the board may receive compensation, in addition to reasonable
per diems, when they render services to the corporation in a capacity other
than as directors/trustees. In the case at bench, Resolution No. 48, s. 1986
granted monthly compensation to private respondents not in their capacity

Page 231 of 1072

as members of the board, but rather as officers of the corporation, more


particularly as Chairman, Vice-Chairman, Treasurer and Secretary of Western
Institute of Technology.
Clearly, therefore, the prohibition with respect to granting
compensation to corporate directors/trustees as such under Section 30 is not
violated in this particular case. Consequently, the last sentence of Section
30 which provides:
xxx xxx. In no case shall the total yearly compensation of
directors, as such directors, exceed ten (10%) percent of the net
income before income tax of the corporation during the
preceding year. does not likewise find application in this case
since the compensation is being given to private respondents in
their capacity as officers of WIT and not as board members.

TOPIC: BOARD OF DIRECTORS HOW COMPENSATED


CENTRAL COOPERATIVE EXCHANGE, INC.
VS.
CONCORDIO TIBE, SR. and THE HONORABLE COURT OF APPEALS
G.R. No. L-27972, June 30, 1970
33 SCRA 593
FACTS:
The petitioner is a national federation of farmers' cooperative
marketing associations, or FACOMAS, scattered throughout the country; its
single majority stockholder is the former Agricultural Credit and Cooperative
Financing Administration (ACCFA), now Agricultural Credit Administration
(ACA). As a member of the petitioner's board of directors from 23 May 1958
to 26 May 1960, representing FACOMAS in Eastern Visayas, respondent
Concordio Tibe, Sr. drew and collected from petitioner CCE cash advances
amounting to P5,668.00; of this sum, respondent had, admittedly, already
liquidated P3,317.25, leaving the sum of P2,350.75 still to be accounted for.
By admission of the petitioner the sum of P2,350.75 has been further
reduced to P2,133.45 as of 31 January 1963 on account of partial payments
made after suit was filed (Petitioner's Brief, page 17). Respondent Tibe had
also drawn several sums, amounting to P14,436.95, representing
commutable per diems for attending meetings of the Board of Directors in
Manila, per diems and transportation expenses for FACOMA visitations,
representation expenses and commutable discretionary funds. All of these
disbursements were based upon several resolutions adopted by CCEs Board

Page 232 of 1072

of Directors and these sums were disbursed with the approval of general
manager, treasurer and auditor of CCE.
ISSUE/S:
Whether or not the board of directors of the CCE had the power and
authority to adopt various resolutions which appropriated the funds
of the corporation for the above-enumerated expenses for the
members of the said board.
RULING:
Section 8 of the By-Laws of petitioner federation provides:
The compensation, if any, and the per diems for
attendance at meetings of the members of the Board of
Directors shall be determined by the members at any
annual meeting or special meeting of the Exchange called
for the purpose.
The Supreme Court agrees with the petitioner that the questioned
resolutions are contrary to the By-Laws of the federation and, therefore, are
not within the power of the board of directors to enact. The By-Laws, in the
aforequoted Section 8, explicitly reserved unto the stockholders the power to
determine the compensation of members of the board of directors, and the
stockholders did restrict such compensation to "actual transportation
expenses plus the per diems of P30.00 and actual expenses while waiting."
Even without the express reservation of said power, the directors are not
entitled to compensation, for
... The law is well-settled that directors of corporations
presumptively serve without compensation and in the absence of
an express agreement or a resolution in relation thereto, no
claim can be asserted therefore (Sec. 2110, 5 Fletcher 375-376).
Thus it has been held that there can be no recovery of
compensation, unless expressly provided for, when a director
serves as president or vice president, as secretary, as treasurer
or cashier, as a member of an executive committee, as chairman
of a building committee, or similar offices (Sec. 2112, 5, Fletcher
381-382). (Alvendia, The Law of Private Corporations in the
Philippines, pages 275-276)
Thus, the directors, in assigning themselves additional duties, such as
the visitation of FACOMAS, acted within their power, but, by voting for
themselves compensation for such additional duties, they acted in excess of
their authority, as expressed in the By-Laws.

Page 233 of 1072

TOPIC: BOARD OF DIRECTORS HOW COMPENSATED


LINGAYEN GULF ELECTRIC POWER COMPANY, INC.
VS.
IRINEO BALTAZAR
G.R. NO. L-4824, June 30, 1953
93 Phil 404
FACTS:
Lingayen Gulf Electric Power Company is a domestic corporation with
an authorized capital stock of P300, 000 divided into 3,000 shares with a par
value of P100 per share to which the defendant, Irineo Baltazar appears to
have subscribed for 600 shares on account of which he had paid upon the
organization of the corporation the sum of P15,000. After incorporation, the
defendant made further payments on account of his subscription, leaving a
balance of P18,500 unpaid for.
On September 28, 1949, the legal counsel of the plaintiff corporation
wrote a letter to the defendant, demanding the payment of the unpaid
balance of his subscription amounting to P18,500. Copy of this letter was
sent by registered mail to the defendant on September 29,1 949. The
defendant ignored the said demand. Thus, Lingayen instituted a civil suit
against Baltazar for collection of sum of money. Defendant on the other
hand, included in his answer a counterclaim against Lingayen claiming from
the plaintiff a reasonable compensation at the rate of P700 per month as

Page 234 of 1072

president of the company, for the period from March 1, 1946 to December
31, 1948.
ISSUE/S:
Whether or not the defendant is entitled to compensation as
president of the plaintiff corporation.
RULING:
No.
As regards the compensation of President claimed by defendant and
appellant, it is clear that he is not entitled to the same. The by-laws of the
company are silent as to the salary of the President. And, while resolutions of
the incorporators and stockholders provide salaries for the general manager,
secretary-treasurer and other employees, there was no provision for the
salary of the President. On the other hand, other resolutions provide for per
diems to be paid to the President and the directors of each meeting
attended, P10 for the President and P8 for each director, which were later
increased to P25 and P15, respectively. This leads to the conclusions that the
President and the board of directors were expected to serve without salary,
and that the per diems paid to them were sufficient compensation for their
services. Furthermore, for defendant's several years of service as President
and up to the filing of the action against him, he never filed a claim for
salary. He thought of claiming it only when this suit was brought against him.

Page 235 of 1072

TOPIC: AUTHORITY OF THE BOARD OF DIRECTORS


LA BUGAL-BLAAN TRIBAL ASSOCIATION, INC. et al.
vs.
VICTOR O. RAMOS, Secretary, Department of Environment and
Natural Resources (DENR), et al.
G.R. No. 127882
December 1, 2004
FACTS:
RA 7942 (The Philippine Mining Act) took effect on April 9, 1995.
Before the effectivity of RA 7942, or on March 30, 1995, the President signed
a Financial and Technical Assistance Agreement (FTAA) with WMCP, a
corporation organized under Philippine laws, covering close to 100, 000
hectares of land in South Cotabato, Sultan Kudarat, Davao del Sur and North
Cotabato.
On August 15, 1995, the DENR Secretary Victor Ramos issued DENR
Administrative Order 95-23, which was later repealed by DENR
Administrative Order 96-40, adopted on December 20, 1996.
Petitioners prayed that RA 7942, its implementing rules, and the FTAA
between the government and WMCP be declared unconstitutional on ground
that they allow fully foreign owned corporations like WMCP to exploit, explore
and develop Philippine mineral resources in contravention of Article XII
Section 2 paragraphs 2 and 4 of the Charter.

Page 236 of 1072

In January 2001, WMC a publicly listed Australian mining and


exploration company sold its whole stake in WMCP to Sagittarius Mines,
60% of which is owned by Filipinos while 40% of which is owned by Indophil
Resources, an Australian company. DENR approved the transfer and
registration of the FTAA in Sagittarius name but Lepanto Consolidated
assailed the same. The latter case is still pending before the Court of
Appeals.
WMCP then points out that the original claim owners of the major
mineralized areas included in the WMCP FTAA, namely, Sagittarius,
Tampakan Mining Corporation, and Southcot Mining Corporation, are all
Filipino-owned corporations, each of which was a holder of an approved
Mineral Production Sharing Agreement awarded in 1994, albeit their
respective mineral claims were subsumed in the WMCP FTAA; and that these
three companies are the same companies that consolidated their interests in
Sagittarius to whom WMC sold its 100% equity in WMCP. WMCP concludes
that in the event that the FTAA is invalidated, the MPSAs of the three
corporations would be revived and the mineral claims would revert to their
original claimants.
ISSUE/S:
Whether or not the sale of the shares of stocks of WMC to
Sagittarius is valid.
RULING:
Yes.
Section 40 expressly applies to the assignment or transfer of the FTAA,
not to the sale and transfer of shares of stock in WMCP as contended by the
petitioners.
Moreover,
when
the
transferee
of
an
FTAA
is
another foreign corporation, there is a logical application of the requirement
of prior approval by the President of the Republic and notification to
Congress in the event of assignment or transfer of an FTAA. In this situation,
such approval and notification are appropriate safeguards, considering that
the new contractor is the subject of a foreign government.
On the other hand, when the transferee of the FTAA happens to be
a Filipino corporation, the need for such safeguard is not critical; hence, the
lack of prior approval and notification may not be deemed fatal as to render
the transfer invalid. In this case, since the transfer was made in favor of a
Filipino corporation, prior approval by the President and notification to
Congress is not really required.

Page 237 of 1072

TOPIC: AUTHORITY OF THE BOARD OF DIRECTORS


SHIPSIDE INCORPORATED
vs.
THE HON. COURT OF APPEALS [Special Former Twelfth Division],
HON. REGIONAL TRIAL COURT, BRANCH 26 (San Fernando City, La
Union) & The REPUBLIC OF THE PHILIPPINES
GR 143377
February 20, 2001
352 SCRA 334
FACTS:
The petitioner filed a certiorari with the CA containing the requisite
certification on non-forum shopping. However, the CA dismissed the petition
on the ground that Lorenzo Balbin, the resident manager for petitioner, who
was the signatory in the verification and certification on non-forum shopping,
failed to show proof that he was authorized by the petitioners board of
directors to file such a petition. The petitioner submits a motion for
reconsideration which attached a secretarys certificate attesting to the
signatorys authority to sign certificates against forum shopping on behalf of
the petitioner. When the court of CA denied the motion, the petitioner sought
relief with the SC.
ISSUE/S:
Whether or not an authorization from petitioners Board of Directors
is still required in order for its resident manager to institute or
commence a legal action for and in behalf of the corporation.

Page 238 of 1072

RULING:
No.
The SC revised the decision of CA recognizing the belated filing of the
certifications against forum shopping as permitted in exceptional
circumstances. It further held that with more reason should a petition be
given due course when this incorporates a certification on non-forum
shopping without evidence that the person signing the certifications was an
authorized signatory and the petitioner subsequently submits a secretarys
certificate attesting to the signatorys authority in its motion for
consideration. The court allows belated submission of certifications showing
proof of the signatorys authority in signing the certification of forum
shopping.
In the instant case, the merits of petitioners case should be considered
special circumstances or compelling reasons that justify tempering the
requirement in regard to the certificate of non-forum shopping. Moreover,
in Loyola, Roadway, and Uy, the Court excused non-compliance with the
requirement as to the certificate of non-forum shopping. With more reason
should we allow the instant petition since petitioner herein did submit a
certification on non-forum shopping, failing only to show proof that the
signatory was authorized to do so. That petitioner subsequently submitted a
secretarys certificate attesting that Balbin was authorized to file an action
on behalf of petitioner likewise mitigates this oversight.

Page 239 of 1072

TOPIC: AUTHORITY OF THE BOARD OF DIRECTORS


ABS-CBN BROADCASTING CORPORATION
vs.
HONORABLE COURT OF APPEALS, REPUBLIC BROADCASTING CORP,
VIVA PRODUCTION, INC., and VICENTE DEL ROSARIO
G.R. No. 128690
January 21, 1999
301 SCRA 572
FACTS:
In
1990, ABS-CBN
and Viva
executed
a Film
Exhibition Agreement whereby ABS-CBN was given the right of first refusal to
the next twenty-four (24) Viva films for TV telecast under such terms as may
be agreed upon by the parties hereto, provided, however, that such right
shall be exercised by ABS-CBN from the actual offer in writing. Consequently,
Viva, through defendant Del Rosario, offered ABS-CBN, through its vicepresident Charo Santos-Concio, a list of three(3) film packages (36 titles)
from which ABS-CBN may exercise its right of first refusal under the aforesaid agreement. ABS CBN rejected said list.
On February 27, 1992, Del Rosario approached Ms. Concio, with
a list consisting of 52 original movie titles, as well as 104 re-runs from which
ABS-CBN may choose another 52 titles, or a total of 156 titles, proposing to
sell to ABS-CBN airing rights over this package of 52 originals and 52 re-runs
forP60,000,000.00. The package was rejected by ABS-CBN. On April 06,

Page 240 of 1072

1992, Del Rosario and Mr. Graciano Gozon of RBS discussed the terms and
conditions of Viva's offer to sell the 104 films.
On April 07, 1992, defendant Del Rosario received through his
secretary, a handwritten note from Ms. Concio which reads: "Here's the draft
of the contract. I hope you find everything in order," to which was attached a
draft exhibition agreement, a counter-proposal covering 53 films for a
consideration of P35million. The said counter-proposal was however rejected
by Viva's Board of Directors. On April 29, 1992, Viva granted RBS the
exclusive right to air 104 Viva-produced and/or acquired films including the
fourteen (14) films subject of the present case.
ABS-CBN then filed a complaint for specific performance. RTC rendered
a decision in favor of RBS and VIVA and against ABS-CBN, ruling that there
was no meeting of minds on the price and terms of the offer. Furthermore,
the right of first refusal under the 1990 Film Exhibition Agreement had
previously been exercised per Ms. Concio's letter to Del Rosario ticking off
ten titles acceptable to them, which would have made the 1992 agreement
an entirely new contract. The Court of Appeals affirmed the decision of the
RTC.
ISSUE/S:
Whether or not there was no perfected contract between petitioner
and private respondent.
RULING:
No.
When Mr. Del Rosario of VIVA met with Mr. Lopez of ABS-CBN at the
Tamarind Grill on 2 April 1992to discuss the package of films, said package of
104 VIVA films was VIVAs offer to ABS-CBN to enter into a new Film
Exhibition Agreement. But ABS-CBN, sent, through Ms. Concio, a counterproposal in the form of a draft contract proposing exhibition of 53 films for a
consideration of P35 million. This counter-proposal could be nothing less than
the counter-offer of Mr. Lopez during his conference with Del Rosario at
Tamarind Grill Restaurant. Clearly, there was no acceptance of VIVAs offer,
for it was met by a counter-offer which substantially varied the terms of the
offer. Even if it be concededarguendo that Del Rosario had accepted the
counter-offer, the acceptance did not bind VIVA, as there was no proof
whatsoever that Del Rosario had the specific authority to do so. That Del
Rosario did not have the authority to accept ABS-CBNs counter-offer was
best evidenced by his submission of the draft contract to VIVAs Board of
Directors for the latters approval. In any event, there was between Del
Rosario and Lopez III no meeting of minds.

Page 241 of 1072

TOPIC: AUTHORITY OF THE BOARD OF DIRECTORS


ASSET PRIVATIZATION TRUST
vs.
COURT OF APPEALS et al.
G.R. No. 121171. December 29, 1998
FACTS:
The development, exploration and utilization of the mineral deposits in
the Surigao Mineral Reservation have been authorized by RA No. 1828, as
amended by RAs No. 2077 and 4167, by virtue of which laws, a MOA was
drawn on July 3, 1968, whereby the Republic thru the Surigao Mineral
Reservation Board, granted MMIC the exclusive right to explore, develop and
exploit minerals in the Surigao Mineral Reservation.
The Philippine Government undertook to support the financing of MMIC
by purchase of MMIC debenture and extension of guarantees. DBP approved
guarantees in favor of MMIC and subsequent requests for guarantees were
based on the unutilized portion of the Government commitment. Thereafter,
the Government extended accommodations to MMIC in various amounts.
On July 13, 1981, MMIC, PNB and DBP executed a Mortgage Trust
Agreement whereby MMIC, as mortgagor, agreed to constitute a mortgage in
favor of PNB and DBP as mortgagees, over all MMICs assets, subject of real

Page 242 of 1072

estate and chattel mortgage executed by the mortgagor, and additional


assets described and identified, including assets of whatever kind, nature or
description.
By 1984, MMIC was having a difficult time meeting its financial
obligations. Thus, a financial restructuring plan (FRP) designed to reduce
MMIC's interest expense through debt conversion to equity was drafted by
the Sycip Gorres Velayo accounting firm. On April 30, 1984, the FRP was
approved by the Board of Directors of the MMIC. However, the proposed FRP
had never been formally adopted, approved or ratified by either PNB or DBP.
However, the as the loans remained unpaid, the DBP and PNB
extrajudicially foreclosed the mortgage. The foreclosed assets were then
transferred to Asset Privatization Trust (APT). Actions for the Annulment of
Foreclosures, Specific Performance and Damages were then filed by MMIC
against DBP and PNB. The case was submitted for arbitration, but was failed,
hence, a petition before the SC was filed.

ISSUE/S:
Whether or not the Financial Restructuring Plan (FRP) which was
approved by the Board of Directors of the MMIC is valid and thus the
foreclosure was not justified.
RULING:
No.
PNB and DBP had the legitimate right to foreclose the mortgages of
MMIC whose obligations were past due. There was no financial restructuring
agreement to speak of that could have constituted an impediment to the
exercise of the banks right to foreclose. As correctly stated by Mr. Jose C.
Sison, a member of the Arbitration Committee who wrote a separate opinion:
(1) the various loans and advances made by DBP and PNB to
MMIC have become overdue and remain unpaid. The fact that a
FRP was drawn up is enough to establish that MMIC has not been
complying with the terms of the loan agreement. Restructuring
simply connotes that the obligations are past due that is why it is
restructurable;
(2) When MMIC thru its board and the stockholders agreed and
adopted the FRP, it only means that MMIC had been informed or

Page 243 of 1072

notified that its obligations were past due and that foreclosure is
forthcoming; xxx
Moreover, PNB and DBP had to initiate foreclosure proceedings as
mandated by P.D. No. 385, which took effect on January 31, 1974. The
decree requires government financial institutions to foreclose collaterals for
loans where the arrearages amount to 20% of the total outstanding
obligations.

TOPIC: AUTHORITY OF THE BOARD OF DIRECTORS


BA SAVINGS BANK, petitioner,
vs.
ROGER T. SIA, TACIANA U. SIA and JOHN DOE, respondents.
G.R. No. 131214
July 27, 2000
336 SCRA 484
FACTS:
The Court of Appeals issued a Resolution denying due course a Petition
for Certiorari filed by BA Savings Bank, on the ground that the Certification
on anti-forum shopping incorporated in the petition was signed not by the
duly authorized representative of the petitioner, as required under Supreme
Court Circular 28-91 but by its counsel, in contravention of said circular.
A Motion for Reconsideration was filed by petitioner, attached to it was
a BA Savings Bank Corporate Secretarys Certificate. The Certificate showed
that the petitioners Board of directors approved a resolution authorizing the
petitioners lawyers to represent it in any action or proceeding before any
court, tribunal or agency; and to sign the Certificate of Non-forum Shopping,
among others. The MR was denied by the Court of Appeals on the ground
that Supreme Court Revised Circular No. 28-91 requires that it is the

Page 244 of 1072

petitioner, not the counsel, who must certify under oath to all of the facts
and undertakings required therein.
ISSUE/S:
Whether or not the Supreme Court Revised Circular No. 28-91 allows
a corporation to authorize its counsel to execute a certificate of nonforum shopping in its behalf.
RULING:
Yes.
A corporation, such as the petitioner, has no powers except those
expressly conferred on it by the Corporation Code and those that are implied
by or are incidental to its existence. In turn, a corporation exercises said
powers through its board of directors and/or its duly authorized officers and
agents. Physical acts, like the signing of documents, can be performed only
by natural persons duly authorized for the purpose by corporate bylaws or by
a specific act of the board of directors. All acts within the powers of a
corporation may be performed by agents of its selection; and, except so far
as limitations or restrictions which may be imposed by special charter, bylaw, or statutory provisions, the same general principles of law which govern
the relation of agency for a natural person govern the officer or agent of a
corporation, of whatever status or rank, in respect to his power to act for the
corporation; and agents once appointed, or members acting in their stead,
are subject to the same rules, liabilities and incapacities as are agents of
individuals and private persons.
In the present case, the corporations board of directors issued a
Resolution specifically authorizing its lawyers to act as their agents in any
action or proceeding before the Supreme Court, the Court of Appeals, or any
other tribunal or agency; and to sign, execute and deliver in connection
therewith the necessary pleadings, motions, verification, affidavit of merit,
certificate of non-forum shopping and other instruments necessary for such
action and proceeding.
The resolution of the Board of Directors was sufficient to vest
petitioners lawyers with authority to bind the corporation and was specific
enough as to the acts they were empowered to do. In the case of natural
persons, Circular 28-91 requires the parties themselves to sign the certificate
of non-forum shopping. However, such requirement cannot be imposed on
artificial persons, like corporations, for the reason that they cannot do the
task themselves. Corporations act only through their officers and duly
authorized agents. The Circular does not require corporate officers to sign

Page 245 of 1072

the certificate. Further, there is no prohibition against authorizing agents to


do so.

TOPIC: AUTHORITY OF BOARD OF DIRECTORS


ALFREDO MONTELIBANO, ET AL., plaintiffs-appellants,
vs.
BACOLOD-MURCIA MILLING CO., INC., defendant-appellee.
G.R. No. L-15092. May 18, 1962
5 SCRA 36
FACTS:
Plaintiffs-appellants, Alfredo Montelibano, Alejandro Montelibano, and
the Limited co-partnership Gonzaga and Company, had been and are sugar
planters adhered to the defendant-appellee's sugar central mill under
identical milling contracts. Originally executed in 1919, said contracts were
stipulated to be in force for 30 years starting with the 1920-21 crops, and
provided that the resulting product should be divided in the ratio of 45% for
the mill and 55% for the planters. Sometime in 1936, it was proposed to
execute amended milling contracts, increasing the planters' share to 60% of
the manufactured sugar and resulting molasses, besides other concessions,
but extending the operation of the milling contract from the original 30 years
to 45 years. To this effect, a printed Amended Milling Contract form was
drawn up. On August 20, 1936, the Board of Directors of the appellee

Page 246 of 1072

Bacolod-Murcia Milling Co., Inc., adopted a resolution granting further


concessions to the planters over and above those contained in the printed
Amended Milling Contract.
Appellants signed and executed the printed Amended Milling Contract
on September 10, 1936, but a copy of the resolution of August 10, 1936,
signed by the Central's General Manager, was not attached to the printed
contract until April 17, 1937.
In 1953, the appellants initiated the present action, contending that
three Negros sugar centrals (La Carlota, Binalbagan-Isabela and San Carlos),
with a total annual production exceeding one-third of the production of all
the sugar central mills in the province, had already granted increased
participation (of 62.5%) to their planters, and that under paragraph 9 of the
resolution of August 20, 1936, heretofore quoted, the appellee had become
obligated to grant similar concessions to the plaintiffs (appellants herein).
ISSUE/S:
Whether of not the stipulations contained in the resolution were
made without consideration; that the resolution in question was,
therefore, null and void ab initio, being in effect a donation that was
ultra vires and beyond the powers of the corporate directors to
adopt.
RULING:
1. The terms embodied in the resolution of August 20, 1936 were
supported by the same causa or consideration underlying the main
amended milling contract; i.e., the promises and obligations
undertaken thereunder by the planters, and, particularly, the
extension of its operative period for an additional 15 years over and
beyond the 30 years stipulated in the original contract. Hence, the
conclusion of the court below that the resolution constituted
gratuitous concessions not supported by any consideration is legally
untenable.
2. There can be no doubt that the directors of the appellee company
had authority to modify the proposed terms of the Amended Milling
Contract for the purpose of making its terms more acceptable to the
other contracting parties.
The rule is that
It is a question, therefore, in each case of the logical
relation of the act to the corporate purpose expressed in
the charter. If that act is one which is lawful in itself, and
not otherwise prohibited, is done for the purpose of serving

Page 247 of 1072

corporate ends, and is reasonably tributary to the


promotion of those ends, in a substantial, and not in a
remote and fanciful sense, it may fairly be considered
within charter powers. The test to be applied is whether
the act in question is in direct and immediate furtherance
of the corporation's business, fairly incident to the express
powers and reasonably necessary to their exercise. If so,
the corporation has the power to do it; otherwise, not.
(Fletcher Cyc. Corp., Vol. 6, Rev. Ed. 1950, pp. 266-268)

3. As the resolution in question was passed in good faith by the board


of directors, it is valid and binding, and whether or not it will cause
losses or decrease the profits of the central, the court has no
authority to review them.
They hold such office charged with the duty to act for the
corporation according to their best judgment, and in so
doing they cannot be controlled in the reasonable exercise
and performance of such duty. Whether the business of a
corporation should be operated at a loss during depression,
or close down at a smaller loss, is a purely business and
economic problem to be determined by the directors of the
corporation and not by the court. It is a well-known rule of
law that questions of policy or of management are left
solely to the honest decision of officers and directors of a
corporation, and the court is without authority to substitute
its judgment of the board of directors; the board is the
business manager of the corporation, and so long as it acts
in good faith its orders are not reviewable by the courts.
(Fletcher on Corporations, Vol. 2, p. 390).

Page 248 of 1072

TOPIC: AUTHORITY OF BOARD OF DIRECTORS


ANTHONY POWERS,et. al., plaintiffs-appellants,
vs.
DONALD I. MARSHALL,et. al., defendants-appellees.
G.R. No. L-48064. May 9, 1988
FACTS:
On July 16, 1975, plaintiffs, all associate members of the International
School, Inc., filed an action for injunction in the CFI of Rizal, against the ten
(10) members of the Board of Trustees of the school. The suit was
precipitated by a letter dated May 19, 1975 which Donald I. Marshall,
president of the Board of Trustees of the International School in Makati,
Metro-Manila, addressed to the parents of the students, giving notice that
the Board of Trustees had decided to embark on a program to construct new
buildings and remodel existing ones to accommodate the increasing
enrollment in the school, and that it was necessary for the school to raise
P35,000,000.00 for this purpose. The Board intended to raise the needed
funds primarily through subscriptions to capital notes and prepayment
certificates, and any deficiency from these sources would be covered by

Page 249 of 1072

collecting a so-called "development fee" of P2,625 from each enrollee


starting with the school year 1975-1976 and continuing up to the school year
1986-1987.
The school superintendent, Dr. Max Snyder, acting under instructions
from the Board of Trustees, wrote a letter to the parents of returning
students, enclosing an Application for Admission which specifically advised
that the payment of the development fee was a pre-requisite for reenrollment.
The plaintiffs protested against the imposition of the development fee.
On June 18, 1975 they requested the Board of Trustees to suspend the
implementation of the requirement of payment. On July 16, 1975 the
plaintiffs filed a complaint for injunction against the school. On July 17, 1975,
the trial court issued an order temporarily restraining the defendants or their
authorized representatives and agents from executing and/or enforcing the
development program.The Court dismissed the complaint for lack of valid
cause of action, and dissolved the restraining order of July 17, 1975. Plaintiffs
appealed to CA.
ISSUE/S:
Whether the Board of Trustees of the International School was
authorized to adopt the development plan for which the disputed
fee was being collected from the students.
RULING:
YES.
Section 2 (b) of P.D. No. 732 granting certain rights to the International
School, Inc., expressly authorized the Board of Trustees "upon consultation
with the Secretary of Education and Culture, . . . to determine the amount of
fees and assessments which may be reasonably imposed upon its students,
to maintain or conform to the school standard of education." Such
consultation had been made with the Secretary of Education and Culture who
expressed his conformity with the reasonableness of the assessment of
P2,625.00 per student for the whole school year to carry out its development
program.
Since the collection of the development fee had been approved by the
Board of Trustees of the International School, Inc., it was a valid exercise of
corporate power by the Board, and said assessment was binding upon all the
members of the corporation. Their action to stop the collection of said fee
was correctly dismissed by the trial court for lack of a valid cause of action
against the school.

Page 250 of 1072

TOPIC: AUTHORITY OF BOARD OF DIRECTORS


PREMIUM MARBLE RESOURCES, INC., petitioner,
vs.
THE COURT OF APPEALS and INTERNATIONAL CORPORATE BANK,
respondents.
G.R. No. 96551. November 4, 1996
264 SCRA 11
FACTS:
Premium Marbles, assisted by Atty. Dumadag as counsel, filed an
action for damages against International Corporate Bank. The latter, on the
other, alleged the Premium has no capacity/personality/authority to sue in
this instance and the complaint should therefore be dismissed for failure to
state a cause of action.
Meanwhile, the same corporation, Premium, but this time represented
by Siguion Reyna et.al. Law office as counsel, filed a motion to dismiss on the

Page 251 of 1072

ground that the filing of the case was without authority from its duly
constituted board of directors as shown by the excerpt of the minutes of the
Premiums board of directors meeting.
Premium through Atty. Dumadag opposed by contending that the
persons who signed the board resolution, the Belen et al are not directors of
the corporation. On the other, Siguion Reyna Law Office asserted that it is
the general information sheet filed with the SEC that is the best evidence
that would show who are the stockholders of the corporation and not the
Articles of Incorporation since the latter does not keep track of the many
changes that take place after new stockholders subscribe to corporate
shares of stocks.
ISSUE/S:
Whether or not the filing of the case for damages against private
respondent was authorized by a duly constituted Board of Directors
of the petitioner Corporation.
RULING:
Petitioner, through the first set of officers, Mario Zavalla, et al
presented the minutes of the meeting of its Board held on April 1, 1982 as
proof that the filing of the case was authorized by the Board. On the other
hand, the second set of officers, Belen et al presented a resolution dated July
30, 1986, to show that Premium did not authorize the filing in its behalf of
any suit against respondent Bank. However while the Minutes of the meeting
dated April 1, 1982 states that the officers were Zavalla et al, petitioner
failed to show proof that this election was reported with the SEC. In the
absence of any board resolution from its board of directors the authority to
act for and in behalf of the corporation, the present action must fail. The
power of the corporation to sue and be sued in any court is lodged with the
board of directors that exercises its corporate powers.
The claimed therefore of petitioners as represented by Atty. Dumadag,
that Zavalla et al are the incumbent officers of Premium has not been fully
substantiated. In the absence of an authority from the board of directors, no
person, not even the officers of the corporation, can validly bind the
corporation.

Page 252 of 1072

TOPIC: AUTHORITY OF BOARD OF DIRECTORS


J. F. RAMIREZ, plaintiff-appellee,
vs.
THE ORIENTALIST CO., and RAMON J. FERNANDEZ, defendantsappellants.
G.R. No. 11897. September 24, 1918
38 PHIL 634
FACTS:
The Orientalist Company is a domestic corporation, was engaged in the
business of maintaining and conducting a theater in the city of Manila for the
exhibition of cinematographic films. The plaintiff J. F. Ramirez was, at the
same time, a resident of the city of Paris, France, and was engaged in the
business of marketing films for a manufacturer or manufacturers, there
engaged in the production or distribution of cinematographic material. The
plaintiff was represented in the city of Manila by his son, Jose Ramirez.

Page 253 of 1072

In July, 1913, negotiations were begun with said officials of the


Orientalist Company by Jose Ramirez, as agent of the plaintiff, for the
purpose of placing the exclusive agency of these films in the hands of the
Orientalist Company. The defendant Ramon J. Fernandez, one of the directors
of the Orientalist Company and also its treasurer, was chiefly active in this
matter.
Near the end of July of the said year, Jose Ramirez, placed in the hands
of Fernandez an offer. Accordingly, Ramon J. Fernandez, on July 30, had an
informal conference with all the members of the company's board of
directors except one, and with the approval of those with whom he had
communicated, addressed a letter to Jose Ramirez, in Manila, accepting the
offer contained in the memorandum for the exclusive agency of the Eclair
films. A few days later, on August 5, he addressed another letter couched in
the same terms, likewise accepting the offer of the exclusive agency for the
Milano films.
Two letters of acceptance were sent by Fernandez to Ramirez. In these
communications it will be noted the separate signature of R. J. Fernandez, as
an individual, is placed somewhat below and to the left of the signature of
the Orientalist Company as signed by R. J. Fernandez, in the capacity of
treasurer.
In due time the films began to arrive in Manila, a draft for the cost and
expenses incident to each shipment being attached to the proper bill of
lading. It appears that the Orientalist Company was without funds to meet
these obligations and the first few drafts were dealt with in the following
manner: The drafts, upon presentment through the bank, were accepted in
the name of the Orientalist Company by its president B. Hernandez, and
were taken by the latter with his own funds. As the drafts had thus been paid
by B. Hernandez, the films which had been procured by the payment of said
drafts were treated by him as his own property; and they in fact never came
into the actual possession of the Orientalist Company as owner at all, though
it is true Hernandez rented the films to the Orientalist Company and they
were exhibited by it in the Oriental Theater under an arrangement which was
made between him and the theater's manager.
During the period between February 27, 1914, and April 30, 1914,
there arrived in the city of Manila several remittances of films from Paris, and
it is these shipments which have given occasion for the present action. All of
the drafts accompanying these films were drawn, as on former occasions,
upon the Orientalist Company; and all were accepted in the name of the
Orientalist Company by its president, B. Hernandez, except the last, which
was accepted by B. Hernandez individually. None of the drafts thus accepted
were taken up by the drawee or by B. Hernandez when they fell due; and it

Page 254 of 1072

was finally necessary for the plaintiff himself to take them up as dishonored
by nonpayment.
Thereupon an action was instituted by the plaintiff on May 19,1914,
against the Orientalist Company, and Ramon J. Fernandez. The court,
appointed a receiver who took charge of the films and sold them. The
amount realized from this sale was applied to the satisfaction of the
plaintiff's claim and was accordingly delivered to him in part payment
thereof. In the judgment of the trial court the Orientalist Company was
declared to be a principal debtor and Ramon J. Fernandez was declared to be
liable subsidiarily as guarantor. From this judgment both of the parties
defendant appealed.
ISSUE/S:
Whether or not trial court correctly declared that the Orientalist
Company was the principal debtor and Ramon J. Fernandez was
liable subsidiarily as guarantor.
RULING:
YES.
Ramon J. Fernandez, as treasurer, had no independent authority to
bind the company by signing its name to the letters in question. It is declared
in section 28 of the Corporation Law that corporate powers shall be
exercised, and all corporate business conducted by the board of directors;
and this principle is recognized in the by-laws of the corporation in question
which contain a provision declaring that the power to make contracts shall
be vested in the board of directors. It is true that it is also declared in the
same by-laws that the president shall have the power, and it shall be his
duty, to sign contracts; but this has reference rather to the formality of
reducing to proper form the contracts which are authorized by the board and
is not intended to confer an independent power to make contracts binding on
the corporation.
The fact that the power to make corporate contracts is thus vested in
the board of directors does not signify that a formal vote of the board must
always be taken before contractual liability can be fixed upon a corporation;
for the board can create liability, like an individual, by other means than by a
formal expression of its will.
As already observed, it is familiar doctrine that if a corporation
knowingly permits one of its officers, or any other agent, to do acts within
the scope of an apparent authority, and thus holds him out to the public as
possessing power to do those acts, the corporation will, as against any one
who has in good faith dealt with the corporation through such agent, be
estopped from denying his authority; and where it is said "if the corporation

Page 255 of 1072

permits" this means the same as "if the thing is permitted by the directing
power of the corporation."
As appears upon the face of the contracts, the signature of Fernandez, in his
individual capacity, is not in line with the signature of the Orientalist
Company, but is set off to the left of the company's signature and somewhat
below. Observation teaches that it is customary for persons who sign
contracts in some capacity other than that of principal obligor to place their
signatures to one side; but the court hardly thinks that this circumstance
alone would justify a court in holding that Fernandez here took upon himself
the responsibility of a guarantor rather than that of a principal obligor. The
Court, however, think that the form in which the contract is signed raises a
doubt as to what the real intention was. In this connection it is entirely clear,
from the testimony of both Ramirez and Ramon J. Fernandez, that the
responsibility of the latter was intended to be that of a guarantor. There is, to
be sure, a certain difference between these witnesses as to the nature of this
guaranty, inasmuch as Fernandez would have us believe that his name was
signed as a guaranty that the contract would be approved by the
corporation, while Ramirez says that the name was put on the contract for
the purpose of guaranteeing, not the approval of the contract but its
performance. We are convinced that the latter was the real intention of the
contracting parties.

TOPIC: CORPORATE OFFICERS/ MEANING OF OFFICE VIS A VIS EMPLOYMENT


RENATO REAL, Petitioner,
vs.
SANGU PHILIPPINES, INC. and/ or KIICHI ABE, Respondents.
G.R. No. 168757: January 19, 2011
FACTS:
Petitioner Renato Real was the Manager of respondent corporation
Sangu Philippines, Inc., a corporation engaged in the business of providing

Page 256 of 1072

manpower for general services, like janitors,


maintenance personnel, to various clients.

janitresses

and

other

Petitioner was removed from his position as Manager through Board


Resolution 2001-03 adopted by respondent corporation's Board of Directors.
Petitioner complained that he was neither notified of the Board Meeting
during which said board resolution was passed nor formally charged with any
infraction. He just received from respondents a letter 4cralaw dated March 26,
2001 stating that he has been terminated from service effective March 25,
2001 for the following reasons: (1) continuous absences at his post at Ogino
Philippines Inc. for several months which was detrimental to the
corporation's operation; (2) loss of trust and confidence; and, (3) to cut down
operational expenses to reduce further losses being experienced by
respondent corporation.
In 2001, petitioner, together with 29 others who were either janitors,
janitresses, leadmen and maintenance men, all employed by respondent
corporation, filed their respective Complaints2cralaw for illegal dismissal.
The Labor arbiter declared the petitioner including his co employees as
illegally dismissed and ordered their reinstatement, however the NLRC
reversed the decision and declared petitioner as validly dismissed and the
Labor Arbiter having no jurisdiction since petitioner is one among the
stockholders, a corporate officer thus belonging to intra-corporate case. The
decision of the NLRC was affirmed by the Court of Appeals.
ISSUE/S:
Whether or not the petitioner is a corporate officer of the respondent
corporation?
RULING:
Corporate officers' in the context of Presidential Decree No. 902-A are
those officers of the corporation who are given that character by the
Corporation Code or by the corporation's by-laws. There are three specific
officers whom a corporation must have under Section 25 of the Corporation
Code. These are the president, secretary and the treasurer. The number of
officers is not limited to these three. A corporation may have such other
officers as may be provided for by its by-laws like, but not limited to, the
vice-president, cashier, auditor or general manager. The number of corporate
officers is thus limited by law and by the corporation's by-laws.
Respondents claim that petitioner was appointed Manager by virtue of
Section 1, Article IV of respondent corporation's By-Laws which provides:

Page 257 of 1072

ARTICLE
IV:OFFICER,
Section
1. Election/Appointment Immediately after their election, the Board of Directors shall
formally organize by electing the President, Vice-President, the
Secretary at said meeting. The Board, may from time to
time, appoint such other officers as it may determine to
be necessary or proper. Any two (2) or more positions may be
held concurrently by the same person, except that no one shall
act as President and Treasurer or Secretary at the same time.
The examination of the record shows that that petitioner's appointment
was not made pursuant to the above-quoted provision of respondent
corporation's By-Laws. No copy of board resolution appointing petitioner as
Manager or any other document showing that he was appointed to said
position by action of the board was submitted by respondents.

TOPIC: CORPORATE OFFICERS/ MEANING OF OFFICE VIS A VIS EMPLOYMENT


MATLING INDUSTRIAL AND COMMERCIAL CORPORATION, RICHARD K.
SPENCER, CATHERINE SPENCER, AND ALEX MANCILLA, Petitioners,
vs.
RICARDO R. COROS, Respondent.
G.R. No. 157802 : October 13, 2010
FACTS:
After his dismissal by Matling as its Vice President for Finance and
Administration, the respondent filed on August 10, 2000 a complaint for

Page 258 of 1072

illegal suspension and illegal dismissal against Matling and some of its
corporate officers (petitioners).
The petitioners moved to dismiss the complaint, raising the ground,
among others, that the complaint pertained to the jurisdiction of the
Securities and Exchange Commission (SEC) due to the controversy being
intra-corporate inasmuch as the respondent was a member of Matlings Board
of Directors aside from being its Vice-President for Finance and
Administration prior to his termination.
The respondent opposed the petitioners motion to dismiss, insisting
that his status as a member of Matlings Board of Directors was doubtful,
considering that he had not been formally elected as such; that he did not
own a single share of stock in Matling, considering that he had been made to
sign in blank an undated endorsement of the certificate of stock he had been
given in 1992; that Matling had taken back and retained the certificate of
stock in its custody; and that even assuming that he had been a Director of
Matling, he had been removed as the Vice President for Finance and
Administration, not as a Director, a fact that the notice of his termination
dated April 10, 2000 showed.
LA granted the petitioners motion to dismiss, ruling that the
respondent was a corporate officer because he was occupying the position of
Vice President for Finance and Administration and at the same time was a
Member of the Board of Directors of Matling.
On March 13, 2001, the NLRC set aside the dismissal, concluding that
the respondents complaint for illegal dismissal was properly cognizable by
the LA, not by the SEC, because he was not a corporate officer by virtue of
his position in Matling, albeit high ranking and managerial, not being among
the positions listed in Matlings Constitution and By-Laws.
The Court of Appeals affirmed the decision of the NLRC.
ISSUE/S:
Whether or not respondent Coros is a corporate officer?
RULING:
The petitioners contend that the position of Vice President for Finance
and Administration was a corporate office, having been created by Matlings
President pursuant to By-Law No. V, as amended, to wit:
BY LAW NO. V: Officers
The President shall be the executive head of the
corporation; shall preside over the meetings of the
stockholders
and
directors;
shall
countersign
all

Page 259 of 1072

certificates, contracts and other instruments of the


corporation as authorized by the Board of Directors; shall
have full power to hire and discharge any or all employees
of the corporation; shall have full power to create new
offices and to appoint the officers thereto as he may deem
proper and necessary in the operations of the corporation
and as the progress of the business and welfare of the
corporation may demand; shall make reports to the
directors and stockholders and perform all such other
duties and functions as are incident to his office or are
properly required of him by the Board of Directors. In case
of the absence or disability of the President, the Executive
Vice President shall have the power to exercise his
functions.
The petitioners argue that the power to create corporate offices and to
appoint the individuals to assume the offices was delegated by Matlings
Board of Directors to its President through By-Law No. V, as amended; and
that any office the President created, like the position of the respondent, was
as valid and effective a creation as that made by the Board of Directors,
making the office a corporate office. In justification, they cite Tabang v.
National Labor Relations Commission, which held that other offices are
sometimes created by the charter or by-laws of a corporation, or the board of
directors may be empowered under the by-laws of a corporation to create
additional officers as may be necessary.
The respondent counters that Matlings By-Laws did not list his position
as Vice President for Finance and Administration as one of the corporate
offices; that Matlings By-Law No. III listed only four corporate officers,
namely: President, Executive Vice President, Secretary, and Treasurer; that
the corporate offices contemplated in the phrase and such other officers as
may be provided for in the by-laws found in Section 25 of the Corporation
Code should be clearly and expressly stated in the By-Laws; that the fact
that Matlings By-Law No. III dealt with Directors & Officers while its By-Law
No. V dealt with Officers proved that there was a differentiation between the
officers mentioned in the two provisions, with those classified under By-Law
No. V being ordinary or non-corporateofficers; and that the officer, to be
considered as a corporate officer, must be elected by the Board of Directors
or the stockholders, for the President could only appoint an employee to a
position pursuant to By-Law No. V.
However, Section 25 of the Corporation Code provides:Corporate
officers, quorum.--Immediately after their election, the directors of a
corporation must formally organize by the election of a president, who shall
be a director, a treasurer who may or may not be a director, a secretary who
shall be a resident and citizen of the Philippines, and such other officers

Page 260 of 1072

as may be provided for in the by-laws. Any two (2) or more positions
may be held concurrently by the same person, except that no one shall act
as president and secretary or as president and treasurer at the same time.
The directors or trustees and officers to be elected shall perform the
duties enjoined on them by law and the by-laws of the corporation. Unless
the articles of incorporation or the by-laws provide for a greater majority, a
majority of the number of directors or trustees as fixed in the articles of
incorporation shall constitute a quorum for the transaction of corporate
business, and every decision of at least a majority of the directors or trustees
present at a meeting at which there is a quorum shall be valid as a corporate
act, except for the election of officers which shall require the vote of a
majority of all the members of the board. Directors or trustees cannot attend
or vote by proxy at board meetings.
Conformably with Section 25, a position must be expressly mentioned
in the By-Laws in order to be considered as a corporate office. Thus, the
creation of an office pursuant to or under a By-Law enabling provision is not
enough to make a position a corporate office. Guerrea v. Lezama, the first
ruling on the matter, held that the only officers of a corporation were those
given that character either by the Corporation Code or by the By-Laws; the
rest of the corporate officers could be considered only as employees or
subordinate officials.

TOPIC: CORPORATE OFFICERS/ MEANING OF OFFICE VIS A VIS EMPLOYMENT


MANILA METAL CONTAINER CORPORATION, petitioner,
REYNALDO C. TOLENTINO, intervenor,
vs.
PHILIPPINE NATIONAL BANK, respondent,DMCI-PROJECT
DEVELOPERS, INC., intervenor.
G.R. No. 166862
December 20, 2006
511 S 444

Page 261 of 1072

FACTS:
Petitioner was the owner of an 8,015 square meter parcel of land. To
secure a P900, 000.00 loans it had obtained from respondent PNB, petitioner
executed a real estate mortgage over the lot. Respondent PNB later granted
petitioner a new credit accommodation of P1, 000,000.00; and, on November
16, 1973, petitioner executed an Amendment of Real Estate Mortgage over
its property. On March 31, 1981, petitioner secured another loan of P653,
000.00 from respondent PNB, payable in quarterly installments
of P32,650.00, plus interests and other charges.
On August 5, 1982, respondent PNB filed a petition for extrajudicial
foreclosure of the real estate mortgage and sought to have the property sold
at public auction for petitioners outstanding obligation to respondent PNB as
of June 30, 1982.
After due notice and publication, the property was sold at public
auction on September 28, 1982 where respondent PNB was declared the
winning bidder for P1,000,000.00. The Certificate of Sale issued in its favor
was registered with the Office of the Register of Deeds of Rizal, and was
annotated at the dorsal portion of the title on February 17, 1983. Thus, the
period to redeem the property was to expire on February 17, 1984.
An offer made by the petitioner for extension of redemption and
repurchase on an installment basis was denied. Meanwhile, some PNB Pasay
City Branch personnel informed petitioner that as a matter of policy, the
bank does not accept "partial redemption."
Since petitioner failed to redeem the property, the Register of Deeds
cancelled TCT No. 32098 on June 1, 1984, and issued a new title in favor of
respondent PNB.
Meanwhile, the Special Assets Management Department (SAMD) had
prepared a statement of account, and as of June 25, 1984 petitioner's
obligation amounted to P1,574,560.47.
In the meantime, the SAMD recommended to the management of
respondent PNB that petitioner be allowed to repurchase the property for P1,
574,560.00. In a letter dated November 14, 1984, the PNB management
informed petitioner that it was rejecting the offer and the recommendation of
the SAMD.
On June 4, 1985, respondent PNB informed petitioner that the PNB
Board of Directors had accepted petitioner's offer to purchase the property,
but for P1,931,389.53 in cash less the P725,000.00 already deposited with it.

Page 262 of 1072

Petitioner rejected respondent's proposal in a letter dated July 14,


1988. It maintained that respondent PNB had agreed to sell the property for
P1,574,560.47, and that since its P725,000.00 downpayment had been
accepted, respondent PNB was proscribed from increasing the purchase price
of the property.
ISSUE/S:
Whether or not SAMD was authorized by respondent's Board of
Directors to accept petitioner's offer and sell the property for
P1,574,560.47.
RULING:
No.
There is no evidence that the SAMD was authorized by respondent's
Board of Directors to accept petitioner's offer and sell the property for
P1,574,560.47. Any acceptance by the SAMD of petitioner's offer would not
bind respondent. As this Court ruled in AF Realty Development, Inc. vs.
Diesehuan Freight Services, Inc.:
Section 23 of the Corporation Code expressly provides that
the corporate powers of all corporations shall be exercised by the
board of directors. Just as a natural person may authorize
another to do certain acts in his behalf, so may the board of
directors of a corporation validly delegate some of its functions
to individual officers or agents appointed by it. Thus, contracts or
acts of a corporation must be made either by the board of
directors or by a corporate agent duly authorized by the board.
Absent such valid delegation/authorization, the rule is that the
declarations of an individual director relating to the affairs of the
corporation, but not in the course of, or connected with the
performance of authorized duties of such director, are held not
binding on the corporation.
Thus, a corporation can only execute its powers and transact its
business through its Board of Directors and through its officers and agents
when authorized by a board resolution or its by-laws.
It appears that the SAMD had prepared a recommendation for
respondent to accept petitioner's offer to repurchase the property even
beyond the one-year period; it recommended that petitioner be allowed to
redeem the property and pay P1,574,560.00 as the purchase price.
Respondent later approved the recommendation that the property be sold to
petitioner. But instead of the P1,574,560.47 recommended by the SAMD and
to which petitioner had previously conformed, respondent set the purchase
price at P2,660,000.00. In fine, respondent's acceptance of petitioner's offer
was qualified, hence can be at most considered as a counter-offer. If

Page 263 of 1072

petitioner had accepted this counter-offer, a perfected contract of sale would


have arisen; as it turns out, however, petitioner merely sought to have the
counter-offer reconsidered. This request for reconsideration would later be
rejected by respondent.

TOPIC: CORPORATE OFFICERS/ MEANING OF OFFICE VIS A VIS EMPLOYMENT


BIENVENIDO ONGKINGCO, as President and GALERIA DE
MAGALLANES CONDOMINIUM ASSOCIATION, INC., petitioners,
vs.
NATIONAL LABOR RELATIONS COMMISSION and FEDERICO B.
GUILAS, respondents.
G.R. No. 119877 March 31, 1997
270 SCRA 613

Page 264 of 1072

FACTS:
Petitioner Galeria de Magallanes Condominium Association, Inc.
(Galeria for brevity) is a non-stock, non-profit corporation formed in
accordance with R.A. No. 4726, otherwise known as the Condominium Act.
"Its primary purpose is to hold title to the common areas of the Galeria de
Magallanes Condominium Project and to manage and administer the same
for the use and convenience of the residents and/or owners. Petitioner
Bienvenido Ongkingco was the president of Galeria at the time private
respondent filed his complaint.
On 1 September 1990, Galeria's Board of Directors appointed private
respondent Federico B. Guilas as Administrator/Superintendent. He was
given a "monthly salary of P10,000 subject to review after five (5) months
and subsequently thereafter as Galeria's finances improved. As
Administrator, private respondent was tasked with the maintenance of the
"performance and elegance of the common areas of the condominium and
external appearance of the compound thereof for the convenience and
comfort of the residents as well as to keep up the quality image, and hence
the value of the investment for the owners thereof.
However, on 17 March 1992, through a resolution passed by the Board
of Directors of Galeria, private respondent was not re-appointed as
Administrator. As a result, private respondent instituted a complaint against
petitioners for illegal dismissal and non-payment of salaries with the NLRC.
ISSUE/S:
Whether or not respondent is corporate officer of Galeria?
RULING:
YES.
Private respondent is an officer of petitioner corporation and not its
mere employee. The by-laws of the Galeria de Magallanes Condominium
Association specifically includes the Superintendent/Administrator in its
roster of corporate officers:
xxx xxx xxx
Sec. 6. The Superintendent or Administrator The Board
of Directors may appoint a Superintendent or Administrator
for the condominium project if the activities and financial
condition of the Association so warrant. If one is so
appointed, he shall be the principal administrative officer
of the Association. He shall attend to routinary and day-today business and activities of the Association and shall
keep regular officer hours for the purpose. He shall have
such other duties and powers as may be conferred upon

Page 265 of 1072

him by the Board of Directors or delegated by the President


of the Association.
The president, vice-president, secretary and treasurer are commonly
regarded as the principal or executive officers of a corporation, and modern
corporation statutes usually designate them as the officers of the
corporation. However, other offices are sometimes created by the charter or
by-laws of a corporation, or the board of directors may be empowered under
the by-laws of a corporation to create additional offices as may be necessary.
It has been held that an "office" is created by the charter of the
corporation and the officer is elected by the directors or stockholders. On the
other hand, an "employee" usually occupies no office and generally is
employed not by action of the directors or stockholders but by the managing
officer of the corporation who also determines the compensation to be paid
to such employee.
In the case at bar, considering that herein respondent, unlike an
ordinary employee, was appointed by petitioner corporation's Board of
Trustees in its memorandum of September 1, 1990, he is deemed an officer
of the corporation. Perforce, Section 5(c) of Presidential Decree No. 902-A,
which provides that the SEC exercises exclusive jurisdiction over
controversies in the election or appointment of directors, trustees, officers or
managers of corporations, partnerships or associations, applies in the
present dispute. Accordingly, jurisdiction over the same is vested in the SEC,
and not in the Labor Arbiter or the NLRC.

TOPIC: CORPORATE OFFICERS/ MEANING OF OFFICE VIS A VIS EMPLOYMENT


ANDRES LAO, petitioner,
vs.
COURT OF APPEALS, THE ASSOCIATED ANGLO-AMERICAN TOBACCO
CORPORATION and ESTEBAN CO, respondents.
G.R. No. 47013
February 17, 2000
325 SCRA 694
FACTS:

Page 266 of 1072

On April 6, 1965, The Associated Anglo-American Tobacco Corporation


(Corporation for brevity) entered into a "Contract of Sales Agent" with Andres
Lao. Under the contract, Lao agreed to sell cigarettes manufactured and
shipped by the Corporation to his business address in Tacloban City. Lao
would in turn remit the sales proceeds to the Corporation. For his services,
Lao would receive commission depending on the kind of cigarettes sold, fixed
monthly salary, and operational allowance. As a guarantee to Lao's
compliance with his contractual obligations, his brother Jose and his father
Tomas executed a deed of mortgage in favor of the Corporation in the
amount of P200, 000.00.
The Corporation awarded him trophies and plaques in recognition of his
outstanding performance from 1966 to 1968. However, in February 1968 and
until about seven (7) months later, Lao failed to accomplish his monthly
sales report. It was then and there established that Lao's liability amounted
to P525, 053.47. And so, Lao and his brother Lao Y Ka enlisted the services of
the Sycip Gorres and Velayo Accounting Firm (SGV) to check and reconcile
the accounts. However, the SGV personnel Lao had employed failed to
conclude their services because the Corporation did not honor its
commitment to assign two of its accountants to assist them. Neither did the
Corporation allow the SGV men access to its records.
Since Lao appeared to encounter difficulties in complying with his
obligations under the contract of agency, the Corporation sent Ngo Kheng to
supervise Lao's sales operations in Leyte and Samar. Ngo Kheng discovered
that, contrary to Lao's allegation that he still had huge collectibles from his
customers; nothing was due the Corporation from Lao's clients. From then
on, Lao no longer received shipments from the Corporation which transferred
its vehicles to another compound controlled by Ngo Kheng. Shipments of
cigarettes and the corresponding invoices were also placed in the name of
Ngo Kheng.
On May 21, 1970, Andres, Jose and Tomas Lao brought a complaint for
accounting and damages with writ of preliminary injunction against the
Corporation.
Petitioner Esteban Co filed a case for estafa against Lao, during the
pendency of the first case.
ISSUE/S:
Whether or not Esteban Co as an officer of the Corporation can be held
personally liable for malicious prosecution in filing estafa case against
Lao.
RULING:

Page 267 of 1072

No.
He is not personally liable since he has the authority to file the case as
impliedly granted by the Corporation for its failure to object on the act of Co.
A perusal of his affidavit-complaint reveals that at the time he filed the
same on June 24, 1974, petitioner Co was the vice-president of the
Corporation. As a corporate officer, his power to bind the Corporation as its
agent must be sought from statute, charter, by-laws, a delegation of
authority to a corporate officer, or from the acts of the board of directors
formally expressed or implied from a habit or custom of doing business. In
this case, no such sources of petitioner's authority from which to deduce
whether or not he was acting beyond the scope of his responsibilities as
corporate vice-president are mentioned, much less proven. It is thus logical
to conclude that the board of directors or by laws- of the corporation vested
petitioner Co with certain executive duties one of which is a case for the
Corporation.
That petitioner Co was authorized to institute the estafa case is
buttressed by the fact that the Corporation failed to make an issue out of his
authority to file said case. Upon well-established principles of pleading, lack
of authority of an officer of a corporation to bind it by contract executed by
him in its name, is a defense which should have been specially pleaded by
the Corporation. The Corporation's failure to interpose such a defense could
only mean that the filing of the affidavit-complaint by petitioner Co was with
the consent and authority of the Corporation. In the same vein, petitioner Co
may not be held personally liable for acts performed in pursuance of an
authority and therefore, holding him solidarily liable with the Corporation for
the damages awarded to respondent Lao does accord with law and
jurisprudence.

TOPIC: CORPORATE OFFICERS/ MEANING OF OFFICE VIS A VIS EMPLOYMENT


MITA PARDO DE TAVERA, plaintiff-appellant,
vs.
PHILIPPINE TUBERCULOSIS SOCIETY, INC., FRANCISCO ORTIGAS, JR.,
MIGUEL CAIZARES, BERNARDO P. PARDO, RALPH NUBLA,
MIDPANTAO ADIL, ENRIQUE GARCIA, ALBERTO G. ROMULO and THE

Page 268 of 1072

PRESENT BOARD OF DIRECTORS, PHILIPPINE TUBERCULOSIS


SOCIETY, INC., defendants- appellees.
GR. No. L-48928 February 25, 1982
112 S 243
FACTS:
Plaintiff is a doctor of Medicine by profession and a recognized
specialist in the treatment of tuberculosis, having been in the continuous
practice of her profession since 1945; that she is a member of the Board of
Directors of the defendant Society, in representation of the Philippine Charity
Sweepstakes Office; that she was duly appointed on April 27, 1973 as
Executive Secretary of the Society; that on May 29, 1974, the past Board of
Directors removed her summarily from her position, the lawful cause of
which she was not informed, through the simple expedient of declaring her
position vacant; that immediately thereafter, defendant Alberto Romulo was
appointed to the position by an affirmative vote of seven directors, with two
abstentions and one objection; and that defendants Pardo, Nubla, Garcia and
Adil, not being members of defendant Society when they were elevated to
the position of members of the Board of Directors, are not qualified to be
elected as such and hence, all their acts in said meeting of May 29, 1974 are
null and void.
The defendants deny that plaintiff was illegally removed from her
position as Executive Secretary and averring that under the Code of By-Laws
of the Society, said position is held at the pleasure of the Board of Directors
and when the pleasure is exercised, it only means that the incumbent has to
vacate the same because her term has expired; that defendants Pardo,
Nubla, Adil and Garcia were, at the time of their election, members of the
defendant Society and qualified to be elected as members of the Board, that
assuming that said defendants were not members of defendant Society at
the time of their election, the question of qualification of the members of the
Board of Directors should have been raised at the time of their election.
ISSUE:
Whether or not petitioner was illegally removed from her position as an
officer of the respondent.

RULING:
Petitioner claims and the respondents do not dispute that the
Executive Secretary is an officer of the Society pursuant to provision in the
Code of By-laws Laws:

Page 269 of 1072

Section 7.01. Officers of the Society. - The executed


officers f the Society shall be the President a VicePresident, a Treasurer who shall be elected by the Board of
Directors, Executive Secretary, and an Auditor, who shall
be appointed by the Board of Directors, all of whom shall
exercise the functions, powers and prerogatives generally
vested upon such officers, the functions hereinafter set out
for their respective offices and such other duties is from
time to time, may be prescribed by the Board of Directors.
On e person may hold more than one office except when
the functions thereof are incompatible with each other.
Section 7.02. Tenure of Office. - All executive officers of the
Society except the Executive Secretary and the Auditor
shall be elected the Board of Directors, for a term of one
rear ind shall hold office until their successors are elected
and have qualified. The Executive secretary, the Auditor
and all other office ers and employees of the Society shall
hold office at the pleasure of the Board of Directors, unless
their term of employment shall have been fixed in their
contract of employment.
xxx xxx xxx
Section 7.04. Removal of Officers and Employees. - All
officers and employees shall be subject to suspension or
removal for a sufficient cause at any time by affirmative
vote of a majority of an the members of the Board of
Directors, except that employees appointed by the
President alone or by the other officers alone at the
pleasure of the officer appointing him.
Petitioner was designated as Acting Executive Secretary with an
honorarium of P200.00 monthly in view of the application of Dr. Jose Y.
Buktaw for leave effective September 1, 1972 for 300 working days. In an
organizational meeting, the president mentioned that there is a need of
appointing a permanent Executive Secretary, thus petitioner was appointed
as Executive Secretary.
Although the minutes of the organizational meeting show that the
Chairman mentioned the need of appointing a "permanent" Executive
Secretary, such statement alone cannot characterize the appointment of
petitioner without a contract of employment definitely fixing her term
because of the specific provision of Section 7.02 of the Code of By-Laws.
Besides the word permanent" could have been used to distinguish the
appointment from acting capacity.

Page 270 of 1072

The absence of a fixed term in the letter addressed to petitioner informing


her of her appointment as Executive Secretary is very significant. This could
have no other implication than that petitioner held an appointment at the
pleasure of the appointing power.
An appointment held at the pleasure of the appointing power is in
essence temporary in nature. It is co-extensive with the desire of the Board
of Directors. Hence, when the Board opts to replace the incumbent,
technically there is no removal but only an expiration of term and in an
expiration of term, there is no need of prior notice, due hearing or sufficient
grounds before the incumbent can be separated from office. The protection
afforded by Section 7.04 of the Code of By-Laws on Removal of Officers and
Employees, therefore, cannot be claimed by petitioner.

TOPIC:MINIMUM SET OF OFFICERS; QUALIFICATIONS; DISQUALIFICATIONS;


AUTHORITY & LIABILITIES
MATLING INDUSTRIAL AND COMMERCIAL CORPORATION, RICHARD K.
SPENCER, CATHERINE SPENCER, AND ALEX MANCILLA, Petitioners,

Page 271 of 1072

vs.
RICARDO R. COROS, Respondent.
G.R. No. 157802. October 13, 2010
FACTS:
After his dismissal by Matling as its VP for Finance and Administration,
the respondent filed a complaint for illegal suspension and illegal dismissal
against Matling and some of its corporate officers. The petitioners moved to
dismiss raising the ground that the complaint pertained to the jurisdiction of
SEC due to the controversy being intra-corporate inasmuch as the
respondent was a member of Matlings BOD aside from being its VP for
Finance and Administration prior to his termination.
The respondent opposed the petitioners motion to dismiss, insisting
that his status as a member of Matlings Board of Directors was doubtful,
considering that he had not been formally elected as such; that he did not
own a single share of stock in Matling, considering that he had been made to
sign in blank an undated indorsement of the certificate of stock he had been
given; that Matling had taken back and retained the certificate of stock in its
custody; and that even assuming that he had been a Director of Matling, he
had been removed as the VP for Finance and Administration, not as a
Director, a fact that the notice of his termination showed.
ISSUE/S:
Whether the respondent was a corporate officer of Matling or not.
RULING:
Matlings By-Laws did not list his position as VPfor Finance and
Administration as one of the corporate offices; that Matlings By-Law No. III
listed only four corporate officers, namely: President, Executive Vice
President, Secretary, and Treasurer; that the corporate offices contemplated
in the phrase "and such other officers as may be provided for in the by-laws"
found in Section 25 of the Corporation Code should be clearly and expressly
stated in the By-Laws; that the fact that Matlings By-Law No. III dealt with
Directors & Officers while its By-Law No. V dealt with Officers proved that
there was a differentiation between the officers mentioned in the two
provisions, with those classified under By-Law No. V being ordinary or noncorporate officers; and that the officer, to be considered as a corporate
officer, must be elected by the Board of Directors or the stockholders, for the
President could only appoint an employee to a position pursuant to By-Law
No. V.
Section 25 of the Corporation Code provides:
Section 25. Corporate officers, quorum.--Immediately after
their election, the directors of a corporation must formally
organize by the election of a president, who shall be a
director, a treasurer who may or may not be a director, a
secretary who shall be a resident and citizen of the
Philippines, and such other officers as may be provided for

Page 272 of 1072

in the by-laws. Any two (2) or more positions may be held


concurrently by the same person, except that no one shall
act as president and secretary or as president and
treasurer at the same time.
The directors or trustees and officers to be elected shall perform the
duties enjoined on them by law and the by-laws of the corporation. Unless
the articles of incorporation or the by-laws provide for a greater majority, a
majority of the number of directors or trustees as fixed in the articles of
incorporation shall constitute a quorum for the transaction of corporate
business, and every decision of at least a majority of the directors or trustees
present at a meeting at which there is a quorum shall be valid as a corporate
act, except for the election of officers which shall require the vote of a
majority of all the members of the board.
Directors or trustees cannot attend or vote by proxy at board meetings.
Conformably with Section 25, a position must be expressly mentioned
in the By-Laws in order to be considered as a corporate office. Thus, the
creation of an office pursuant to or under a By-Law enabling provision is not
enough to make a position a corporate office. The only officers of a
corporation were those given that character either by the Corporation Code
or by the By-Laws; the rest of the corporate officers could be considered only
as employees or subordinate officials. Thus, an "office" is created by the
charter of the corporation and the officer is elected by the directors or
stockholders. On the other hand, an employee occupies no office and
generally is employed not by the action of the directors or stockholders but
by the managing officer of the corporation who also determines the
compensation to be paid to such employee.
In this case, respondent was appointed vice president for nationwide
expansion by Malonzo, petitioner's general manager, not by the board of
directors of petitioner. It was also Malonzo who determined the compensation
package of respondent. Thus, respondent was an employee, not a "corporate
officer."
This interpretation is the correct application of Section 25 of the
Corporation Code, which plainly states that the corporate officers are the
President, Secretary, Treasurer and such other officers as may be provided
for in the By-Laws. Accordingly, the corporate officers in the context of PD
No. 902-A are exclusively those who are given that character either by the
Corporation Code or by the corporations By-Laws.
A different interpretation can easily leave the way open for the Board
of Directors to circumvent the constitutionally guaranteed security of tenure
of the employee by the expedient inclusion in the By-Laws of an enabling
clause on the creation of just any corporate officer position.

Page 273 of 1072

It is relevant to state in this connection that the SEC, the primary


agency administering the Corporation Code, adopted a similar interpretation
of Section 25 of the Corporation Code in its Opinion dated November 25,
1993,21 to wit:
Thus, pursuant to the above provision (Section 25 of the Corporation
Code), whoever are the corporate officers enumerated in the by-laws are the
exclusive Officers of the corporation and the Board has no power to create
other Offices without amending first the corporate By-laws. However, the
Board may create appointive positions other than the positions of corporate
Officers, but the persons occupying such positions are not considered as
corporate officers within the meaning of Section 25 of the Corporation Code
and are not empowered to exercise the functions of the corporate Officers,
except those functions lawfully delegated to them. Their functions and duties
are to be determined by the Board of Directors/Trustees.
Moreover, the Board of Directors of Matling could not validly delegate
the power to create a corporate office to the President, in light of Section 25
of the Corporation Code requiring the Board of Directors itself to elect the
corporate officers. Verily, the power to elect the corporate officers was a
discretionary power that the law exclusively vested in the Board of Directors,
and could not be delegated to subordinate officers or agents. The office of
Vice President for Finance and Administration created by Matlings President
pursuant to By Law No. V was an ordinary, not a corporate, office.
To emphasize, the power to create new offices and the power to
appoint the officers to occupy them vested by By-Law No. V merely allowed
Matlings President to create non-corporate offices to be occupied by
ordinary employees of Matling. Such powers were incidental to the
Presidents duties as the executive head of Matling to assist him in the daily
operations of the business.

Page 274 of 1072

TOPIC:MINIMUM SET OF OFFICERS; QUALIFICATIONS; DISQUALIFICATIONS;


AUTHORITY
& LIABILITIES
E. B. VILLAROSA & PARTNER CO., LTD., petitioner,
vs.
HON. HERMINIO I. BENITO, in his capacity as Presiding Judge, RTC,
Branch 132, Makati City and IMPERIAL DEVELOPMENT
CORPORATION, respondent.
G.R. No. 136426. August 6, 1999
312 SCRA 65
FACTS:
Herein Petitioner, a limited partnership with principal office address at
102 Juan Luna St., Davao City and with branch offices at 2492 Bay View
Drive, Tambo, Paraaque, Metro Manila and Kolambog, Lapasan, Cagayan de
Oro City. Petitioner and private respondent executed a Deed of Sale with
Development Agreement wherein the former agreed to develop certain
parcels of land located at Barrio Carmen, Cagayan de Oro belonging to the
latter into a housing subdivision for the construction of low cost housing
units. They further agreed that in case of litigation regarding any dispute
arising therefrom, the venue shall be in the proper courts of Makati.
Private respondent filed a complaint for breach of contract and
damages against petitioner before RTC of Makati for failure to comply with
the contract. Summons together with the complaint were served upon the
petitioner through its Branch Manager Wendell Sabultero at Cagayan De Oro
City.
Petitioner sought the dismissal of the complaint on the ground of
improper service of summons and lack of jurisdiction over the person of the
petitioner. It contends that the trial court did not acquire jurisdiction over its
person since the summons was improperly served upon its employee in its
branch office at Cagayan De Oro City who is not one of those persons named
in Section 11, Rule 14 of the 1997 Rules of Civil Procedure upon whom the
service of summons maybe made.
The Trial court issued an order stating that since the summons and
copy of the complaint were in fact received by the corporation through its
branch manager, there was substantial compliance with the rule on service
of summons and consequently, it validly acquires jurisdiction over the person
of the petitioner.
ISSUE/S:

Page 275 of 1072

Whether or not the trial court acquired jurisdiction over the person
of petitioner upon service of summons on its Branch manager.
RULING:
No.
Accordingly, the Supreme Court ruled that the service of summons
upon the branch manager of petitioner at its branch office at Cagayan de
Oro, instead of upon the general manager at its principal office at Davao City
is improper. Consequently, the trial court did not acquire jurisdiction over the
person of the petitioner.
The fact that defendant filed a belated motion to dismiss did not
operate to confer jurisdiction upon its person. There is no question that the
defendant's voluntary appearance in the action is equivalent to service of
summons. Before, the rule was that a party may challenge the jurisdiction of
the court over his person by making a special appearance through a motion
to dismiss and if in the same motion, the movant raised other grounds or
invoked affirmative relief which necessarily involves the exercise of the
jurisdiction of the court.This doctrine has been abandoned in the case of La
Naval Drug Corporation vs. Court of Appeals, et al., which became the basis
of the adoption of a new provision in the former Section 23, which is now
Section 20 of Rule 14 of the 1997 Rules. Section 20 now provides that "the
inclusion in a motion to dismiss of other grounds aside from lack of
jurisdiction over the person of the defendant shall not be deemed a
voluntary appearance." The emplacement of this rule clearly underscores the
purpose to enforce strict enforcement of the rules on summons. Accordingly,
the filing of a motion to dismiss, whether or not belatedly filed by the
defendant, his authorized agent or attorney, precisely objecting to the
jurisdiction of the court over the person of the defendant can by no means
be deemed a submission to the jurisdiction of the court. There being no
proper service of summons, the trial court cannot take cognizance of a case
for lack of jurisdiction over the person of the defendant. Any proceeding
undertaken by the trial court will consequently be null and void.

Page 276 of 1072

TOPIC:MINIMUM SET OF OFFICERS; QUALIFICATIONS; DISQUALIFICATIONS;


AUTHORITY
& LIABILITIES
SUPREME STEEL PIPE CORPORATION and REGAN SY, Petitioners
vs.
ROGELIO BARDAJE, Respondent.
G.R. No. 170811. April 24, 2007
522 SCRA 155
FACTS:
Petitioner Supreme Steel Pipe Corporation (SSPC), a domestic
corporation primarily engaged in the business of manufacturing steel pipes,
employed respondent Rogelio Bardaje as a warehouseman. SSPC employees
were required to wear a uniform (a yellow t-shirt with a logo and the marking
Supreme) while at work.
On August 19, 1999, respondent reported for work at 6:45 a.m. It was
a common practice among warehousemen to wear long-sleeved shirts over
their uniforms to serve as protection from heat and dust while working, and
on this day, respondent had on a green long-sleeved shirt over his uniform.
Momentarily, security guard Christopher Barrios called him in a loud voice,
and arrogantly ordered him to remove and turn-over to him (Barrios) the
long-sleeved shirt. Insulted and feeling singled-out from the other
warehousemen who were also wearing long-sleeved shirts over their
uniforms, respondent replied: Ano ba ang gusto mo, hubarin ko o
magsuntukan na lang tayo sa labas? A heated exchange of words ensued,
but the brewing scuffle between the two was averted by a co-employee from
the Production Division, Albert A. Bation. A security guard, Ricky Narciso, was
able to keep the parties apart. Barrios reported the incident to the SSPC
management. Because of the incident, Bardaje was dismissed.
ISSUE/S:
Can Regan Sy being the President of SSPC be held liable?
RULING:
No.
It appears that respondent impleaded SSPC President Regan Sy only
because he is an officer/agent of the company. However, petitioner Sy
cannot be held solidarily liable with petitioner SSPC for the termination of
respondents employment, since there is no showing that the dismissal was
attended with malice or bad faith.

Page 277 of 1072

TOPIC:MINIMUM SET OF OFFICERS; QUALIFICATIONS; DISQUALIFICATIONS;


AUTHORITY
& LIABILITIES
CAGAYAN VALLEY DRUG CORPORATION, petitioner,
vs.
COMMISSIONER OF INTERNAL REVENUE, respondent.
G.R. No. 151413. February 13, 2008
545 SCRA 10
FACTS:
Petitioner, a corporation duly organized and existing under Philippine
laws, is a duly licensed retailer of medicine and other pharmaceutical
products. It operates two drugstores, one in Tuguegarao, Cagayan, and the
other in Roxas, Isabela, under the name and style of "Mercury Drug."
In 1995, Cagayan granted 20% sales discounts to qualified senior
citizens on purchases of medicine pursuant to RA 7432. It treated such
discounts as deductions from the gross sales pursuant to Revenue Regulation
No. 2-49 instead of treating them as tax credit as provided under Section 4 of
RA 7432.
Cagayan filed with the BIR a claim for tax refund of the full amount of
the 20% sales discount it granted to senior citizens for the year 1995.
BIR failed to act on the petition. Thus, Cagayan filed a petition for
review with the CTA but the latter dismissed the petition for lack of merit.
On appeal to CA, the latter dismissed the petition on the ground that
the person who signed the verification and certification of non-forum
shopping, a certain Jacinto Concepcion, President of Cagayan, failed to
adduce proof that he was duly authorized by the board of directors to do so.
ISSUE/S:
Whether petitioners president can sign the subject verification and
certification sans the approval of its Board of Directors.
RULING:
YES.
Section 23 in relation to Section 25 of the Corporation Code enunciates
that all corporate powers are exercised, all business conducted, and all
properties controlled by the board of directors. Thus, an individual corporate

Page 278 of 1072

officer cannot solely exercise any corporate power pertaining to the


corporation without authority from the board.
However, in some cases decided by SC, the following officials or
employees of the company can sign the verification and certification without
need of a board resolution:
1. The Chairperson of the board of directors (Lepanto Case,
Sept. 24, 2003);
2. the President of a corporation (Lepanto,supra);
3. the General Manager or Acting Manager (Mactan-Cebu
Case, Nov. 7, 2000); Personnel Officer (Novelty Philippines
Case, Sept 17, 2003);
4. an Employment Specialist in Labor case (Pfizer Case, May
25, 2001).
The rationale apllied in the forefoing cases is to justify the authority of
corporate officers to sign being in a position to verify the truthfulness and
correctness of the allegations in the petition.
In the recent case of Philippine Airlines(Jan 24, 2006), SC held that
only individuals vested with authority by a valid board resolution may sign
the verification and certification. In this case, SC said that appending the
board resolution to the complaint or petition is the better procedure to
obviate any question on the authority of the signatory to the verification and
certification.
In the case at bar, Cagayan substantially complied with Secs. 4 and 5,
Rule 7 of the Rules of Court because:
1. the requisite board resolution has been submitted albeit
belatedly;
2. application of Lepanto case with the rationale that the
President of Cagayan is in a position to verify the
truthfulness and correctness of the allegations in the
petition;
3. the President has signed the complaint before the CTA at
the inception of the judicial claim for refund.

Page 279 of 1072

TOPIC:MINIMUM SET OF OFFICERS; QUALIFICATIONS; DISQUALIFICATIONS;


AUTHORITY
& LIABILITIES
VIOLETA TUDTUD BANATE, MARY MELGRID M. CORTEL, BONIFACIO
CORTEL, ROSENDO MAGLASANG, and PATROCINIA MONILAR,
Petitioners,
vs.
PHILIPPINE COUNTRYSIDE RURAL BANK (LILOAN, CEBU), INC. and
TEOFILO SOON, JR., Respondents.
G.R. No. 163825, July 13, 2010
FACTS:
On July 22, 1997, petitioner spouses Rosendo Maglasang and Patrocinia
Monilar obtained a loan from PCRB for P1,070,000.00. The subject loan was
evidenced by a promissory note and was payable on January 18, 1998. To
secure the payment of the subject loan, the spouses Maglasang executed, in
favor of PCRB a real estate mortgage over their property, Lot 12868-H-3-C, 6
including the house constructed thereon, owned by petitioners Mary Melgrid
and Bonifacio Cortel, the spouses Maglasangs daughter and son-in-law,
respectively. Aside from the subject loan, the spouses Maglasang obtained
two other loans from PCRB which were covered by separate promissory notes
and secured by mortgages on their other properties.
Sometime in November 1997 (before the subject loan became due),
the spouses Maglasang and the spouses Cortel asked PCRBs permission to
sell the subject properties. They likewise requested that the subject
properties be released from the mortgage since the two other loans were
adequately secured by the other mortgages. The spouses Maglasang and the
spouses Cortel claimed that the PCRB, acting through its Branch Manager,
Pancrasio Mondigo, verbally agreed to their request but required first the full
payment of the subject loan. The spouses Maglasang and the spouses Cortel
thereafter sold to petitioner Violeta Banate the subject properties for
P1,750,000.00. The spouses Magsalang and the spouses Cortel used the
amount to pay the subject loan with PCRB. After settling the subject loan,
PCRB gave the owners duplicate certificate of title of Lot 12868-H-3-C to
Banate, who was able to secure a new title in her name. The title, however,
carried the mortgage lien in favor of PCRB, prompting the petitioners to
request from PCRB a Deed of Release of Mortgage. As PCRB refused to
comply with the petitioners request, the petitioners instituted an action for
specific performance before the RTC to compel PCRB to execute the release
deed.

Page 280 of 1072

After trial, the RTC ruled in favor of the petitioners. It noted that the
petitioners, as "necessitous men," could not have bargained on equal footing
with PCRB in executing the mortgage, and concluded that it was a contract of
adhesion. Therefore, any obscurity in the mortgage contract should not
benefit PCRB.
On appeal, the CA reversed the RTCs decision. The CA did not consider
as valid the petitioners new agreement with Mondigo, which would novate
the original mortgage contract containing the cross-collateral stipulation. It
ruled that Mondigo cannot orally amend the mortgage contract between
PCRB, and the spouses Maglasang and the spouses Cortel.
ISSUE/S:
Whether or not the act of Mondigo in verbally entering into
agreement with spouses Maglasang and Sps. Cortel binds the
corporation.
RULING:
Section 23 of the Corporation Code expressly provides that the
corporate powers of all corporations shall be exercised by the board of
directors. The power and the responsibility to decide whether the corporation
should enter into a contract that will bind the corporation are lodged in the
board, subject to the articles of incorporation, bylaws, or relevant provisions
of law. In the absence of authority from the board of directors, no person, not
even its officers, can validly bind a corporation.
However, just as a natural person may authorize another to do certain
acts for and on his behalf, the board of directors may validly delegate some
of its functions and powers to its officers, committees or agents. The
authority of these individuals to bind the corporation is generally derived
from law, corporate bylaws or authorization from the board, either expressly
or impliedly by habit, custom or acquiescence in the general course of
business.
Notably, the petitioners action for specific performance is premised on
the supposed actual or apparent authority of the branch manager, Mondigo,
to release the subject properties from the mortgage, although the other
obligations remain unpaid. In light of our discussion above, proof of the
branch managers authority becomes indispensable to support the
petitioners contention. The petitioners make no claim that Mondigo had
actual authority from PCRB, whether express or implied. Rather, adopting the
trial courts observation, the petitioners posited that PCRB should be held
liable for Mondigos commitment, on the basis of the latters apparent
authority.
The Court disagrees with this position.
Under the doctrine of apparent authority, acts and contracts of the
agent, as are within the apparent scope of the authority conferred on him,
although no actual authority to do such acts or to make such contracts has

Page 281 of 1072

been conferred, bind the principal. The principals liability, however, is


limited only to third persons who have been led reasonably to believe by the
conduct of the principal that such actual authority exists, although none was
given. In other words, apparent authority is determined only by the acts of
the principal and not by the acts of the agent. There can be no apparent
authority of an agent without acts or conduct on the part of the principal;
such acts or conduct must have been known and relied upon in good faith as
a result of the exercise of reasonable prudence by a third party as claimant,
and such acts or conduct must have produced a change of position to the
third partys detriment.
In the present case, the decision of the trial court was utterly silent on
the manner by which PCRB, as supposed principal, has "clothed" or "held
out" its branch manager as having the power to enter into an agreement, as
claimed by petitioners. No proof of the course of business, usages and
practices of the bank about, or knowledge that the board had or is presumed
to have of, its responsible officers acts regarding bank branch affairs, was
ever adduced to establish the branch managers apparent authority to
verbally alter the terms of mortgage contracts. Neither was there any
allegation, much less proof, that PCRB ratified Mondigos act or is estopped
to make a contrary claim.
Further, we would be unduly stretching the doctrine of apparent
authority were we to consider the power to undo or nullify solemn
agreements validly entered into as within the doctrines ambit. Although a
branch manager, within his field and as to third persons, is the general agent
and is in general charge of the corporation, with apparent authority
commensurate with the ordinary business entrusted him and the usual
course and conduct thereof, yet the power to modify or nullify corporate
contracts remains generally in the board of directors.26 Being a mere branch
manager alone is insufficient to support the conclusion that Mondigo has
been clothed with "apparent authority" to verbally alter terms of written
contracts, especially when viewed against the telling circumstances of this
case: the unequivocal provision in the mortgage contract; PCRBs vigorous
denial that any agreement to release the mortgage was ever entered into by
it; and, the fact that the purported agreement was not even reduced into
writing considering its legal effects on the parties interests. To put it simply,
the burden of proving the authority of Mondigo to alter or novate the
mortgage contract has not been established.
It is a settled rule that persons dealing with an agent are bound at their
peril, if they would hold the principal liable, to ascertain not only the fact of
agency but also the nature and extent of the agents authority, and in case
either is controverted, the burden of proof is upon them to establish it. As
parties to the mortgage contract, the petitioners are expected to abide by its
terms. The subsequent purported agreement is of no moment, and cannot
prejudice PCRB, as it is beyond Mondigos actual or apparent authority, as
above discussed.

Page 282 of 1072

TOPIC: MINIMUM SET OF OFFICERS; QUALIFICATIONS; DISQUALIFICATIONS;


AUTHORITY AND LIABILITIES:
MILLENIUM INDUSTRIAL COMMERCIAL CORPORATION
vs.
JACKSON TAN
G.R. No. 131724
February 28, 2000
383 Phil. 468
FACTS:
In December 1994, Millenium Industrial Commercial Corporation,
petitioner herein, executed a Deed of Real Estate Mortgage over its real
property covered by TCT No. 24069 in favor of respondent Jackson Tan. The
mortgage was executed to secure payment of petitioner's indebtedness to
respondent in the amount of P2 million, without monthly interest, but which,
at maturity date on June 10, 1995, was payable in the amount of P4 million.
On November 9, 1995, respondent filed against petitioner a complaint
for foreclosure of mortgage in the Regional Trial Court, Branch 6, Cebu City.
On November 21, 1995, summons and a copy of the complaint were served
upon petitioner through a certain Lynverd Cinches, described in the sheriff's
return, dated November 23, 1995, as "a Draftsman, a person of sufficient age
and (discretion) working therein, he is the highest ranking officer or Officerin-Charge of defendant's Corporation, to receive processes of the Court."
Petitioner moved for the dismissal of the complaint on the ground that
there was no valid service of summons upon it, as a result of which the trial
court did not acquire jurisdiction over it. Petitioner invoked Rule 14, 13 of
the 1964 Rules of Court and contended that service on Lynverd Cinches, as
alleged in the sheriff's return, was invalid as he is not one of the authorized
persons on whom summons may be served and that, in fact, he was not
even its employee.

Page 283 of 1072

Petitioner also sought the dismissal of the complaint against it on the


ground that it had satisfied its obligation to respondent. Petitioner further
prayed for "other reliefs just and equitable under the premises." On
December 15, 1995, the trial court denied petitioner's Motion to Dismiss. The
petitioners motion for reconsideration was denied by the trial court. The
Court of Appeals dismissed the petitioners petition for certiorari.
Hence, this petition.

ISSUE:
Whether it is allowable to merely infer actual receipt of summons by
the corporation through the person on whom summons was served.
RULING:
No.
Summons is the means by which the defendant in a case is notified of
the existence of an action against him and, thereby, the court is conferred
jurisdiction over the person of the defendant. If the defendant is corporation,
Rule 14, 13 requires that service of summons be made upon the
corporations president, manager, secretary, cashier, agent, or any of its
directors. The rationale of the rule is that service must be made on a
representative so integrated with the corporation sued as to make it a
priori presumable that he will realize his responsibilities and know what he
should do with any legal papers received by him.
Petitioner contends that the enumeration in Rule 14, 13 is exclusive
and that service of summons upon one who is not enumerated therein is
invalid. This is the general rule. However, it is settled that substantial
compliance by serving summons on persons other than those mentioned in
the above rule may be justified. We hold that it cannot be allowed. For there
to be substantial compliance, actual receipt of summons by the corporation
through the person served must be shown. Where a corporation only learns
of the service of summons and the filing of the complaint against it through
some person or means other than the person actually served, the service of
summons becomes meaningless. This is particularly true in the present case
where there is serious doubt if Lynverd Cinches, the person on whom service
of summons was effected, is in fact an employee of the corporation. Except
for the sheriff's return, there is nothing to show that Lynverd Cinches was
really a draftsman employed by the corporation.

Page 284 of 1072

TOPIC: MINIMUM SET OF OFFICERS; QUALIFICATIONS; DISQUALIFICATIONS;


AUTHORITY AND LIABILITIES:
SALOME PABON and VICENTE CAMONAYAN
vs.
NATIONAL LABOR RELATIONS COMMISSION and SENIOR MARKETING
CORPORATION
G.R. No. 120457
September 24, 1998
357 Phil. 7
FACTS:
On May 24, 1994 and June 22, 1994, complaints for illegal dismissal
and non-payment of benefits were filed by petitioners Salome Pabon and
Vicente Camonayan against private respondent Senior Marketing Corporation
(SMC) and its Field Manager, R-Jay Roxas Summons and notices of hearings
were sent to Roxas at private respondents provincial office in 13 Valley
Homes, Patul Road, Santiago, Isabela which were received by its bookkeeper,
Mina Villanueva.
On September 15, 1994, the Labor Arbiter rendered a judgment by
default after finding that private respondent tried to evade all the summons
and orders of hearing by refusing to claim all the registered mail addressed
to it.
Instead of appealing the Labor Arbiters decision to the National Labor
Relations Commission (NLRC), within ten (10) days from receipt of the said
decision, private respondent filed a motion for reconsideration/new
trial before the Labor Arbiter. It was only after the said ten-day period had
lapsed that private respondent appealed to the NLRC which, in a
Decision promulgated on March 31, 1995, setting aside the Labor Arbiters
decision.
Hence, this petition.
ISSUE/S:

Page 285 of 1072

Whether respondent was properly served with summons in


accordance with the Rules of Court through its bookkeeper at its
provincial office address.
RULING:
Yes.
We are of the view that a bookkeeper can be considered as an agent of
private Respondent Corporation within the purview of Section 13, Rule 14 of
the old Rules of Court. The rationale of all rules with respect to service of
process on a corporation is that such service must be made to an agent of a
representative so integrated with the corporation sued as to make it
a priori supposable that he will realize his responsibilities and know what he
should do with any legal papers served on him. The bookkeepers task is one
under consideration. The job of a bookkeeper is so integrated with the
corporation that his regular recording of the corporations business
accounts and essential facts about the transactions of a business enterprise
safeguards the corporation from possible fraud being committed adverse to
its own corporate interest.
Although it may be true that the service of summons was made on a
person not authorized to receive the same in behalf of the petitioner,
nevertheless since it appears that the summons and complaint were in fact
received by the corporation through its said clerk, the Court finds that there
was a substantial compliance with the rule on service of summons. Indeed
the purpose of said rule as above stated to assure service of summons on
the corporation had thereby been attained. The need for speedy justice must
prevail over technicality.

Page 286 of 1072

TOPIC: MINIMUM SET OF OFFICERS; QUALIFICATIONS;


DISQUALIFICATIONS; AUTHORITY AND LIABILITIES:
VLASON ENTERPRISES CORPORATION
vs.
COURT OF APPEALS and DURAPROOF SERVICES, represented by its
General Manager, Cesar Urbino Sr.
G.R. Nos. 121662-64
July 6, 1999
310 SCRA 26
FACTS:
Poro Point Shipping Services, then acting as the local agent of Omega
Sea Transport Company of Honduras & Panama, a Panamanian company,
(hereafter referred to as Omega), requested permission for its vessel M/V
Star Ace, which had engine trouble, to unload its cargo and to store it at the
Philippine Ports Authority (PPA) compound in San Fernando, La Union while
awaiting transhipment to Hongkong. The request was approved by the
Bureau of Customs. Despite the approval, the customs personnel boarded
the vessel when it docked on January 7, 1989, on suspicion that it was the
hijacked M/V Silver Med owned by Med Line Philippines Co., and that its
cargo would be smuggled into the country. The district customs collector
seized said vessel and its cargo pursuant to Section 2301, Tariff and Customs
Code. A notice of hearing of SFLU Seizure Identification No. 3-89 was served
on its consignee, Singkong Trading Co. of Hongkong, and its shipper, Dusit
International Co., Ltd. of Thailand.
While seizure proceedings were ongoing, La Union was hit by three
typhoons, and the vessel ran aground and was abandoned. On June 8, 1989,
its authorized representative, Frank Cadacio, entered into a salvage
agreement with private respondent to secure and repair the vessel at the
agreed consideration of $1 million and fifty percent (50%) of the cargo after
all expenses, cost and taxes.
Finding that no fraud was committed, the District Collector of Customs,
Aurelio M. Quiray, lifted the warrant of seizure on July 16, 1989. However, in
a Second Indorsement dated November 11, 1989, then Customs
Commissioner Salvador M. Mison declined to issue a clearance for Quirays
Decision; instead, he forfeited the vessel and its cargo in accordance with
Section 2530 of the Tariff and Customs Code. Accordingly, acting District
Collector of Customs John S. Sy issued a Decision decreeing the forfeiture
and the sale of the cargo in favor of the government.
To enforce its preferred salvors lien, herein Private Respondent
Duraproof Services filed with the Regional Trial Court of Manila a Petition
for Certiorari,
Prohibition
and Mandamus assailing
the
actions
of
Commissioner Mison and District Collector Sy. Also impleaded as

Page 287 of 1072

respondents; were PPA Representative Silverio Mangaoang and Med Line


Philippines, Inc.
On January 10, 1989, private respondent amended its Petition to
include former District Collector Quiray; PPA Port Manager Adolfo Ll. Amor Jr;
Petitioner Vlason Enterprises as represented by its president, Vicente
Angliongto; Singkong Trading Company as represented by Atty. Eddie
Tamondong; Banco Du Brasil; Dusit International Co., Inc.; Thai-Nan
Enterprises Ltd. and Thai-United Trading Co., Ltd. In both Petitions, private
respondent plainly failed to include any allegation pertaining to petitioner, or
any prayer for relief against it.
Summonses for the amended Petition were served on Atty. Joseph
Capuyan for Med Line Philippines: Angliongto (through his secretary, Betty
Bebero), Atty. Tamondong and Commissioner Mison. Upon motion of the
private respondent, the trial court allowed summons by publication to be
served upon the alien defendants who were not residents and had no direct
representatives in the country.
After much ado, the CA ruled that the judgment sought to be reviewed
has become final and executory and ordered the Regional Trial Court to take
appropriate action on the urgent ex-parte motion for issuance of a writ of
execution filed by private respondent. Pursuant thereto, the Regional Trial
Court of Manila issued a writ of possession thus placing private respondent in
possession of petitioners barge Lawin.
Hence, this petition.
ISSUE:
Whether Petitioner Company was served with summons.
RULING:
Yes.
A corporation may be served summons through its agents or officers
who under the Rules are designated to accept service of process. A
summons addressed to a corporation and served on the secretary of its
president binds that corporation. This is based on the rationale that service
must be made on a representative so integrated with the corporation sued,
that it is safe to assume that said representative had sufficient responsibility
and discretion to realize the importance of the legal papers served and to
relay the same to the president or other responsible officer of the corporation
being sued. The secretary of the president satisfies this criterion. This rule
requires, however, that the secretary should be an employee of the
corporation sought to be summoned. Only in this manner can there be an
assurance that the secretary will bring home to the corporation [the] notice
of the filing of the action against it.

Page 288 of 1072

In the present case, Bebero was the secretary of Angliongto, who was
president of both VSI and petitioner, but she was an employee of VSI, not of
petitioner. The piercing of the corporate veil cannot be resorted to when
serving summons. Doctrinally, a corporation is a legal entity distinct and
separate from the members and stockholders who compose it. However,
when the corporate fiction is used as a means of perpetrating a fraud,
evading an existing obligation, circumventing a statute, achieving or
perfecting a monopoly or, in generally perpetrating a crime, the veil will be
lifted to expose the individuals composing it. None of the foregoing
exceptions has been shown to exist in the present case. Quite the contrary,
the piercing of the corporate veil in this case will result in manifest
injustice. This we cannot allow. Hence, the corporate fiction remains.

Page 289 of 1072

TOPIC:MINIMUM SET OF OFFICERS; QUALIFICATIONS; DISQUALIFICATIONS;


AUTHORITY AND LIABILITIES:
PRIME WHITE CEMENT CORPORATION
vs.
HONORABLE INTERMEDIATE APPELLATE COURT and ALEJANDRO TE
G.R. No. L-68555
March 19, 1993
220 SCRA 103
FACTS:
On or about 16 July 1969, Alejandro Te and Prime White Cement
Corporation (PWCC) thru its President, Mr. Zosimo Falcon and Justo C. Trazo,
as Chairman of the Board, entered into a dealership agreement whereby Te
was obligated to act as the exclusive dealer and/or distributor of PWCC of its
cement products in the entire Mindanao area for a term of 5 years and
providing among others that (a) the corporation shall, commencing
September, 1970, sell to and supply Te, as dealer with 20,000 bags (94
lbs/bag) of white cement per month; (b) Te shall pay PWCC P9.70, Philippine
Currency, per bag of white cement, FOB Davao and Cagayan de Oro ports;
(c) Te shall every time PWCC is ready to deliver the good, open with any bank
or banking institution a confirmed, unconditional, and irrevocable letter of
credit in favor of PWCC and that upon certification by the boat captain on the
bill of lading that the goods have been loaded on board the vessel bound for
Davao the said bank or banking institution shall release the corresponding
amount as payment of the goods so shipped." Right after Te entered into the
dealership agreement, he placed an advertisement in a national, circulating
newspaper the fact of his being the exclusive dealer of PWWC's white
cement products in Mindanao area, more particularly, in the Manila Chronicle
dated 16 August 1969 and was even congratulated by his business
associates, so much so, he was asked by some of his businessmen friends
and close associates if they can be his sub-dealer in the Mindanao area.
Relying heavily on the dealership agreement, Te sometime in the months of
September, October, and December, 1969, entered into a written agreement
with several hardware stores dealing in buying and selling white cement in
the Cities of Davao and Cagayan de Oro which would thus enable him to sell
his allocation of 20,000 bags regular supply of the said commodity, by
September, 1970.
After Te was assured by his supposed buyer that his allocation of
20,000 bags of white cement can be disposed of, he informed the defendant
corporation in his letter dated 18 August 1970 that he is making the
necessary preparation for the opening of the requisite letter of credit to
cover the price of the due initial delivery for the month of September 1970,
looking forward to PWCC's duty to comply with the dealership agreement. In
reply to the aforesaid letter of Te, PWCC thru its corporate secretary, replied

Page 290 of 1072

that the board of directors of PWCC decided to impose the following


conditions: (a) Delivery of white cement shall commence at the end of
November, 1970; (b) Only 8,000 bags of white cement per month for only a
period of three (3) months will be delivered; (c) The price of white cement
was priced at P13.30 per bag; (d) The price of white cement is subject to
readjustment unilaterally on the part of the defendant; (e) The place of
delivery of white cement shall be Austurias; (f) The letter of credit may be
opened only with the Prudential Bank, Makati Branch; (g) Payment of white
cement shall be made in advance and which payment shall be used by the
defendant as guaranty in the opening of a foreign letter of credit to cover
costs and expenses in the procurement of materials in the manufacture of
white cement. Several demands to comply with the dealership agreement
were made by Te to PWCC, however, PWCC refused to comply with the same,
and Te by force of circumstances was constrained to cancel his agreement
for the supply of white cement with third parties, which were concluded in
anticipation of, and pursuant to the said dealership agreement.
Notwithstanding that the dealership agreement between Te and PWCC was in
force and subsisting, PWCC, in violation of, and with evident intention not to
be bound by the terms and conditions thereof, entered into an exclusive
dealership agreement with a certain Napoleon Co for the marketing of white
cement in Mindanao. Te filed suit. After trial, the trial court adjudged PWCC
liable to Alejandro Te in the amount of P3,302,400.00 as actual
damages,P100,000.00 as moral damages, and P10,000 00 as and for
attorney's fees and costs. The appellate court affirmed the said decision.
Hence, this petition.
ISSUE/S:
Whether or not the "dealership agreement" referred by the
President and Chairman of the Board of petitioner corporation is a
valid and enforceable contract.
RULING:
No.
Under the Corporation Law, which was then in force at the time this
case arose, as well as under the present Corporation Code, all corporate
powers shall be exercised by the Board of Directors, except as otherwise
provided by law. Although it cannot completely abdicate its power and
responsibility to act for the juridical entity, the Board may expressly delegate
specific powers to its President or any of its officers. In the absence of such
express delegation, a contract entered into by its President, on behalf of the
corporation, may still bind the corporation if the board should ratify the same
expressly or impliedly. Implied ratification may take various forms like
silence or acquiescence; by acts showing approval or adoption of the
contract; or by acceptance and retention of benefits flowing therefrom.
Furthermore, even in the absence of express or implied authority by
ratification, the President as such may, as a general rule, bind the

Page 291 of 1072

corporation by a contract in the ordinary course of business, provided the


same is reasonable under the circumstances. These rules are basic, but are
all general and thus quite flexible. They apply where the President or other
officer, purportedly acting for the corporation, is dealing with a third
person, i. e., a person outside the corporation.
The situation is quite different where a director or officer is dealing with
his own corporation. In the instant case respondent Te was not an ordinary
stockholder; he was a member of the Board of Directors and Auditor of the
corporation as well. He was what is often referred to as a "self-dealing"
director. A director of a corporation holds a position of trust and as such, he
owes a duty of loyalty to his corporation. In case his interests conflict with
those of the corporation, he cannot sacrifice the latter to his own advantage
and benefit. As corporate managers, directors are committed to seek the
maximum amount of profits for the corporation. This trust relationship "is not
a matter of statutory or technical law. It springs from the fact, that directors
have the control and guidance of corporate affairs and property and hence,
of the property interests of the stockholders."

Page 292 of 1072

TOPIC:MINIMUM SET OF OFFICERS; QUALIFICATIONS; DISQUALIFICATIONS;


AUTHORITY AND LIABILITIES:
LOUIS VUITTON S.A.
vs.
JUDGE FRANCISCO DIAZ VILLANUEVA, presiding Judge, Branch 36,
The Metropolitan Trial Court at Quezon City, Metro Manila
A.M. No. MTJ-92-643
November 27, 1992
216 SCRA 121
FACTS:
The complaining witness in this case is the representative and
attorney-in-fact, counsel of Louis Vuitton, S.A. French Company with business
address at Paris, France. Private complainant is suing the accused for the
protection of the trade mark Louis Vuitton and the L.V. logo which are duly
registered with the Philippine Patent Office. On May 10, 1989, Atty. Felino
Padlan of the Quasha Law Office brought a letter to the COD informing the
latter to cease and desist from selling leather articles bearing the trade
marks Louis Vuitton and L.V. logo as the same is the registered trademarks
belonging to the private complainant which has not authorized any person in
the Philippines to sell such articles. On September 6, 1989, Mrs. Domingo
bought from the same store a wallet with a trade mark and logo of Louis
Vuitton. On September 28 1989, the NBI, upon the request of the Quasha law
Firm applied for a Search Warrant at the Metropolitan Trial Court in Quezon
City; that the application was granted and the Search Warrant was issued
against COD and was enforced on the same date. From the implementation
of the said Search Warrant, about seventy-two (72) leather products were
seized. The accused signed the inventory of the seized articles.
The accused, on the other hand, claimed: that he is not the
manufacturer or seller of the seized articles; that the said articles were sold
in the store by a concessionaire by the name of Erlinda Tan who is doing
business under the name of Hi-Tech Bags and wallets. The trial court
acquitted the accused.
Hence, this petition.
ISSUE/S:
Whether Jose V. Rosarion who was accused as owner/proprietor of
COD was not properly charged as his personality is distinct from
that of the COD's.
RULING:
Jose v. Rosario was charged as owner/proprietor. COD is not a single
proprietorship but one that is run and owned by a corporation, Rosario Bros.,

Page 293 of 1072

Inc., of which the accused is stockholder and Executive Vice-President. A


stockholder generally does not have a hand in the management of the
corporate affairs. On the other hand, the Vice-President had no inherent
power to bind the corporation. As general rule, his duties must be specified
in the by-laws. In the criminal case, the information did not specify his duties
as Executive Vice-President. The trial court had no basis for holding that as
such, the accused entered into a contract with the concessionaire thereby
giving the latter an opportunity to practice unfair competition. Whereas,
Section 23 of the Corporation Code is explicit that the directors, acting as a
body, exercise corporation powers and conduct the corporation's business.
The board has the sole power and responsibility to decide whether a
corporation should enter into any contract or perform any act. The
amendment of the charge, as proposed by the private prosecutor, would not
in any way affect the application of the doctrine that the corporation has a
personality distinct from that of its owners.
Moreover, the finding of the trial court that there is no unfair
competition rendered the consideration of the motions insignificant. If there
was unfair competition, so would there be no offense of giving others an
opportunity to engage in unfair competition since there was no unfair
competition to begin with.

Page 294 of 1072

TOPIC: DOCTRINE OF APPARENT AUTHORITY


SARGASSO CONSTRUCTION & DEVELOPMENT CORPORATION/PICK &
SHOVEL, INC.,/ATLANTIC ERECTORS, INC. (JOINT VENTURE)
VS.
PHILIPPINE PORTS AUTHORITY
G.R. No. 170530, July 5, 2010
FACTS:
Plaintiff Sargasso Construction and Development Corporation, Pick and
Shovel, Inc. and Atlantic Erectors, Inc., a joint venture, was awarded the
construction of Pier 2 and the rock causeway (R.C. Pier 2) for the port of San
Fernando, La Union, after a public bidding conducted by the defendant PPA.
Implementation of the project commenced on August 14, 1990. Adjacent to
Pier 2 is an area of P4,280 square meters intended for the reclamation
project as part of the overall port development plan.
In a letter dated October 1, 1992 of Mr. Melecio J. Go, Executive
Director of the consortium, plaintiff offered to undertake the reclamation
between the Timber Pier and Pier 2 of the Port of San Fernando, La Union, as
an extra work to its existing construction of R.C. Pier 2 and Rock Causeway
for a price of P36,294,857.03. Defendant replied thru its Assistant General
Manager Teofilo H. Landicho in a letter that the proposed project cost is not
acceptable but the proposal may be considered if the price be reduced.
On August 26, 1993, a Notice of Award signed by PPA General Manager
Rogelio Dayan was sent to plaintiff for the phase I Reclamation Contract in
the reduced amount and instructing it to "enter into and execute the contract
agreement with this Office" and to furnish the documents representing
performance security and credit line.
At its meeting held on September 9, 1994, the Board decided not to
approve the contract proposal since there is no strong legal basis for
Management to award the supplemental contract through negotiation. The
Board noted that the Pier 2 Project was basically for the construction of a pier
while the supplemental agreement refers to reclamation. Thus there is no
basis to compare the terms and conditions of the reclamation project with
the original contract.
On June 30, 1997, plaintiff filed a complaint for specific performance
and damages against respondent.

Page 295 of 1072

ISSUE/S:
Whether or not the general manager of PPA is vested with authority
to enter into a contract for and on behalf of PPA.
RULING:
No.
Under Section 51 of contracts in behalf of the Republic of the
Philippines shall be executed by the President unless authority therefore is
expressly vested by law or by him in any other public officer.
Contracts in behalf of the political subdivisions and corporate agencies
or instrumentalities shall be approved by their respective governing boards
or councils and executed by their respective executive heads. Contracts to
which the government is a party are generally subject to the same laws and
regulations which govern the validity and sufficiency of contracts between
private individuals. A government contract, however, is perfected only upon
approval by a competent authority, where such approval is required.
P.D. 857 states that one of the corporate powers of respondents Board
of Directors is to "reclaim any part of the lands vested in the Authority." It
also "exercises all the powers of a corporation under the Corporation Law."
On the other hand, the law merely vests the general manager the "general
power to sign contracts" and "to perform such other duties as the Board
may assign" Therefore, unless respondents Board validly authorizes its
general manager, the latter cannot bind respondent PPA to a contract.
Precisely, the Board of Directors of the respondent did not see fit to
approve the contract by negotiation after finding that "the Pier 2 Project was
basically for the construction of a pier while the supplemental agreement
refers to reclamation. Thus, there is no basis to compare the terms and
conditions of the reclamation project with the original contract (Pier 2
Project) of Sargasso." The negotiated contract itself basically contravenes
stringent legal requirements aimed at protecting the interest of the public.
The facts here do not conform to what the law requires.

Page 296 of 1072

TOPIC: DOCTRINE OF APPARENT AUTHORITY


EMILIANO ACUA,
vs.
BATAC PRODUCERS COOPERATIVE MARKETING ASSOCIATION, INC. et
al
G.R. No. L-20333 June 30, 1967
FACTS:
On August 9, 1962, Emiliano Acua filed a complaint, which was later
amended on August 13, against the defendant Batac Producers Cooperative
Marketing Association, Inc.The complaint alleged, inter alia, that on or about
May 5, 1962 it was tentatively agreed upon between plaintiff and defendant
Leon Q. Verano, as Manager of the defendant Batac Procoma, Inc., that the
former would seek and obtain the sum of not less, than P20,000.00 to be
advanced to the defendant Batac Procoma, Inc., to be utilized by it as
additional funds for its Virginia tobacco buying operations during the current
redrying season.
The said tentative agreement was favorably received by the Board of
Directors of the defendant Batac Procoma Inc., and on May 6, 1962 all the
defendants named above, who constituted the entire Board of Directors of
said corporation unanimously authorized defendant Leon Q. Verano, by a
formal resolution. He was assured by these defendants that a formal
approval of said "Agreement" by the Board was no longer necessary, as it
was a mere "formality" appended to its authorizing resolution and as all the
members of the Board had already agreed to the same. But the "Agreement"
was disapproved by its Board of Directors on June 6, 1962.
On September 10, 1962, the trial court dismissed the complaint .
ISSUE/S:
Whether or not the agreement is binding between the parties
RULING:
Yes.
There is abundant authority in support of the proposition that
ratification may be express or implied, and that implied ratification may take
diverse forms, such as by silence or acquiescence; by acts showing approval
or adoption of the contract; or by acceptance and retention of benefits
flowing therefrom.

Page 297 of 1072

The very resolution of the Board of Directors relied upon by defendants


appears to militate against their contention. It refers to plaintiff's failure to
comply with certain promises he had made, as well as to his interpretation of
the contract with respect to his remuneration which, according to the Board,
was contrary to the intention of the parties. The resolution then proceeds to
"disapprove and/or rescind" the said contract. The idea of conflicting
interpretation, or rescission on the ground that one of the parties has failed
to fulfill his obligation under the contract, is certainly incompatible with
defendants' theory here that no contract had yet been perfected for lack of
approval by the Board of Directors.
Hence, the order of the trial court is set aside and the case was
remanded for further proceedings.

TOPIC: DOCTRINE OF APPARENT AUTHORITY

Page 298 of 1072

THE BOARD OF LIQUIDATORS representing THE GOVERNMENT OF


THE REPUBLIC OF THE PHILIPPINES, plaintiff-appellant,
vs.
HEIRS OF MAXIMO M. KALAW, JUAN BOCAR, ESTATE OF THE
DECEASED CASIMIRO GARCIA, and LEONOR MOLL, defendantsappellees.
G.R. No. L-18805. August 14, 1967
FACTS:
The National Coconut Corporation (NACOCO, for short) was chartered
as a non-profit governmental organization on May 7, 1940 by Commonwealth
Act 518 avowedly for the protection, preservation and development of the
coconut industry in the Philippines. On August 1, 1946, NACOCO's charter
was amended [Republic Act 5]. General manager and board chairman was
Maximo M. Kalaw; defendants Juan Bocar and Casimiro Garcia were members
of the Board; defendant Leonor Moll became director only on December 22,
1947.
NACOCO, after the passage of Republic Act 5, embarked on copra trading
activities. Several contracts executed by general manager Kalaw became
sources of dispute. An unhappy chain of events conspired to deter NACOCO
from fulfilling these contracts. Four devastating typhoons visited the
Philippines. Coconut trees throughout the country suffered extensive
damage. Copra production decreased. Cash requirements doubled.
Deprivation of export facilities increased the time necessary to accumulate
shiploads of copra. Quick turnovers became impossible, financing a problem.
When it became clear that the contracts would be unprofitable Kalaw
submitted them to the board for approval. It was not until December 22,
1947 when the membership was completed. Defendant Moll took her oath on
that date. A meeting was then held. Kalaw made a full disclosure of the
situation apprised the board of the impending heavy losses. No action was
taken on the contracts. Neither did the board vote thereon at the meeting of
January 7, 1948 following. Then, on January 11, 1948, President Roxas made
a statement that the NACOCO head did his best to avert the losses,
emphasized that government concerns faced the same risks that confronted
private companies, that NACOCO was recouping its losses, and that Kalaw
was to remain in his post. Not long thereafter, that is, on January 30, 1948,
the board met again with Kalaw, Bocar, Garcia and Moll in attendance. They
unanimously approved the contracts. As was to be expected, NACOCO but
partially performed the contracts.
The buyers threatened damage suits. Some of the claims were settled.
But one buyer, Louis Dreyfus & Co. (Overseas) Ltd., did in fact sue before the
Court of First Instance of Manila. The corporation thereunder paid Dreyfus
P567,024.52 representing 70% of the total claims. With particular reference
to the Dreyfus claims, NACOCO put up the defenses that: (1) the contracts
were void because Louis Dreyfus & Co. (Overseas) Ltd. did not have license

Page 299 of 1072

to do business here; and (2) failure to deliver was due to force majeure, the
typhoons.
All the settlements sum up to P1,343,274.52. In 1949, NACOCO seeks
to recover the said sum from general manager and board chairman Maximo
M. Kalaw, and directors Juan Bocar, Casimiro Garcia and Leonor Moll. It
charges Kalaw with negligence under Article 2176, new Civil Code; and
defendant board members, including Kalaw, with bad faith and/or breach
trust for having approved the contracts. The lower court came out with a
judgment dismissing the complaint without costs as well as defendant's
counterclaims, except that plaintiff was ordered to pay the heirs of Maximo
Kalaw the sum of P2,601.94 for unpaid salaries and cash deposit due the
deceased Kalaw from NACOCO.
ISSUE/S:
Did the Board of Liquidators has lost its legal personality to continue
with this suit due to its
winding-up?
Is the Defendant's second that the action is unenforceable against
the heirs of Kalaw because it is a personal action and at that did not
survive his death correct?
Did Kalaw justifiedly enter into the controverted contracts without
the prior approval of the corporation's directorate?
RULING:
1. NO.
The three methods by which a corporation may wind up its affairs: (1)
under Section 3, Rule 104, of the Rules of Court [which superseded Section
66 of the Corporation Law]
whereby, upon voluntary dissolution of a
corporation, the court may direct "such disposition of its assets as justice
requires, and may appoint a receiver to collect such assets and pay the
debts of the corporation;" (2) under Section 77 of the Corporation Law,
whereby a corporation whose corporate existence is terminated, "shall
nevertheless be continued as a body corporate for three years after the time
when it would have been so dissolved, for the purpose of prosecuting and
defending suits by or against it and of enabling it gradually to settle and
close its affairs, to dispose of and convey its property and to divide its capital
stock, but not for the purpose of continuing the business for which it was
established"; and (3) under Section 78 of the Corporation Law, by virtue of
which the corporation, within the three-year period just mentioned, "is
authorized and empowered to convey all of its property to trustees for the
benefit of members, stockholders, creditors, and others interested."
Section 1 of Executive Order 372, whereby the corporate existence of
NACOCO was continued for a period of three years from the effectivity
of the order for "the purpose of prosecuting and defending suits by or
against it and of enabling the Board of Liquidators gradually to settle
and close its affairs, to dispose of and convey its property in the

Page 300 of 1072

manner hereinafter provided", is to be read not as an isolated provision


but in conjunction with the whole. So reading, it will be readily
observed that no time limit has been tackled to the existence of the
Board of Liquidators and its function of closing the affairs of the various
government-owned corporations, including NACOCO.
By Section 2 of the executive order, while the boards of directors of the
various corporations were abolished, their powers and functions and
duties under existing laws were to be assumed and exercised by the
Board of Liquidators. The President thought it best to do away with the
boards of directors of the defunct corporations; at the same time,
however, the President had chosen to see to it that the Board of
Liquidators step into the vacuum. And nowhere in the executive order
was there any mention of the lifespan of the Board of Liquidators. A
glance at the other provisions of the executive order buttresses our
conclusion. Thus, liquidation by the Board of Liquidators may, under
section 1, proceed in accordance with law, the provisions of the
executive order, "and/or in such manner as the President of the
Philippines may direct. Implicit in all these, is that the term of life of the
Board of Liquidators is without time limit. Executive Order 372, the
government, the sole stockholder, abolished NACOCO, and placed its
assets in the hands of the Board of Liquidators. The Board of
Liquidators thus became the trustee on behalf of the government. It
was an express trust. The legal interest became vested in the trustee
the Board of Liquidators. The beneficial interest remained with the
sole stockholder the government. At no time had the government
withdrawn the property, or the authority to continue the present suit,
from the Board of Liquidators. The provisions of Section 78 of the
Corporation Law the third method of winding up corporate affairs
find application.
2. NO.
The suit here revolves around the alleged negligent acts of Kalaw for
having entered into the questioned contracts without prior approval of the
board of directors, to the damage and prejudice of plaintiff; and is against
Kalaw and the other directors for having subsequently approved the said
contracts in bad faith and/or breach of trust. Clearly then, the present case is
not a mere action for the recovery of money nor a claim for money arising
from contract. The suit involves alleged tortious acts. And the action is
embraced in suits filed "to recover damages for an injury to person or
property, real or personal," which survive.
3. YES.
Settled jurisprudence has it that where similar acts have been
approved by the directors as a matter of general practice, custom, and
policy, the general manager may bind the company without formal

Page 301 of 1072

authorization of the board of directors. In varying language, existence of


such authority is established, by proof of the course of business, the usages
and practices of the company and by the knowledge which the board of
directors has, or must be presumed to have, of acts and doings of its
subordinates in and about the affairs of the corporation. So also,
". . . authority to act for and bind a corporation may be presumed from acts
of recognition in other instances where the power was in fact exercised."
". . . Thus, when, in the usual course of business of a corporation, an officer
has been allowed in his official capacity to manage its affairs, his authority to
represent the corporation may be implied from the manner in which he has
been permitted by the directors to manage its business." In the case at bar,
the practice of the corporation has been to allow its general manager to
negotiate and execute contracts in its copra trading activities for and in
NACOCO's behalf without prior board approval. If the by-laws were to be
literally followed, the board should give its stamp of prior approval on all
corporate contracts. But that board itself, by its acts and through
acquiescence practically laid aside the by-law requirement of prior approval.
Under the given circumstances, the Kalaw contracts are valid corporate acts.

Page 302 of 1072

TOPIC: DOCTRINE OF APPARENT AUTHORITY


TRINIDAD J. FRANCISCO, plaintiff-appellee,
vs.
GOVERNMENT SERVICE INSURANCE SYSTEM, defendant-appellant.
G.R. No. L-18287. March 30, 1963
TRINIDAD J. FRANCISCO, plaintiff-appellant,
vs.
GOVERNMENT SERVICE INSURANCE SYSTEM, defendant-appellee.
G.R. No. L-18155. March 30, 1963
FACTS:
On 10 October 1956, Trinidad J. Francisco, in consideration of a loan in
the amount of P400,000.00, out of which the sum of P336,100.00 was
released to her, mortgaged in favor of the defendant, GSIS a parcel of land.
On 6 January 1959, the System extrajudicially foreclosed the mortgage on
the ground that up to that date the plaintiff-mortgagor was in arrears on her
monthly installments in the amount of P52,000.00. The System itself was the
buyer of the property in the foreclosure sale.
On 20 February 1959, the plaintiff's father, Atty. Vicente J. Francisco,
sent a letter for a proposal of payment to the general manager of the
defendant corporation. On the same day Francisco received a telegram
approving his request by the board of the GSIS. The defendant received the
amount of P30,000.00, and issued therefor its official receipt No. 1209874,
dated 4 March 1959. It did not, however, take over the administration of the
compound. GSIS sent 3 letters, one dated 29 January 1960, which was signed
by its assistant general manager, and the other two letters, dated 19 and 26
February 1960, respectively, which were signed by Andal, asking the plaintiff
for a proposal for the payment of her indebtedness, since according to the
System the one-year period for redemption had expired.
In reply, Atty. Francisco sent a letter, dated 11 March 1960, protesting
against the System's request for proposal of payment and inviting its
attention to the concluded contract generated by his offer of 20 February
1959, and its acceptance by telegram of the same date.
ISSUE/S:
Whether or not the offer of compromise has been validly
accepted

Page 303 of 1072

RULING:
Yes.
The offer of compromise made by plaintiff in the letter, Exhibit "A", had
been validly accepted, and was binding on the defendant. The terms of the
offer were clear, and over the signature of defendant's general manager,
Rodolfo Andal, plaintiff was informed telegraphically that her proposal had
been accepted. There was nothing in the telegram that hinted at any
anomaly, or gave ground to suspect its veracity, and the plaintiff, therefore,
can not be blamed for relying upon it.
If a private corporation intentionally or negligently clothes its officers
or agents with apparent power to perform acts for it, the corporation will be
estopped to deny that such apparent authority is real, as to innocent third
persons dealing in good faith with such officers or agents. Hence, even if it
were the board secretary who sent the telegram, the corporation could not
evade the binding effect produced by the telegram.

Page 304 of 1072

TOPIC: DOCTRINE OF APPARENT AUTHORITY


NEW DURAWOOD CO., INC. petitioner,
vs.
COURT OF APPEALS, HON. FELIX S. CABALLES, as Judge, RTC of
Antipolo, Rizal, Branch 71, WILSON M. GAW, ORLANDO S. BONGAT ,
DURAWOOD CONSTRUCTION AND LUMBER SUPPLY CO., INC.,
respondents.
G.R. No. 111732, Feb 20, 1996
FACTS:
On February 14, 1990, a "Petition for Judicial Reconstitution of the Lost
Owner's Duplicate Certificates of TCT Nos. 140486; 156454 and 140485"
was filed in the Regional Trial Court, Branch LXXI, Antipolo, Rizal by
petitioner-corporation, represented by its Branch Manager, Wilson M. Gaw.
Attached to said petition was an "Affidavit of Loss" of respondent Orlando S.
Bongat, one of the stockholders of petitioner-corporation.
Finding the petition "to be sufficient in form and in substance,"
respondent Judge set the case for hearing on March 18, 1991. On April 16,
1991, respondent Judge issued the questioned order.
Sometime in May, 1991, petitioner discovered that the original TCT
Nos. N-140485, N-140486 and 156454 on file with the Register of Deeds of
Rizal had been cancelled and, in lieu thereof, TCT Nos. 200100, 200101 and
200102 had been issued in the name of respondent Durawood Construction
and Lumber Supply, Inc. Surprised by this cancellation, petitioner after
investigation found out about the reconstitution proceeding in the
respondent trial court. So, on July 17, 1991, petitioner filed suit in the Court
of Appeals praying for the annulment of the assailed order in LRC Case No.
91-924 penned by respondent Judge. It also prayed for the cancellation of
the new certificates (TCT Nos. 200100, 200101 and 200102). On May 31,
1993, the respondent Court of Appeals rendered the assailed Decision and
on August 30, 1993, the Resolution denying the motion for reconsideration.
ISSUE/S:
Whether or not Gaw's act of filing the reconstitution proceedings
is binding upon petitioner-corporation

Page 305 of 1072

RULING:
No.
The appellate court erred in its explanation that while there may not
have been a quorum during the board meeting of petitioner-corporation on
May 10, 1984 when a resolution authorizing Gaw to sue on its behalf was
allegedly passed, this did "not mean however, that New Durawood Co., Inc.
cannot be bound by Gaw's action because "no howl of protest, complaint or
denial came from (said corporation), and that said corporation in fact had
taken advantage of the benefits therefrom. Hence, petitioner is estopped
from questioning Gawls acts. The appellate Court was of the belief that
petitioner-corporation ratified Gaw's "authority" by acquiescence to his acts.
The respondent Court thus concluded that petitioner-corporation's "claim of
being a victim of extrinsic fraud is baseless."
It is clear that, there having been no quorum present during the
meeting in question, the board of directors could not have validly given Gaw
any express authority to file the petition. Upon the other hand, the doctrine
of "apparent authority" cannot apply as to Gaw because, being a mere
branch manager, he could not be looked upon as a corporate officer clothed
with the implied or "apparent" power to file suit for and in behalf of a
corporation. Neither will estoppel prevent the corporation from questioning
Gaw's acts. Precisely, these acts were hidden from the company and its top
officers. How then can estoppel attach?
By his surreptitious filing of the petition for reconstitution without
authority express or implied of his employer, Gaw enabled respondent
corporation to acquire the certificates of title in a manner contrary to law.
In petitions for issuance of new owner's duplicate copies of Torrens
titles, it is essential as provided under Sec. 109 of P.D. 1529 as amended
(supra) that the trial court take steps to assure itself that the petitioner is
the "registered owner or other person in interest". Otherwise, new owner's
duplicate certificates might be issued in favor of impostors who could
fraudulently dispose, hypothecate or otherwise deal in and with real estate in
mockery of the Torrens system of titling properties.

Page 306 of 1072

TOPIC:DOCTRINE OF APPARENT AUTHORITY


PEOPLE'S AIRCARGO AND WAREHOUSING CO. INC.,
vs.
COURT OF APPEALS and STEFANI SAO,
G.R. No. 117847. October 7, 1998
297 S 170
FACTS:
Petitioner is a domestic corporation, which was organized in the middle
of 1986 to operate a customs bonded warehouse at the old Manila
International Airport in Pasay City.
To obtain a license for the corporation from the Bureau of Customs, Antonio
Punsalan Jr., the corporation president, solicited a proposal from private
respondent for the preparation of a feasibility study. Private respondent
submitted a letter-proposal.
Initially, Cheng Yong, the majority stockholder of petitioner, objected to
private respondent's offer, as another company priced a similar proposal at
only P15,000. However, Punsalan preferred private respondent's service
because of the latter's membership in the task force, which was supervising
the transition of the Bureau of Customs from the Marcos government to the
Aquino administration.
On October 17, 1986, pertitioner, through Punsalan, sent private
respondent a letter, confirming their agreement.
Accordingly, private respondent prepared a feasibility study for
petitioner which eventually paid him the balance of the contract price,
although not according to the schedule agreed upon.
On December 4, 1986, upon Punsalan's request, private respondent
sent petitioner another letter-proposal.
On January 10, 1987, Andy Villaceren, vice president of petitioner,
received the operations manual prepared by private respondent. Petitioner
submitted said operations manual to the Bureau of Customs is connection
with the former's application to operate a bonded warehouse; thereafter, in
May 1987, the Bureau issued to it a license to operate, enabling it to become
one of the three public bonded warehouses at the international airport.
Private respondent also conducted, in the third week of January 1987 in the
warehouse of petitioner, a three-day training seminar for the latter's
employees.

Page 307 of 1072

On March 25, 1987, private respondent joined the Bureau of Customs


as special assistant to then Commissioner Alex Padilla, a position he held
until he became technical assitant to then Commissioner Miriam DefensorSantiago on March 7, 1988.
Meanwhile, Punsalan sold his shares in
petitioner-corporation and resigned as its president in 1987.
On February 9, 1988, private respondent filed a collection suit against
petitioner.
ISSUE/S:
Whether the subject letter-agreement for services was binding on
the corporation simply because it was entered into by its
president
RULING:
The general rule is that, in the absence of authority from the board of
directors, no person, not even its officers, can validly bind a corporation. A
corporation is a juridical person, separate and distinct from its stockholders
and members, "having . . . powers, attributes and properties expressly
authorized by law or incident to its existence."
Being a juridical entity, a corporation may board of directors, which
exercises almost all corporate powers, lays down all corporate business
policies and is responsible for the efficiency of management, as provided in
Section 23 of the Corporation Code of the Philippines.
Under this provision, the power and the responsibility to decide
whether the corporation should enter into a contract that will bind the
corporation is lodged in the board, subject to the articles of incorporaration,
bylaws, or relevant provisions of law. Howeever, just as a natural person
may authorize another to do certain acts for and on his behalf, the board of
directors may validly delegate some of its functions and powers to officers,
committees or agents. The authority of such individuals to bind the
corporation is generally derived from law, corporate bylaws or authorization
from the board, either expressly or impliedly by habit, custom or
acquiescence in the general course of business.
In the case at bar, petitioner, through its president Antonio Punsalan Jr.,
entered into the First Contract without first securing board approval. Despite
such lack of board approval, petitioner did not object to or repudiate said
contract, thus "clothing" its president with the power to bind the corporation.
The grant of apparent authority to Punsalan is evident in the testimony of
Yong senior vice president, treasurer and major stockholder of petitioner.
Hence, private respondent should not be faulted for believing that Punsalan's
conformity to the contract in dispute was also binding on petitioner. It is

Page 308 of 1072

familiar doctrine that if a corporation knowingly permits one of its officers, or


any other agent, to act within the scope of an apparent authority, it holds
him out to the public as possessing the power to do those acts; and thus, the
corporation will, as against anyone who has in good faith dealt with it
through such agent, be estopped from denying the agent's authority.

Page 309 of 1072

TOPIC: DOCTRINE OF APPARENT AUTHORITY


SAN JUAN STRUCTURAL AND STEEL FABRICATORS, INC.,
vs.
COURT OF APPEALS, MOTORICH SALES CORPORATION, NENITA LEE
GRUENBERG, ACL DEVELOPMENT CORP. and JNM REALTY AND
DEVELOPMENT CORP.,
G.R. No. 129459. September 29, 1998
FACTS:
Plaintiff-appellant San Juan Structural and Steel Fabricators, Inc.'s
amended complaint alleged that on 14 February 1989, plaintiff-appellant
entered into an agreement with defendant-appellee Motorich Sales
Corporation for the transfer to it of a parcel of land identified as Lot 30, Block
1 of the Acropolis Greens Subdivision containing an area of Four Hundred
Fourteen (414) square meters.
That as stipulated in the Agreement of 14 February 1989, plaintiffappellant paid the down payment in the sum of One Hundred Thousand
(P100, 000.00) Pesos, the balance to be paid on or before March 2, 1989;
that on March 1, 1989.
Mr. Andres T. Co, president of plaintiff-appellant Corporation, wrote a
letter to defendant-appellee Motorich Sales Corporation requesting for a
computation of the balance to be paid plaintiff-appellant was ready with the
amount corresponding to the balance, payable to defendant-appellee
Motorich Sales Corporation; that plaintiff-appellant and defendant-appellee
Motorich Sales Corporation were supposed to meet in the office of plaintiffappellant but defendant-appellee's treasurer, Nenita Lee Gruenberg, did not
appear; that defendant-appellee Motorich Sales Corporation despite
repeated demands and in utter disregard of its commitments had refused to
execute the Transfer of Rights/Deed of Assignment which is necessary to
transfer the certificate of title.
ISSUE/S:
Was there a valid contract of sale between petitioner and
Motorich?
RULING:
In the case at bar, Respondent Motorich categorically denies that it
ever authorized Nenita Gruenberg, its treasurer, to sell the subject parcel of
land. Consequently, petitioner had the burden of proving that Nenita
Gruenberg was in fact authorized to represent and bind Motorich in the
transaction. Petitioner failed to discharge this burden. Its offer of evidence
before the trial court contained no proof of such authority. It has not shown

Page 310 of 1072

any provision of said respondent's articles of incorporation, bylaws or board


resolution to prove that Nenita Gruenberg possessed such power.
That Nenita Gruenberg is the treasurer of Motorich does not free
petitioner from the responsibility of ascertaining the extent of her authority
to represent the corporation. Petitioner cannot assume that she, by virtue of
her position, was authorized to sell the property of the corporation. Selling is
obviously foreign to a corporate treasurer's function, which generally has
been described as "to receive and keep the funds of the corporation, and to
disburse them in accordance with the authority given him by the board or
the properly authorized officers."
Neither was such real estate sale shown to be a normal business
activity of Motorich. The primary purpose of Motorich is marketing,
distribution, export and import in relation to a general merchandising
business. Unmistakably, its treasurer is not cloaked with actual or apparent
authority to buy or sell real property, an activity which falls way beyond the
scope of her general authority.
As a general rule, the acts of corporate officers within the scope of
their authority are binding on the corporation. But when these officers
exceed their authority, their actions "cannot bind the corporation, unless it
has ratified such acts or is estopped from disclaiming them."
In this case, there is a clear absence of proof that Motorich ever
authorized Nenita Gruenberg, or made it appear to any third person that she
had the authority, to sell its land or to receive the earnest money. Neither
was there any proof that Motorich ratified, expressly or impliedly, the
contract. Petitioner rests its argument on the receipt which, however, does
not prove the fact of ratification. The document is a hand-written one, not a
corporate receipt, and it bears only Nenita Gruenberg's signature. Certainly,
this document alone does not prove that her acts were authorized or ratified
by Motorich.
Because Motorich had never given a written authorization to
Respondent Gruenberg to sell its parcel of land, we hold that the February
14, 1989 Agreement entered into by the latter with petitioner is void under
Article 1874 of the Civil Code. Being inexistent and void from the beginning,
said contract cannot be ratified.

Page 311 of 1072

TOPIC:DOCTRINE OF APPARENT AUTHORITY


RURAL BANK OF MILAOR (CAMARINES SUR)
vs.
FRANCISCA OCFEMIA, ROWENA BARROGO, MARIFE O. NIO,
FELICISIMO OCFEMIA, RENATO OCFEMIA JR, and WINSTON OCFEMIA
G.R. No. 137686.February 8, 2000
325 SCRA 99
FACTS:
On April 10, 1996, petitioner was declared in default on motion of the
respondents for failure to file an answer within the reglementary-period after
it was duly served with summons. On April 26, 1996, herein petitioner filed a
motion to set aside the order of default with objection thereto filed by herein
respondents.
On June 17, 1996, an order was issued denying petitioner's motion to
set aside the order of default. The defendant filed a motion for
reconsideration of the order of June 17, 1996 with objection thereto by
respondents. An order was issued denying petitioner's motion for
reconsideration. On July 31, 1996, respondents filed a motion to set case for
hearing. A copy thereof was duly furnished the petitioner but the latter did
not file any opposition and so respondents were allowed to present their
evidence ex-parte. A certiorari case was filed by the petitioner with the Court
of Appeals but the petition was denied in a decision rendered on March 31,
1997 and the same is now final.
The evidence presented by the respondents, shows that she is the
daughter of Francisca Ocfemia, a co-respondent in this case, and the late
Renato Ocfemia who died on July 23, 1994. The parents of her father, Renato
Ocfemia, were Juanita Arellano Ocfemia and Felicisimo Ocfemia. Her other
co-respondents Rowena O. Barrogo, Felicisimo Ocfemia, Renato Ocfemia, Jr.
and Winston Ocfemia are her brothers and sisters.
Marife O. Nio knows the five parcels of land described in paragraph 6 of the
petition and that they are the ones possessing them which were originally
owned by her grandparents, respondents mortgaged the said five parcels of
land and two others to the petitioner Rural Bank of Milaor as shown by the
Deed of Real Estate Mortgage.
The spouses Felicisimo Ocfemia and Juanita Arellano Ocfemia were not
able to redeem the mortgaged properties consisting of seven (7) parcels of
land and so the mortgage was foreclosed and thereafter ownership thereof
was transferred to the petitioner bank. Out of the seven (7) parcels that were

Page 312 of 1072

foreclosed, five (5) of them are in the possession of the respondents because
these five (5) parcels of land described in paragraph 6 of the petition were
sold by the petitioner bank to the parents of Marife O. Nio.
The aforementioned five (5) parcels of land subject of the deed of sale
have not been, however transferred in the name of the parents of Merife O.
Nio after they were sold to her parents by the petitioner bank because
according to the Assessor's Office the five (5) parcels of land, subject of the
sale, cannot be transferred in the name of the buyers as there is a need to
have the document of sale registered with the Register of Deeds of
Camarines Sur.
In view of the foregoing, Marife O. Nio went to the Register of Deeds of
Camarines Sur with the Deed of Sale in order to have the same registered.
The Register of Deeds, however, informed her that the document of sale
cannot be registered without a board resolution of the petitioner Bank.
The petitioner bank refused her request for a board resolution and
made many alibis.
ISSUE/S:
Whether the bank manager can enter into a contract of sale.
RULING:
In any event, the bank acknowledged, by its own acts or failure to act,
the authority of Fe S. Tena to enter into binding contracts. After the execution
of the Deed of Sale, respondents occupied the properties in dispute and paid
the real estate taxes due thereon. If the bank management believed that it
had title to the property, it should have taken some measures to prevent the
infringement or invasion of its title thereto and possession thereof.
Likewise, Tena had previously transacted business on behalf of the bank, and
the latter had acknowledged her authority. A bank is liable to innocent third
persons where representation is made in the course of its normal business by
an agent like Manager Tena, even though such agent is abusing her
authority.
Clearly, persons dealing with her could not be blamed for
believing that she was authorized to transact business for and on behalf of
the bank.
Notwithstanding the putative authority of the manager to bind the
bank in the Deed of Sale, petitioner has failed to file an answer to the
Petition below within the reglementary period, let alone present evidence
controverting such authority. Indeed, when one of herein respondents, Marife
S. Nino, went to the bank to ask for the board resolution, she was merely told
to bring the receipts. The bank failed to categorically declare that Tena had
no authority. This Court stresses the following:

Page 313 of 1072

. . . Corporate transactions would speedily come to a


standstill were every person dealing with a corporation
held duty-bound to disbelieve every act of its responsible
officers, no matter how regular they should appear on their
face.
In this light, the bank is estopped from questioning the authority of the
bank manager to enter into the contract of sale. If a corporation knowingly
permits one of its officers or any other agent to act within the scope of an
apparent authority, it holds the agent out to the public as possessing the
power to do those acts; thus, the corporation will, as against anyone who has
in good faith dealt with it through such agent, be estopped from denying the
agent's authority.
Unquestionably, petitioner has authorized Tena to enter into the Deed
of Sale. Accordingly, it has a clear legal duty to issue the board resolution
sought by respondent's. Having authorized her to sell the property, it
behooves the bank to confirm the Deed of Sale so that the buyers may enjoy
its full use.

Page 314 of 1072

TOPIC: PERSONAL LIABILITY OF DIRECTORS AND OTHER CORPORATE


OFFICERS
IMELDA O. COJUANGCO, PRIME HOLDINGS, INC., AND THE ESTATE OF
RAMON U. COJUANGCO
vs.
SANDIGANBAYAN, REPUBLIC OF THE PHILIPPINES, AND THE SHERIFF
OF SANDIGANBAYAN
G.R. No. 183278.April 24, 2009
FACTS:
On July 16, 1987, respondent Republic of the Philippines filed before
the Sandiganbayan a "Complaint for Reconveyance, Reversion, Accounting,
Restitution and Damages," praying for the recovery of alleged ill-gotten
wealth from the late President Marcos and former First Lady Imelda Marcos
and their cronies, including some 2.4 million shares of stock in the Philippine
Long Distance Telephone Company
The complaint, which was later amended to implead herein petitioners
Ramon and Imelda Cojuangco alleged that the Marcoses ill-gotten wealth
included shares in the registered in the name of Prime Holdings, Inc. (Prime
Holdings).
The Sandiganbayan dismissed the complaint with respect to the
recovery of the PLDT shares, hence, the Republic appealed to this Court.
The Decision became final and executory on October 26, 2006, hence,
the Republic filed on November 20, 2006 with the Sandiganbayan a Motion
for the Issuance of a Writ of Execution, praying for the cancellation of the
111,415 shares/certificates of stock registered in the name of Prime Holdings
and the annotation of the change of ownership on PTICs Stock and Transfer
Book.
ISSUE/S:
whether the Sandiganbayan gravely abused its discretion in
ordering the accounting, delivery, and remittance to the Republic
of the stock, cash, and property dividends pertaining to the
111,415 PTIC shares of Prime Holdings

Page 315 of 1072

RULING:
This Court, in directing the reconveyance to the Republic of the
111,415 shares of PLDT stock owned by PTIC in the name of Prime Holdings,
declared the Republic as the owner of said shares and, necessarily, the
dividends and interests accruing thereto.
Ownership is a relation in law by virtue of which a thing pertaining to
one person is completely subjected to his will in everything not prohibited by
law or the concurrence with the rights of another. Its traditional elements or
attributes include jus utendi or the right to receive from the thing what it
produces.
It would be absurd to award the shares to the Republic as their owner
and not include the dividends and interests accruing thereto. An owner who
cannot exercise the "juses" or attributes of ownership -- the right to possess,
to use and enjoy, to abuse or consume, to accessories, to dispose or
alienate, to recover or vindicate, and to the fruits - is a crippled owner.
Respecting petitioners argument that the Republic has yielded its right
to the fruits of the shares when it sold them to Metro Pacific Assets Holdings,
Inc., (Metro Pacific), the same does not lie.
Dividends are payable to the stockholders of record as of the date of
the declaration of dividends or holders of record on a certain future date, as
the case may be, unless the parties have agreed otherwise. And a transfer of
shares which is not recorded in the books of the corporation is valid only as
between the parties, hence, the transferor has the right to dividends as
against the corporation without notice of transfer but it serves as trustee of
the real owner of the dividends, subject to the contract between the
transferor and transferee as to who is entitled to receive the dividends.
It is thus clear that the Republic is entitled to the dividends accruing
from the subject 111,415 shares since 1986 when they were sequestered up
to the time they were transferred to Metro Pacific via the Sale and Purchase
Agreement of February 28, 2007; and that the Republic has since the latter
date been serving as trustee of those dividends for the Metro Pacific up to
the present, subject to the terms and conditions of the said agreement they
entered into.

Page 316 of 1072

TOPIC: PERSONAL LIABILITY OF DIRECTORS AND OTHER CORPORATE


OFFICERS
ARNEL U. TY, MARIE ANTONETTE TY, JASON ONG, WILLY DY, and
ALVIN TY
vs.
NBI SUPERVISING AGENT MARVIN E. DE JEMIL, PETRON GASUL
DEALERS ASSOCIATION, and TOTALGAZ DEALERS ASSOCIATION
G.R. No. 182147.December 15, 2010
FACTS:
Petitioners are stockholders of Omni Gas Corporation as per Omnis
General Information Sheet submitted to the Securities and Exchange
Commission. Omni is in the business of trading and refilling of Liquefied
Petroleum Gas cylinders and holds Pasig City Mayors Permit.
The case all started when Joaquin Guevara Adarlo & Caoile Law Offices
sent a letter to the NBI requesting, on behalf of their clients Shellane Dealers
Association, Inc., Petron Gasul Dealers Association, Inc., and Totalgaz Dealers
Association, Inc., for the surveillance, investigation, and apprehension of
persons or establishments in Pasig City that are engaged in alleged illegal
trading of petroleum products and underfilling of branded LPG cylinders in
violation of Batas Pambansa Blg. 33.
Agents De Jemil and Kawada attested to conducting surveillance of
Omni in the months of March and April 2004 and doing a test-buy on April
15, 2004. They brought eight branded LPG cylinders of Shellane, Petron
Gasul, Totalgaz, and Superkalan Gaz to Omni for refilling. The branded LPG
cylinders were refilled, for which the National Bureau of Investigation (NBI)
agents paid PhP 1,582 issued by Omni on April 15, 2004. The refilled LPG
cylinders were without LPG valve seals and one of the cylinders was actually
underfilled, as found by LPG Inspector Noel N. Navio of the Liquefied
Petroleum Gas Industry Association who inspected the eight branded LPG
cylinders on April 23, 2004 which were properly marked by the NBI after the
test-buy.
On the same day of the filing of the application for search warrants on
April 28, 2004, the RTC, Branch 167 in Pasig City issued Search Warrants. The

Page 317 of 1072

NBI served the warrants the next day or on April 29, 2004 resulting in the
seizure of several items from Omnis premises duly itemized in the NBIs
Receipt/Inventory of Property/Item Seized.
ISSUE/S:
Whether or not petitioners can be held liable under BP 33 for
being mere directors, not actually in charge of the management
of the Business Affairs of the Corporation
RULING:
When the offender is a corporation, partnership, or other juridical
person, the president, the general manager, managing partner, or such
other officer charged with the management of the business affairs thereof, or
employee responsible for the violation shall be criminally liable; in case the
offender is an alien, he shall be subject to deportation after serving the
sentence.
If the offender is a government official or employee, he shall be
perpetually disqualified from office.
On this point, we agree with petitioners except as to petitioner Arnel U.
Ty who is indisputably the President of Omni.
It may be noted that Sec. 4 above enumerates the persons who may
be held liable for violations of the law, viz:
1. the president,
2. general manager,
3. managing partner,
4. such other officer charged with the management of the
business affairs of the corporation or juridical entity, or
5. the employee responsible for such violation.
A common thread of the first four enumerated officers is the fact that
they manage the business affairs of the corporation or juridical entity. In
short, they are operating officers of a business concern, while the last in the
list is self-explanatory.
It is undisputed that petitioners are members of the board of directors
of Omni at the time pertinent. There can be no quibble that the enumeration
of persons who may be held liable for corporate violators of BP 33, as
amended, excludes the members of the board of directors. This stands to
reason for the board of directors of a corporation is generally a policy making
body. Even if the corporate powers of a corporation are reposed in the board
of directors under the first paragraph of Sec. 23of the Corporation Code, it is
of common knowledge and practice that the board of directors is not directly
engaged or charged with the running of the recurring business affairs of the

Page 318 of 1072

corporation. Depending on the powers granted to them by the Articles of


Incorporation, the members of the board generally do not concern
themselves with the day-to-day affairs of the corporation, except those
corporate officers who are charged with running the business of the
corporation and are concomitantly members of the board, like the President.
Section 25 of the Corporation Code requires the president of a corporation to
be also a member of the board of directors.
Thus, the application of the legal maxim expressio unius est exclusio
alterius, which means the mention of one thing implies the exclusion of
another thing not mentioned. If a statute enumerates the thing upon which it
is to operate, everything else must necessarily and by implication be
excluded from its operation and effect. The fourth officer in the enumerated
list is the catch-all "such other officer charged with the management of the
business affairs" of the corporation or juridical entity which is a factual issue
which must be alleged and supported by evidence.

Page 319 of 1072

TOPIC: PERSONAL LIABLITY OF DIRECTORS AND OTHER CORPORATE


OFFICERS
QUEENSLAND-TOKYO COMMODITIES, INC., ROMEO Y. LAU, and
CHARLIE COLLADO, Petitioners,
vs.
THOMAS GEORGE, Respondent.
G.R. No. 172727. September 8, 2010
FACTS:
QTCI is a duly licensed broker engaged in the trading of commodity
futures. In 1995, Guillermo Mendoza, Jr. and OnilerLontoc of QTCI met with
respondent Thomas George, encouraging the latter to invest with QTCI. On
July 7, 1995, upon Mendozas prodding, respondent finally invested with
QTCI. On the same day, Collado, in behalf of QTCI, and respondent signed the
Customers Agreement. Forming part of the agreement was the Special
Power of Attorneyexecuted by respondent, appointing Mendoza as his
attorney-in-fact with full authority to trade and manage his account.
On June 20, 1996, the Securities and Exchange Commission (SEC)
issued a Cease-and-Desist Order (CDO) against QTCI. Alarmed by the
issuance of the CDO, respondent demanded from QTCI the return of his
investment, but it was not heeded. He then sought legal assistance, and
discovered that Mendoza and Lontoc were not licensed commodity futures
salesmen.
On February 4, 1998, respondent filed a complaint for Recovery of
Investment with Damageswith the SEC against QTCI, Lau, and Collado,
petitioners, and against the unlicensed salesmen, Mendoza and Lontoc. The
case was docketed as SEC Case No. 02-98-5886, and was raffled to SEC
Hearing Officer Julieto F. Fabrero.
Only petitioners answered the complaint, as Mendoza and Lontoc had
since vanished into thin air. Traversing the complaint, petitioners denied the
material allegations in the complaint and alleged lack of cause of action, as a
defense. Petitioners averred that QTCI only assigned duly qualified persons to
handle the accounts of its clients; and denied allowing unlicensed brokers or
agents to handle respondents account. They claimed that they were not

Page 320 of 1072

aware of, nor were they privy to, any arrangement which resulted in the
account of respondent being handled by unlicensed brokers. They added that
even assuming that the subject account was handled by an unlicensed
broker, respondent is now estopped from raising it as a ground for the return
of his investment. They pointed out that respondent transacted business
with QTCI for almost a year, without questioning the license or the authority
of the traders handling his account. It was only after it became apparent that
QTCI could no longer resume its business transactions by reason of the CDO
that respondent raised the alleged lack of authority of the brokers or traders
handling his account. The losses suffered by respondent were due to
circumstances beyond petitioners control and could not be attributed to
them. Respondents remedy, they added, should be against the unlicensed
brokers who handled the account. Thus, petitioners prayed for the dismissal
of the complaint.
ISSUE/S:
Whether or not individual petitioners are solidarily liable for the
damages and awards due the respondent.
RULING:
The Supreme Court is not persuaded by petitioners assertion that they
had no hand in Mendozas designation as respondents attorney-in-fact. As
pointed out by the CA, the Special Power of Attorney formed part of
respondents agreement with QTCI, and under the Customers
Agreement,only a licensed or registered dealer or investment consultant may
be appointed as attorney-in-fact.
The evidence on record established that petitioners indeed permitted
an unlicensed trader and salesman, like Mendoza, to handle respondents
account. On the other hand, the record is bereft of proof that respondent had
knowledge that the person handling his account was not a licensed trader.
Respondent can, therefore, recover the amount he had given under the
contract. The SEC Hearing Officer and the CA, therefore, committed no
reversible error in holding that respondent is entitled to a full recovery of his
investments. Doctrine dictates that a corporation is invested by law with a
personality separate and distinct from those of the persons composing it,
such that, save for certain exceptions, corporate officers who entered into
contracts in behalf of the corporation cannot be held personally liable for the
liabilities of the latter. Personal liability of a corporate director, trustee, or
officer, along (although not necessarily) with the corporation, may validly
attach, as a rule, only when
1. he assents to a patently unlawful act of the corporation,
or when he is guilty of bad faith or gross negligence in
directing its affairs, or when there is a conflict of interest
resulting in damages to the corporation, its
stockholders, or other persons;

Page 321 of 1072

2. he consents to the issuance of watered down stocks or


who, having knowledge thereof, does not forthwith file
with the corporate secretary his written objection
thereto;
3. he agrees to hold himself personally and solidarily liable
with the corporation; or (4) he is made by a specific
provision of law personally answerable for his corporate
action.
Petitioner Romeo Lau, as president of petitioner QTCI, cannot feign
innocence on the existence of these unlawful activities within the company,
especially so that Collado, himself a ranking officer of QTCI, is involved in the
unlawful execution of customer orders. Petitioner Lau, being the chief
operating officer, cannot escape the fact that had he exercised a modicum of
care and discretion in supervising the operations of QTCI, he could have
detected and prevented the unlawful acts of [petitioner] Collado and
Mendoza.
It is therefore safe to conclude that although Lau may not have
participated nor been aware of the unlawful acts, he is however deemed to
have been grossly negligence in directing the affairs of QTCI.
In all, it having been established by substantial evidence that
petitioner Collado assented to the unlawful act of QTCI, and that petitioner
Lau is grossly negligent in directing the affairs of QTCI, and pursuant to
Section 31 of the Corporation Code, they are therefore, jointly and severally
liable with QTCI for all the damages and awards due to the respondent.
We find no compelling reason to depart from the conclusion of the SEC
Hearing Officer, which was affirmed by the CA. We are in full accord with his
reasons for holding Lau and Collado jointly and severally liable with QTCI for
the payment of respondents claim.

Page 322 of 1072

TOPIC: PERSONAL LIABLITY OF DIRECTORS AND OTHER CORPORATE


OFFICERS
WENSHA SPA CENTER, INC. and/or XU ZHI JIE, Petitioners,
vs.
LORETA T. YUNG, Respondent.
G.R. No. 185122.August 16, 2010
FACTS:
Wensha Spa Center, Inc. (Wensha) in Quezon City is in the business of
sauna bath and massage services. Xu Zhi Jie a.k.a. Pobby Co is its
president,respondent Loreta T. Yung was its administrative manager at the
time of her termination from employment. Loreta stated that she used to be
employed by Manmen Services Co., Ltd. where Xu was a client. Xu was
apparently impressed by Loretas performance. After he established Wensha,
he convinced Loreta to transfer and work at Wensha.
Loreta recounted that on August 10, 2004, she was asked to leave her
office because Xu and a Feng Shui master were exploring the premises. Later
that day, Xu asked Loreta to go on leave with pay for one month. She did so
and returned on September 10, 2004. Upon her return, Xu and his wife asked
her to resign from Wensha because, according to the Feng Shui master, her
aura did not match that of Xu. Loreta refused but was informed that she
could no longer continue working at Wensha. That same afternoon, Loreta
went to the NLRC and filed a case for illegal dismissal against Xu and
Wensha.
Wensha and Xu denied illegally terminating Loretas employment. They
claimed that two months after Loreta was hired, they received various
complaints against her from the employees so that on August 10, 2004, they
advised her to take a leave of absence for one month while they conducted
an investigation on the matter. Based on the results of the investigation, they
terminated Loretas employment on August 31, 2004 for loss of trust and
confidence.
ISSUE/S:

Page 323 of 1072

Whether or not the Court of Appelas committed grave abuse of


discretion and serious errors when it held that petitioner XU ZHI
JIE to be solidarily liable with WENSHA, assuming that respondent
was illegally dismissed.
RULING:
The Court finds merit in the argument of petitioner Xu that the CA
erred in ruling that he is solidarily liable with Wensha.
Elementary is the rule that a corporation is invested by law with a
personality separate and distinct from those of the persons composing it and
from that of any other legal entity to which it may be related. Mere
ownership by a single stockholder or by another corporation of all or nearly
all of the capital stock of a corporation is not of itself sufficient ground for
disregarding the separate corporate personality.
In the subject decision, the CA concluded that petitioner Xu and
Wensha are jointly and severally liable to Loreta. We have read the decision
in its entirety but simply failed to come across any finding of bad faith or
malice on the part of Xu. There is, therefore, no justification for such a ruling.
To sustain such a finding, there should be an evidence on record that an
officer or director acted maliciously or in bad faith in terminating the services
of an employee. Moreover, the finding or indication that the dismissal was
effected with malice or bad faith should be stated in the decision itself.

Page 324 of 1072

TOPIC: PERSONAL LIABLITY OF DIRECTORS AND OTHER CORPORATE


OFFICERS
CEBU MACTAN MEMBERS CENTER, INC., Petitioner,
vs.
MASAHIRO TSUKAHARA, Respondent.
G.R. No. 159624.July 17, 2009
FACTS:
In February 1994, petitioner Cebu Mactan Members Center, Inc.
(CMMCI), through Mitsumasa Sugimoto (Sugimoto), the President and
Chairman of the Board of Directors of CMMCI, obtained a loan amounting to
P6,500,000 from respondent Masahiro Tsukahara. As payment for the loan,
CMMCI issued seven postdated checks of CMMCI payable to Tsukahara. On
13 April 1994, CMMCI, through Sugimoto, obtained another loan amounting
to P10,000,000 from Tsukahara. Sugimoto executed and signed a promissory
note in his capacity as CMMCI President and Chairman, as well as in his
personal capacity.
Tsukahara alleged that the amount of P16,500,000 was used by CMMCI
for the improvement of its beach resort, which included the construction of a
wave fence, the purchase of airconditioners and curtains, and the provision
of salaries of resort employees. He also asserted that Sugimoto, as the
President of CMMCI, "has the power to borrow money for said corporation by
any legal means whatsoever and to sign, endorse and deliver all checks and
promissory notes on behalf of the corporation.
CMMCI, on the other hand, denied borrowing the amount from
Tsukahara, and claimed that both loans were personal loans of Sugimoto. The
company also contended that if the loans were those of CMMCI, the same
should have been supported by resolutions issued by CMMCIs Board of
Directors.
ISSUE/S:

Page 325 of 1072

Whether the Court of Appeals erred in holding that CMMCI is


liable for the loan contracted by its President without a resolution
issued by the CMMCI Board of Directors.
RULING:
A corporate officer or agent may represent and bind the corporation in
transactions with third persons to the extent that the authority to do so has
been conferred upon him, and this includes powers which have been
intentionally conferred, and also such powers as, in the usual course of the
particular business, are incidental to, or may be implied from, the powers
intentionally conferred, powers added by custom and usage, as usually
pertaining to the particular officer or agent, and such apparent powers as the
corporation has caused persons dealing with the officer or agent to believe
that it has conferred.
It is clear from the foregoing that the president of CMMCI is given the
power to borrow money, execute contracts, and sign and indorse checks and
promissory notes, in the name and on behalf of CMMCI. With such powers
expressly conferred under the corporate by-laws, the CMMCI president, in
exercising such powers, need not secure a resolution from the companys
board of directors. The corporation is now estopped from denying the
authority of its president to bind the former into contractual relations.
Thus, given the presidents express powers under the CMMCIs by-laws,
Sugimoto, as the president of CMMCI, was more than equipped to enter into
loan transactions on CMMCIs behalf. Accordingly, the loans obtained by
Sugimoto from Tsukahara on behalf of CMMCI are valid and binding against
the latter, and CMMCI may be held liable to pay such loans.

Page 326 of 1072

TOPIC: PERSONAL LIABLITY OF DIRECTORS AND OTHER CORPORATE


OFFICERS
ARMANDO DAVID, Petitioner,
vs.
NATIONAL FEDERATION OF LABOR UNION and MARIVELES APPAREL
CORPORATION, Respondents.
G.R. Nos. 148263 and 148271-72.April 21, 2009
FACTS:
MAC hired David as IMPEX and Treasury Manager on 16 September
1988. David began serving as MACs President in May 1990. David served as
President in the nature of a nominee as he did not own any of MACs shares.
David tendered his irrevocable resignation from MAC on 30 September 1993.
Davids resignation was made effective on 15 October 1993.
In a complaint for illegal dismissal dated 12 August 1993, National
Federation of Labor Unions (NAFLU) and Mariveles Apparel Corporation Labor
Union (MACLU) alleged that MAC ceased operations on 8 July 1993 without
prior notice to its employees. MAC allegedly gave notice of its closure on the
same day that it ceased operations. MACLU and NAFLU further alleged that,
at the time of MACs closure, employees who had rendered one to two weeks
work were not paid their corresponding salaries.
ISSUE/S:
Whether or not in finding petitioner guilty of illegal closure and
making him personally liable for payment of private respondents
claims.
RULING:
Assuming arguendo that the NLRC and the Labor Arbiter had
jurisdiction over David, we rule that it was still improper to hold David liable
for MACs obligations to its employees.
However, Article 212(e) of the Labor Code, by itself, does not make a
corporate officer personally liable for the debts of the corporation because
Section 31 of the Corporation Code is still the governing law on personal

Page 327 of 1072

liability of officers for the debts of the corporation. Section 31 of the


Corporation Code provides:
Liability of directors, trustees or officers. Directors or trustees who
willfully and knowingly vote for or assent to patently unlawful acts of the
corporation or who are guilty of gross negligence or bad faith in directing the
affairs of the corporation or acquire any personal or pecuniary interest in
conflict with their duty as such directors, or trustees shall be liable jointly
and severally for all damages resulting therefrom suffered by the
corporation, its stockholders or members and other persons.
There was no showing of David willingly and knowingly voting for or
assenting to patently unlawful acts of the corporation, or that David was
guilty of gross negligence or bad faith.

Page 328 of 1072

TOPIC: PERSONAL LIABLITY OF DIRECTORS AND OTHER CORPORATE


OFFICERS
HILARIO P. SORIANO and ROSALINDA ILAGAN, Petitioners,
vs.
PEOPLE OF THE PHILIPPINES, BANGKO SENTRAL NG PILIPINAS (BSP),
and PHILIPPINE DEPOSIT INSURANCE CORPORATION (PDIC),
Respondents.
G.R. No. 159517-18.June 30, 2009
FACTS:
Hilario P. Soriano and Rosalinda Ilagan were the President and General
Manager, respectively, of the Rural Bank of San Miguel (Bulacan), Inc.
(RBSM). Allegedly, on June 27, 1997 and August 21, 1997, during their
incumbency as president and manager of the bank, petitioners indirectly
obtained loans from RBSM. They falsified the loan applications and other
bank records, and made it appear that Virgilio J. Malang and Rogelio Maaol
obtained loans of P15,000,000.00 each, when in fact they did not.
Accordingly, on May 4, 2000, State Prosecutor Josefino A. Subia
charged Soriano in the Regional Trial Court (RTC) of Malolos, Bulacan, with
violation of Section 83 of Republic Act No. 337 (R.A. No. 337) or the General
Banking Act, as amended by Presidential Decree No. 1795, or Violation of the
Director, Officer, Stockholder or Related Interest (DOSRI) Rules (DOSRI
Rules). On the same date, an information for estafa thru falsification of
commercial document was also filed against Soriano and Ilagan.
ISSUE/S:
Whether or not petitioners can be sued at the same time with
violation of DOSRI rules and estafa.
RULING:
The Court had ruled that a single act or incident might offend against
two or more entirely distinct and unrelated provisions of law thus justifying
the prosecution of the accused for more than one offense. The only limit to

Page 329 of 1072

this rule is the Constitutional prohibition that no person shall be twice put in
jeopardy of punishment for "the same offense."
Consequently, the filing of the multiple charges against petitioners,
although based on the same incident, is consistent with settled doctrine.
As aptly pointed out by the BSP in its memorandum, there are
differences between the two (2) offenses. A DOSRI violation consists in the
failure to observe and comply with procedural, reportorial or ceiling
requirements prescribed by law in the grant of a loan to a director, officer,
stockholder and other related interests in the bank, i.e. lack of written
approval of the majority of the directors of the bank and failure to enter such
approval into corporate records and to transmit a copy thereof to the BSP
supervising department. The elements of abuse of confidence, deceit, fraud
or false pretenses, and damage, which are essential to the prosecution for
estafa, are not elements of a DOSRI violation. The filing of several charges
against Soriano was, therefore, proper.

Page 330 of 1072

TOPIC: PERSONAL LIABILITY OF DIRECTORS AND OTHER


CORPORATE OFFICERS
CEBU COUNTRY CLUB, INC., SABINO R. DAPAT, RUBEN D.
ALMENDRAS, JULIUS Z. NERI, DOUGLAS L. LUYM, CESAR T. LIBI,
RAMONTITOE. GARCIA and JOSE B. SALA, petitioners,
vs.
RICARDO F. ELIZAGAQUE, respondent.
G.R. No. 160273. January 18, 2008
FACTS:
Cebu Country Club, Inc.,( CCCI) is a domestic corporation operating as
a non-profit and non-stock private membership club its principal place of
business in Banilad, Cebu. Petitioners herein are the members of the Board
of Directors.
Respondent Elizagaque as the Senior Vice-president and Operations
Manager for the Visayas and Mindanao of San Miguel Corp., a special
company proprietary of CCCI, was designated as a special non-proprietary
member of CCCI as approved by the CCCIs board.
Later, respondent, thru the indorsenment of 2 proprietary members of
CCCI and upon purchasing the share of a certain Dr. Bulatid in the amount of
P 3.5 Million, filed an application for proprietary membership with CCCI.
However, said application was deferred and later on disapproved by the
Board of Directors of CCCI. Thus, respondent wrote 2 letters of
reconsideration, but CCCI never answered.
Subsequently, respondent filed a complaint for damages against
petitioners wherein the RTC acted in his favor by ordering herein petitioners
jointly and severally liable.
The CA affirmed and modified the decision of the RTC.
ISSUE/S:
Whether In Disapproving Respondents Application For
Proprietary Membership With CCCI, Petitioners Are Liable To
Respondent For Damages, And
If So, Whether Their Liability Is Joint And Several.

Page 331 of 1072

RULING:
In rejecting respondents application for proprietary membership, the
Supreme Court finds that petitioners violated the rules governing human
relations, the basic principles to be observed for the rightful relationship
between human beings and for the stability of social order. The trial court
and the Court of Appeals aptly held that petitioners committed fraud and
evident bad faith in disapproving respondents applications. This is contrary
to morals, good custom or public policy. Hence, petitioners are liable for
damages pursuant to Article 19 in relation to Article 21 of the same Code.
It bears stressing that the amendment to Section 3(c) of CCCIs
Amended By-Laws requiring the unanimous vote of the directors present at a
special or regular meeting was not printed on the application form
respondent filled and submitted to CCCI. What was printed thereon was the
original provision of Section 3(c) which was silent on the required number of
votes needed for admission of an applicant as a proprietary member.
Petitioners explained that the amendment was not printed on the
application form due to economic reasons. We find this excuse flimsy and
unconvincing. Such amendment, aside from being extremely significant, was
introduced way back in 1978 or almost twenty (20) years before respondent
filed his application. We cannot fathom why such a prestigious and exclusive
golf country club, like the CCCI, whose members are all affluent, did not have
enough money to cause the printing of an updated application form.
It is thus clear that respondent was left groping in the dark wondering
why his application was disapproved. He was not even informed that a
unanimous vote of the Board members was required. When he sent a letter
for reconsideration and an inquiry whether there was an objection to his
application, petitioners apparently ignored him. Certainly, respondent did not
deserve this kind of treatment. Having been designated by San Miguel
Corporation as a special non-proprietary member of CCCI, he should have
been treated by petitioners with courtesy and civility. At the very least, they
should have informed him why his application was disapproved.
The exercise of a right, though legal by itself, must nonetheless be in
accordance with the proper norm. When the right is exercised arbitrarily,
unjustly or excessively and results in damage to another, a legal wrong is
committed for which the wrongdoer must be held responsible. It bears
reiterating that the trial court and the Court of Appeals held that petitioners
disapproval of respondents application is characterized by bad faith.
Thus, petitioners argument that they could not be held jointly and
severally liable for damages because only one (1) voted for the disapproval
of respondents application lacks merit.

Page 332 of 1072

Section 31 of the Corporation Code provides:


SEC. 31. Liability of directors, trustees or officers.
Directors or trustees who willfully and knowingly vote for or
assent to patently unlawful acts of the corporation or who
are guilty of gross negligence or bad faith in directing the
affairs of the corporation or acquire any personal or
pecuniary interest in conflict with their duty as such
directors, or trustees shall be liable jointly and severally
for all damages resulting therefrom suffered by the
corporation, its stockholders or members and other
persons.

Page 333 of 1072

TOPIC: PERSONAL LIABILITY OF DIRECTORS AND OTHER


CORPORATE OFFICERS
CALTEX (PHILS.), INC. (now CHEVRON PHILIPPINES, INC.), Petitioner,
vs.
NATIONAL LABOR RELATIONS
COMMISSION AND ROMEO T. STO. TOMAS, Respondents.
G.R. No. 159641. October 15, 2007
FACTS:
Private respondent Romeo T. Sto Tomas was a regular employee of
petitioner since February 2, 1984. He was a Senior Accounting Analyst
receiving a monthly salary of P29, 860.00 at the time of his termination on
July 31, 1997.
In a letter dated October 21, 1996, petitioner informed the Department
of Labor and Employment (DOLE) of its plan to implement a redundancy
program in its Marketing Division and some departments in its Batangas
Refinery for the period starting October 1996 to December 1998. The letter
alleged that the redundancy program is a response to the market situation
which constrained petitioner to rationalize and simplify its business
processes; that petitioner undertook a review, restructuring and streamlining
of its organization which resulted in consolidation, abolition and outsourcing
of certain functions and in the identification of certain redundant positions.
The letter also states that petitioner will provide the DOLE a list of affected
employees as it implements each phase of the redundancy program.
Petitioner, through a letter dated June 30, 1997, notified private
respondent of his termination effective July 31, 1997 due to the redundancy
of his position and awarded him a separation package in the amount of
P559,458.90 consisting of the following:
Regular separation/retirement benefits P352,721.25
under the New Retirement Plan; and
Ex-gratia payment computed at months
Basic pay for every year of service 206,737.65
TOTAL: Php 559, 458. 905
On June 8, 1998, respondent filed with the Labor Arbiter a complaint
for illegal dismissal against petitioner and its President and Chief Executive
Officer, Mr. Clifton Hon.
ISSUE/S:
Whether private respondents termination on the ground of
redundancy was valid.
RULING:

Page 334 of 1072

NO.
In the instant case, we find no reversible error committed by the CA in
upholding the findings of the NLRC that there was no substantial evidence
presented by petitioner to justify private respondent's dismissal due to
redundancy. As correctly found by the CA, petitioners evidence to show
redundancy merely consisted of a copy of petitioners letter to the DOLE
informing the latter of its intention to implement a redundancy program and
nothing more. The letter which merely stated that petitioner undertook a
review, restructuring and streamlining of its organization which resulted in
consolidation, abolition and outsourcing of certain functions; and which
resulted in identified and redundant positions instead of simplifying its
business process restructuring, does not satisfy the requirement of
substantial evidence, that is, the amount of evidence which a reasonable
mind might accept as adequate to justify a conclusion.
Petitioner failed to demonstrate the superfluity of private respondents
position as there was nothing in the records that would establish any
concrete and real factors recognized by law and relevant jurisprudence, such
as overhiring of workers, decreased volume of business, or dropping of a
particular product line or service activity previously manufactured or
undertaken by the enterprise, which were adopted by petitioner in
implementing the redundancy program.
Petitioner also failed to show any fair and reasonable criteria in
ascertaining what positions are redundant and how the selection of
employees to be dismissed was made.

Page 335 of 1072

TOPIC: PERSONAL LIABILITY OF DIRECTORS AND OTHER


CORPORATE OFFICERS
ATRIUM MANAGEMENT CORPORATION, petitioner,
vs.
COURT OF APPEALS, E.T. HENRY AND CO., LOURDES VICTORIA M. DE
LEON, RAFAEL DE LEON, JR., AND HI-CEMENT CORPORATION,
respondents.
G.R. No. 109491. February 28, 2001
FACTS:
On January 3, 1983, Atrium Management Corporation filed with the
Regional Trial Court, Manila an action for collection of the proceeds of four
postdated checks in the total amount of P2 million. Hi-Cement Corporation
through its corporate signatories, petitioner Lourdes M. de Leon, treasurer,
and the late Antonio de las Alas, Chairman, issued checks in favor of E.T.
Henry and Co. Inc., as payee. E.T. Henry and Co., Inc., in turn, endorsed the
four checks to petitioner Atrium Management Corporation for valuable
consideration. Upon presentment for payment, the drawee bank dishonored
all four checks for the common reason "payment stopped". Atrium, thus,
instituted this action after its demand for payment of the value of the checks
was denied.
After due proceedings, on July 20, 1989, the trial court rendered a
decision ordering Lourdes M. de Leon, her husband Rafael de Leon, E.T.
Henry and Co., Inc. and Hi-Cement Corporation to pay petitioner Atrium,
jointly and severally, the amount of P2 million corresponding to the value of
the four checks, plus interest and attorney's fees.
On appeal to the Court of Appeals, on March 17, 1993, the Court of
Appeals promulgated its decision modifying the decision of the trial court,
absolving Hi-Cement Corporation from liability and dismissing the complaint
as against it. The appellate court ruled that:
1. Lourdes M. de Leon was not authorized to issue the subject
checks in favor of E.T. Henry, Inc.;
2. The issuance of the subject checks by Lourdes M. de Leon
and the late Antonio de las Alas constituted ultra vires
acts; and
3. The subject checks were not issued for valuable
consideration.
ISSUE/S:

Page 336 of 1072

Whether Lourdes M. de Leon and Antonio de las Alas were


personally liable for the checks issued as corporate officers and
authorized signatories of the check.
RULING:
Personal liability of a corporate director, trustee or officer along
(although not necessarily) with the corporation may so validly attach, as a
rule, only when:
1. He assents
a. to a patently unlawful act of the corporation, or
b. for bad faith or gross negligence in directing its
affairs, or
c. for conflict of interest, resulting in damages to
the corporation, its stockholders or other
persons;
2. He consents to the issuance of watered down stocks or who,
having
knowledge thereof, does not forthwith file with the
corporate secretary
his written objection thereto;
3. He agrees to hold himself personally and solidarily liable with
the corporation;
or
4. He is made, by a specific provision of law, to personally
answer for his
corporate action."
In the case at bar, Lourdes M. de Leon and Antonio de las Alas as
treasurer and Chairman of Hi-Cement were authorized to issue the checks.
However, Ms. de Leon was negligent when she signed the confirmation letter
requested by Mr. Yap of Atrium and Mr. Henry of E.T. Henry for the
rediscounting of the crossed checks issued in favor of E.T. Henry. She was
aware that the checks were strictly endorsed for deposit only to the payee's
account and not to be further negotiated. What is more, the confirmation
letter contained a clause that was not true, that is, "that the checks issued to
E.T. Henry were in payment of Hydro oil bought by Hi-Cement from E.T.
Henry". Her negligence resulted in damage to the corporation.
Hence, Ms. de Leon may be held personally liable therefor.

Page 337 of 1072

TOPIC: PERSONAL LIABILITY OF DIRECTORS AND OTHER


CORPORATE OFFICERS
ARB CONSTRUCTION CO., INC., and MARK MOLINA, petitioners,
vs.
COURT OF APPEALS, TBS SECURITY AND INVESTIGATION AGENCY
represented by CECILIA R. BACLAY, respondents.
G.R. No. 126554. May 31, 2000
FACTS:
On 15 August 1993 TBS Security and Investigation Agency (TBSS)
entered into two (2) Service Contracts with Petitioner Corporation wherein
TBSS agreed to provide and post security guards in the five (5)
establishments being maintained by ARBC. Clause 10 of the Service
Contracts provides
10. This contract shall be effective for a period of one (1)
year commencing from 15th August 1993 and shall be
considered automatically renewed for the same period
unless otherwise a written notice of termination shall have
been given by one party to the other party thirty (30) days
in advance.
On February 23, 1994 ARBC informed TBSS of its desire to terminate
the Service Contracts effective thirty (30) days after receipt of the letter.
Also, in a letter dated 22 March 1994, ARBC through its Vice President for
Operations, Mark Molina, informed TBSS that it was replacing its security
guards with those of Global Security Investigation Agency (GSIA).
In response to both letters, TBSS informed ARBC that the latter could
not preterminate the Service Contracts nor could it post security guards from
GSIA as it would run counter to the provisions of their Service Contracts.
On 23 March 1994 Molina wrote TBSS conceding that indeed the
"security contract dated 15 August 1993 stipulates that the duration of the
service shall be for a period of one year, ending on 15 August 1994 . . . and
could not be preterminated until then." 1 Nevertheless, Molina decreased the
security guards to only one (1) allegedly pursuant to Clause 2 of the Service
Contracts which provides
2. The AGENCY shall adopt a guarding system and post
guards in accordance thereof, in the premises of the client

Page 338 of 1072

throughout the whole 24 hours daily, using variable shifts


of the guards at such hours as may be designated by the
CLIENT or AGENCY. As required by the CLIENT, the security
guards to be assigned by the AGENCY shall consist initially
of the following . . . subject to be increased or decreased
by the CLIENT at its sole discretion depending on the
security situation or the exigency of the service, by giving
the AGENCY at least SEVEN (7) days prior notice.
Thus, TBSS filed a complaint agains petitioners.
ISSUE/S:
Whether or not petitioner Molina is liable to private respondent in
his personal capacity.
RULING:
It is basic that a corporation is invested by law with a personality
separate and distinct from those of the persons composing it as well as from
that of any other legal entity to which it may be related. As a general rule, a
corporation may not be made to answer for acts or liabilities of its
stockholders or those of the legal entities to which it may be connected and
vice versa. However, the veil of corporate fiction may be pierced when it is
used as a shield to further an end subversive of justice; or for purposes that
could not have been intended by the law that created it; or to defeat public
convenience, justify wrong, protect fraud, or defend crime; or to perpetuate
deception; or as an alter ego, adjunct or business conduit for the sole benefit
of the stockholders.
Prescinding from the foregoing, the general rule is that officers of a
corporation are not personally liable for their official acts unless it is shown
that they have exceeded their authority.
Article 31 of the Corporation Code is in point
Sec. 31. Liability of directors, trustees or officers.
Directors or trustees who willfully and knowingly vote for or
assent to patently unlawful acts of the corporation or who
are guilty of gross negligence or bad faith in directing the
affairs of the corporation or acquire any personal or
pecuniary interest conflict with their duty as such directors,
or trustees shall be liable jointly and severally for all
damages resulting therefrom suffered by the corporation,
its stockholders or members and other persons . . . .
On the basis hereof, petitioner Molina could not be held jointly and
severally liable for any obligation which petitioner ARBC may be held
accountable for, absent any proof of bad faith or malice on his part.
Corollarily, it is also incorrect on the part of the Court of Appeals to conclude
that there was a sufficient cause of action against Molina as to make him
personally liable for his actuations as Vice President for Operations of ARBC.

Page 339 of 1072

A cursory reading of the records of the instant case would reveal that Molina
did not summarily withhold certain amounts from the payrollof TBSS.
Instead, he enumerated instances which in his view were enough bases to do
so.

TOPIC: PERSONAL LIABILITY OF DIRECTORS & OTHER CORPORATE OFFICERS


RUFINA LUY LIM, petitioner,
vs.
COURT OF APPEALS, AUTO TRUCK TBA CORPORATION, et
al respondents.
G.R. No. 124715, JANUARY 24, 2000
323 SCRA 102
FACTS:
Petitioner Rufina Luy Lim is the surviving spouse of late Pastor Y. Lim
whose estate is the subject of probate proceedings while private respondents
Auto Truck Corporation, Alliance Marketing Corporation, Speed Distributing,
Inc., Active Distributing, Inc. and Action Company are corporations formed,
organized and existing under Philippine laws and which owned real
properties covered under the Torrens system and whose properties were
included in the inventory of the estate of Pastor Y. Lim.
Although the above business entities dealt and engaged in business
with the public as corporations, all their capital, assets and equity were
however, were personally owned by the late Pastor Y Lim. Hence the alleged
stockholders and officers appearing in the respective articles of incorporation
of the above business entities were mere dummies of Pastor Y. Lim, and they
were listed therein only for purposes of registration with the Securities and
Exchange Commission.
ISSUE/S:
Whether or not the officers of the corporation shall be held liable
for the debts of the corporation.
RULING:
No.
The real properties included in the inventory of the estate of the Late
Pastor Y. Lim are in the possession of and are registered in the name of
private respondent corporations, which under the law possess a personality
separate and distinct from their stockholders. It is settled that a corporation
is clothed with personality separate and distinct from that of the persons

Page 340 of 1072

composing it. It may not generally be held liable for that of the persons
composing it. It may not be held liable for the personal indebtedness of its
stockholders or those of the entities connected with it.
Mere ownership by a single stockholder or by another corporation of all
or nearly all of the capital stock of a corporation is not of itself a sufficient
reason for disregarding the fiction of separate corporate personalities.
Moreover, to disregard the separate juridical personality of a corporation, the
wrong-doing must be clearly and convincingly established. The reliance
reposed by petitioner on the affidavits executed by Teresa Lim and Lani
Wenceslao is unavailing considering that the aforementioned documents
possess no weighty probative value pursuant to the hearsay rule, as the
affiants were not at all presented during the course of the proceedings in the
lower court.

Page 341 of 1072

TOPIC:PERSONAL LIABILITY OF DIRECTORS & OTHER CORPORATE OFFICERS


ADALIA B. FRANCISCO and MERRYLAND DEVELOPMENT
CORPORATION, petitioners,
vs.
RITA C. MEJIA, as Executrix of Testate Estate of ANDREA CORDOVA
VDA. DE GUTERREZ, respondent.
G.R. NO. 141617, August 14, 2001
FACTS:
Andrea Cordova Vda. de Gutierrez was the registered owner of a parcel
of land. Gutierrez and Cardale Financing and Realty Corporation executed a
Deed of Sale with Mortgage relating to the lots secured by a mortgage on
three of the four parcels of land. Cardale failed to settle its mortgage
obligation thus Gutierrez filed a complaint for rescission of the contract and
Cardale was represented by petitioner Adalia B. Francisco in her capacity as
Vice-President and Treasurer of Cardale.
The mortgaged parcels of land became delinquent in the payment of
real estate taxes culminated in their levy and auction sale with petitioner
Merryland as the highest bidder and whose President and majority
stockholder is Francisco. Appellee Francisco knew that Cardale of which she
was vice-president and treasurer had an outstanding obligation to Gutierrez
for the unpaid balance of the real properties. She also knew that Gutierrez
had a mortgage lien on the said properties to secure payment of the
aforesaid obligation. Despite such knowledge, appellee Francisco did not
inform Gutierrez's Estate or the Executrix as well as the trial court.
ISSUE/S:
Whether or not Francisco, a corporate officer should be held
personally liable.
RULING:
Yes.
With specific regard to corporate officers, the general rule is that the
officer cannot be held personally liable with the corporation, whether civilly

Page 342 of 1072

or otherwise, for the consequences of his acts, if he acted for and in behalf of
the corporation, within the scope of his authority and in good faith. In such
cases, the officer's acts are properly attributed to the corporation. However,
if it is proven that the officer has used the corporate fiction to defraud a third
party, or that he has acted negligently, maliciously or in bad faith, then the
corporate veil shall be lifted and he shall be held personally liable for the
particular corporate obligation involved.
The totality of the circumstances appertaining conduce to the
inevitable conclusion that petitioner Francisco acted in bad faith. It has been
established that Cardale failed to comply with its obligation to pay the
balance of the purchase price for the four parcels of land it bought from
Gutierrez where the case dragged on for about fourteen years with Francisco
being the Vice-President and Treasurer of the same. Francisco knew that the
properties subject of the mortgage had become tax delinquent, being the
officer charged with and yet she did not inform the Gutierrez estate or the
trial court of the tax arrears and of the notice from the City Treasurer so as to
prevent the auction sale and to protect its interests in the mortgaged
properties. Finally, in 1983, the properties were sold at public auction
wherein Merryland a corporation where Francisco is a stockholder and
concurrently acts as President and director was the highest bidder which
incident only serves to shed more light upon Francisco's fraudulent purposes.

Page 343 of 1072

TOPIC:PERSONAL LIABILITY OF DIRECTORS & OTHER CORPORATE OFFICERS


DEVELOPMENT BANK OF THE PHILIPPINES, petitioner,
vs.
HONORABLE COURT OF APPEALS and
REMINGTON INDUSTRIAL SALES CORPORATION, respondents.
G.R. NO. 126200, AUGUST 16, 2001
FACTS:
Marinduque Mining-Industrial Corporation obtained from the PNB
various loan accommodations it secured by a Deed of Real Estate Mortgage
and Chattel Mortgage in favor of PNB and another Mortgage Trust Agreement
on July 13, 1981, which was further amended to favor PNB and DBP of all of
its other real and personal properties and other real rights subsequently
acquired by Marinduque Mining. Marinduque Mining failed to settle its loan
obligations thus PNB and DBP instituted an extrajudicial foreclosure
proceedings over the mortgaged properties.
Remington Industrial Sales Corporation (Remington) for which
Marinduque Mining purchased construction materials and other merchandise
which remained unpaid filed a complaint for a sum of money and damages
impleading therein
PNB and DBP, Nonoc Mining, Maricalum Mining
Corporation (Maricalum Mining) and Island Cement Corporation (Island
Cement) as co-defendants. Accordingly, NMIC, Maricalum and Island Cement
which are newly created entities are practically owned wholly by defendants
PNB and DBP, and managed by their officers, aside from the fact that the
aforesaid co-defendants NMIC, Maricalum and Island Cement were organized
in such a hurry and in such suspicious circumstances by co-defendants PNB
and DBP after the supposed extrajudicial foreclosure of MMIC's assets as to
make their supposed projects assets, machineries and equipment which
were originally owned by co-defendant MMIC in fraud of the latters creditors.
ISSUE/S:
Whether or not the officers of the corporation should be held
personally liable.
HELD:

Page 344 of 1072

No.
PNB and DBP are mandated to foreclose on the mortgage when the
past due account had incurred arrearages pursuant to Section 1 of
Presidential Decree No. 385 (The Law on Mandatory Foreclosure). The banks
had no choice but to obey the statutory command.
The Court of Appeals made reference to two principles in corporation
law. The first pertains to transactions between corporations with interlocking
directors resulting in the prejudice to one of the corporations. This rule does
not apply in this case, however, since the corporation allegedly prejudiced
(Remington) is a third party, not one of the corporations with interlocking
directors (Marinduque Mining and DBP).
The second principle invoked by respondent court involves "directors x
x x who are creditors" which is also inapplicable herein. Here, the creditor of
Marinduque Mining is DBP, not the directors of Marinduque Mining. Neither
do we discern any bad faith on the part of DBP by its creation of Nonoc
Mining, Maricalum and Island Cement. As Remington itself concedes, DBP is
not authorized by its charter to engage in the mining business. The creation
of the three corporations was necessary to manage and operate the assets
acquired in the foreclosure sale lest they deteriorate from non-use and lose
their value.

Page 345 of 1072

TOPIC:PERSONAL LIABILITY OF DIRECTORS & OTHER CORPORATE OFFICERS


AHS/PHILIPPINES, INC., GERVACIO R. AMISTOSO and CONSTANCIO V.
HALILI, petitioners,
vs.
COURT OF APPEALS and ALFONSO R. BAYANI, respondents.
G.R. No. 111807, June 14, 1996
FACTS:
Petitioner Corporation was engaged in the sale and manufacture of
medicines and pharmaceuticals in the country and did substantial business
with government hospitals. On 1 June 1970 it hired private respondent as an
Area Manager for Visayas and Mindanao, and later appointed him Manager of
its Cebu branch. On 30 January 1978 private respondent was dismissed from
the service on ground of insubordination. Private respondent filed a
complaint for damages alleging that petitioners were directly encouraging,
abetting and promoting bribery in the guise of "commissions,"
"entertainment expenses" and "representation expenses" which were given
to various government hospital officials in exchange for favorable
recommendations, approvals and actual purchases of medicines and
pharmaceuticals.
ISSUE/S:
Whether or not petitioners Amistoso as president and Halili as
vice-president of the same corporation be held joint and
solidarily liable with the corporation for the dismissal of
respondent.
RULING:
No.
The Supreme Court have already said that corporate officers are not
personally liable for money claims of discharged corporate employees unless
they acted with evident malice and bad faith in terminating their
employment. In the case at bar, while petitioners Amistoso and Halili may
have had a hand in the relief of respondent, Bayani, there are no indications
of malice and bad faith on their part. On the contrary it is apparent that the
relief order was a business judgment on the part of the officers, with the best

Page 346 of 1072

interest of the corporation in mind, based on their opinion that respondent


Bayani had failed to perform the duties expected of him.

TOPIC: PERSONAL LIABILITY OF DIRECTORS AND OTHER CORPORATE


OFFICERS
COMPLEX ELECTRONICS EMPLOYEES ASSOCIATION (CEEA)
represented by its union president CECILIA TALAVERA, GEORGE
ARSOLA, MARIO DIAGO AND SOCORRO BONCAYAO, petitioners,
vs.
THE NATIONAL LABOR RELATIONS COMMISSION, COMPLEX
ELECTRONICS CORPORATION, IONICS CIRCUIT, INC., LAWRENCE QUA,
REMEDIOS DE JESUS, MANUEL GONZAGA, ROMY DELA ROSA,
TERESITA ANDINO, ARMAN CABACUNGAN, GERRY GABANA, EUSEBIA
MARANAN and BERNADETH GACAD, respondents.
G.R. No. 121315. July 19, 1999
310 SCRA 143
FACTS:
Complex Electronics Corporation (Complex) was engaged in the
manufacture of electronic products. It was actually a subcontractor of
electronic products where its customers gave their job orders, sent their own
materials and consigned their equipment to it. The customers were foreignbased companies with different product lines and specifications requiring the
employment of workers with specific skills for each product line. Thus, there
was the AMS Line for the Adaptive Micro System, Inc., the Heril Line for Heril
Co., Ltd., the Lite-On Line for the Lite-On Philippines Electronics Co., etc.
The rank and file workers of Complex were organized into a union
known as the Complex Electronics Employees Association, herein referred to
as the Union. Complex received a facsimile message from Lite-On Philippines
Electronics Co., requiring it to lower its price by 10%. Consequently, a
meeting was held between Complex and the personnel of the Lite-On
Production Line. Complex informed its Lite-On personnel that such request of
lowering their selling price by 10% was not feasible as they were already
incurring losses at the present prices of their products. Under such
circumstances, Complex regretfully informed the employees that it was left
with no alternative but to close down the operations of the Lite-On Line.

Page 347 of 1072

The Union, on the other hand, pushed for a retrenchment pay


equivalent to one (1) month salary for every year of service, which Complex
refused. Complex filed a notice of closure of the Lite-On Line with the
Department of Labor and Employment (DOLE) and the retrenchment of the
ninety-seven (97) affected employees. Thus, the Union filed a notice of strike
with the National Conciliation and Mediation Board (NCMB). Two days
thereafter, the Union conducted a strike vote which resulted in a "yes" vote.
A complaint was, thereafter, filed with the Labor Arbitration Branch of
the NLRC for unfair labor practice, illegal closure/illegal lockout, money
claims for vacation leave, sick leave, unpaid wages, 13th month pay,
damages and attorney's fees. The Union alleged that the pull-out of the
machinery, equipment and materials from the company premises, which
resulted to the sudden closure of the company was in violation of Section 3
and 8, Rule XIII, Book V of the Labor Code of the Philippines and the existing
CBA. Ionics was impleaded as a party defendant because the officers and
management personnel of Complex were also holding office at Ionics with
Lawrence Qua as the President of both companies.
ISSUE/S:
Whether Lawrence Qua can be held personally liable as director
of said corporation.
RULING:
In the matter of personal liability of Lawrence Qua, it is settled that in
the absence of malice or bad faith, a stockholder or an officer of a
corporation cannot be made personally liable for corporate liabilities. In the
present case, while it may be true that the equipment, materials and
machinery were pulled-out of Complex and transferred to Ionics during the
night, their action was sufficiently explained by Lawrence Qua that the pullout of the machinery, equipment and materials was effected during
nighttime is not per se an indicia of bad faith on the part of respondent Qua
since he had no other recourse, and the same was dictated by the prevailing
mood of unrest as the laborers were already vandalizing the equipment, bent
on picketing the company premises and threats to lock out the company
officers were being made. Such acts of respondent Qua were, in fact, made
pursuant to the demands of Complex's customers who were already alarmed
by the pending labor dispute and imminent strike to be stage by the
laborers, to have their equipment, machinery and materials pull out of
Complex. As such, these acts were merely done pursuant to his official
functions and were not, in any way, made with evident bad faith.
The Supreme Court perceives no intention on the part of Lawrence Qua
and the other officers of Complex to defraud the employees and the Union.
They were compelled to act upon the instructions of their customers who
were the real owners of the equipment, materials and machinery. The

Page 348 of 1072

prevailing labor unrest permeating within the premises of Complex left the
officers with no other choice but to pull them out of Complex at night to
prevent their destruction. Thus, we see no reason to declare Lawrence Qua
personally liable to the Union.
TOPIC: PERSONAL LIABILITY OF DIRECTORS AND OTHER CORPORATE
OFFICERS
ERNESTINA CRISOLOGO-JOSE, petitioner,
vs.
COURT OF APPEALS and RICARDO S. SANTOS, JR. in his own behalf
and as Vice-President for Sales of Mover Enterprises, Inc.,
respondents.
G.R. No. 80599. September 15, 1989
FACTS:
In 1980, plaintiff Ricardo S. Santos, Jr. was the vice-president of Mover
Enterprises, Inc. in-charge of marketing and sales; and the president of the
said corporation was Atty. Oscar Z. Benares. On April 30, 1980, Atty. Benares,
in accommodation of his clients, the spouses Jaime and Clarita Ong, issued
Check No. 093553 drawn against Traders Royal Bank, dated June 14, 1980, in
the amount of P45,000.00 payable to defendant Ernestina Crisologo-Jose.
Since the check was under the account of Mover Enterprises, Inc., the same
was to be signed by its president, Atty. Oscar Z. Benares, and the treasurer of
the said corporation. However, since at that time, the treasurer of Mover
Enterprises was not available, Atty. Benares prevailed upon the plaintiff,
Ricardo S. Santos, Jr., to sign the aforesaid chEck as an alternate story.
Plaintiff Ricardo S. Santos, Jr. did sign the check.
ISSUE/S:
Whether or not respondent being only a co-signatory does not
detract him from his personal liability.
RULING:
Respondent Santos is an accommodation party and is, therefore, liable
for the value of the check. The fact that he was only a co-signatory does not
detract from his personal liability. A co-maker or co-drawer under the
circumstances in this case is as much an accommodation party as the other
co-signatory or, for that matter, as a lone signatory in an accommodation
instrument. Under the doctrine in Philippine Bank of Commerce vs. Aruego,
supra, he is in effect a co-surety for the accommodated party with whom he
and his co-signatory, as the other co-surety, assume solidary liability ex lege
for the debt involved. With the dishonor of the check, there was created a
debtor-creditor relationship, as between Atty. Benares and respondent

Page 349 of 1072

Santos, on the one hand, and petitioner, on the other. This circumstance
enables respondent Santos to resort to an action of consignation where his
tender of payment had been refused by petitioner.
TOPIC: PERSONAL LIABILITY OF DIRECTORS AND OTHER CORPORATE
OFFICERS
FCY CONSTRUCTION GROUP, INC., and FRANCIS C. YU, petitioners,
vs.
THE COURT OF APPEALS, THE HON. JOSE C. DE LA RAMA, Presiding
Judge, Branch 139, Regional Trial Court, NCJR, Makati City, MetroManila, and LEY CONSTRUCTION AND DEVELOPMENT CORPORATION,
respondents.
G.R. No. 123358. February 1, 2000
324 SCRA 270
FACTS:
On June 29, 1993, private respondent Ley Construction and
Development Corporation filed a Complaint for collection of a sum of money
with application for preliminary attachment against petitioner FCY
Construction Group, Inc. and Francis C. Yu with the Makati Regional Trial
Court which was docketed as Civil Case No. 93-2112. Private respondent
alleged that it had a joint venture agreement with petitioner FCY
Construction Group, Inc. (wherein petitioner Francis C. Yu served as
President) over the Tandang Sora Commonwealth Flyover government
project, for which it had provided funds and construction materials. The
Complaint was filed in order to compel petitioners to pay its half share in the
collections received in the project as well as those yet to be received therein.
In support of its application for a writ of attachment, private respondent
alleged that petitioners were guilty of fraud in incurring the obligation and
had fraudulently misapplied or converted the money paid them, to which it
had an equal share.
ISSUE/S:
Whether or not petitioner Francis Yu can be held personally liable
to the acts of the corporation.
RULING:
Petitioner Francis Yu cannot be made liable in his individual capacity if
he indeed entered into and signed the contract in his official capacity as
President, in the absence of stipulation to that effect, due to the personality
of the corporation being separate and distinct from the persons composing it.
However, while we agree that petitioner Francis Yu cannot be held solidarily
liable with Petitioner Corporation merely because he is the President thereof

Page 350 of 1072

and was involved in the transactions with Private Corporation, we also note
that there exists instances when corporate officers may be held personally
liable for corporate acts.
Such exceptions were outlined in Tramat Mercantile, Inc. vs. Court of
Appeals, as follows Personal liability of a corporate director, trustee or
officer along (although not necessarily) with the corporation may so validly
attach, as a rule, only when
1. He assents
a) to a patently unlawful act of the corporation, or
b) for bad faith or gross negligence in directing its affairs, or
c) for conflict of interest, resulting in damages to the
corporation, its stockholders or other persons;
2. He consents to the issuance of watered down stocks or who,
having knowledge thereof, does not forthwith file with the
corporate secretary his written objection thereto;
3. He agrees to hold himself personally and solidarily liable with the
corporation; or
4. He is made, by a specific provision of law, to personally answer
for his corporate action.

Page 351 of 1072

TOPIC: PERSONAL LIABILITY OF DIRECTORS AND OTHER CORPORATE


OFFICERS
RICARDO A. LLAMADO, petitioner,
vs.
COURT OF APPEALS and PEOPLE OF THE PHILIPPINES, respondents.
G.R. No. 99032. March 26, 1997
270 SCRA 423
FACTS:
It was the practice in the corporation for petitioner to sign blank checks
and leave them with Pascual so that Pascual could make disbursements and
enter into transactions even in the absence of petitioner. One of the checks
which petitioner signed in blank and gave to Pascual is the check in question,
Exhibit "A."
The check was later issued to private complainant, filled up with the
amount P186,500.00 and date November 4, 1983.The check was dishonored
when private complainant presented it for payment because its payment had
been stopped. However, there were also no sufficient funds in the account to
cover the amount of the check.
Private complainant went to see Aida Tan, the "Secretary" of Pan-Asia
Finance Corporation, about the dishonor of the check because "she was the
one who handled the check and gave it to me." He returned the check to
Aida Tan who gave him a receipt for it, and promised "to return the cash
money." However, she did not do so. Instead, she returned the check to
private complainant.
On November 11, 1983, private complainant entered into an
agreement with petitioner whereby Pan-Asia Finance Corporation would pay
private complainant 10% of the P186,500.00 by November 14, or 15, and the
balance will be rolled over for 90 days. Private respondent was not however
paid as agreed upon.
ISSUE/S:
Whether petitioner can be held personally liable for the amount
of the check because he signed the same in his capacity as
Treasurer of the corporation.
RULING:

Page 352 of 1072

Petitioner's argument that he should not be held personally liable for


the amount of the check because it was a check of the Pan Asia Finance
Corporation and he signed the same in his capacity as Treasurer of the
corporation is also untenable. The third paragraph of Section 1 of BP Blg. 22
states:
Where the check is drawn by a corporation, company or
entity, the person or p ersons who actually signed the
check in behalf of such drawer shall be liable under this
Act.

Page 353 of 1072

TOPIC: PERSONAL LIABILITY OF DIRECTORS AND OTHER CORPORATE


OFFICERS
MAM REALTY DEVELOPMENT CORPORATION and MANUEL CENTENO,
petitioners, vs.
NATIONAL LABOR RELATIONS COMMISSION and CELSO B.
BALBASTRO respondents.
G.R. No. 114787. June 2, 1995
244 SCRA797
FACTS:
The case originated from a complaint filed with the Labor Arbiter by
private respondent Celso B. Balbastro against herein petitioners, MAM Realty
Development Corporation ("MAM") and its Vice President Manuel P. Centeno,
for wage differentials, "ECOLA," overtime pay, incentive leave pay, 13th
month pay (for the years 1988 and 1989), holiday pay and rest day pay.
Balbastro alleged that he was employed by MAM as a pump operator in 1982
and had since performed such work at its Rancho Estate, Marikina, Metro
Manila. He earned a basic monthly salary of P1,590.00 for seven days of
work a week that started from 6:00 a.m. to up until 6:00 p.m. daily.
MAM countered that Balbastro had previously been employed by
Francisco Cacho and Co., Inc., the developer of Rancho Estates. Sometime in
May 1982, his services were contracted by MAM for the operation of the
Rancho Estates' water pump. He was engaged, however, not as an
employee, but as a service contractor, at an agreed fee of P1,590.00 a
month. Similar arrangements were likewise entered into by MAM with one
Rodolfo Mercado and with a security guard of Rancho Estates III
Homeowners' Association. Under the agreement, Balbastro was merely made
to open and close on a daily basis the water supply system of the different
phases of the subdivision in accordance with its water rationing scheme. He
worked for only a maximum period of three hours a day, and he made use of
his free time by offering plumbing services to the residents of the
subdivision. He was not at all subject to the control or supervision of MAM
for, in fact, his work could so also be done either by Mercado or by the
security guard.
On 23 May 1990, prior to the filing of the complaint, MAM executed a
Deed of Transfer, effective 01 July 1990, in favor of the Rancho Estates Phase
III Homeowners Association, Inc., conveying to the latter all its rights and
interests over the water system in the subdivision.
ISSUE/S:

Page 354 of 1072

Whether petitioner Centeno can be held solidary liabile to the


acts of the corporation.
RULING:
A corporation, being a juridical entity, may act only through its
directors, officers and employees. Obligations incurred by them, acting as
such corporate agents, are not theirs but the direct accountabilities of the
corporation they represent. True, solidary liabilities may at times be incurred
but only when exceptional circumstances warrant such as, generally, in the
following cases:
1. When directors and trustees or, in appropriate cases, the
officers of a corporation
a) vote for or assent to patently unlawful acts of the
corporation;
b) act in bad faith or with gross negligence in directing the
corporate affairs;
c) are guilty of conflict of interest to the prejudice of the
corporation, its stockholders or members, and other
persons.
2. When a director or officer has consented to the issuance of
watered stocks or who, having knowledge thereof, did not
forthwith file with the corporate secretary his written objection
thereto.
3. When a director, trustee or officer has contractually agreed or
stipulated to hold himself personally and solidarily liable with
the Corporation.
4. When a director, trustee or officer is made, by specific
provision of law, personally liable for his corporate action.
In labor cases, for instance, the Court has held corporate directors and
officers solidarily liable with the corporation for the termination of
employment of employees done with malice or in bad faith. In the case at
Bench, there is nothing substantial on record that can justify, prescinding
from the foregoing, petitioner Centeno's solidary liability with the
corporation.

Page 355 of 1072

TOPIC: PERSONAL LIABILITY OF DIRECTORS AND OTHER CORPORATE


OFFICERS
SERGIO F. NAGUIAT, doing business under the name and style
SERGIO F. NAGUIAT ENT., INC., & CLARK FIELD TAXI, INC.
VS.
NATIONAL LABOR RELATIONS COMMISSION (THIRD DIVISION),
NATIONAL ORGANIZATION OF WORKINGMEN and its members,
LEONARDO T. GALANG, et al.
G.R. No. 116123, March 13, 1997
FACTS:
CFTI and NEI are both family-owned corporations where Sergio is the
President and Antolin Naguiat as VP. CFTI hold a concessionaire's contract
with the Army Air Force Exchange Services (AAFES) for the operation of taxi
services within Clark Air Base. Due to the phase-out of the US military bases
in the Philippines, from which Clark Air Base was not spared, the AAFES was
dissolved, and the services of individual respondents were officially
terminated. The Drivers Union and CFTI negotiated as to the severance pay
to which the members of the union, except the herein respondents,
accepted. Respondents disaffiliated themselves with the Union and joined
the NOWM. They filed a complaint against "Sergio F. Naguiat doing business
under the name and style Sergio F. Naguiat Enterprises, Inc., AAFES with
Mark Hooper as Area Service Manager, Pacific Region, and AAFES Taxi
Drivers Association with Eduardo Castillo as President," for payment of
separation pay due to termination/ phase-out. The complaint was later
amended to include additional taxi drivers who were similarly situated as
complainants, and CFTI with Antolin T. Naguiat as vice president and general
manager, as party respondent.
In their complaint, they alleged that they are regular employees of NEI
although their individual applications for employment were approved by
CFTI. They claimed to have been assigned to NEI after having been hired by
CFTI, and that the former thence managed, controlled and supervised their
employment.

Page 356 of 1072

Private respondents were held liable by the labor arbiter prompting


them to file their individual appeal before the NLRC, which affirmed the same
subject to some modifications.
ISSUE/S:
Whether or not Sergio F. Naguiat at and Antolin Naguiat, as
officers and SHs of CFTI could be held personally accountable for
corporate debts; and who were not impleaded as defendants.
RULING:
Our jurisprudence is wanting as to the definite scope of "corporate
tort." Essentially, "tort" consists in the violation of a right given or the
omission of a duty imposed by law. Simply stated, tort is a breach of a legal
duty. Article 283 of the Labor Code mandates the employer to grant
separation pay to employees in case of closure or cessation of operations of
establishment or undertaking not due to serious business losses or financial
reverses, which is the condition obtaining at bar. CFTI failed to comply with
this law-imposed duty or obligation. Consequently, its stockholder who was
actively engaged in the management or operation of the business should be
held personally liable.
Furthermore, in MAM Realty Development vs. NLRC, the Court
recognized that a director or officer may still be held solidarily liable with a
corporation by specific provision of law. Thus: . . . A corporation, being a
juridical entity, may act only through its directors, officers and employees.
Obligations incurred by them, acting as such corporate agents, are not theirs
but the direct accountabilities of the corporation they represent. True,
solidary liabilities may at times be incurred but only when exceptional
circumstances warrant such as, generally, in the following cases: xxx xxx xxx
4. When a director, trustee or officer is made, by specific provision of law,
personally liable for his corporate action.
As pointed out earlier, the fifth paragraph of Section 100 of the
Corporation Code specifically imposes personal liability upon the stockholder
actively managing or operating the business and affairs of the close
corporation. In fact, in posting the surety bond required by this Court for the
issuance of a temporary restraining order enjoining the execution of the
assailed NLRC Resolutions, only Sergio F. Naguiat, in his individual and
personal capacity, principally bound himself to comply with the obligation
thereunder, i.e., "to guarantee the payment to private respondents of any
damages which they may incur by reason of the issuance of a temporary
restraining order sought, if it should be finally adjudged that said principals
were not entitled thereto.

Page 357 of 1072

The Court here finds no application to the rule that a corporate officer
cannot be held solidarily liable with a corporation in the absence of evidence
that he had acted in bad faith or with malice. In the present case, Sergio
Naguiat is held solidarily liable for corporate tort because he had actively
engaged in the management and operation of CFTI, a close corporation.
Antolin T. Naguiat was the vice president of the CFTI. Although he
carried the title of "general manager" as well, it had not been shown that he
had acted in such capacity. Furthermore, no evidence on the extent of his
participation in the management or operation of the business was preferred.
In this light, he cannot be held solidarily liable for the obligations of CFTI and
Sergio Naguiat to the private respondents.

Page 358 of 1072

TOPIC: PERSONAL LIABILITY OF DIRECTORS AND OTHER CORPORATE


OFFICERS
PROGRESS HOMES and ERMELO ALMEDA
VS.
NATIONAL LABOR RELATIONS COMMISSION, GREGORIO A. MEDRANO,
DANTE BAGUIO, JAIME GUAN, JOSE SAPALARAN, RONNIE DELPINO,
DIONISIO FRANCISCO and ELMER BAGUIO
G.R. No. 106212, March 7, 1997
269 SCRA 274
FACTS:
Petitioner Progress Homes Subdivision is a housing project undertaken
by the Ermelo M. Almeda Foundation, Inc., a non-stock organization duly
registered with the Securities and Exchange Commission (SEC). When it
engaged in constructing low-cost housing units for low-income employees, it
named its project "Progress Homes Subdivision" in Camarines Sur. The other
petitioner, Ermelo Almeda, is the President and General Manager of Progress
Homes and the owner of the land where the Progress Homes Subdivision is
located.
Private respondents allegedly were among the workers employed by
petitioners in their construction and development of the subdivision from
1986 to 1988. They were paid varying salaries.
Forty of these workers, including private respondents, filed before the
NLRC Arbitration Branch a petition for reinstatement, salary adjustment,
ECOLA, overtime pay and 13th month pay.
Petitioners amicably settled the case with thirty-three of the laborers,
leaving private respondents as the only claimants. Private respondents
alleged that they worked as laborers and carpenters for 8.5 hours a day at a
salary below the minimum wage and that when they demanded payment of
the benefits due them, they were summarily dismissed and barred from
entering the workplace.
Petitioners denied that private respondents were regular employees
claiming that they were only project employees and that there was no
employer-employee relationship between them.

Page 359 of 1072

ISSUE/S:
Whether or not petitioner Almeda could be held jointly and
severally liable with Progress Homes.
RULING:
NO.
The NLRC committed grave abuse of discretion when it affirmed the
Labor Arbiter's decision holding petitioner Almeda jointly and severally liable
with Progress Homes. The Court has held that corporate directors and
officers are solidarily liable with the corporation for the termination of
employment of employees only if the termination is done with malice or in
bad faith.The Labor Arbiter's decision failed to disclose why Almeda was
made personally liable. There appears no evidence on record that he acted
maliciously or in bad faith in terminating the services of private
respondents.Petitioner Almeda, therefore, should not have been made
personally answerable for the payment of private respondents' salaries.

Page 360 of 1072

TOPIC: PERSONAL LIABILITY OF DIRECTORS AND OTHER CORPORATE


OFFICERS
REAHS CORPORATION, SEVERO CASTULO, ROMEO PASCUA, and
DANIEL VALENZUELA
VS.
NATIONAL LABOR RELATIONS COMMISSION, BONIFACIO RED
VICTORIA PADILLA, MA. SUSAN R. CALWIT, SONIA DE LA CRUZ,
SUSAN DE LA CRUZ, EDNA WAHINGON, NANCY B. CENITA and
BENEDICTO A. TULABING
G.R. No. 117473 April 15, 1997
FACTS:
Complainant Bonifacio Red, Benedicto Tulabing, Nancy Cenita and
Susan Calwit Edna Wahingon, Susan dela Cruz, Sonia dela Cruz and Victoria
Padilla instituted an action before a the labor arbiter for unfair labor practice
and illegal dismissal, claims for separation pay, underpayment of wages,
holiday pay and 13th month pay.
On the other hand, respondents allege that a certain Ms Soledad
Domingo, the sole proprietress and operator of Rainbow Sauna located at
316 Araneta Avenue, Quezon City, offered to sell her business to respondent
Reah's Corporation After the sale, all the assets of Ms Domingo were turned
over to respondent Reah's, which put a sing-along coffee shop and massage
clinic; that complainant Red started his employment on the first week of
December 1988 as a roomboy at P50.00/day and was given living quarters
inside the premises as he requested; that sometime in March 1989,
complainant Red asked permission to go to Bicol for a period of ten (10)
days, which was granted, and was given an advance money of P1,200.00 to
bring some girls from the province to work as attendants at the respondent's
massage clinic, that it was only on January 1, 1990 that complainant Red
returned and was re-hired under the same terms and conditions of his
previous employment with the understanding that he will have to refund the
P1,200.00 cash advance given to him; that due to poor business, increase in
the rental cost and the failure of Meralco to reconnect the electrical services
in the establishment, it suffered losses leading to its closure.
The labor arbiter rendered judgment dismissing private respondents'
complaints for unfair labor practice and illegal dismissal but upholding the
claims for separation pay, underpayment of wages, holiday pay and 13th
month pay. The NLRC dismissed the appeal.

Page 361 of 1072

ISSUE/S:
Whether or not petitioners-officers can be held jointly and
severally liable with the corporation in the payment of separation
pay to private respondents under Article 283 of the Labor Code.
RULING:
As a general rule established by legal fiction, the corporation has a
personality separate and distinct from its officers, stockholders and
members. Hence, officers of a corporation are not personally liable for their
official acts unless it is shown that they have exceeded their authority. This
fictional veil, however, can be pierced by the very same law which created it
when "the notion of the legal entity is used as a means to perpetrate fraud,
an illegal act, as a vehicle for the evasion of an existing obligation, and to
confuse legitimate issues". Under the Labor Code, for instance, when a
corporation violates a provision declared to be penal in nature, the penalty
shall be imposed upon the guilty officer or officers of the corporation.
In the case at bar, the thrust of petitioners' arguments was aimed at
confining liability solely to the corporation, as if the entity were an
automaton designed to perform functions at the push of a button. The issue,
however, is not limited to payment of separation pay under Article 283 but
also payment of labor standard benefits such as underpayment of wages,
holiday pay and 13th month pay to two of the private respondents. While
there is no sufficient evidence to conclude that petitioners have
indiscriminately stopped the entity's business, at the same time, petitioners
have opted to abstain from presenting sufficient evidence to establish the
serious and adverse financial condition of the company.

Page 362 of 1072

TOPIC: PERSONAL LIABILITY OF DIRECTORS AND OTHER CORPORATE


OFFICERS
LYDIA SANTOS
VS.
NATIONAL LABOR RELATIONS COMMISSION AND SECURITY BANK
AND TRUST COMPANY
G.R. NO. 76721 SEPTEMBER 21, 1987
254 SCRA 673
FACTS:
Private respondent was hired to be the project accountant for MMDC's
mining operations in Gatbo, Bacon, Sorsogon. Private respondent sent to Mr.
Gil Abao, the MMDC corporate treasurer, a memorandum calling the latter's
attention to the failure of the company to comply with the withholding tax
requirements of, and to make the corresponding monthly remittances to, the
Bureau of Internal Revenue ("BIR") on account of delayed payments of
accrued salaries to the company's laborers and employees.
In a letter, private respondent was informed that his employment
contract was terminated as decided by the Board of MMDC. Private
respondent filed with the NLRC a complaint for illegal dismissal against
MMDC. Millena alleged, among other things, that his dismissal was merely an
offshoot of his letter of 12 August 1986 to Abao about the company's
inability to pay its workers and to remit withholding taxes to the BIR. LA
Aurellano ruled that there was no valid cause for terminating complainant's
employment. Upon appeal, the NLRC affirmed the decision of the Labor
Arbiter.
ISSUE/S:
Whether or not petitioner is solidarily liable with MMDC even in
the absence of bad faith and malice on his part.
RULING:
A corporation is a juridical entity with legal personality separate and
distinct from those acting for and in its behalf and, in general, from the
people comprising it. The rule is that obligations incurred by the corporation,
acting through its directors, officers and employees, are its sole liabilities.
Nevertheless, being a mere fiction of law, peculiar situations or valid grounds
can exist to warrant, albeit done sparingly, the disregard of its independent
being and the lifting of the corporate veil. As a rule, this situation might arise
when a corporation is used to evade a just and due obligation or to justify a
wrong, to shield or perpetrate fraud, to carry out similar other unjustifable

Page 363 of 1072

aims or intentions, or as a subterfuge to commit injustice and so circumvent


the law.
In Tramat Mercantile, Inc., vs. Court of Appeals,the Court has collated
the settled instances when, without necessarily piercing the veil of corporate
fiction, personal civil liability can also be said to lawfully attach to a
corporate director, trustee or officer; to wit: When
1. He assents
a) to a patently unlawful act of the corporation, or
b) for bad faith or gross negligence in directing its affairs,
or
c) for conflict of interest, resulting in damages to the
corporation, its stockholders or other persons;
2. He consents to the issuance of watered stocks or who, having
knowledge thereof, does not forthwith file with the corporate
secretary his written objection thereto;
3. He agrees to hold himself personally and solidarily liable with
the corporation; or
4. He is made, by a specific provision of law, to personally
answer for his corporate action.
The case of petitioner is way off these exceptional instances. It is not
even shown that petitioner has had a direct hand in the dismissal of private
respondent enough to attribute to him (petitioner) a patently unlawful act
while acting for the corporation. Neither can Article 289 of the Labor Code be
applied since this law specifically refers only to the imposition of penalties
under the Code. It is undisputed that the termination of petitioner's
employment has, instead, been due, collectively, to the need for a further
mitigation of losses, the onset of the rainy season, the insurgency problem in
Sorsogon and the lack of funds to further support the mining operation in
Gatbo.

Page 364 of 1072

TOPIC: PERSONAL LIABILITY OF DIRECTORS AND OTHER CORPORATE


OFFICERS
JOSE O. SIA
VS.
THE PEOPLE OF THE PHILIPPINES
G.R. No. L-30896, April 28, 1983
121 SCRA 655
FACTS:
Jose 0. Sia was General Manager of the Metal Manufacturing Company
of the Philippines, Inc. Because his company was in need of raw materials to
be imported from abroad, he applied for a letter of credit to import steel
sheets from Mitsui Bussan Kaisha, Ltd. of Tokyo, Japan directed to the
Continental Bank. His application has been approved and the letter of credit
was opened in the amount of $18,300 and the goods arrived sometime in
July, 1963. The bill of exchange issued for the purpose of collecting the
unpaid account thereon having fallen due neither accused nor his company
having made payment thereon notwithstanding demands, and the accounts
having reached the sum in pesos of P46,818.68 after deducting his deposit
valued at P28,736.47; that was the reason why upon complaint by
Continental Bank, the Fiscal filed the information after preliminary
investigation.
ISSUE/S:
Whether petitioner Jose O. Sia, having only acted for and in
behalf of the Metal Manufacturing Company of the Philippines
(Metal Company, for short) as President thereof in dealing with
the complainant, the Continental Bank, (Bank for short) he may
be liable for the crime charged.
RULING:
The case cited by the Court of Appeals in support of its stand-Tan Boon
Kong case, supra-may however not be squarely applicable to the instant case
in that the corporation was directly required by law to do an act in a given
manner, and the same law makes the person who fails to perform the act in
the prescribed manner expressly liable criminally. The performance of the act
is an obligation directly imposed by the law on the corporation. Since it is a

Page 365 of 1072

responsible officer or officers of the corporation who actually perform the act
for the corporation, they must of necessity be the ones to assume the
criminal liability; otherwise this liability as created by the law would be
illusory, and the deterrent effect of the law, negated.
In the present case, a distinction is to be found with the Tan Boon Kong
case in that the act alleged to be a crime is not in the performance of an act
directly ordained by law to be performed by the corporation. The act is
imposed by agreement of parties, as a practice observed in the usual pursuit
of a business or a commercial transaction. The offense may arise, if at all,
from the peculiar terms and condition agreed upon by the parties to the
transaction, not by direct provision of the law. The intention of the parties,
therefore, is a factor determinant of whether a crime was committed or
whether a civil obligation alone intended by the parties. With this
explanation, the distinction adverted to between the Tan Boon Kong case and
the case at bar should come out clear and meaningful. In the absence of an
express provision of law making the petitioner liable for the criminal offense
committed by the corporation of which he is a president as in fact there is no
such provisions in the Revised Penal Code under which petitioner is being
prosecuted, the existence of a criminal liability on his part may not be said to
be beyond any doubt. In all criminal prosecutions, the existence of criminal
liability for which the accused is made answerable must be clear and certain.
The maxim that all doubts must be resolved in favor of the accused is always
of compelling force in the prosecution of offenses. This Court has thus far not
ruled on the criminal liability of an officer of a corporation signing in behalf of
said corporation a trust receipt of the same nature as that involved herein. In
the case of Samo vs. People, L-17603-04, May 31, 1962, the accused was not
clearly shown to be acting other than in his own behalf, not in behalf of a
corporation.

Page 366 of 1072

TOPIC: PERSONAL LIABILITY OF DIRECTORS AND OTHER CORPORATE


OFFICERS
TRAMAT MERCANTILE, INC. AND DAVID ONG
VS.
HON. COURT OF APPEALS AND MELCHOR DE LA CUESTA
G.R. No. 111008, November 7, 1994
FACTS:
Melchor de la Cuesta, doing business under the name and style of
"Farmers Machineries," sold to Tramat Mercantile, Inc. ("Tramat"), 1 unit
HINOMOTO TRACTOR Model. In payment, David Ong, Tramat's president and
manager, issued a postdated check for P33,500.00. Tramat, in turn, sold the
tractor, together with an attached lawn mower fabricated by it to NAWASA.
David Ong caused a "stop payment" of the check when NAWASA refused to
pay the tractor and lawn mower after discovering that, aside from some
stated defects of the attached lawn mower, the engine (sold by de la Cuesta)
was a reconditioned unit.
Melchor de la Cuesta filed an action for the recovery of P33,500.00.
Ong, in his answer, averred, among other things, that de la Cuesta had no
cause of action and that the questioned transaction was between Farmers
Machineries and Tramat Mercantile, Inc., and not with Ong in his personal
capacity and that the payment of the check was stopped because the
subject tractor had been priced as a brand new, not as a reconditioned unit.
The trial court ruled in favor of Farmers Machineries as represented by
de la Cuesta and holding David Ong jointly and severally liable with TRAMAT
which was affirmed by the CA on appeal.
ISSUE/S:
Whether or not defendant Ong could be held jointly and severally
liable with TRAMAT to de la Cuesta?
RULING:
NO.
Personal liability of a corporate director, trustee or officer along
(although not necessarily) with the corporation may so validly attach, as a
rule, only when
1. He assents

Page 367 of 1072

a) to a patently unlawful act of the corporation, or


b) for bad faith, or gross negligence in directing its affairs, or
c) for conflict of interest, resulting in damages to the corporation,
its stockholders or other persons;
2. He consents to the issuance of watered stocks or who, having
knowledge thereof, does not forthwith file with the corporate secretary
his written objection thereto;
3. He agrees to hold himself personally and solidarily liable with the
corporation; or
4. He is made, by a specific provision of law, to personally answer for his
corporate action.
In the case at bench, there is no indication that petitioner David Ong
could be held personally accountable under any of the abovementioned
cases.

Page 368 of 1072

TOPIC: DISLOYALTY
JOHN GOKONGWEI, JR., petitioner,
vs.
SECURITIES AND EXCHANGE COMMISSION, ANDRES M. SORIANO,
JOSE M. SORIANO, ENRIQUE ZOBEL, ANTONIO ROXAS, EMETERIO
BUNAO, WALTHRODE B. CONDE, MIGUEL ORTIGAS, ANTONIO PRIETO,
SAN MIGUEL CORPORATION, EMIGDIO TANJUATCO, SR., and
EDUARDO R. VISAYA, respondents.
G.R. No. L-45911 April 11, 1979
89 SCRA 336
FACTS:
The Universal Robina Corporation, a corporation engaged in business
competitive to that of respondent corporation, began acquiring shares
therein until September 1976 when its total holding amounted to 622,987
shares: that in October 1972, the Consolidated Foods Corporation (CFC)
likewise began acquiring shares in respondent corporation until its total
holdings amounted to P543,959.00 in September 1976; that on January 12,
1976, petitioner, who is president and controlling shareholder of Robina and
CFC (both closed corporations) purchased 5,000 shares of stock of
respondent corporation, and thereafter, in behalf of himself, CFC and Robina,
"conducted malevolent and malicious publicity campaign against SMC" to
generate support from the stockholder "in his effort to secure for himself and
in representation of Robina and CFC interests, a seat in the Board of
Directors of SMC", that in the stockholders' meeting of March 18, 1976,
petitioner was rejected by the stockholders in his bid to secure a seat in the
Board of Directors on the basic issue that petitioner was engaged in a
competitive business and his securing a seat would have subjected
respondent corporation to grave disadvantages; that "petitioner nevertheless
vowed to secure a seat in the Board of Directors at the next annual meeting;
that thereafter the Board of Directors amended the by-laws as afore-stated.
On October 22, 1976, petitioner, as stockholder of respondent San
Miguel Corporation, filed with the Securities and Exchange Commission (SEC)
a petition for "declaration of nullity of amended by-laws, cancellation of
certificate of filing of amended by- laws, injunction and damages with prayer
for a preliminary injunction" against the majority of the members of the
Board of Directors and San Miguel Corporation as an unwilling petitioner.
Causes of action are the following: individual respondents amended by
bylaws of the corporation, basing their authority to do so on a resolution of
the stockholders adopted on March 13, 1961; the authority granted in 1961
has already been exercised in 1962-63; petitioner averred that the
membership of the Board of Directors had changed since the authority was
given in 1961, there being six (6) new directors; Soriano et. Al purposely

Page 369 of 1072

provided Gokongweis disqualification and deprived him of his right and


finally the corporation has no inherent power to disqualify him as stockholder
from being elected as Board of Director.
ISSUE/S:
Whether or not the amended by-laws of SMC of disqualifying a
competitor from nomination or election to the Board of Directors of
SMC are valid and reasonable.
RULING:
The validity or reasonableness of a by-law of a corporation is purely a
question of law. Whether the by-law is in conflict with the law of the land, or
with the charter of the corporation, or is in a legal sense unreasonable and
therefore unlawful is a question of law. This rule is subject, however, to the
limitation that where the reasonableness of a by-law is a mere matter of
judgment, and one upon which reasonable minds must necessarily differ, a
court would not be warranted in substituting its judgment instead of the
judgment of those who are authorized to make by-laws and who have
exercised their authority.
The exclusion of a competitor from the Board is legitimate corporate
purpose, considering that being a competitor, petitioner cannot devote an
unselfish and undivided Loyalty to the corporation; that it is essentially a
preventive measure to assure stockholders of San Miguel Corporation of
reasonable protective from the unrestrained self-interest of those charged
with the promotion of the corporate enterprise; that access to confidential
information by a competitor may result either in the promotion of the interest
of the competitor at the expense of the San Miguel Corporation, or the
promotion of both the interests of petitioner and respondent San Miguel
Corporation, which may, therefore, result in a combination or agreement in
violation of Article 186 of the Revised Penal Code by destroying free
competition to the detriment of the consuming public. It is further argued
that there is not vested right of any stockholder under Philippine Law to be
voted as director of a corporation. It is alleged that petitioner, as of May 6,
1978, has exercised, personally or thru two corporations owned or controlled
by him, control over the following shareholdings in San Miguel Corporation.
Private respondents contend that the disputed amended by laws were
adopted by the Board of Directors of San Miguel Corporation a-, a measure of
self-defense to protect the corporation from the clear and present danger
that the election of a business competitor to the Board may cause upon the
corporation and the other stockholders inseparable prejudice.
In this
jurisdiction, under section 21 of the Corporation Law, a corporation may
prescribe in its by-laws "the qualifications, duties and compensation of
directors, officers and employees ... " This must necessarily refer to a
qualification in addition to that specified by section 30 of the Corporation
Law, which provides that "every director must own in his right at least one
share of the capital stock of the stock corporation of which he is a director.

Page 370 of 1072

Thus, it has been held that an officer of a corporation cannot engage in


a business in direct competition with that of the corporation where he is a
director by utilizing information he has received as such officer, under "the
established law that a director or officer of a corporation may not enter into a
competing enterprise which cripples or injures the business of the
corporation of which he is an officer or director. It is also well established
that corporate officers "are not permitted to use their position of trust and
confidence to further their private interests." The doctrine of "corporate
opportunity" is precisely a recognition by the courts that the fiduciary
standards could not be upheld where the fiduciary was acting for two entities
with competing interests. This doctrine rests fundamentally on the
unfairness, in particular circumstances, of an officer or director taking
advantage of an opportunity for his own personal profit when the interest of
the corporation justly calls for protection . It is not denied that a member of
the Board of Directors of the San Miguel Corporation has access to sensitive
and highly confidential information, such as:
a) marketing strategies and pricing structure;
b) budget for expansion and diversification;
c) research and development; and
d) sources of funding, availability of personnel,
proposals of mergers or tie-ups with other firms.
It is obviously to prevent the creation of an opportunity for an officer or
director of San Miguel Corporation, who is also the officer or owner of a
competing corporation, from taking advantage of the information which he
acquires as director to promote his individual or corporate interests to the
prejudice of San Miguel Corporation and its stockholders, that the questioned
amendment of the by-laws was made. Certainly, where two corporations are
competitive in a substantial sense, it would seem improbable, if not
impossible, for the director, if he were to discharge effectively his duty, to
satisfy his loyalty to both corporations and place the performance of his
corporation duties above his personal concerns.
Indeed, access by a competitor to confidential information regarding
marketing strategies and pricing policies of San Miguel Corporation would
subject the latter to a competitive disadvantage and unjustly enrich the
competitor, for advance knowledge by the competitor of the strategies for
the development of existing or new markets of existing or new products
could enable said competitor to utilize such knowledge to his advantage.

Page 371 of 1072

TOPIC: DISLOYALTY
ELEANOR ERICA STRONG AND RICHARD P. STRONG, plaintiffsappellees,
vs.
FRANCISCO GUTIERREZ REPIDE, defendant-appellant.
November15, 1906 G.R. No. L-2101
41 Phil 947
FACTS:
This action was brought to recover 800 shares of the capital stock of
the Philippine Sugar Estates Development Company, Limited, an anonymous
society formed to hold the Dominican friar lands.
The shares were the property of one of the plaintiffs, Mrs. Strong, as
part of the estate of her first husband. They were purchased by the
defendant through a broker who dealt with her agent, one Jones, who had
the script in her possession and who had made the sale without the
knowledge of the plaintiff. The defendant was a director, was the managing
agent, and was in his own right the majority stockholder of the society.
The government of United States wants to secure title of the friar
lands. Thus the Governor made an offer for purchase for the total sum of S6,
043, 219.47 in gold for the friar lands though owned by different owners.
However ,before the final offer has been made by the Governor, the
defendant although still holding out for a higher price for the lands, took
steps to purchase the 800 shares of stock in his company owned by Mrs.
Strong, which he knew were in possession of Jones as her agent.
Defendant employed one Kauffman and the latter employed a certain
Mr. Sloan, a broker to purchase the stock for him and told Sloan that the
stock was for a member of his wifes family. Mr. Sloan communicated with Mr.
Strong but he was referred to Mr. Jones.
Mr. Jones sold the 800 shares of stock for S16, 000.00 Mexican currencies
delivering to Kauffman at S18, 000.00, S1, 800 for Kauffmans services.
ISSUE/S:
Whether or not it is the duty of the defendant to disclose the facts
bearing upon or which might affect the value of the stock.
RULING:
A Director must always be loyal and act in good faith not only for the
corporation but for the stock holder as well. Directors of a corporation are
but the agents and trustees of the company; they have power only to act for
the interest of the company and not against it. Any agreement to influence

Page 372 of 1072

their action for the benefit of others and to the prejudice of the company is
fraudulent
and
void.
It is one of the fundamental principles of law that a person acting as an
agent or in a fiduciary relation cannot act for himself and at the same time
as agent for another whose interests are conflicting. Thus a person cannot be
a purchaser of property and at the same time agent of the vendor. The law
will always condemn the transactions of such a party in his own behalf or in
respect to matters concerning him as agent of others and will give relief
against such acts whenever their enforcement is seasonably resisted.
Directors and general managers of corporation and all other persons who
sustain any fiduciary relation to other parties and are clothed with power to
act for them are subject to this rule.
Concealing his identity when procuring the purchase of the stock by his
agent, was in itself a strong evidence of fraud on the part of the defendant.
The concealment was not a mere inadvertent omission, an omission without
any fraudulent or deceitful interest but was studied and intentional omission,
to be characterized as part of the deceitful machination to obtain the
purchase without giving any information whatever as to the state and
probable result of the negotiations, to the vendor of the stocks , in that way
obtained the same at a lower price.
They are not liable for errors of judgment and mistakes of fact or law
when they act in good faith and with proper carte. They are liable, however,
when by their secret connivance or by fraudulent conduct they have made
great
profits
at
the
expense
of
the
stockholders.

Page 373 of 1072

TOPIC: WATERED STOCK


LIRAG TEXTILE MILLS, INC., and BASILIO L. LIRAG, Petitioners,
vs.
SOCIAL SECURITY SYSTEM, and HON. PACIFICO DE CASTRO,
Respondents
G.R. No. L-33205 August 31, 1987
FACTS:
The plaintiff [herein respondent Social Security System] and the
defendants [herein petitioners] Lirag Textile Mills, Inc. and Basilio Lirag
entered into a Purchase Agreement under which the plaintiff agreed to
purchase from the said defendant preferred shares of stock worth ONE
MILLION PESOS P1,000,000.00 subject to the conditions set forth in such
agreement.
The plaintiff, on January 31, 1962, paid the defendant Lirag Textile
Mills, Inc. the sum of FIVE HUNDRED THOUSAND PESOS P500, 000.00 for
which the said defendant issued to plaintiff 5,000 preferred shares with a par
value of one hundred pesos P10000 per share as evidenced by stock
Certificate No. 128, and further made another payment of P500,000.00 for
which the said defendant issued to plaintiff 5,000 preferred shares with a par
value of one hundred pesos P100.00 per share as evidenced by Stock
Certificate No. 139.
Purchase Agreement provides for the repurchase by the Lirag Textile
Mills, Inc. of the shares of stock at regular intervals of one year beginning
with the 4th year following the date of issue, Stock Certificates Nos. 128 and
139 were to be repurchased by the Lirag Textile Mills, Inc. thus, to guarantee
the redemption of the stocks purchased by the plaintiff, the payment of
dividends, as well as the other obligations of the Lirag Textile Mills, Inc.,
defendants Basilio L. Lirag signed the Purchase Agreement of September 4,
1961 not only as president of the defendant corporation, but also as surety
so that should the Lirag Textile Mills, Inc. fail to perform any of its obligations
in the said Purchase Agreement, the surety shall immediately pay to the
vendee the amounts then outstanding.
Defendant corporation failed to redeem certificates of Stock Nos. 128
and 139 by payment of the amounts and Lirag Textile Mills, Inc. has not paid
dividends despite notices to defendant and Basilio as surety. Defendant
claims that it was not able to pay its obligation due to financial reverses.
It has been the policy of the plaintiff to be represented in the board of
directors of the corporation or entity which has obtained financial assistance
from the System be it in terms of loans, mortgages or equity investments,

Page 374 of 1072

thus Messrs. Rene Espina, Bernardino Abes and Heber Catalan were each
issued one common share of stock as a qualifying share to their election to
the Board of Directors of the Lirag Textiles Mills, Inc. However, the per diems
received by the SSS representative do not go to the coffers of the System but
personally to the representative in the said board of directors.
Lirag Textile Mills, Inc. and Basilio L. Lirag denied the existence of any
obligation on their part to redeem the preferred stocks, on the ground that
the SSS became and still is a preferred stockholder of the corporation so that
redemption of the shares purchased depended upon the financial ability of
said corporation. Insofar as defendant Basilio Lirag is concerned, it was
alleged that his liability arises only if the corporation is liable and does not
perform its obligations under the Purchase Agreement. They further
contended that no liability on their part has arisen because of the financial
condition of the corporation upon which such liability was made to depend,
particularly the non-realization of any profit or earned surplus. Thus, the
other claims for dividends, liquidated damages and exemplary damages are
allegedly without basis.
ISSUE/S:
Whether or not respondent SSS is a stock holder of Petitioner
Corporation.
RULING:
The Purchase Agreement is, indeed, a debt instrument. Its terms and
conditions unmistakably show that the parties intended the repurchase of
the preferred shares on the respective scheduled dates to be an absolute
obligation which does not depend upon the financial ability of petitioner
corporation. This absolute obligation on the part of petitioner corporation is
made manifest by the fact that a surety was required to see to it that the
obligation is fulfilled in the event of the principal debtor's inability to do so.
The unconditional undertaking of petitioner corporation to redeem the
preferred shares at the specified dates constitutes a debt which is defined
"as an obligation to pay money at some fixed future time, or at a time which
becomes definite and fixed by acts of either party and which they expressly
or impliedly, agree to perform in the contract.
The rights given by the Purchase Agreement to respondent SSS are
rights not enjoyed by ordinary stockholders. The aforementioned rights
specially stipulated for the benefit of the plaintiff [respondent SSS] suggest
eloquently an intention on the part of the plaintiff [respondent SSS] to
facilitate a loan to the defendant corporation upon the latter's request. In
order to afford protection to the plaintiff which otherwise is provided by
means of collaterals, as the plaintiff exacts in its grants of loans in its
ordinary transactions of this kind, as it is looked upon more as a lending
institution rather than as an investing agency, the purchase agreement

Page 375 of 1072

supplied these protective rights which would otherwise be furnished by


collaterals to the loan. Thus, the membership in the board is to have a
watchdog in the operation of the business of the corporation, so as to insure
against mismanagement which may result in losses not entirely unavoidable
since payment for purposes of redemption as well as the dividends is
expressly stipulated to come from profits and/or surplus. Such a right is
never exacted by an ordinary stockholder merely investing in the
corporation.
Moreover, the Purchase Agreement provided that failure on the part of
petitioner to repurchase the preferred shares on the scheduled due dates
renders the entire obligation due and demandable, with petitioner in such
eventuality liable to pay 12% of the then outstanding obligation as liquidated
damages. These features of the Purchase Agreement, taken collectively,
clearly show the intent of the parties to be bound therein as debtor and
creditor, and not as corporation and stockholder.

Page 376 of 1072

TOPIC: WATERED STOCK


RICARDO A. NAVA, petitioner-appellant.
vs.
PEERS MARKETING CORPORATION, RENATO R. CUSI and AMPARO
CUSI, respondents-appellees.
G.R. No. L-28120 November 25, 1976
FACTS:
Teofilo Po as an incorporator subscribed to eighty shares of Peers
Marketing Corporation at one hundred pesos a share or a total par value of
eight thousand pesos. Po paid two thousand pesos or twenty-five percent of
the amount of his subscription. No certificate of stock was issued to him or,
for that matter, to any incorporator, subscriber or stockholder.
On April 2, 1966 Po sold to Ricardo A. Nava for two thousand pesos
twenty of his eighty shares. In the deed of sale Po represented that he was
"the absolute and registered owner of twenty shares" of Peers Marketing
Corporation.
Nava requested the officers of the corporation to register the sale in
the books of the corporation. The request was denied because Po has not
paid fully the amount of his subscription. Nava was informed that Po was
delinquent in the payment of the balance due on his subscription and that
the corporation had a claim on his entire subscription of eighty shares which
included the twenty shares that had been sold to Nava.
Nava filed this mandamus action in the Court of First Instance of
Negros Occidental, Bacolod City Branch to compel the corporation and
Renato R. Cusi and Amparo Cusi, its executive vice-president and secretary,
respectively, to register the said twenty shares in Nava's name in the
corporation's transfer book.
ISSUE/S:
Whether or not the Corporation and its officers can be compelled to
register the alleged purchased shares of stock by Nava.

Page 377 of 1072

RULING:
NO
SEC. 35 of the Corporation Code provides that: the capital stock of
stock corporations shall be divided into shares for which certificates signed
by the president or the vice-president, countersigned by the secretary or
clerk and sealed with the seal of the corporation, shall be issued in
accordance with the by-laws. Shares of stock so issued are personal property
and may be transferred by delivery of the certificate indorsed by the owner
or his attorney in fact or other person legally authorized to make the transfer.
No transfer, however, shall be valid, except as between the, parties, until the
transfer is entered and noted upon the books of the corporation so as to
show the names of the parties to the transaction, the date of the transfer,
the number of the certificate, and the number of shares transferred.
No share of stock against which the corporation holds any unpaid claim
shall be transferable on the books of the corporation.
As prescribed in section 35, shares of stock may be transferred by delivery to
the transferee of the certificate properly indorsed. "Title may be vested in
the transferee by delivery of the certificate with a written assignment or
endorsement thereof. There should be compliance with the mode of transfer
prescribed by law.
The usual practice is for the stockholder to sign the form on the back of
the stock certificate. The certificate may thereafter be transferred from one
person to another. If the holder of the certificate desires to assume the legal
rights of a shareholder to enable him to vote at corporate elections and to
receive dividends, he fills up the blanks in the form by inserting his own
name as transferee. Then he delivers the certificate to the secretary of the
corporation so that the transfer may be entered in the corporation's books.
The certificate is then surrendered and a new one issued to the transferee.
(Hager vs. Bryan, 19 Phil. 138, 143-4).
That procedure cannot be followed in the instant case because, as
already noted, the twenty shares in question are not covered by any
certificate of stock in Po's name. Moreover, the corporation has a claim on
the said shares for the unpaid balance of Po's subscription. A stock
subscription is a subsisting liability from the time the subscription is made.
The subscriber is as much bound to pay his subscription as he would be to
pay any other debt. The right of the corporation to demand payment is no
less incontestable. (Velasco vs. Poizat, 37 Phil. 802; Lumanlan vs. Cura, 59
Phil. 746).
A corporation cannot release an original subscriber from paying for his
shares without a valuable consideration (Philippine National Bank vs. Bitulok
Sawmill,
Inc.,
L-24177-85, June 29, 1968, 23 SCRA 1366) or without the unanimous consent

Page 378 of 1072

of the stockholders (Lingayen Gulf Electric Power Co., Inc. vs. Baltazar, 93
Phil 404).
Under the facts of this case, there is no clear legal duty on the part of the
officers of the corporation to register the twenty shares in Nava's name;
hence, there is no cause of action for mandamus.

TOPIC: SELF-DEALING DIRECTOR/OFFICER


CHARLES W. MEAD, plaintiff-appellant,
vs.
E. C. McCullough, ET AL., and THE PHILIPPINE ENGINEERING AND
CONSTRUCTION COMPANY, defendant-appellants.
G.R. No. 6217 December 26, 1911
21 Phil 95
FACTS:
On March 15, 1902, Mead and McCullough organized the "Philippine
Engineering and Construction Company," the incorporators being the only
stockholders and also the directors of said company, with general ordinary
powers. One of the contracts and work undertaken by the company during
the management of Mead were the wrecking contract with the Navy
Department at Cavite for the raising of the Spanish ships sunk by Admiral
Dewey.
Shortly after the Mead left the Philippine Islands for China, the other
directors, the defendants in this case, held a meeting on December 24, 1903,
for the purpose of discussing the condition of the company at that time and
determining what course to pursue and at the same date, entered into a
contract with the defendant McCullough selling the wrecking contract with
the naval authorities to the latter.
On the 28th of the same month, McCullough in turn transferred his
right, title and interest over the wrecking contract by virtue of a contract to
R. W. Brown, H. D. C. Jones, John T. Macleod, and T. H. Twentyman. The
assignees of the wrecking contract, including McCullough, formed was now
known as the "Manila Salvage Association." This association paid to
McCullough $15,000 Mexican Currency cash for the assignment of said
contract. In addition to this payment, McCullough retained a one-sixth
interest in the new company or association.
ISSUE:

Page 379 of 1072

Whether or not an officer or directors of the corporation can


purchase the corporate property.
RULING:
While a corporation remains solvent, we can see no reason why a
director or officer, by the authority of a majority of the stockholders or board
of managers, may not deal with the corporation, loan it money or buy
property from it, in like manner as a stranger. So long as a purely private
corporation remains solvent, its directors are agents or trustees for the
stockholders. They owe no duties or obligations to others. But the moment
such a corporation becomes insolvent, its directors are trustees of all the
creditors, whether they are members of the corporation or not, and must
manage its property and assets with strict regard to their interest; and if they
are themselves creditors while the insolvent corporation is under their
management, they will not be permitted to secure to themselves by
purchasing the corporate property or otherwise any personal advantage over
the other creditors. Nevertheless, a director or officer may in good faith and
for an adequate consideration purchase from a majority of the directors or
stockholders the property even of an insolvent corporation, and a sale thus
made to him is valid and binding upon the minority. (Beach et al. vs. Miller,
supra; Twin-Lick Oil Company vs. Marbury, supra; Drury vs. Cross, 7 Wall.,
299; Curran vs. State of Arkansas, 15 How., 304; Richards vs. New
Hamphshire Insurance Company, 43 N. H., 263; Morawetz on Corporations
(first edition), sec. 579; Haywood vs. Lincoln Lumber Company et al., 64 Wis.,
639; Port vs. Russels, 36 Ind., 60; Lippincott vs. Shaw Carriage Company, 21
Fed. Rep., 577.)
The sale or transfer of the corporate property in the case at bar was
made by three directors who were at the same time a majority of
stockholders. If a majority of the stockholders have a clear and a better right
to sell the corporate property than a majority of the directors, then it can be
said that a majority of the stockholders made this sale or transfer to the
defendant McCullough.
But as we have said when the sale or transfer under consideration took
place, there were three directors present, and all voted in favor of making
this sale. It was not necessary for the president, McCullough, to vote. There
was a quorum without him: a quorum of the directors, and at the same time
a majority of the stockholders.
A corporation is essential a partnership, except in form. "The directors
are the trustees or managing partners, and the stockholders are the cestui
que trust and have a joint interest in all the property and effects of the
corporation." (Per Walworth, Ch., in Robinson vs. Smith, 3 Paige, 222, 232; 5
idem, 607; Slee vs. Bloom, 19 Johns., 479; Hoyt vs. Thompson, 1 Seld., 320.)

Page 380 of 1072

The Philippine Engineering and Construction Company was an artificial


person, owning its property and necessarily acting by its agents; and these
agents were the directors. McCullough was then an agent or a trustee, and
the stockholders the principal. Or say (as corporation was insolvent) that he
was an agent or trustee and the creditors were the beneficiaries. This being
the true relation, then the rules of the law (art. 1713 of the Civil Code)
applicable to sales and purchases by agents and trustees would not apply to
the purchase in question for the reason that there was a quorum without
McCullough, and for the further reason that an officer or director of a
corporation, being an agent of an artificial person and having a joint interest
in the corporate property, is not such an agent as that treated of in article
1713 of the Civil Code.

Page 381 of 1072

TOPIC: SELF-DEALING DIRECTOR/OFFICER


PRIME WHITE CEMENT CORPORATION
VS.
HONORABLE INTERMEDIATE APPELLATE COURT AND ALEJANDRO TE
G.R. NO. 68555, MARCH 19, 1993
220 SCRA 103
FACTS:
Prime White Cement Corporation, herein plaintiff, entered into a
dealership agreement with herein private respondent Alejandro Te, who is
likewise a member of the Board of Directors and Auditor of the said
corporation, whereby the latter shall act as an exclusive dealer and/or
distributor of Prime Whites cement products in the entire Mindanao area for
a period of five years at a fixed price of P9.70 per bag of cement.
Thus, relying heavily on the dealership agreement, Te entered into written
agreements between several hardware stores to whom he would supply
Prime Whites cement. Te even wrote the corporation to inform the latter that
he is already making the necessary preparation for the opening of the
requisite letter of credit to cover the price of the initial delivery due for the
month of September. However, the corporation replied through its corporate
secretary that the Board of Prime White decided to impose certain conditions
which effectively modified the dealership agreement that they have entered
into. Said conditions effectively reduced the number of cement bags to be
delivered, shortening the period of the exclusive dealership agreement, as
well as fixing the price of white cement at P13.30 per bag.
Te demanded for the corporation to comply with the dealership
agreement. However, said corporation refused, forcing him to cancel his
agreement to supply white cement with third parties. Thus, Te filed an action
for damages against the corporation which the trial court ruled in favour of
Te. Said decision was affirmed by the IAC.
ISSUE/S:
Whether or not the dealership agreement entered into by a
self-dealing director and a corporation is valid and enforceable.
RULING:
Inthe instant case, respondent Te was not an ordinary stockholder; he
was a member of the Board of Directors and Auditor of the corporation as
well. He was what is often referred to as a self-dealing director.
Section 32 of the Corporation Code provides, thus:
Sec. 32. Dealings, of directors, trustees or officers with the
corporation. A contract of the corporation with one or more of its
directors or trustees or officers is voidable, at the option of such
corporation, unless all the following conditions are present:

Page 382 of 1072

1. That the presence of such or trustee in the board meeting in


which the contract was approved was not necessary to
constitute a quorum for such meeting;
2. That the vote of such director or trustee was not necessary for
the approval of the contract;
3. That the contract is fair and reasonable under the
circumstances; and
4. That in the case of an officer, the contract with the officer has
been previously authorized by the Board of Directors.
Where any of the first two conditions set forth in the preceding
paragraph is absent, in the case of a contract with a director or trustee, such
contract may be ratified by the vote of the stockholders at least two-thirds
(2/3) of the outstanding capital stock of two-thirds (2/3) of the members in a
meeting called for the purpose: Provided, That full disclosure of the adverse
interest of the directors or trustees involved is made at such meeting:
Provided, however, That the contract is fair and reasonable under the
circumstances.
In the case at bar, the Supreme Court held that the contract was
neither fair nor reasonable. The dealership agreement entered into in July
1969 was to sell and supply to respondent Te 20,000 bags of white cement
per month, for five years starting September 1970, at the fixed price of P9.70
per bag. Respondent Te is a businessman himself and must have known, or
at least must be presumed to know, that at that time, prices of commodities
in general, and white cement in particular, were not stable and were
expected to rise. Despite this, no provision was made in the dealership
agreement to allow for an increase in price mutually acceptable to the
parties.
In view of the foregoing, the Decision and Resolution of the
Intermediate Appellate Court dated March 30, 1984 and August 6, 1984,
respectively, are hereby SET ASIDE. Private respondent Alejandro Te is
hereby ordered to pay petitioner corporation the sum of P20,000.00 for
attorneys fees, plus the cost of suit and expenses of litigation.

Page 383 of 1072

TOPIC: CONTRACTS
DIRECTORS

BETWEEN

CORPORATIONS

WITH

INTERLOCKING

PEDRO PALTING
VS.
SAN JOSE PETROLEUM INCORPORATED
G.R. NO. L-14441, DECEMBER 17, 1966
18 SCRA 924
FACTS:
On September 7, 1956, SAN JOSE PETROLEUM filed with the Philippine
Securities and Exchange Commission a sworn registration statement, for the
registration and licensing for sale in the Philippines Voting Trust Certificates
representing 2,000,000 shares of its capital stock of a par value of $0.35 a
share, at P1.00 per share. It was alleged that the entire proceeds of the sale
of said securities will be devoted or used exclusively to finance the
operations of San Jose Oil Company, Inc. (a domestic mining corporation
hereafter to be referred to as SAN JOSE OIL) which has 14 petroleum
exploration concessions covering an area of a little less than 1,000,000
hectares, located in the provinces of Pangasinan, Tarlac, Nueva Ecija, La
Union, Iloilo, Cotabato, Davao and Agusan.
Respondent SAN JOSE PETROLEUM, whose shares of stock were
allowed registration for sale in the Philippines, was incorporated under the
laws of Panama in April, 1956 with an authorized capital stock of
$500,000.00, American currency, divided into 50,000,000 shares at par value
of $0.01 per share.
Its Articles of Incorporation include, among others, a provision which
states that no contract or transaction between the corporation and any
other association or partnership will be affected, except in case of fraud, by
the fact that any of the directors or officers of the corporation is interested
in, or is a director or officer of, such other association or partnership, and
that no such contract or transaction of the corporation with any other person
or persons, firm, association or partnership shall be affected by the fact that
any director or officer of the corporation is a party to or has an interest in,
such contract or transaction, or has in anyway connected with such other
person or persons, firm, association or partnership; and finally, that all and
any of the persons who may become director or officer of the corporation
shall be relieved from all responsibility for which they may otherwise be
liable by reason of any contract entered into with the corporation, whether it
be for his benefit or for the benefit of any other person, firm, association or
partnership in which he may be interested.
ISSUE/S:

Page 384 of 1072

Whether or not the aforementioned provision in the Articles of


Incorporation of San Jose Petroleum is valid under Philippine
laws.
RULING:
These provisions are in direct opposition to our corporation law and
corporate practices in this country. These provisions alone would outlaw any
corporation locally organized or doing business in this jurisdiction. Consider
the unique and unusual provision that no contract or transaction between
the company and any other association or corporation shall be affected
except in case of fraud, by the fact that any of the directors or officers of the
company may be interested in or are directors or officers of such other
association or corporation; and that none of such contracts or transactions of
this company with any person or persons, firms, associations or corporations
shall be affected by the fact that any director or officer of this company is a
party to or has an interest in such contract or transaction or has any
connection with such person or persons, firms associations or corporations;
and that any and all persons who may become directors or officers of this
company are hereby relieved of all responsibility which they would otherwise
incur by reason of any contract entered into which this company either for
their own benefit, or for the benefit of any person, firm, association or
corporation in which they may be interested.
The impact of these provisions upon the traditional judiciary
relationship between the directors and the stockholders of a corporation is
too obvious to escape notice by those who are called upon to protect the
interest of investors. The directors and officers of the company can do
anything, short of actual fraud, with the affairs of the corporation even to
benefit themselves directly or other persons or entities in which they are
interested, and with immunity because of the advance condonation or relief
from responsibility by reason of such acts. This and the other provision which
authorizes the election of non-stockholders as directors, completely
disassociate the stockholders from the government and management of the
business in which they have invested.

Page 385 of 1072

TOPIC: CONTRACTS
DIRECTORS

BETWEEN

CORPORATIONS

WITH

INTERLOCKING

DEVELOPMENT BANK OF THE PHILIPPINES


VS.
HONORABLE COURT OF APPEALS and REMINGTON INDUSTRIAL
SALES CORPORATION
G.R. No. 126200, August 16, 2001
363 SCRA 307
FACTS:
Marinduque Mining-Industrial Corporation, obtained from the Philippine
National Bank
various loan accommodations. To secure the loans,
Marinduque Mining executed on October 9, 1978 a Deed of Real Estate
Mortgage and Chattel Mortgage in favor of PNB. The mortgage covered all of
Marinduque Mining's real properties, located at Surigao del Norte, Sipalay,
Negros Occidental, and at Antipolo, Rizal, including the improvements
thereon. As of November 20, 1980, the loans extended by PNB amounted to
P4 Billion, exclusive of interest and charges.
For failure of Marinduque Mining to settle its loan obligations, PNB and
DBP instituted sometime on July and August 1984 extrajudicial foreclosure
proceedings over the mortgaged properties.
In the ensuing public auction sales PNB and DBP emerged and were
declared the highest bidders over the foreclosed real properties, buildings,
mining claims, leasehold rights together with the improvements thereon as
well as machineries and equipments of MMIC.
PNB and DBP thereafter thru several Deed of Transfers assigned and
transferred to Nonoc Mining and Industrial Corporation all their rights,
interest and participation over the foreclosed properties of MMIC located at
Nonoc Island, Surigao del Norte for an initial consideration of
P14,361,000,000.00, in favor of Maricalum Mining Corp. all its rights, interest
and participation over the foreclosed properties of MMIC at Sipalay, Negros
Occidental for an initial consideration of P325,800,000.00 and to the National
Government through the Asset Privatization Trust all its existing rights and
interest over the assets of MMIC, earlier assigned to Nonoc Mining and
Industrial Corporation, Maricalum Mining Corporation and Island Cement
Corporation.
However, between July 16, 1982 to October 4, 1983, Marinduque
Mining purchased and caused to be delivered construction materials and
other merchandise from Remington Industrial Sales Corporation worth
P921,755.95. The purchases remained unpaid as of August 1, 1984 when
Remington filed a complaint for a sum of money and damages against

Page 386 of 1072

Marinduque Mining for the value of the unpaid construction materials and
other merchandise purchased by Marinduque Mining, as well as interest,
attorney's fees and the costs of suit. The complaint was later amended to
include PNB and DBP, Nonoc Mining, Maricalum Mining and Island Cement
based on the claim of Remington that these mining corporations must be
treated in law as one and the same entity by disregarding the veil of
corporate fiction since:
1. Co-defendants NMIC, Maricalum and Island Cement which
are newly created entities are practically owned wholly by
defendants PNB and DBP, and managed by their officers,
aside from the fact that the aforesaid co-defendants NMIC,
Maricalum and Island Cement were organized in such a
hurry and in such suspicious circumstances by codefendants PNB and DBP after the supposed extrajudicial
foreclosure of MMIC's assets as to make their supposed
projects assets, machineries and equipment which were
originally owned by co-defendant MMIC beyond the reach
of creditors of the latter.
2. The personnel, key officers and rank-and-file workers and
employees of co-defendants NMIC, Maricalum and Island
Cement creations of co-defendants PNB and DBP were the
personnel of co-defendant MMIC such that . . . practically
there has only been a change of name for all legal purpose
and intents
The RTC and CA ruled in favour of Remington.
ISSUE:
Whether or not the presence of interlocking directors warrant the
piercing of the veil of corporate fiction.
RULING:
The Court of Appeals made reference to two principles in corporation
law. The first pertains to transactions between corporations with interlocking
directors resulting in the prejudice to one of the corporations. This rule does
not apply in this case, however, since the corporation allegedly prejudiced
(Remington) is a third party, not one of the corporations with interlocking
directors (Marinduque Mining and DBP).
The second principle invoked by respondent court involves "directors x x x
who are creditors" which is also inapplicable herein. Here, the creditor of
Marinduque Mining is DBP, not the directors of Marinduque Mining.
Neither do we discern any bad faith on the part of DBP by its creation
of Nonoc Mining, Maricalum and Island Cement. As Remington itself
concedes, DBP is not authorized by its charter to engage in the mining
business. The creation of the three corporations was necessary to manage
and operate the assets acquired in the foreclosure sale lest they deteriorate

Page 387 of 1072

from non-use and lose their value. In the absence of any entity willing to
purchase these assets from the bank, what else would it do with these
properties in the meantime? Sound business practice required that they be
utilized for the purposes for which they were intended.
Remington also asserted in its third amended complaint that the use of
Nonoc Mining, Maricalum and Island Cement of the premises of Marinduque
Mining and the hiring of the latter's officers and personnel also constitute
badges of bad faith.
Assuming that the premises of Marinduque Mining were not among
those acquired by DBP in the foreclosure sale, convenience and practicality
dictated that the corporations so created occupy the premises where these
assets were found instead of relocating them. No doubt, many of these
assets are heavy equipment and it may have been impossible to move them.
The same reasons of convenience and practicality, not to mention efficiency,
justified the hiring by Nonoc Mining, Maricalum and Island Cement of
Marinduque Mining's personnel to manage and operate the properties and to
maintain the continuity of the mining operations.

Page 388 of 1072

TOPIC: DERIVATICE SUIT: REMEDIES TO ENFORCE PERSONAL LIABILITY


LEGASPI TOWERS 300, INC Vs.AMELIA P. MUER et.al
G.R. No. 170783 - June 18, 2012
FACTS:
Pursuant to the by-laws of Legaspi Towe, petitioners Lilia Marquinez
et.al ., The incumbent Board of Directors, set the annual meeting of the
members of the condominium corporation and the election of the new Board
of Directors.
Out of a total number of 5,723 members who were entitled to vote,
1,358 were supposed to vote through their respective proxies and their votes
were critical in determining the existence of a quorum, which was at least
2,863 (50% plus 1). The Committee on Elections of Legaspi Towers 300, Inc.,
however, found most of the proxy votes, at its face value, irregular, thus,
questionable; and for lack of time to authenticate the same, petitioners
adjourned the meeting for lack of quorum.
Despite petitioners' insistence that no quorum was obtained during the
annual meeting respondents pushed through with the scheduled election and
were elected as the new Board of Directors and officers of Legaspi Towers
Subsequently, they submitted a General Information Sheet to the Securities
and Exchange Commission (SEC) with the new set of officers.
Petitioners filed a Complaint for the Declaration of Nullity of Elections
ISSUE:
Whether or not petitioners have no right as board of directors to bring
an action in behalf of Legaspi towers.
HELD:
The complaint is meant to be a derivative suit filed by petitioners in
behalf of the corporation. The sudden takeover by private respondents of the
management of Legaspi Towers
has only proven the rightfulness of
petitioners move to include Legaspi Towers as party-plaintiff. This is because
every resolution passed by private respondents sitting as a board result[s] in
violation of Legaspi Towers right to be managed and represented by herein
petitioners.

Page 389 of 1072

Since it is the corporation that is the real party-in-interest in a


derivative suit, then the reliefs prayed for must be for the benefit or interest
of the corporation. When the reliefs prayed for do not pertain to the
corporation, then it is an improper derivative suit.
The requisites for a derivative suit are as follows:
a) The party bringing suit should be a shareholder as of the time of the act
or transaction complained of, the number of his shares not being material;
b) He has tried to exhaust intra-corporate remedies, i.e., has made a demand
on the board of directors for the appropriate relief but the latter has failed or
refused to heed his plea; and
c)
The cause of action actually devolves on the corporation, the
wrongdoing or harm having been, or being caused to the corporation and not
to the particular stockholder bringing the suit.
Petitioners complaint seek to nullify the said election, and to protect
and enforce their individual right to vote. Petitioners seek the nullification of
the election of the Board of Directors composed of respondents, who
pushed through with the election even if petitioners had adjourned the
meeting allegedly due to lack of quorum. Petitioners are the injured party,
whose rights to vote and to be voted upon were directly affected by the
election of the new set of board of directors.

Page 390 of 1072

TOPIC: DERIVATICE SUIT: REMEDIES TO ENFORCE PERSONAL LIABILITY


LISAM ENTERPRISES, INC vs. BANCO DE ORO UNIBANK, INC.
G.R. No. 143264 -April 23, 2012
FACTS:
Petitioners filed a Complaint against respondents for Annulment of
Mortgage. Petitioner Lolita Soriano alleged that she is a stockholder of
petitioner Lisam and a member of its Board of Directors, designated as its
Corporate Secretary. Plaintiff LEI, in the course of its business operation,
acquired by purchase a parcel of residential land with improvement.
Spouses Soriano in their personal capacity and for their own use and
benefit obtained a loan as Banco de Oro Unibank, Inc. in the total amount of
P20 Million. That as security for the payment of the credit accommodation
spouses Soriano without authority and consent of the board and with the use
of a falsified board resolution, executed a real estate mortgage on property
of plaintiff LEI in favor of Banco de oro. The Spouses Soriano, with intent to
defraud and prejudice plaintiff LEI and its stockholders, falsified the
signatures of Lolita A. Soriano
ISSUE:
Whether or not the derivative suit is proper.
Held:
Yes, With the amendment stating that plaintiff Lolita A. Soriano
likewise made demands upon the Board of Directors of Lisam Enterprises,
Inc., to make legal steps to protect the interest of the corporation from said
fraudulent transaction, but unfortunately, until now, no such legal step was
ever taken by the Board, hence, this action for the benefit and in behalf of
the corporation. In Hi-Yield Realty, Incorporated v. Court of Appeals the Court
enumerated the requisites for filing a derivative suit, as follows:

a) The party bringing the suit should be a shareholder as of the time of


the act or transaction complained of, the number of his shares not being
material;

Page 391 of 1072

b) He has tried to exhaust intra-corporate remedies, i.e., has made a


demand on the board of directors for the appropriate relief but the latter has
failed or refused to heed his plea; and
c) The cause of action actually devolves on the corporation, the
wrongdoing or harm having been, or being caused to the corporation and not
to the particular stockholder bringing the suit.[7]
the amended complaint will reveal that all the foregoing requisites had
been alleged therein. Hence, the amended complaint remedied the defect in
the original complaint and now sufficiently states a cause of action.

TOPIC: DERIVATICE SUIT: REMEDIES TO ENFORCE PERSONAL LIABILITY

Page 392 of 1072

STRATEGIC ALLIANCE DEVELOPMENT CORPORATION, Petitioner,


vs.
RADSTOCK SECURITIES LIMITED and PHILIPPINE NATIONAL
CONSTRUCTION CORPORATION, Respondents.
ASIAVEST MERCHANT BANKERS BERHAD, Intervenor.
G.R. No. 178158 December 4, 2009
FACTS:
PNCC was incorporated in 1966 for a term of fifty years under the
Corporation Code with the name Construction Development Corporation of
the Philippines (CDCP). PD 1113, issued on 31 March 1977, granted CDCP a
30-year franchise to construct, operate and maintain toll facilities in the
North and South Luzon Tollways. PD 1894, issued on 22 December 1983,
amended PD 1113 to include in CDCPs franchise the Metro Manila
Expressway, which would "serve as an additional artery in the transportation
of trade and commerce in the Metro Manila area."
Sometime between 1978 and 1981, Basay Mining Corporation (Basay
Mining), an affiliate of CDCP, obtained loans from Marubeni Corporation of
Japan (Marubeni) amounting to 5,460,000,000 yen and US$5 million. A CDCP
official issued letters of guarantee for the loans, committing CDCP to pay
solidarily for the full amount of the 5,460,000,000 yen loan and to the extent
of P20 million for the US$5 million loan. However, there was no CDCP Board
Resolution authorizing the issuance of the letters of guarantee. Later, Basay
Mining changed its name to CDCP Mining Corporation (CDCP Mining). CDCP
Mining secured the Marubeni loans when CDCP and CDCP Mining were still
privately owned and managed.
Subsequently in 1983, CDCP changed its corporate name to PNCC to
reflect the extent of the Government's equity investment in the company,
which arose when government financial institutions converted their loans to
PNCC into equity following PNCCs inability to pay the loans. Various
government financial institutions held a total of seventy-seven point fortyeight percent (77.48%) of PNCCs voting equity, most of which were later
transferred to the Asset Privatization Trust (APT) under Administrative Orders
No. 14 and 64, series of 1987 and 1988, respectively. Also, the Presidential
Commission on Good Government holds some 13.82% of PNCCs voting
equity under a writ of sequestration and through the voluntary surrender of
certain PNCC shares. In fine, the Government owns 90.3% of the equity of
PNCC and only 9.70% of PNCCs voting equity is under private ownership.
Meanwhile, the Marubeni loans to CDCP Mining remained unpaid. On
20 October 2000, during the short-lived Estrada Administration, the PNCC
Board of Directors (PNCC Board) passed Board Resolution No. BD-092-2000

Page 393 of 1072

admitting PNCCs liability to Marubeni for P10,743,103,388 as of 30


September 1999.
This was the first PNCC Board Resolution admitting PNCCs liability for
the Marubeni loans. Previously, for two decades the PNCC Board consistently
refused to admit any liability for the Marubeni loans.
On 22 November 2000, the PNCC Board passed Board Resolution No.
BD-099-2000 amending Board Resolution No. BD-092-2000. PNCC Board
Resolution No.
In January 2001, barely three months after the PNCC Board first
admitted liability for the Marubeni loans, Marubeni assigned its entire credit
to Radstock for US$2 million or less than P100 million. In short, Radstock
paid Marubeni less than 10% of the P10.743 billion admitted amount.
Radstock immediately sent a notice and demand letter to PNCC.
On 15 January 2001, Radstock filed an action for collection and
damages against PNCC before the Regional Trial Court. In its order of 23
January 2001, the trial court issued a writ of preliminary attachment against
PNCC. The trial court ordered PNCCs bank accounts garnished and several of
its real properties attached. PNCC filed a petition for certiorari before the
Court of Appeals, assailing the denial of the motion to dismiss. On 30 August
2002, the Court of Appeals denied PNCCs petition. PNCC filed a motion for
reconsideration, which the Court of Appeals also denied. PNCC filed a petition
for review before this Court, docketed as G.R. No. 156887.
Meanwhile, on 19 June 2001, at the start of the Arroyo Administration,
the PNCC Board, under a new President and Chairman, revoked Board
Resolution No. BD-099-2000. The trial court rendered a decicion in favor of
the plaintiff and the defendant was directed to pay the total amount of
Thirteen Billion One Hundred Fifty One Million Nine Hundred Fifty Six
thousand Five Hundred Twenty Eight Pesos (P13,151,956,528.00) with
interest from October 15, 2001 plus Ten Million Pesos (P10,000,000.00) as
attorneys fees. PNCC appealed the trial courts decision to the Court of
Appeals.
ISSUE/S:
When to intervene?
Who may intervene?
RULING:
The Court of Appeals denied STRADECs motion for intervention on the
ground that the motion was filed only after the Court of Appeals and the trial
court had promulgated their respective decisions.
Section 2, Rule 19 of the 1997 Rules of Civil Procedure provides:

Page 394 of 1072

SECTION 2. Time to intervene. The motion to intervene may be


filed at any time before rendition of judgment by the trial court. A
copy of the pleading-in-intervention shall be attached to the
motion and served on the original parties.
The rule is not absolute. The rule on intervention, like all other rules of
procedure, is intended to make the powers of the Court completely available
for justice. It is aimed to facilitate a comprehensive adjudication of rival
claims, overriding technicalities on the timeliness of the filing of the claims.
Concededly, STRADEC has no legal interest in the subject matter of the
Compromise Agreement. Section 1, Rule 19 of the 1997 Rules of Civil
Procedure states:
SECTION 1. Who may intervene. - A person who has a legal
interest in the matter in litigation, or in the success of either of
the parties, or an interest against both, or is so situated as to be
adversely affected by a distribution or other disposition of
property in the custody of the court or of an officer thereof may,
with leave of court, be allowed to intervene in the action. The
Court shall consider whether or not the intervention will unduly
delay or prejudice the adjudication of the rights of the original
parties, and whether or not the intervenors rights may be fully
protected in a separate proceeding.
A derivative action is a suit by a stockholder to enforce a corporate
cause of action. Under the Corporation Code, where a corporation is an
injured party, its power to sue is lodged with its board of directors or
trustees. However, an individual stockholder may file a derivative suit on
behalf of the corporation to protect or vindicate corporate rights whenever
the officials of the corporation refuse to sue, or are the ones to be sued, or
hold control of the corporation. In such actions, the corporation is the real
party-in-interest while the suing stockholder, on behalf of the corporation, is
only a nominal party.
In this case, the PNCC Board cannot conceivably be expected to attack
the validity of the Compromise Agreement since the PNCC Board itself
approved the Compromise Agreement. In fact, the PNCC Board steadfastly
defends the Compromise Agreement for allegedly being advantageous to
PNCC.
Besides, the circumstances in this case are peculiar. Sison, as former
PNCC President and Chairman of the PNCC Board, was responsible for the
approval of the Board Resolution issued on 19 June 2001 revoking the
previous Board Resolution admitting PNCCs liability for the Marubeni loans.
Such revocation, however, came after Radstock had filed an action for
collection and damages against PNCC on 15 January 2001. Then, when the
trial court rendered its decision on 10 December 2002 in favor of Radstock,

Page 395 of 1072

Sison was no longer the PNCC President and Chairman, although he remains
a stockholder of PNCC.

TOPIC: DERIVATICE SUIT: REMEDIES TO ENFORCE PERSONAL LIABILITY


ANTHONY S. YU, ROSITA G. YU and JASON G. YU, Petitioners,
vs.
JOSEPH S. YUKAYGUAN, NANCY L. YUKAYGUAN, JERALD NERWIN
L. YUKAYGUAN, and JILL NESLIE L. YUKAYGUAN, [on their own behalf
and on behalf of] WINCHESTER INDUSTRIAL SUPPLY, INC.,
Respondents.
G.R. No. 177549. June 18, 2009
FACTS:
Petitioners and the respondents were all stockholders of Winchester.
Respondents filed against petitioners complaint for Accounting, Inspection of
Corporate Books and Damages through Embezzlement and Falsification of
Corporate Records and Accounts, in their own behalf and as a derivative suit
on behalf of Winchester.
According to respondents, Winchester, Inc. was established and
incorporated, with petitioner Anthony as one of the incorporators, holding
1,000 shares of stock worth P100,000.00. Petitioner Anthony paid for the said
shares of stock with respondent Josephs money, thus, making the former a
mere trustee of the shares for the latter. Petitioner Anthony ceded 800 of his
1,000 shares of stock in Winchester to respondent Joseph, as well as Yu Kay
Guan, Siao So Lan, and John S. Yu. Petitioner Anthony remained as trustee for
respondent Joseph of the 200 shares of stock in Winchester, Inc., still in
petitioner Anthonys name.
Respondents then alleged that, Winchester bought from its
incorporators, excluding petitioner Anthony, their accumulated 8,500 shares
in the corporation. Subsequently, Winchester sold the same 8,500 shares to
other persons, who included respondents Nancy, Jerald, and Jill; and
petitioners Rosita and Jason.
Respondents further averred that although respondent Joseph
appeared as the Secretary and Treasurer in the corporate records of
Winchesterpetitioners actually controlled and ran the said corporation as if it
were their own family business. Petitioner Rosita handled the money market
placements of the corporation to the exclusion of respondent Joseph, the
designated Treasurer of Winchester. Petitioners were also misappropriating

Page 396 of 1072

the funds and properties of Winchester. by understating the sales, charging


their personal and family expenses to the said corporation, and withdrawing
stocks for their personal use without paying for the same. Respondents
attached to the Complaint various receipts to prove the personal and family
expenses charged by petitioners to Winchester.

ISSUE/S:
Whether or not a derivative suit may be converted into liquidation
proceedings.
RULING:
The general rule is that where a corporation is an injured party,
its power to sue is lodged with its board of directors or trustees. Nonetheless,
an individual stockholder is permitted to institute a derivative suit on behalf
of the corporation wherein he holds stocks in order to protect or vindicate
corporate rights, whenever the officials of the corporation refuse to sue, or
are the ones to be sued, or hold the control of the corporation. In such
actions, the suing stockholder is regarded as a nominal party, with the
corporation as the real party in interest. A derivative action is a suit by a
shareholder to enforce a corporate cause of action. The corporation is a
necessary party to the suit. And the relief which is granted is a judgment
against a third person in favor of the corporation. Similarly, if a corporation
has a defense to an action against it and is not asserting it, a stockholder
may intervene and defend on behalf of the corporation.
In contrast, liquidation is a necessary consequence of the dissolution of
a corporation. It is specifically governed by Section 122 of the Corporation
Code, which reads:
SEC. 122. Corporate liquidation. Every corporation whose
charter expires by its own limitation or is annulled by forfeiture
or otherwise, or whose corporate existence for other purposes is
terminated in any other manner, shall nevertheless be continued
as a body corporate for three (3) years after the time when it
would have been so dissolved, for the purpose of prosecuting
and defending suits by or against it and enabling it to settle and
close its affairs, to dispose of and convey its property and to
distribute its assets, but not for the purpose of continuing the
business for which it was established.
At any time during said three (3) years, said corporation is authorized
and empowered to convey all of its property to trustees for the benefit of
stockholders, members, creditors, and other persons in interest. From and

Page 397 of 1072

after any such conveyance by the corporation of its property in trust for the
benefit of its stockholders, members, creditors and others in interest, all
interest which the corporation had in the property terminates, the legal
interest vests in the trustees, and the beneficial interest in the stockholders,
members, creditors or other persons in interest.
Upon winding up of the corporate affairs, any asset distributable to any
creditor or stockholder or member who is unknown or cannot be found shall
be escheated to the city or municipality where such assets are located.
Except by decrease of capital stock and as otherwise allowed by this
Code, no corporation shall distribute any of its assets or property except
upon lawful dissolution and after payment of all its debts and liabilities.
Following the voluntary or involuntary dissolution of a corporation,
liquidation is the process of settling the affairs of said corporation, which
consists of adjusting the debts and claims, that is, of collecting all that is due
the corporation, the settlement and adjustment of claims against it and the
payment of its just debts. More particularly, it entails the following:
Winding up the affairs of the corporation means the collection of all
assets, the payment of all its creditors, and the distribution of the remaining
assets, if any among the stockholders thereof in accordance with their
contracts, or if there be no special contract, on the basis of their respective
interests. The manner of liquidation or winding up may be provided for in the
corporate by-laws and this would prevail unless it is inconsistent with law.
It may be undertaken by the corporation itself, through its Board of
Directors; or by trustees to whom all corporate assets are conveyed for
liquidation; or by a receiver appointed by the SEC upon its decree dissolving
the corporation.
Glaringly, a derivative suit is fundamentally distinct and independent
from liquidation proceedings. They are neither part of each other nor the
necessary consequence of the other. There is totally no justification for the
Court of Appeals to convert what was supposedly a derivative suit instituted
by respondents, on their own behalf and on behalf of Winchester against
petitioners, to a proceeding for the liquidation of Winchester.

Page 398 of 1072

TOPIC: DERIVATICE SUIT: REMEDIES TO ENFORCE PERSONAL LIABILITY


VIRGINIA O. GOCHAN, et. al., petitioner,
vs.
RICHARD G. YOUNG, et. al., respondents.
G.R. No. 131889. March 12, 2001
FACTS:
In June, 1951, Felix Gochan and Sons Realty Corporation was registered
with the SEC, with Felix Gochan, Sr., Maria Pan Nuy Go Tiong, Pedro Gochan,
Tomasa Gochan, Esteban Gochan and Crispo Gochan as its incorporators.
Felix Gochan Sr.'s daughter, Alice, mother of herein respondents,
inherited 50 shares of stock in Gochan Realty from the former.
Alice died in 1955, leaving the 50 shares to her husband, John Young,
Sr.
In 1962, the Regional Trial Court of Cebu adjudicated 6/14 of these
shares to her children, herein respondents Richard Young, David Young, Jane
Young Llaban, John Young Jr., Mary Young Hsu and Alexander Thomas Young.
Having earned
September 1979.

dividends,

these

stocks

numbered

179

by

20

Five days later (25 September), at which time all the children had
reached the age of majority, their father John Sr., requested Gochan Realty to
partition the shares of his late wife by cancelling the stock certificates in his
name and issuing in lieu thereof, new stock certificates in the names of
herein respondents.
On 17 October 1979, respondent Gochan Realty refused, citing as
reason, the right of first refusal granted to the remaining stockholders by the
Articles of Incorporation.
Thereafter, John, Sr. died, leaving the shares to the respondents.
On 8 February 1994, Cecilia Gochan Uy and Miguel Uy filed a complaint
with the SEC for issuance of shares of stock to the rightful owners,

Page 399 of 1072

nullification of shares of stock, reconveyance of property impressed with


trust, accounting, removal of officers and directors and damages against
respondents. A Notice of Lis Pendens was annotated upon the real properties
of the corporation.
.
ISSUE/S:
Whether or not herein respondents have the personality to file a
derivative suit.
RULING:
Yes.
The fact that certain persons are not registered as stockholders in the
books of the corporation will not bar them from filing a derivative suit, if it is
evident from the allegations in the complaint that they are bona fide
stockholders.
The complaint, which was deemed admitted by the adverse party due
to their failure to so specifically deny, alleges that the Youngs are, as in fact
they are, the heirs of John Young, Jr.- husband of Alice Gochan who
predeceased her husband and transferred the share of stock in question to
him. As such, it cannot be denied that the children are the beneficial owners
of the stocks, who, after partition of their fathers estate, would be the legal
owners thereof. Thus, they have the personality to file the derivative suit not
only in behalf of the corporation but also in their own right as heirs and as
stockholders.

Page 400 of 1072

TOPIC: DERIVATICE SUIT: REMEDIES TO ENFORCE PERSONAL LIABILITY


TAM WING TAK, petitioner,
vs.
HON. RAMON P. MAKASIAR (in his Capacity as Presiding Judge of the
Regional Trial Court of Manila, Branch 35) and ZENON DE GUIA (in
his capacity as Chief State Prosecutor), respondents.
G.R. No. 122452. January 29, 2001

FACTS:
On November 11, 1992, petitioner, in his capacity as director of
Concord-World Properties, Inc., a domestic corporation, filed an affidavitcomplaint charging Vic Ang Siong with violation of B.P. Blg. 22. The complaint
alleged that a check for the amount of P83, 550,000.00, issued by Vic Ang
Siong in favor of Concord, was dishonored when presented for encashment.
Vic Ang Siong sought the dismissal of the case on two grounds: First,
that petitioner had no authority to file the case on behalf of Concord, the
payee of the dishonored check, since the firms board of directors had not
empowered him to act on its behalf. Second, he and Concord had already
agreed to amicably settle the issue after he made a partial payment of P19,
000, 000.00 on the dishonored check.
ISSUE/S:
Whether or not petitioner had the capacity to sue in behalf of
Concord.
RULING:
No.
The Court held that it is not disputed in the instant case that Concord,
a domestic corporation, was the payee of the bum check, not
petitioner. Therefore, it is Concord, as payee of the bounced check, which is
the injured party. Since petitioner was neither a payee nor a holder of the

Page 401 of 1072

bad check, he had neither the personality to sue nor a cause of action
against Vic Ang Siong. Under Section 36 of the Corporation Code, read in
relation to Section 23, it is clear that where a corporation is an injured party,
its power to sue is lodged with its board of directors or trustees. Note that
petitioner failed to show any proof that he was authorized or deputized or
granted specific powers by Concords board of director to sue Victor Ang
Siong for and on behalf of the firm. Clearly, petitioner as a minority
stockholder and member of the board of directors had no such power or
authority to sue on Concords behalf. Nor can the Court upheld his act as a
derivative suit. For a derivative suit to prosper, it is required that the
minority stockholder suing for and on behalf of the corporation must allege in
his complaint that he is suing on a derivative cause of action on behalf of the
corporation and all other stockholders similarly situated who may wish to join
him in the suit. There is no showing that petitioner has complied with the
foregoing requisites. It is obvious that petitioner has not shown any clear
legal right which would warrant the overturning of the decision of public
respondents to dismiss the complaint against Vic Ang Siong.

Page 402 of 1072

TOPIC: DERIVATICE SUIT: REMEDIES TO ENFORCE PERSONAL LIABILITY


GILDA C. LIM, et al
vs.
PATRICIA LIM-YU, in her capacity as a minority stockholder of
LIMPAN INVESTMENT CORPORATION
G.R. No. 138343. February 19, 2001
FACTS:
At a special meeting on 07 October 1994, the Board of Directors of
Limpan Investment Corporation approved a resolution to pay the legal
services of Gilda C. Lim in the handling of various cases on behalf of, or
involving the corporation in the amount of P1, 551,500.00 to be paid in
equivalent value in shares of stock of the corporation totaling 15,515 shares.
Due to the issuance of the unsubscribed shares to the petitioner Gilda C. Lim,
all of LIMPAN's authorized capital stock became fully subscribed, with Gilda
C. Lim ending up controlling 62.5% of the shares.
In July 1996, the respondent Patricia Lim Yu, a sister of the petitioner,
LIM, filed a complaint against the members of the Board of Directors of
LIMPAN who approved the aforesaid resolution.
ISSUE/S:
Whether or not respondent has the legal capacity to file a derivative
suit against the petitioners.
RULING:
No.
The Court held that respondent cannot transact in representation of or
for the benefit of her parents, brothers or sisters, or the Limpan Investment
Corporation. She can act only on and in her own behalf, not that of
petitioners or the Corporation.

Page 403 of 1072

There appears to be a confusion on the nature of the suit initiated


before the SEC. Petitioners describe it as a derivative suit, which has been
defined as "an action brought by minority shareholders in the name of the
corporation to redress wrongs committed against it, for which the directors
refuse to sue. It is a remedy designed by equity and has been the principal
defense of the minority shareholders against abuses by the majority." In a
derivative action, the real party in interest is the corporation itself, not the
shareholder(s) who actually instituted it.
If the suit filed by respondent was indeed derivative in character, then
respondent may not have the capacity to sue. The reason is that she would
be acting in representation of the corporation, an act which the TRO enjoins
her from doing.
However, that the suit of respondent cannot be characterized as
derivative, because she was complaining only of the violation of her
preemptive right under Section 39 of the Corporation Code. She was merely
praying that she be allowed to subscribe to the additional issuances of stocks
in proportion to her shareholdings to enable her to preserve her percentage
of ownership in the corporation. She was therefore not acting for the benefit
of the corporation. Quite the contrary, she was suing on her own behalf, out
of a desire to protect and preserve her preemptive rights. Unquestionably,
the TRO did not prevent her from pursuing that action.
Petitioners fail to appreciate the distinction between the act itself and
its net result. The act of filing the suit did not in any way bind the
corporation.

Page 404 of 1072

TOPIC: DERIVATIVE SUIT: REMEDIES TO ENFORCE PERSONAL LIABILITY


WESTERN INSTITUTE OF TECHNOLOGY, INC., HOMERO L. VILLASIS,
DIMAS ENRIQUEZ, PRESTON F. VILLASIS & REGINALD F. VILLASIS
vs.
RICARDO T. SALAS, SOLEDAD SALAS-TUBILLEJA, ANTONIO S. SALAS,
RICHARD S. SALAS & HON. JUDGE PORFIRIO PARIAN
G.R. No. 113032
August 21, 1997
278 SCRA 216, 223
FACTS:
Ricardo T. Salas, Salvador T. Salas, Soledad Salas-Tubilleja, Antonio S.
Salas, and Richard S. Salas, belonging to the same family, are the majority
and controlling members of the Board of Trustees of Western Institute of
Technology, Inc. (WIT), a stock corporation engaged in the operation, among
others, of an educational institution. According to Homero L. Villasis, Dimas
Enriquez, Peston F. Villasis, and Reginald F. Villasis, the minority stockholders
of WIT, sometime on 1 June 1986 in the principal office of WIT at La Paz, Iloilo
City, a Special Board Meeting was held. In attendance were other members
of the Board including Reginald Villasis.
Prior to said Special Board Meeting, copies of notice thereof, dated 24
May 1986, were distributed to all Board Members. The notice allegedly
indicated that the meeting to be held on 1 June 1986 included Item 6 which
states that "Possible implementation of Art. III, Sec. 6 of the Amended ByLaws of Western Institute of Technology, Inc. on compensation of all officers
of the corporation." In said meeting, the Board of Trustees passed Resolution
48, series 1986, granting monthly compensation to Salas, et. al. as corporate
officers retroactive 1 June 1985, in the following amounts: Chairman
9,000.00/month, Vice Chairman P3,500.00/month, Corporate Treasurer
P3,500.00/month and Corporate Secretary P3,500.00/month, retroactive June
1, 1985 and the ten per centum of the net profits shall be distributed equally

Page 405 of 1072

among the ten members of the Board of Trustees. This shall amend and
supersede any previous resolution.
A few years later, or on 13 March 1991, Homero Villasis, Preston
Villasis, Reginald Villasis and Dimas Enriquez filed an affidavit-complaint
against Salas, et. al. before the Office of the City Prosecutor of Iloilo, as a
result of which2 separate criminal informations, one for falsification of a
public document under Article 171 of the Revised Penal Code and the other
for estafa under Article 315, par. 1(b) of the RPC, were filed before Branch 33
of the Regional Trial Court of Iloilo City. The charge for falsification of public
document was anchored on Salas, et. al.'s submission of WIT's income
statement for the fiscal year 1985-1986 with the Securities and Exchange
Commission (SEC) reflecting therein the disbursement of corporate funds for
the compensation of Salas, et.al. based on Resolution 4, series of 1986,
making it appear that the same was passed by the board on 30March 1986,
when in truth, the same was actually passed on 1 June 1986, a date not
covered by the corporation's fiscal year 1985-1986 (beginning May 1, 1995
and ending April 30, 1986).
Thereafter, trial for the two criminal cases (Criminal Cases 37097 and
37098), was consolidated. After a full-blown hearing, Judge Porfirio Parian
handed down a verdict of acquittal on both counts dated 6 September 1993
without imposing any civil liability against the accused therein. Villasis, et. al.
filed a Motion for Reconsideration of the civil aspect of the RTC Decision
which was, however, denied in an Order dated 23 November 1993.Villasis,
et. al. filed the petition for review on certiorari. Significantly on 8 December
1994, a Motion for Intervention, dated 2 December 1994, was filed before
this Court by Western Institute of Technology, Inc., disowning its inclusion in
the petition and submitting that Atty. Tranquilino R. Gale, counsel for Villasis,
et.al., had no authority whatsoever to represent the corporation in filing the
petition. Intervenor likewise prayed for the dismissal of the petition for being
utterly without merit. The Motion for Intervention was granted on January 16,
1995.
ISSUE/S:
Whether the grant of compensation to Salas, et. al. is proscribed
under Section 30 of the Corporation Code.
RULING:
Directors or trustees, as the case may be, are not entitled to salary or
other compensation when they perform nothing more than the usual and
ordinary duties of their office. This rule is founded upon a presumption that
directors/trustees render service gratuitously, and that the return upon their
shares adequately furnishes the motives for service, without compensation.
Under Section 30 of the Corporation Code, there are only two (2) ways by

Page 406 of 1072

which members of the board can be granted compensation apart from


reasonable per diems:
1. when there is a provision in the by-laws fixing their
compensation; and
2. when the stockholders representing a majority of the
outstanding capital stock at a regular or special stockholders'
meeting agree to give it to them.
Also, the proscription, however, against granting compensation to
director/trustees of corporation is not a sweeping rule. Worthy of note is the
clear phraseology of Section 30 which state: "The directors shall not receive
any compensation, as such directors." The phrase as such directors is not
without significance for it delimits the scope of the prohibition to
compensation given to them for services performed purely in their capacity
as directors or trustees. The unambiguous implication is that members of the
board may receive compensation, in addition to reasonable per diems, when
they render services to the corporation in a capacity other than as
directors/trustees. Herein, resolution 48, s. 1986 granted monthly
compensation to Salas, et. al. not in their capacity as members of the board,
but rather as officers of the corporation, more particularly as Chairman, ViceChairman, Treasurer and Secretary of Western Institute of Technology.
Clearly, therefore, the prohibition with respect to granting compensation to
corporate directors/trustees as such under Section 30 is not violated in this
particular case. Consequently, the last sentence of Section 30 which provides
that "In no case shall the total yearly compensation of directors, as such
directors, exceed ten (10%) percent of the net income before income tax of
the corporation during the preceding year" does not likewise find application
in this case since the compensation is being given to Salas, et. al. in their
capacity as officers of WIT and not as board members.

Page 407 of 1072

TOPIC: DERIVATIVE SUIT: REMEDIES TO ENFORCE PERSONAL LIABILITY


FIRST PHILIPPINE INTERNATIONAL BANK (Formerly Producers Bank
of the Philippines) and MERCURIO RIVERA,
vs.
COURT OF APPEALS, CARLOS EJERCITO, in substitution of DEMETRIO
DEMETRIA, and JOSE JANOLO.
G.R. No. 115849
January 24, 1996
FACTS:
First Philippine International Bank acquired six parcels of land with a
total area of 101 hectares located at Don Jose, Sta. Rose, Laguna through its
banking operations. The said parcels of land formerly belonged to BYME
Investment and Development Corp, as they were mortgaged as collateral for
a bank loan. The original plaintiffs, Demetrio and Jose Janolo, wanted to
purchase the property and thus initiated negotiations for that purpose. In
the early part of August 1987, Demetrio and Jose, upon the suggestion of
BYME investment's legal counsel, met with Mercurio Rivera, Manager of the
Property Management Department of the defendant bank. The meeting was
held pursuant to Demetrios and Joses plan to buy the property. After the
meeting, plaintiff Jose, following the advice of Mercurio, made a formal
purchase offer to the bank through a letter. The offer to buy was accepted by
the bank. Then, the conservator of the bank (which has been placed under
conservatorship by the Central Bank since 1984) was replaced by an Acting
Conservator in the person of defendant Leonida T. Encarnacion. Mercurio
wrote Demetrio a letter stating that the land to be purchased is under study
yet as of this time by the newly created committee for submission to the
newly designated Acting Conservator of the bank. This was then followed by
a number of demands for the bank to comply with what was deemed as a
perfected contract of sale by former plaintiffs. These were then refused by
the bank.
Demetrio and Jose, through a letter to defendant Mercurio, already
tendered payment of the amount of P5.5 million "pursuant to (our) perfected
sale agreement." The Bank refused to receive both the payment and the
letter. Instead, the parcels of land involved in the transaction were advertised
by the bank for sale to any interested buyer. Demetrio and Jose demanded
the execution by the bank of the documents on what was considered as a
"perfected agreement" through another letter. The Bank acknowledged the
receipt of the letter but remained adamant in its position. Demetrio and
Jose, through counsel, made a final demand for compliance by the bank with
its obligations under the considered perfected contract of sale. The Bank
filed a reply repudiating the authority of Mercurio and claimed that his
dealings with the Demetrio and Jose, particularly his counter-offer of P5.5

Page 408 of 1072

Million are unauthorized or illegal; thus, there is no perfected contract of


sale.
On that basis, the Bank justified the refusal of the tenders of payment
and the non-compliance with the obligations under what the Demetrio and
Jose considered to be a perfected contract of sale. Other major stockholders
of the Bank filed a motion to intervene as a derivative suit with the Regional
Trial Court of Makati against the plaintiffs "to declare any perfected sale of
the property as unenforceable and to stop Ejercito from enforcing or
implementing the sale In his answer, Jose argued that the Second Case was
barred by litis pendentia by virtue of the case then pending in the Court of
Appeals. During the pre-trial conference in the Second Case, plaintiffs filed a
Motion for Leave of Court to Dismiss the Case without Prejudice. "The Bank
opposed this motion on the ground, among others, that plaintiff's act of
forum shopping justifies the dismissal of both cases, with prejudice. The
Bank, in its memorandum, averred that this motion is still pending in the
Makati RTC.
The Bank vigorously argues that in spite of this verification, Demetrio, Jose,
etc. are guilty of actual forum shopping because the instant petition pending
before this Court involves "identical parties or interests represented, rights
asserted and reliefs sought (as that) currently pending before the Regional
Trial Court, Makati Branch 134 in the Second Case. In fact, the issues in the
two cases are so intertwined that a judgment or resolution in either case will
constitute res judicata in the other."
On the other hand, Demetrio, Jose, etc. explain that there is no forumshopping because:
1. In the earlier or "First Case" from which this proceeding arose,
the Bank was impleaded as a defendant, whereas in the "Second
Case" (assuming the Bank is the real party in interest in a
derivative suit), it was plaintiff;
2. "The derivative suit is not properly a suit for and in behalf of the
corporation under the circumstances"
3. Although the CERTIFICATION/VERIFICATION signed by the Bank
president and attached to the Petition identifies the action as a
"derivative suit," it "does not mean that it is one" and "that is a
legal question for the courts to decide"
ISSUE:
Whether or not the 2nd Case filed was a derivative suit.
RULING:
Yes.

Page 409 of 1072

In the instant case before us, there is also identity of parties, or at


least, of interests represented. Although the plaintiffs in the Second Case
(Henry L. Co. et al.) are not name parties in the First Case, they represent the
same interest and entity, namely, petitioner Bank, because:
Firstly, they are not suing in their personal capacities, for they
have no direct personal interest in the matter in controversy.
They are not principally or even subsidiary liable; much less are
they direct parties in the assailed contract of sale; and
Secondly, the allegations of the complaint in the Second Case
show that the stockholders are bringing a "derivative suit". In the
caption itself, petitioners claim to have brought suit "for and in
behalf of the Producers Bank of the Philippines. Indeed, this is
the very essence of a derivative suit:
An individual stockholder is permitted to institute a
derivative suit on behalf of the corporation wherein he
holds stock in order to protect or vindicate corporate
rights, whenever the officials of the corporation refuse to
sue, or are the ones to be sued or hold the control of the
corporation. In such actions, the suing stockholder is
regarded as a nominal party, with the corporation as the
real party in interest. (Gamboa v. Victoriano, 90 SCRA 40,
47 [1979]).
In the face of the damaging admissions taken from the complaint in
the Second Case, petitioners, quite strangely, sought to deny that the
Second Case was a derivative suit, reasoning that it was brought, not by the
minority shareholders, but by Henry Co et al., who not only own, hold or
control over 80% of the outstanding capital stock, but also constitute the
majority in the Board of Directors of petitioner Bank. That being so, then they
really represent the Bank. So, whether they sued "derivatively" or directly,
there is undeniably an identity of interests/entity represented.
Petitioner also tried to seek refuge in the corporate fiction that the
personality Of the Bank is separate and distinct from its shareholders. But
the rulings of this Court are consistent: "When the fiction is urged as a
means of perpetrating a fraud or an illegal act or as a vehicle for the evasion
of an existing obligation, the circumvention of statutes, the achievement or
perfection of a monopoly or generally the perpetration of knavery or crime,
the veil with which the law covers and isolates the corporation from the
members or stockholders who compose it will be lifted to allow for its
consideration merely as an aggregation of individuals."
In addition to the many cases where the corporate fiction has been
disregarded, we now add the instant case, and declare herewith that the

Page 410 of 1072

corporate veil cannot be used to shield an otherwise blatant violation of the


prohibition against forum-shopping. Shareholders, whether suing as the
majority in direct actions or as the minority in a derivative suit, cannot be
allowed to trifle with court processes, particularly where, as in this case, the
corporation itself has not been remiss in vigorously prosecuting or defending
corporate causes and in using and applying remedies available to it. To rule
otherwise would be to encourage corporate litigants to use their
shareholders as fronts to circumvent the stringent rules against forum
shopping.

Page 411 of 1072

TOPIC: DERIVATIVE SUIT: REMEDIES TO ENFORCE PERSONAL LIABILITY


COMMART (PHILS.) INC., JESUS, CORAZON, ALBERTO, AND BERNARD
all surnamed MAGLUTAC
vs.
SECURITIES & EXCHANGE COMMISSION and ALICE MAGLUTAC
G.R. No. 85318
June 3, 1991
198 SCRA 73
FACTS:
Two brothers, Jesus and Mariano Maglutac organized Commart (Phils.),
Inc. Sometime in June 1984, the two brothers agreed to go their separate
ways, with Mariano being persuaded to sell to Jesus his shareholdings in
Commart amounting to 25% of the outstanding capital stock. Marianos wife,
Alice M. Maglutac (Private respondent herein) who has been for years a
stockholder and director of Commart, did not dispose of her shareholdings,
and thus continued as such even after the sale of Marianos equity.
Shortly after the sale of his equity in Commart to Jesus, Mariano
allegedly discovered that for several years, Jesus and his wife Corazon (who
was herself a director) had been siphoning and diverting to their private
bank accounts in the United States and in Hongkong gargantuan amounts
sliced off from commissions due Commart from some foreign suppliers.
Consequently, spouses Mariano and Alice Maglutac filed a complaint with the
SEC against Jesus Maglutac and rest of the members of the Board of
Directors of Commart. SEC held that the suit is a derivative suit.
ISSUE/S:
Whether or not SEC correctly held that the case was a derivative
suit.
RULING:
Yes.
SEC correctly held that the case was a minority stockholdings
derivative suit and correctly sustained the hearing panels denial of the
motions to dismiss. Readily shows that it avers the diversion of corporate
income into the private bank accounts of petitioner Jesus T. Maglutac and his
wife. Likewise, the principal relief prayed for in the complaint is the recovery
of a sum of money in favor of the corporation. This being the case, the
complaint is definitely a derivative suit. Consequently, the SEC correctly
held that the case was a minority stockholders derivative suit and correctly
sustained the hearing panels denial- insofar as Alice Maglutac was
concerned- of the motions to dismiss it.

Page 412 of 1072

TOPIC: DERIVATIVE SUIT: REMEDIES TO ENFORCE PERSONAL LIABILITY


ELTON W. CHASE, as minority stockholder and on behalf of the
stockholders similarly situated and for the benefit of AMERICAN
MACHINERY AND PARTS MANUFACTURING, INC.,
vs.
THE COURT of FIRST INSTANCE OF MANILA, BRANCH XIV, DR. VICTOR
BUENCAMINO, SR., VICTOR BUENCAMINO, JR., DOLORES A.
BUENCAMINO and JULIO B. FRANCIA, JR.
G.R. No. L-20457
October 29, 1966
FACTS:
On August 20, 1960, petitioner, a minority stockholder of AMPARTS,
filed a derivative suit in the CFI of Manila against Dr. Victor Buencamino Sr.,
Victor Buencamino, Jr., Dolores A. Buencamino and Julio B. Francia, Jr.,
majority stockholders and corporate directors of AMPARTS charging them
with breach of trust; praying for their removal as directors and, if necessary,
for the dissolution and liquidation of said corporation. Attached to the
complaint was an application for the appointment of a receiver of AMPARTS.
After a hearing on the application the court, then presided by the Hon.
Magno S. Gatmaitan, issued an order dated June 10, 1961 denying the same,
but requiring respondents to file bond in the amount of P100,000.00 to
answer for whatever damages petitioner might suffer by reason of the
denial. Petitioner's motion for reconsideration was likewise denied. After trial
on the merits, the court rendered judgment finding Dr. Buencamino guilty of
mismanagement and condemning him "to pay Amparts the sum of
P1,970,200 with legal interest from date of the filing of the complaint; he is
also prohibited from collecting any interest on the sum of P300,000.00 paid
by him on the 15th July, 1955 on the initial subscription, and such interest as
has already been paid to him is ordered refunded with legal interest from the
date of the filing of the complaint . . ."
On May 8, 1962, petitioner filed a motion for the appointment of
Lawrence Moran as receiver of Amparts until the full amount of the above
judgment against respondent Buencamino is fully satisfied or until the
dissolution or liquidation of said corporation.
On May 12, 1962, the Court issued the following order:
(1) Mr. Chase shall have free access to AMPARTS and its
records personally and/or through representative duly
authorized;

Page 413 of 1072

(2) Decisions of Dr. Buencamino and/or management of


AMPARTS shall be made known to Chase who shall have
the right to object and if so, the matter shall be notified to
the Court which shall resolve the difficulties; in the interim,
pending the objection, the decision shall not be enforced or
made operative;
With this resolution, the Court disposes for the present of the
issue of receivership.
Supplementing the above-quoted order, the respondent court, now
presided by the Hon. Jesus De Veyra, issued the following order of August 27,
1962:
As for the appointment of a receiver, Judge Gatmaitan decided on the
temporary measure of giving petitioner herein a veto right, appealable
to this Court, on all decisions of management. Considering that up to
the present, the Buencaminos own 2/3 of the stock of the corporation,
the solution is equitable and must be allowed to continue subject to
the condition that once a decision of management is made known to
plaintiff, he must make known his objection thereto to the Court within
five (5) days from receipt of said decision, otherwise he shall be
deemed to have waived any objection to the decision.

ISSUE/S:
Whether or not the respondent court committed a grave abuse of
discretion in issuing its orders of June 10, 1961, June 21, 1961,
May 12, 1962, and August 27 of the same year mentioned
heretofore.
RULING:
It is well settled in this jurisdiction that where corporate directors are
guilty of a breach of trust and intra-corporate remedy is futile, the minority
stockholders may resort to the courts for appropriate relief and, incidentally,
ask for the appointment of a receiver for the protection of their rights. In
such case, however, the appointment of a receiver is a matter addressed to
the sound discretion of the court, and it has been frequently held that such
discretion to appoint a receiver who would take over the administration of

Page 414 of 1072

the corporate business should be exercised with great caution and only when
the necessity therefore is clear.
The facts of the present case show that, in connection with the order of
June 10, 1961, which denied petitioner's application for the appointment of a
receiver, the court required respondents herein to file a bond in the amount
of P100,000.00 to answer for whatever damages petitioner might suffer by
reason of the denial. Again, perhaps by reason of the judgment rendered
against Dr. Buencamino finding him guilty of mismanagement etc., the
respondent court, through the Hon. Jesus de Veyra, issued the order of
August 27, 1962 whose pertinent portion is quoted above.
Upon the facts of the case, and considering the precautionary
measures adopted by the respondent court for the protection of petitioner's
rights and interest in AMPARTS, We cannot find our way clear to ruling that
said court had committed a grave abuse of discretion in issuing the orders
complained of.
WHEREFORE, the petition for certiorari is dismissed.

Page 415 of 1072

TOPIC:DERIVATIVE SUIT: REMEDIES TO ENFORCE PERSONAL LIABILITY


SAN MIGUEL CORPORATION, represented by EDUARDO DE LOS
ANGELES,
vs.
ERNEST KAHN, ANDRES SORIANO III, BENIGNO TODA, JR., ANTONIO
ROXAS, ANTONIO PRIETO, FRANCISCO EIZMENDI, JR., EDUARDO
SORIANO, RALPH KAHN and RAMON DEL ROSARIO, JR.
G.R. No. 85339. August 11, 1989
FACTS:
On December 15, 1983, 33,133,266 shares of the outstanding capital
stock of the San Miguel Corporation were acquired by fourteen other
corporations, and were placed under a Voting Trust Agreement in favor of
the late Andres Soriano, Jr. When the latter died, Eduardo M. Cojuangco, Jr.
was elected Substitute Trustee on April 9, 1984 with power to delegate the
trusteeship in writing to Andres Soriano III. Shortly after the Revolution of
February, 1986, Cojuangco left the country amid "persistent reports" that
"huge and unusual cash disbursements from the funds of SMC" had been
irregularly made, and the resources of the firm extensively used in support of
the candidacy of Ferdinand Marcos during the snap elections in February,
1986.
On March 26, 1986, an "Agreement" was executed between Andres
Soriano III, as "Buyer," and the 14 corporations, as "Sellers," for the purchase
by Soriano, "for himself and as agent of several persons," of the 33,133,266
shares of stock at the price of P100.00 per share, or "an aggregate sum of
Three Billion Three Hundred Thirteen Million Three Hundred Twenty Six
Thousand Six Hundred (P3,313,326,600.00) Pesos payable in specified
installments. The Agreement revoked the voting trust above mentioned, and
expressed the desire of the corporations to sell the shares of stock "to pay
certain outstanding and unpaid debts," and Soriano's own wish to purchase
the same "in order to institutionalize and stabilize the management of the
COMPANY in and the professional officer corps, mandated by the COMPANY's
By- laws, and to direct the COMPANY towards giving the highest priority to its
principal products and extensive support to agriculture programme of' the
Government ...
At this point the 33,133,266 SMC shares were sequestered by the
Presidential Commission on Good Government (PCGG), on the ground that
the stock belonged to Eduardo Cojuangco, Jr., allegedly a close associate and

Page 416 of 1072

dummy of former President Marcos, and the sale thereof was "in direct
contravention of .. Executive Orders Numbered 1 and 2 which prohibit.
At the meeting of the SMC Board on January 30, 1987, Eduardo de los
Angeles, one of the PCGG representatives in the SMC board, impugned said
Resolution No. 86-12-2, denying that it was ever adopted, and stating that
what in truth was agreed upon at the meeting of December 4, 1986 was
merely a "further study" by Director Ramon del Rosario of a plan presented
by him for the assumption of the loan. De los Angeles also pointed out
certain "deleterious effects" thereof. He was however overruled by private
respondents. When his efforts to obtain relief within the corporation and
later the PCGG proved futile, he repaired to the Securities and Exchange
Commission.
.
ISSUE/S:
Whether or not de los Angeles could file a derivative suit as
stockholder and/or director of the San Miguel Corporation.
RULING:
The bona fide ownership by a stockholder of stock in his own right
suffices to invest him with standing to bring a derivative action for the
benefit of the corporation. The number of his shares is immaterial since he is
not suing in his own behalf, or for the protection or vindication of his own
particular right, or the redress of a wrong committed against him,
individually, but in behalf and for the benefit of the corporation.
Neither can the "conflict-of-interest" theory be upheld. From the conceded
premise that de los Angeles now sits in the SMC Board of Directors by the
grace of the PCGG, it does not follow that he is legally obliged to vote as the
PCGG would have him do, that he cannot legitimately take a position
inconsistent with that of the PCGG, or that, not having been elected by the
minority stockholders, his vote would necessarily never consider the latter's
interests. The proposition is not only logically indefensible, non sequitur, but
also constitutes an erroneous conception of a director's role and function, it
being plainly a director's duty to vote according to his own independent
judgment and his own conscience as to what is in the best interests of the
company. Moreover, it is undisputed that apart from the qualifying shares
given to him by the PCGG, he owns 20 shares in his own right, as regards
which he cannot from any aspect be deemed to be "beholden" to the PCGG,
his ownership of these shares being precisely what he invokes as the source
of his authority to bring the derivative suit.

Page 417 of 1072

It is also theorized, on the authority of the BASECO decision, that the


PCGG has no power to vote sequestered shares of stock as an act of
dominion but only in pursuance to its power of administration. The
inference is that the PCGG's act of voting the stock to elect de los Angeles to
the SMC Board of Directors was unauthorized and void; hence, the latter
could not bring suit in the corporation's behalf. The argument is strained and
obviously of no merit. As already more than plainly indicated, it was not
necessary for de los Angeles to be a director in order to bring a derivative
action; all he had to be was a stockholder, and that he was owning in his own
right 20 shares of stock, a fact not disputed by the respondents.
TOPIC: DERIVATICE SUIT: REMEDIES TO ENFORCE PERSONAL LIABILITY
HARRIE S. EVERETT, CRAL G. CLIFFORD, ELLIS H. TEAL and GEORGE
W. ROBINSON,
vs.
THE ASIA BANKING CORPORATION, NICHOLAS E. MULLEN, ERIC
BARCLAY, ALFRED F. KELLY, JOHN W. MEARS and CHARLES D.
MACINTOSH,
G.R. No.L-25241.November 3, 1926
49 Phil 512
FACTS:
Defendant Asia Banking Corporation, a foreign banking corporation
duly licensed to transact banking business in the Philippine Islands, with its
principal office and place of business at Manila and that said Corporation was
never been empowered by law or licensed to do any business other than
commercial banking in the Philippine. Defendants Nicholas E. Mullen, Alfred
F. Kelly, John W. Mears, and Charles D. Macintosh were residents of said City
of Manila and were officers, agents and employees of the said Corporation,
the said Mullen being the General Manager thereof in said City; That the
defendant Eric Barclay is a now a resident of Los Angeles, California, and the
defendant Mcintosh is also residing in the United States, his exact residence
being unknown.
Teal and Company, a domestic corporation duly incorporated under the
laws of the Philippine Islands and with principal office and place of business
at Manila. That during said times the plaintiffs Everett, Clifford, Teal and
Robinson were the principal stockholders in the Company owning a total of
4,478 shares therein and that the defendant Barclay was the only other
stockholder, owning one share thereof.
In the year 1921, Teal and Company has become indebted to the firm
of H. W. Peabody and Company in the sum of P300, 000, being for tractors,
plows and parts which had been ordered and delivered, the Bank and other

Page 418 of 1072

banks in Manila held drafts accepted by the Company under said H. W.


Peabody and Company's guarantee.
Thereafter, the Bank persuaded the Company and the said H. W.
Peabody and Co. and Smith, Kirkpatrick and Co. to enter into a so called
"creditors agreement" with itself, wherein it was mutually agreed that neither
of the parties should take action to collect its debts from the Company for
the term of two years after the date thereof. That these plaintiffs have no
copy of said agreement but beg leave to refer to the original of same, in
possession of the Bank, for greater certainty.
The said Company was indebted to the Bank in about the sum of
P750,000, which said sum was secured by mortgage on its personal property
and the improvements upon the real estate occupied by it.
It was further represented by the Bank and Mullen that in order to
protect the mutual interests of the Bank and the Company it was necessary
to carry into effect the said proposed voting trust without the knowledge of
the creditors above named and thereby place the Bank in an advantageous
position with regard to them. Relying upon the previous friendly relations
between the bank and the Company and between the individual defendants
and these plaintiffs and relying upon the promise and representations of the
defendants, these plaintiffs were induced to sign and did sign and deliver to
the Bank simultaneously a so-called "Voting Trust Agreement," executed by
the plaintiff stockholders and a Memorandum of Agreement executed by the
Company.
By reason of the facts above set forth and of their reliance upon the
good faith and good-will of the defendants these plaintiffs were induced to
sign the "Memorandum of Agreement," and "Voting Trust Agreement.
ISSUE/S:
Whether or not corporation Teal and Company is a necessary
party plaintiff.
RULING:
Invoking the well-known rule that shareholders cannot ordinarily sue in
equity to redress wrongs done to the corporation, but that the action must be
brought by the Board of Directors, the appellees argue and the court
below held that the corporation Teal and Company is a necessary party
plaintiff and that the plaintiff stockholders, not having made any demand on
the Board to bring the action, are not the proper parties plaintiff. But, like
most rules, the rule in question has its exceptions. It is alleged in the
complaint and, consequently, admitted through the demurrer that the
corporation Teal and Company is under the complete control of the principal

Page 419 of 1072

defendants in the case, and, in these circumstances, it is obvious that a


demand upon the Board of Directors to institute an action and prosecute the
same effectively would have been useless, and the law does not require
litigants to perform useless acts.
The conclusion of the court below that the plaintiffs, not being
stockholders in the Philippine Motors Corporation, had no legal right to
proceed against that corporation in the manner suggested in the complaint
evidently rest upon a misconception of the character of the action. In this
proceeding it was necessary for the plaintiffs to set forth in full the history of
the various transactions which eventually led to the alleged loss of their
property and, in making a full disclosure, references to the Philippine Motors
Corporation appear to have been inevitable. It is to be noted that the
plaintiffs seek no judgment against the corporation itself at this stage of the
proceedings.
The court below also erred in holding that the investigation of the
transaction referred to in the complaint is not within the province of the
courts, but should be conducted by some other agency. That discovery, such
as that demanded in the present action, is one of the functions of a court of
equity is so well established as to require no discussion.

Page 420 of 1072

TOPIC: DERIVATICE SUIT: REMEDIES TO ENFORCE PERSONAL LIABILITY


RICARDO L. GAMBOA, LYDIA R. GAMBOA, HONORIO DE 1A RAMA,
EDUARDO DE LA RAMA, and the HEIRS OF MERCEDES DE LA RAMABORROMEO
vs.
HON. OSCAR R. VICTORIANO as Presiding Judge of the Court of First
Instance of Negros Occidental, Branch II, BENJAMIN LOPUE, SR.,
BENJAMIN LOPUE, JR., LEONITO LOPUE, and LUISA U. DACLES
G.R. No.L-40620. May 5, 1979
90 SCRA 40
FACTS:
The plaintiffs, with the exception of Anastacio Dacles who was joined
as a formal party, are the owners of 1,328 shares of stock of the Inocentes
de la Rama, Inc., a domestic corporation, with an authorized capital stock of
3,000 shares, with a par value of P100.00 per share, 2,177 of which were
subscribed and issued, thus leaving 823 shares unissued; that upon the
plaintiffs' acquisition of the shares of stock held by Rafael Ledesma and Jose
Sicangco, Jr., then President and Vice-President of the corporation,
respectively, the defendants Mercedes R. Borromeo, Honorio de la Rama, and
Ricardo Gamboa, remaining members of the board of directors of the
corporation, in order to forestall the takeover by the plaintiffs of the aforenamed corporation, surreptitiously met and elected Ricardo L. Gamboa and
Honorio de la Rama as president and vice-president of the corporation,
respectively.
Thereafter passed a resolution authorizing the sale of the 823 unissued
shares of the corporation to the defendants, Ricardo L. Gamboa, Lydia R.
Gamboa, Honorio de la Rama, Ramon de la Rama, Paz R. Battistuzzi Eduardo
de la Rama, and Mercedes R. Borromeo, at par value, after which the
defendants Honorio de la Rama, Lydia de la Rama-Gamboa, and
EnzoBattistuzzi were elected to the board of directors of the corporation; that
the sale of the unissued 823 shares of stock of the corporation was in
violation of the plaintiffs' and pre-emptive rights and made without the
approval of the board of directors representing 2/3 of the outstanding capital

Page 421 of 1072

stock, and is in disregard of the strictest relation of trust existing between


the defendants, as stockholders thereof; and that the defendants Lydia de la
Rama-Gamboa, Honorio de la Rama, and Enzo Battistuzzi were not legally
elected to the board of directors of the said corporation and has unlawfully
usurped or intruded into said office to the prejudice of the plaintiffs.
Private respondents, entered into a compromise agreement with the
defendants Ramon de la Rama, Paz de la Rama Battistuzzi and Enzo
Battistuzzi , whereby the contracting parties withdrew their respective claims
against each other and the aforenamed defendants waived and transferred
their rights and interests over the questioned 823 shares of stock in favor of
the plaintiffs.
ISSUE/S:
Whether or not a derivative suit is proper in this case
RULING:
The petitioners further contend that the proper remedy of the plaintiffs
would be to institute a derivative suit against the petitioners in the name of
the corporation in order to secure a binding relief after exhausting all the
possible remedies available within the corporation.
An individual stockholder is permitted to institute a derivative suit on
behalf of the corporation wherein he holds stock in order to protect or
vindicate corporate rights, whenever the officials of the corporation refuse to
sue, or are the ones to be sued or hold the control of the corporation. In such
actions, the suing stockholder is regarded as a nominal party, with the
corporation as the real party in interest. In the case at bar, however, the
plaintiffs are alleging and vindicating their own individual interests or
prejudice, and not that of the corporation. At any rate, it is yet too early in
the proceedings since the issues have not been joined. Besides, misjoinder of
parties is not a ground to dismiss an action.

Page 422 of 1072

TOPIC: DERIVATICE SUIT: REMEDIES TO ENFORCE PERSONAL LIABILITY


CATALINA R. REYES
vs.
HON. BIENVENIDO A. TAN, as Judge of the Court of First Instance of
Manila, Branch XIII and FRANCISCA R. JUSTINIANI
G.R. No.L-16982.September 30, 1961
3 SCRA 198
FACTS:
The corporation, Roxas-Kalaw Textile Mills, Inc., was organized on June
5, 1954 by defendants Cesar K. Roxas, Adelia K. Roxas, Benjamin M. Roxas,
Jose Ma. Barcelona and Morris Wilson, for and on behalf of the following
primary principals with the following shareholdings: Adelia K. Roxas, 1200
Class A shares; I. Sherman, 900 Class A shares; Robert W. Born, 450 Class A
shares and Morris Wilson, 450 Class A shares; that the plaintiff holds both
Class A and Class B shares and number and value thereof, the Board of
Directors approved a resolution designating one Dayaram as co-manager
with the specific understanding that he was to act as defendant
WadhumalDalamal's designee, Morris Wilson was likewise designated as comanager with responsibilities for the management of the factory only, that
an office in New York was opened for the purpose of supervising purchases,
which purchases must have the unanimous agreement of Cesar K. Roxas,
New York resident member of the board of directors, Robert Born and
Wadhumal Dalamal or their respective representatives; that several
purchases aggregating $289,678.86 were made in New York for raw
materials such as greige cloth, rayon and grey goods for the textile mill and
shipped to the Philippines, which shipment were found out to consist not of
raw materials but already finished products, such as, West Point Khaki rayon
suiting materials dyed in the piece, finished rayon tafetta in cubes, cotton
eyelets, etc., for which reasons the Central Bank of the Philippines stopped
all dollar allocations for raw materials for the corporation which necessarily

Page 423 of 1072

led to the paralyzation of the operation of the textile mill and its business;
that the supplier of the aforesaid finished goods was the United Commercial
Company of New York in which defendant Dalamal had interests and the
letter of credit for said goods were guaranteed by the Indian Commercial
Company and the Indian Traders in which firms defendant Dalamal likewise
held interests; that the resale of the finished goods was the business of the
Indian Commercial Company of Manila, which company could not obtain
dollar allocations for importations of finished goods under the Central Bank
regulations; that plaintiff and some members of the board of directors urged
defendants to proceed against Dalamal, exposing his offense to the Central
Bank, and to initiate suit against Dalamal for his fraud against the
corporation; that defendants refused to proceed against Dalamal and instead
continued to deal with the Indian Commercial Company to the damage and
prejudice of the corporation.
ISSUE/S:
Whether or not derivative suit is proper in this case
RULING:
It is well settled in this jurisdiction that where corporate directors are
guilty of a breach of trust not of mere error of judgment or abuse of
discretion and intracorporate remedy is futile or useless, a stockholder
may institute a suit in behalf of himself and other stockholders and for the
benefit of the corporation, to bring about a redress of the wrong inflicted
directly upon the corporation and indirectly upon the stockholders.
The claim that respondent Justiniani did not take steps to remedy the
illegal importation for a period of two years is also without merit. During that
period of time respondent had the right to assume and expect that the
directors would remedy the anomalous situation of the corporation brought
about by their own wrong doing. Only after such period of time had elapsed
could respondent conclude that the directors were remiss in their duty to
protect the corporation property and business.
The directors permitted the fraudulent transaction to go unpunished
and nothing appears to have been done to remove the erring purchasing
managers. In a way the appointment of a receiver may have been thought of
by the court below so that the dollar allocation for raw material may be
revived and the textile mill placed on an operating basis. It is possible that if
a receiver in which the Central Bank may have confidence is appointed, the
dollar allocation for raw material may be restored. Claim is made that if a
receiver is appointed, the Philippine National Bank to which the corporation
owes considerable sums of money might be led to foreclose the mortgage.
Precisely the appointment of a receiver in whom the bank may have had
confidence might rehabilitate the business and bring a restoration of the
dollar allocation much needed for raw material and an improvement in the

Page 424 of 1072

business and assets the corporation, thus insuring the collection of the
bank's loan.
Considering the above circumstances we are led to agree with the
judge below that the appointment of a receiver was not only expedient but
also necessary to restore the faith and confidence of the Central Bank
authorities in the administration of the affairs of the corporation, thus
ultimately leading to a restoration of the dollar allocation so essential to the
operation of the textile mills. The first assignment of error is, therefore,
overruled.

TOPIC: DERIVATICE SUIT: REMEDIES TO ENFORCE PERSONAL LIABILITY


CANDIDO PASCUAL
vs.
EUGENIO DEL SAZ OROZCO, ET AL.
G.R. No. L-5174. March 17, 1911
FACTS:
That during the years 1903, 1904, 1905, and 1907 the defendants and
appellees, without the knowledge, consent, or acquiescence of the
stockholders, deducted their respective compensation from the gross income
instead of from the net profits of the bank, thereby defrauding the bank and
its stockholders of approximately P20,000 per annum; that though due
demands has been made upon them therefor, defendants refuse to refund to
the bank the sums so misappropriated, or any part thereof; that defendants
constitute a majority of the present board of directors of the bank, who alone
can authorize an action against them in the name of the corporation, and
that prior to the filing of the present suit plaintiff exhausted every remedy in
the premises within this banking corporation.
The second cause of action sets forth that defendants' and appellees'
immediate predecessors in office in this bank during the years 1899, 1900,
1901, and 1902, committed the same illegality as to their compensation as is
charged against the defendants themselves; that in the four years
immediately following the year 1902, the defendants and appellees were the
only officials or representatives of the bank who could and should investigate
and take action in regard to the sums of money thus fraudulently
appropriated by their predecessors; that they were the only persons
interested in the bank who knew of the fraudulent appropriation by their
predecessors; that they wholly neglected to take any action in the premises
or inform the stockholders thereof; that due demand has been made upon
defendants to reimburse the bank for this loss; that the bank itself can not
bring an action in its own name against the defendants and appellees, for

Page 425 of 1072

the reason already stated, and that there remains no remedy within the
corporation itself.
ISSUE/S:
Whether or not plaintiff by reason of the fact that he is a
stockholder in the bank has a right to maintain a suit for and on
behalf of the bank
RULING:
The right of individual stockholders to maintain suits for and on behalf
of the corporation was denied until within a comparatively short time, but his
right is now no longer doubted.
Notwithstanding this fact, however, that it was the duty and right of
the corporation to bring suit remedy these wrongs, it gradually became
apparent that frequently the corporation was helpless and unable to institute
the suit. It was found, where the guilty parties themselves controlled the
directors and also a majority of the stock, that the corporation was in their
power, was unable to institute suit, and that the minority of the stockholders
were being defrauded of their rights and were without remedy. Where
corporate directors have committed a breach of trust either by their frauds,
ultra vires acts, or negligence, and the corporation is unable or unwilling to
institute suit to remedy the wrong, a single stockholder may institute that
suit, suing on behalf of himself and other stockholders and for the benefit of
the corporation, to bring about a redress of the wrong done directly to the
corporation and indirectly to the stockholders.
So it is clear that the plaintiff, by reason of the fact that he is a
stockholder in the bank has a right to maintain a suit for and on behalf of the
bank, but the extent of such a right must depend upon when, how, and for
what purpose he acquired the shares which he now owns. In the
determination of these questions we can not see how, if it be true that the
bank is a quasi-public institution, it can affect in any way the final result.

Page 426 of 1072

TOPIC: POWERS OF CORPORATIONS THEORY OF SPECIAL CAPACITY VS.


THEORY OF GENERAL CAPACITIES
ACEBEDO OPTICAL COMPANY, INC.
vs.
COURT OF APPEALS, et al
G.R. No. 100152. March 31, 2000
FACTS:
Petitioner applied with the Office of the City Mayor of Iligan for a
business permit. After consideration of petitioners application and the
opposition interposed thereto by local optometrists, respondent City Mayor
issued Business Permit No. 5342 subject to the following conditions:
1. Since it is a corporation, Acebedo cannot put up an optical
clinic but only a commercial store;
2. Acebedo cannot examine and/or prescribe reading and similar
optical glasses for patients, because these are functions of
optical clinics;
3. Acebedo cannot sell reading and similar eyeglasses without a
prescription having first been made by an independent
optometrist (not its employee) or independent optical clinic.
Acebedo can only sell directly to the public, without need of a
prescription, Ray-Ban and similar eyeglasses;
4. Acebedo cannot advertise optical lenses and eyeglasses, but
can advertise Ray-Ban and similar glasses and frames;
5. Acebedo is allowed to grind lenses but only upon the
prescription of an independent optometrist.
On December 5, 1988, private respondent Samahan ng Optometrist Sa
Pilipinas (SOPI), Iligan Chapter, through its Acting President, Dr. Frances B.
Apostol, lodged a complaint against the petitioner before the Office of the

Page 427 of 1072

City Mayor, alleging that Acebedo had violated the conditions set forth in its
business permit and requesting the cancellation and/or revocation of such
permit.
ISSUE/S:
Whether or not Acebedos act of hiring optometrists is
considered the practice by the corporation itself of the profession
of optometry.
RULING:
No.
Courts have distinguished between optometry as a learned profession
in the category of law and medicine, and optometry as a mechanical art.
And, insofar as the courts regard optometry as merely a mechanical art, they
have tended to find nothing objectionable in the making and selling of
eyeglasses, spectacles and lenses by corporations so long as the patient is
actually examined and prescribed for by a qualified practitioner.
The primary purpose of the statute regulating the practice of
optometry is to insure that optometrical services are to be rendered by
competent and licensed persons in order to protect the health and physical
welfare of the people from the dangers engendered by unlicensed practice.
Such purpose may be fully accomplished although the person rendering the
service is employed by a corporation.
Furthermore, it was ruled that the employment of a qualified
optometrist by a corporation is not against public policy. Unless prohibited by
statutes, a corporation has all the contractual rights that an individual
hasand it does not become the practice of medicine or optometry because of
the presence of a physician or optometrist. The manufacturing, selling,
trading and bartering of eyeglasses and spectacles as articles of
merchandise do not constitute the practice of optometry.
Moreover, distinction must be made between the grant of a license or
permit to do business and the issuance of a license to engage in the practice
of a particular profession. The first is usually granted by the local authorities
and the second is issued by the Board or Commission tasked to regulate the
particular profession. A business permit authorizes the person, natural or
otherwise, to engage in business or some form of commercial activity. A
professional license, on the other hand, is the grant of authority to a natural
person to engage in the practice or exercise of his or her profession.
What is sought by petitioner from respondent City Mayor is a permit
engage in the business of running an optical shop. It does not purport
seek a license to engage in the practice of optometry as a corporate body
entity, although it does have in its employ, persons who are duly licensed

to
to
or
to

Page 428 of 1072

practice optometry by the Board of Examiners in Optometry. The objective of


the imposition of subject conditions on petitioners business permit could be
attained by requiring the optometrists in petitioners employ to produce a
valid certificate of registration as optometrist, from the Board of Examiners
in Optometry. A business permit is issued primarily to regulate the conduct of
business and the City Mayor cannot, through the issuance of such permit,
regulate the practice of a profession, like that of optometry. Such a function
is within the exclusive domain of the administrative agency specifically
empowered by law to supervise the profession, in this case the Professional
Regulations Commission and the Board of Examiners in Optometry.

TOPIC: EXPRESS, IMPLIED AND INCIDENTAL POWERS


PILIPINAS LOAN COMPANY, INC.
vs.
SEC AND FILIPINAS PAWNSHOP, INC.
G.R. No. 104720. April 4, 2001
356 SCRA 193
FACTS:
Private respondent Filipinas Pawnshop, Inc. is a duly organized
corporation registered with the SEC. The AI of private respondent states that
its primary purpose is to extend loans at legal interest on the security of
either personal properties or on the security of real properties, and to finance
installment sales of motor vehicles, home appliances and other chattels.
Petitioner is a lending corporation duly registered with the SEC. On
September 11, 1990, private respondent filed a complaint against petitioner.
The complaint alleged that petitioner, contrary to the restriction set by the
Commission, has been operating and doing business as a pawnbroker,
pawnshop or "sanglaan" in the same neighborhood where private respondent
has had its own pawnshop for 30 years in violation of its primary purpose
and without the imprimatur of the Central Bank to engage in the pawnshop
business thereby causing unjust and unfair competition with private
respondent.
ISSUE/S:
Whether or not private respondent can engage in pawnbroking.
RULING:
No.

Page 429 of 1072

The Court held that a corporation, under the Corporation Code, has
only such powers as are expressly granted to it by law and by its articles of
incorporation,those which may be incidental to such conferred powers, those
reasonably necessary to accomplish its purposes and those which may be
incident to its existence. In the case at bar, the limit of the powers of
petitioner as a corporation is very clear, it is categorically prohibited from
"engaging in pawnbroking as defined under PD 114". Hence, in determining
what constitutes pawnbrokerage, the relevant law to consider is PD 114. This
reference to PD 114 is also in line with Article 2123 of the Civil Code that
states that: "Art. 2123. With regard to pawnshops and other establishments,
which are engaged in making loans secured by pledges, the special laws and
regulations concerning them shall be observed, and subsidiarily, the
provisions of this Title."
Moreover, a careful examination and analysis of the records of this
case indicates that petitioner has indeed engaged in the business of
pawnbroking. It is not argued that petitioner does lend money on the security
of personal property. What must be observed though are the very prominent
words "SANGLAAN" found on its billboards which cannot but give the
impression to the public that its establishment is more of a pawnshop than a
lending institution servicing different kinds of loans. The word "SANGLAAN",
especially in big cities, have come to be associated with pawnshops and it
denotes the idea of a place where one presents personal property for a loan,
which is the exclusive domain of a pawnshop. Thus, the use of such word by
petitioner was more calculated to attract customers who will acquire loans on
the security of personal properties alone. That this activity is in fact
undertaken can be readily deduced from the graphic and unmistakable setup of petitioners place of business which is a picture of a typical pawnshop
where a person transacts through small glass openings labeled sangla and
tubos. Moreover, the supposed "promissory note" evidencing a customers
transaction with petitioner, is more of a pawnticket than what it represents.

Page 430 of 1072

TOPIC: EXPRESS, IMPLIED AND INCIDENTAL POWERS


LUNETA MOTOR COMPANY
vs
A.D. SANTOS, INC., ET AL.
G.R. No. L-17716. July 31, 1962
5 SCRA 809
FACTS:
On December 31, 1941, to secure payment of a loan evidenced by a
promissory note executed by Nicolas Concepcion in favor of petitioner,
Concepcion executed a chattel mortgage covering the above mentioned
certificate in favor of petitioner. The certificate was later sold to Francisco
Benitez, Jr., who resold it to Rodi Taxicab Company. On October 10, 1953
petitioner filed an action to foreclose the chattel mortgage executed in its
favor by Concepcion in view of the failure of the latter and his guarantor,
Placido Esteban, to pay their overdue account.
On June 9, 1958, the CFI of Manila adjudged Concepcion indebted to
petitioner and ordered that the certificate of public convenience subject
matter of the chattel mortgage be sold at public auction in accordance with
law. Accordingly, on March 3, 1959 said certificate was sold at public auction
to petitioner, and six days thereafter the Sheriff of the City of Manila issued
in its favor the corresponding certificate of sale. Thereupon petitioner filed
the application for the approval of the sale. Respondent A.D. Santos, Inc.
opposed petitioner's application, filed a motion to dismiss based on the
ground that under the petitioner's Articles of Incorporation, it was not
authorized to engage in the taxicab business or operate as a common
carrier.

Page 431 of 1072

The respondent Commission held that under petitioner's articles of


incorporation it had no authority to engage in the taxicab business or
operate as a common carrier, and that, is a result, it could not acquire by
purchase the certificate of public convenience referred to above. Hence, the
present appeal interposed by petitioner who claims that, in accordance with
the Corporation Law and its articles of incorporation, it can acquire by
purchase the certificate of public convenience in question, maintaining
inferentially that, after acquiring said certificate, it could make use of it by
operating a taxicab business or operate is a common carrier by land.
ISSUE/S:
Whether or not under the Corporation Law and petitioner's AI, it
may acquire by purchase a certificate of public convenience and
after its acquisition; it may hold the certificate and thereunder
operate as a common carrier by land.
RULING:
No.
It is not denied that under Section 13 (5) of the Corporation Law, a
corporation created thereunder may purchase, hold, etc., and otherwise deal
in such real and personal property is the purpose for which the corporation
was formed may permit, and the transaction of its lawful business may
reasonably and necessarily require. The issue here is precisely whether the
purpose for which petitioner was organized and the transaction of its lawful
business reasonably and necessarily require the purchase and holding by it
of a certificate of public convenience like the one in question and thus give it
additional authority to operate thereunder as a common carrier by land.
Petitioner claims in this regard that its corporate purposes are to carry
on a general mercantile and commercial business, etc., and that it is
authorized in its articles of incorporation to operate and otherwise deal in
and concerning automobiles and automobile accessories' business in all its
multifarious ramification and to operate, etc., and otherwise dispose of
vessels and boats, etc., and to own and operate steamship and sailing ships
and other floating craft and deal in the same and engage in the Philippine
Islands and elsewhere in the transportation of persons, merchandise and
chattels by water; all this incidental to the transportation of automobiles
The Court found nothing in the legal provision and the provisions of
petitioner's AI relied upon that could justify petitioner's contention in this
case. To the contrary, they are precisely the best evidence that it has no
authority at all to engage in the business of land transportation and operate
a taxicab service. That it may operate and otherwise deal in automobiles and
automobile accessories; that it may engage in the transportation of
persons by water does not mean that it may engage in the business of land

Page 432 of 1072

transportation an entirely different line of business. If it could not thus


engage in the line of business, it follows that it may not acquire an certificate
of public convenience to operate a taxicab service, such as the one in
question, because such acquisition would be without purpose and would
have no necessary connection with petitioner's legitimate business.

TOPIC: EXPRESS, IMPLIED AND INCIDENTAL POWERS


TERESA ELECTRIC AND POWER CO., INC., petitioner,
vs.
PUBLIC SERVICE COMMISSION and FILIPINAS CEMENT
CORPORATION, respondents.
G. R. No. L- 21804. September 25, 1967
FACTS:
The Teresa Electric Light and Power Co., Inc. is a domestic corporation
operating an electric plant in Teresa, Rizal, under a subsisting certificate of
public convenience and necessity issued on June 2, 1960, while the
respondent Filipinas is likewise a domestic corporation engaged in the
manufacture and sale of cement.
On May 24, 1962 Filipinas filed an application with the Public Service
Commission for a certificate of public convenience to install, maintain and
operate an electric plant in sitio Kaysapon of barrio Pamanaan, municipality
of Teresa, Rizal, for the purpose of supplying electric power and light to its
cement factory and its employees living within its compound.
Petitioner filed its written opposition alleging:
1. that it is the duly authorized operator of an electric light,
heat and power service in Teresa, Rizal;
2. that Filipinas is not authorized by its articles of
incorporation to operate an electric plant;
3. that the Municipal Council of Teresa had not authorized it
either to operate the proposed service;

Page 433 of 1072

4. that it is willing to supply Filipinas' need for electricity; and


5. that Filipinas' principal business does not come within the
jurisdiction of the respondent Commission.
ISSUE/S:
Whether under its articles of incorporation Filipinas is authorized
to operate and maintain an electric plant.
RULING:
Yes.
It appears that the Articles of Incorporation of Filipinas (paragraph 7)
provide for authority to secure from any governmental, state, municipality, or
provincial, city or other authority, and to utilize and dispose of in any lawful
manner, rights, powers, privileges, franchises and concessions obviously
necessary or at least related to the operation of its cement factory. Moreover,
said Articles of Incorporation also provide that the corporation may generally
perform any and all acts connected with the business of manufacturing
portland cement or arising therefrom or incidental thereto.
It can not be denied that the operation of an electric light, heat and
power plant is necessarily connected with the business of manufacturing
cement. If in the modern world where we live today electricity is virtually a
necessity for our daily needs, it is more so in the case of industries like the
manufacture of cement.
Upon the last question, petitioner claims that Filipinas is not entitled to
a certificate of public convenience to maintain and operate electric service
for its cement plant and its employees because petitioner is operating an
electric plant in the same municipality where Filipinas cement plant is
located.
While it is true that operators of public convenience and service
deserve some protection from unnecessary or unlawful competition, yet the
rule is that nobody has any exclusive right to secure a franchise or a
certificate of public convenience. Above any or all considerations, the grant
of franchises and certificates of public convenience and service should be
guided by public service and interest; the latter are the primordial
considerations to be taken into account.
Moreover, it has been established in this case that petitioner was in no
condition to supply the power needs of Filipinas, because its load capacity
was only 200 kilowatts while Filipinas was in need of 6,000 Kilowatts power
to operate its cement factory.

Page 434 of 1072

TOPIC: EXPRESS, IMPLIED AND INCIDENTAL POWERS


ANTHONY POWERSet. al
VS.
DONALD I. MARSHALL et. Al
G.R. No. L-48064. May 09, 1988
FACTS:
On July 16, 1975, plaintiffs, all associate members of the International
School, Inc., filed an action for injunction in the CFI of Rizal, against the ten
(10) members of the Board of Trustees of the school. The suit was
precipitated by a letter dated May 19, 1975 which Donald I. Marshall,
president of the Board of Trustees of the International School in Makati,
Metro-Manila, addressed to the parents of the students, giving notice that
the Board of Trustees had decided to embark on a program to construct new
buildings and remodel existing ones to accommodate the increasing
enrollment in the school, and that it was necessary for the school to raise
P35, 000,000.00 for this purpose. The Board intended to raise the needed
funds primarily through subscriptions to capital notes and prepayment
certificates, and any deficiency from these sources would be covered by
collecting a so-called "development fee" of P2,625 from each enrollee
starting with the school year 1975-1976 and continuing up to the school year
1986-1987.
The school superintendent, Dr. Max Snyder, acting under instructions
from the Board of Trustees, wrote a letter to the parents of returning
students, enclosing an Application for Admission which specifically advised

Page 435 of 1072

that the payment of the development fee was a pre-requisite for reenrollment.
The plaintiffs protested against the imposition of the development fee.
On June 18, 1975 they requested the Board of Trustees to suspend the
implementation of the requirement of payment. On July 16, 1975 the
plaintiffs filed a complaint for injunction against the school. On July 17, 1975,
the trial court issued an order temporarily restraining the defendants or their
authorized representatives and agents from executing and/or enforcing the
development program.The Court dismissed the complaint for lack of valid
cause of action, and dissolved the restraining order of July 17, 1975. Plaintiffs
appealed to CA.
ISSUE/S:
Whether or not the Board of Trustees has the power to
implement the development plan.
RULING:
Yes.
The Court held that the by-laws of the school authorized the BOT to
exercise such powers which may be lawfully exercised by the corporation,
subject to applicable laws, the Articles of Incorporation and the by-lays.
The law authorizes the BOT to determine the amount of fees which
may reasonably be imposed to maintain or conform to the school standard of
education upon consultation and approval of the Secretary of Education.
Aside from the authority emanating from both the law and the by-laws,
the development plan and the consequential increase of school fees had
been approved by the schools Board and by the Secretary of Education.
Thus, the questioned act is a valid exercise of Board power.

Page 436 of 1072

TOPIC:POWER TO HAVE/USE CORPORATE NAME OR SEAL


LAUREANO INVESTMENT & DEVELOPMENT CORPORATION
VS.
COURT OF APPEALS AND BORMAHECO, INC.
G.R. NO. 100468 MAY 6, 1997
FACTS:
Spouses Reynaldo Laureano and Florence Laureano are majority
stockholders of petitioner Corporation who entered into a series of loan and
credit transactions with Philippine National Cooperative Bank (PNCB). To
secure payment of the loans, they executed Deeds of Real Estate Mortgage
however, in view of their failure to pay their indebtedness, PNCB applied for
extrajudicial foreclosure of the real estate mortgages. The bank was the
purchaser of the properties in question in the foreclosure sale and titles
thereof were consolidated in PNCB's name.
Private respondent Bormaheco, Inc. became the successor of the
obligations and liabilities of PNCB over subject lots by virtue of a Deed of
Sale/Assignment wherein Bormaheco bought from PNCB under a bulk sale
114 titled and untitled properties including the two parcels of land in
question, formerly registered in the name of the Laureano spouses.
Bormaheco filed an Ex-Parte Petition for the Issuance of Writ of
Possession of the two parcels of land of petitioner. Petitioner Corporation filed
its Motion for Intervention and to Admit Attached Complaint in Intervention in
said case. However, it sued under the name Lideco Corporation and not

Page 437 of 1072

under Laureano Investment and Development Corporation. After an


exchange of pleadings, respondent Court ruled that petitioner corporation
(Lideco Corporation) has no personality to file the instant motion because it
is separate and distinct from Laureano Investment and Development
Corporation. Thus, it has no interest in the properties in question. A MR was
filed but it was denied. Hence this petition.
ISSUE/S:
Whether or not petitioner corporation has a legal personality to
sue under the name Lideco Corporation.
RULING:
Article 36 of the Corporation Code similarly provides: Corporate powers
and capacity. Every corporation incorporated under this Code has the power
and capacity:
1. To sue and be sued in its corporate name;
As the trial and appellate courts have held, "Lideco
Corporation" had no personality to intervene since it had
not been duly registered as a corporation. If petitioner
legally and truly wanted to intervene, it should have used
its corporate name as the law requires and not another
name which it had not registered. Indeed, as the
Respondent Court found, nowhere in the motion for
intervention and complaint in intervention does it appear
that "Lideco Corporation" stands for Laureano Investment
and Development Corporation. Bormaheco, Inc., thus, was
not estopped from questioning the juridical personality of
"Lideco Corporation," even after the trial court had allowed
it to intervene in the case.

Page 438 of 1072

TOPIC:POWER TO SUE AND BE SUED


TAM WING TAK, petitioner,
vs.
HON. RAMON P. MAKASIAR (in his Capacity as Presiding Judge of the
Regional Trial Court of Manila, Branch 35) and ZENON DE GUIA (in
his capacity as Chief State Prosecutor), respondents.
G.R. No. 122452. January 29, 2001
FACTS:
On November 11, 1992, petitioner, in his capacity as director of
Concord-World Properties, Inc., a domestic corporation, filed an affidavitcomplaint with the Quezon City Prosecutors Office, charging Vic Ang Siong
with violation of B.P. Blg. 22. The complaint alleged that a check issued by
Vic Ang Siong in favor of Concord, was dishonored when presented for
encashment.
Vic Ang Siong sought the dismissal of the case on two grounds: First,
that petitioner had no authority to file the case on behalf of Concord, the
payee of the dishonored check, since the firms board of directors had not
empowered him to act on its behalf. Second, he and Concord had already
agreed to amicably settle the issue after he made a partial payment of
P19,000,000.00 on the dishonored check.

Page 439 of 1072

The City Prosecutor dismissed the complaint refused to file the


Information.
ISSUE/S:
Whether or not the petitioner had the personality to file the case
on behalf of Concord.
RULING:
It is not disputed in the instant case that Concord, a domestic
corporation, was the payee of the bum check, not petitioner. Therefore, it is
Concord, as payee of the bounced check, which is the injured party. Since
petitioner was neither a payee nor a holder of the bad check, he had neither
the personality to sue nor a cause of action against Vic AngSiong. Under
Section 36 of the Corporation Code, read in relation to Section 23, it is clear
that where a corporation is an injured party, its power to sue is lodged with
its board of directors or turstees. Note that petitioner failed to show any
proof that he was authorized or deputized or granted specific powers by
Concord's board of director to sue Victor And Siong for and on behalf of the
firm. Clearly, petitioner as a minority stockholder and member of the board
of directors had no such power or authority to sue on Concord's behalf. Nor
can we uphold his act as a derivative suit. For a derivative suit to prosper, it
is required that the minority stockholder suing for and on behalf of the
corporation must allege in his complaint that he is suing on a derivative
cause of action on behalf of the corporation and all other stockholders
similarly situated who may wish to join him in the suit. There is no showing
that petitioner has complied with the foregoing requisites. It is obvious that
petitioner has not shown any clear legal right which would warrant the
overturning of the decision of public respondents to dismiss the complaint
against Vic AngSiong. A public prosecutor, by the nature of his office, is
under no compulsion to file a criminal information where no clear legal
justification has been shown, and no sufficient evidence of guilt nor prima
facie case has been presented by the petitioner. No reversible error may be
attributed to the court a quo when it dismissed petitioner's special civil
action for mandamus.

Page 440 of 1072

TOPIC:POWER TO SUE AND BE SUED


NORA A. BITONG, petitioner,
vs.
COURT OF APPEALS (FIFTH DIVISION), EUGENIA D. APOSTOL, JOSE A.
APOSTOL, MR. & MS. PUBLISHING CO., LETTY J. MAGSANOC, AND
ADORACION G. NUYDA, respondents.
G.R. No. 123553. July 13, 1998
FACTS:
Alleging before the SEC that petitioner had been the Treasurer and a
Member of the Board of Directors of Mr. & Ms. from the time it was
incorporated on 29 October 1976 to 11 April 1989, and was the registered
owner of 1,000 shares of stock out of the 4,088 total outstanding shares,
petitioner complained of irregularities committed from 1983 to 1987 by
Eugenia D. Apostol, President and Chairperson of the Board of Directors.
Petitioner claimed that except for the sale of the name Philippine Inquirer to
Philippine Daily Inquirer (PDI hereafter) all other transactions and
agreements entered into byMr. & Ms. with PDI were not supported by any
bond and/or stockholders' resolution. And, upon instructions of Eugenia D.
Apostol, Mr. & Ms. made several cash advances to PDI on various occasions
amounting to P3.276 million. On some of these borrowings PDI paid no
interest whatsoever. Despite the fact that the advances made by Mr. & Ms. to
PDI were booked as advances to an affiliate, there existed no board or

Page 441 of 1072

stockholders' resolution, contract nor any other document which could


legally authorize the creation of and support to an affiliate.
Petitioner further alleged that respondents Eugenia and Jose Apostol
were stockholders, directors and officers in both Mr. & Ms. and PDI. In fact on
2 May 1986 respondents Eugenia D. Apostol, Leticia J. Magsanoc and
Adoracion G. Nuyda subscribed to PDI shares of stock at P50,000.00 each or
a total of P150,000.00. The stock subscriptions were paid for by Mr. & Ms.
and initially treated, as receivables from officers and employees. But, no
payments were ever received from respondents, Magsanoc and Nuyda.
ISSUE/S:
Whether or not a stockholder may institute in behalf of himself
and other stockholders and for the benefit of the corporation.
RULING:
It is well settled in this jurisdiction that where corporate directors are
guilty of a breach of trust, not of mere error of judgment or abuse of
discretion, and intra corporate remedy is futile or useless, a stockholder may
institute a suit in behalf of himself and other stockholders and for the benefit
of the corporation, to bring about a redress of the wrong inflicted directly
upon the corporation and indirectly upon the stockholders. The stockholder's
right to institute a derivative suit is not based on any express provision of
The Corporation Code but is impliedly recognized when the law makes
corporate directors or officers liable for damages suffered by the corporation
and its stockholders for violation of their fiduciary duties.
Hence, a stockholder may sue for mismanagement, waste or
dissipation of corporate assets because of a special injury to him for which
he is otherwise without redress. In effect, the suit is an action for specific
performance of an obligation owed by the corporation to the stockholders to
assist its rights of action when the corporation has been put in default by the
wrongful refusal of the directors or management to make suitable measures
for its protection.
The basis of a stockholder's suit is always one in equity. However, it
cannot prosper without first complying with the legal requisites for its
institution. The most important of these is the bona fide ownership by a
stockholder of a stock in his own right at the time of the transaction
complained of which invests him with standing to institute a derivative action
for the benefit of the corporation.
WHEREFORE, the petition is DENIED. The 31 August 1995 Decision of
the Court of Appeals dismissing the complaint of petitioner Nora A. Bitong in
CA-G.R. No. SP 33291, and granting the petition for certiorari and prohibition
filed by respondent Edgardo U. Espiritu as well as annulling the 5 November

Page 442 of 1072

1993, 24 January 1993 and 18 February 1994 Orders of the SEC En Banc in
CA-G.R. No. SP 33873, is AFFIRMED. Costs against petitioner.

TOPIC:POWER TO SUE AND BE SUED


SPECIAL SERVICE CORPORATION
VS.
CENTRO LA PAZ, A CHAPTER OF UNION ESPIRITISTA CRISTIANA DE
FILIPINAS, INC.
G.R. NO. L-44100; APRIL 28, 1983
FACTS:
Alejandro Estudillo was one of the registered owners indicated in a
titled land which became the subject of a Writ of Execution to satisfy the
judgement rendered in favor of Petitoner Special Services Corp.
Before the scheduled auction sale of Estudillos rights and interest to
the property, the latter filed a "Motion to Dissolve and/or Cancel the Notice of
Levy" alleging that he and the other registered owners indicated on the title
merely held in trust the properties and improvements thereon in favor of
respondent Centro La Paz (Samahang Espiritista Sa Lunduyang La Paz) a
Chapter of Union Espiritista Cristiana de Filipinas, Inc. (hereinafter referred to
as CENTRO, for brevity), as evidenced by "Acknowledgments.
A thirdy party claim also was filed by CENTRO alleging exclusive
ownership over the subject property the registered owners thereof had
publicly acknowledged their possession of said properties in the concept of
trustees.

Page 443 of 1072

The lower court ruled in favor of Estudillo, and established that


CENTRO is indeed the true owner of the property. The same was affirmed by
the Court of Appeals.
ISSUE/S:
Whether or not Centro La Paz which is merely a Chapter of Union
Espiritista de Filipinas, Inc. has a juridical personality of its own
and may thus have the power to sue.
RULING:
Evident from all the foregoing is that although it was CENTRO that was
actively prosecuting the case, in substance, it was representing the mother
organization, the Union Espiritista Cristiana de Filipinas, Inc., which is the real
party in interest and is itself named in the Complaint. It is an organization
that is duly registered with the Securities and Exchange Commission, and
thus possessed of a juridical personality to sue and be sued.
As found by both the Trial Court and respondent Appellate Court, the
evidence sufficiently establishes that the registered owners of the parcels of
land covered by TCT 51837, all of whom are members of CENTRO, hold the
properties in trust for CENTRO by virtue of the indubitable documents
executed even before the institution of suit. In the same manner that the real
property, registered solely in the name of a husband, can be proven to be
conjugal property with his wife, the fact of registration in the name of
Alejandro Estudillo and others does not bar evidence to show that the
registered owners hold the properties in trust for CENTRO.
Admittedly, the trust was not registered in accordance with section 65
of Act 496 (the former Land Registration Law). The absence of said
registration, however, cannot be taken against CENTRO inasmuch as, if the
public auction sale had actually been held, with petitioner as the successful
buyer, petitioner could not have been considered a purchaser for value and
in good faith at said sale since it had knowledge of CENTRO's claim,
particularly when the latter had filed a third-party-claim with the Sheriff of
Manila before the scheduled auction sale, which knowledge was equivalent
to registration of the several "Acknowledgments" in the Registry of Deeds.
The conclusion follows that inasmuch as Estudillo has no interest in the
properties in question, there is nothing that petitioner can levy upon. The
power of a Court in the execution of its judgment extends only over
properties unquestionably belonging to the judgment debtor.

Page 444 of 1072

TOPIC:POWER TO SUE AND BE SUED


R. TRANSPORT CORPORATION
VS.
COURT OF APPEALS
G.R. NO. 111187 FEBRUARY 1, 1995

FACTS:
On November 22, 1991, a complaint for damages arising from breach
of contract of carriage was filed against petitioner with the RTC. The trial
court upon ex parte motion of private respondent, declared petitioner in
default and appointed a commissioner to receive evidence ex parte.
Petitioner filed a Motion to Dismiss and to Stop Ex Parte Reception of
Evidence. It asserted that it was not properly served with summons and
consequently, the trial court did not acquire jurisdiction over its person. It
argued that none of the officers enumerated in Section 13, Rule 14 of the
Revised Rules of Court (namely, the corporation's president, manager,
secretary, cashier, agent or any of its directors) received any summons. The
trial court denied petitioner's motion and allowed private respondent to
adduce its evidence ex parte. Petitioner filed a motion for reconsideration
giving as an additional ground therefor that summons was served at Sucat,
Paraaque, where its bus terminal was located, and not at its principal office
at No. 4474 Singian Street, Makati, Metro Manila, where its president, general
manager, secretary, agent and directors hold office. Petitioner asked, inter
alia, that the trial court direct "the Clerk of Court to issue another summons
together with a copy of the complaint and serve such summons to the

Page 445 of 1072

President, General Manager, Cashier, or any of its Directors, with offices at


Rizal Towers, 4474 Singian St., Makati, Metro Manila, who are authorized by
law to receive these summons on behalf of the defendant corporation". The
trial court denied petitioner's motion for reconsideration for lack of merit.
Hence, petitioner filed a petition for certiorari with the Court of Appeals
to nullify the above three orders of the trial court. The Court of Appeals
dismissed the petition ruling that the trial court did not commit any grave
abuse of discretion in declaring the petitioner in default and in denying
petitioner's motion for reconsideration. Petitioner moved for reconsideration
of the appellate court's decision, submitting the affidavit of its President to
the effect that its Operations Manager was a certain Roger F. Lemi and not
Cesar Pasquin. The appellate court denied the motion.
ISSUE/S:
Whether or not there was valid service of summons for it to be
liable for the case filed.
RULING:
As a general rule, service of summons must be made on the persons
named in Section 13, Rule 14 of the Revised Rules of Court which provides:
Service upon private domestic corporation or partnership. If the defendant is
a corporation organized under the laws of the Philippines or a partnership
duly registered, service may be made on the president, manager, secretary,
cashier, agent or any of its directors. Thus service on persons other than
those mentioned in said Rule has been held as improper.
The rationale of all rules for service of process on corporations is that
service must be made on a representative so integrated with the corporation
sued as to make it a priori supposable that he will realize his responsibilities
and know what he should do with any legal papers served on him.
Thus, we hold that service of summons on petitioner's Operations
Manager was valid. He is an officer who may be relied upon to appreciate the
importance of the papers served on him. The purpose of Section 13 of Rule
14 was served. The fact that service was made at petitioner's bus terminal at
the address stated in the summons and not at its office in Makati does not
render the service of summons invalid. In Villa Rey Transit, Inc., supra, we
held valid the service of summons made on the corporation's Assistant
General Manager for Operations holding office at the "sub-station" in
Sampaloc, Manila.
Petitioner is engaged in the transportation business, operating over
100 buses. Its central bus terminal is located at Sucat, Paraaque, from
where it conducts the bulk of its business. It was at that terminal where
petitioner's Operations Manager was found and upon whom service was

Page 446 of 1072

made. We distinguish the instant case from First Integrated Bonding &
Insurance Co., Inc. v. Dizon, 125 SCRA 440 (1983), where we held that a
branch manager does not come within the enumeration of Section 13, Rule
14, who are officers whose duties generally pertain to the overall
transportation business of the corporation and not merely to a branch or
department thereof.

TOPIC: POWER TO ACQUIRE, DISPOSE, ENCUMBER PROPERTY


THE DIRECTOR OF LANDS, petitioner,
vs.
THE HONORABLE COURT OF APPEALS and IGLESIA NI CRISTO,
respondents.
G.R. No. L-56613. March 14, 1988
FACTS:
The CFI of Cavite granted Iglesia ni Cristos application for registration
of title. It found that respondent and its predecessors-in-interest had been in
continuous, open and adverse possession of the subject property in the
concept of owner for more than forty years and that the land was not within
any military and naval reservation, nor covered by any kind of public land
application or patent, as it is within the proposed alienable or disposable
block of the proposed LC Project No. 5-A of Amadeo, Cavite.
The Director of Lands appealed the decision of the land registration
court to the CA believing that respondent did not sufficiently Identify the land
in question by reason of its failure to submit the original tracing cloth plan
thereof and that it was disqualified from holding, except by lease, alienable
lands of the public domain under Section 11, Article XIV of the 1973
Constitution.

Page 447 of 1072

The CA affirmed in toto the assailed decision.


ISSUE/S:
Whether or not Iglesia ni Cristo, a corporation sole, may have the
said land registered in its name under the 1973 Constitution.
RULING:
Yes.
The land under consideration was acquired by private respondent from
Aquelina de la Cruz in 1947, who, in turn, acquired by same by purchase
from the Ramos brothers and sisters in 1936.
Under section 48[b] of Commonwealth Act No. 141, as amended,
"those who by themselves or through their predecessors-in-interest have
been in open, continuous, exclusive and notorious possession and occupation
of agricultural lands of the public domain, under a bona fide claim of
acquisition or ownership, for at least thirty years immediately preceding the
filing of the application for confirmation of title except when prevented by
war or force majeure" may apply to the CFI of the province where the land is
located for confirmation of their claims, and the issuance of a certificate of
title therefor, under the Land Registration Act. It further provides that "these
shall be conclusively presumed to have performed all the conditions
essential to a Government grant and shall be entitled to a certificate of title
under the provisions of this chapter."
Taking the year 1936 as the reckoning point, there being no showing as
to when the Ramoses first took possession and occupation of the land in
question, the 30-year period of open, continuous, exclusive and notorious
possession and occupation required by law was completed in 1966. The
completion by private respondent of this statutory 30-year period has dual
significance in the light of Section 48[b] of Commonwealth Act No. 141, as
amended and prevailing jurisprudence:
1. at this point, the land in question ceased by operation
of law to be part of the public domain; and
2. private respondent could have its title thereto confirmed
through the appropriate proceedings as under the
Constitution then in force, private corporations or
associations were not prohibited from acquiring public
lands, but merely prohibited from acquiring, holding or
leasing such type of land in excess of 1,024 hectares.
If in 1966, the land in question was converted ipso jure into private
land, it remained so in 1974 when the registration proceedings were
commenced. This being the case, the prohibition under the 1973 Constitution
would have no application. Otherwise construed, if in 1966, private
respondent could have its title to the land confirmed, then it had acquired a

Page 448 of 1072

vested right thereto, which the 1973 Constitution can neither impair nor
defeat.

TOPIC: POWER TO MAKE DONATIONS


MARIA CLARA PIROVANA ET AL., plaintiffs-appellees,
vs.
THE DE LA RAMA STEAMSHIP CO., defendant-appellant.
G.R. No. L-5377. December 29, 1954
96 Phil 335
FACTS:
Defendant is a corporation duly organized in accordance with law and
with an authorized capital of P500, 000, divided into 5,000 shares, with a par
value of P100 each share. When Enrico Pirovano became the president of the
corporation and at the time of his execution during the Japanese
ccupation,said corporation grew and progressed until it became a multimillion corporation. Thus, the Defendant Corporations Board of Directors
adopted a resolution entitled SPECIAL PAYMENT TO MINORS HEIRS OF THE
LATE ENRICO PIROVANO granting to the Pirovano children the proceeds of
the insurance policies taken on his life by the defendant company. However,
the Corporation assailed the same contending that it is ultra vires.
Plaintiffs herein are the minor children of the late Enrico Pirovano
represented by their mother and judicial guardian Estefania R. Pirovano. They
seek to enforce the aboe stated resolutions adopted by the Board of
Directors and stockholders of the defendant company giving to said minor

Page 449 of 1072

children of the proceeds of the insurance policies taken on the life of their
deceased father Enrico Pirovano with the company as beneficiary.
The Court of First Instance of Rizal declared the donation made by the
defendant in favor of the minor children of the late Enrico Pirovano on the
proceeds of the insurance policies taken on his life valid and binding, and
ordering said defendant to pay to said minor children the sum of
P583,813.59, with interest thereon at the rate of per cent from the date of
filing of the complaint, plus an additional amount equivalent to 20 per cent of
said sum of P538,813.59 as damages by way of attorney's fees and the costs
of action.
ISSUE/S:
Can Defendant Corporation give by way of donation the proceeds
of said insurance policies to the minor children of the late Enrico
Pirovano under the law or its articles of corporation, or is that
donation an ultra vires act?
RULING:
The Supreme Court finds that the corporation was given broad and
almost unlimited powers to carry out the purposes for which it was organized
among them,
1. "To invest and deal with the moneys of the company not
immediately required, in such manner as from time to time may
be determined" and,
2. "to aid in any other manner any person, association, or
corporation of which any obligation or in which any interest is
held by this corporation or in the affairs or prosperity of which
this corporation has a lawful interest."
The world deal is broad enough to include any manner of disposition,
and refers to moneys not immediately required by the corporation, and such
disposition may be made in such manner as from time to time may be
determined by the corporations. The donation in question undoubtedly
comes within the scope of this broad power for it is a fact appearing in the
evidence that the insurance proceeds were not immediately required when
they were given away. In fact, the evidence shows that the corporation
declared a 100 per cent cash dividend, or P2,000,000, and later on another
30 per cent cash dividend. This is clear proof of the solvency of the
corporation. It may be that, as insinuated, Don Esteban wanted to make use
of the insurance money to rehabilitate the central owned by a sister
corporation, known as Hijos de I. de la Rama and Co., Inc., situated in Bago,
Negros Occidental, but this, far from reflecting against the solvency of the De
la Rama company, only shows that the funds were not needed by the
corporation.

Page 450 of 1072

Since it is not contended that the donation under consideration is


illegal, or contrary to any of the express provision of the articles of
incorporation, nor prejudicial to the creditors of the defendant corporation,
we cannot but logically conclude, on the strength of the authorities we have
quoted above, that said donation, even if ultravires in the supposition we
have adverted to, is not void, and if voidable its infirmity has been cured by
ratification and subsequent acts of the defendant corporation. The defendant
corporation, therefore, is now prevented or estopped from contesting the
validity of the donation. This is specially so in this case when the very
directors who conceived the idea of granting said donation are practically the
stockholders themselves, with few nominal exception. This applies to the
new stockholder Jose Cojuangco who acquired his interest after the donation
has been made because of the rule that a "purchaser of shares of stock
cannot avoid ultra vires acts of the corporation authorized by its vendor,
except those done after the purchase" (7 Fletcher, Cyc. Corps. section 3456,
p. 603; Pascual vs. Del Saz Orozco, 19 Phil., 82.) Indeed, how can the
stockholders now pretend to revoke the donation which has been partly
consummated? How can the corporation now set at naught the transfer
made to Mrs. Pirovano of the property in New York, U.S.A., the price of which
was paid by her but of the proceeds of the insurance policies given as
donation. To allow the corporation to undo what it has done would only be
most unfair but would contravene the well-settled doctrine that the defense
of ultra vires cannot be set up or availed of in completed transactions (7
Fletcher, Cyc. Corps. Section 3497, p. 652; 19 C.J.S., 431).
TOPIC: POWER TO INCREASE OR DECREASE CAPITAL STOCK
MADRIGAL & COMPANY, INC., petitioner,
vs.
HON. RONALDO B. ZAMORA, PRESIDENTIAL ASSISTANT FOR LEGAL
AFFAIRS, THE HON. SECRETARY OF LABOR, and MADRIGAL CENTRAL
OFFICE EMPLOYEES UNION, respondents.
G.R. No. L-48237. June 30, 1987
FACTS:
On July 29, 1974, by an alleged resolution of its stockholders, the
petitioner reduced its capital stock from 765,000 shares to 267,366
shares.This was effected through the distribution of the marketable securities
owned by the petitioner to its stockholders in exchange for their shares in an
equivalent amount in the corporation.
On August 22, 1975, by yet another alleged stockholders' action, the
petitioner reduced its authorized capitalization from 267,366 shares to
110,085 shares, again, through the same scheme.

Page 451 of 1072

After the petitioner's failure to sit down with the respondent union, the
latter, on August 28, 1974, commenced Case No. LR-5415 with the National
Labor Relations Commission on a complaint for unfair labor practice. Pending
the resolution of Case No. LR-5415, the petitioner, in a letter dated
November 17, 1975, informed the Secretary of Labor that Rizal Cement Co.,
Inc., "from which it derives income" "as the General Manager or Agent" had
"ceased operating temporarily." "In addition, "because of the desire of the
stockholders to phase out the operations of the Madrigal & Co., Inc. due to
lack of business incentives and prospects, and in order to prevent further
losses,"it had to reduce its capital stock on two occasions "As the situation,
therefore, now stands, the Madrigal & Co., Inc. is without substantial income
to speak of, necessitating a reorganization, by way of retrenchment, of its
employees and operations."The petitioner then requested that it "be allowed
to effect said reorganization gradually considering all the circumstances, by
phasing out in at least three (3) stages, or in a manner the Company deems
just, equitable and convenient to all concerned, about which your good office
will be apprised accordingly." The letter, however, was not verified and
neither was it accompanied by the proper supporting papers. For this reason,
the Department of Labor took no action on the petitioner's request.
ISSUE/S:
Whether or not petitioner can use as a defense the reduction of
its capital stock in the labor case.
RULING:
The Supreme Court agrees with the National Labor Relations
Commission that "the dividends received by the company are corporate
earnings arising from corporate investment."Indeed, as found by the
Commission, the petitioner had entered such earnings in its financial
statements as profits, which it would not have done if they were not in fact
profits.
Moreover, it is incorrect to say that such profits in the form of
dividends are beyond the reach of the petitioner's creditors since the
petitioner had received them as compensation for its management services
in favor of the companies it managed as a shareholder thereof. As such
shareholder, the dividends paid to it were its own money, which may then be
available for wage increments. It is not a case of a corporation distributing
dividends in favor of its stockholders, in which case, such dividends would be
the absolute property of the stockholders and hence, out of reach by
creditors of the corporation. Here, the petitioner was acting as stockholder
itself, and in that case, the right to a share in such dividends, by way of
salary increases, may not be denied its employees.

Page 452 of 1072

Accordingly, this court is convinced that the petitioner's capital


reduction efforts were, to begin with, a subterfuge, a deception as it were, to
camouflage the fact that it had been making profits, and consequently, to
justify the mass layoff in its employee ranks, especially of union members.
They were nothing but a premature and plain distribution of corporate assets
to obviate a just sharing to labor of the vast profits obtained by its joint
efforts with capital through the years.

TOPIC: POWER TO INCREASE OR DECREASE CAPITAL STOCK


PHILIPPINE TRUST COMPANY, as assignee in insolvency of "La
Cooperativa Naval Filipina," plaintiff-appellee,
vs.
MARCIANO RIVERA, defendant-appellant.
G.R. No. L-19761. January 29, 1923
FACTS:
In 1918 the Cooperativa Naval Filipina was duly incorporated under the
laws of the Philippine Islands, with a capital of P100,000, divided into one
thousand shares of a par value of P100 each. Among the incorporators of this
company was numbered the defendant Mariano Rivera, who subscribed for
450 shares representing a value of P45,000, the remainder of the stock being
taken by other persons. The articles of incorporation were duly registered in
the Bureau of Commerce and Industry on October 30 of the same year.

Page 453 of 1072

In the course of time the company became insolvent and went into the
hands of the Philippine Trust Company, as assignee in bankruptcy; and by it
this action was instituted to recover one-half of the stock subscription of the
defendant, which admittedly has never been paid.
The reason given for the failure of the defendant to pay the entire
subscription is, that not long after the Cooperativa Naval Filipina had been
incorporated, a meeting of its stockholders occurred, at which a resolution
was adopted to the effect that the capital should be reduced by 50 per
centum and the subscribers released from the obligation to pay any unpaid
balance of their subscription in excess of 50 per centum of the same. As a
result of this resolution it seems to have been supposed that the subscription
of the various shareholders had been cancelled to the extent stated; and
fully paid certificate were issued to each shareholders for one-half of his
subscription. It does not appear that the formalities prescribed in section 17
of the Corporation Law (Act No. 1459), as amended, relative to the reduction
of capital stock in corporations were observed, and in particular it does not
appear that any certificate was at any time filed in the Bureau of Commerce
and Industry, showing such reduction.
ISSUE/S:
Whether or not defendant was still liable for the unpaid balance
of his subscription.
RULING:
Yes.
It is established doctrine that subscription to the capital of a
corporation constitute a find to which creditors have a right to look for
satisfaction of their claims and that the assignee in insolvency can maintain
an action upon any unpaid stock subscription in order to realize assets for
the payment of its debts. (Velasco vs. Poizat, 37 Phil., 802.) A corporation has
no power to release an original subscriber to its capital stock from the
obligation of paying for his shares, without a valuable consideration for such
release; and as against creditors a reduction of the capital stock can take
place only in the manner an under the conditions prescribed by the statute
or the charter or the articles of incorporation. Moreover, strict compliance
with the statutory regulations is necessary (14 C. J., 498, 620).
In the case before us the resolution releasing the shareholders from
their obligation to pay 50 per centum of their respective subscriptions was
an attempted withdrawal of so much capital from the fund upon which the
company's creditors were entitled ultimately to rely and, having been
effected without compliance with the statutory requirements, was wholly
ineffectual.

Page 454 of 1072

TOPIC:TO DENY PRE-EMPTIVE RIGHT


DATU TAGORANAO BENITO, petitioner,
vs.
SEC & JAMIATUL
PHIL AL ISLAMIA, INC., respondents
G.R. NO. L 56655, JULY 25, 1983
FACTS:
Respondent Corporation filed a certificate of increase of its capita stock
from P200, 000.00 to P1, 000,000.00 for which P 110, 980 worth of shares
were subsequently issued by the corporation from the unissued portion of
the authorized capital stock.
Petitioner filed a petition alleging that the additional issue worth P 110,
980.00 of previously subscribed shares of the corporation was made in
violation of his pre-emptive right to said additional shares and that the
increase in the authorized capital stock on the corporation from P200, 000.00
to P1, 000,000.00 was illegal as there was no notice given to the
stockholders of the meeting wherein the proposed increase was the agenda.

Page 455 of 1072

ISSUE/S:
Whether or not there was a denial of pre-emptive right.

RULING:
No.
Pre-emptive rights are recognized only with respect to new issue of
shares, and not with respect to additional issues of originally authorized
shares as in this case. This is on the theory that when a corporation at its
inception offers its first shares, it is presumed to have presumed to have
offered all of those which it is authorized to issue. An original subscriber is
deemed to have taken his shares knowing that they form a definite
proportionate part of the whole number of authorized shares. When the
shares left unsubscribed are later re-offered, he cannot therefore claim a
dilution of interest.

TOPIC:TO DENY PRE-EMPTIVE RIGHT


PEDRO LOPEZ DEE, petitioner,
vs.
SECURITIES AND EXCHANGE COMMISSION, HEARING OFFICER
EMMANUEL SISON, NAGA TELEPHONE CO., INC., COMMUNICATION
SERVICES, INC., LUCIANO MAGGAY, AUGUSTO FEDERIS, NILDA
RAMOS, FELIPA JAVALERA, DESIDERIO SAAVEDRA, respondents.
G.R. No. L-63922 July 16, 1991
FACTS:
On April 12, 1977, NATELCO entered into a contract with
Communication Services, Inc. (CSI for short) for the "manufacture, supply,
delivery and installation" of telephone equipment. In accordance with this
contract, NATELCO issued 24,000 shares of common stocks to CSI on the
same date as part of the downpayment.

Page 456 of 1072

On May 5, 1979, another 12,000 shares of common stocks were issued


to CSI. In both instances, no prior authorization from the Board of
Communications, now the National Telecommunications Commission, was
secured pursuant to the conditions imposed by the decision in BOC Case NO.
74-84.
On May 19, 1979, the stockholders of the Natelco held their annual
stockholders' meeting to elect their seven directors to their Board of
Directors, for the year 1979-1980. In this election Pedro Lopez Dee was
unseated as Chairman of the Board and President of the Corporation, but was
elected as one of the directors, together with his wife, Amelia Lopez Dee.
In the election, CSI was able to gain control of NATELCO, when the
latter's legal counsel, Atty. Luciano Maggay won a seat in the Board with the
help of CSI. In the reorganization Atty. Maggay became president. Petitioner
Dee having been unseated in the election, filed a petition in the SEC
docketed as SEC Case No. 1748, questioning the validity of the elections of
May 19, 1979 upon the main ground that there was no valid list of
stockholders through which the right to vote could be determined. During the
tenure of the Maggay Board, from June 22, 1979 to March 10, 1980, it did not
reform the contract of April 12, 1977, and entered into another contract with
CSI for the supply and installation of additional equipment but also issued to
CSI 113,800 shares of common stock. These shares were claimed to be
issued by the corporation against the pre-emptive rights of the stockholders.

ISSUE/S:
Whether or not the Natelco stockholders have a right of pre-emption to
the 113, 800 shares.
RULING:
NO.
The questioned issuance of the 113,800 stocks is not invalid even
assuming that it was made without notice to the stockholders as claimed by
the petitioner. The power to issue shares of stocks in a corporation is lodged
in the board of directors and no stockholders meeting is required to consider
it because additional issuance of shares of stocks does not need approval of
the stockholders. Consequently, no pre-emptive right of Natelco stockholders
was violated by the issuance of the 113,800 shares to CSI

Page 457 of 1072

TOPIC:TO DENY PRE-EMPTIVE RIGHT


PRESIDENTIAL COMMISSION ON GOOD GOVERNMENT, petitioner,
vs.
HON. BENJAMIN M. AQUINO, JR., as Presiding Judge, Regional Trial
Court, NCJR Branch LXXII Malabon, Metro Manila, and MARCELO
FIBERGLASS CORPORATION, respondents.
G.R. No. 77816, June 30, 1988
FACTS:
Records show that on June 18, 1982, Edward T. Marcelo as president of
private respondent Marcelo Fiberglass Corporation [MFC] entered into a
Contract to Buy and Sell with the Philippine Navy represented by Rear
Admiral Simeon M. Alejandro, then Flag Officer in Command, for the
construction and delivery by the former of fifty-five 551 units of fiberglass
high-speed patrol boats at P7,200,000 each plus spare parts amounting to
P29,700,000 for a total contract price of P425,700,000. It was stipulated in
the contract that the patrol boats would be delivered within thirty-six [36]

Page 458 of 1072

months from the date the Philippine Navy pays to private respondent the
stipulated down payment of thirty per cent [30%] of the contract price.
On July 28,1983, the Philippine Navy, with the approval of former
President Marcos paid private respondent the amount of P127,710,000
representing the 30% initial down payment stipulated in the contract through
Land Bank of the Philippines Cashier's Check No. 009369 3 in violation of the
contract which required that payment shall be made by confirmed,
irrevocable, divisible letter of credit established by the Philippine Navy in
favor of private respondent.
ISSUE/S:
Whether or not there was a denial of pre-emptive right.
RULING:
NO.
The sequestration of the assets of MFC is in accordance with the
powers and functions of the PCGG. Suffice it to say that the matters involved
in these cases are orders of the PCGG issued in the exercise of its powers
and functions for they involve the sequestration of the assets of private
respondent Marcelo Fiberglass Corporation and Edward T. Marcelo, its
president. The propriety of said sequestration and any incident arising from,
incidental or related to such sequestration is within the exclusive jurisdiction
of the Sandiganbayan.

TOPIC: TO DENY PRE-EMPTIVE RIGHT


REPUBLIC OF THE PHILIPPINES, Petitioner,
vs.
SANDIGANBAYAN (First Division), EDUARDO M. COJUANGCO, JR.,
PHILIPPINE COCONUT PRODUCERS FEDERATION, INC., MARIA CLARA
L. LOBREGAT, and THE CORPORATE SECRETARY OF THE SAN MIGUEL
CORPORATION, Respondents.
G.R. No. 118661, JUNE 30, 1988
FACTS:
This case revolves around the corporations organized and the
investments acquired or funded allegedly from the coconut levy fund. While
it came from levies on the sale of copra or equivalent coconut products
exacted for the most part from coconut farmers, the Fund went or was known
under various names, such as Coconut Consumers Development Fund,

Page 459 of 1072

Coconut Industry Investment Fund and Coconut Industry Stabilization Fund


(CISF). Through the years, a part of the Fund went to various projects, was
converted into different assets or invested. Among the assets allegedly
acquired thru the direct or indirect use of the Fund was a block of San Miguel
Corporation (SMC) shares of stock. This case sought by the PCGG to set aside
the joint petition for approval of a compromise agreement and
settlement interposed by certain corporations involving sequestered SMC
shares of stocks. The said stocks are in the name of the CIIF Corporations,
independent of the transaction involving the contracting parties in the
Compromise Agreement and it appearing further that the said sequestered
SMC shares of stock have not been taken over by the PCGG.
In August 1994, the PCGG and the Government Service Insurance
System (GSIS) entered into a Stock Purchase Agreement in which the PCGG,
thereat styled as "owner" "with clear title" over a block of SMC shares
consisting of 14,519,996 shares of stock the arbitration fee shares and their
stock dividends sold the same to the GSIS for the total consideration of
P1.452 Billion. Cojuangco, Jr. and the COCOFED-Lobregat group, having
learned of the conclusion of the aforesaid Stock Purchase Agreement
opposed and moved for the annulment of the sale.
ISSUE/S:
Whether or not there is a violation of pre-emptive right.
RULING:
NO.
First, the subject 33.1 million SMC shares sold by the apparent owners
thereof, the CIIF Holding Companies, to Soriano III "for himself and as agent
of several persons," which later became the subject of the Compromise
Agreement between the UCPB group and the SMC group are sequestered
shares. They appear to be still sequestered. Petitioner PCGG, no less, admits
the sequestered character of the subject shares.
Second, the sequestered character of the original 5.5 Million transferred
shares necessarily attaches on the fruits or dividends accruing thereon.
These shares are alleged to be part of the alleged ill-gotten wealth which are
property in custodia legis. It is not yet known whether the shares are part of
the alleged ill-gotten wealth of former President Marcos and his cronies. Any
Compromise Agreement concerning these sequestered shares has to be
approved by the Sandiganbayan.

Page 460 of 1072

TOPIC: TO SELL OR OTHERWISE DISPOSE OF ALL OR SUBSTANTIALLY ALL OF


CORPORATE
ASSETS
PHILIPPINE NATIONAL BANK & NATIONAL SUGAR DEVELOPMENT
CORPORATION, petitioners,
vs.
ANDRADA ELECTRIC & ENGINEERING COMPANY, respondent.
G.R. No. 142936. April 17, 2002
381 SCRA 244
FACTS:
Respondent is a partnership duly organized, existing, and operating
under the laws of the Philippines, with office and principal place of business
at Nos. 794-812 Del Monte [A]venue, Quezon City, while the defendant
[herein petitioner] Philippine National Bank (herein referred to as PNB), is a
semi-government corporation duly organized, existing and operating under
the laws of the Philippines, with office and principal place of business at
Escolta Street, Sta. Cruz, Manila; whereas, the other defendant, the National
Sugar Development Corporation (NASUDECO in brief), is also a semigovernment corporation and the sugar arm of the PNB, with office and

Page 461 of 1072

principal place of business at the 2nd Floor, Sampaguita Building, Cubao,


Quezon City; and the defendant Pampanga Sugar Mills (PASUMIL in short), is
a corporation organized, existing and operating under the 1975 laws of the
Philippines, and had its business office before 1975 at Del Carmen,
Floridablanca, Pampanga.
Plaintiff is engaged in the business of general construction for the
repairs and/or construction of different kinds of machineries and buildings;
that on August 26, 1975, the defendant PNB acquired the assets of the
defendant PASUMIL that were earlier foreclosed by the Development Bank of
the Philippines (DBP) under LOI No. 311; that the defendant PNB organized
the defendant NASUDECO in September, 1975, to take ownership and
possession of the assets and ultimately to nationalize and consolidate its
interest in other PNB controlled sugar mills; that prior to October 29, 1971,
the defendant PASUMIL engaged the services of plaintiff for electrical
rewinding and repair, most of which were partially paid by the defendant
PASUMIL, leaving several unpaid accounts with the plaintiff; that finally, on
October 29, 1971, the plaintiff and the defendant PASUMIL entered into a
contract for the plaintiff and aside from the work contract the defendant
PASUMIL required the plaintiff to perform extra work, and provide electrical
equipment and spare parts.
That out of the total obligation of P777,263.80, the defendant
PASUMIL had paid only P250,000.00, leaving an unpaid balance, as of June
27, 1973, amounting to P527,263.80, as shown in the Certification of the
chief accountant of the PNB, a machine copy of which is appended as Annex
C of the complaint; that out of said unpaid balance of P527,263.80, the
defendant PASUMIL made a partial payment to the plaintiff of P14,000.00, in
broken amounts, covering the period from January 5, 1974 up to May 23,
1974, leaving an unpaid balance of P513,263.80; that the defendant
PASUMIL and the defendant PNB, and now the defendant NASUDECO, failed
and refused to pay the plaintiff their just, valid and demandable obligation;
that the President of the NASUDECO is also the Vice-President of the PNB,
and this official holds office at the 10th Floor of the PNB, Escolta, Manila, and
plaintiff besought this official to pay the outstanding obligation of the
defendant PASUMIL, inasmuch as the defendant PNB and NASUDECO now
owned and possessed the assets of the defendant PASUMIL, and these
defendants all benefited from the works, and the electrical, as well as the
engineering and repairs, performed by the plaintiff; that because of the
failure and refusal of the defendants to pay their just, valid, and demandable
obligations, plaintiff suffered actual damages in the total amount of
P513,263.80; and that in order to recover these sums, the plaintiff was
compelled to engage the professional services of counsel, to whom the
plaintiff agreed to pay a sum equivalent to 25% of the amount of the
obligation due by way of attorneys fees.
ISSUE/S:
Whether PNB is liable for the unpaid debts of PASUMIL to respondent.

Page 462 of 1072

RULING:
Petitioners posit that they should not be held liable for the corporate
debts of PASUMIL, because their takeover of the latters foreclosed assets did
not make them assignees. On the other hand, respondent asserts that
petitioners and PASUMIL should be treated as one entity and, as such, jointly
and severally held liable for PASUMILs unpaid obligation.
As a rule, a corporation that purchases the assets of another will not be
liable for the debts of the selling corporation, provided the former acted in
good faith and paid adequate consideration for such assets, except when any
of the following circumstances is present:
1. where the purchaser expressly or impliedly agrees to assume
the debts,
2. where the transaction amounts to a consolidation or merger of
the corporations,
3. where the purchasing corporation is merely a continuation of
the selling corporation, and (4) where the transaction is
fraudulently entered into in order to escape liability for those
debts.

TOPIC: TO SELL OR OTHERWISE DISPOSE OF ALL OR SUBSTANTIALLY ALL OF


CORPORATE
ASSETS
ISLAMIC DIRECTORATE OF THE PHILIPPINES, MANUEL F. PEREA and
SECURITIES & EXCHANGE COMMISSION, petitioners,
vs.
COURT OF APPEALS and IGLESIA NI CRISTO, respondents.
G.R. No. 117897. May 14, 1997
272 SCRA 454
FACTS:
Petitioner IDP-Tamano Group alleges that sometime in 1971, Islamic
leaders of all Muslim major tribal groups in the Philippines headed by Dean
Cesar Adib Majul organized and incorporated the ISLAMIC DIRECTORATE OF
THE PHILIPPINES (IDP), the primary purpose of which is to establish an
Islamic Center in Quezon City for the construction of a "Mosque (prayer

Page 463 of 1072

place), Madrasah (Arabic School), and other religious infrastructures" so as to


facilitate the effective practice of Islamic faith in the area.
Towards this end, that is, in the same year, the Libyan government
donated money to the IDP to purchase land at Culiat, Tandang Sora, Quezon
City, to be used as a Center for the Islamic populace. The land, with an area
of 49,652 square meters, was covered by two titles: Transfer Certificate of
Title Nos. RT-26520 (176616) and RT-26521 (170567), both registered in the
name of IDP.
It appears that in 1971, the Board of Trustees of the IDP was composed
of the following per Article 6 of its Articles of Incorporation: Senator Mamintal
Tamano, Congressman Ali Dimaporo, Congressman Salipada Pendatun, Dean
Cesar Adib Majul, Sultan Harun Al-Rashid Lucman, Delegate Ahmad Alonto,
Commissioner Datu Mama Sinsuat, Mayor Aminkadra Abubakar.
According to the petitioner, in 1972, after the purchase of the land by
the Libyan government in the name of IDP, Martial Law was declared by the
late President Ferdinand Marcos. Most of the members of the 1971 Board of
Trustees like Senators Mamintal Tamano, Salipada Pendatun, Ahmad Alonto,
and Congressman Al-Rashid Lucman flew to the Middle East to escape
political persecution.
Thereafter, two Muslim groups sprung, the Carpizo Group, headed by
Engineer Farouk Carpizo, and the Abbas Group, led by Mrs. Zorayda Tamano
and Atty. Firdaussi Abbas. Both groups claimed to be the legitimate IDP.
Significantly, on October 3, 1986, the SEC, in a suit between these two
contending groups, came out with a Decision in SEC Case No. 2687 declaring
the election of both the Carpizo Group and the Abbas Group as IDP board
members to be null and void.
ISSUE/S:
Whether the sale by Carpizo Group-INC was valid.
RULING:
NO.
The Carpizo Group-INC sale is further deemed null and void ab initio
because of the Carpizo Group's failure to comply with Section 40 of the
Corporation Code pertaining to the disposition of all or substantially all
assets of the corporation.
The Tandang Sora property, it appears from the records, constitutes
the only property of the IDP. Hence, its sale to a third-party is a sale or
disposition of all the corporate property and assets of IDP falling squarely
within the contemplation of the foregoing section. For the sale to be valid,
the majority vote of the legitimate Board of Trustees, concurred in by the

Page 464 of 1072

vote of at least 2/3 of the bona fide members of the corporation should have
been obtained. These twin requirements were not met as the Carpizo Group
which voted to sell the Tandang Sora property was a fake Board of Trustees,
and those whose names and signatures were affixed by the Carpizo Group
together with the sham Board Resolution authorizing the negotiation for the
sale were, from all indications, not bona fide members of the IDP as they
were made to appear to be. Apparently, there are only fifteen (15) official
members of the petitioner corporation including the eight (8) members of the
Board of Trustees.
All told, the disputed Deed of Absolute Sale executed by the fake
Carpizo Board and private respondent INC was intrinsically void ab initio.

TOPIC: TO SELL OR OTHERWISE DISPOSE OF ALL OR SUBSTANTIALLY ALL OF


CORPORATE
ASSETS
THE EDWARD J. NELL COMPANY, petitioner,
vs.
PACIFIC FARMS, INC., respondent.
G.R. No. L-20850. November 29, 1965
15 SCRA 415
FACTS:
On October 9, 1958, appellant secured in Civil Case No. 58579 of the
Municipal Court of Manila against Insular Farms, Inc. hereinafter referred to
as Insular Farms a judgment for the sum of P1,853.80 representing the
unpaid balance of the price of a pump sold by appellant to Insular Farms
with interest on said sum, plus P125.00 as attorney's fees and P84.00 as

Page 465 of 1072

costs. A writ of execution, issued after the judgment had become final, was,
on August 14, 1959, returned unsatisfied, stating that Insular Farms had no
leviable property. Soon thereafter, or on November 13, 1959, appellant filed
with said court the present action against Pacific Farms, Inc. hereinafter
referred to as appellee for the collection of the judgment aforementioned,
upon the theory that appellee is the alter ego of Insular Farms, which
appellee has denied. In due course, the municipal court rendered judgment
dismissing appellant's complaint. Appellant appealed, with the same result,
to the court of first instance and, subsequently, to the Court of Appeals.
Hence this appeal by certiorari.
ISSUE/S:
Whether appellee is an alter ego of Insular Farms, or is liable for its
debts.
RULING:
Generally where one corporation sells or otherwise transfers all of its
assets to another corporation, the latter is not liable for the debts and
liabilities of the transferor, except:
1. where the purchaser expressly or impliedly agrees to
assume such debts;
2. where the transaction amounts to a consolidation or
merger of the corporations;
3. where the purchasing corporation is merely a continuation
of the selling corporation; and
4. where the transaction is entered into fraudulently in order
to escape liability for such debts.
In the case at bar, there is neither proof nor allegation that appellee
had expressly or impliedly agreed to assume the debt of Insular Farms in
favor of appellant herein, or that the appellee is a continuation of Insular
Farms, or that the sale of either the shares of stock or the assets of Insular
Farms to the appellee has been entered into fraudulently, in order to escape
liability for the debt of the Insular Farms in favor of appellant herein. In fact,
these sales took place (March, 1958) not only over six (6) months before the
rendition of the judgment (October 9, 1958) sought to be collected in the
present action, but, also, over a month before the filing of the case (May 29,
1958) in which said judgment was rendered. Moreover, appellee purchased
the shares of stock of Insular Farms as the highest bidder at an auction sale
held at the instance of a bank to which said shares had been pledged as
security for an obligation of Insular Farms in favor of said bank. It has, also,
been established that the appellee had paid P285,126.99 for said shares of
stock, apart from the sum of P10,000.00 it, likewise, paid for the other assets
of Insular Farms.
Neither is it claimed that these transactions have resulted in the
consolidation or merger of the Insular Farms and appellee herein. On the

Page 466 of 1072

contrary, appellant's theory to the effect that appellee is an alter ego of the
Insular Farms negates such consolidation or merger, for a corporation cannot
be its own alter ego.

TOPIC: TO SELL OR OTHERWISE DISPOSE OF ALL OR SUBSTANTIALLY ALL OF


CORPORATE
ASSETS
JULIETA V. ESGUERRA, petitioner,
vs.
COURT OF APPEALS and SURESTE PROPERTIES, INC., respondents
G.R. No. 119310. February 3, 1997
FACTS:
Petitioner Julieta Esguerra filed a complaint for administration of
conjugal partnership or separation of property against her husband Vicente
Esguerra, Jr. before (the trial) court. The said complaint was later amended
on 31 October 1985 impleading V. Esguerra Construction Co., Inc. (VECCI for
brevity) and other family corporations as defendants.

Page 467 of 1072

The parties entered into a compromise agreement which was submitted to


the court. On the basis of the said agreement, the court on 11 January 1990
rendered two partial judgments: one between Vicente and (herein petitioner)
and the other as between the latter and VECCI.
By virtue of said agreement, Esguerra Bldg. I located at 140 Amorsolo
St., Legaspi Village was sold and the net proceeds distributed according to
the agreement. The controversy arose with respect to Esguerra Building II
located at 104 Amorsolo St., Legaspi Village, Makati. (Herein petitioner)
started claiming one-half of the rentals of the said building which VECCI
refused. Thus, petitioner filed a motion with respondent court praying that
VECCI be ordered to remit one-half of the rentals to her effective January
1990 until the same be sold. VECCI opposed said motion.
ISSUE/S:
Whether or not the sale to VECCI is valid.
RULING:
As far as private respondent Sureste Properties, Inc. is concerned, the
sale to it by VECCI was completely valid and legal because it was executed in
accordance with the compromise agreement, authorized not only by the
parties thereto, who became co-principals in a contract of agency created
thereby, but by the approving court as well. Consequently, the sale to
Sureste Properties, Inc. of Esguerra Building II cannot in any manner or guise
be deemed unenforceable, as contended by petitioner.
TOPIC: TO SELL OR OTHERWISE DISPOSE OF ALL OR SUBSTANTIALLY ALL OF
CORPORATE
ASSETS
LOPEZ REALTY, INC., AND ASUNCION LOPEZ GONZALES, petitioners,
vs.
FLORENTINA FONTECHA, ET AL., AND THE NATIONAL LABOR
RELATIONS COMMISSION, respondents.
G.R. No. 76801. August 11, 1995
247 SCRA 183
FACTS:
Lopez Realty, Inc., is a corporation engaged in real estate business,
while petitioner Asuncion Lopez Gonzales is one of its majority shareholders.
Except for Arturo F. Lopez, the rest of the shareholders also sit as members
of the Board of Directors. Arturo Lopez submitted a proposal relative to the

Page 468 of 1072

distribution of certain assets of Petitioner Corporation among its three (3)


main shareholders. The proposal had three (3) aspects, viz:
1. the sale of assets of the company to pay for its obligations;
2. the transfer of certain assets of the company to its three
3. main shareholders, while some other assets shall remain with
the company: and
4. the reduction of employees with provision for their gratuity
pay. The proposal was deliberated upon and approved in a
special meeting of the board of directors.
It appears that petitioner corporation approved two (2) resolutions
providing for the gratuity pay of its employees.
At that time, however, petitioner Asuncion Lopez Gonzales was still
abroad. Allegedly, while she was still out of the country, she sent a
cablegram to the corporation, objecting to certain matters taken up by the
board in her absence, such as the sale of some of the assets of the
corporation. Upon her return, she filed a derivative suit with the Securities
and Exchange Commission (SEC) against majority shareholder Arturo F.
Lopez.
ISSUE/S:
Whether or not the private respondents are entitled to receive their
gratuity pay under the assailed board resolutions.
RULING:
They are entitled. The assailed resolutions before us cover a subject
which concerns the benefit and welfare of the company's employees. To
stress, providing gratuity pay for its employees is one of the express powers
of the corporation under the corporation Code, hence, petitioners cannot
invoke the doctrine of ultra vires to avoid any liability arising from the
issuance the subject resolutions.
It will be observed that, except for Arturo Lopez, the stockholders of
Petitioner Corporation also sit as members of the board of directors. Under
the circumstances in field, it will be illogical and superfluous to require the
stockholders' approval of the subject resolutions. Thus, even without the
stockholders approval of the subject resolutions, petitioners are still liable to
pay private respondents' gratuity pay.

Page 469 of 1072

TOPIC:TO INVEST COPRPORATE FUNDS IN ANOTHER CORPORATION OR


BUSINESS
JOHN GOKONGWEI, JR.
VS.
SECURITIES AND EXCHANGE COMMISSION, ANDRES M. SORIANO,
JOSE M. SORIANO, ENRIQUE ZOBEL, ANTONIO ROXAS, EMETERIO
BUNAO, WALTHRODE B. CONDE, MIGUEL ORTIGAS, ANTONIO PRIETO,
SAN MIGUEL CORPORATION, EMIGDIO TANJUATCO, SR., AND
EDUARDO R. VISAYA
G.R. NO. L-45911, APRIL 11, 1979
FACTS:
John Gokongwei, desiring to have a seat in the Board of directors of
SMC, moved to annul the amended by- laws of SMC and to cancel the
certificate of filing thereof. He also filed a petition to declare Andres and Jose
Soriano, as well as SMC guilty of violating the Corporation Code having
discovered that respondent corporation has been investing corporate funds

Page 470 of 1072

in other corporations and businesses outside of the primary purpose clause


of the corporation.
Pending resolution of the cases, SMC proceeded its annual
stockholders meeting, which approved among others, the reaffirmation of
authorization granted by the stockholders to the board to invest corporate
funds in other businesses other than the purposes for which the corporation
was established. Respondents contend that such investment of corporate
funds in a foreign corporation is in furtherance of the same business which
the corporation stated in its articles.
ISSUES:
Whether or not respondent SEC gravely abused its discretion in
allowing the stockholders of Respondent Corporation to ratify the
investment of corporate funds in a foreign corporation
RULING:
Section 17-1/2 of the Corporation Law allows a corporation to "invest
its funds in any other corporation or business or for any purpose other than
the main purpose for which it was organized" provided that its Board of
Directors has been so authorized by the affirmative vote of stockholders
holding shares entitling them to exercise at least two-thirds of the voting
power. If the investment is made in pursuance of the corporate purpose, it
does not need the approval of the stockholders. It is only when the purchase
of shares is done solely for investment and not to accomplish the purpose of
its incorporation that the vote of approval of the stockholders holding shares
entitling them to exercise at least two-thirds of the voting power is
necessary.
As stated by Respondent Corporation, the purchase of beer
manufacturing facilities by SMC was an investment in the same business
stated as its main purpose in its Articles of Incorporation, which is to
manufacture and market beer. It appears that the original investment was
made in 1947-1948, when SMC, then San Miguel Brewery, Inc., purchased a
beer brewery in Hongkong (Hongkong Brewery & Distillery, Ltd.) for the
manufacture and marketing of San Miguel beer thereat. Restructuring of the
investment was made in 1970-1971 thru the organization of SMI in Bermuda
as tax free reorganization.

Page 471 of 1072

TOPIC:TO INVEST COPRPORATE FUNDS IN ANOTHER CORPORATION OR


BUSINESS
RAMON DE LA RAMA, FRANCISCO RODRIGUEZ, HORTENCIA SALAS,
PAZ SALAS AND PATRIA SALAS, HEIRS OF MAGDALENA SALAS, AS
STOCKHOLDERS ON THEIR OWN BEHALF AND FOR THE BENEFIT OF
THE MA-AO SUGAR CENTRAL CO., INC., AND OTHER STOCKHOLDERS
THEREOF WHO MAY WISH TO JOIN IN THIS ACTION VS.
MA-AO SUGAR CENTRAL CO., INC., J. AMADO ARANETA, MRS. RAMON
S. ARANETA, ROMUALDO M. ARANETA, AND RAMON A. YULO
G.R. NO. L-17504 & L-17506, FEBRUARY 28, 1969
7 SCRA 247
FACTS:
MSCCI, through its President, J. Amado, subscribed for P300k worth of
capital stock of the PFPC. Payments of the subscription were made on 3
installments. At the time the first two payments were made there was no
board resolution authorizing the investment. It was only on November 26,

Page 472 of 1072

1951, that J. Amado was so authorized by the BOD, by the way, making the
third payment made in March 1952 authorized. In addition, 355k shares of
PFPC, owned by LIC were transferred to MSCCI. Again, the investment was
made without prior board resolution, the authorizing resolution having been
subsequently approved only on June 4, 1952. A derivative suit was filed by 4
minority SHs of MSCCI which stated 5 causes of action:
1) for alleged illegal and ultra-vires acts consisting of self-dealing,
irregular loans, and unauthorized investments;
2) for alleged gross mismanagement;
3) for alleged forfeiture of corporate rights warranting dissolution;
4) for alleged damages and attorney's fees; and
5) for receivership.
ISSUE/S:
Whether or not the lower court erred in holding that the investment of
corporate funds of the MSCCI in the PFPC Inc. was not a violation of
sec. 17- of the Corporation Law.
RULING:
In his work entitled "The Philippine Corporation Law," now in its 5th
edition, Professor Sulpicio S. Guevara of the University of the Philippines,
College of Law, a well-known authority in commercial law, reconciled these
two apparently conflicting legal provisions, as follows:
j. Power to acquire or dispose of shares or securities.
A private corporation, in order to accomplish its
purpose as stated in its articles of incorporation, and
subject to the limitations imposed by the Corporation
Law, has the power to acquire, hold, mortgage,
pledge or dispose of shares, bonds, securities, and
other evidences of indebtedness of any domestic or
foreign corporation. Such an act, if done in
pursuance of the corporate purpose, does not need
the approval of the stockholders; but when the
purchase of shares of another corporation is done
solely for investment and not to accomplish the
purpose of its incorporation, the vote of approval of
the stockholders is necessary. In any case, the
purchase of such shares or securities must be subject
to the limitations established by the Corporation Law;
namely, (a) that no agricultural or mining corporation
shall in anywise be interested in any other
agricultural or mining corporation; or (b) that a nonagricultural or non-mining corporation shall be
restricted to own not more than 15% of the voting

Page 473 of 1072

stock of any agricultural or mining corporation; and


(c) that such holdings shall be solely for investment
and not for the purpose of bringing about a
monopoly in any line of commerce or combination in
restraint of trade. (The Philippine Corporation Law by
Sulpicio S. Guevara, 1967 Ed., p. 89.)
40. Power to invest corporate funds. A private
corporation has the power to invest its corporate funds in
any other corporation or business, or for any purpose other
than the main purpose for which it was organized, provided
that 'its board of directors has been so authorized in a
resolution by the affirmative vote of stockholders holding
shares in the corporation entitling them to exercise at least
two-thirds of the voting power on such a proposal at a
stockholders' meeting called for that purpose,' and
provided further, that no agricultural or mining corporation
shall in anywise be interested in any other agricultural or
mining corporation. When the investment is necessary to
accomplish its purpose or purposes as stated in it articles
of incorporation, the approval of the stockholders is not
necessary.
The Court therefore agrees with the finding of the Lower Court that the
investment in question does not fall under the purview of Sec. 17- of the
Corporation Law.
TOPIC:TO ACQUIRE OWN SHARES
BOMAN ENVIRONMENTAL DEVELOPMENT CORPORATION
Vs.
HON. COURT OF APPEALS and NILCAR Y. FAJILAN
G.R. No. 77860, November 22, 1988
FACTS:
Nilcar Y. Fajilan offered in writing to resign as President and Member of
the Board of Directors BEDECO and to sell to the company all his shares,
rights, and interests therein for P 300,000 plus the transfer to him of the
company's Isuzu pick-up truck which he had been using.
Fajilan's resignation as president was accepted and new officers were
elected. Fajilan's offer to sell his shares back to the corporation was
approved, the Board promising to pay for them on a staggered basis from
July 15, 1984 to December 15, 1984. However, BEDECO paid only P50, 000

Page 474 of 1072

on July 15, 1984 and another P50, 000 on August 31, 1984 and defaulted in
paying the balance of P200, 000.
Fajilan filed a complaint in the Regional Trial Court of Makati for
collection of that balance from BEDECO. The trial court dismissed the
complaint for lack of jurisdiction. It ruled that the controversy arose out of
intracorporate relations, hence, the Securities and Exchange Commission has
original and exclusive jurisdiction to hear and decide it.
ISSUE/S:
Whether or not a suit brought by a withdrawing stockholder against the
corporation to enforce payment of the balance due on the
consideration (evidenced by a corporate promissory note) for the
surrender of his shares of stock and interests in the corporation,
involves an intra-corporate dispute.
RULING:
Fajilan's suit against the corporation to enforce the latter's promissory
note or compel the corporation to pay for his shareholdings is cognizable by
the SEC alone which shall determine whether such payment will not
constitute a distribution of corporate assets to a stockholder in preference
over creditors of the corporation. The SEC has exclusive supervision, control
and regulatory jurisdiction to investigate whether the corporation has
unrestricted retained earnings to cover the payment for the shares, and
whether the purchase is for a legitimate corporate purpose as provided in
Sections 41 and 122 of the Corporation Code, which reads as follows:
SEC. 41. Power to acquire own shares.A stock corporation shall
have the power to purchase or acquire its own shares for a
legitimate corporate purpose or purposes, including but not
limited to the following cases: Provided, That the corporation has
unrestricted retained earnings in its books to cover the shares to
be purchased or acquired;
1) To eliminate fractional shares arising out of
stock dividends;
2) To collect or compromise an indebtedness to
the corporation, arising out of unpaid
subscription, in a delinquency sale, and to
purchase delinquent shares sold during said
sale; and
3) To pay dissenting or withdrawing stockholders
entitled to payment for their shares under the
provisions of this Code,

Page 475 of 1072

Sec. 12. Corporate liquidation. ...


xxx xxx xxx
Except by decrease of capital stock and as otherwise allowed by
this Code, no corporation shall distribute any of its assets or
property except upon lawful dissolution and after payment of all
its debts and liabilities, (77a, 89a, 16a).

TOPIC:TO ACQUIRE OWN SHARES


C. H. STEINBERG, AS RECEIVER OF THE SIBUGUEY TRADING
COMPANY, INCORPORATED
VS.
GREGORIO VELASCO, ET AL.
G.R. NO. L-30460, MARCH 12, 1929
52 Phil 953
FACTS:
Gregorio Velasco, as president, Felix del Castillo, as vice-president,
Andres L. Navallo, as secretary-treasurer, and Rufino Manuel, as director of
Trading Company approved and authorized various lawful purchases already
made of a large portion of the capital stock of the company from its various
stockholders, thereby diverting its funds to the injury, damage and in fraud
of the creditors of the corporation. The total amount of the capital stock
unlawfully purchased was P3, 300. That at the time of such purchase, the
corporation had accounts payable amounting to P13,807.50, most of which
were unpaid at the time petition for the dissolution of the corporation was
financial condition, in contemplation of an insolvency and dissolution.

Page 476 of 1072

Plaintiff alleges that the officers and directors of the corporation


approved a resolution for the payment of P3, 000 as dividends to its
stockholders, which was wrongfully done, and in bad faith, and to the injury
and fraud of its creditors. That at the time the petition for the dissolution of
the corporation was presented it had accounts payable in the sum of P9,
241.19, "and practically worthless accounts receivable.
ISSUE/S:
Whether or not Sibuguey could legally purchase its own stock.
RULING:
It is very apparent that on June 24, 1922, the board of directors acted
on assumption that, because it appeared from the books of the corporation
that it had accounts receivable of the face value of P19, 126.02, therefore it
had a surplus over and above its debts and liabilities. But as stated there is
no stipulation as to the actual cash value of those accounts, and it does
appear from the stipulation that on February 28, 1924, P12, 512.47 of those
accounts had but little, if any, value, and it must be conceded that, in the
purchase of its own stock to the amount of P3, 300 and in declaring the
dividends to the amount of P3, 000, the real assets of the corporation were
diminished P6, 300. It also appears from paragraph 4 of the stipulation that
the corporation had a "surplus profit" of P3, 314.72 only. It is further
stipulated that the dividends should "be made in installments so as not to
effect financial condition of the corporation." In other words, that the
corporation did not then have an actual bona fide surplus from which the
dividends could be paid, and that the payment of them in full at the time
would "affect the financial condition of the corporation."
Upon each of those points, the rule is well stated in Ruling Case Law,
vol. 7, p. 473, section 454 where it is said:
General Duty to Exercise Reasonable Care. The directors
of a corporation are bound to care for its property and
manage its affairs in good faith, and for a violation of these
duties resulting in waste of its assets or injury to the
property they are liable to account the same as other
trustees. Are there can be no doubt that if they do acts
clearly beyond their power, whereby loss ensues to the
corporation, or dispose of its property or pay away its
money without authority, they will be required to make
good the loss out of their private estates. This is the rule
where the disposition made of money or property of the
corporation is one either not within the lawful power of the

Page 477 of 1072

corporation, or, if within the authority of the particular


officer or officers.
And section 458 which say:
Want of Knowledge, Skill, or Competency. It has
been said that directors are not liable for losses resulting to
the corporation from want of knowledge on their part; or
for mistake of judgment, provided they were honest, and
provided they are fairly within the scope of the powers and
discretion confided to the managing body. But the
acceptance of the office of a director of a corporation
implies a competent knowledge of the duties assumed, and
directors cannot excuse imprudence on the ground of their
ignorance or inexperience; and if they commit an error of
judgment through mere recklessness or want of ordinary
prudence or skill, they may be held liable for the
consequences. Like a mandatory, to whom he has been
likened, a director is bound not only to exercise proper care
and diligence, but ordinary skill and judgment. As he is
bound to exercise ordinary skill and judgment, he cannot
set up that he did not possess them.
Creditors of a corporation have the right to assume that so long as
there are outstanding debts and liabilities, the board of directors will not use
the assets of the corporation to purchase its own stock, and that it will not
declare dividends to stockholders when the corporation is insolvent.
TOPIC:TO DECLARE DIVIDENDS KINDS: CASH, STOCK, PROPERTY, AND
SCRIP
IMELDA O. COJUANGCO, PRIME HOLDINGS, INC. AND THE ESTATE OF
RAMON U. COJUANGCO
VS.
SANDIGANBAYAN, REPUBLIC OF THE PHILIPPINES, AND THE SHERIFF
OF SANDIGANBAYAN
G.R. NO. 183278, APRIL 24, 2009
FACTS:
As a consequence of the finality of the Sandiganbayans decision in
Republic of the Philippines vs. Ferdinand Marcos, et.al. declaring the Republic
to be the owner of 111,415 PTIC shares registered in the name of Prime
Holdings, the Republic filed on November 20, 2006 with the Sandiganbayan a
Motion for the Issuance of a Writ of Execution, praying for the cancellation of
the 111,415 shares/certificates of stock registered in the name of Prime
Holdings and the annotation of the change of ownership on PTICs Stock and

Page 478 of 1072

Transfer Book. The Republic further prayed for the issuance of an order for
PTIC to account for all cash and stock dividends declared and/or issued by
PLDT in favor of PTIC from 1986 up to the present including compounded
interests appurtenant thereto.
By Resolution dated December 14, 2006, the Sandiganbayan granted
the Motion for the Issuance of a Writ of Execution with respect to the
reconveyance of the shares, but denied the prayer for accounting of
dividends.
The Republic filed a Motion of Reconsideration and the same was partly
granted by the Sandiganbayan. On the other hand, the petitioners assert
that the Republic has no more rights to the said dividends because it already
sold its shares to Metro Pacific Assets Holdings, Inc.,
ISSUE/S:
Whether the Sandiganbayan gravely abused its discretion in
ordering the accounting, delivery, and remittance to the Republic
of the stock, cash, and property dividends pertaining to the
111,415 PTIC shares of Prime Holdings, this Courts Decision in
G.R. No. 153459 not having even discussed the same.
RULING:
The term "dividend" in its technical sense and ordinary acceptation is
that part or portion of the profits of the enterprise which the corporation, by
its governing agents, sets apart for ratable division among the holders of the
capital stock.5 It is a payment to the stockholders of a corporation as a return
upon their investment,6 and the right thereto is an incident of ownership of
stock.
This Court, in directing the reconveyance to the Republic of the
111,415 shares of PLDT stock owned by PTIC in the name of Prime Holdings,
declared the Republic as theowner of said shares and, necessarily, the
dividends and interests accruing thereto.
Ownership is a relation in law by virtue of which a thing pertaining to
one person is completely subjected to his will in everything not prohibited by
law or the concurrence with the rights of another. Its traditional elements or
attributes include jus utendi or the right to receive from the thing what it
produces.
In G.R. No. 153459, although the inclusion of the dividends, interests,
and earnings of the 111,415 PTIC shares as belonging to the Republic was
not mentioned in the dispositive portion of the Courts Decision, it is clear
from its body that what was being adjudicated in favor of the Republic was
the whole block of shares and the fruits thereof, said shares having been

Page 479 of 1072

found to be part of the Marcoses ill-gotten wealth, and therefore, public


money.
It would be absurd to award the shares to the Republic as their owner
and not include the dividends and interests accruing thereto. An owner who
cannot exercise the "juses" or attributes of ownership -- the right to possess,
to use and enjoy, to abuse or consume, to accessories, to dispose or
alienate, to recover or vindicate, and to the fruits - is a crippled owner.

Page 480 of 1072

TOPIC:DIVIDENDS DECLARATION, PAYMENT & RECORD DATES


IMELDA O. COJUANGCO, PRIME HOLDINGS, INC. AND THE ESTATE OF
RAMON U. COJUANGCO
VS.
SANDIGANBAYAN, REPUBLIC OF THE PHILIPPINES, AND THE SHERIFF
OF SANDIGANBAYAN
G.R. NO. 183278, APRIL 24, 2009
FACTS:
As a consequence of the finality of the Sandiganbayans decision in
Republic of the Philippines vs. Ferdinand Marcos, et.al. declaring the Republic
to be the owner of 111,415 PTIC shares registered in the name of Prime
Holdings, the Republic filed on November 20, 2006 with the Sandiganbayan a
Motion for the Issuance of a Writ of Execution, praying for the cancellation of
the 111,415 shares/certificates of stock registered in the name of Prime
Holdings and the annotation of the change of ownership on PTICs Stock and
Transfer Book. The Republic further prayed for the issuance of an order for
PTIC to account for all cash and stock dividends declared and/or issued by
PLDT in favor of PTIC from 1986 up to the present including compounded
interests appurtenant thereto.
By Resolution dated December 14, 2006, the Sandiganbayan granted
the Motion for the Issuance of a Writ of Execution with respect to the
reconveyance of the shares, but denied the prayer for accounting of
dividends.
The Republic filed a Motion of Reconsideration and the same was partly
granted by the Sandiganbayan. On the other hand, the petitioners assert
that the Republic has no more rights to the said dividends because it already
sold its shares to Metro Pacific Assets Holdings, Inc.,
ISSUE/S:
Whether or not the Republic, having transferred the shares to a
third party, is entitled to the dividends, interests, and earnings
thereof.
RULING:
Dividends are payable to the stockholders of record as of the date of
the declaration of dividends or holders of record on a certain future date, as
the case may be, unless the parties have agreed otherwise. A transfer of
shares which is not recorded in the books of the corporation is valid only as
between the parties; hence, the transferor has the right to dividends as
against the corporation without notice of transfer but it serves as trustee of
the real owner of the dividends, subject to the contract between the
transferor and transferee as to who is entitled to receive the dividends.

Page 481 of 1072

It is thus clear that the Republic is entitled to the dividends accruing


from the subject 111,415 shares since 1986 when they were sequestered up
to the time they were transferred to Metro Pacific via the Sale and Purchase
Agreement of February 28, 2007; and that the Republic has since the latter
date been serving as trustee of those dividends for the Metro Pacific up to
the present, subject to the terms and conditions of the said agreement they
entered into.

Page 482 of 1072

TOPIC: LIMITATION ON RETENTION OF SURPLUS PROFITS


C. H. STEINBERG, as Receiver of the Sibuguey Trading Company,
Incorporated, plaintiff-appellant,
vs.
GREGORIO VELASCO, ET AL., defendants-appellees.
G.R. NO. L-30460, MARCH 12, 1929
52 Phil 953
FACTS:
Plaintiff is the receiver of the Sibuguey Trading Company, a domestic
corporation. The defendants are residents of the Philippine Islands.
Plaintiff alleged that the defendants, Gregorio Velasco, as president,
Felix del Castillo, as vice-president, Andres L. Navallo, as secretary-treasurer,
and Rufino Manuel, as director of Trading Company, at a meeting of the
board of directors held on July 24, 1922, approved and authorized various
lawful purchases already made of a large portion of the capital stock of the
company from its various stockholders, thereby diverting its funds to the
injury, damage and in fraud of the creditors of the corporation. He
furthermore alleges that on July 24, 1922, the officers and directors of the
corporation approved a resolution for the payment of P3,000 as dividends to
its stockholders, which was wrongfully done and in bad faith, and to the
injury and fraud of its creditors. That at the time the petition for the
dissolution of the corporation was presented it had accounts payable in the
sum of P9,241.19, "and practically worthless accounts receivable."
ISSUE/S:
Whether or not Sibuguey Trading Company, Incorporated, could
legally purchase its own stock.
Whether or not the Board of Directors can legally declare a dividend
of P3, 000.
RULING:
1. From all of which, it appears that on June 30, 1922, the board of
directors of the corporation authorized the purchase of, purchased and
paid for, 330 shares of the capital stock of the corporation at the
agreed price of P3, 300, and that at the time the purchase was made,
the corporation was indebted in the sum of P13, 807.50, and that
according to its books, it had accounts receivable in the sum of P19,
126.02. That on September 11, 1923, when the petition was filed for its
dissolution upon the ground that it was insolvent, its accounts payable
amounted to P9,241.19, and its accounts receivable P12,512.47, or an
apparent asset of P3,271.28 over and above its liabilities. But it will be
noted that there is no stipulation or finding of facts as to what was the
actual cash value of its accounts receivable. Neither is there any

Page 483 of 1072

stipulation that those accounts or any part of them ever have been or
will be collected, and it does appear that after his appointment on
February 28, 1924, the receiver made a diligent effort to collect them,
and that he was unable to do so, and it also appears from the minutes
of the board of directors that the president and manager
"recommended that P3,000 out of the surplus account to be set
aside for dividends payable, and that payments be made in
installments so as not to effect the financial condition of the
corporation."
If in truth and in fact the corporation had an actual bona fide surplus
of P3,000 over and above all of its debt and liabilities, the payment of the
P3,000 in dividends would not in the least impair the financial condition of
the corporation or prejudice the interests of its creditors.
It is very apparent that on June 24, 1922, the board of directors
acted on assumption that, because it appeared from the books of the
corporation that it had accounts receivable of the face value of P19,
126.02, therefore it had a surplus over and above its debts and liabilities.
But as stated there is no stipulation as to the actual cash value of those
accounts, and it does appear from the stipulation that on February 28,
1924, P12, 512.47 of those accounts had but little, if any, value, and it
must be conceded that, in the purchase of its own stock to the amount of
P3, 300 and in declaring the dividends to the amount of P3, 000, the real
assets of the corporation were diminished P6, 300. It also appears from
paragraph 4 of the stipulation that the corporation had a "surplus profit"
of P3, 314.72 only. It is further stipulated that the dividends should "be
made in installments so as not to effect financial condition of the
corporation." In other words, that the corporation did not then have an
actual bona fide surplus from which the dividends could be paid, and that
the payment of them in full at the time would "affect the financial
condition of the corporation."
2. Creditors of a corporation have the right to assume that so long as
there are outstanding debts and liabilities, the board of directors will
not use the assets of the corporation to purchase its own stock, and
that it will not declare dividends to stockholders when the corporation
is insolvent.

Page 484 of 1072

TOPIC: LIMITATION ON RETENTION OF SURPLUS PROFITS


NIELSON & COMPANY, INC.
VS.
LEPANTO CONSOLIDATED MINING COMPANY
G.R. No. L-21601, December 28, 1968
26 SCRA 540
FACTS:
Nielson and Lepanto entered into a management contract whereby the
former shall operate and develop the mining properties of Lepanto for five
years. In 1940, there was some dispute between Lepanto and Nielson
regarding the application and interpretation of certain provisions of the
original contract particularly with regard to the 10% participation of Nielson
in the net profits.
Thus, a suit ensued whereby the Supreme Court ruled in favor of
Nielson awarding to it undetermined amount of shares of stock.
ISSUE:
Whether or not the court erred in awarding to Nielson an
undetermined amount of shares of stock and/or cash, which
award cannot be ascertained and executed without further
litigation.
RULING:
Per Section 16 of the Corporation Law, the considerations for which
shares of stock may be issued are:
(1) cash;
(2) property; and
(3) undistributed profits.
Shares of stock are given the special name "stock dividends" only if
they are issued in lieu of undistributed profits. If shares of stocks are issued
in exchange of cash or property then those shares do not fall under the
category of "stock dividends". A corporation may legally issue shares of stock
in consideration of services rendered to it by a person not a stockholder, or
in payment of its indebtedness. A share of stock issued to pay for services
rendered is equivalent to a stock issued in exchange of property, because
services are equivalent to property. Likewise a share of stock issued in
payment of indebtedness is equivalent to issuing a stock in exchange for
cash. But a share of stock thus issued should be part of the original capital
stock of the corporation upon its organization, or part of the stocks issued
when the increase of the capitalization of a corporation is properly
authorized. In other words, it is the shares of stock that are originally issued
by the corporation and forming part of the capital that can be exchanged for

Page 485 of 1072

cash or services rendered, or property; that is, if the corporation has original
shares of stock unsold or unsubscribed, either coming from the original
capitalization or from the increased capitalization. Those shares of stock may
be issued to a person who is not a stockholder, or to a person already a
stockholder in exchange for services rendered or for cash or property. But a
share of stock coming from stock dividends declared cannot be issued to one
who is not a stockholder of a corporation.
A "stock dividend" is any dividend payable in shares of stock of the
corporation declaring or authorizing such dividend. It is, what the term itself
implies, a distribution of the shares of stock of the corporation among the
stockholders as dividends. A stock dividend of a corporation is a dividend
paid in shares of stock instead of cash, and is properly payable only out of
surplus profits. So, a stock dividend is actually two things:
(1) A dividend, and
(2) The enforced use of the dividend money to purchase
additional shares of stock at par.
When a corporation issues stock dividends, it shows that the
corporation's accumulated profits have been capitalized instead of
distributed to the stockholders or retained as surplus available for
distribution, in money or kind, should opportunity offer. Far from being a
realization of profits for the stockholder, it tends rather to postpone said
realization, in that the fund represented by the new stock has been
transferred from surplus to assets and no longer available for actual
distribution. Thus, it is apparent that stock dividends are issued only to
stockholders. This is so because only stockholders are entitled to dividends.
They are the only ones who have a right to a proportional share in that part
of the surplus which is declared as dividends. A stock dividend really adds
nothing to the interest of the stockholder; the proportional interest of each
stockholder remains the same. If a stockholder is deprived of his stock
dividends - and this happens if the shares of stock forming part of the stock
dividends are issued to a non-stockholder then the proportion of the
stockholder's interest changes radically. Stock dividends are civil fruits of the
original investment, and to the owners of the shares belong the civil fruits.
The term "dividend" both in the technical sense and its ordinary
acceptation, is that part or portion of the profits of the enterprise which the
corporation, by its governing agents, sets apart for ratable division among
the holders of the capital stock. It means the fund actually set aside, and
declared by the directors of the corporation as dividends and duly ordered by
the director, or by the stockholders at a corporate meeting, to be divided or
distributed among the stockholders according to their respective interests.
It is Courts considered view, therefore, that under Section 16 of the
Corporation Law stock dividends cannot be issued to a person who is not a

Page 486 of 1072

stockholder in payment of services rendered. And so, in the case at bar


Nielson cannot be paid in shares of stock which form part of the stock
dividends of Lepanto for services it rendered under the management
contract. We sustain the contention of Lepanto that the understanding
between Lepanto and Nielson was simply to make the cash value of the stock
dividends declared as the basis for determining the amount of compensation
that should be paid to Nielson, in the proportion of 10% of the cash value of
the stock dividends declared. And this conclusion of ours finds support in the
record.

Page 487 of 1072

TOPIC: LIMITATION ON RETENTION OF SURPLUS PROFITS


COMMISSIONER OF INTERNAL REVENUE
vs.
JOHN L. MANNING, W.D. McDONALD, E.E. SIMMONS and THE COURT
OF TAX APPEALS
G.R. No. L-28398, August 6, 1975
66 SCRA 14
FACTS:
In 1952 the MANTRASCO had an authorized capital stock of P2,500,000
divided into 25,000 common shares; 24,700 of these were owned by Julius S.
Reese, and the rest, at 100 shares each, by the three respondents.
Upon the death of Reese, and in accordance to his desire, On February
2, 1955, after MANTRASCO made a partial payment of Reese's shares, the
certificate for the 24,700 shares in Reese's name was cancelled and a new
certificate was issued in the name of MANTRASCO. On the same date, and in
the meantime that Reese's interest had not been fully paid, the new
certificate was endorsed to the law firm of Ross, Selph, Carrascoso and
Janda, as trustees for and in behalf of MANTRASCO.
On November 25, 1963 the entire purchase price of Reese's interest in
MANTRASCO was finally paid in full by the latter, On May 4, 1964 the trust
agreement was terminated and the trustees delivered to MANTRASCO all the
shares which they were holding in trust.
Meanwhile, on September 14, 1962, an examination of MANTRASCO's
books was ordered by the Bureau of Internal Revenue. The examination
disclosed that (a) as of December 31, 1958 the 24,700 shares declared as
dividends had been proportionately distributed to the respondents,
representing a total book value or acquisition cost of P7,973,660; (b) the
respondents failed to declare the said stock dividends as part of their taxable
income for the year 1958.
On the basis of their examination, the BIR examiners concluded that
the distribution of Reese's shares as stock dividends was in effect a
distribution of the "asset or property of the corporation as may be gleaned
from the payment of cash for the redemption of said stock and distributing
the same as stock dividend." On April 14, 1965 the Commissioner of Internal
Revenue issued notices of assessment for deficiency income taxes to the
respondents for the year 1958.
The respondents unsuccessfully challenged the foregoing assessments
and, failing to secure a favorable reconsideration, appealed to the Court of
Tax Appeals.

Page 488 of 1072

On October 30, 1967 the CTA rendered judgment absolving the respondents
from any liability for receiving the questioned stock dividends on the ground
that their respective one-third interest in MANTRASCO remained the same
before and after the declaration of stock dividends and only the number of
shares held by each of them had changed.
ISSUE/S:
Whether or not the CTA erred in absolving the respondents from
any liability for receiving stock dividends
RULING:
The manifest intention of the parties to the trust agreement was, in
sum and substance, to treat the 24,700 shares of Reese as absolutely
outstanding shares of Reese's estate until they were fully paid. Such being
the true nature of the 24,700 shares, their declaration as treasury stock
dividend in 1958 was a complete nullity and plainly violative of public policy.
A stock dividend, being one payable in capital stock, cannot be declared out
of outstanding corporate stock, but only from retained earnings.
Of pointed relevance is this useful discussion of the nature of a stock
dividend:
"'A stock dividend always involves a transfer of surplus (or profit)
to capital stock.' Graham and Katz, Accounting in Law Practice,
2d ed. 1938, No. 70. As the court said in United States vs. Siegel,
8 Cir., 1931, 52 F 2d 63, 65, 78 ALR 672: 'A stock dividend is a
conversion of surplus or undivided profits into capital stock,
which is distributed to stockholders in lieu of a cash dividend.'
Congress itself has defined the term 'dividend' in No. 115(a) of
the Act as meaning any distribution made by a corporation to its
shareholders, whether in money or in other property, out of its
earnings or profits. In Eisner v. Macomber, 1920, 252 US 189, 40
S Ct 189, 64 L Ed 521, 9 ALR 1570, both the prevailing and the
dissenting opinions recognized that within the meaning of the
revenue acts the essence of a stock dividend was the
segregation out of surplus account of a definite portion of the
corporate earnings as part of the permanent capital resources of
the corporation by the device of capitalizing the same, and the
issuance to the stockholders of additional shares of stock
representing the profits so capitalized."
The declaration by the respondents and Reese's trustees of
MANTRASCO's alleged treasury stock dividends in favor of the former, brings,
however, into clear focus the ultimate purpose which the parties to the trust
instrument aimed to realize: to make the respondents the sole owners of
Reese's interest in MANTRASCO by utilizing the periodic earnings of that

Page 489 of 1072

company and its subsidiaries to directly subsidize their purchase of the said
interests, and by making it appear outwardly, through the formal declaration
of non-existent stock dividends in the treasury, that they have not received
any income from those firms when, in fact, by that declaration they secured
to themselves the means to turn around as full owners of Reese's shares. In
other words, the respondents, using the trust instrument as a convenient
technical device, bestowed unto themselves the full worth and value of
Reese's corporate holdings with the use of the very earnings of the
companies. Such package device, obviously not designed to carry out the
usual stock dividend purpose of corporate expansion reinvestment, e.g. the
acquisition of additional facilities and other capital budget items, but
exclusively for expanding the capital base of the respondents in
MANTRASCO, cannot be allowed to deflect the respondents' responsibilities
toward our income tax laws. The conclusion is thus ineluctable that
whenever the companies involved herein parted with a portion of their
earnings "to buy" the corporate holdings of Reese, they were in ultimate
effect and result making a distribution of such earnings to the respondents.
All these amounts are consequently subject to income tax as being, in truth
and in fact, a flow of cash benefits to the respondents.

Page 490 of 1072

TOPIC: LIMITATION ON RETENTION OF SURPLUS PROFITS


MADRIGAL & COMPANY, INC.
VS.
HON. RONALDO ZAMORA
G.R. NO. L-48237, JUNE 30, 1987
151 SCRA 355
FACTS:
The Madrigal Central Office Employees Union sought the renewal of its
collective bargaining agreement with the petitioner which was due to expire.
Specifically, it proposed a wage increase of P200.00 a month, an allowance
of P100.00 a month, and other economic benefits. However, petitioner
requested for a deferment in the said negotiations. During the period of
deferment, and by the alleged resolution of Madrigals stockholders, the
petitioner twice reduced its capital stock which was effected through the
distribution of the marketable securities owned by the petitioner to its
stockholders in exchange for their shares in an equivalent amount in the
corporation.
After the petitioners failure to sit down with the respondent union, the
latter filed a complaint for unfair labor practice. In due time, petitioner filed
its position paper, alleging operational losses. The labor arbiter rendered a
decision granting, among other things, a general wage increase of P200.00 a
month beginning March 1, 1974 plus a monthly living allowance of P100.00
in favor of the petitioners employees. The arbiter specifically found that the
petitioner had been making substantial profits in its operation since 1972
through 1975.
ISSUE/S:
Whether or not NLRC erred in not finding that the economic
benefits granted in the form of salary increases are unfair and
violative of the mandatory guidelines prescribed under
Presidential Decree No. 525 and ignoring the undisputed fact
that petitioner had virtually ceased operations after having twice
decreased its capital stocks and, therefore, not financially
capable to absorb such award of benefits.
RULING:
The petitioner would, however, have us believe that it in fact sustained
losses. Whatever profits it earned, so it claims were in the nature of
dividends "declared on its shareholdings in other companies in the earning of
which the employees had no participation whatsoever." "Cash dividends,"
according to it, "are the absolute property of the stockholders and cannot be
made available for disposition if only to meet the employees' economic
demands."

Page 491 of 1072

There is no merit in this contention. We agree with the National Labor


Relations Commission that "[t]he dividends received by the company are
corporate earnings arising from corporate investment." Indeed, as found by
the Commission, the petitioner had entered such earnings in its financial
statements as profits, which it would not have done if they were not in fact
profits.
Moreover, it is incorrect to say that such profits in the form of
dividends are beyond the reach of the petitioner's creditors since the
petitioner had received them as compensation for its management services
in favor of the companies it managed as a shareholder thereof. As such
shareholder, the dividends paid to it were its own money, which may then be
available for wage increments. It is not a case of a corporation distributing
dividends in favor of its stockholders, in which case, such dividends would be
the absolute property of the stockholders and hence, out of reach by
creditors of the corporation. Here, the petitioner was acting as stockholder
itself, and in that case, the right to a share in such dividends, by way of
salary increases, may not be denied its employees.
Accordingly, this court is convinced that the petitioner's capital
reduction efforts were, to begin with, a subterfuge, a deception as it were, to
camouflage the fact that it had been making profits, and consequently, to
justify the mass layoff in its employee ranks, especially of union members.
They were nothing but a premature and plain distribution of corporate assets
to obviate a just sharing to labor of the vast profits obtained by its joint
efforts with capital through the years. Surely, we can neither countenance
nor condone this. It is an unfair labor practice.

Page 492 of 1072

TOPIC: LIMITATION ON THE RETENTION OF SURPLUS PROFITS


REPUBLIC PLANTERS BANK, petitioner,
vs.
HON. ENRIQUE A. AGANA, SR., as Presiding Judge, Court of First
Instance of Rizal, Branch XXVIII, Pasay City, ROBES-FRANCISCO
REALTY & DEVELOPMENT CORPORATION and ADALIA F. ROBES,
respondents.
[G.R. No. 51765. March 3, 1997]
269 SCRA 1
FACTS:
On September 18, 1961, private respondent Corporation secured a
loan from petitioner in the amount of P120,000.00. As part of the proceeds of
the loan, preferred shares of stocks were issued to private respondent
Corporation, through its officers then, private respondent Adalia F. Robes and
one Carlos F. Robes. In other words, instead of giving the legal tender totaling
to the full amount of the loan, which is P120, 000.00, petitioner lent such
amount partially in the form of money and partially in the form of stock
certificates numbered 3204 and 3205, each for 400 shares with a par value
of P10.00 per share, or for P4,000.00 each, for a total of P8,000.00. Said
stock certificates were in the name of private respondent Adalia F. Robes and
Carlos F. Robes, who subsequently, however, endorsed his shares in favor of
Adalia F. Robes.
Said certificates of stock bear the following terms and conditions:
"The Preferred Stock shall have the following rights,
preferences, qualifications and limitations, to wit:
1. Of the right to receive a quarterly dividend of One
Per Centum (1%), cumulative and participating.
2. That such preferred shares may be redeemed, by the
system of drawing lots, at any time after two (2)
years from the date of issue at the option of the
Corporation. x x x."
On January 31, 1979, private respondents proceeded against petitioner
and filed a Complaint anchored on private respondents' alleged rights to
collect dividends under the preferred shares in question and to have
petitioner redeem the same under the terms and conditions of the stock
certificates. Private respondents attached to their complaint, a letter-demand
dated January 5, 1979.

Page 493 of 1072

ISSUE/S:
Whether or not the petitioner is liable for redemption price.
RULING:
Preferred shares take a multiplicity of forms. The most common forms
may be classified into two:
1. preferred shares as to assets; and
2. preferred shares as to dividends, this entitle the holder to
receive dividends on said share to the extent agreed upon before
any dividends at all are paid to the holders of common stock.
There is no guaranty, however, that the share will receive any
dividends. Under the old Corporation Law in force at the time the contract
between the petitioner and the private respondents was entered into, it was
provided that "no corporation shall make or declare any dividend except from
the surplus profits arising from its business, or distribute its capital stock or
property other than actual profits among its members or stockholders until
after the payment of its debts and the termination of its existence by
limitation or lawful dissolution. Similarly, the present Corporation Code
provides that the board of directors of a stock corporation may declare
dividends only out of unrestricted retained earnings.
Thus, the declaration of dividends is dependent upon the availability of
surplus profit or unrestricted retained earnings, as the case may be.
Preferences granted to preferred stockholders, moreover, do not give them a
lien upon the property of the corporation nor make them creditors of the
corporation, the right of the former being always subordinate to the
latter. Dividends are thus payable only when there are profits earned by the
corporation and as a general rule, even if there are existing profits, the board
of directors has the discretion to determine whether or not dividends are to
be declared. Shareholders, both common and preferred, are considered risk
takers who invest capital in the business and who can look only to what is
left after corporate debts and liabilities are fully paid.
Redeemable shares, on the other hand, are shares usually preferred,
which by their terms are redeemable at a fixed date, or at the option of
either issuing corporation, or the stockholder, or both at a certain
redemption price. A redemption by the corporation of its stock is, in a sense,
a repurchase of it for cancellation. The present Code allows redemption of
shares even if there are no unrestricted retained earnings on the books of
the corporation. This is a new provision which in effect qualifies the general
rule that the corporation cannot purchase its own shares except out of
current retained earnings. However, while redeemable shares may be

Page 494 of 1072

redeemed regardless of the existence of unrestricted retained earnings, this


is subject to the condition that the corporation has, after such redemption,
assets in its books to cover debts and liabilities inclusive of capital stock.
Redemption, therefore, may not be made where the corporation is insolvent
or if such redemption will cause insolvency or inability of the corporation to
meet its debts as they mature.

Page 495 of 1072

TOPIC: LIMITATION ON THE RETENTION OF SURPLUS PPROFITS


NORA A. BITONG, petitioner,
vs.
COURT OF APPEALS (FIFTH DIVISION), EUGENIA D. APOSTOL, JOSE A.
APOSTOL, MR. & MS. PUBLISHING CO., LETTY J. MAGSANOC, AND
ADORACION G. NUYDA, respondents.
(CA-G.R. No. 33873) July 13, 1998
292 S 503
FACTS:
Bitong had been the Treasurer and a Member of the Board of Directors
of Mr. & Ms. from the time it was incorporated on 29 October 1976 to 11 April
1989, and was the registered owner of 1,000 shares of stock out of the 4,088
total outstanding shares, petitioner complained of irregularities committed
from 1983 to 1987 by Eugenia D. Apostol, President and Chairperson of the
Board of Directors. Petitioner claimed that except for the sale of the
name Philippine Inquirer to Philippine Daily Inquirer (PDI hereafter) all other
transactions and agreements entered into by Mr. & Ms. with PDI were not
supported by any bond and/or stockholders' resolution. And, upon
instructions of Eugenia D. Apostol, Mr. & Ms. made several cash advances to
PDI on various occasions amounting to P3.276 million. On some of these
borrowings PDI paid no interest whatsoever. Despite the fact that the
advances made by Mr. & Ms. to PDI were booked as advances to an affiliate,
there existed no board or stockholders' resolution, contract nor any other
document which could legally authorize the creation of and support to an
affiliate.
Petitioner further alleged that respondents Eugenia and Jose Apostol
were stockholders, directors and officers in both Mr. & Ms. and PDI. In fact on
2 May 1986 respondents Eugenia D. Apostol, Leticia J. Magsanoc and
Adoracion G. Nuyda subscribed to PDI shares of stock at P50,000.00 each or
a total of P150,000.00. The stock subscriptions were paid for by Mr. & Ms.
and initially treated, as receivables from officers and employees. But, no
payments were ever received from respondents, Magsanoc and Nuyda.
Private respondents denied all the allegations and averred that all the
PDI shares owned by respondents Eugenia and Jose Apostol were acquired
through their own private funds and that the loan of P750,000.00 by PDI from
Mr. & Ms. had been fully paid with 20% interest per annum. And, it was PDI,
not Mr. & Ms., which loaned off P250,000.00 each to respondents Magsanoc
and Nuyda. Private respondents further argued that petitioner was not the
true party to this case, the real party being JAKA which continued to be the
true stockholder of Mr. & Ms.; hence, petitioner did not have the personality

Page 496 of 1072

to initiate and prosecute the derivative suit which, consequently, must be


dismissed.
ISSUE/S:
Whether or not the petitioner has the personality to sue.
RULING:
NO.
Petitioner Bitong has no personality to sue the respondents as she was
a stockholder thereof. As found by the Hearing Panel and affirmed by
respondent Court of Appeals, there is overwhelming evidence that despite
what appears on the certificate of stock and stock and transfer book,
petitioner was not a bona fide stockholder of Mr. & Ms. before March 1989 or
at the time the complained acts were committed to qualify her to institute a
stockholder's derivative suit against private respondents. Aside from
petitioner's own admissions, several corporate documents disclose that the
true party-in-interest is not petitioner but JAKA.
It should be emphasized that on 10 May 1983 JAKA executed, a deed of
sale over 1,000 Mr. & Ms. shares in favor of respondent Eugenio D. Apostol.
On the same day, respondent Apostol signed a declaration of trust stating
that she was the registered owner of 1,000 Mr. & Ms. shares covered by
Certificate of Stock No. 007.
The declaration of trust further showed that although respondent
Apostol was the registered owner, she held the shares of stock and dividends
which might be paid in connection therewith solely in trust for the benefit of
JAKA, her principal. It was also stated therein that being a trustee,
respondent Apostol agreed, on written request of the principal, to assign and
transfer the shares of stock and any and all such distributions or dividends
unto the principal or such other person as the principal would nominate or
appoint.
Petitioner was well aware of this trust, being the person in charge of this
documentation and being one of the witnesses to the execution of this
document. Hence, the mere alleged endorsement of Certificate of Stock No.
001 by Senator Enrile or by a duly authorized officer of JAKA to effect the
transfer of shares of JAKA to petitioner could not have been legally feasible
because Certificate of Stock No. 001 was already canceled by virtue of the
deed of sale to respondent Apostol.
There is nothing in the records which shows that JAKA had revoked the
trust it reposed on respondent Eugenia D. Apostol. Neither was there any
evidence that the principal had requested her to assign and transfer the
shares of stock to petitioner. If it was true that the shares of stock covered by
Certificate of Stock No. 007 had been transferred to petitioner, the person
who could legally endorse the certificate was private respondent Eugenia D.

Page 497 of 1072

Apostol, she being the registered owner and trustee of the shares of stock
covered by Certificate of Stock No. 007. It is a settled rule that the trustee
should endorse the stock certificate to validate the cancellation of her share
and to have the transfer recorded in the books of the corporation.
That JAKA retained its ownership of its Mr. & Ms. shares was clearly
shown by its receipt of the dividends issued in December 1986. This only
means, very obviously, that Mr. & Ms. shares in question still belonged to
JAKA and not to petitioner. For, dividends are distributed to stockholders
pursuant to their right to share in corporate profits. When a dividend is
declared, it belongs to the person who is the substantial and beneficial owner
of the stock at the time regardless of when the distribution profit was earned.
Finally, this Court takes notice of the glaring and open admissions of
petitioner made, not just seven (7) but nine (9) times, during the 22
September 1988 meeting of the board of directors that the Enriles were her
principals or shareholders, as shown by the minutes thereof which she duly
signed .
The admissions of a party against his interest inscribed upon the
record books of a corporation are competent and persuasive evidence
against him. 35 These admissions render nugatory any argument that
petitioner is a bona fide stockholder of Mr. & Ms. at any time before 1988 or
at the time the acts complained of were committed. There is no doubt that
petitioner was an employee of JAKA as its managing officer, as testified to by
Senator Enrile himself. 36 However, in the absence of a special authority from
the board of directors of JAKA to institute a derivative suit for and in its
behalf, petitioner is disqualified by law to sue in her own name. The power to
sue and be sued in any court by a corporation even as a stockholder is
lodged in the board of directors that exercises its corporate powers and not
in the president or officer thereof.

Page 498 of 1072

TOPIC: LIMITATION ON THE RETENTION OF SURPLUS PROFITS


COMMISSIONER OF INTERNAL REVENUE, petitioner,
vs.
THE COURT OF APPEALS, COURT OF TAX APPEALS and A. SORIANO
CORP., respondents
G.R. No. 108576 January 20, 1999
301 S 152
FACTS:
Sometime in the 1930s, Don Andres Soriano, a citizen and resident of
theUnited States, formed the corporation "A. Soriano Y Cia", predecessor of
ANSCOR, with a P1,000,000.00 capitalization divided into 10,000 common
shares at a par value of P100/share. ANSCOR is wholly owned and controlled
by the family of Don Andres, who are all non-resident aliens. In 1937, Don
Andres subscribed to 4,963 shares of the 5,000 shares originally issued.
On September 12, 1945, ANSCOR's authorized capital stock was
increased to P2,500,000.00 divided into 25,000 common shares with the
same par value of the additional 15,000 shares, only 10,000 was issued
which were all subscribed by Don Andres, after the other stockholders
waived in favor of the former their pre-emptive rights to subscribe to the new
issues. This increased his subscription to 14,963 common shares. A month
later, 8 Don Andres transferred 1,250 shares each to his two sons, Jose and
Andres, Jr., as their initial investments in ANSCOR.9 Both sons are foreigners.
By 1947, ANSCOR declared stock dividends. Other stock dividend
declarations were made between 1949 and December 20, 1963. 11 On
December 30, 1964 Don Andres died. As of that date, the records revealed
that he has total shareholdings of 185,154 shares 50,495 of which are
original issues and the balance of 134.659 shares as stock dividend
declarations. 13Correspondingly, one-half of that shareholdings or 92,577
shares were transferred to his wife, Doa Carmen Soriano, as her conjugal
share. The other half formed part of his estate.
After the death of Don Andres, ANSCOR increased its capital stock to P20M
and to P30M in 1966. They declared stock dividends worth 46,290 and 46,
287 shares were respectively received by the estate of Don Andres and Dona
Carmen. By 1968, ANSCOR reclassified its existing common shares into 150,
000 common and 150,00 preferred shares.
On March 31, 1968 Doa Carmen exchanged her whole 138,864 common
shares for 138,860 of the newly reclassified preferred shares. The estate of

Page 499 of 1072

Don Andres in turn, exchanged 11,140 of its common shares, for the
remaining 11,140 preferred shares, thus reducing its (the estate) common
shares to 127,727.
On June 30, 1968, pursuant to a Board Resolution, ANSCOR redeemed
28,000 common shares from the Don Andres' estate. By November 1968, the
Board further increased ANSCOR's capital stock to P75M divided into 150,000
preferred shares and 600,000 common shares. 27 About a year later, ANSCOR
again redeemed 80,000 common shares from the Don Andres' estate, 28
further reducing the latter's common shareholdings to 19,727. As stated in
the Board Resolutions, ANSCOR's business purpose for both redemptions of
stocks is to partially retire said stocks as treasury shares in order to reduce
the company's foreign exchange remittances in case cash dividends are
declared.
Revenue examiners issued a report proposing that ANSCOR be
assessed for deficiency withholding tax-at-source, pursuant to Sections 53
and 54 of the 1939 Revenue Code,for the year 1968 and the second quarter
of 1969 based on the transactions of exchange 31 and redemption of stocks.
ANSCOR alleged that it should not be held liable for tax since it availed of the
tax amnesty under Presidential Decree.
ISSUE/S:
Whether or not ANSCOR is liable for tax in declaring stock dividends
and redeeming those dividends.
RULING:
YES.
ANCOR is liable because ANSCOR's redemption of 82,752.5 stock
dividends is herein considered as essentially equivalent to a distribution of
taxable dividends for which it is LIABLE for the withholding tax-at-source.
Sec. 83(b) of the 1939 NIRC provides that:
A stock dividend representing the transfer of surplus
to capital account shall not be subject to tax. However, if a
corporation cancels or redeems stock issued as a dividend
at such time and in such manner as to make the
distribution and cancellation or redemption, in whole or in
part, essentially equivalent to the distribution of a taxable
dividend, the amount so distributed in redemption or
cancellation of the stock shall be considered as taxable
income to the extent it represents a distribution of earnings
or profits accumulated after March first, nineteen hundred
and thirteen.
For the exempting clause of Section, 83(b) to apply, it is indispensable
that:
a. there is redemption or cancellation;

Page 500 of 1072

b. the transaction involves stock dividends and


c. the "time and manner" of the transaction makes it
"essentially equivalent to a distribution of taxable
dividends."
Of these, the most important is the third.
In the instant case, there is no dispute that ANSCOR redeemed shares
of stocks from a stockholder (Don Andres) twice (28,000 and 80,000 common
shares). But where did the shares redeemed come from? If its source is the
original capital subscriptions upon establishment of the corporation or from
initial capital investment in an existing enterprise, its redemption to the
concurrent value of acquisition may not invite the application of Sec. 83(b)
under the 1939 Tax Code, as it is not income but a mere return of capital. On
the contrary, if the redeemed shares are from stock dividend declarations
other than as initial capital investment, the proceeds of the redemption is
additional wealth, for it is not merely a return of capital but a gain thereon.
It is not the stock dividends but the proceeds of its redemption that
may be deemed as taxable dividends. Here, it is undisputed that at the time
of the last redemption, the original common shares owned by the estate
were only 25,247.5. This means that from the total of 108,000 shares
redeemed from the estate, the balance of 82,752.5 (108,000 less 25,247.5)
must have come from stock dividends. Besides, in the absence of evidence
to the contrary, the Tax Code presumes that every distribution of corporate
property, in whole or in part, is made out of corporate profits such as stock
dividends. The capital cannot be distributed in the form of redemption of
stock dividends without violating the trust fund doctrine wherein the
capital stock, property and other assets of the corporation are regarded as
equity in trust for the payment of the corporate creditors. Once capital, it is
always capital. That doctrine was intended for the protection of corporate
creditors.
ANSCOR redeemed stock dividends issued just 2 to 3 years earlier. The
time alone that lapsed from the issuance to the redemption is not a sufficient
indicator to determine taxability. It is a must to consider the factual
circumstances as to the manner of both the issuance and the redemption.
The "time" element is a factor to show a device to evade tax and the scheme
of cancelling or redeeming the same shares is a method usually adopted to
accomplish the end sought.
The issuance of stock dividends and its subsequent redemption must
be separate, distinct, and not related, for the redemption to be considered a
legitimate tax scheme. Redemption cannot be used as a cloak to distribute
corporate earnings. Otherwise, the apparent intention to avoid tax becomes
doubtful as the intention to evade becomes manifest. It has been ruled that:
An operation with no business or corporate purpose is a mere devise

Page 501 of 1072

which put on the form of a corporate reorganization as a disguise for


concealing its real character, and the sole object and accomplishment of
which was the consummation of a preconceived plan, not to reorganize a
business or any part of a business, but to transfer a parcel of corporate
shares to a stockholder.
Depending on each case, the exempting provision of Sec. 83 (b) of the
1939 Code may not be applicable if the redeemed shares were issued with
bona fide business purpose, which is judged after each and every step of the
transaction have been considered and the whole transaction does not
amount to a tax evasion scheme.
ANSCOR invoked two reasons to justify the redemptions
1. the alleged "filipinization" program and
2. the reduction of foreign exchange remittances in case cash
dividends are declared.
The Court is not concerned with the wisdom of these purposes
but on their relevance to the whole transaction which can be
inferred from the outcome thereof. Again, it is the "net effect rather
than the motives and plans of the taxpayer or his corporation that
is the fundamental guide in administering Sec. 83 (b). This tax
provision is aimed at the result. It also applies even if at the time of the
issuance of the stock dividend, there was no intention to redeem it as a
means of distributing profit or avoiding tax on dividends. The existence of
legitimate business purposes in support of the redemption of stock dividends
is immaterial in income taxation. It has no relevance in determining
"dividend equivalence. Such purposes may be material only upon the
issuance of the stock dividends. The test of taxability under the exempting
clause, when it provides "such time and manner" as would make the
redemption "essentially equivalent to the distribution of a taxable dividend",
is whether the redemption resulted into a flow of wealth. If no wealth is
realized from the redemption, there may not be a dividend equivalence
treatment. In the metaphor of Eisner v. Macomber, income is not deemed
"realize" until the fruit has fallen or been plucked from the tree.
The two purposes invoked by ANSCOR, under the facts of this case is
no excuse for its tax liability. First, the alleged "filipinization" plan cannot be
considered legitimate as it was not implemented until the BIR started making
assessments on the proceeds of the redemption. Such corporate plan was
not stated in or supported by any Board Resolution but a mere afterthought
interposed by the counsel of ANSCOR. Being a separate entity, the
corporation can act only through its Board of Directors. The Board
Resolutions authorizing the redemptions state only one purpose reduction
of foreign exchange remittances in case cash dividends are declared. Not
even this purpose can be given credence. Records show that despite the
existence of enormous corporate profits no cash dividend was ever declared

Page 502 of 1072

by ANSCOR from 1945 until the BIR started making assessments in the early
1970's. Although a corporation under certain exceptions, has the prerogative
when to issue dividends, yet when no cash dividends was issued for about
three decades, this circumstance negates the legitimacy of ANSCOR's
alleged purposes. Moreover, to issue stock dividends is to increase the
shareholdings of ANSCOR's foreign stockholders contrary to its "filipinization"
plan. This would also increase rather than reduce their need for foreign
exchange remittances in case of cash dividend declaration, considering that
ANSCOR is a family corporation where the majority shares at the time of
redemptions were held by Don Andres' foreign heirs.
TOPIC: TO ENTER INTO MANAGEMENT CONTRACT
WOLRGANG AURBACH, JOHN GRIFFIN, DAVID P. WHITTINGHAM and
CHARLES CHAMSAY, petitioners,
vs.
SANITARY WARES MANUFACTURING CORPORATOIN, ERNESTO V.
LAGDAMEO, ERNESTO R. LAGDAMEO, JR., ENRIQUE R. LAGDAMEO,
GEORGE F. LEE, RAUL A. BONCAN, BALDWIN YOUNG and AVELINO V.
CRUZ, respondents.
G.R. No. 75875 December 15, 1989
180 SCRA 131
FACTS:
In 1961, Saniwares, a domestic corporation was incorporated for the
primary purpose of manufacturing and marketing sanitary wares. One of the
incorporators, Mr. Baldwin Young went abroad to look for foreign
partners,who could help in its expansion plans. On August 15, 1962, ASI, a
foreign corporation domiciled in Delaware, United States entered into an
Agreement with Saniwares and some Filipino investors whereby ASI and the
Filipino investors agreed to participate in the ownership of an enterprise
which would engage primarily in the business of manufacturing in the
Philippines and selling here and abroad vitreous china and sanitary wares.
The parties agreed that the business operations in the Philippines shall be
carried on by an incorporated enterprise and that the name of the
corporation shall initially be "Sanitary Wares Manufacturing Corporation."
The Agreement has the following provisions relevant to the issues in
these cases on the nomination and election of the directors of the
corporation:
3. Articles of Incorporation
(a) The Articles of Incorporation of the Corporation
shall be substantially in the form annexed hereto as
Exhibit A and, insofar as permitted under Philippine
law, shall specifically provide for
(1) Cumulative voting for directors:

Page 503 of 1072

xxx xxx xxx


5. Management
(a) The management of the Corporation shall be
vested in a Board of Directors, which shall consist of
nine individuals. As long as American-Standard shall
own at least 30% of the outstanding stock of the
Corporation, three of the nine directors shall be
designated by American-Standard, and the other six
shall be designated by the other stockholders of the
Corporation.
Later, the 30% capital stock of ASI was increased to 40%. The
corporation was also registered with the Board of Investments for availment
of incentives with the condition that at least 60% of the capital stock of the
corporation shall be owned by Philippine nationals.
On March 8, 1983, the annual stockholders' meeting was held. The
meeting was presided by Baldwin Young. After disposing of the preliminary
items in the agenda, the stockholders then proceeded to the election of the
members of the board of directors. The ASI group nominated three persons
namely; Wolfgang Aurbach, John Griffin and David P. Whittingham. The
Philippine investors nominated six, namely; Ernesto Lagdameo, Sr., Raul A.
Boncan, Ernesto R. Lagdameo, Jr., George F. Lee, and Baldwin Young. Mr.
Eduardo R, Ceniza then nominated Mr. Luciano E. Salazar, who in turn
nominated Mr. Charles Chamsay. The chairman, Baldwin Young ruled the last
two nominations out of order on the basis of section 5 (a) of the Agreement,
the consistent practice of the parties during the past annual stockholders'
meetings to nominate only nine persons as nominees for the nine-member
board of directors, and the legal advice of Saniwares' legal counsel. The ASI
Group, Luciano E. Salazar and other stockholders, allegedly representing 53
or 54% of the shares of Saniwares, decided to continue the meeting at the
elevator lobby of the American Standard Building. The continued meeting
was presided by Luciano E. Salazar, while Andres Gatmaitan acted as
Secretary. On the basis of the cumulative votes cast earlier in the meeting,
the ASI Group nominated its four nominees; Wolfgang Aurbach, John Griffin,
David Whittingham and Charles Chamsay. Luciano E. Salazar voted for
himself, thus the said five directors were certified as elected directors by the
Acting Secretary, Andres Gatmaitan, with the explanation that there was a
tie among the other six (6) nominees for the four (4) remaining positions of
directors and that the body decided not to break the tie.
ISSUE/S:
Whether or not a joint venture or a corporation was established?
Whether or not the ASI Group may vote their additional 10% equity
during elections of Saniwares' board of directors.

Page 504 of 1072

RULING:
A joint venture was established by the parties. In the instant cases, our
examination of important provisions of the Agreement as well as the
testimonial evidence presented by the Lagdameo and Young Group shows
that the parties agreed to establish a joint venture and not a corporation.
The history of the organization of Saniwares and the unusual arrangements
which govern its policy making body are all consistent with a joint venture
and not with an ordinary corporation. As stated by the SEC:
According to the unrebutted testimony of Mr. Baldwin Young, he
negotiated the Agreement with ASI in behalf of the Philippine nationals. He
testified that ASI agreed to accept the role of minority vis-a-vis the Philippine
National group of investors, on the condition that the Agreement should
contain provisions to protect ASI as the minority.
An examination of the Agreement shows that certain provisions were
included to protect the interests of ASI as the minority. For example, the vote
of 7 out of 9 directors is required in certain enumerated corporate acts [Sec.
3 (b) (ii) (a) of the Agreement]. ASI is contractually entitled to designate a
member of the Executive Committee and the vote of this member is required
for certain transactions [Sec. 3 (b) (i)].
The Agreement also requires a 75% super-majority vote for the
amendment of the articles and by-laws of Saniwares [Sec. 3 (a) (iv) and (b)
(iii)]. ASI is also given the right to designate the president and plant manager
[Sec. 5 (6)]. The Agreement further provides that the sales policy of
Saniwares shall be that which is normally followed by ASI [Sec. 13 (a)] and
that Saniwares should not export "Standard" products otherwise than
through ASI's Export Marketing Services [Sec. 13 (6)]. Under the Agreement,
ASI agreed to provide technology and know-how to Saniwares and the latter
paid royalties for the same.
It is pertinent to note that the provisions of the Agreement requiring a
7 out of 9 votes of the board of directors for certain actions, in effect gave
ASI (which designates 3 directors under the Agreement) an effective veto
power. Furthermore, the grant to ASI of the right to designate certain officers
of the corporation; the super-majority voting requirements for amendments
of the articles and by-laws; and most significantly to the issues of the case,
the provision that ASI shall designate 3 out of the 9 directors and the other
stockholders shall designate the other 6, clearly indicate that there are two
distinct groups in Saniwares, namely ASI, which owns 40% of the capital
stock and the Philippine National stockholders who own the balance of 60%,
and that 2) ASI is given certain protections as the minority stockholder.
As to the second issue, as in other joint venture companies, the extent of
ASI's participation in the management of the corporation is spelled out in the
Agreement. Section 5(a) hereof says that three of the nine directors shall be
designated by ASI and the remaining six by the other stockholders, i.e., the

Page 505 of 1072

Filipino stockholders. This allocation of board seats is obviously in


consonance with the minority position of ASI.
Having entered into a well-defined contractual relationship, it is
imperative that the parties should honor and adhere to their respective
rights and obligations there under. The clearly established minority position
of ASI and the contractual allocation of board seats cannot be disregarded.
The rights of the stockholders to cumulative voting should also be protected.
To allow the ASI Group to vote their additional equity to help elect even
a Filipino director who would be beholden to them would obliterate their
minority status as agreed upon by the parties. As aptly stated by the
appellate court:
... ASI, however, should not be allowed to interfere in the
voting within the Filipino group. Otherwise, ASI would be able to
designate more than the three directors it is allowed to designate
under the Agreement, and may even be able to get a majority of
the board seats, a result which is clearly contrary to the
contractual intent of the parties.
Such a ruling will give effect to both the allocation of the board seats
and the stockholder's right to cumulative voting. Moreover, this ruling will
also give due consideration to the issue raised by the appellees on possible
violation or circumvention of the Anti-Dummy Law (Com. Act No. 108, as
amended) and the nationalization requirements of the Constitution and the
laws if ASI is allowed to nominate more than three directors.
Equally important as the consideration of the contractual intent of the
parties is the consideration as regards the possible domination by the foreign
investors of the enterprise in violation of the nationalization requirements
enshrined in the Constitution and circumvention of the Anti-Dummy Act. In
this regard, petitioner Salazar's position is that the Anti-Dummy Act allows
the ASI group to elect board directors in proportion to their share in the
capital of the entity. It is to be noted, however, that the same law also limits
the election of aliens as members of the board of directors in proportion to
their allowance participation of said entity. In the instant case, the foreign
Group ASI was limited to designate three directors. This is the allowable
participation of the ASI Group. Hence, in future dealings, this limitation of six
to three board seats should always be maintained as long as the joint
venture agreement exists considering that in limiting 3 board seats in the 9man board of directors there are provisions already agreed upon and
embodied in the parties' Agreement to protect the interests arising from the
minority status of the foreign investors.
The insinuation that the ASI Group may be able to control the
enterprise under the cumulative voting procedure cannot, however, be

Page 506 of 1072

ignored. The validity of the cumulative voting procedure is dependent on the


directors thus elected being genuine members of the Filipino group, not
voters whose interest is to increase the ASI share in the management of
Saniwares. The joint venture character of the enterprise must always be
taken into account, so long as the company exists under its original
agreement. Cumulative voting may not be used as a device to enable
ASI to achieve stealthily or indirectly what they cannot accomplish
openly. There are substantial safeguards in the Agreement which are
intended to preserve the majority status of the Filipino investors as well as to
maintain the minority status of the foreign investors group as earlier
discussed. They should be maintained.
TOPIC: TO ENTER INTO MANAGEMENT CONTRACT
PHILIPPINE NATIONAL BANK, plaintiff-appellant,
vs. .
PRODUCERS' WAREHOUSE ASSOCIATION, defendant-appellee.
G.R. No. L- 16510 January 9, 1922
42 Phil 608
FACTS:
The plaintiff is a corporation organized under the banking laws of the
Philippine Islands with its principal office in the city of Manila. The defendant
is a domestic corporation doing a general warehouse business and domiciled
at Manila, and the Philippine Fiber and Produce Company, to which we will
hereafter refer as the Produce Company, is another domestic corporation
with its principal office also at Manila.
In May, 1916, the defendant, as party of the first part, entered into a
written contract with the Produce Company, as party of the second part, in
and by which "the above-named party of the second part is hereby named,
constituted, and appointed as the general manager of the business of the
party of the first part, in all of the branches thereof, with the duties, powers,
authority and compensation hereinafter provided." "The said party of the
second part shall exercise a general and complete supervision over and
management of the business of the party of the first part," and "shall direct,
manage, promote and advance the said business, subject only to the control
and instructions of the board of directors of the party of the first part." That
said party of the second part, as general manager, shall have all powers and
authorities necessary, proper or usual for the due transaction of the business
of the party of the first part, including the power to sign the company's
name, save and except such power or authority as shall have been expressly
reserved to itself, by the board of directors of the party of the first part,
provided "that such reservations by the board of directors shall not be

Page 507 of 1072

employed to unreasonably hamper or interfere with the due management of


said business and shall, at no time, reduce the powers and authorities of said
general manager below the usual and ordinary standard in business of like
kind."
It is then agreed that the Produce Company shall have an annual salary
of P7,500 for its services as general manager, and that the defendant will
also pay the local agents of the Produce Company P300 per month for their
services. The agreement also provides that it shall remain in force and effect
ten years from date, with the right of the Produce Company to renew it for a
further period of one to ten years at its option. In the months of November
and December, 1918, and while the contract was in force and effect, the
defendant duly issued to the Produce Company its negotiable quedans Nos.
1255, 1266, 1273, 1275, 1277, 1279, and 1283 for 15,699.34 piculs of copra
in and by which, subject to the terms and conditions therein stated, it agreed
to deliver that amount of copra to the Produce Company or its order.
The Produce Company arranged for an overdraft with the plaintiff of P1,
000,000. To secure such overdraft, and as collateral from and after the dates
of their issuance, the quedans in question were endorsed in blank by the
Produce Company, and delivered to the plaintiff, which became and is now
the owner and holder thereof. Without making a tender of any charges, on
March 21, 1919, the plaintiff requested the delivery of the copra described in
the respective quedans, and, for its failure to do so, commenced this action
on April 23, 1919, to recover its value alleged to be P240,689, with interest
from March 21, 1919, at the rate of 6 per cent per annum. it is alleged that,
in good faith, the plaintiff purchased these quedans, and that it is the owner,
and recites all of the conditions printed on the back, and made a part of the
quedans.
It is then alleged that on July 30, 1919, the plaintiff requested the
defendant to register the quedans in the name of the plaintiff, and to deliver
to it the 14,587.19 piculs of copra, and, upon that date, that it had offered to
satisfy any lien that defendant might have, to surrender the receipts with
such endorsement that it might require, and the receipt therefore, when the
goods were delivered, if such signature is requested by the defendant. "That
the defendant refused to comply with the demands of the plaintiff, stating
that it could not deliver the goods mentioned in the receipts as said goods
are not in the warehouse, said defendant still refusing to make such delivery.
ISSUE/S:
Whether or not the officers of the defendant have the authority to
issue receipts in the name of the defendant?
RULING:
The Produce Company was the general manager of the defendant's
warehouse business, and that it had authority to issue quedans in its name,

Page 508 of 1072

and as its corporate act and deed. That the quedans in question are duly
authenticated, and were duly issued by the defendant to, and in the name of,
the Produce Company, and when issued were duly endorsed, and delivered
to the plaintiff for value. For aught that appears in the record, the bank was
acting in good faith, and the quedans were duly issued, endorsed and
delivered to it as collateral in the ordinary course of business. Although there
may have been fraud, there is no allegation or proof that the bank was a
party to it, or had any knowledge of it, and this court has no right to assume
that the bank was a party to a fraud. Giving to the quedans their legal force
and effect, it must follow that at the time the demand was made; the bank
was the owner and entitled to the possession of the copra therein described.
The receipts call for 15,699.34 piculs of copra, but plaintiff admits that, with
its consent, 1,112.15 piculs of copra, of the declared value of P18, 350, were
delivered to the Produce Company from and out of receipt No. 1255. This
would leave 14,587.19 piculs of copra evidenced by the quedans.
TOPIC: TO ENTER INTO MANAGEMENT CONTRACT
NIELSON & COMPANY, INC., plaintiff-appellant,
vs.
LEPANTO CONSOLIDATED MINING COMPANY, defendant-appellee.
G.R. No. L-21601
December 28, 1968
26 SCRA 541
FACTS:
Under the contract, Nielson had agreed, for a period of five years, with
the right to renew for a like period, to explore, develop and operate the
mining claims of Lepanto, and to mine, or mine and mill, such pay ore as
may be found therein and to market the metallic products recovered there
from which may prove to be marketable, as well as to render for Lepanto
other services specified in the contract. Nielson was to take complete charge
subject at all times to the general control of the Board of Directors of
Lepanto, of the exploration and development of the mining claims, of the
hiring of a sufficient and competent staff and of sufficient and capable
laborers, of the prospecting and development of the mine, of the erection
and operation of the mill, and of the benefication and marketing of the
minerals found on the mining properties; and in carrying out said obligation
Nielson should proceed diligently and in accordance with the best mining
practice. In connection with its work Nielson was to submit reports, maps,
plans and recommendations with respect to the operation and development
of the mining properties, make recommendations and plans on the erection
or enlargement of any existing mill, dispatch mining engineers and
technicians to the mining properties as from time to time may reasonably be
required to investigate and make recommendations without cost or expense
to Lepanto. Nielson was also to "act as purchasing agent of supplies,

Page 509 of 1072

equipment and other necessary purchases by Lepanto, provided, however,


that no purchase shall be made without the prior approval of Lepanto; and
provided further, that no commission shall be claimed or retained by Nielson
on such purchase"; and "to submit all requisition for supplies, all constricts
and arrangement with engineers, and staff and all matters requiring the
expenditures of money, present or future, for prior approval by Lepanto; and
also to make contracts subject to the prior approve of Lepanto for the sale
and marketing of the minerals mined from said properties, when said
products are in a suitable condition for marketing.
Lepanto contends that the management contract in question is one of
agency because:
1. Nielson was to manage and operate the mining
properties and mill on behalf, and for the account, of
Lepanto; and
2. Nielson was authorized to represent Lepanto in
entering, on Lepanto's behalf, into contracts for the
hiring of laborers, purchase of supplies, and the sale
and marketing of the ores mined.
All these, Lepanto claims, show that Nielson was, by the terms of the
contract, destined to execute juridical acts not on its own behalf but on
behalf of Lepanto under the control of the Board of Directors of Lepanto "at
all times". Hence Lepanto claims that the contract is one of agency. Lepanto
then maintains that an agency is revocable at the will of the principal (Article
1733 of the Old Civil Code), regardless of any term or period stipulated in the
contract, and it was in pursuance of that right that Lepanto terminated the
contract in 1945 when it took over and assumed exclusive management of
the work previously entrusted to Nielson under the contract. Lepanto finally
maintains that Nielson as an agent is not entitled to damages since the law
gives to the principal the right to terminate the agency at will.
ISSUE/S:
Whether or not the contract was a contract of agency or management
contract.
RULING:
Statements in the annual report for 1936, and from the provision of
paragraph XI of the Management contract, that the employment by Lepanto
of Nielson to operate and manage its mines was principally in consideration
of the know-how and technical services that Nielson offered Lepanto. The
contract thus entered into pursuant to the offer made by Nielson and
accepted by Lepanto was a "detailed operating contract". It was not a
contract of agency. Nowhere in the record is it shown that Lepanto
considered Nielson as its agent and that Lepanto terminated the
management contract because it had lost its trust and confidence in Nielson.

Page 510 of 1072

The contention of Lepanto that it had terminated the management


contract in 1945, following the liberation of the mines from Japanese control,
because the relation between it and Nielson was one of agency and as such
it could terminate the agency at will, is, therefore, untenable. On the other
hand, it can be said that, in asserting that it had terminated or cancelled the
management contract in 1945, Lepanto had thereby violated the express
terms of the management contract. The management contract was renewed
to last until January 31, 1947, so that the contract had yet almost two years
to go upon the liberation of the mines in 1945. There is no showing that
Nielson had ceased to prosecute the operation and development of the
mines in good faith and in accordance with approved mining practice which
would warrant the termination of the contract upon ninety days written
notice. In fact there was no such written notice of termination. It is an
admitted fact that Nielson ceased to operate and develop the mines because
of the war a cause beyond the control of Nielson. Indeed, if the
management contract in question was intended to create a relationship of
principal and agent between Lepanto and Nielson, paragraph XI of the
contract should not have been inserted because, as provided in Article 1733
of the old Civil Code, agency is essentially revocable at the will of the
principal that means, with or without cause. But precisely said paragraph
XI was inserted in the management contract to provide for the cause for its
revocation. The provision of paragraph XI must be given effect.
By express stipulation of the parties, the management contract in
question is not revocable at the will of Lepanto. We rule that this
management contract is not a contract of agency as defined in Article 1709
of the old Civil Code, but a contract of lease of services as defined in Article
1544 of the same Code. This contract cannot be unilaterally revoked by
Lepanto.

Page 511 of 1072

TOPIC: TO ENTER INTO MANAGEMENT CONTRACT


J. M. TUASON & CO., INC., represented by it Managing PARTNER,
GREGORIA ARANETA, INC., plaintiff-appellee,
vs.
QUIRINO BOLAOS, defendant-appellant.
G.R. No. L-4935
May 28, 1954
FACTS:
Plaintiff filed a complaint for recovery of possession of land, the
complaint was amended three times to conform with respect to the extent
and description of the land sought to be recovered. The original complaint
described the land as a portion of a lot registered in plaintiff's name under
Transfer Certificate of Title No. 37686 containing an area of 13 hectares more
or less. But the complaint was amended by reducing the area of 6 hectares,
more or less, after the defendant had indicated the plaintiff's surveyors the
portion of land claimed and occupied by him. The second amendment
became necessary and was allowed following the testimony of plaintiff's
surveyors that a portion of the area was embraced in another certificate of
title, which was plaintiff's Transfer Certificate of Title No. 37677. And still
later, in the course of trial, after defendant's surveyor and witness, Quirino
Feria, had testified that the area occupied and claimed by defendant was
about 13 hectares, plaintiff again, with the leave of court, amended its
complaint to make its allegations conform to the evidence.

Page 512 of 1072

Defendant, in his answer, sets up prescription and title in him thru


"open, continuous, exclusive and public and notorious possession (of land in
dispute) under claim of ownership, adverse to the entire world by defendant
and his predecessor in interest" from "time in-memorial". The answer further
alleges that registration of the land in dispute was obtained by plaintiff or its
predecessors in interest thru "fraud or error and without knowledge (of) or
interest either personal or thru publication to defendant and/or predecessors
in interest." The answer therefore prays that the complaint be dismissed with
costs and plaintiff required to reconvey the land to defendant or pay its
value.
The trial court ruled in favor of the plaintiff.
ISSUE/S:
Whether or not the case was raised by the proper party.
RULING
YES.
The case was properly raised by the managing partner of J. M. Tuason
and Co. Inc., which is GREGORIA ARANETA, INC. What the Rules of Court
require is that an action be brought in the name of, but not necessarily by,
the real party in interest. (Section 2, Rule 2.) In fact the practice is for an
attorney-at-law to bring the action that is to file the complaint, in the name
of the plaintiff. That practice appears to have been followed in this case,
since the complaint is signed by the law firm of Araneta and Araneta,
"counsel for plaintiff" and commences with the statement "comes now
plaintiff, through its undersigned counsel."
It is true that the complaint also states that the plaintiff is "represented
herein by its Managing Partner Gregorio Araneta, Inc.", another corporation,
but there is nothing against one corporation being represented by another
person, natural or juridical, in a suit in court. The contention that Gregorio
Araneta, Inc. cannot act as managing partner for plaintiff on the theory that
it is illegal for two corporations to enter into a partnership is without merit,
for the true rule is that "though a corporation has no power to enter into a
partnership, it may nevertheless enter into a joint venture with another
where the nature of that venture is in line with the business authorized by its
charter." (Wyoming-Indiana Oil Gas Co. vs. Weston, 80 A. L. R., 1043, citing 2
Fletcher Cyc. of Corp., 1082.) There is nothing in the record to indicate that
the venture in which plaintiff is represented by Gregorio Araneta, Inc. as "its
managing partner" is not in line with the corporate business of either of
them.

Page 513 of 1072

TOPIC: ULTRA VIRES ACTS


HEIRS OF ANTONIO PAEL
vs.
COURT OF APPEALS
G.R. No. 133843
November 11, 2003
325 SCRA 341, 358
FACTS:
The facts reveal that on December 9, 1993, Maria Destura filed a
complaint before the Regional Trial Court for the annulment of the
memorandum of agreement (MOA). The MOA stated that Chin and Mallari
had a buyer of the lots and they promised to pay Destura one hundred
million pesos (P100,000,000.00) upon finality of the sale; that the sale did
not materialize and the payment of the promised amount has become
uncertain, to the prejudice of the Destura spouses.
On January 24, 1995, the trial court rendered a judgment by default.
The trial court nullified the MOA in question. Chin and Mallari, assisted by
new counsel, Atty. Samuel Alentaje, filed before the Court of Appeals a
Petition for Annulment of Judgment. On April 29, 1998, the Court of Appeals
rendered a decision in favor of Chin and Mallari. Further, the appellate court
upheld the validity of the sale. The Court rendered a Decision denying both
petitions and affirming the title of Chin and Mallari over the property. The
Court further denied the motions for reconsideration, but granted the motion

Page 514 of 1072

for intervention filed by UP. The Court remanded the case to the Court of
Appeals for reception of evidence on the conflicting claims over the property
in question by Chin and Mallari as against UP.
ISSUE:
Should the judgment of ownership that was already ruled be
upheld against UP?
RULING:
NO.
Although there was already a court judgment, the disputed property,
however, is part of the UP Diliman Campus. It was established, after the
survey conducted by the Department of Environment and Natural Resources,
National Capital Region (DENR-NCR) that the property claimed by Chin and
Mallari overlaps the property covered by UPs title. The superiority of UPs
title over that of the Paels has been recognized by the courts in an earlier
case filed by Roberto Pael, et al. against UP.
It is judicial notice that the legitimacy of UPs title has been settled in
several other cases decided by this Court. The Decision upholding the
superior rights of Chin and Mallari over those of the petitioners was based on
its findings on the sale of the property by the Paels and a certain Menor to
Chin and Mallari, overlooking the ownership of U.P.

Page 515 of 1072

TOPIC: ULTRA VIRES ACTS


PILIPINAS LOAN COMPANY, INC.
vs.
HON. SECURITES AND EXCHANGE COMMISSION AND FILIPINAS
PAWNSHOP, INC.
G.R. No. 104720
April 4, 2001

FACTS:
On September 11, 1990, private respondent (Filipinas Pawnshop, Inc.)
filed a complaint against petitioner with the Prosecution and Enforcement
Department (PED) of the SEC docketed as PED CASE No. 90-0737.
The complaint alleged that: (1) petitioner, contrary to the restriction
set by the Commission, has been operating and doing business as a
pawnbroker, pawnshop or sanglaan in the same neighborhood where
private respondent has had its own pawnshop for 30 years in violation of its
primary purpose and without the imprimatur of the Central Bank to engage
in the pawnshop business thereby causing unjust and unfair competition with
private respondent; and (2) the business name of petitioner, PILIPINAS
Loan, bears similarity in spelling and phonetics with the corporate name of
private respondent, FILIPINAS Pawnshop, creating constant confusion in the
minds of the public and the customers of private respondent. In the same
complaint, private respondent urged the SEC to: (1) order petitioner to
change its business name, Pilipinas Loan, and cease from using it in the near
future; (2) order Pilipinas Loan to cease and desist from engaging in the
business of pawn broking as defined under PD No. 114; and (3) impose upon

Page 516 of 1072

the director, officers, employees or persons responsible such penalties as


may be proper under the law.
On April 8, 1991, the PED of the SEC issued an Order directing
petitioner to amend its articles of incorporation by changing the word
Pilipinas in its corporate name, and to cease and desist from further
engaging in the business of pawnshop or sanglaan.
On August 13, 1991, the SEC en banc rendered a Decision affirming
with modification the aforementioned Order. On October 31, 1991 the Court
of Appeals rendered a Decision affirming with modification the decision of
the SEC. On October 31, 1991 the Court of Appeals rendered a Decision
affirming with modification the decision of the SEC.
Hence, this petition.
ISSUE:
Whether it is ultra vires for SEC to determine whether a
registered entity is violating PD 114.
RULING:
No.
A corporation, under the Corporation Code, has only such powers as
are expressly granted to it by law and by its articles of incorporation, those
which may be incidental to such conferred powers, those reasonably
necessary to accomplish its purposes and those which may be incident to its
existence. In the case at bar, the limit of the powers of petitioner as a
corporation is very clear, it is categorically prohibited from engaging in
pawn broking as defined under PD 114. Hence, in determining what
constitutes pawn brokerage, the relevant law to consider is PD 114.
Indispensable therefore to the determination of whether or not
petitioner had violated its articles of incorporation, was an inquiry by the SEC
if petitioner was holding out itself to the public as a pawnshop. It must be
stressed that the determination of whether petitioner violated PD 114 was
merely incidental to the regulatory powers of the SEC, to see to it that a
corporation does not go beyond the powers granted to it by its articles of
incorporation.
Jurisprudence has laid down the principle that it is the certificate of
incorporation that gives juridical personality to a corporation and places it
within SEC jurisdiction.

Page 517 of 1072

TOPIC: ULTRA VIRES ACTS


ERNESTINA CRISOLOGO-JOSE
vs.
COURT OF APPEALS and RICARDO S. SANTOS, JR.
G.R. No. 80599 September 15, 1989
177 SCRA 594
FACTS:
The president of Movers Enterprises, to accommodate its clients
Spouses Ong, issued a check in favor of petitioner Crisologo-Jose. This
was in consideration of a quitclaim by petitioner over a parcel of land, which
the GSIS agreed to sell to spouses Ong, with the understanding that
upon approval of the compromise agreement, the check will be
encashed accordingly.
As the compromise agreement wasn't approved
during the expected period of time, the aforesaid check was replaced with
another one for the same value.
Upon deposit though of the checks
by petitioner, it was dishonored. This prompted the petitioner to file a case
against Atty. Bernares and Santos for violation of BP22. Meanwhile, during
the preliminary investigation, Santos tried to tender a cashiers check
for the value of the dishonored check but petitioner refused to accept such.
This was consigned by Santos with the clerk of court and he instituted
charges against petitioner.
The trial court held that consignation wasn't applicable to the case at
bar but was reversed by the CA.
On appeal, petitioner averred that it is not Santos who is the
accommodation party to the instrument but the corporation itself.
ISSUE:
Whether or not Court of Appeals erred in holding that private
respondent, one of the signatories of the check issued under the
account of Mover Enterprises, Inc., is an accommodation party
under the Negotiable Instruments Law and a debtor of petitioner
to the extent of the amount of said check.
RULING:
NO.
Under the Negotiable Instruments Law, Section 29 thereof provides:
Sec.
29.
Liability
of
accommodation
party
an
accommodation party is one who has signed the
instrument as maker, drawer, acceptor, or indorser,
without receiving value therefore, and for the purpose of

Page 518 of 1072

lending his name to some other person. Such a person is


liable on the instrument to a holder for value,
notwithstanding such holder, at the time of taking the
instrument, knew him to be only an accommodation party.
Consequently, to be considered an accommodation party, a person
must
(1) Be a party to the instrument, signing as maker, drawer,
acceptor, or indorser,
(2) Not receive value therefore, and
(3) Sign for the purpose of lending his name for the credit
of some other person.
Based on the foregoing requisites, it is not a valid defense that the
accommodation party did not receive any valuable consideration when he
executed the instrument. But assuming arguendo that the corporation is
the accommodation party, it cannot be held liable to the check issued
in favor of petitioner. The rule on accommodation party doesn't include or
apply to corporations which are accommodation parties. This is because the
issue or indorsement of another is ultra vires. Hence, one who has taken the
instrument with knowledge of the accommodation nature thereof cannot
recover against a corporation where it is only an accommodation party.
If the form of the instrument, or the nature of the transaction, is such as to
charge the indorsee with the knowledge that the issue or indorsement of
the instrument by the corporation is for the accommodation of another,
he cannot recover against the corporation thereon.
By way of exception, an officer or agent of a corporation shall
have the power to execute or indorse a negotiable paper in the name
of the corporation for the accommodation of a third party only is
specifically authorized to do so. Corollarily, corporate officers have no
power to execute for mere accommodation a negotiable instrument
of the corporation for their individual debts and transactions arising
from or in relation to matters in which the corporation has no
legitimate concern. Since such accommodation paper cannot be
enforced
against
the corporation, the signatories thereof shall be
personally liable therefore, as well as the consequences arising from their
acts in connection therewith.

Page 519 of 1072

TOPIC: ULTRA VIRES ACTS


IRINEO G. CARLOS
vs.
MINDORO SUGAR CO., ET AL.
G.R. No. L-36207
October 26, 1932
FACTS:
Appellant Irineo Carlos brought this action to recover from the
defendants the value of four bonds, with due and unpaid interest thereon,
issued by the Mindoro Sugar Company (MSC) and placed in trust with the
Philippine Trust Company (PTC) which, in turn, guaranteed them for value
received. Said plaintiff appealed from the judgment rendered by the CFI of
Manila absolving the defendants from the complaint, excepting the MSC,
which was sentenced to pay the value of the four bonds with interest at 8 per
cent per annum, plus costs.
On November 17, 1917, PTC, adopted a resolution authorizing its
president to purchase at par and in the name and for the use of the trust
corporation all or such part as he may deem expedient, of the bonds in the
value of P3,000,000 that the MSC was about to issue, and to resell them,
with or without the guarantee of said trust corporation, at a price not less
than par, and to guarantee to the Philippine National Bank the payment of
the indebtedness to said bank by the MSC, up to P2,000,000. In pursuance
of this resolution, on December 21, 1917, MSC executed in favor of the PTC
the deed of trust, transferring all of its property to it in consideration of the
bonds it had issued to the value of P3,000,000, the value of each bond being
$1,000, which par value, with interest at 8 per cent per annum, the PTC had
guaranteed to the holders, and in consideration, furthermore, of said trust
corporation having guaranteed to the Philippine National Bank all the
obligations contracted by MSC, up to the aforesaid amount of P2,000,000.
The aforementioned deed was approved by his Excellency, the GovernorGeneral, upon recommendation of the Secretary of Agriculture and Natural
Resources, and in accordance with the provisions of Act No. 2720 of the
Philippine Legislature.
The PTC paid the appellant, upon presentation of the coupons, the
stipulated interest from the date of their maturity until the 1st of July, 1928,
when it stopped payments; and thenceforth it alleged that it did not deem
itself bound to pay such interest or to redeem the obligation because the
guarantee given for the bonds was illegal and void.
The appellant now contends that the judgment appealed from is
untenable, assigning the following errors:

Page 520 of 1072

1) The lower court erred in holding that the Philippine Trust


Company has no power to guarantee the obligation of
another juridical personality, for value received.
2) The lower court erred in not recognizing the validity and
effect of the guarantee subscribed by the Philippine Trust
Company for the payment of the four bonds claimed in the
complaint, endorsed upon them, and in absolving said
institution from the complaint.
ISSUE/S:
Whether the Philippine Trust Company acted within its corporate
powers in acquiring and guaranteeing the four bonds.
RULING:
The Supreme Court held that: Firstly, the Philippine Trust Company,
although secondarily engaged in banking, was primarily organized as a trust
corporation with full power to acquire personal property such as the bonds in
question according to both section 13 (par. 5) of the Corporation Law and its
duly registered by-laws and articles of incorporation; Secondly, that being
thus authorized to acquire the bonds, it was given implied power to
guarantee them in order to place them upon the market under better, more
advantageous conditions, and thereby secure the profit derived from their
sale:
It is not; however, ultra vires for a corporation to enter into
contracts of guaranty or suretyship where it does so in the
legitimate furtherance of its purposes and business. And it is well
settled that where a corporation acquires commercial paper or
bonds in the legitimate transaction of its business it may sell
them, and in furtherance of such a sale it may, in order to make
them the more readily marketable, indorse or guarantee their
payment. (7 R. C. L., p. 604 and cases cited.)
There are other considerations leading to the same result even in the
supposition that the Philippine Trust Company did not acquire the bonds in
question, but only guaranteed them. In such a case the guarantee of these
bonds would at any rate, be valid and the said corporation would be bound
to pay the appellant their value with the accrued interest in view of the fact
that they become due on account of the lapse of sixty (60) days, without the
accrued interest due having been paid; and the reason is that it is estopped
from denying the validity of its guarantee.
When a contract is not on its face necessarily beyond the scope of the
power of the corporation by which it was made, it will, in the absence of
proof to the contrary, be presumed to be valid. Corporations are presumed to
contract within their powers. The doctrine of ultra vires, when invoked for or

Page 521 of 1072

against a corporation, should not be allowed to prevail where it would defeat


the ends of justice or work a legal wrong. (Coleman vs. Hotel de France Co.,
29 Phil., 323.)
Guaranties of payment of bonds taken by a loan and trust company in
the ordinary course of its business, made in connection with their sale, are
not ultra vires, and are binding. (Broadway Nat. Bank vs. Baker, 57 N. E., p.
603.)

Page 522 of 1072

TOPIC: ULTRA VIRES ACTS


MARIA CLARA PIROVANO ET AL.
vs.
THE DE LA RAMA STEAMSHIP CO.
G.R. No. L-5377
December 29, 1954
96 Phil. 335
FACTS:
Enrico Pirovano, who was executed during the Japanese occupation,
was the president of the defendant company.
He managed the said
company until it became a multi-million corporation. After his death, the
companys Board of Directors then passed a resolution that out of the
proceeds, the sum of P400,000 be set aside for equal division among the 4
minor children, convertible into shares of stock of the De la Rama Steamship
Company, at par and, for that purpose, that the present registered
stockholders of the corporation be requested to waive their preemptive right
to 4,000 shares of the unissued stock of the company in order to enable each
of the 4 minor heirs to obtain 1,000 shares at par.
If the Pirovano children would be given shares of stock, the voting
strength of the 5 daughters of Don Esteban would be adversely affected
and Mrs. Pirovano would have a voting power twice that of her sisters.
Lourdes de la Rama wrote secretary of the corporation, Atty. Marcial
Lichauco, asking him to cancel the waiver she supposedly gave of her preemptive rights. The company amended the resolution turning it into a loan
with 5% interest payable when the obligation can be met. The company
revoked its donation of the life premium proceeds since it is not in
compliance with the SEC. The minor children of the late Enrico represented
by their mother and judicial guardian demanded the payment of the credit
due them as of December 31, 1951, amounting to P564, 980.89. The trial
court then ruled that the contract or donation is not ultra vires
ISSUE/S:
Whether the corporation donation of the proceeds of insurance
policies is an ultra vires act.
RULING:
No.
The donation of the proceeds was valid and binding. Accordingly, a
remunerative donation is that which is made to a person in consideration of
his merits or for services rendered to the donor, provided they do not
constitute recoverable debts, or that in which a burden less than the value of
the thing given is imposed upon the donee, is also a donation." (Art. 619, old
Civil Code). In donations made to a person for services rendered to the

Page 523 of 1072

donor, the donor's will is moved by acts which directly benefit him. The
motivating cause is gratitude, acknowledgment of a favor, a desire to
compensate. The donation has reached the stage of perfection which is valid
and binding upon the corporation and as such cannot be rescinded unless
there is exists legal grounds for doing so. The donation was embodied in a
resolution duly approved by the Board of Directors on January 6, 1947.
There are 2 reasons given for the rescission of donation in the
resolution of the corporation adopted on March 8, 1951 - valid and legal as to
justify the rescission, which are:
(1) Corporation failed to comply with the conditions to which the
above donation was made subject.
(2) In the opinion of the Securities and Exchange Commission said
donation is ultra vires.
Further, the articles of incorporation state that to invest and deal with
the moneys of the company and immediately required, in such manner as
from time to time may be determined. Additionally, to aid in any other
manner any person, association, or corporation of which any obligation or in
which any interest is held by this corporation or in the affairs or prosperity of
which this corporation has a lawful interest.
By ratification the infirmity of the corporate act has been obliterated
thereby making it perfectly valid and enforceable. This is especially so if the
donation is not merely executory but executed and consummated and no
creditors are prejudice, or if there are creditors affected, the latter has
expressly given their confirmity

Page 524 of 1072

TOPIC: ULTRA VIRES ACTS


REPUBLIC OF THE PHILIPPINES
Vs.
ACOJE MINING COMPANY, INC.
G.R. No. L-18062
February 28, 1963
3 SCRA 361
FACTS:
In its mining camp in Zambales, Acoje Mining Company requested the
opening of a post office to service the employees living in the camp. This was
then agreed to by the Director of Posts, provided that in the meantime those
funds are not available, the company would provide for all essential
equipment and assign a responsible employee to perform the duties of a
postmaster. He also added that the company shall assume direct
responsibility for whatever pecuniary loss may be suffered by the Bureau
of Posts by reason of the dishonesty or negligence of the employee assigned.
A Resolution by the Acoje Board of Directors was passed. Hilario
Sanchez, the postmaster assigned, , went on leave and never returned. It
was soon discovered that a shortage was incurred in the amount of P
13,867.24, apparently embezzled by Sanchez. The Bureau of Posts now sues
for the shortage. On the other hand, Acoje Mining denied its liability
contending that the resolution issued by the board was ultra-vires and its
liability if any would only be that of a guarantor only.
ISSUE/S:
Whether the resolution issued by the board was ultra-vires.
RULING:
It should be noted that it was Acoje itself that requested for the setting
up of a post office for the convenience of its employees, which the SC held to
cover a subject which is a reasonable and proper adjunct to the conduct of
the business of Acoje Mining.
An ultra vires act is one committed outside the object for which a
corporation is created, but there are certain corporate acts that may be
performed outside the scope of the powers expressly conferred if they are
necessary to promote the interest and welfare of the corporation.Even in
the case of ultra vires acts which are not illegal per se, a corporation cannot
be heard to complain that it is not liable for the acts of its board, because of
estoppel by representation.

Page 525 of 1072

The term ultra vires should be distinguished from an illegal act for the
former is merely voidable which may be enforced by performance,
ratification, or estoppel, while the latter is void and cannot be invalidated. It
being merely voidable, an ultra vires act can be enforced or validated if there
are equitable grounds for taking such action.
In this case, it is fair that the resolution be upheld at least on the
ground of estoppel. The defense of ultra vires rests on violation of trust or
duty towards the stockholders, and should not be entertained where its
allowance will do greater wrong to innocent parties dealing with the
corporation. The acceptance of benefits arising from the performance of the
other party gives rise to an estoppel precluding the repudiation of the
contract.

TOPIC: ULTRA VIRES ACTS

Page 526 of 1072

REPUBLIC OF THE PHILIPPINES


vs.
SECURITY CREDIT AND ACCEPTANCE CORPORATION, ROSENDO T.
RESUELLO, PABLO TANJUTCO, ARTURO SORIANO, RUBEN BELTRAN,
BIENVENIDO V. ZAPA, PILAR G. RESUELLO, RICARDO D. BALATBAT,
JOSE SEBASTIAN and VITO TANJUTCO JR.
G.R. No. L-20583
January 23, 1967
19 SCRA 58
FACTS:
This is an original quo warranto proceeding, initiated by the Solicitor
General, to dissolve the Security and Acceptance Corporation for allegedly
engaging in banking operations without the authority required therefore by
the General Banking Act (Republic Act No. 337).
On May 18, 1962, the Municipal Court of Manila issued Search Warrant
No. A-1019 and that, pursuant thereto, members of the intelligence division
of the Central Bank and of the Manila Police Department searched the
premises of the corporation and seized documents and records thereof
relative to its business operations. Upon the return of said warrant, the
seized documents and records were placed under the custody of the Central
Bank of the Philippines. Upon examination and evaluation of said documents
and records, the intelligence division of the Central Bank submitted, to the
Acting Deputy Governor thereof, a memorandum dated September 10, 1962,
finding that the corporation is soliciting and accepting savings deposits from
the general public when the company's articles of incorporation authorize it
only to engage primarily in financing agricultural, commercial and industrial
projects, and secondarily, in buying and selling stocks and bonds of any
corporation, thereby exceeding the scope of its powers and authority as
granted under its charter; consequently such acts are ultra-vires.
Accordingly, on December 6, 1962, the Solicitor General commenced
this quo warranto proceedings for the dissolution of the corporation, with a
prayer that, meanwhile, a writ of preliminary injunction be issued ex parte,
enjoining the corporation and its branches, as well as its officers and agents,
from performing the banking operations complained of, and that a receiver
be appointed pendente lite.
In their answer, defendants admitted practically all of the allegations of
fact made in the petition.
ISSUE/S:
Whether the ultra-vires acts performed by the defendants
warrant their dissolution.

Page 527 of 1072

RULING:
Yes.
Although, admittedly, defendant corporation has not secured the
requisite authority to engage in banking, defendants deny that its
transactions partake of the nature of banking operations. It is conceded,
however, that, in consequence of a propaganda campaign therefore, a total
of 59,463 savings account deposits have been made by the public with the
corporation and its 74 branches, with an aggregate deposit of P1,689,136.74,
which has been lent out to such persons as the corporation deemed suitable
therefore. It is clear that these transactions partake of the nature of banking,
as the term is used in Section 2 of the General Banking Act. Indeed, a bank
has been defined as:
... a moneyed institute [Talmage vs. Pell 7 N.Y. (3 Seld. ) 328, 347, 348]
founded to facilitate the borrowing, lending and safe-keeping of money
(Smith vs. Kansas City Title & Trust Co., 41 S. Ct. 243, 255 U.S. 180,
210, 65 L. Ed. 577) and to deal, in notes, bills of exchange, and credits
(State vs. Cornings Sav. Bank, 115 N.W. 937, 139 Iowa 338). (Banks &
Banking, by Zellmann Vol. 1, p. 46).
Moreover, it has been held that:
An investment company which loans out the money of its
customers, collects the interest and charges a commission
to both lender and borrower, is a bank. (Western
Investment Banking Co. vs. Murray, 56 P. 728, 730, 731; 6
Ariz 215.)
... any person engaged in the business carried on by
banks of deposit, of discount, or of circulation is doing a
banking business, although but one of these functions is
exercised. (MacLaren vs. State, 124 N.W. 667, 141 Wis.
577, 135 Am. S.R. 55, 18 Ann. Cas. 826; 9 C.J.S. 30.)
Accordingly, defendant corporation has violated the law by engaging in
banking without securing the administrative authority required in Republic
Act No. 337.
That the illegal transactions thus undertaken by defendant corporation
warrant its dissolution is apparent from the fact that the foregoing misuser of
the corporate funds and franchise affects the essence of its business, that it
is willful and has been repeated 59,463 times, and that its continuance
inflicts injury upon the public, owing to the number of persons affected
thereby. Wherefore, the writ prayed for should be, as it is hereby granted
and Defendant Corporation is, accordingly, ordered dissolved.
TOPIC: BY LAWS : FUNCTIONS

Page 528 of 1072

DILY DANY NACPIL, petitioner,


vs.
INTERNATIONAL BROADCASTING CORPORATION, respondent.
G.R. No. 144767. March 21, 2002
370 SCRA 653
FACTS:
Petitioner was Assistant General Manager for Finance/Administration
and Comptroller of IBC. When Emiliano Templo assumed the IBC presidency,
he allegedly harassed, insulted, humiliated and pressured petitioner into
resigning until the latter was forced to retire. Hence, petitioner filed with the
Labor Arbiter a complaint for illegal dismissal and non-payment of benefits.
IBC filed a motion to dismiss contending that petitioner was a
corporate officer who was duly elected by the Board of Directors of IBC;
hence, the case qualifies as an intra-corporate dispute falling within the
jurisdiction of the SEC. On the other hand, petitioner argues that he is not a
corporate officer of IBC but an employee thereof since he had not been
elected nor appointed as Comptroller and Assistant Manager by the IBC's
Board of Directors but by an IBC General Manager. This is also because the
IBC's By-Laws do not even include the position of comptroller in its roster of
corporate officers.He therefore contends that his dismissal is a controversy
falling within the jurisdiction of the labor courts.
ISSUE/S:
Whether or not petitioner is a corporate officer although the position
of comptroller is not expressly mentioned in the by-laws.
RULING:
YES.
The fact that the position of Comptroller is not expressly mentioned
among the officers of the IBC in the By-Laws is of no moment, because the
IBC's Board of Directors is empowered under Section 25 of the Corporation
Code and under the corporation's By-Laws to appoint such other officers as it
may deem necessary.
The By-Laws of the IBC categorically provides:
XII. OFFICERS: The officers of the corporation shall
consist of a President, a Vice-President, a SecretaryTreasurer, a General Manager, and such other officers as
the Board of Directorsmay from time to time does fit to

Page 529 of 1072

provide for. Said officers shall be elected by majority vote


of the Board of Directors and shall have such powers and
duties as shall hereinafter provide.
The by-laws may and usually do provide for such other officers," and
that where a corporate office is not specifically indicated in the roster of
corporate offices in the by-laws of a corporation, the board of directors may
also be empowered under the by-lawsto create additional officers as may be
necessary. Furthermore, as petitioner's appointment as comptroller required
the approval and formal action of the IBC's Board of Directors to become
valid, it is clear therefore holds that petitioner is a corporate officer whose
dismissal may be the subject of a controversy cognizable by the SEC under
Section 5(c) of P.D. 902-A which includes controversies involving both
election and appointmentof corporate directors, trustees, officers, and
managers. Had petitioner been an ordinary employee, such board action
would not have been required.
Additional Notes:
Who are officers of the corporation?
1. Those statutory corporate officers provided in the
Corporation Code such as the president, treasurer
and secretary.
2. Those who have been named in the by-laws of the
corporation.
3. Those who may be appointed by the Board of
Directors for as long as the Board of Directors has
the authority to do so in the by-laws of the
corporation.

TOPIC: BY LAWS : FUNCTIONS

Page 530 of 1072

PMI COLLEGES, petitioner,


vs.
THE NATIONAL LABOR RELATIONS COMMISSION and ALEJANDRO GA
LVA N, respondents.
G.R. No. 121466. August 15, 1997
277 SCRA 462
FACTS:
On July 7, 1991, PMI Colleges, an educational institution offering
courses on basic seamans training and other related courses, hired
Alejandro Galvan as contractual instructor. Initially, Galvan and other
instructors were compensated for services rendered during the first 3 periods
of the abovementioned contract. However, for reasons unknown to him, he
stopped receiving payment for the succeeding rendition of services. A letter
of demand for payment of such services was sent by Acting Director
Aguinaldo to PMI Colleges President, Atty. Pastor. This request of the acting
director apparently went unheeded. Repeated demands having likewise
failed, Galvan was constrained to file a complaint with the NCR-Arbitration
Branch.
Later in the proceedings, PMI Colleges manifested that Mr. Tomas Cloma Jr., a
member of the board of trustees write a letter to the Chairman of the Board,
clarifying the case of Galvan and stating therein, inter alia, that under PMIs
by-laws only the Chairman is authorized to sign any contract and that
Galvan, in any event, failed to submit documents on the alleged shipyard
and plant visits in Cavite Naval Base.
ISSUE/S:
Is the contract of employment of Galvan valid even if the signatory
therein was not the Chairman of the Board?
RULING:
Yes.
The contract of employment is valid. The contract remained valid even
if the signatory thereon was not the chairman of the board which allegedly
violated petitioners by-laws. Since by-laws operate merely as internal rules
among the stockholders, they can not affect or prejudice third persons who
deal with the corporation, unless they have knowledge of the same. No proof
appears on record that private respondent ever knew anything about the
provisions of the said by-laws. In fact, petitioner itself merely asserts the
same without even bothering to attach a copy or excerpt thereof to show
that there is such provision. How can it now expect the Labor Arbiter and the
NLRC to believe it? That this allegation has never been denied to private
respondent does not necessarily signify admission of its existence because

Page 531 of 1072

technicalities of law and procedure and the rules obtaining in the courts of
law do not strictly apply to proceeding of this nature.

TOPIC: BY LAWS : FUNCTIONS

Page 532 of 1072

LOYOLA GRAND VILLAS HOMEOWNERS (SOUTH) ASSOCIATION, INC.


vs.
CA HOME INSURANCE AND GUARANTY CORPORATION, EMDEN
ENCARNACION and HORATIO AYCARDO,
G.R. No. 117188 August 7, 1997
276 SCRA 681
FACTS:
Loyola Grand Villas Homeowners Association (LGVHA) was organized
on February 8, 1983 as the association of homeowners and residents of the
Loyola Grand Villas. It was registered with the Home Financing Corporation,
the predecessor of herein respondent Home Insurance and Guaranty
Corporation (HIGC), as the sole homeowners' organization in the said
subdivision. It was organized by the developer of the subdivision and its first
president was Victorio V. Soliven, himself the owner of the developer. For
unknown reasons, however, LGVHAI did not file its corporate by-laws.
Sometime in 1988, the officers of the LGVHAI tried to register its bylaws but failed to do so. Then the officers that there were two other
organizations within the subdivision the Loyola Grand Villas homeowners
North Association Incorporated (North Association) and the Loyola Grand
Villas homeowners South Association Incorporated (South Association).
According to private respondents, a non-resident and Soliven himself,
respectively headed these associations. They also discovered that these
associations had five (5) registered homeowners each who were also the
incorporators, directors and officers thereof. None of the members of the
LGVHAI was listed as member of the North Association while three (3)
members of LGVHAI were listed as members of the South Association. When
Soliven inquired about the status of LGVHAI, Atty. Joaquin A. Bautista, the
head of the legal department of the HIGC, informed him that LGVHAI had
been automatically dissolved because it did not submit its by-laws within the
period required by the Corporation Code and there was non-user of corporate
charter because HIGC had not received any report on the association's
activities. Apparently, this information resulted in the registration of the
North and South Association.
ISSUE/S:
Whether or not failure of LGVHAI to file its by-laws within one month
from the date of its incorporation, as mandated by Section 46 of the
Corporation Code, result in its automatic dissolution.
RULING:
NO.

Page 533 of 1072

The SC ruled that the non-filing of the bylaws within the period of 1
month from the issuance by SEC of the Certificate of Incorporation will not
result to the automatic dissolution of the corporation because the word
MUST in Sec 46 of the Corporation Code is merely directory not mandatory
in meaning. In fact the second paragraph allows the filing of bylaws even
prior to incorporation. This provision in the same section of the Code rules
out mandatory compliance with the requirement of filing the by-laws "within
one (1) month after receipt of official notice of the issuance of its certificate
of incorporation by the Securities and Exchange Commission." It necessarily
follows that failure to file the by-laws within that period does not imply the
"demise" of the corporation. By-laws may be necessary for the "government"
of the corporation but these are subordinate to the articles of incorporation
as well as to the Corporation Code and related statutes.
TOPIC: BY LAWS : FUNCTIONS
CITIBANK, N.A., petitioner,
vs.
HON. SEGUNDINO G. CHUA, SANTIAGO M. KAPUNAN and LUIS L.
VICTOR, ASSOCIATE JUSTICES OF THE HON. COURT OF APPEALS,
THIRD DIVISION, MANILA, HON. LEONARDO B. CANARES, Judge of
Regional, Trial Court of Cebu, Branch 10, and SPOUSES CRESENCIO
AND ZENAIDA VELEZ, respondents.
G.R. No. 102300. March 17, 1993
220 SCRA 75
FACTS:
Citibank is a foreign commercial banking corporation duly licensed to
do business in the Philippines. Private respondents, spouses Cresencio and
Zenaida Velez, who were good clients alleged that the petitioner bank
extended to them credit lines sufficiently secured with real estate and chattel
mortgages on equipment. They claim that a restructuring agreement has
been entered into between them and the bank. However, the bank failed to
comply thereto thus spouses Velez sued for specific performance and
damages.
On March 30, 1990, the date of the pre-trial conference, counsel for
petitioner bank appeared, presenting a special power of attorney executed
by Citibank officer Florencia Tarriela in favor of petitioner bank's counsel, the
J.P. Garcia & Associates, to represent and bind petitioner bank at the pre-trial
conference of the case at bar. Inspite of this special power of attorney,
counsel for spouses Velez orally moved to declare petitioner bank as in
default on the ground that the special power of attorney was not executed by
the Board of Directors of Citibank. Thus petitioner bank executed another
special power of attorney made by William W. Ferguson, Vice President and
highest ranking officer of Citibank, Philippines, constituting and appointing

Page 534 of 1072

the J.P. Garcia & Associates to represent and bind the BANK. Unsatisfied,
private respondents moved again for declaration of default. Though the bank
again executed another special power of attorney through William W.
Ferguson in favor of Citibank employees, the court issued an order declaring
petitioner bank as in default.
The CA dismissed the petition filed by the bank. The CA relied on
Section 46 of the Corporation Code to support its conclusion that the by-laws
in question are without effect because they were not approved by the SEC.
ISSUE/S:
Whether petitioner bank's by-laws, which constitute the basis for
Ferguson's special power of attorney in favor of petitioner bank's
legal counsel are effective, considering that petitioner bank has
been previously granted a license to do business in the Philippines.
RULING:
Yes.
Section 46 (which was relied upon by the CA) starts with the phrase
"Every corporation formed under this Code", which can only refer to
corporations incorporated in the Philippines. Hence, Section 46, in so far as it
refers to the effectivity of corporate by-laws, applies only to domestic
corporations and not to foreign corporations.
On the other hand, Section 125 of the same Code requires that a
foreign corporation applying for a license to transact business in the
Philippines must submit, among other documents, to the SEC, a copy of its
articles of incorporation and by-laws, certified in accordance with law. Unless
these documents are submitted, the application cannot be acted upon by the
SEC. In the following section, the Code specifies when the SEC can grant the
license applied for. Section 126 provides in part:"SEC. 126. Issuance of a
license. - If the Securities and Exchange Commission is satisfied that the
applicant has complied with all the requirements of this Code and other
special laws, rules and regulations, the Commission shall issue a license to
the applicant to transact business in the Philippines for the purpose or
purposes specified in such license . . ."
Since the SEC will grant a license only when the foreign corporation
has complied with all the requirements of law, it follows that when it decides
to issue such license, it is satisfied that the applicant's by-laws, among the
other documents, meet the legal requirements. This, in effect, is an approval
of the foreign corporations by-laws. It may not have been made in express
terms, still it is clearly an approval. Therefore, petitioner bank's by-laws,
though originating from a foreign jurisdiction, are valid and effective in the
Philippines.

Page 535 of 1072

TOPIC: BY LAWS : WHEN TO ADOPT AND FILE


LOYOLA GRAND VILLAS HOMEOWNERS (SOUTH) ASSOCIATION, INC.
vs.
CA HOME INSURANCE AND GUARANTY CORPORATION, EMDEN
ENCARNACION and HORATIO AYCARDO,
G.R. No. 117188 August 7, 1997
276 SCRA 681
FACTS:
Loyola Grand Villas Homeowners Association (LGVHA) was organized
on February 8, 1983 as the association of homeowners and residents of the
Loyola Grand Villas. It was registered with the Home Financing Corporation,
the predecessor of herein respondent Home Insurance and Guaranty
Corporation (HIGC), as the sole homeowners' organization in the said
subdivision. It was organized by the developer of the subdivision and its first
president was Victorio V. Soliven, himself the owner of the developer. For
unknown reasons, however, LGVHAI did not file its corporate by-laws.
Sometime in 1988, the officers of the LGVHAI tried to register its bylaws but failed to do so. Then the officers that there were two other
organizations within the subdivision the Loyola Grand Villas homeowners
North Association Incorporated (North Association) and the Loyola Grand
Villas homeowners South Association Incorporated (South Association).
According to private respondents, a non-resident and Soliven himself,
respectively headed these associations. They also discovered that these
associations had five (5) registered homeowners each who were also the
incorporators, directors and officers thereof. None of the members of the
LGVHAI was listed as member of the North Association while three (3)
members of LGVHAI were listed as members of the South Association. When
Soliven inquired about the status of LGVHAI, Atty. Joaquin A. Bautista, the

Page 536 of 1072

head of the legal department of the HIGC, informed him that LGVHAI had
been automatically dissolved because it did not submit its by-laws within the
period required by the Corporation Code and there was non-user of corporate
charter because HIGC had not received any report on the association's
activities. Apparently, this information resulted in the registration of the
North and South Association.
ISSUE/S:
Whether or not failure of LGVHAI to file its by-laws within one month
from the date of its incorporation, as mandated by Section 46 of the
Corporation Code, result in its automatic dissolution.
RULING:
NO.
The SC ruled that the non-filing of the bylaws within the period of 1
month from the issuance by SEC of the Certificate of Incorporation will not
result to the automatic dissolution of the corporation because the word
MUST in Sec 46 of the Corporation Code is merely directory not mandatory
in meaning. In fact the second paragraph allows the filing of bylaws even
prior to incorporation. This provision in the same section of the Code rules
out mandatory compliance with the requirement of filing the by-laws "within
one (1) month after receipt of official notice of the issuance of its certificate
of incorporation by the Securities and Exchange Commission." It necessarily
follows that failure to file the by-laws within that period does not imply the
"demise" of the corporation. By-laws may be necessary for the "government"
of the corporation but these are subordinate to the articles of incorporation
as well as to the Corporation Code and related statutes.

Page 537 of 1072

TOPIC:AUTHORITY TO ELECT ADDITIONAL BY-LAWS OFFICERS


HENRY FLEISCHER,
vs.
BOTICA NOLASCO CO., INC.,
G.R. No.L-23241.March 14, 1925
45 Phil 583
FACTS:
Manuel Gonzalez was the original owner of the five shares of stock in
question, Nos. 16, 17, 18, 19 and 20 of the Botica Nolasco, Inc.; on March
11, 1923, he assigned and delivered said five shares to the plaintiff, Henry
Fleischer, by accomplishing the form of endorsement provided on the back
thereof, together with other credits, in consideration of a large sum of money
owed by Gonzalez to Fleischer. On March 13, 1923, Dr. Eduardo Miciano, who
was the secretary-treasurer of said corporation, offered to buy from Henry
Fleischer, on behalf of the corporation, said shares of stock, at their par value
of P100 a share, for P500; that by virtue of article 12 of the by-laws of Botica
Nolasco, Inc., said corporation had the preferential right to buy from Manuel
Gonzalez said shares that the plaintiff refused to sell them to the defendant;
that the plaintiff requested Doctor Miciano to register said shares in his
name; that Doctor Miciano refused to do so, saying that it would be in
contravention of the by-laws of the corporation.
Two days after the assignment of the shares to the plaintiff, Manuel
Gonzales made a written statement to the Botica Nolasco, Inc., requesting
that the five shares of stock sold by him to Henry Fleischer be noted
transferred to Fleischer's name. He also acknowledged in said written
statement the preferential right of the corporation to buy said five shares. On
June 14, 1923, Gonzalez wrote a letter to the Botica Nolasco, withdrawing
and cancelling his written statement of March 13, 1923, to which letter the
Botica Nolasco on June 15, 1923, replied, declaring that his written
statement was in conformity with the by-laws of the corporation; that his

Page 538 of 1072

letter of June 14th was of no effect, and that the shares in question had been
registered in the name of the Botica Nolasco, Inc.
ISSUE/S:
Whether or not article 12 of the by-laws of the Botica Nolasco, Inc.,
is in conflict with the provisions of the Corporation Law (Act No.
1459)
RULING:
As a general rule, the by-laws of a corporation are valid if they are
reasonable and calculated to carry into effect the objects of the corporation,
and are not contradictory to the general policy of the laws of the land.
On the other hand, it is equally well settled that by-laws of a
corporation must be reasonable and for a corporate purpose, and always
within the charter limits. They must always be strictly subordinate to the
constitution and the general laws of the land. They must not infringe the
policy of the state, nor be hostile to public welfare. They must not disturb
vested rights or impair the obligation of a contract, take away or abridge the
substantial rights of stockholder or member, affect rights of property or
create obligations unknown to the law.
The foregoing authorities go farther than the stand we are taking on
this question. They hold that the power of a corporation to enact by-laws
restraining the sale and transfer of shares, should not only be in harmony
with the law or charter of the corporation, but such power should be
expressly granted in said law or charter.
A by-law which prohibits a transfer of stock without the consent or
approval of all the stockholders or of the president or board of directors is
illegal as constituting undue limitation on the right of ownership and in
restraint of trade.
And moreover, the by-laws now in question cannot have any effect on
the appellee. He had no knowledge of such by-law when the shares were
assigned to him. He obtained them in good faith and for a valuable
consideration. He was not a privy to the contract created by said by-law
between the shareholder Manuel Gonzalez and the Botica Nolasco, Inc. Said
by-law cannot operate to defeat his rights as a purchaser.

Page 539 of 1072

TOPIC: AUTHORITY TO ELECT ADDITIONAL BY-LAWS OFFICERS


JOHN GOKONGWEI, JR.,
vs
SECURITIES AND EXCHANGE COMMISSION, ANDRES M. SORIANO,
JOSE M. SORIANO, ENRIQUE ZOBEL, ANTONIO ROXAS, EMETERIO
BUNAO, WALTHRODE B. CONDE, MIGUEL ORTIGAS, ANTONIO PRIETO,
SAN MIGUEL CORPORATION, EMIGDIO TANJUATCO, SR., and
EDUARDO R. VISAYA,
G.R. No. L-45911. April 11, 1979
89 SCRA 336
FACTS:
Petitioner alleged that on September 18, 1976, individual respondents
amended by bylaws of the corporation, basing their authority to do so on a
resolution of the stockholders adopted on March 13, 1961, when the
outstanding
capital
stock
of
respondent
corporation
was
only
P70,139.740.00, divided into 5,513,974 common shares at P10.00 per share
and 150,000 preferred shares at P100.00 per share. At the time of the
amendment, the outstanding and paid up shares totalled 30,127,047 with a
total par value of P301,270,430.00. It was contended that according to
section 22 of the Corporation Law and Article VIII of the by-laws of the
corporation, the power to amend, modify, repeal or adopt new by-laws may
be delegated to the Board of Directors only by the affirmative vote of
stockholders representing not less than 2/3 of the subscribed and paid up
capital stock of the corporation, which 2/3 should have been computed on
the basis of the capitalization at the time of the amendment. Since the
amendment was based on the 1961 authorization, petitioner contended that
the Board acted without authority and in usurpation of the power of the
stockholders.

Page 540 of 1072

ISSUE/S:
Whether or not the amended by-laws of SMC of disqualifying a
competitor from nomination or election to the Board of Directors of
SMC are valid and reasonable .
RULING:
Whether the by-law is in conflict with the law of the land, or with the
charter of the corporation, or is in a legal sense unreasonable and therefore
unlawful is a question of law. This rule is subject, however, to the limitation
that where the reasonableness of a by-law is a mere matter of judgment, and
one upon which reasonable minds must necessarily differ, a court would not
be warranted in substituting its judgment instead of the judgment of those
who are authorized to make by-laws and who have exercised their authority.
It is recognized by an authorities that 'every corporation has the
inherent power to adopt by-laws 'for its internal government, and to regulate
the conduct and prescribe the rights and duties of its members towards itself
and among themselves in reference to the management of its affairs. At
common law, the rule was "that the power to make and adopt by-laws was
inherent in every corporation as one of its necessary and inseparable legal
incidents. And it is settled throughout the United States that in the absence
of positive legislative provisions limiting it, every private corporation has this
inherent power as one of its necessary and inseparable legal incidents,
independent of any specific enabling provision in its charter or in general
law, such power of self-government being essential to enable the corporation
to accomplish the purposes of its creation.
In this jurisdiction, under section 21 of the Corporation Law, a
corporation may prescribe in its by-laws "the qualifications, duties and
compensation of directors, officers and employees ... " This must necessarily
refer to a qualification in addition to that specified by section 30 of the
Corporation Law, which provides that "every director must own in his right at
least one share of the capital stock of the stock corporation of which he is a
director.

Page 541 of 1072

TOPIC: AUTHORITY TO ELECT ADDITIONAL BY-LAWS OFFICERS


THE GOVERNMENT OF THE PHILIPPINE ISLANDS
(on relation of the Attorney-General)
vs.
EL HOGAR FILIPINO
G.R. No. L-26649.July 13, 1927
50 Phil 399
FACTS:
On March 1, 1906, the Philippine Commission enacted what is known
as the Corporation Law (Act No. 1459) effective upon April 1 of the same
year. Section 171 to 190, inclusive, of this Act are devoted to the subject of
building and loan associations, defining their objects making various
provisions governing their organization and administration, and providing for
the supervision to be exercised over them. These provisions appear to be
adopted from American statutes governing building and loan associations
and they of course reflect the ideals and principles found in American law
relative to such associations.
The respondent, El Hogar Filipino, was apparently the first corporation
organized in the Philippine Islands under the provisions cited, and the
association has been favored with extraordinary success. The articles of
incorporation bear the date of December 28, 1910, at which time capital
stock in the association had been subscribed to the amount of P150,000 of
which the sum of P10,620 had been paid in.
Under the law as it then stood, the capital of the Association was not
permitted to exceed P3,000,000, but by Act No. 2092, passed December 23,
1911, the statute was so amended as to permit the capitalization of building

Page 542 of 1072

and loan associations to the amount of ten millions. Soon thereafter the
association took advantage of this enactment by amending its articles so as
to provide that the capital should be in an amount not exceeding the then
lawful limit.
From the time of its first organization the number of shareholders has
constantly increased, with the result that on December 31, 1925, the
association had 5,826 shareholders holding 125,750 shares, with a total
paid-up value of P8,703,602.25. During the period of its existence prior to
the date last above-mentioned the association paid to withdrawing
stockholders the amount of P7,618,257,.72; and in the same period it
distributed in the form of dividends among its stockholders the sum of
P7,621,565.81.
ISSUE/S:
Whether or not the by-laws is valid
RULING:
This by-law is of course a patent nullity, since it is in direct conflict with
the latter part of section 187 of the Corporation Law, which expressly
declares that the board of directors shall not have the power to force the
surrender and withdrawal of unmatured stock except in case of liquidation of
the corporation or of forfeiture of the stock for delinquency. It is agreed that
this provision of the by-laws has never been enforced, and in fact no attempt
has ever been made by the board of directors to make use of the power
therein conferred. In November, 1923, the Acting Insular Treasurer addressed
a letter to El Hogar Filipino, calling attention to article 10 of its by-laws and
expressing the view that said article was invalid. It was therefore suggested
that the article in question should be eliminated from the by-laws. At the
next meeting of the board of directors the matter was called to their
attention and it was resolved to recommend to the shareholders that in their
next annual meeting the article in question be abrogated. It appears,
however, that no annual meeting of the shareholders called since that date
has been attended by a sufficient number of shareholders to constitute a
quorum, with the result that the provision referred to has not been
eliminated from the by-laws, and it still stands among the by-laws of the
association, notwithstanding its patent conflict with the law.
It is supposed, in the fourth cause of action, that the existence of this
article among the by-laws of the association is a misdemeanor on the part of
the respondent which justifies its dissolution. In this view we are unable to
concur. The obnoxious by-law, as it stands, is a mere nullity, and could not be
enforced even if the directors were to attempt to do so. There is no provision
of law making it a misdemeanor to incorporate an invalid provision in the by-

Page 543 of 1072

laws of a corporation; and if there were such, the hazards incident to


corporate effort would certainly be largely increased.

TOPIC: AMENDMENT AND/OR REJECTION OF BY LAWS


ENRIQUE SALAFRANCA, petitioner,
vs.
PHILAMLIFE (PAMPLONA) VILLAGE HOMEOWNERS ASSOCIATION,
INC., BONIFACIO DAZO and THE SECOND DIVISION, NATIONAL LABOR
RELATIONS COMMISSION (NLRC), respondents.
G.R. No. 121791 December 23, 1998
300 SCRA 469
FACTS:
Petitioner Enrique Salafranca started working with the private
respondent Philamlife Village Homeowners Association on May 1, 1981 as
administrative officer for a period of six months. From this date until
December 31, 1983, petitioner was reappointed to his position three more
times. As administrative officer, petitioner was generally responsible for the
management of the village's day to day activities. After petitioner's term of
employment expired on December 31, 1983, he still continued to work in the
same capacity, albeit, without the benefit of a renewed contract. Sometime
in 1987, private respondent decided to amend its by-laws. Included therein
was a provision regarding officers, specifically, the position of administrative
officer under which said officer shall hold office at the pleasure of the Board
of Directors. Private respondent informed the petitioner that his term of
office shall be coterminous with the Board of Directors which appointed him
to his position. Furthermore, until he submits a medical certificate showing
his state of health, his employment shall be on a month-to-month basis.
Oddly, notwithstanding the failure of herein petitioner to submit his medical

Page 544 of 1072

certificate, he continued working until his termination in December 1992.


Claiming that his services had been unlawfully and unceremoniously
dispensed with, petitioner filed a complaint for illegal dismissal with money
claims and for damages.
After the submission by the parties of their respective position papers
and other pleadings, the Labor Arbiter rendered a decisionordering private
respondent to pay the petitioner. The Labor Arbiter elucidated:
Respondents' contention that complainant's term of employment was coterminus with the term of Office of the Board of Directors, is wanting in merit.
Records show that complainant had been hired in 1981 while the
Amendment of the respondents' By-Laws making the position of an
Administrative Officer co-terminus with the term of the Board of Directors
was made in 1987. Evidently, the said Amendment would not be applicable
to the case of complainant who had become a regular employee long time
before the Amendment took place. Moreover, the Amendment should be
applied prospectively and not retroactively.
The NLRC reversed the decision of the Labor Arbiter and rendered a
new onereducing petitioner's monetary award to only one-half (1/2) month
pay for every year of service representing his retirement pay. The NLRC
viewed the dismissal of the petitioner as a valid act by the private
respondent. The fact that he continued to perform the function of the office
of administrative officer without extension or re-appointment thereafter, to
our mind, did not in any way make his employment permanent as in fact, he
was even reminded of the nature of his position by then president of the
association Jaime Y. Ladao in a letter of 3 July 1987. His reply to the aforesaid
letter, claiming his employment regular, and viz a viz, referring to submit his
medical certificate, notwithstanding, to our mind, merely underscored the
need to define his position as, in fact, the Association's Rules and
Regulations were amended if but to put to rest the tenural limit of the office
of the Administrative Officer in accordance with its earlier intention, that it is
co-terminus with that of the members of the Board of Directors.

ISSUE/S:
Whether or not Salafrancas position is co-terminus with that of the
Village's Board of Directors, as provided for in its amended by-laws.
RULING:
Admittedly, the right to amend the by-laws lies solely in the discretion
of the employer, this being in the exercise of management prerogative or
business judgment. However this right, extensive as it may be, cannot impair
the obligation of existing contracts or rights.

Page 545 of 1072

Prescinding from these premises, private respondent's insistence that


it can legally dismiss petitioner on the ground that his tenure has expired is
untenable. To reiterate, petitioner, being a regular employee, is entitled to
security of tenure, hence, his services may only be terminated for
causesprovided by law. A contrary interpretation would not find justification
in the laws or the Constitution. If the Court were to rule otherwise, it would
enable an employer to remove any employee from his employment by the
simple expediency of amending its by-laws and providing that his/her
position shall cease to exist upon the occurrence of a specified event.
If private respondent wanted to make the petitioner's position coterminus with that of the Board of Directors, then the amendment must be
effective after petitioner's stay with the private respondent, not during his
term. Obviously, the measure taken by the private respondent in amending
its by-laws is nothing but a devious, but crude, attempt to circumvent
petitioner's right to security of tenure as a regular employee guaranteed
under the Labor Code.

TOPIC:MEETINGS OF STOCKHOLDERS AND BOARD OF DIRECTORS


ROSITA PEA
vs.
THE COURT OF APPEALS, SPOUSES RISING T. YAP and CATALINA YAP,
PAMPANGA BUS CO., INC., JESUS DOMINGO, JOAQUIN BRIONES,
SALVADOR BERNARDEZ, MARCELINO ENRIQUEZ and EDGARDO A.
ZABAT
G.R. No. 91478. February 7, 1991
193 S 717
FACTS:
A reading of the records shows that Pampanga Bus Co., original owners
of the lots in question under TCT Nos. 4314, 4315 and 4316, mortgaged the
same to the Development Bank of the Philippines (DBP) on January 3, 1962
in consideration of the amount of P935,000.00. This mortgage was
foreclosed. In the foreclosure sale under Act No. 3135 held on October 25,
1974, the said properties were awarded to Rosita Pea as highest bidder. A
certificate of sale was issued in her favor by the Senior Deputy Sheriff of
Pampanga, Edgardo A. Zabat, upon payment of the sum of P128, 000.00 to
the Office of the Provincial Sheriff . The certificate of sale was registered on
October 29, 1974.
On November 19, 1974, the board of directors of PAMBUSCO, through
three out of its five directors, resolved to assign its right of redemption over
the aforesaid lots and authorized one of its members, Atty. Joaquin Briones

Page 546 of 1072

"to execute and sign a Deed of Assignment for and in behalf of PAMBUSCO in
favor of any interested party . .Consequently, on March 18, 1975, Briones
executed a Deed of Assignment of PAMBUSCO's redemption right over the
subject lots in favor of Marcelino Enriquez. The latter then redeemed the said
properties and a certificate of redemption dated August 15, 1975 was issued
in his favor by Sheriff Zabat upon payment of the sum of one hundred forty
thousand, four hundred seventy four pesos P140,474.00) to the Office of the
Provincial Sheriff of Pampanga.
A day after the aforesaid certificate was issued, Enriquez executed a
deed of absolute sale of the subject properties in favor of plaintiffsappellants, the spouses Rising T. Yap and Catalina Lugue, for the sum of
P140,000.00.
On August 18, 1975, a levy on attachment in favor of Capitol Allied
Trading was entered as an additional encumbrance on TCT Nos. 4314, 4315
and 4316 and a Notice of a pending consulta was also annotated on the
same titles concerning the Allied Trading case.
ISSUE/S:
Whether or not the transaction was valid
RULING:
In this case, neither petitioner nor respondents Yap spouses are
stockholders or officers of PAMBUSCO. Consequently, the issue of the validity
of the series of transactions resulting in the subject properties being
registered in the names of respondents Yap may be resolved only by the
regular courts.
There can be no question in this case that the questioned resolution
and series of transactions resulting in the registration of the properties in the
name of respondent Yap spouses adversely affected the rights of petitioner
to the said properties. Consequently, petitioner has the legal standing to
question the validity of said resolution and transactions.
In the instant case, however, there was no proof whatsoever, either by
way of documentary or testimonial evidence, that there was such a failure or
irregularity of notice as to make the aforecited provision apply. There was not
even such an allegation in the Answer that should have necessitated a proof
thereof. The fact alone that only three (3) out of five (5) members of the
board of directors attended the subject special meeting, was not enough to
declare the aforesaid proceeding void ab initio, much less the board
resolution borne out of it, when there was no proof of irregularity nor failure
of notice and when the defense made in the Answer did not touch upon the
said failure of attendance. Therefore, the judgment declaring the nullity of
the subject board resolution must be set aside for lack of proof.

Page 547 of 1072

The by-laws of a corporation are its own private laws which


substantially have the same effect as the laws of the corporation. They are in
effect, written, into the charter. In this sense they become part of the
fundamental law of the corporation with which the corporation and its
directors and officers must comply.
Apparently, only three (3) out of five (5) members of the board of
directors of respondent PAMBUSCO convened on November 19, 1974 by
virtue of a prior notice of a special meeting. There was no quorum to validly
transact business since, under Section 4 of the amended by-laws
hereinabove reproduced, at least four (4) members must be present to
constitute a quorum in a special meeting of the board of directors of
respondent PAMBUSCO.

TOPIC:MEETINGS OF STOCKHOLDERS AND BOARD OF DIRECTORS


THE BOARD OF LIQUIDATORS representing THE GOVERNMENT OF
THE REPUBLIC OF THE PHILIPPINES
vs.
HEIRS OF MAXIMO M. KALAW,2 JUAN BOCAR, ESTATE OF THE
DECEASED CASIMIRO GARCIA,3 and LEONOR MOLL
G.R. No. L-18805. August 14, 1967
105 Phil 426
FACTS:
The National Coconut Corporation was chartered as a non-profit
governmental organization on May 7, 1940 by Commonwealth Act 518
avowedly for the protection, preservation and development of the coconut
industry in the Philippines. On August 1, 1946, NACOCOs charter was
amended to grant that corporation the express power to buy, sell, barter,
export, and in any other manner deal in, coconut, copra, and dessicated
coconut, as well as their by-products, and to act as agent, broker or
commission merchant of the producers, dealers or merchants thereof. The
charter amendment was enacted to stabilize copra prices, to serve coconut
producers by securing advantageous prices for them, to cut down to a

Page 548 of 1072

minimum, if not altogether eliminate, the margin of middlemen, mostly


aliens.
General manager and board chairman was Maximo M. Kalaw;
defendants Juan Bocar and Casimiro Garcia were members of the Board;
defendant Leonor Moll became director only on December 22, 1947.
NACOCO, after the passage of Republic Act 5, embarked on copra
trading activities. Amongst the scores of contracts executed by general
manager Kalaw are the disputed contracts.
An unhappy chain of events conspired to deter NACOCO from fulfilling
these contracts. Nature supervened. Four devastating typhoons visited the
Philippines: the first in October, the second and third in November, and the
fourth in December, 1947. Coconut trees throughout the country suffered
extensive damage. Copra production decreased. Prices ]piraled. Warehouses
were destroyed. Cash requirements doubled. Deprivation of export facilities
increased the time necessary to accumulate shiploads of copra. Quick
turnovers became impossible, financing a problem.
When it became clear that the contracts would be unprofitable, Kalaw
submitted them to the board for approval. It was not until December 22,
1947 when the membership was completed. Defendant Moll took her oath on
that date. A meeting was then held. Kalaw made a full disclosure of the
situation, apprised the board of the impending heavy losses. No action was
taken on the contracts. Neither did the board vote thereon at the meeting of
January 7, 1948 following.
Then, on January 11, 1948, President Roxas made a statement that the
NACOCO head did his best to avert the losses, emphasized that government
concerns faced the same risks that confronted private companies, that
NACOCO was recouping its losses, and that Kalaw was to remain in his post.
Not long thereafter, that is, on January 30, 1948, the board met again with
Kalaw, Bocar, Garcia and Moll in attendance. They unanimously approved the
contracts hereinbefore enumerated.
ISSUE/S:
Whether or not the action is unenforceable against the heirs of
Kalaw
RULING:
Settled jurisprudence has it that where similar acts have been
approved by the directors as a matter of general practice, custom, and
policy, the general manager may bind the company without formal
authorization of the board of directors. In varying language, existence of

Page 549 of 1072

such authority is established, by proof of the course of business, the usage


and practices of the company and by the knowledge which the board of
directors has, or must be presumed to have, of acts and doings of its
subordinates in and about the affairs of the corporation. So also,
x x x authority to act for and bind a corporation may
be presumed from acts of recognition in other instances
where the power was in fact exercised.
x x x Thus, when, in the usual course of business of a
corporation, an officer has been allowed in his official
capacity to manage its affairs, his authority to represent
the corporation may be implied from the manner in which
he has been permitted by the directors to manage its
business.
In the case at bar, the practice of the corporation has been to allow its
general manager to negotiate and execute contracts in its copra trading
activities for and in NACOCOs behalf without prior board approval. If the bylaws were to be literally followed, the board should give its stamp of prior
approval on all corporate contracts. But that board itself, by its acts and
through acquiescence, practically laid aside the by-law requirement of prior
approval.
Under the given circumstances, the Kalaw contracts are valid corporate acts.

TOPIC: QUORUM REQUIRED


ABELARDO JAVELLANA, TOMAS JONCO, RUDICO HABANA, EXEQUIEL
GOLEZ, ALFREDO ANG, and FILIPINAS SOLEDAD, in their capacities
as Councilors of the Municipal Municipality of Buenavista, Province
of Iloilo, petitioners appellees,
vs.
SUSANO TAYO, as Mayor of the Municipal Municipality of Buenavista,
Iloilo, respondent-appellant.
G.R. No. L-18919. December 29, 1962
FACTS:
Petitioners were members of the municipal council. On several
sessions, the mayor, herein defendant, was absent prompting the council to
decide among themselves as to who to appoint as presiding officers. The
mayor refused to act on the resulting minutes also refused to sign the

Page 550 of 1072

payrolls of the council covering the per diems of the petitioners, alleging that
the proceedings were illegal due to his absence.
Despite the Provincial Fiscal and the Provincial Board upholding the
controverted sessions of the Municipal Council, the Mayor refused and still
refuses to recognize the validity of the acts of the Municipal Council and the
legality of its regular session held in his absence.
The trial court ruled that attendance of the Mayor is not essential to the
validity of the session as long as there is quorum constituted in accordance
with law. To declare that the proceedings of the petitioners were null and
void, is to encourage recalcitrant public officials who would frustrate valid
sessions for political end or consideration.
ISSUE/S:
Whether or not the sessions held by petitioners were valid and
legal, having constituted a quorum, and despite the absence of the
defendant.
RULING:
The term "quorum" has been defined as "that number of members of
the body which, when legally assembled in their proper places, will enable
the body to transact its proper business, or, in other words, that number that
makes a lawful body and gives it power to pass a law or ordinance or do any
other valid corporate act.
The Revised Administrative Code states that for the majority of the
members of the council to constitute a quorum to do business, the council
"shall be presided by the Mayor and no one else.
The procedure, as provided in the Administrative Code, provides that in
case of temporary incapacity of the mayor, the council member having the
highest number of votes can sit as presiding officer. This rule on incapacity
was declared as valid by the court in the case. Thus, the quorum
requirement was satisfied despite the continuous absence of the mayor on
those scheduled sessions.
Thus, the questioned sessions and the resulting resolutions were
declared valid.

Page 551 of 1072

TOPIC: WHO COULD ATTEND AND VOTE


JULIO E. T. SALES and GEORGE V. AGONIAS, in their own behalf, and
in behalf of SIPALAY MINING EXPLORATION CORPORATION, as
minority stockholders thereof, and SIPALAY MINING EXPLORATION
CORPORATION, petitioners,
vs.
SECURITIES AND EXCHANGE COMMISSION, STATE INVESTMENT
HOUSE, INC., represented by its President, ANSELMO TRINIDAD;
ANSELMO TRINIDAD CO., INC., represented by its President,
ANSELMO TRINIDAD; and VULCAN INDUSTRIAL AND MINING CORP.,
represented by its President, WALTER W. BROWN; AFREDO C.
RAMOS, ANNABELLE P. BROWN, WALTER W. BROWN, MANUEL C.
DIAZ, and AUGUSTO B. SUNICO, respondents.
G.R. No. L-54330 January 13, 1989

Page 552 of 1072

FACTS:
Respondent State Investment House, Inc. entered into a sales
agreement with Sipaly Mining whereby the latter sold to the former
200,000,000 common shares of its capital stock in the amount of P2.6
Million. Sipalay Investment addressed to Sipalay Mining requesting that the
latter transfer the said share to Anselmo Trinidad & Co. Inc. (ATCO). ATCO
voted them in the stockholders meeting of Sipalay Mining. ATCO in turn sold
198,500,000 shares to VULCAN.
Eight days prior to the scheduled annual stockholders meeting of
Sipalay Mining, petitioners filed before the SEC a petition to nullify the sale of
the shares to VULCAN, with the prayer for the issuance of a writ of
preliminary injunction to enjoin VULCAN from voting the shares.
The SEC temporary restrained VULCAN from voting the 198,500,000
share. The annual stockholders meeting of Sipalay Mining proceeded
without the participation of VULCANs 198,500,000 shares and the members
of Board of Directors were elected.
ISSUE/S:
Whether or not SEC acted with grave abuse of discretion in not
permanently enjoining VIMC in voting.
RULING:
The Supreme Court found no grave abuse of discretion on the part of
the SEC in not restraining VIMC.
It adopted the SEC resolution stating that the sale of the shares of
stock had long been perfected and is presumed valid until declared
otherwise. As against this presumption, petitioners' prayer for injunction
cannot prevail as the issue of the validity of the sale is still to be resolved by
the SEC. Further, the directive of the BOD of SMEC to its President to sign the
stock certificate that would evidence the ownership of the shares by VIMC
militates against a finding that petitioners have established a case for
injunction.
Furthermore, SC also held that it is not at liberty to review whether or
not the decision of the board to direct its President to sign the stock
certificate was to the best interest of the corporation as it is a well known
rule of law that questions of policy or of management are left solely to the

Page 553 of 1072

honest decision of officers and directors of a corporation, and the court is


without authority to substitute its judgment for the judgment of the BOD.
Moreover, considering that the shares constitute the majority, it is
more equitable that the same be allowed to vote rather than be enjoined. As
it has been ruled the removal of a majority SH from the management of the
corporation and/or the dissolution of a corporation in a suit filed by a
minority SH is a drastic measure. It should be resorted to only when the
necessity is clear. With more reason, the Court will not deprive a SH of his
right to vote his shares in the annual SHs' meeting, except upon a clear
showing of its lawful denial under the articles of incorporation or by-laws of
the corporation, as it is a right inherent in stock ownership.

TOPIC: WHO COULD ATTEND AND VOTE


DOMINGO PONCE AND BUHAY L. PONCE, petitioners,
vs.
DEMETRIO B. ENCARNACION, Judge of the Court of First Instance of
Manila, Branch I, and POTENCIANO GAPOL, respondents.
G.R. No. L-5883. November 28, 1953
94 Phil 81
FACTS:
Daguhoy Enterprises, Inc., was duly registered as such on 24 June
1948. On 16 April 1951 at a meeting duly called, the voluntary dissolution of
the corporation and the appointment of Gapol as receiver were agreed upon
and to that end a petition for voluntary dissolution was drafted which was
sent to, and signed by, the petitioner Domingo Ponce. Instead of filing the
petition for voluntary dissolution of the corporation as agreed upon, Gapol,
who is the largest stockholder, changed his mind and filed a complaint in the

Page 554 of 1072

CFI of Manila to compel the petitioners to render an accounting of the funds


and assets of the corporation, to reimburse it, jointly and severally, a total
sum of P18,690, plus interest, which have been converted by the petitioner
Domingo Ponce to his own use and benefit.
On 18 May 1951 Gapol filed a motion praying that the petitioners be
removed as members of the board of directors which was denied by the
court.
On 3 January 1952 Gapol filed a petition praying for an order directing
him to call a meeting of the stockholders of the corporation and to preside at
such meeting in accordance with section 26 of the Corporation Law.
Two-days later, without notice to the petitioners and to the other
members of the board of directors and in violation of the Rules of Court
which require that the adverse parties be notified of the hearing of the
motion three days in advance, the respondent court issued the order as
prayed for.
ISSUE/S:
Whether under and pursuant to section 26 of the Corporation Law,
the respondent court may issue the order complained of.
RULING:
A showing of good cause exists when the court is apprised of the fact
that the by-laws of the corporation require the calling of a general meeting of
the stockholders to elect the board of directors but the call for such meeting
has not been done. The requirement that "on the showing of good cause
therefore," the court may grant to a stockholder the authority to call such
meeting and to preside thereat does not mean that the petition must be set
for hearing with notice served upon the board of directors.
The respondent court was satisfied that there was a showing of good
cause for authorizing the respondent Potenciano Gapol to call a meeting of
the stockholders for the purpose of electing the board of directors as
required and provided for in the by-laws, because the chairman of the board
of directors called upon to do so had failed, neglected, or refused to perform
his duty. It may be likened to a writ of preliminary injunction or of attachment
which may be issued ex-parte upon compliance with the requirements of the
rules and upon the court being satisfied that the same should issue. Such
provisional reliefs have not been deemed and held as violative of the due
process of law clause of the Constitution.
Petitioners were not deprived of their right without due process of law.
They had no right to continue as directors of the corporation unless reelected
by the stockholders in a meeting called for that purpose every even year.

Page 555 of 1072

The alleged illegality of the election of one member of the board of


directors at the meeting called by Gapol as authorized by the court being
subsequent to the order complained of cannot affect the validity and legality
of the order. If it be true that one of the directors elected at the meeting
called by Gapol, as authorized by the order of the court complained of, was
not qualified in accordance with the provisions of the by-laws, the remedy of
an aggrieved party would be quo warranto. Also, the alleged previous
agreement to dissolve the corporation does not affect or render illegal the
order issued by the respondent court.

TOPIC: WHO COULD ATTEND AND VOTE


SALVADOR P. LOPEZ, President of the University of the Philippines;
BOARD OF REGENTS, University of the Philippines; and OSEAS DEL
ROSARIO, Officer-in-Charge, College of Education, University of the
Philippines, petitioners,
vs.
HON. VICENTE ERICTA, Judge of the Court of First Instance of Rizal,
Branch XVIII (Quezon City), and DR. CONSUELO S. BLANCO,
respondents.
G.R. No. L-32991 June 29, 1972
45 SCRA 539
FACTS:
The first such appointment was extended on April 27, 1970, "effective
May 1, 1970 until April 30, 1971, unless sooner terminated and subject to the
approval of the Board of Regents and to pertinent University regulations."

Page 556 of 1072

Pursuant thereto Dr. Blanco assumed office as ad interim Dean on May 1,


1970.
The Board of Regents met on May 26, 1970, and President Lopez
submitted to it the ad interim appointment of Dr. Blanco for reconsideration.
The minutes of that meeting disclose that "the Board voted to defer action
on the matter in view of the objections cited by Regent Kalaw based on the
petition against the appointment, addressed to the Board, from a majority of
the faculty and from a number of alumni ..." Dr. Blanco's appointment had
lapsed.
On May 26, 1970, President Lopez extended another ad interim
appointment to her, effective from May 26, 1970 to April 30, 1971, with the
same conditions as the first. However, such ad interim appointment had not
been confirmed by the Board of Regents. Due to the following votes: 5-yes,
3-no and 4-abstain.
On August 18, 1970 Dr. Blanco wrote the President of the University,
protesting the appointment of Oseas A. del Rosario as Officer-in-Charge of
the College of Education. Neither communication having elicited any official
reply, Dr. Blanco went to the Court of First Instance of Quezon City on a
petition for certiorari and prohibition with preliminary injunction.
ISSUE/S:
Whether or not respondent Dr. Consuelo S. Blanco was duly elected
Dean of the College of Education, University of the Philippines, in
the meeting of the Board of Regents on July 9, 1970.
RULING:
No.
The votes of abstention, viewed in their setting, can in no way be
construed as votes for confirmation of the appointment. There can be no
doubt whatsoever as to the decision and recommendation of the three
members of the Personnel Committee: it was for rejection of the
appointment. No inference can be drawn from this that the members of the
Personnel Committee, by their abstention, intended to acquiesce in the
action taken by those who voted affirmatively. Neither, for that matter, can
such inference be drawn from the abstention that he was abstaining because
he was not then ready to make a decision.
Dr. Blanco was clearly not the choice of a majority of the members of
the Board of Regents, as unequivocally demonstrated by the transcript of the
proceedings. This fact cannot be ignored simply because the Chairman, in
submitting the question to the actual vote, did not frame it as accurately as
the preceding discussion called for, such that two of the Regents present
(Silva and Kalaw) had to make some kind of clarification.

Page 557 of 1072

TOPIC: VOTING; WHO MAY EXERCISE


WILSON P. GAMBOA, Petitioner,
vs.
FINANCE SECRETARY MARGARITO B. TEVES, et.al., Respondents.
PABLITO V. SANIDAD and ARNO V. SANIDAD, Petitioners-in-Intervention.
G.R. No. 176579.June 28, 2011
FACTS:
On 28 November 1928, the Philippine Legislature enacted Act No.
3436 which granted PLDT a franchise and the right to engage in
telecommunications business. In 1969, General Telephone and Electronics
Corporation (GTE), an American company and a major PLDT stockholder, sold

Page 558 of 1072

26 percent of the outstanding common shares of PLDT to PTIC. In 1977,


Prime Holdings, Inc. (PHI) was incorporated by several persons, including
Roland Gapud and Jose Campos, Jr. Subsequently, PHI became the owner of
111,415 shares of stock of PTIC by virtue of three Deeds of Assignment
executed by PTIC stockholders Ramon Cojuangco and Luis Tirso Rivilla. In
1986, the 111,415 shares of stock of PTIC held by PHI were sequestered by
the Presidential Commission on Good Government (PCGG). The 111,415 PTIC
shares, which represent about 46.125 percent of the outstanding capital
stock of PTIC, were later declared by this Court to be owned by the Republic
of the Philippines.
In 1999, First Pacific, a Bermuda-registered, Hong Kong-based
investment firm, acquired the remaining 54 percent of the outstanding
capital stock of PTIC.
Thereafter, First Pacific announced that it would exercise its right of first
refusal as a PTIC stockholder and buy the 111,415 PTIC shares by matching
the bid price of Parallax. However, First Pacific failed to do so by the 1
February 2007 deadline set by IPC and instead, yielded its right to PTIC itself
which was then given by IPC until 2 March 2007 to buy the PTIC shares. On
14 February 2007, First Pacific, through its subsidiary, MPAH, entered into a
Conditional Sale and Purchase Agreement of the 111,415 PTIC shares, or
46.125 percent of the outstanding capital stock of PTIC, with the Philippine
Government for the price of P25,217,556,000 or US$510,580,189. The sale
was completed on 28 February 2007.
Since PTIC is a stockholder of PLDT, the sale by the Philippine
Government of 46.125 percent of PTIC shares is actually an indirect sale of
12 million shares or about 6.3 percent of the outstanding common shares of
PLDT.
ISSUE/S:
Whether the term "capital" in Section 11, Article XII of the
Constitution refers to the total common shares only or to the total
outstanding capital stock (combined total of common and nonvoting preferred shares) of PLDT, a public utility.
RULING:
The Supreme Court agrees with petitioner and petitioners-inintervention. The term "capital" in Section 11, Article XII of the Constitution
refers only to shares of stock entitled to vote in the election of directors, and
thus in the present case only to common shares, and not to the total
outstanding capital stock comprising both common and non-voting preferred
shares.
The shares of stock of stock corporations may be divided into classes
or series of shares, or both, any of which classes or series of shares may

Page 559 of 1072

have such rights, privileges or restrictions as may be stated in the articles of


incorporation: Provided, That no share may be deprived of voting rights
except those classified and issued as "preferred" or "redeemable" shares,
unless otherwise provided in this Code: Provided, further, That there shall
always be a class or series of shares which have complete voting rights. Any
or all of the shares or series of shares may have a par value or have no par
value as may be provided for in the articles of incorporation: Provided,
however, That banks, trust companies, insurance companies, public utilities,
and building and loan associations shall not be permitted to issue no-par
value shares of stock.
Indisputably, one of the rights of a stockholder is the right to
participate in the control or management of the corporation. This is exercised
through his vote in the election of directors because it is the board of
directors that controls or manages the corporation. In the absence of
provisions in the articles of incorporation denying voting rights to preferred
shares, preferred shares have the same voting rights as common shares.
However, preferred shareholders are often excluded from any control, that is,
deprived of the right to vote in the election of directors and on other matters,
on the theory that the preferred shareholders are merely investors in the
corporation for income in the same manner as bondholders.
Holders of PLDT preferred shares are explicitly denied of the right to vote in
the election of directors. PLDTs Articles of Incorporation expressly state that
"the holders of Serial Preferred Stock shall not be entitled to vote at any
meeting of the stockholders for the election of directors or for any other
purpose or otherwise participate in any action taken by the corporation or its
stockholders, or to receive notice of any meeting of stockholders.
WHEREFORE, we PARTLY GRANT the petition and rule that the term
"capital" in Section 11, Article XII of the 1987 Constitution refers only to
shares of stock entitled to vote in the election of directors, and thus in the
present case only to common shares, and not to the total outstanding capital
stock (common and non-voting preferred shares). Respondent Chairperson of
the Securities and Exchange Commission is DIRECTED to apply this definition
of the term "capital" in determining the extent of allowable foreign ownership
in respondent Philippine Long Distance Telephone Company, and if there is a
violation of Section 11, Article XII of the Constitution, to impose the
appropriate sanctions under the law.

Page 560 of 1072

TOPIC: VOTING; WHO MAY EXERCISE


PHILIPPINE COCONUT PRODUCERS FEDERATION, INC. (COCOFED),
MANUEL V. DEL ROSARIO, DOMINGO P. ESPINA, SALVADOR P.
BALLARES, JOSELITO A. MORALEDA, PAZ M. YASON, VICENTE A.
CADIZ, CESARIA DE LUNA TITULAR, and RAYMUNDO C. DE VILLA,
Petitioners,
vs.
REPUBLIC OF THE PHILIPPINES, Respondent.
JOVITO R. SALONGA, WIGBERTO E. TAADA, OSCAR F. SANTOS, ANA
THERESIA HONTIVEROS, and TEOFISTO L. GUINGONA III, OppositorsIntervenors.

Page 561 of 1072

WIGBERTO E. TAADA, OSCAR F. SANTOS, SURIGAO DEL SUR


FEDERATION OF AGRICULTURAL COOPERATIVES (SUFAC) and MORO
FARMERS ASSOCIATION OF ZAMBOANGA DEL SUR (MOFAZS),
represented by ROMEO C. ROYANDOYAN; and PAMBANSANG
KILUSAN NG MGA SAMAHAN NG MAGSASAKA (PAKISAMA),
represented by VICENTE FABE, Movants-Intervenors.
G.R. Nos. 177857-58.February 11, 2010
DANILO B. URUSA, Petitioner,vs.REPUBLIC OF THE PHILIPPINES,
Respondent.
G.R. No. 178193
EDUARDO M. COJUANGCO, JR., Petitioner,vs.REPUBLIC OF THE
PHILIPPINES, Respondent.
G.R. No. 180705
FACTS:
As may be recalled, the Court, in its resolution adverted to, approved,
upon motion of petitioner Philippine Coconut Producers Federation, Inc.
(COCOFED), the conversion of the sequestered 753,848,312 Class "A" and
"B" common shares of San Miguel Corporation (SMC), registered in the name
of Coconut Industry Investment Fund (CIIF) Holding Companies (hereunder
referred to as SMC Common Shares), into 753,848,312 SMC Series Preferred
Shares.
Oppositors-intervenors harp on the perceived economic disadvantages
and harm that the government would likely suffer by the approval of the
proposed conversion. Pursuing this point, it is argued that the Court missed
the fact that the current value of the shares in question is increasing and the
"perceived advantages of pegging the issue price at PhP 75 are dwindling on
a daily basis."
Salonga, et al. also argue that the proposed redemption is a right to
buy the preferred shares at less than the market value. That the market
value of the preferred shares may be higher than the issue price of PhP 75
per share at the time of redemption is possible. But then the opposite
scenario is also possible. Again, the Court need not delve into policy
decisions of government agencies because of their expertise and special
knowledge of these matters. Suffice it to say that all indications show that
SMC will redeem said preferred shares in the third year and not later because
the dividend rate of 8% it has to pay on said shares is higher than the
interest it will pay to the banks in case it simply obtains a loan. When market
prices of shares are low, it is possible that interest rate on loans will likewise
be low.
On the other hand, if SMC has available cash, it would be prudent for it
to use such cash to redeem the shares than place it in a regular bank deposit
which will earn lower interests. It is plainly expensive and costly for SMC to

Page 562 of 1072

keep on paying the 8% dividend rate annually in the hope that the market
value of the shares will go up before it redeems the shares. Likewise, the
conclusion that respondent Republic will suffer a loss corresponding to the
difference between a high market value and the issue price does not take
into account the dividends to be earned by the preferred shares for the three
years prior to redemption. The guaranteed PhP 6 per share dividend
multiplied by three years will amount to PhP 18. If one adds PhP 18 to the
issue price of PhP 75, then the holders of the preferred shares will have
actually attained a price of PhP 93 which hews closely to the speculative
PhP 100 per share price indicated by movants-intervenors.
ISSUE/S:
Whether or not the conversion of the shares is patently
disadvantageous to the government and the coconut farmers, given
that SMCs option to redeem ensures that the shares will be bought
at less than their market value.
RULING:
It should be remembered that the SMC shares allegedly owned by the
CIIF companies are sequestered assets under the control and supervision of
the PCGG pursuant to Executive Order No. 1, Series of 1986. Be that as it
may, it is the duty of the PCGG to preserve the sequestered assets and
prevent their dissipation. In the exercise of its powers, the PCGG need not
seek or obtain the consent or even the acquiescence of the sequestered
assets owner with respect to any of its acts intended to preserve such assets.
Otherwise, it would be well-nigh impossible for PCGG to perform its duties
and exercise its powers under existing laws, for the owner of the sequestered
assets will more often than not oppose or resist PCGGs actions if their
consent is a condition precedent. The act of PCGG of proposing the
conversion of the sequestered SMC shares to Series 1 Preferred Shares was
clearly an exercise of its mandate under existing laws, where the consent of
the CIIF Companies is rendered unnecessary.
Besides, since the subject sequestered SMC shares are under custodia
legis, the Court has certain control over them and their fruits. Nonetheless,
the PCGG, having administrative control over the subject sequestered shares
pending resolution of the actual ownership thereof, possesses discretion,
taking into account the greater interest of the government and the farmers,
to decide on where to deposit on escrow the net dividend earnings of, and/or
redemption proceeds from, the Series 1 Preferred Shares of SMC. The
depository bank may be the DBP/LBP or the UCPB.
WHEREFORE, the Court resolves to DENY for lack of merit the: (1)
Motion for Reconsideration dated October 7, 2009 filed by oppositorsintervenors Jovito R. Salonga, Wigberto E. Taada, Oscar F. Santos, Ana
Theresa Hontiveros, and Teofisto L. Guingona III; and (2) Motion to Admit

Page 563 of 1072

Motion for Reconsideration with Motion for Reconsideration [Re: Conversion


of SMC Shares] dated October 16, 2009 filed by movants-intervenors
Wigberto E. Taada, Oscar F. Santos, SUFAC, MOFAZS, represented by Romeo
C. Royandoyan, and PAKISAMA, represented by Vicente Fabe.
The Court PARTIALLY GRANTS the Motion for Leave to Intervene and to
File and Admit Attached Motion for Partial Reconsideration dated October 5,
2009, and the Motion for Partial Reconsideration dated October 6, 2009 filed
by movant-intervenor UCPB.
The Court AMENDS its Resolution dated September 17, 2009 to give to the
PCGG the discretion in depositing on escrow the net dividend earnings on,
and/or redemption proceeds from, the Series 1 Preferred Shares of SMC,
either with the Development Bank of the Philippines/Land Bank of the
Philippines or with the United Coconut Planters Bank, having in mind the
greater interest of the government and the coconut farmers.

TOPIC: VOTING; WHO MAY EXERCISE


REPUBLIC OF THE PHILIPPINES, represented by the PRESIDENTIAL
COMMISSION ON GOOD GOVERNMENT (PCGG), petitioner,
vs.

Page 564 of 1072

COCOFED, ET AL. and BALLARES, ET AL.,1 EDUARDO M. COJUANGCO


JR. and the SANDIGANBAYAN (First Division) respondents.
G.R. No. 147062-64.December 14, 2001
372 SCRA 462
FACTS:
Immediately after the 1986 EDSA Revolution, then President Corazon
C. Aquino issued Executive Order (EO) Nos. 1, 2 and 14. Pursuant to these
laws, the PCGG issued and implemented numerous sequestrations, freeze
orders and provisional takeovers of allegedly ill-gotten companies, assets
and properties, real or personal.
Among the properties sequestered by the Commission were shares of
stock in the United Coconut Planters Bank (UCPB) registered in the names of
the alleged "one million coconut farmers," the so-called Coconut Industry
Investment Fund companies (CIIF companies) and Private Respondent
Eduardo Cojuangco Jr. (hereinafter "Cojuangco").
In connection with the sequestration of the said UCPB shares, the PCGG, on
July 31, 1987, instituted an action for reconveyance, reversion, accounting,
restitution and damages docketed as Case No. 0033 in the Sandiganbayan.
On November 15, 1990, upon Motion of Private Respondent COCOFED,
the Sandiganbayan issued a Resolution lifting the sequestration of the
subject UCPB shares on the ground that herein private respondents in
particular, COCOFED and the so-called CIIF companies had not been
impleaded by the PCGG as parties-defendants in its July 31, 1987 Complaint
for reconveyance, reversion, accounting, restitution and damages.
The Sandiganbayan ruled that the Writ of Sequestration issued by the
Commission was automatically lifted for PCGG's failure to commence the
corresponding judicial action within the six-month period ending on August 2,
1987 provided under Section 26, Article XVIII of the 1987 Constitution. The
anti-graft court noted that though these entities were listed in an annex
appended to the Complaint, they had not been named as partiesrespondents.

ISSUE/S:
Who may vote the sequestered UCPB shares while the main case for
their reversion to the State is pending in the Sandiganbayan?
RULING:
The Supreme Court holds that the government should be allowed to
continue voting those shares inasmuch as they were purchased with coconut

Page 565 of 1072

levy funds that are prima facie public in character or, at the very least, are
clearly affected with public interest.
The Court granted PCGG the right to vote the sequestered shares
because they appeared to be "assets belonging to the government itself." In
short, when sequestered shares registered in the names of private
individuals or entities are alleged to have been acquired with ill-gotten
wealth, then the two-tiered test is applied. However, when the sequestered
shares in the name of private individuals or entities are shown, prima facie,
to have been
1. originally government shares, or
2. purchased with public funds or those affected with
public interest, then the two-tiered test does not apply.
Rather, the public character exceptions in Baseco v. PCGG and
Cojuangco Jr. v. Roxas prevail; that is, the government shall vote the shares.
In sum, we hold that the Sandiganbayan committed grave abuse of
discretion in grossly contradicting and effectively reversing existing
jurisprudence, and in depriving the government of its right to vote the
sequestered UCPB shares which are prima facie public in character.
WHEREFORE, the Petition is hereby GRANTED and the assailed Order
SET ASIDE. The PCGG shall continue voting the sequestered shares until
Sandiganbayan Civil Case Nos. 0033-A, 0033-B and 0033-F are finally and
completely resolved. Furthermore, the Sandiganbayan is ORDERED to decide
with finality the aforesaid civil cases within a period of six (6) months from
notice. It shall report to this Court on the progress of the said cases every
three (3) months, on pain of contempt. The Petition in Intervention is
DISMISSED inasmuch as the reliefs prayed for are not covered by the main
issues in this case. No costs.

TOPIC: VOTING; WHO MAY EXERCISE


RAMON C. LEE and ANTONIO DM. LACDAO, petitioners,
vs.
THE HON. COURT OF APPEALS, SACOBA MANUFACTURING CORP.,
PABLO GONZALES, JR. and THOMAS GONZALES, respondents.
G.R. No. 93695. February 4, 1992

Page 566 of 1072

FACTS:
On November 15, 1985, a complaint for a sum of money was filed by
the International Corporate Bank, Inc. against the private respondents who,
in turn, filed a third party complaint against ALFA and the petitioners on
March 17, 1986.On September 17, 1987, the petitioners filed a motion to
dismiss the third party complaint which the Regional Trial Court of Makati,
Branch 58 denied in an Order dated June 27, 1988.
On July 18, 1988, the petitioners filed their answer to the third party
complaint.
Meanwhile, on July 12, 1988, the trial court issued an order requiring the
issuance of an alias summons upon ALFA through the DBP as a consequence
of the petitioner's letter informing the court that the summons for ALFA was
erroneously served upon them considering that the management of ALFA
had been transferred to the DBP.
In a manifestation dated July 22, 1988, the DBP claimed that it was not
authorized to receive summons on behalf of ALFA since the DBP had not
taken over the company which has a separate and distinct corporate
personality and existence.
ISSUE/S:
Whether or not the execution of the voting trust agreement by a
stockholders whereby all his shares to the corporation have been
transferred to the trustee deprives the stockholders of his position
as director of the corporation; to rule otherwise, would be violative
of section 23 of the Corporation Code.
RULING:
By its very nature, a voting trust agreement results in the separation
of the voting rights of a stockholder from his other rights such as the right to
receive dividends, the right to inspect the books of the corporation, the right
to sell certain interests in the assets of the corporation and other rights to
which a stockholder may be entitled until the liquidation of the corporation.
In the instant case, the point of controversy arises from the effects of
the creation of the voting trust agreement. The petitioners maintain that with
the execution of the voting trust agreement between them and the other
stockholders of ALFA, as one party, and the DBP, as the other party, the
former assigned and transferred all their shares in ALFA to DBP, as trustee.
The facts of this case show that the petitioners, by virtue of the voting
trust agreement executed in 1981 disposed of all their shares through
assignment and delivery in favor of the DBP, as trustee. Consequently, the
petitioners ceased to own at least one share standing in their names on the
books of ALFA as required under Section 23 of the new Corporation Code.

Page 567 of 1072

They also ceased to have anything to do with the management of the


enterprise. The petitioners ceased to be directors. Hence, the transfer of the
petitioners' shares to the DBP created vacancies in their respective positions
as directors of ALFA.

TOPIC: VOTING; WHO MAY EXERCISE


REPUBLIC OF THE PHILIPPINES (PRESIDENTIAL COMMISSION ON
GOOD GOVERNMENT), petitioner,
vs.
THE HONORABLE SANDIGANBAYAN (THIRD DIVISION) and VICTOR
AFRICA, respondents.AEROCOM INVESTORS AND MANAGERS, INC.,
BENITO NIETO, CARLOS NIETO, MANUEL NIETO III, RAMON NIETO,
ROSARIO ARELLANO, VICTORIA LEGARDA, ANGELA LOBREGAT, MA.

Page 568 of 1072

RITA DE LOS REYES, CARMEN TUAZON and RAFAEL VALDEZ,


intervenors.
G.R. Nos. 107789 & 147214.April 30, 2003
402 SCRA 84
FACTS:
On August 7, 1991, the Presidential Commission on Good Government
(PCGG) conducted an ETPI stockholders meeting during which a PCGG
controlled board of directors was elected. A special stockholders meeting
was later convened by the registered ETPI stockholders wherein another set
of board of directors was elected, as a result of which two sets of such board
and officers were elected. Africa, a stockholder of ETPI, alleging that the
PCGG had since January 29, 1988 been"illegally 'exercising' the rights of
stockholders of ETPI," especially in the election of the members of the board
of directors, filed the above-said motion before the Sandiganbayan.
The PCGG did not object to Africa's motion provided that, an Order be
issued upholding the right of PCGG to vote all the Class "A" shares of ETPI, in
the alternative, in the remote event that PCGG's right to vote the
sequestered shares be not upheld, an Order be issued: a. Disregarding the
Stock and Transfer Book and Booklet of Stock Certificates of ETPI in
determining who can vote the shares in an Annual Stockholders Meeting of
ETPI, b. Allowing PCGG to vote twenty-three and 90/100 percent (23.9%) of
the total subscription in ETPI, and c. Directing the amendment of the Articles
of Incorporation and By-laws of ETPI providing for the minimum safeguards
for the conservation of assets, prior to the calling of a stockholders meeting.
ISSUE/S:
Whether respondent Sandiganbayan acted with grave abuse of
discretion in ruling that the registered stockholders of ETPI had the
right to vote in spite ETPIs stock and transfer book was altered and
cannot be used as the basis to determine who can vote in a stock
holders meeting.
RULING:
In the case at bar, there was adequate justification to vote the
incumbent directors out of office and elect others in their stead because the
evidence showed prima facie that the former were just tools of President
Marcos and were no longer owners of any stock in the firm, if they ever were
at all. It must however be emphasized that the conduct of the PCGG
nominees in the BASECO Board in the management of the company's affairs
should henceforth be guided and governed by the norms herein laid down.
They should never for a moment allow themselves to forget they are
conservators, not owners of the business; they are fiduciaries, trustees, of

Page 569 of 1072

whom the highest degree of diligence and rectitude is, in the premises,
required. The PCGG cannot thus vote sequestered shares, except when there
are "demonstrably weighty and defensible grounds" or "when essential to
prevent disappearance or wastage of corporate property.

TOPIC: VOTING TRUST AGREEMENT


ROSAURA P. CORDON, complainant,
vs.
JESUS BALICANTA, respondent.
A.C. No. 2797. October 4, 2002
FACTS:

Page 570 of 1072

When her husband Felixberto C. Jaldon died, Rosaura Cordon and her
daughter Rosemarie inherited 21 parcels of land located in Zamboanga
City. Respondent, the lawyer who helped in the settlement of estate of the
deceased, enticed complainant and her daughter to organize a corporation
that would develop the said real properties into a high-scale commercial
complex with a beautiful penthouse for complainant. Relying on these
apparently sincere proposals, complainant and her daughter assigned 19
parcels of land to Rosaura Enterprises, Incorporated, a newly-formed and
duly registered corporation in which they assumed majority ownership. The
subject parcels of land were then registered in the name of the corporation.
Thereafter, respondent single-handedly ran the affairs of the corporation in
his capacity as Chairman of the Board, President, General Manager and
Treasurer.

Complainant and her daughter made several demands on respondent


for the delivery of the real properties they allegedly assigned to the
corporation, for an accounting of the proceeds of the LBP loan and as well as
the properties sold, and for the rentals earned by BCC. But the demands
remained unheeded. Hence, complainant and her daughter, terminated the
services of respondent as their lawyer and repeated their demands for
accounting and turn-over of the corporate funds, and the return of the 19
titles that respondent transferred to the corporation.

For his defense, respondent, in his comment and position paper,


denied employing deceit and machination in convincing complainant and her
daughter to assign their real properties to the corporation; that they freely
and voluntary executed the deeds of assignment and the voting trust
agreement that they signed.

ISSUE/S:
Did complainant and her daughter freely and voluntary executed
the deeds of assignment and the voting trust agreement that they
allegedly signed.
RULING:
No.

Page 571 of 1072

The voting trust referred to by respondent, even if it were assumed to


be valid, covered only 266 shares of complainants yet she owned a total of
1,039 shares after she and her daughter ceded in favor of the corporation 19
parcels of land. Being a former lawyer to complainant, respondent should
have ensured that her interest was safeguarded. Yet, complainant was
apparently and deliberately left it on the pretext that, she had executed a
voting trust agreement in favor of respondent. It is suspicious that
complainant was made to sign a voting trust agreement on 21 August 1981
and immediately thereafter, the resolutions authorizing respondent to obtain
a loan and to mortgage the 9 parcels of land were passed and approved. It is
further worth noting that complainants voting trust where she allegedly
entrusted 266 shares to respondent on August 21, 1981 had only a validity of
5 years. Thus, she should have had her entire holdings of 1,283 shares back
in her name in August 1986.Respondents purported minutes of
stockholders meeting do not reflect this. There was no explanation
whatsoever from respondent on how complainant and her daughter lost their
97% control holding in the corporation. Respondent cannot take refuge in the
contested voting trust agreement supposedly executed by complainant and
her daughter for the reason that it authorized respondent to represent
complainant for such matters.
Moreover the factual findings of the investigating commission, affirmed
by the IBP Board, disclosed that complainant and her daughter own 1,711
out of 1,750 shares of the outstanding capital stock of the corporation, based
on the Articles of Incorporation and deeds of transfer of the properties. But
respondents evidence showed that complainant had only 266 shares of
stock in the corporation while her daughter had none, notwithstanding the
fact that there was nothing to indicate that complainant and her daughter
ever conveyed their shares to others.
Respondent likewise did not explain why he did not return the certificates
representing the 266 shares after the lapse of 5 years from the time the
voting trust certificate was executed in 1981.
The records show that up to now, the complainant and her daughter
own 97% of the outstanding shares but respondent never bothered to
explain why they were never asked to participate in or why they were never
informed of important corporate decisions.

Page 572 of 1072

TOPIC:VOTING TRUST AGREEMENT


NATIONAL INVESTMENT AND DEVELOPMENT CORPORATION, EUSEBIO
VILLATUYA MARIO Y. CONSING and ROBERTO S. BENEDICTO,
petitioners,
vs.
HON. BENJAMIN AQUINO, in his official capacity as Presiding Judge of
Branch VIII of the Court of First Instance of Rizal, BATJAK INC.,
GRACIANO A. GARCIA and MARCELINO CALINAWAN JR., respondents.
G.R. No. L-34192. June 30, 1988
163 SCRA 153
FACTS:
Batjak, (Basic Agricultural Traders Jointly Administered Kasamahan) is a
Filipino-American corporation organized under the laws of the Philippines,
primarily engaged in the manufacture of coconut oil and copra cake for
export. In 1965, Batjak's financial condition deteriorated to the point of
bankruptcy. As of that year, Batjak's indebtedness to some private banks and
to the Philippine National Bank (PNB) amounted to P11, 915,000.00.
Thus, as security for the payment of its obligations and advances
against shipments, Batjak mortgaged its three (3) coco-processing oil mills in
Sasa, Davao City, Jimenez, Misamis Occidental and Tanauan, Leyte to Manila
Banking Corporation (Manila Bank), Republic Bank (RB), and Philippine
Commercial and Industrial Bank (PCIB), respectively. In need for additional
operating capital to place the three (3) coco-processing mills at their
optimum capacity and maximum efficiency and to settle, pay or otherwise
liquidate pending financial obligations with the different private banks,
Batjak applied to PNB for additional financial assistance. Consequently, a
Financial Agreement was submitted by PNB to Batjak for acceptance. The
terms and conditions of the Financial Agreement were duly accepted by
Batjak. Under said Agreement, NIDC would, as it actually did, invest P6,
722,500.00 in Batjak in the form of preferred shares of stock convertible
within five (5) years at par into common stock, to liquidate Batjak's
obligations to Republic Bank (RB), Manufacturers Bank and Trust Company
(MBTC) and Philippine Commercial & Industrial Bank (PCIB), and the balance
of the investment was to be applied to Batjak's past due account of P 5
million with the PNB.
Moreover, the financial accommodation that had been extended by
PNB to Batjak amounted to a total of P 14,207,859.51. Batjak also executed a

Page 573 of 1072

first mortgage in favor of PNB on all its properties located at Jimenez,


Misamis Occidental and Tanauan, Leyte. And, a Voting Trust Agreement was
executed on 26 October 1965 in favor of NIDC by the stockholders
representing 60% of the outstanding paid-up and subscribed shares of
Batjak. This agreement was for a period of five (5) years and, upon its
expiration, was to be subject to negotiation between the parties.
Forced by the insolvency of Batjak, PNB instituted extrajudicial
foreclosure proceedings against the oil mills of Batjak located in Tanauan,
Leyte and Jimenez, Misamis Occidental. The properties were sold to PNB as
the highest bidder, after Batjak failed to exercise its right to redeem the
foreclosed properties within the allowable one year period of redemption.
Subsequently, PNB transferred the ownership of the two (2) oil mills to NIDC
which, as afore stated, was a wholly-owned PNB subsidiary.
ISSUE/S:
Whether or not NIDC should turn over the assets of the three (3)
oil mills to Batjak after the alleged expiration of the Voting Trust
Agreement entered into between them.
RULING:
No.
It is clear that what was assigned to NIDC was the power to vote the
shares of stock of the stockholders of Batjak, representing 60% of Batjak's
outstanding shares, and who are the signatories to the agreement. The
power entrusted to NIDC also included the authority to execute any
agreement or document that may be necessary to express the consent or
assent to any matter, by the stockholders. Nowhere in the said provisions or
in any other part of the Voting Trust Agreement is mention made of any
transfer or assignment to NIDC of Batjak's assets, operations, and
management. NIDC was constituted as trustee only of the voting rights of
60% of the paid-up and outstanding shares of stock in Batjak.
Moreover, what was to be returned by NIDC as trustee to Batjak's
stockholders, upon the termination of the agreement, are the certificates of
shares of stock belonging to Batjak's stockholders, not the properties or
assets of Batjak itself which were never delivered, in the first place to NIDC,
under the terms of said Voting Trust Agreement.
In any event, a voting trust transfers only voting or other rights
pertaining to the shares subject of the agreement or control over the stock.
The law on the matter is Section 59, Paragraph 1 of the Corporation Code (BP
68) which provides:
Sec. 59. Voting Trusts One or more stockholders of a
stock corporation may create a voting trust for the purpose

Page 574 of 1072

of conferring upon a trustee or trusties the right to vote


and other rights pertaining to the shares for a period not
exceeding five (5) years at any one time: ...
The acquisition by PNB-NIDC of the properties in question was not made or
effected under the capacity of a trustee but as a foreclosing creditor for the
purpose of recovering on a just and valid obligation of Batjak.
TOPIC:VOTING TRUST AGREEMENT
LEON J. LAMBERT, plaintiff-appellant,
Vs.
T. J. FOX, defendant-appellee.
G.R. No. L-7991. January 29, 1914
FACTS:
The firm known as John r. Edgar & Co. engaged in the retail and
stationery business was taken over by its creditors, including herein plaintiffappellant and defendant- appellee as the two largest stockholders in the new
corporation called John R. Edgar & Co., Incorporated.
Thereafter, plaintiff- appellant and defendant-appellee entered into the
following agreement:
Not to sell, transfer, or otherwise dispose of any part of
their present holdings of stock in said corporation, till after
one year from date thereof
Either party violating said agreement shall pay to the other
the sum of one thousand (P1, 000) pesos as liquidated
damages, unless previous consent in writing to such sale,
transfer, or other disposition be obtained.
Despite said agreement and protest of Lambert, Fox sold his stock to
E.C. Mc Cullough & Co., a strong competitor of the said John R. Edgar & Co.,
Inc.
The trial court decided the case in favor of the defendant upon the
ground that the intention of the parties as it appeared from the contract in
question was to the effect that the agreement should be good and continue
only until the corporation reached a sound financial basis.
ISSUE/S:
Whether or not the stipulation in the contract suspending the
power to sell the stock referred to therein is an illegal stipulation,
is in restraint of trade and, therefore, offends public policy.
RULING:

Page 575 of 1072

No.
The suspension of the power to sell has a beneficial purpose, results in
the protection of the corporation as well as of the individual parties to the
contract, and is reasonable as to the length of time of the suspension. The
Supreme Court does not here undertake to discuss the limitations to the
power to suspend the right of alienation of stock, limiting them to the
statement that the suspension in this particular case is legal and valid.
The judgment is reversed, the case remanded with instructions to
enter a judgment in favor of the plaintiff and against the defendant for P1,
000, with interest; without costs in this instance.

Page 576 of 1072

TOPIC: CAPITAL STOCK DISTINGUISHED FROM CAPITAL


WILSON P. GAMBOA, Petitioner,
vs.
FINANCE SECRETARY MARGARITO B. TEVES, FINANCE
UNDERSECRETARY JOHN P. SEVILLA, AND COMMISSIONER RICARDO
ABCEDE OF THE PRESIDENTIAL COMMISSION ON GOOD
GOVERNMENT (PCGG) IN THEIR CAPACITIES AS CHAIR AND
MEMBERS, RESPECTIVELY, OF THE PRIVATIZATION COUNCIL,
CHAIRMAN ANTHONI SALIM OF FIRST PACIFIC CO., LTD. IN HIS
CAPACITY AS DIRECTOR OF METRO PACIFIC ASSET HOLDINGS INC.,
CHAIRMAN MANUEL V. PANGILINAN OF PHILIPPINE LONG DISTANCE
TELEPHONE COMPANY (PLDT) IN HIS CAPACITY AS MANAGING
DIRECTOR OF FIRST PACIFIC CO., LTD., PRESIDENT NAPOLEON L.
NAZARENO OF PHILIPPINE LONG DISTANCE TELEPHONE COMPANY,
CHAIR FE BARIN OF THE SECURITIES EXCHANGE COMMISSION, and
PRESIDENT FRANCIS LIM OF THE PHILIPPINE STOCK EXCHANGE,
Respondents.
G.R. No. 176579. June 28, 2011
FACTS:
Petitioner, a stockholder of PLDT states that Congress enacted Act No.
3436 which granted PLDT a franchise and the right to engage in
telecommunications business. Then, General Telephone and Electronics
Corporation (GTE), an American company and major PLDT stock, sold 26% of
the Outstanding Common Shares of PLDT to PTIC. Thereafter, Prime Holdings,
Inc, (PHI) was incorporated. Subsequently, PHI became the owner of 111,
415 shares of stock of PTIC thru 3 Deeds of Assignments. Thereafter, the said
111, 415 shares of stock, about 46.125% of the Outstanding Capital Stock of
PTIC were sequestered by the PCGG and later declared by the Supreme Court
to be owned by the Republic of the Philippines. The remaining 54% of the
outstanding capital stock of PTIC was acquired by First Pacific, a Bermudaregistered, and Hongkong- based investment firm.
ISSUE/S:
Whether the term "capital" in Section 11, Article XII of the
Constitution refers to the total common shares only or to the
total outstanding capital stock (combined total of common and
non-voting preferred shares) of PLDT, a public utility.
RULING:

Page 577 of 1072

The Supreme Court agrees with petitioner and petitioners-inintervention. The term "capital" in Section 11, Article XII of the Constitution
refers only to shares of stock entitled to vote in the election of directors, and
thus in the present case only to common shares, and NOT to the total
outstanding capital stock comprising both common and non-voting preferred
shares.
The Corporation Code of the Philippines classifies shares as common or
preferred, thus:
Sec. 6. Classification of shares. - The shares of stock of
stock corporations may be divided into classes or series of
shares, or both, any of which classes or series of shares
may have such rights, privileges or restrictions as may be
stated in the articles of incorporation: Provided, That no
share may be deprived of voting rights except those
classified and issued as "preferred" or "redeemable"
shares, unless otherwise provided in this Code:
Provided, further, That there shall always be a class or
series of shares which have complete voting rights. Any or
all of the shares or series of shares may have a par value
or have no par value as may be provided for in the articles
of incorporation: Provided, however, That banks, trust
companies, insurance companies, public utilities, and
building and loan associations shall not be permitted to
issue no-par value shares of stock.
Preferred shares of stock issued by any corporation may be given
preference in the distribution of the assets of the corporation in case of
liquidation and in the distribution of dividends, or such other preferences as
may be stated in the articles of incorporation which are not violative of the
provisions of this Code: Provided, That preferred shares of stock may be
issued only with a stated par value. The Board of Directors, where authorized
in the articles of incorporation, may fix the terms and conditions of preferred
shares of stock or any series thereof: Provided, That such terms and
conditions shall be effective upon the filing of a certificate thereof with the
Securities and Exchange Commission.
Shares of capital stock issued without par value shall be deemed fully
paid and non-assessable and the holder of such shares shall not be liable to
the corporation or to its creditors in respect thereto: Provided; That shares
without par value may not be issued for a consideration less than the value
of five (P5.00) pesos per share: Provided, further, That the entire
consideration received by the corporation for its no-par value shares shall be
treated as capital and shall not be available for distribution as dividends.

Page 578 of 1072

A corporation may, furthermore, classify its shares for the purpose of


insuring compliance with constitutional or legal requirements.
Except as otherwise provided in the articles of incorporation and stated
in the certificate of stock, each share shall be equal in all respects to every
other share.
Where the articles of incorporation provide for non-voting shares in the
cases allowed by this Code, the holders of such shares shall nevertheless be
entitled to vote on the following matters:
1. Amendment of the articles of incorporation;
2. Adoption and amendment of by-laws;
3. Sale, lease, exchange, mortgage, pledge or other disposition
of all or substantially all of the corporate property;
4. Incurring, creating or increasing bonded indebtedness;
5. Increase or decrease of capital stock;
6. Merger or consolidation of the corporation with another
corporation or other corporations;
7. Investment of corporate funds in another corporation or
business in accordance with this Code; and
8. Dissolution of the corporation.
Except as provided in the immediately preceding paragraph, the vote
necessary to approve a particular corporate act as provided in this Code shall
be deemed to refer only to stocks with voting rights.
Indisputably, one of the rights of a stockholder is the right to
participate in the control or management of the corporation. This is exercised
through his vote in the election of directors because it is the board of
directors that controls or manages the corporation. In the absence of
provisions in the articles of incorporation denying voting rights to preferred
shares, preferred shares have the same voting rights as common shares.
However, preferred shareholders are often excluded from any control, that is,
deprived of the right to vote in the election of directors and on other matters,
on the theory that the preferred shareholders are merely investors in the
corporation for income in the same manner as bondholders. In fact, under
the Corporation Code only preferred or redeemable shares can be deprived
of the right to vote. Common shares cannot be deprived of the right to vote
in any corporate meeting, and any provision in the articles of incorporation
restricting the right of common shareholders to vote is invalid
Considering that common shares have voting rights which translate to
control, as opposed to preferred shares which usually have no voting rights,
the term "capital" in Section 11, Article XII of the Constitution refers only to
common shares. However, if the preferred shares also have the right to vote
in the election of directors, then the term "capital" shall include such
preferred shares because the right to participate in the control or

Page 579 of 1072

management of the corporation is exercised through the right to vote in the


election of directors. In short, the term "capital" in Section 11, Article
XII of the Constitution refers only to shares of stock that can vote in
the election of directors.
This interpretation is consistent with the intent of the framers of the
Constitution to place in the hands of Filipino citizens the control and
management of public utilities. As revealed in the deliberations of the
Constitutional Commission, "capital" refers to the voting stock or
controlling interest of a corporation

Page 580 of 1072

TOPIC: LEGAL OR STATED CAPITAL


PHILIPPINE LONG DISTANCE TELEPHONE COMPANY, petitioner,
vs.
NATIONAL TELECOMMUNICATIONS COMMISSION, JOSEPH
A.SANTIAGO, in his capacity as NTC Commissioner, and EDGARDO
CABARRIOS, in his capacity as Chief, CCAD, respondents
G.R. No. 152685. December 4, 2007
FACTS:
The NTC, as authorized under the Public Service Act to collect from
public telecommunication companies Supervision and Regulation Fees (SRF)
of P0.50 for every P 100 or a fraction of the Capital and Stock Subscribed or
Paid for of a stock corporation, partnership or single proprietorship of the
capital invested or of the property and equipment, whichever is higher, sent
SRF assessments to PLDT based on the Market Value of the Outstanding
capital stock, including Stock Dividends.
Consequently, PLDT protested the assessments contending that the
SRF ought to be based on the par value of its outstanding capital stock.
However, NTC denied said protest. Its Motion for reconsideration was also
denied.
On appeal filed by PLDT, the CA modified the decision of the NTC by
holding that the SRF should be assessed at par value of the outstanding
capital stock of PLDT, excluding stock dividends.
ISSUE/S:
Are all the stock dividends that are part of the outstanding
capital stock of PLDT subject to the SRF?
RULING:
Yes.
Dividends, regardless of the form these are declared, that is, cash,
property or stocks, are valued at the amount of the declared dividend taken
from the unrestricted retained earnings of a corporation. Thus, the value of
the declaration in the case of a stock dividend is the actual value of the
original issuance of said stocks. In G.R. No. 127937 we said that "in the case

Page 581 of 1072

of stock dividends, it is the amount that the corporation transfers from its
surplus profit account to its capital account" or "it is the amount that the
corporation receives in consideration of the original issuance of the shares."
It is "the distribution of current or accumulated earnings to the shareholders
of a corporation pro rata based on the number of shares owned." Such
distribution in whatever form is valued at the declared amount or monetary
equivalent.
Thus, it cannot be said that no consideration is involved in the issuance
of stock dividends. In fact, the declaration of stock dividends is akin to a
forced purchase of stocks. By declaring stock dividends, a corporation
ploughs back a portion of its entire unrestricted retained earnings either to
its working capital or for capital asset acquisition or investments. It is
simplistic to say that the corporation did not receive any actual payment for
these. When the dividend is distributed, it ceases to be a property of the
corporation as the entire or portion of its unrestricted retained earnings is
distributed pro rata to corporate shareholders.
When stock dividends are distributed, the amount declared ceases to
belong to the corporation but is distributed among the shareholders.
Consequently, the unrestricted retained earnings of the corporation are
diminished by the amount of the declared dividend while the stockholders
equity is increased. Furthermore, the actual payment is the cash value from
the unrestricted retained earnings that each shareholder foregoes for
additional stocks/shares which he would otherwise receive as required by the
Corporation Code to be given to the stockholders subject to the availability
and conditioned on a certain level of retained earnings. Else wise put, where
the unrestricted retained earnings of a corporation are more than 100% of
the paid-in capital stock, the corporate Board of Directors is mandated to
declare dividends which the shareholders will receive in cash unless
otherwise declared as property or stock dividends, which in the latter case
the stockholders are forced to forego cash in lieu of property or stocks.
In essence, therefore, the stockholders by receiving stock dividends
are forced to exchange the monetary value of their dividend for capital stock,
and the monetary value they forego is considered the actual payment for the
original issuance of the stocks given as dividends. Therefore, stock dividends
acquired by shareholders for the monetary value they forego are under the
coverage of the SRF and the basis for the latter is such monetary value as
declared by the board of directors.

Page 582 of 1072

TOPIC:CONTROL TEST v. GRANDFATHERS RULE


WILSON P. GAMBOA, Petitioner,
vs.
FINANCE SECRETARY MARGARITO B. TEVES, FINANCE
UNDERSECRETARY JOHN P. SEVILLA, et al, Respondents.
PABLITO V. SANIDAD and ARNO V. SANIDAD, Petitioners-in-Intervention.
G.R. No. 176579, June 28, 2011
FACTS:
Petitioner Wilson P. Gamboa was a stockholder of Philippine Long
Distance Telephone Company (PLDT). General Telephone and Electronics
Corporation (GTE), an American company and a major PLDT stockholder, sold
26 percent of the outstanding common shares of PLDT to PTIC which was
owned by PHI by virtue of three Deeds of Assignment executed by PTIC
stockholders. In 1986, the 111,415 shares of stock of PTIC held by PHI were
sequestered by the Presidential Commission on Good Government (PCGG)
about 46.125 percent of the outstanding capital stock of PTIC, which were
later declared by this Court to be owned by the Republic of the Philippines.
In 1999, First Pacific, a Bermuda-registered, Hong Kong-based
investment firm, acquired the remaining 54 percent of the outstanding
capital stock of PTIC. Since PTIC is a stockholder of PLDT, the sale by the
Philippine Government of 46.125 percent of PTIC shares is actually an
indirect sale of 12 million shares or about 6.3 percent of the outstanding
common shares of PLDT. With the sale, First Pacifics common shareholdings
in PLDT increased from 30.7 percent to 37 percent, thereby increasing the
common shareholdings of foreigners in PLDT to about 81.47 percent, in
violation of Section 11, Article XII of the 1987 Philippine Constitution which
limits foreign ownership of the capital of a public utility to not more than 40
percent.
ISSUE/S:
Whether or not there was a violation of the constitutional
provision on the control test in this case.
RULING:

Page 583 of 1072

Section 11, Article XII (National Economy and Patrimony) of the 1987
Constitution mandates the Filipinization of public utilities, to wit:
No franchise, certificate, or any other form of
authorization for the operation of a public utility shall be
granted except to citizens of the Philippines or to
corporations or associations organized under the laws of
the Philippines, at least sixty per centum of whose capital
is owned by such citizens
Clearly, therefore, the forty percent (40%) foreign equity limitation in
public utilities prescribed by the Constitution refers to ownership of shares of
stock entitled to vote, i.e., common shares. Furthermore, ownership of record
of shares will not suffice but it must be shown that the legal and beneficial
ownership rests in the hands of Filipino citizens. Consequently, in the case of
petitioner PLDT, since it is already admitted that the voting interests of
foreigners which would gain entry to petitioner PLDT by the acquisition of
SMART shares through the Questioned Transactions is equivalent to 82.99%,
and the nominee arrangements between the foreign principals and the
Filipino owners is likewise admitted, there is, therefore, a violation of Section
11, Article XII of the Constitution. The term "capital" in Section 11, Article XII
of the Constitution refers only to shares of stock entitled to vote in the
election of directors, and thus in the present case only to common
shares, and not to the total outstanding capital stock comprising both
common and non-voting preferred shares. This interpretation is consistent
with the intent of the framers of the Constitution to place in the hands of
Filipino citizens the control and management of public utilities as revealed in
the deliberations of the Constitutional Commission that "capital" refers to the
voting stock or controlling interest.

Page 584 of 1072

TOPIC:CONTROL TEST v. GRANDFATHERS RULE


DEMOSTHENES P. AGAN, JR., et. al, petitioners,
vs.
PHILIPPINE INTERNATIONAL AIR TERMINALS CO., INC., et. al
respondents,
G.R. No. 155001, January 21, 2004
FACTS:
These are separate Motions for Reconsideration filed by respondents
declaring the contracts for the NAIA IPT III project null and void.
Briefly, the proceedings. On October 5, 1994, Asias Emerging Dragon
Corp. (AEDC) submitted an unsolicited proposal to the Philippine Government
through the Department of Transportation and Communication (DOTC) and
Manila International Airport Authority (MIAA) for the construction and
development of the NAIA IPT III under a build-operate-and-transfer
arrangement pursuant to R.A. No. 6957, as amended by R.A. No. 7718 (BOT
Law).
The Government, through then DOTC Secretary Arturo T. Enrile, and
PIATCO, through its President, Henry T. Go, signed the "Concession
Agreement for the Build-Operate-and-Transfer Arrangement of the Ninoy
Aquino International Airport Passenger Terminal III" (1997 Concession
Agreement) which were later amended and superseded.
ISSUE/S:
Whether or not the grandfathers rule is applicable in this case.
RULING:
It is settled that public interest on the occasion of a national
emergency is the primary consideration when the government decides to
temporarily take over or direct the operation of a public utility or a business
affected with public interest. The nature and extent of the emergency is the
measure of the duration of the takeover as well as the terms thereof. It is the
State that prescribes such reasonable terms which will guide the
implementation of the temporary takeover as dictated by the exigencies of

Page 585 of 1072

the time, which power of the State can not be negated by any party nor
should its exercise be a source of obligation for the State.
Section 17, Article XII of the 1987 Constitution grants the State in
times of national emergency the right to temporarily take over the operation
of any business affected with public interest. This right is an exercise of
police power which is one of the inherent powers of the State. It consists of
two essential elements. First, it is an imposition of restraint upon liberty or
property. Second, the power is exercised for the benefit of the common good.
It is and still is the "most essential, insistent, and illimitable" of the States
powers. Unlike the power of eminent domain, police power is exercised
without provision for just compensation for its paramount consideration is
public welfare.

Page 586 of 1072

TOPIC: CLASSIFICATION OF SHARES (VOTING/NON VOTING)


WILSON P. GAMBOA, Petitioner,
vs.
FINANCE SECRETARY MARGARITO B. TEVES, et. al., Respondents.
PABLITO V. SANIDAD and ARNO V. SANIDAD, Petitioners-in-Intervention.
G.R. No. 176579. June 28, 2011
FACTS:
On 28 November 1928, the Philippine Legislature enacted Act No. 3436
which granted PLDT a franchise and the right to engage in
telecommunications business. In 1969, General Telephone and Electronics
Corporation (GTE), an American company and a major PLDT stockholder, sold
26 percent of the outstanding common shares of PLDT to PTIC. In 1977,
Prime Holdings, Inc. (PHI) was incorporated by several persons, including
Roland Gapud and Jose Campos, Jr. Subsequently, PHI became the owner of
111,415 shares of stock of PTIC by virtue of three Deeds of Assignment
executed by PTIC stockholders Ramon Cojuangco and Luis Tirso Rivilla. In
1986, the 111,415 shares of stock of PTIC held by PHI were sequestered by
the Presidential Commission on Good Government (PCGG). The 111,415 PTIC
shares, which represent about 46.125 percent of the outstanding capital
stock of PTIC, were later declared by this Court to be owned by the Republic
of the Philippines.
In 1999, First Pacific, a Bermuda-registered, Hong Kong-based
investment firm, acquired the remaining 54 percent of the outstanding
capital stock of PTIC. On 20 November 2006, the Inter-Agency Privatization
Council (IPC) of the Philippine Government announced that it would sell the
111,415 PTIC shares, or 46.125 percent of the outstanding capital stock of
PTIC, through a public bidding to be conducted on 4 December 2006.
Subsequently, the public bidding was reset to 8 December 2006, and only
two bidders, Parallax Venture Fund XXVII (Parallax) and Pan-Asia Presidio
Capital, submitted their bids. Parallax won with a bid of P25.6 billion or
US$510 million.
Thereafter, First Pacific announced that it would exercise its right of
first refusal as a PTIC stockholder and buy the 111,415 PTIC shares by
matching the bid price of Parallax. However, First Pacific failed to do so by
the 1 February 2007 deadline set by IPC and instead, yielded its right to PTIC
itself which was then given by IPC until 2 March 2007 to buy the PTIC shares.

Page 587 of 1072

On 14 February 2007, First Pacific, through its subsidiary, MPAH, entered into
a Conditional Sale and Purchase Agreement of the 111,415 PTIC shares, or
46.125 percent of the outstanding capital stock of PTIC, with the Philippine
Government for the price of P25, 217,556,000 or US$510,580,189. The sale
was completed on 28 February 2007.
Since PTIC is a stockholder of PLDT, the sale by the Philippine Government of
46.125 percent of PTIC shares is actually an indirect sale of 12 million shares
or about 6.3 percent of the outstanding common shares of PLDT. With the
sale, First Pacifics common shareholdings in PLDT increased from
30.7 percent to 37 percent, thereby increasing the common
shareholdings of foreigners in PLDT to about 81.47 percent. This
violates Section 11, Article XII of the 1987 Philippine Constitution which limits
foreign ownership of the capital of a public utility to not more than 40
percent.
ISSUE/S:
Whether the term "capital" in Section 11, Article XII of the
Constitution refers to the total common shares only or to the
total outstanding capital stock (combined total of common and
non-voting preferred shares) of PLDT, a public utility.
RULING:
Any citizen or juridical entity desiring to operate a public utility must
meet the minimum nationality requirement prescribed in Section 11, Article
XII of the Constitution. Hence, for a corporation to be granted authority to
operate a public utility, at least 60 percent of its "capital" must be owned by
Filipino citizens.
Petitioner submits that the 40 percent foreign equity limitation in
domestic public utilities refers only to common shares because such shares
are entitled to vote and it is through voting that control over a corporation is
exercised. Petitioner posits that the term "capital" in Section 11, Article XII of
the Constitution refers to "the ownership of common capital stock subscribed
and outstanding, which class of shares alone, under the corporate set-up of
PLDT, can vote and elect members of the board of directors." It is undisputed
that PLDTs non-voting preferred shares are held mostly by Filipino citizens.
This arose from Presidential Decree No. 217, issued on 16 June 1973 by then
President Ferdinand Marcos, requiring every applicant of a PLDT telephone
line to subscribe to non-voting preferred shares to pay for the investment
cost of installing the telephone line.
The forty percent (40%) foreign equity limitation in public utilities
prescribed by the Constitution refers to ownership of shares of stock entitled
to vote, i.e., common shares. Furthermore, ownership of record of shares will
not suffice but it must be shown that the legal and beneficial ownership rests

Page 588 of 1072

in the hands of Filipino citizens. Consequently, in the case of petitioner PLDT,


since it is already admitted that the voting interests of foreigners which
would gain entry to petitioner PLDT by the acquisition of SMART shares
through the Questioned Transactions is equivalent to 82.99%, and the
nominee arrangements between the foreign principals and the Filipino
owners is likewise admitted, there is, therefore, a violation of Section 11,
Article XII of the Constitution.

TOPIC: CLASSIFICATION OF SHARES (VOTING/NON VOTING)


CECILIA CASTILLO, OSCAR DEL ROSARIO, ARTURO S. FLORES,
XERXES NAVARRO, MARIA ANTONIA TEMPLO and MEDICAL CENTER
PARAAQUE, INC., petitioners,
vs.
ANGELES BALINGHASAY, RENATO BERNABE, ALODIA DEL ROSARIO,
ROMEO FUNTILA, TERESITA GAYANILO, RUSTICO JIMENEZ, ARACELI**
JO, ESMERALDA MEDINA, CECILIA MONTALBAN, VIRGILIO OBLEPIAS,
CARMENCITA PARRENO, CESAR REYES, REYNALDO SAVET, SERAPIO
TACCAD, VICENTE VALDEZ, SALVACION VILLAMORA, and HUMBERTO
VILLAREAL, respondents.
G.R. No. 150976. October 18, 2004
FACTS:
Petitioners and the respondents are stockholders of MCPI, with the
former holding Class "B" shares and the latter owning Class "A" shares.MCPI
is a domestic corporation with offices at Dr. A. Santos Avenue, Sucat,
Paraaque City. It was organized sometime in September 1977. At the time
of its incorporation, Act No. 1459, the old Corporation Law was still in force
and effect. Article VII of MCPIs original Articles of Incorporation, as approved
by the Securities and Exchange Commission (SEC).
On July 31, 1981, Article VII of the Articles of Incorporation of MCPI was
amended, of which the authorized capital stock of the corporation is increase
from two million to FIVE MILLION (P5,000,000.00) PESOS. Only holders of
Class A shares have the right to vote and the right to be elected as directors
or as corporate officers.
The foregoing amendment was approved by the SEC on June 7, 1983.
While the amendment granted the right to vote and to be elected as
directors or corporate officers only to holders of Class "A" shares, holders of
Class "B" stocks were granted the same rights and privileges as holders of
Class "A" stocks with respect to the payment of dividends.

Page 589 of 1072

On September 9, 1992, Article VII was again amended to and that the
authorized capital stock of the corporation is now THIRTY TWO MILLION
PESOS (P32, 000,000.00). Except when otherwise provided by law, only
holders of Class "A" shares have the right to vote and the right to be elected
as directors or as corporate officers . The SEC approved the foregoing
amendment on September 22, 1993.
On February 9, 2001, the shareholders of MCPI held their annual
stockholders meeting and election for directors. During the course of the
proceedings, respondent Rustico Jimenez, citing Article VII, as amended, and
notwithstanding MCPIs history, declared over the objections of herein
petitioners, that no Class "B" shareholder was qualified to run or be voted
upon as a director. In the past, MCPI had seen holders of Class "B" shares
voted for and serve as members of the corporate board and some Class "B"
share owners were in fact nominated for election as board members.
Nonetheless, Jimenez went on to announce that the candidates holding Class
"A" shares were the winners of all seats in the corporate board.
The petitioners protested, claiming that Article VII was null and void
for depriving them, as Class "B" shareholders, of their right to vote and to be
voted upon, in violation of the Corporation Code (Batas Pambansa Blg. 68),
as amended. After their protest was given short shrift, herein petitioners filed
a Complaint for Injunction, Accounting and Damages. Said complaint was
founded on two (2) principal causes of action, namely: a. Annulment of the
declaration of directors of the MCPI made during the February 9, 2001 Annual
Stockholders Meeting, and for the conduct of an election whereat all
stockholders, irrespective of the classification of the shares they hold, should
be afforded their right to vote and be voted for; and b. Stockholders
derivative suit challenging the validity of a contract entered into by the
Board of Directors of MCPI for the operation of the ultrasound unit.
Subsequently, the complaint was amended to implead MCPI as party-plaintiff
for purposes only of the second cause of action.
ISSUE/S:
Whether or not holders of Class "B" shares of the MCPI may be
deprived of the right to vote and be voted for as directors in
MCPI.
RULING:
When Article VII of the Articles of Incorporation of MCPI was amended
in 1992, the phrase "except when otherwise provided by law" was inserted in
the provision governing the grant of voting powers to Class "A" shareholders.
This particular amendment is relevant for it speaks of a law providing for
exceptions to the exclusive grant of voting rights to Class "A" stockholders.
The determination of which law to apply is necessary. There are two laws
being cited and relied upon by the parties in this case. In this instance, the
law in force at the time of the 1992 amendment was the Corporation Code

Page 590 of 1072

(B.P. Blg. 68), not the Corporation Law (Act No. 1459), which had been
repealed by then. The law referred to in the amendment to Article VII refers
to the Corporation Code and no other law. At the time of the incorporation of
MCPI in 1977, the right of a corporation to classify its shares of stock was
sanctioned by Section 5 of Act No. 1459. The law repealing Act No. 1459, B.P.
Blg. 68, retained the same grant of right of classification of stock shares to
corporations, but with a significant change. Under Section 6 of B.P. Blg. 68,
the requirements and restrictions on voting rights were explicitly provided
for, such that "no share may be deprived of voting rights except those
classified and issued as "preferred" or "redeemable" shares, unless otherwise
provided in this Code" and that "there shall always be a class or series of
shares which have complete voting rights." Section 6 of the Corporation
Code being deemed written into Article VII of the Articles of Incorporation of
MCPI, it necessarily follows that unless Class "B" shares of MCPI stocks are
clearly categorized to be "preferred" or "redeemable" shares, the holders of
said Class "B" shares may not be deprived of their voting rights. Note that
there is nothing in the Articles of Incorporation nor an iota of evidence on
record to show that Class "B" shares were categorized as either "preferred"
or "redeemable" shares. The only possible conclusion is that Class "B" shares
fall under neither category and thus, under the law, are allowed to exercise
voting rights.
One of the rights of a stockholder is the right to participate in the
control and management of the corporation that is exercised through his
vote. The right to vote is a right inherent in and incidental to the ownership
of corporate stock, and as such is a property right. The stockholder cannot
be deprived of the right to vote his stock nor may the right be essentially
impaired, either by the legislature or by the corporation, without his consent,
through amending the charter, or the by-laws.
The non-impairment clause is inapplicable in this instance. When
Article VII of the Articles of Incorporation of MCPI were amended in 1992, the
board of directors and stockholders must have been aware of Section 6 of
the Corporation Code and intended that Article VII be construed in harmony
with the Code, which was then already in force and effect. Since Section 6 of
the Corporation Code expressly prohibits the deprivation of voting rights,
except as to "preferred" and "redeemable" shares, then Article VII of the
Articles of Incorporation cannot be construed as granting exclusive voting
rights to Class "A" shareholders, to the prejudice of Class "B" shareholders,
without running afoul of the letter and spirit of the Corporation Code.

Page 591 of 1072

TOPIC: CLASSIFICATION OF SHARES (VOTING/NON VOTING)


JULIO E. T. SALES and GEORGE V. AGONIAS, in their own behalf, and
in behalf of SIPALAY MINING EXPLORATION CORPORATION, as
minority stockholders thereof, and SIPALAY MINING EXPLORATION
CORPORATION, petitioners,
vs.
SECURITIES AND EXCHANGE COMMISSION, STATE INVESTMENT
HOUSE, INC., represented by its President, ANSELMO TRINIDAD;
ANSELMO TRINIDAD CO., INC., represented by its President,
ANSELMO TRINIDAD; and VULCAN INDUSTRIAL AND MINING CORP.,
represented by its President, WALTER W. BROWN; AFREDO C.
RAMOS, ANNABELLE P. BROWN, WALTER W. BROWN, MANUEL C.
DIAZ, and AUGUSTO B. SUNICO, respondents.
G.R. No. L-54330 January 13, 1989
169 SCRA 109
FACTS:
On or about June 13, 1974, respondent State Investment House, Inc.
(formerly State Financing Center, Inc.) entered into a sales agreement with
Sipalay Mining whereby the latter sold to the former 200,000,000 common
shares of its capital stock in the amount of P2, 600,000.00. The 200,000,000
shares of stock of Sipalay Mining, covered by ten certificates of stock, were
delivered to State Investment. Subsequently, the restriction on the sale of
the shares was modified. On October 19, 1974, the Board of Directors of
Sipalay Mining approved the amendment of the sales agreement by allowing
sale in blocks of 5,000,000 shares per buyer.
On December 22, 1975, State Investment addressed a letter to Sipalay
Mining requesting that the latter transfer the 200,000,000 shares to Anselmo

Page 592 of 1072

Trinidad & Co., Inc. (hereinafter referred to as ATCO), to which it had sold the
shares. Sipalay Mining complied with this request. During the time that ATCO
held the shares, it voted them in the stockholders' meetings of Sipalay
Mining.
On July 17, 1978, or some two and a half years later, ATCO in turn sold
198,500,000 of the shares to respondent VULCAN. Sipalay Mining was
requested by ATCO to transfer the 198,500,000 shares to the name of
VULCAN. By resolution of the Board of Directors of Sipalay Mining, its
President was directed to sign the certificate of stock that would effect the
transfer.
Eight days prior to the scheduled annual stockholders' meeting of
Sipalay Mining on July 18,1979, petitioners filed before the SEC a petition to
nullify the sale of the shares to VULCAN, with a prayer for the issuance of a
writ of preliminary injunction to enjoin VULCAN from voting the shares.
The SEC temporarily restrained VULCAN from voting its 198,500,000
shares at the 1979 annual stockholders' meeting pending resolution of
petitioners' petition for the issuance of a writ of preliminary injunction.
The annual stockholders' meeting of Sipalay Mining proceeded on July
18, 1979 without the participation of VULCAN's 198,500,000 shares and the
members of the Board of Directors were elected. Meanwhile, hearings on
petitioners' petition for injunction continued. In the March 10, 1980 issue of
the Bulletin Today, a Notice of Call was published, calling for the payment of
twenty percent (20%) of unpaid subscriptions in Sipalay Mining on or before
April 15, 1980. VULCAN immediately petitioned the SEC to issue a writ of
injunction.
ISSUE/S:
Whether the sale of the shares to VULCAN and the right to vote
them in the annual stockholders' meeting squarely falls within
the original and exclusive jurisdiction of the SEC.
RULING:
The sale of the shares to VULCAN and the right to vote them in the
annual stockholders' meeting squarely falls within the original and exclusive
jurisdiction of the SEC. The court likewise finds that it was within the powers
of the SEC to compel the officers of Sipalay Mining to call a stockholders'
meeting under its supervision. Under Section 5 of P.D. No. 902-A, the SEC
had original and exclusive jurisdiction over the controversy. It was "in order
to effectively exercise such jurisdiction", to borrow the language of P.D. No.
902-A, that the SEC ordered the creation of the committee, in the exercise of
its broad powers of control and supervision over corporations and its more

Page 593 of 1072

specific power to compel the officers of a corporation to call meetings of


stockholders under its supervision.
The Court finds the functions delegated to the committee to be in
accordance with the SEC's mandate. The powers delegated to the committee
were all confined to the holding of the stockholders' meeting and the conduct
of the election of directors in connection therewith. This displays the
circumspect and cautious manner in which the SEC exercised its broad
powers under P.D. No. 902-A.
The Court, therefore, finds no basis to sustain petitioners' contention
that the SEC acted arbitrarily and gravely abused its discretion when it
ordered the creation of a committee to supervise the stockholders' meeting
and election of directors.

TOPIC: REDEEMABLE SHARES


REPUBLIC PLANTERS BANK, petitioner,
vs.
HON. ENRIQUE A. AGANA, SR., as Presiding Judge, Court of First
Instance of Rizal, Branch XXVIII, Pasay City, ROBES-FRANCISCO
REALTY & DEVELOPMENT CORPORATION and ADALIA F. ROBES,
respondents.
G.R. No. 51765. March 3, 1997
269 SCRA
FACTS:
On September 18, 1961, private respondent Corporation secured a
loan from petitioner in the amount of P120, 000.00. As part of the proceeds
of the loan, preferred shares of stocks were issued to private respondent
Corporation, through its officers then, private respondent Adalia F. Robes and
one Carlos F. Robes. In other words, instead of giving the legal tender totaling
to the full amount of the loan, which is P120, 000.00, petitioner lent such
amount partially in the form of money and partially in the form of stock
certificates numbered 3204 and 3205, each for 400 shares with a par value
of P10.00 per share, or for P4, 000.00 each, for a total of P8, 000.00. Said
stock certificates were in the name of private respondent Adalia F. Robes and
Carlos F. Robes, who subsequently, however, endorsed his shares in favor of
Adalia F. Robes.
On January 31, 1979, private respondents proceeded against petitioner
and filed a Complaint anchored on private respondents' alleged rights to

Page 594 of 1072

collect dividends under the preferred shares in question and to have


petitioner redeem the same under the terms and conditions of the stock
certificates. Private respondents attached to their complaint, a letter-demand
dated January 5, 1979 which, significantly, was not formally offered in
evidence.
ISSUE/S:
Whether petitioner can be compelled to redeem the preferred
shares issued to the private respondent.
RULING:
The redemption of said shares cannot be allowed. As pointed out by
the petitioner, the Central Bank made a finding that said petitioner has been
suffering from chronic reserve deficiency,and that such finding resulted in a
directive, issued on January 31, 1973 by then Gov. G.S. Licaros of the Central
Bank, to the President and Acting Chairman of the Board of the petitioner
bank prohibiting the latter from redeeming any preferred share, on the
ground that said redemption would reduce the assets of the Bank to the
prejudice of its depositors and creditors. Redemption of preferred shares was
prohibited for a just and valid reason. The directive issued by the Central
Bank Governor was obviously meant to preserve the status quo, and to
prevent the financial ruin of a banking institution that would have resulted in
adverse repercussions, not only to its depositors and creditors, but also to
the banking industry as a whole. The directive, in limiting the exercise of a
right granted by law to a corporate entity, may thus be considered as an
exercise of police power. The respondent judge insists that the directive
constitutes an impairment of the obligation of contracts. It has, however,
been settled that the Constitutional guaranty of non-impairment of
obligations of contract is limited by the exercise of the police power of the
state, the reason being that public welfare is superior to private rights.
Since the stock certificate granted the private respondents the right to
receive a quarterly dividend of One Per Centum (1%) cumulative and
participating, it "clearly and unequivocally indicates that the same are
"interest bearing stocks" or stocks issued by a corporation under an
agreement to pay a certain rate of interest thereon. As such, plaintiffs
(private respondents herein) become entitled to the payment thereof as a
matter of right without necessity of a prior declaration of dividend.There is
no legal basis for this observation. Both Sec. 16 of the Corporation Law and
Sec. 43 of the present Corporation Code prohibit the issuance of any stock
dividend without the approval of stockholders, representing not less than
two-thirds (2/3) of the outstanding capital stock at a regular or special
meeting duly called for the purpose. These provisions underscore the fact
that payment of dividends to a stockholder is not a matter of right but a
matter of consensus. Furthermore, "interest bearing stocks", on which the

Page 595 of 1072

corporation agrees absolutely to pay interest before dividends are paid to


common stockholders, is legal only when construed as requiring payment of
interest as dividends from net earnings or surplus only.Clearly, the
respondent judge, in compelling the petitioner to redeem the shares in
question and to pay the corresponding dividends, committed grave abuse of
discretion amounting to lack or excess of jurisdiction in ignoring both the
terms and conditions specified in the stock certificate, as well as the clear
mandate of the law.

TOPIC:CLASSIFICATION OF SHARES TREASURY


COMMISSIONER OF INTERNAL REVENUE
vs.
JOHN L. MANNING et al.
G.R. NO. L-28398, August 06, 1975
66 SCRA 14
FACTS:
In 1952 the MANTRASCO had an authorized capital stock of P2,500,000
divided into25,000 common shares; 24,700 of these were owned by Julius S.
Reese, and the rest, at 100 shares each, by the three respondents.
On February 2, 1955, after MANTRASCO made a partial payment of
Reese's shares, the certificate for the 24,700 shares in Reese's name was
cancelled and a new certificate was issued in the name of MANTRASCO. On
the same date, and in the meantime that Reese's interest had not been fully
paid, the new certificate was endorsed to the law firm until it was finally paid
in full by MANTRASCO. The trustees delivered to MANTRASCO all the shares
which they were holding in trust. The BIR examiners concluded that the
distribution of Reese'ss hares as stock dividends was in effect a distribution
of the "asset or property of the corporation as may be gleaned from
the payment of cash for the redemption of said stock and distributing the
same as stock dividend."

Page 596 of 1072

Commissioner maintains that the full value (P7,973,660) of the shares


redeemed from Reese by MANTRASCO which were subsequently distributed
to the respondents as stock dividends in1958 should be taxed as income of
the respondents for that year, the said distribution being in effect a
distribution of cash. The respondents' interests in MANTRASCO, he further
argues, were only .4% prior to the declaration of the stock dividends in
1958, but rose to 33 1/3% each after the said declaration. In submitting their
respective contentions, it is the assumption of both parties that the 24,700
shares declared as stock dividends were treasury shares.
ISSUE/S:
Whether or not the shares in question treasury shares.
RULING:
Treasury shares are stocks issued and fully paid for and re-acquired by
the corporation either by purchase, donation, forfeiture or other means.
Treasury shares are therefore issued shares, but being in the treasury they
do not have the status of outstanding shares. Consequently, although a
treasury share, not having been retired by the corporation re-acquiring it,
may be re-issued or sold again, such share, as long as it is held by
the corporation as a treasury share, participates neither in dividends,
because dividends cannot be declared by the corporation to itself, nor in the
meetings of the corporation as voting stock, for otherwise equal distribution
of voting powers among stockholders will be effectively lost and the directors
will be able to perpetuate their control of the corporation, though it still
represents a paid-for interest in the property of the corporation.
The foregoing essential features of a treasury stock are lacking in the
questioned shares. The manifest intention of the parties to the trust
agreement was, in sum and substance, to treat the 24,700 shares of
Reese as absolutely outstanding shares of Reese's estate until they were
fully paid. Such being the true nature of the 24,700 shares, their declaration
as treasury stock dividend in 1958 was a complete nullity and plainly
violative of public policy. A stock dividend, being one payable in capital stock,
cannot be declared out of outstanding corporate stock, but only from
retained earnings. A stock dividend always involves a transfer of surplus (or
profit) to capital stock.

Page 597 of 1072

TOPIC:CLASSIFICATION OF SHARES TREASURY


SAN MIGUEL CORPORATION, NEPTUNIA CORPORATION LIMITED,
ANDRES SORIANO III AND ANSCOR-HAGEDORN SECURITIES,
INC., petitioners,
vs.
SANDIGANBAYAN (FIRST DIVISION), PHILIPPINE COCONUT
PRODUCERS FEDERATION, INC. (COCOFED), et. al., respondents.
G.R. NOS. 104637-38, SEPTEMBER 14, 2000
FACTS:
It appears that on March 26, 1986, the Coconut Industry Investment
Fund Holding Companies ("CIIF" for brevity) sold 33,133,266 shares of the
outstanding capital stock of San Miguel Corporation to Andres Soriano III of
the SMC Group payable in four (4) installments. Soriano III paid the initial
P500 million to the UCPB as administrator of the CIIF registered in the name
of Anscor-Hagedorn Securities, Inc. (AHSI).
On April 7, 1986, the Presidential Commission on Good Government
(PCGG) sequestered the shares of stock subject of the sale. Due to the
sequestration, the SMC Group (hereinafter referred to as the petitioners)

Page 598 of 1072

suspended payment of the balance of the purchase price of the subject


stocks. In retaliation, the UCPB Group rescinded the sale.
SMC claimed that the First Installment Shares shall revert to the
SMC treasury for dispersal pursuant to the SMC Stock Dispersal Plan to be
sold to raise funds at the soonest possible time for the expansion program of
SMC.
ISSUE/S:
Whether or not the claim of SMC that the 26.45 million shares
are now Treasury Shares.
RULING:
The SMC Group's primary justification for non-compliance with the
Resolution of this Court requiring it to turn over the certificates of stock for
the 25.45 million sequestered shares as well as the cash dividends already
accrued thereon is the fact that the shares of stock have allegedly now
become Treasury Shares.
The SMC Group, however, forgets two things:
a. Under the Corporation Code 'Treasury shares are shares of
stock which have been issued and fully paid for, but
subsequently reacquired by, the issuing corporation by
purchase, redemption, donation or through some lawful
means . . .' (Sec. 9, B.P. Blg. 68, Corporation Code). These
26.45 million shares of stock or any portion thereof can,
therefore, become Treasury Shares, i.e., property of the
San Miguel Corporation, only if the sale between the UCPB
Group and the SMC Group is allowed; otherwise these
shares cannot even begin to be deemed to have been 'reacquired by the issuing corporation,' i.e., the San Miguel
Corporation;
b. Even then, under the AGREEMENT between the UCPB
Group and the SMC Group on March 26, 1986 for the sale
of 33.1 million shares of SMC, the buyers were not only the
San Miguel Corporation but also Andres Soriano, III, the
Neptunia Corporation Limited of Hongkong and the AnscorHagedorn Securities, Inc.Under the letter of the PCGG
Commissioner Ramon Diaz dated May 19, 1986, the
Corporate Secretary of the San Miguel Corporation was
forbidden from recording the transfer, conveyance, and
encumbrance of these shares without the PCGG's
approval. This was by virtue of the PCGG's powers under
Sec. 2 of E.O. No. 2.

Page 599 of 1072

Unless, therefore, the right of Neptunia, Andres Soriano, III and the
Anscor-Hagedorn Securities, Inc. to these 26.45 million shares shall have
been transferred to the SMC, the SMC cannot be deemed to have
'reacquired' these shares. They would remain co-owned by all four (4)
entities.
But even if, indeed, these shares are treasury shares, they remain
sequestered so that any movement of these shares cannot be of any
permanent character that will alter their being sequestered shares and,
therefore, in 'custodia legis,' that is to say, under the control and disposition
of this Court.

TOPIC:TRUST FUND DOCTRINE


NATIONAL TELECOMMUNICATIONS COMMISSION, petitioner,
vs.
HONORABLE COURT OF APPEALS and PHILIPPINE LONG DISTANCE
TELEPHONE COMPANY, respondents.
G.R. No. 127937. July 28, 1999
FACTS:
Sometime in 1988, the National Telecommunications Commission (NTC)
served on the Philippine Long Distance Telephone Company (PLDT)
assessment notices and demands for payment. PLDT challenged the
aforesaid assessments. On September 29, 1993, the NTC rendered a
Decision, denying the protest of PLDT.
On October 22, 1993, PLDT interposed a Motion for Reconsideration,
which was denied by NTC in an Order issued on May 3, 1994. On May 12,
1994, PLDT appealed the aforesaid Decision to the Court of Appeals, which
came out with its questioned Decision of October 30, 1996, modifying the
disposition of NTC.

Page 600 of 1072

On November 20, 1996, NTC moved for partial reconsideration of the


Decision, with respect to the basis of the assessment under Section 40(e),
i.e., par value of the subscribed capital stock. It also sought a partial
reconsideration of the fee of fifty (P0.50) centavos for the issuance or
increasing of the capital stock under Section 40 (f). With the denial of its
motions for reconsideration by the Resolution of the Court of Appeals dated
January 27, 1997, petitioner found its way to the Court via the Petition for
Certiorari.

ISSUE/S:
Whether or not the Court of Appeals erred in holding that the
computation of Supervision and regulation Fees under Section
40(f) of the Public Service Act should be based on the par value
of the subscribed capital stock and not on the market value of
PLDTs outstanding capital stock inclusive of stock dividends and
premium.
RULING:
Succinct and clear is the ruling of this Court in the case of Philippine
Long Distance Telephone Company vs. Public Service Commission, 66 SCRA
341, that the basis for computation of the fee to be charged by NTC on PLDT,
is the capital stock subscribed or paid and not, alternatively, the property
and equipment. The fee in question is based on the capital stock subscribed
or paid, nothing less nothing more.
The term capital and other terms used to describe the capital
structure of a corporation are of universal acceptance, and their usages have
long been established in jurisprudence. Briefly, capital refers to the value of
the property or assets of a corporation. The capital subscribed is the total
amount of the capital that persons (subscribers or shareholders) have agreed
to take and pay for, which need not necessarily be, and can be more than,
the par value of the shares. In fine, it is the amount that the corporation
receives, inclusive of the premiums if any, in consideration of the original
issuance of the shares. In the case of stock dividends, it is the amount that
the corporation transfers from its surplus profit account to its capital
account. It is the same amount that can loosely be termed as the trust
fund of the corporation. The Trust Fund doctrine considers this subscribed
capital as a trust fund for the payment of the debts of the corporation, to

Page 601 of 1072

which the creditors may look for satisfaction. Until the liquidation of the
corporation, no part of the subscribed capital may be returned or released to
the stockholder (except in the redemption of redeemable shares) without
violating this principle. Thus, dividends must never impair the subscribed
capital; subscription commitments cannot be condoned or remitted; nor can
the corporation buy its own shares using the subscribed capital as the
consideration therefore.
In the same way that the Court in PLDT vs. PSC has rejected the value
of the property and equipment as being the proper basis for the fee
imposed by Section 40(e) of the Public Service Act, as amended by Republic
Act No. 3792, so also must the Court disallow the idea of computing the fee
on the par value of [PLDTs] capital stock subscribed or paid excluding stock
dividends, premiums, or capital in excess of par. Neither, however, is the
assessment made by the National Telecommunications Commission on the
basis of the market value of the subscribed or paid-in capital stock
acceptable since it is itself a deviation from the explicit language of the law.

TOPIC:TRUST FUND DOCTRINE


ONG YONG, JUANITA TAN ONG, WILSON T. ONG, ANNA L. ONG,
WILLIAM T. ONG, WILLIE T. ONG, And JULIE ONG ALONZO, Petitioners,
vs.
DAVID S. TIU, CELY Y. TIU,et. al , Respondents.
G.R. No. 144476. April 8, 2003
401 SCRA 1
FACTS:
The construction of the Masagana Citimall was threatened with
stoppage and incompletion when its owner, FLADC which was owned by the
Tius, encountered dire financial difficulties. It was heavily indebted to the
PNB for P190 million. To stave off foreclosure of the mortgage on the two lots
where the mall was being built, the Tius invited Ong Yong, Juanita Tan Ong,
Wilson T. Ong, Anna L. Ong, William T. Ong and Julia Ong Alonzo (the Ongs),

Page 602 of 1072

to invest in FLADC. Under the Pre-Subscription Agreement, the Ongs and the
Tius agreed to maintain equal shareholdings in FLADC: the Ongs were to
subscribe to 1,000,000 shares at a par value of P100.00 each while the Tius
were to subscribe to an additional 549,800 shares at P100.00 each in
addition to their already existing subscription of 450,200 shares.
Accordingly, the Ongs paid P100M in cash for their subscription to 1M
shares of stock while the Tius committed to contribute to FLADC a four-storey
building and two parcels of land respectively valued at P20M for 200K shares,
P30M for 300K shares and P49.8M or 49,800 shares to cover their additional
549,800 stock subscription therein. The Ongs paid in another P70M to FLADC
and P20M to the Tius over and above their P100M investment, the total sum
of which P190M was used to settle the P190M mortgage indebtedness of
FLADC to PNB.
The Tius, on February 23, 1996, rescinded the Pre-Subscription
Agreement. The Tius accused the Ongs of (1) refusing to credit to them the
FLADC shares covering their real property contributions; (2) preventing David
S. Tiu and Cely Y. Tiu from assuming the positions of and performing their
duties as Vice-President and Treasurer, respectively, and (3) refusing to give
them the office spaces agreed upon.
The business harmony between the parties were short-lived when the
Tius rescinded unilaterally the Pre-subcription agreement accussing the
Ongs of refusing to credit to them the FLADC shares covering their real
property contributions and preventing the Tius from assuming the positions
of and performing their duties and refusing to give them office spaces agreed
upon.
ISSUE/S:
Whether or not the trust fund doctrine is violated in this case.
RULING:
The Trust Fund Doctrine, first enunciated by this Court in the 1923 case
of Philippine Trust Co. vs. Rivera,provides that subscriptions to the capital
stock of a corporation constitute a fund to which the creditors have a right to
look for the satisfaction of their claims.This doctrine is the underlying
principle in the procedure for the distribution of capital assets, embodied in
the Corporation Code, which allows the distribution of corporate capital only
in three instances:
(1)amendment of the Articles of Incorporation to reduce the
authorized capital stock,
(2)purchase of redeemable shares by the corporation, regardless of
the existence of unrestricted retained earnings, and
(3)dissolution and eventual liquidation of the corporation.

Page 603 of 1072

Furthermore, the doctrine is articulated in Section 41 on the power of a


corporation to acquire its own shares and in Section 122 on the prohibition
against the distribution of corporate assets and property unless the stringent
requirements therefor are complied with.
In the instant case, the rescission of the Pre-Subscription Agreement
will effectively result in the unauthorized distribution of the capital assets
and property of the corporation, thereby violating the Trust Fund Doctrine
and the Corporation Code, since rescission of a subscription agreement is not
one of the instances when distribution of capital assets and property of the
corporation is allowed. Contrary to the Tius' allegation, rescission will, in the
final analysis, result in the premature liquidation of the corporation without
the benefit of prior dissolution. The Tius maintain that rescinding the
subscription contract is not synonymous to corporate liquidation because all
rescission will entail would be the simple restoration of the status quo
ante and a return to the two groups of their cash and property contributions.
The Tius' case for rescission cannot validly be deemed a petition to
decrease capital stock because such action never complied with the formal
requirements for decrease of capital stock under Section 33 of the
Corporation Code. No majority vote of the board of directors was ever taken.
Neither was there any stockholders meeting at which the approval of
stockholders owning at least two-thirds of the outstanding capital stock was
secured. There was no revised treasurer's affidavit and no proof that said
decrease will not prejudice the creditors' rights. On the contrary, all their
pleadings contained were alleged acts of violations by the Ongs to justify an
order of rescission.

TOPIC:WHAT IS SUBSCRIPTION
ONG YONG, JUANITA TAN ONG, WILSON T. ONG, ANNA L. ONG,
WILLIAM T. ONG, WILLIE T. ONG, And JULIE ONG ALONZO, Petitioners,
vs.
DAVID S. TIU, CELY Y. TIU,et. al , Respondents.
G.R. No. 144476. April 8, 2003
401 SCRA 1
FACTS:
The construction of the Masagana Citimall was threatened with
stoppage and incompletion when its owner, FLADC which was owned by the
Tius, encountered dire financial difficulties. It was heavily indebted to the
PNB for P190 million. To stave off foreclosure of the mortgage on the two lots

Page 604 of 1072

where the mall was being built, the Tius invited Ong Yong, Juanita Tan Ong,
Wilson T. Ong, Anna L. Ong, William T. Ong and Julia Ong Alonzo (the Ongs),
to invest in FLADC. Under the Pre-Subscription Agreement, the Ongs and the
Tius agreed to maintain equal shareholdings in FLADC: the Ongs were to
subscribe to 1,000,000 shares at a par value of P100.00 each while the Tius
were to subscribe to an additional 549,800 shares at P100.00 each in
addition to their already existing subscription of 450,200 shares.
Accordingly, the Ongs paid P100M in cash for their subscription to 1M
shares of stock while the Tius committed to contribute to FLADC a four-storey
building and two parcels of land respectively valued at P20M for 200K shares,
P30M for 300K shares and P49.8M or 49,800 shares to cover their additional
549,800 stock subscription therein. The Ongs paid in another P70M to FLADC
and P20M to the Tius over and above their P100M investment, the total sum
of which P190M was used to settle the P190M mortgage indebtedness of
FLADC to PNB.
The Tius, on February 23, 1996, rescinded the Pre-Subscription
Agreement. The Tius accused the Ongs of (1) refusing to credit to them the
FLADC shares covering their real property contributions; (2) preventing David
S. Tiu and Cely Y. Tiu from assuming the positions of and performing their
duties as Vice-President and Treasurer, respectively, and (3) refusing to give
them the office spaces agreed upon.
The business harmony between the parties were short-lived when the
Tius rescinded unilaterally the Pre-subcription agreement accussing the
Ongs of refusing to credit to them the FLADC shares covering their real
property contributions and preventing the Tius from assuming the positions
of and performing their duties and refusing to give them office spaces agreed
upon.
ISSUE/S:
Whether or not the Tius are justified in rescinding the presubscription agreement.
RULING:
A subscription contract necessarily involves the corporation as one of
the contracting parties since the subject matter of the transaction is a
property owned by the corporation, its shares and stocks. Considering
therefore that the real contracting parties to the subscription agreement
were FLADC and the Ongs alone, the action of rescission by the Tius in their
personal capacity will not prosper.
In the instant case, the rescission of the Pre-subscription Agreement
will effectively result in the unauthorized distribution of the capital assets

Page 605 of 1072

and property of the corporation, thereby violating the Trust Fund Doctrine
and the Corporation Code, since rescission of a subscription agreement is not
one of the instances when distribution of capital assets and property of the
corporation is allowed.
Furthermore, it is an improper judicial intrusion into the internal affairs
of the corporation to compel FLADC to file at the SEC a petition for the
issuance of a certificate of decrease of stock. Decreasing a corporations
authorized capital stock is an amendment of the Articles of Incorporation. It
is a decision only the stockholders and the Directors can make, considering
they are the contracting parties.

TOPIC:WHAT IS SUBSCRIPTION
SOFRONIO T. BAYLA, ET AL.
VS.
SILANG TRAFFIC CO., INC.
SILANG TRAFFIC CO. V. SOFRONIO BAYLA, ET AL.
G.R. Nos. L-48195 and 48196, May 1, 1942
73 Phil 557
FACTS:

Page 606 of 1072

Petitioners instituted this action against Silang Traffic Co., Inc. to


recover sums of money which they had paid severally to the corporation on
account of shares of stock they individually agreed to take and pay for under
certain specified terms and conditions, wherein subscriber promised to pay
at the Municipality of Silang, Province of Cavite, Philippine Islands, P1,500 as
purchase price of 15 shares of capital stock to be paid as follows, to wit: 5%
upon the execution of the contract and the remainder in installments of 5%
payable within the first month of each and every quarter thereafter,
commencing on the 1st day of July, 1935 and the subscriber agreed that if he
fails to pay any of said installment when due, or to perform any of the
aforesaid conditions, or if said shares shall be attached or levied upon by
creditors of the said subscriber, then the said shares are to revert to the
seller and the payments already made are to be forfeited in favor of said
seller, and the latter may then take possession, without resorting to court
proceedings.
The respondent corporation set up the following defenses:
(1)That the above-quoted resolution is not applicable to
the petitioners Sofronio T. Bayla, Josefa Naval, and Paz
Toledo because on the date thereof "their subscribed
shares of stock had already automatically reverted to
the defendant, and the installments paid by them had
already been forfeited"; and
(2)that said resolution of August 1, 1937, was revoked and
cancelled by a subsequent resolution of the board of
directors of the defendant corporation dated August 22,
1937.
The trial court absolved the defendant from the complaint and
declared canceled (forfeited) in favor of the defendant the shares of stock in
question. It held that the resolution of August 1, 1937, was null and void.
ISSUE/S:
What is a subscription contract and is the contract entered into
by the parties a subscription contract
RULING:
Whether a particular contract is a subscription or a sale of stock is a
matter of construction and depends upon its terms and the intention of the
parties. In the Unson case the Court held that a subscription to stock in an
existing corporation is, as between the subscriber and the corporation,
simply a contract of purchase and sale.

Page 607 of 1072

It seems clear from the terms of the contracts in question that they are
contracts of sale and not of subscription. The lower courts erred in
overlooking the distinction between subscription and purchase "A
subscription, properly speaking, is the mutual agreement of the subscribers
to take and pay for the stock of a corporation, while a purchase is an
independent agreement between the individual and the corporation to buy
shares of stock from it at stipulated price." In some particulars the rules
governing subscriptions and sales of shares are different. For instance, the
provisions of our Corporation Law regarding calls for unpaid subscription and
assessment of stock (sections 37-50) do not apply to a purchase of stock.
Likewise the rule that corporation has no legal capacity to release an original
subscriber to its capital stock from the obligation to pay for his shares, is
inapplicable to a contract of purchase of shares.

TOPIC:WHAT IS SUBSCRIPTION
SALMON, DEXTER & CO.
VS.
TIMOTEO UNSON
G.R. No. L-23608, March 17, 1925
47 PHIL 649
FACTS:

Page 608 of 1072

Salmon, Dexter and Company, a domestic corporation, was organized


under the name of C.S. Salmon and Company, with a capital stock of P250,
000. Pursuant to a resolution of the board of directors of the corporation, a
meeting of the stockholders was held at which the capital stock of C.S.
Salmon and Company was increased to P500, 000. The certificate of increase
of capital stock from P250, 000 to P500, 000, and articles of incorporation, as
amended, of Salmon, Dexter and Company were filed with the Mercantile
Registry of the Bureau of Commerce and Industry.
Timoteo Unson became a subscriber of C.S. Salmon and Company, by
signing an agreement in writing and delivering the same to C.S. Salmon and
Company, the name of which company was later changed to Salmon, Dexter
and Company. The agreement as accomplished by Timoteo Unson on July
28, 1920, was for ten shares of the capital stock of C.S. Salmon and
Company, "authorized capital P250, 000, It must be noted that two weeks
before, on July 14, 1920, the stockholders of C.S. Salmon and Company,
without the acquiescence or participation of Unson, had authorized an
increase of the capital stock of the corporation to P500, 000.
ISSUE/S:
Is the contract of Unson a contract of subscription to the capital stock
of C.S. Salmon and Company, or is it a contract to purchase stock in
the corporation?
Whether one or the other, is Unson released from his obligation on the
subscription agreement on account of the increase of the capital stock
of C.S. Salmon and Company from P250, 000 to P500, 000?
RULING:
After incorporation, one may become a shareholder by subscription, or
by purchasing stock directly from the corporation, or from individual owners
thereof. A distinction is drawn by the authorities between a subscription to
the capital stock of the corporation after its organization and a sale of shares
by it. Whether a particular contract is a subscription or a sale of stock is a
matter of construction, and depends upon its terms and the intention of the
parties. It has been held that a subscription to stock in an existing
corporation is, as between the subscriber and the corporation, simply a
contract of purchase and sale. (Bole vs. Fulton [1912], 233 Pa., 609; 2
Fletcher, Cyclopedia of Corporations, pp. 1120 et seq.)
The allegation of the complaint is that defendant is a "subscriber."
Exhibit A, on its face, purports to be a "subscription for capital stock." The
intention of the parties as gleaned from this contract was undoubtedly to
consider it as such. Admitting, however, that the terminology of the
agreement is not conclusive, and admitting that it is a contract between a
subscriber and the corporation, and thus simply a contract of purchase and

Page 609 of 1072

sale, then under the last hypothesis we have to determine if the contract is
avoided by misrepresentation.
Plaintiff's right of recovery rests exclusively upon the written
agreement. The promise of Unson in this agreement was to subscribe for ten
shares of the capital stock, authorized capital P250, 000, of C.S. Salmon and
Company. One of the essential conditions of this subscription or contract of
sale was that the authorized capital stock of the company was P250, 000. As
far as we are informed, Unson would have never have put his name to the
agreement if he had known that two weeks before, the capital had been
increased to P500, 000. If knowledge of this increase had been brought home
to Unson before he signed, that would be a different question. But the record
is silent on this point. So should the contract be enforced? Unson would be
required to take and pay for a 1/500 part of the capital stock of Salmon,
Dexter and Company, whereas his obligation was to take and pay for a 1/250
part of the capital stock. Paraphrasing the United States Supreme Court in
the case of Chicago City Railway Company vs. Allerton ( [1874], 18 Wall.,
233), a change in the capital stock without the consent of the stockholder
would make him a member of an association in which he never consented to
become such. "It would change the relative influence, control and profit of
each member."

TOPIC:WHAT IS SUBSCRIPTION
SUNSET VIEW CONDOMINIUM CORPORATION
VS.
THE HON. JOSE C. CAMPOS, JR. OF THE COURT OF FIRST INSTANCE,
BRANCH XXX, PASAY CITY and AGUILAR-BERNARES REALTY
G.R. No. L-52361, April 27, 1981
FACTS:

Page 610 of 1072

Sunset View Condominium Corporation is a condominium corporation


within the meaning of Republic Act No. 4726 in relation to a duly registered
Amended Master Deed with Declaration of Restrictions of the Sunset View
Condominium Project of which petitioner is the Management Body holding
title to all the common and limited common areas.
Aguilar-Bernares Realty is the assignee of a unit, "Solana", in the
Sunset View Condominium Project with La Perla Commercial, Incorporated, as
assignor. The La Perla Commercial, Incorporated bought the "Solana" unit on
installment from the Tower Builders, Inc. Sunset View Condominium
Corporation, filed for the collection of assessments levied on the unit against
Aguilar-Bernares Realty. Petitioner filed its amended complaint for the
collection of overdue accounts on assessments and insurance premiums and
the interest thereon against the private respondent Lim Siu Leng to whom
was assigned a unit called "Alegria" of the Sunset View Condominium Project
by Alfonso Uy who had entered into a "Contract to Buy and Sell" with Tower
Builders, Inc. over the said unit on installment basis.
Private respondent filed a motion to dismiss on the ground of lack of
jurisdiction, alleging that the amount sought to be collected is an
assessment. The correctness and validity of which is certain to involve a
dispute between her and the petitioner corporation; that she has
automatically become, as a purchaser of the condominium unit, a
stockholder of the petitioner pursuant to Section 2 of the Condominium Act,
Republic Act No. 4726; that the dispute is intra-corporate and is
consequently under the exclusive jurisdiction of the Securities & Exchange
Commission as provided in Section 5 of P.D. No. 902-A. The petitioner filed
its opposition thereto, alleging that the private respondent who had not fully
paid for the unit was not the owner thereof, consequently was not the holder
of a separate interest which would make her a stockholder, and that hence
the case was not an intra-corporate dispute.
ISSUE/S:
Is a purchaser of a condominium unit in the condominium project
managed by the petitioner, who has not yet fully paid the purchase
price thereof, automatically a, stockholder of the petitioner
Condominium Corporation
RULING:
That only the owner of a unit is a stockholder of the Condominium
Corporation is inferred from Section 10 of the Condominium Act which reads:
SEC. 10. ... Membership in a condominium corporation, regardless of whether
it is a stock or non-stock corporation, shall not be transferable separately
from the condominium unit of which it is an appurtenance When a member

Page 611 of 1072

or stockholder ceases is to own a unit in the project in which the


condominium corporation owns or holds the common areas, he shall
automatically cease to be a member or stockholder of the condominium
corporation.
Pursuant to the above statutory provision, ownership of a unit is a
condition sine qua non to being a shareholder in the condominium
corporation. It follows that a purchaser of a unit who is not yet the owner
thereof for not having fully paid the full purchase price, is not a shareholder
By necessary implication, the "separate interest" in a condominium, which
entitles the holder to become automatically a share holder in the
condominium corporation, as provided in Section 2 of the Condominium Act,
can be no other than ownership of a unit. This is so because nobody can be a
shareholder unless he is the owner of a unit and when he ceases to be the
owner, he also ceases automatically to be a shareholder.
The private respondents, therefore, who have not fully paid the
purchase price of their units and are consequently not owners of their units
are not members or shareholders of the petitioner condominium corporation,

TOPIC:WHAT IS SUBSCRIPTION
MIGUEL VELASCO, assignee of The Philippine Chemical Product Co.
(Ltd.), plaintiff-appellant,
vs.
JEAN M. POIZAT, defendant-appellee.
G.R. No. L-11528. March 15, 1918
37 PHIL 802

Page 612 of 1072

FACTS:
Plaintiff, as assignee in insolvency of "The Philippine Chemical Product
Company" (Ltd.) is seeking to recover of the defendant, Jean M. Poizat, the
sum of P1,500, upon a subscription made by him to the corporate stock of
said company. The defendant subscribed for 20 shares of the stock of the
company, and paid in upon his subscription the sum of P500, the par value of
5 shares. The action was brought to recover the amount subscribed upon the
remaining shares.
It appears that the defendant was a stock holder in the company from
the inception of the enterprise, and for sometime acted as its treasurer and
manager. While serving in this capacity he called in and collected all
subscriptions to the capital stock of the company, except the aforesaid 15
shares subscribed by himself and another 15 shares owned by Jose R.
Infante.
Upon July 13, 1914, a meeting of the board of directors of the company
was held at which two resolutions. The first was a proposal that the directors,
or shareholders, of the company should make good by new subscriptions, in
proportion to their respective holdings, 15 shares which had been
surrendered by Infante. It seems that this shareholder had already paid 25
per cent of his subscription upon 20 shares, leaving 15 shares unpaid for,
and an understanding had been reached by him and the management by
which he was to be released from the obligation of his subscription, it being
understood that what he had already paid should not be refunded.
Accordingly the directors present at this meeting subscribed P1, 200 toward
taking up his shares, leaving a deficiency of P300 to be recovered by
voluntary subscriptions from stockholders not present at the meeting.
The other proposition was to the effect that Juan [Jean] M. Poizat, who
was absent, should be required to pay the amount of his subscription upon
the 15 shares for which he was still indebted to the company. The resolution
further provided that, in case he should refuse to make such payment, the
management of the corporation should be authorized to undertake judicial
proceedings against him.
As the company soon went into voluntary insolvency, Velasco was
named as the assignee. The Court of First Instance rendered judgment in
favor of the defendant, and the complaint was dismissed. From this action
the plaintiff has appealed.
ISSUE/S:
Whether or not Poizat is liable upon his subscription.

Page 613 of 1072

RULING:
It evidently cannot be permitted that a subscriber should escape from
his lawful obligation by reason of the failure of the officers of the corporation
to perform their duty in making a call; and when the original model of
making the call becomes impracticable, the obligation must be treated as
due upon demand. If the corporation must be treated still an active entity
and this action should be dismissed for irregularity in the making of the call,
other steps could be taken by the board to cure the defect and another
action could be brought; but where the company is being wound up, no such
procedure would be practicable. The better doctrine is that when insolvency
supervenes all unpaid subscriptions become at once due and enforceable.
The circumstance that the board of directors in their meeting of July
13, 1914, resolved to release Infante from his obligation upon a subscription
for 15 shares is no wise prejudicial to the right of the corporation or its
assignee to recover from Poizat upon a subscription made by him. In
releasing Infante the board transcended its powers, and he no doubt still
remained liable on such of his shares as were not taken up and paid for by
other persons.

TOPIC: ACQUISITION AND OWNERSHIP OF SHARES IN A CORPORATION;


EXTENT OF
PROPRIETARY RIGHT
REPUBLIC OF THE PHILIPPINES
VS.
SANDIGANBAYAN (FIRST DIVISION) EDUARDO M. COJUANGCO, JR.
G.R. No. 166859, April 12, 2011

Page 614 of 1072

FACTS:
As a consequence of the CIIF block of San Miguel Corporation (smc)
shares of stock totalling 33,133,266 shares as of 1983 together with all
dividends declared, paid and issued thereon as well as any increments
thereto arising from, but not limited to, exercise of pre-emptive rights being
declared to be owned by the government in trust for all the coconut farmers
and ordered reconveyed to the government, the Republic filed a motion for
execution. However, during the pendency of the Republics motion for
execution, Cojuangco, et al. filed a Motion for Authority to Sell San Miguel
Corporation (SMC) shares, praying for leave to allow the sale of SMC shares
to proceed, exempted from the conditions set forth in the resolutions
promulgated on October 3, 2003 and June 24, 2005. The Republic opposed,
contending that the requested leave to sell would be tantamount to
removing jurisdiction over the res or the subject of litigation.
However, the Sandiganbayan eventually granted the Motion for
Authority to Sell San Miguel Corporation (SMC) shares.
Thereafter, Cojuangco, et al. manifested to the Sandiganbayan that the
shares would be sold to the San Miguel Corporation Retirement Plan.
ISSUE/S:
Whether or not the subject shares in SMC, which were acquired
by, and are in the respective names of respondents Cojuangco,
Jr. And the Cojuangco companies, should be reconveyed to the
republic of the Philippines for having been acquired using
coconut levy fund.
RULING:
Referring to plaintiffs causes of action against defendants Cojuangco,
et al., the Court finds its evidence insufficient to prove that the source of
funds used to purchase SMC shares indeed came from coconut levy funds. In
fact, there is no direct link that the loans obtained by defendant Cojuangco,
Jr. were the same money used to pay for the SMC shares. The scheme
alleged to have been taken by defendant Cojuangco, Jr. was not even
established by any paper trail or testimonial evidence that would have
identified the same. On account of his positions in the UCPB, PCA and the
CIIF Oil Mills, the Court cannot conclude that he violated the fiduciary
obligations of the positions he held in the absence of proof that he was so
actuated and that he abused his positions.
It was plain, indeed, that Cojuangco, et al. had tendered genuine issues
through their responsive pleadings and did not admit that the acquisition of
the Cojuangco block of SMC shares had been illegal, or had been made with
public funds. As a result, the Republic needed to establish its allegations with
preponderant competent evidence, because, as earlier stated, the fact that

Page 615 of 1072

property was ill gotten could not be presumed but must be substantiated
with competent proof adduced in proper judicial proceedings. That the
Republic opted not to adduce competent evidence thereon despite stern
reminders and warnings from the Sandiganbayan to do so revealed that the
Republic did not have the competent evidence to prove its allegations
against Cojuangco, et al.
Still, the Republic, relying on the 2001 holding in Republic v. COCOFED,
pleads in its petition for review (G.R. No. 180702) that:
With all due respect, the Honorable Sandiganbayan failed to
consider legal precepts and procedural principles vis--vis the
records of the case showing that the funds or "various loans" or
"advances" used in the acquisition of the disputed SMC Shares
ultimately came from the coconut levy funds.
As discussed hereunder, respondents own admissions in their Answers
and Pre-Trial Briefs confirm that the "various sources" of funds utilized in the
acquisition of the disputed SMC shares came from "borrowings" and
"advances" from the UCPB and the CIIF Oil Mills.
Thereby, the Republic would have the Sandiganbayan pronounce the
block of SMC shares of stock acquired by Cojuangco, et al. as ill-gotten
wealth even without the Republic first presenting preponderant evidence
establishing that such block had been acquired illegally and with the use of
coconut levy funds.
The Court cannot heed the Republics pleas for the following reasons:
To begin with, it is notable that the decision of November 28,
2007 did not rule on whether coconut levy funds were public
funds or not. The silence of the Sandiganbayan on the matter
was probably due to its not seeing the need for such ruling
following its conclusion that the Republic had not preponderantly
established the source of the funds used to pay the purchase
price of the concerned SMC shares, and whether the shares had
been acquired with the use of coconut levy funds.

Page 616 of 1072

Secondly, the ruling in Republic v. COCOFED determined only whether


certain stockholders of the UCPB could vote in the stockholders meeting
that had been called. The issue now before the Court could not be controlled
by the ruling in Republic v. COCOFED, however, for even as that ruling
determined the issue of voting, the Court was forthright enough about not
thereby pre-empting the Sandiganbayans decisions on the merits on illgotten wealth in the several cases then pending, including this one, TOPIC:
ACQUISITION AND OWNERSHIP OF SHARES IN A CORPORATION; EXTENT OF
PROPRIETARY RIGHT
SIXTO P. CRISOSTOMO
VS.
SECURITIES AND EXCHANGE COMMISSION, SPOUSES SHOJI YAMADA
and MICHIYO YAMADA and SPOUSES TOMOTADA ENATSU and EDITA
ENATSU
G.R. Nos. 89095 & 89555, November 6, 1989
179 SCRA 146
FACTS:
Sixto Crisostomo, et.al.,known as the Crisostomo group were the
original stockholders of the United Doctors Medical Center which was
organized in 1968 with an authorized capital stock of P1,000,000 which was
later increased to P15,000,000 in 1972. They owned approximately 40% of
UDMC's outstanding capital stock, while the 60% majority belonged to the
members of the United Medical Staff Association, composed of doctors and
medical personnel of UDMC. Herein petitioner is the director and legal
counsel of UDMC.
In 1988, UDMC defaulted in paying its loan obligation of approximately
P55 million to the DBP and is now facing foreclosure by the Asset
Privatization Trust which has taken over UDMCs loan obligation to DBP.
To stave off the threatened foreclosure, UDMC, through its principal
officers, Ricardo Alfonso and Juanito Crisostomo persuaded Japanese doctors
Yamadas and Enatsu to invest fresh capital in UDMC. The wife of Tomotada
Enatsu, Edita Enatsu, is a Filipina. They invested approximately P57 million in
UDMC which was effected by means of a Stock Purchase Agreement and an
Amended Memorandum of Agreement whereby the group subscribed to
82.09% of the outstanding shares of UDMC.
Both transactions were duly authorized by the board of directors and
stockholders of UDMC. They were submitted to, scrutinized by, and, finally,
approved by the Board of Investments, the Central Bank of the Philippines,
and the Securities and Exchange Commission. The elaborate governmental
approval process was done openly and with full knowledge of all concerned,

Page 617 of 1072

including Sixto Crisostomo, the corporate legal counsel. Upon the completion
of the governmental approval process, shares of stock, duly signed by
UDMC's authorized officers, were issued to the Yamadas and Enatsus.
However, on the eve of the meetings, i.e., on August 19, 1988, Sixto
Crisostomo, supposedly acting for himself, filed SEC Case No. 3420 against
Juanito Crisostomo, Ricardo Alfonso, Shoji Yamada, Michiyo Yamada,
Tomotada Enatsu and Edita Enatsu, praying, among other things, (1) to stop
the holding of the stockholder's and board of directors' meetings; (2) to
disqualify the Japanese investors from holding a controlling interest in UDMC
and from being elected directors or officers of UDMC; and (3) to annul the
Memorandum of Agreement and Stock Purchase Agreement because they
allegedly did not express the true agreement of the parties .
ISSUE/S:
Whether or not the issuance of shares of stock to the Japanese
group violates the constitution on ownership of corporations

RULING:
The P57 million investment of the Japanese group in UDMC violates the
constitutional provisions restricting the transfer or conveyance of private
lands (Art. XIII, Sec. 7, 1987 Constitution) and the ownership of educational
institutions (Art. XVI, Sec. 14[a], 1987 Constitution), to citizens of the
Philippines or corporations at least 60% of the capital of which is owned by
Filipino citizens. While 82% of UDMC's capital stock is indeed subscribed by
the Japanese group, only 30% (equivalent to 171,721 shares or P17, 172.00)
is owned by the Japanese citizens, namely, the Yamada spouses and
Tomotada Enatsu. 52% is owned by Edita Enatsu, who is a Filipino.
Accordingly, in its application for approval/registration of the foreign equity
investments of these investors, UDMC declared that 70% of its capital stock
is owned by Filipino citizens, including Edita Enatsu. That application was
approved by the Central Bank on August 3, 1988.
The investments in UDMC of Doctors Yamada and Enatsu do not violate
the Constitutional prohibition against foreigners practicing a profession in the
Philippines (Section 14, Article XII, 1987 Constitution) for they do not practice
their profession (medicine) in the Philippines, neither have they applied for a
license to do so. They only own shares of stock in a corporation that operates
a hospital. No law limits the sale of hospital shares of stock to doctors only.
The ownership of such shares does not amount to engaging (illegally,) in the
practice of medicine, or, nursing. If it were otherwise, the petitioner's
stockholding in UDMC would also be illegal.

Page 618 of 1072

TOPIC: ACQUISITION AND OWNERSHIP OF SHARES IN A CORPORATION;


EXTENT OF
PROPRIETARY RIGHT
TIRSO GARCIA, IN HIS CAPACITY AS RECEIVER OF THE MERCANTILE
BANK OF CHINA,
VS.
LIM CHU SING
G.R. NO. L-39427, FEBRUARY 24, 1934
FACTS:
On June 20, 1930, Lim Chu Sing executed and delivered to the
Mercantile Bank of China promissory note for the sum of P19, 605.17 with
interest thereon at 6 per cent per annum, payable in monthly installments
until the amount of the promissory note together with the interest thereon is
fully paid. One of the conditions stipulated in said promissory note is that in
case of defendant's default in the payment of any of the monthly
installments, as they become due, the entire amount or the unpaid balance
thereof together with interest thereon at 6 per cent per annum, shall become
due and payable on demand. The defendant had been, making several
partial payments thereon, leaving an unpaid balance of P9,105.17. However,
he defaulted in the payment of several installments by reason of which the
unpaid balance of P9,105.17 on the promissory note has ipso facto become
due and demandable.
The debt which is the subject matter of the complaint was not really an
indebtedness of the defendant but of Lim Cuan Sy, who had an account with
the plaintiff bank in the form of "trust receipts" guaranteed by the defendant
as surety and with chattel mortgage securities. The plaintiff bank, without
the knowledge and consent of the defendant, foreclosed the chattel
mortgage and privately sold the property covered thereby. Inasmuch as Lim
Cuan Sy failed to comply with his obligations, the plaintiff required the
defendant, as surety, to sign a promissory note for the sum of P19,105.17
payable in the manner hereinbefore stated. The defendant had been paying
the corresponding installments until the debt was reduced to the sum of
P9,105.17 claimed in the complaint. The defendant is the owner of shares of
stock of the plaintiff Mercantile Bank of China amounting to P10,000. The
plaintiff bank is now under liquidation.
ISSUE/S:
Whether or not it is proper to compensate the defendantappellant's indebtedness of P9, 105.17, which is claimed in the

Page 619 of 1072

complaint, with the sum of P10, 000 representing the value of his
shares of stock with the Mercantile Bank of China.
RULING:
According to the weight of authority, a share of stock or the certificate
thereof is not an indebtedness to the owner nor evidence of indebtedness
and, therefore, it is not a credit Stockholders, as such, are not creditors of
the corporation. It is the prevailing doctrine of the American courts,
repeatedly asserted in the broadest terms, that the capital stock of a
corporation is a trust fund to be used more particularly for the security of
creditors of the corporation, who presumably deal with it on the credit of its
capital stock. Therefore, the defendant-appellant Lim Chu Sing not being a
creditor of the Mercantile Bank of China, although the latter is a creditor of
the former, there is no sufficient ground to justify compensation.
In view of the foregoing, this court is of the opinion and so holds:
1. That failure to file an exception to a ruling rendered in open
court denying a motion for the inclusion of a party as
defendant deprives the petitioner, upon appeal of the right
to raise the question whether such denial proper or
improper;
2. that the shares of a banking corporation do not constitute
an indebtedness of the corporation to the stockholder and,
therefore, the latter is not a creditor of the former for such
shares;
3. That the indebtedness of a shareholder to a banking
corporation cannot be compensated with the amount of his
shares therein, there being no relation of creditor and
debtor with respect to such shares; and (4) that the
percentage stipulated in a contract, for costs and
attorney's fees for the collection of indebtedness, includes
judicial costs.

Page 620 of 1072

TOPIC:ACQUISITION AND OWNERSHIP OF SHARES IN A CORPORATION;


EXTENT OF
PROPRIETARY RIGHT
CONCEPCION MAGSAYSAY-LABRADOR, ET.AL.,
VS.
THE COURT OF APPEALS and ADELAIDA RODRIGUEZ-MAGSAYSAY,
Special Administratrix of the Estate of the late Genaro F. Magsaysay
G.R. No. 58168 December 19, 1989
180 SCRA 266
FACTS:
On February 9, 1979, Adelaida Rodriguez-Magsaysay, widow and
special administratix of the estate of the late Senator Genaro Magsaysay,
brought before the then Court of First Instance of Olongapo an action against
Artemio
Panganiban,
Subic
Land
Corporation
(SUBIC),
Filipinas
Manufacturer's Bank (FILMANBANK) and the Register of Deeds of Zambales
concerning Pequena Island which she claims was bought by conjugal funds
and that the same was unlawfully assigned by her late husband to Subic
Land.
On March 7, 1979, herein petitioners, sisters of the late senator, filed a
motion for intervention on the ground that on June 20, 1978, their brother
conveyed to them one-half (1/2 ) of his shareholdings in SUBIC or a total of
416,566.6 shares and as assignees of around 41 % of the total outstanding
shares of such stocks of SUBIC, they have a substantial and legal interest in
the subject matter of litigation and that they have a legal interest in the
success of the suit with respect to SUBIC. Invoking the principle enunciated
in the case of PNB v. Phil. Veg. Oil Co., 49 Phil. 857,862 & 853 (1927),
petitioners strongly argue that their ownership of 41.66% of the entire
outstanding capital stock of SUBIC entitles them to a significant vote in the
corporate affairs; that they are affected by the action of the widow of their
late brother for it concerns the only tangible asset of the corporation and
that it appears that they are more vitally interested in the outcome of the
case than SUBIC.
On July 26, 1979, the court denied the motion for intervention, and
ruled that petitioners have no legal interest whatsoever in the matter in
litigation and their being alleged assignees or transferees of certain shares in
SUBIC cannot legally entitle them to intervene because SUBIC has a
personality separate and distinct from its stockholders.
ISSUE/S:
Whether or not the court erred in denying the motion for
intervention filed by the petitioners

Page 621 of 1072

RULING:
To allow intervention,
a) it must be shown that the movant has legal interest in the matter
in litigation, or otherwise qualified; and
b) consideration must be given as to whether the adjudication of
the rights of the original parties may be delayed or prejudiced, or
whether the intervenor's rights may be protected in a separate
proceeding or not.
Both requirements must concur as the first is not more important
than the second.
The interest which entitles a person to intervene in a suit between
other parties must be in the matter in litigation and of such direct and
immediate character that the intervenor will either gain or lose by the direct
legal operation and effect of the judgment. Otherwise, if persons not parties
of the action could be allowed to intervene, proceedings will become
unnecessarily complicated, expensive and interminable. And this is not the
policy of the law.
The words "an interest in the subject" mean a direct interest in the
cause of action as pleaded, and which would put the intervenor in a legal
position to litigate a fact alleged in the complaint, without the establishment
of which plaintiff could not recover.
Here, the interest, if it exists at all, of petitioners-movants is indirect,
contingent, remote, conjectural, consequential and collateral. At the very
least, their interest is purely inchoate, or in sheer expectancy of a right in the
management of the corporation and to share in the profits thereof and in the
properties and assets thereof on dissolution, after payment of the corporate
debts and obligations.
While a share of stock represents a proportionate or aliquot interest in
the property of the corporation, it does not vest the owner thereof with any
legal right or title to any of the property, his interest in the corporate
property being equitable or beneficial in nature. Shareholders are in no legal
sense the owners of corporate property, which is owned by the corporation
as a distinct legal person.

Page 622 of 1072

TOPIC:ACQUISITION AND OWNERSHIP OF SHARES IN A CORPORATION;


EXTENT OF
PROPRIETARY RIGHT
ROY NICOLAS
VS.
THE HONORABLE COURT OF APPEALS (Sixth Division) and BLESILO
F.B. BUAN
G.R. NO. 122857. MARCH 27, 1998
FACTS:
On February 19, 1987, petitioner Roy Nicolas and private respondent
Blesito Buan entered into a Portfolio Management Agreement, wherein the
former was to manage the stock transactions of the latter for a period of
three months with an automatic renewal clause. Under the Portfolio
Management Agreement, it was agreed that private respondent would pay
the petitioner 20% of all realized profits every end of the month as his
management fees. However, upon the initiative of the private respondent
the agreement was terminated on August 19, 1987, and thereafter he
requested for an accounting of all transactions made by the petitioner.
Three weeks after the termination of the agreement, petitioner
demanded from the private respondent the amount of P68, 263.67
representing his alleged management fees covering the periods of June 30,
July 31 and August 19, 1987 as provided for in the Portfolio Management
Agreement. But the demands went unheeded, much to the chagrin of the
petitioner.
Rebuffed, petitioner filed a complaint for collection of sum of money
against the private respondent before the trial court. In his answer, private
respondent contended that petitioner mismanaged his transactions resulting
in losses, thus, he was not entitled to any management fees.
After hearing, the trial court rendered its decision in favor of Nicolas.
When Buan appealed the decision to the Court of Appeals, the latter finding
merit in his case, reversed the RTC decision.
ISSUE/S:
Whether or not Nicolas is entitled to his alleged management
fees
RULING:
Evidently, the key word in the provision is profits. Simply put, profit
has been defined as the excess of return over expenditure in a transaction

Page 623 of 1072

or series of transactions or the series of an amount received over the


amount paid for goods and services.
Admittedly, like any services rendered or performed stock brokers are
entitled to commercial fees or compensation pursuant to the Revised
Securities Act Rule 19-13, which reads:
RSA Rule 19-13. Charges for Services Performed.
Charges by brokers or dealers, if any, for service performed,
including miscellaneous services such as collection of monies
due for principal, dividends, interests, exchange or transfer of
securities, appeals, safekeeping or custody of securities, and
other services, shall be reasonable and not unfairly
discriminatory between customers.
Moreover, the same law provides that any fee or commission must be
with due regard to relevant circumstances.
Unfortunately, the profit and loss statements presented by the
petitioner are nothing but bare assertions, devoid of any concrete basis or
specifics as to the method of arriving at the amounts indicated in the
documents. In fact, it did not even state when the stocks were purchased,
the type of stocks (whether Class A or B or common or preferred) bought,
when the stocks were sold, the acquisition and selling price of each stock,
when the profits, if any, were delivered to the private respondent, the cost of
safekeeping or custody of the stocks, as well as the taxes paid for each
transaction. With respect to the alleged losses, it has been held that where a
profit or loss statement shows a loss, the statement must show income and
items of expense to explain the method of determining such loss. However,
in the instant petition, petitioner hardly elucidated the reasons and the
factors behind the losses incurred in the course of the transactions.
In short, no evidentiary value can be attributed to the profit and loss
statements submitted by the petitioner. These documents can hardly be
considered a credible or true reflection of the transactions.
It is an
incomplete record yielding easily to the inclusion or deletion of certain
matters. The contents are subject to suspicion since they are not reflective
of all pertinent and relevant data. Thus, even assuming the admissibility of
these alleged profit and loss statements, they are devoid of any evidentiary
weight, for the amounts are conclusions without premises, its bases left to
speculation, conjectures, assertions and guesswork.
The futility of petitioner's action became more pronounced by the fact
that he traded securities for the account of others without the
necessarylicense from the Securities and Exchange Commission (SEC).
Clearly, such omission was in violation of Section 19 of the Revised Securities
Act which provides that no broker shall sell any securities unless he is

Page 624 of 1072

registered with the SEC. The purpose of the statute requiring the registration
of brokers selling securities and the filing of data regarding securities which
they propose to sell, is to protect the public and strengthen the securities
mechanism.
American jurisprudence emphasizes the principle that:
x x x, an unlicensed person may not recover compensation for
services as a broker where a statute or ordinance requiring a
license is applicable and such statute or ordinance is of a
regulatory nature, was enacted in the exercise of the police
power for the purpose of protecting the public, requires a license
as evidence of qualification and fitness, and expressly precludes
an unlicensed person from recovering compensation by suit, or
at least manifests an intent to prohibit and render unlawful the
transaction of business by an unlicensed person.
The Supreme Court sees no reason not to apply the same rule in our
jurisdiction.
Stock market trading, a technical and highly specialized
institution in the Philippines, must be entrusted to individuals with proven
integrity, competence and knowledge, who have due regard to the
requirements of the law.

Page 625 of 1072

TOPIC: ACQUISITION AND OWNERSHIP OF SHARES IN A CORPORATION;


EXTENT OF
PROPRIETARY RIGHT
ALFREDO C. RAMOS
VS.
THE HON. COURT OF APPEALS, LOPEZ, LOCSIN, LEDESMA & CO., INC.
and CMS STOCK BROKERAGE INC.
G.R. No. L-41295 December 4, 1989
FACTS:
On August 14 and 26, 1969, CMS Stock Brokerage, Inc. sold to Lopez,
Locsin, Ledesma & Co., Inc. of the Makati Stock Exchange 2,650 shares of
Benguet Consolidated Corporation for P297,650 on a delayed delivery basis
of 10 to 20 days, evidenced by Exchange Contracts Nos. B-11807 and B11814 both dated August 14, 1969 and B-13084 dated August 26, 1969. LLL
bought the shares for the account of its clients, the third-party defendants,
Rene Ledesma, Jose Maria Lopez, Cesar A. Lopez, Jr. and Alfredo Ramos. CMS
failed to deliver the shares of stocks within the agreed period, but LLL did not
demand delivery.
On January 6, 1970, CMS informed LLL that it would deliver the shares
the next day. LLL wrote CMS that it would not accept the shares because its
principals had cancelled their orders. In its reply, CMS insisted that LLL take
delivery of the Benguet shares.
In CMS's Clearing House Report of January 9, 1970, the disposition of
the shares in favor of LLL appeared, but the latter refused to acknowledge
receipt of the covering disposal letter. CMS then deposited the letter in the
Office of the Exchange Executive, Secretary with the notation "Refused
acceptance pending decision of the Exchange".
When the controversy was submitted to the Board of Governors of the
Exchange for determination, the Board issued Resolution No. 523 on August
10, 1970 advising the parties to litigate the matter in court.
Accordingly, CMS filed in the CFI of Rizal a complaint to compel LLL to
accept the Benguet shares, to pay the price of P297,650, as well as P25,000
as attorney's fees and costs.
ISSUE/S:
Whether or not the appellate court erred in refusing to exonerate
the petitioner from personal liability for the orders he placed with
LLL for the account of the Alakor Corporation.
RULING:

Page 626 of 1072

Section 1, Article V of the Rules and Regulations of the Exchange


provides:
SEC. 1. -In the event of a Selling Member failing to make delivery
within a reasonable period of time of shares sold under delayed
delivery contract, it shall be the Buying Member's duty to advise
the Selling Member in writing, giving him one (1) full business
day from the time of receipt of said letter of demand, to make
delivery.
The Buying Member shall obtain a written receipt from the Selling
Member on the duplicate copy of the letter of demand. This receipt must
state the time of delivery of the letter of demand to the Selling Member.
Fifteen (15) days shall be considered a reasonable period of time within
which to effect delivery unless otherwise stated in the sales contract.
In the event a Selling Member is unable to make delivery within said
period, the Buying Member shall deliver a copy of his letter of demand to the
Chairman of the Floor Trading & Arbitration Committee who may purchase
the shares for the Selling Member's account.
Paragraph 5, Section 1, Article XI of the same Rules provides:
... Delayed Delivery TransactionsOn the agreed day of delivery, notice in writing shall be
given by the selling broker to the buying broker, said notice
to reach the buying broker not later than 4:00 p.m. ...
Section 1, Article V of the Exchange Rules and Regulations covers not
only contracts without a fixed period, but also those contracts where a period
for delayed delivery is stipulated.
In the case at bar, the stock purchases of LLL were on a 10-20 day
delayed delivery basis. Accordingly, after that period lapsed, the Buying
Member (LLL) should have advised the Selling Member CMS in writing, giving
CMS one (1) full business day from receipt of said letter of demand to
comply. Since the selling member was unable to make delivery within the
stated one (1) full business day from receipt of the demand letter, the buying
member should have delivered a copy of his letter of demand to the
Chairman of the Floor Trading and Arbitration Committee who would have
purchased the shares for the selling member's account.
As observed by the trial court, Section 1, Article V of the Exchange
Rules does not vest on the buyer, respondent LLL, a right to rescind its
contract with CMS upon the latter's default. The Exchange Rules obligate the
buyer to make a demand, and if the selling member fails to deliver the
ordered shares despite the demand, the buyer is further obligated to deliver
a copy of his demand letter to the Chairman of the Floor Trading and

Page 627 of 1072

Arbitration Committee so that the latter may purchase the shares for the
selling member's account. Said rules were held binding on members of the
Exchange (Lopez, Locsin, Ledesma & Co., Inc. vs. Court of Appeals, G.R. No.
L-41291, December 8, 1988). Inasmuch as petitioner placed his order for
Benguet shares through a member of the Exchange (LLL), he is indirectly
bound by the rules of the Exchange.

Page 628 of 1072

TOPIC: ACQUISITION AND OWNERSHIP OF SHARES IN A CORPORATION;


EXTENT OF
PROPRIETARY RIGHT
RUBEN SAW, DIONISIO SAW, LINA S. CHUA, LUCILA S. RUSTE AND
EVELYN SAW
VS.
HON. COURT OF APPEALS, HON. BERNARDO P. PARDO
G.R. No. 90580 April 8, 1991
195 SCRA 740
FACTS:
A collection suit with preliminary attachment was filed by Equitable
Banking Corporation against Freeman, Inc. and Saw Chiao Lian, its President
and General Manager. The petitioners moved to intervene, alleging that
(1)the loan transactions between Saw Chiao Lian and
Equitable Banking Corp. were not approved by the
stockholders representing at least 2/3 of corporate capital;
(2)Saw Chiao Lian had no authority to contract such loans;
and
(3)there was collusion between the officials of Freeman, Inc.
and Equitable Banking Corp. in securing the loans.
The motion to intervene was denied, and the petitioners appealed to
the Court of Appeals.
ISSUE:
Whether or not the Honorable Court of Appeals erred in holding
that the petitioners cannot intervene in Civil Case No. 88-44404
because their rights as stockholders of Freeman are merely
inchoate and not actual, material, direct and immediate prior to
the dissolution of the corporation
RULING:
After examining the issues and arguments of the parties, the Court
finds that the respondent court committed no reversible error in sustaining
the denial by the trial court of the petitioners' motion for intervention.
The words "an interest in the subject" mean a direct interest in the
cause of action as pleaded, and which would put the intervenor in a legal
position to litigate a fact alleged in the complaint, without the establishment
of which plaintiff could not recover.

Page 629 of 1072

Here, the interest, if it exists at all, of petitioners-movants is indirect,


contingent, remote, conjectural, consequential and collateral. At the very
least, their interest is purely inchoate, or in sheer expectancy of a right in the
management of the corporation and to share in the profits thereof and in the
properties and assets thereof on dissolution, after payment of the corporate
debts and obligations.
While a share of stock represents a proportionate or aliquot interest in
the property of the corporation, it does not vest the owner thereof with any
legal right or title to any of the property, his interest in the corporate
property being equitable or beneficial in nature. Shareholders are in no legal
sense the owners of corporate property, which is owned by the corporation
as a distinct legal person.

Page 630 of 1072

TOPIC: CONSIDERATION FOR STOCKS


ERNESTO M. APODACA
vs.
NATIONAL LABOR RELATIONS COMMISSION, JOSE M. MIRASOL and
INTRANS PHILS., INC.,
G.R. No. 80039 April 18, 1989
172 SCRA 442
FACTS:
Petitioner was employed in Respondent Corporation.
On August 28, 1985, respondent Jose M. Mirasol persuaded petitioner
to subscribe to 1,500 shares of Respondent Corporation at P100.00 per share
or a total of P150, 000.00. He made an initial payment of P37, 500.00. On
September 1, 1975, petitioner was appointed President and General Manager
of the respondent corporation. However, on January 2, 1986, he resigned. On
December 19, 1986, petitioner instituted with the NLRC a complaint against
private respondents for the payment of his unpaid wages, his cost of living
allowance, the balance of his gasoline and representation expenses and his
bonus compensation for 1986.
In a decision dated April 28, 1987, the labor arbiter sustained the claim
of petitioner for P17, 060.07 on the ground that the employer has no right to
withhold payment of wages already earned under Article 103 of the Labor
Code. Upon the appeal of the private respondents to public respondent
NLRC, the decision of the labor arbiter was reversed in a decision dated
September 18, 1987. The NLRC held that a stockholder who fails to pay his
unpaid subscription on call becomes a debtor of the corporation and that the
set-off of said obligation against the wages and others due to petitioner is
not contrary to law, morals and public policy.
ISSUE/S:
Whether or not the petitioner is entitled to his money claimed.
RULING:
Firstly, the NLRC has no jurisdiction to determine such intra-corporate
dispute between the stockholder and the corporation as in the matter of
unpaid subscriptions. This controversy is within the exclusive jurisdiction of
the Securities and Exchange Commission.
Secondly, assuming arguendo that the NLRC may exercise jurisdiction
over the said subject matter under the circumstances of this case, the
unpaid subscriptions are not due and payable until a call is made by the
corporation for payment. 2 Private respondents have not presented a
resolution of the board of directors of Respondent Corporation calling for the

Page 631 of 1072

payment of the unpaid subscriptions. It does not even appear that a notice of
such call has been sent to petitioner by the respondent corporation.
What the records show is that the respondent corporation deducted
the amount due to petitioner from the amount receivable from him for the
unpaid subscriptions. 3 No doubt such set-off was without lawful basis, if not
premature. As there was no notice or call for the payment of unpaid
subscriptions, the same is not yet due and payable.
Lastly, assuming further that there was a call for payment of the
unpaid subscription, the NLRC cannot validly set it off against the wages and
other benefits due petitioner. Article 113 of the Labor Code allows such a
deduction from the wages of the employees by the employer, only in three
instances, to wit::
a) In cases where the worker is insured with his consent by the
employer, and the deduction is to recompense the employer for
the amount paid by him as premium on the insurance;
b) For union dues, in cases where the right of the worker or his
union to check off has been recognized by the employer or
authorized in writing by the individual worker concerned; and
c)
In cases where the employer is authorized by law or regulations
issued by the Secretary of Labor.

TOPIC: CONSIDERATION FOR STOCKS

Page 632 of 1072

FUA CUN (alias Tua Cun), plaintiff-appellee,


vs.
RICARDO SUMMERS, in his capacity as Sheriff ex-oficio of the City of
Manila, and the CHINA BANKING CORPORATION, defendantsappellants.
G.R. No. L-19441. March 27, 1923
44 PHIL 705
FACTS:
It appears from the evidence that on August 26, 1920, one Chua Soco
subscribed for five hundred shares of stock of the defendant Banking
Corporation at a par value of P100 per share, paying the sum of P25, 000,
one-half of the subscription price, in cash, for which a receipt was issued.
On May 18, 1921, Chua Soco executed a promissory note in favor of
the plaintiff Fua Cun for the sum of P25, 000 payable in ninety days and
drawing interest at the rate of 1 per cent per month, securing the note with a
chattel mortgage on the shares of stock subscribed for by Chua Soco, who
also endorsed the receipt above mentioned and delivered it to the
mortgagee. The plaintiff thereupon took the receipt to the manager of the
defendant Bank and informed him of the transaction with Chua Soco, but
was told to await action upon the matter by the Board of Directors.
In the meantime Chua Soco appears to have become indebted to the
China Banking Corporation in the sum of P37,731.68 for dishonored
acceptances of commercial paper and in an action brought against him to
recover this amount, Chua Soco's interest in the five hundred shares
subscribed for was attached and the receipt seized by the sheriff. The
attachment was levied after the defendant bank had received notice of the
facts that the receipt had been endorsed over to the plaintiff.
Fua Cun thereupon brought the present action maintaining that by
virtue of the payment of the one-half of the subscription price of five hundred
shares Chua Soco in effect became the owner of two hundred and fifty
shares and praying that his, the plaintiff's, lien on said shares, by virtue of
the chattel mortgage, be declared to hold priority over the claim of the
defendant Banking Corporation; that the defendants be ordered to deliver
the receipt in question to him; and that he be awarded the sum of P5,000 in
damages for wrongful attachment.
ISSUE/S:
Whether or not the plaintiff-appellee has rights over the stock
subject of the instant case.

Page 633 of 1072

RULING:
Turning now to the rights of the plaintiff in the stock in question, it is
argued that the interest held by Chua Soco was merely an equity which could
not be made the subject of a chattel mortgage. Though the courts have
uniformly held that chattel mortgages on shares of stock and other choses in
action are valid as between the parties, there is still much to be said in favor
of the defendants' contention that the chattel mortgage here in question
would not prevail over liens of third parties without notice; an equity in
shares of stock is of such an intangible character that it is somewhat difficult
to see how it can be treated as a chattel and mortgaged in such a manner
that the recording of the mortgage will furnish constructive notice to third
parties. But a determination of this question is not essential in the present
case. There can be no doubt that an equity in shares of stock may be
assigned and that the assignment is valid as between the parties and as to
persons to whom notice is brought home. Such an assignment exists here,
though it was made for the purpose of securing a debt.
As against the rights of the plaintiff the defendant bank had, as we
have seen, no lien unless by virtue of the attachment. But the attachment
was levied after the bank had received notice of the assignment of Chua
Soco's interests to the plaintiff and was therefore subject to the rights of the
latter. It follows that as against these rights the defendant bank holds no lien
whatever. As we have already stated, the court erred in holding the plaintiff
as the owner of two hundred and fifty shares of stock; "the plaintiff's rights
consist in equity in five hundred shares and upon payment of the unpaid
portion of the subscription price he becomes entitledto the issuance of
certificate for said five hundred shares in his favor.

TOPIC: CONSIDERATION FOR STOCKS

Page 634 of 1072

THE NATIONAL EXCHANGE CO., INC


vs.
I. B. DEXTER
G.R. No. L-27872
February 25, 1928
51 PHIL 601
FACTS:
It appears that on August 10, 1919, the defendant, I. B. Dexter, signed
a written subscription to the corporate stock of C. S. Salmon & Co. in the
following form: I hereby subscribe for three hundred (300) shares of the
capital stock of C. S. Salmon and Company, payable from the first dividends
declared on any and all shares of said company owned by me at the time
dividends are declared, until the full amount of this subscription has been
paid.
Upon this subscription the sum of P15,000 was paid in January, 1920,
from a dividend declared at about that time by the company, supplemented
by money supplied personally by the subscriber. Beyond this nothing has
been paid on the shares and no further dividend has been declared by the
corporation. There is therefore a balance of P15,000 still paid upon the
subscription.
ISSUE/S:
Whether the stipulation contained in the subscription to the
effect that the subscription is payable from the first dividends
declared on the shares has the effect of relieving the subscriber
from personal liability in an action to recover the value of the
shares. The trial court held, in effect, that the stipulation
mentioned is invalid.
RULING:
The prohibition against the issuance of shares by corporations except
for actual cash to the par value of the stock to its full equivalent in property
is thus enshrined in both the organic and statutory law of the Philippine;
Islands; and it would seem that our lawmakers could scarely have chosen
language more directly suited to secure absolute equality stockholders with
respect to their liability upon stock subscriptions. Now, if it is unlawful to
issue stock otherwise than as stated it is self-evident that a stipulation such
as that now under consideration, in a stock subcription, is illegal, for this
stipulation obligates the subcriber to pay nothing for the shares except as
dividends may accrue upon the stock. In the contingency that dividends are
not paid, there is no liability at all. This is a discrimination in favor of the
particular subcriber, and hence the stipulation is unlawful.

Page 635 of 1072

The rule thus stated is supported by a long line of decisions from


numerous courts, with little or no diversity of opinion. As stated in the
headnote to the opinion of the Supreme Court of United States in the case of
Putnan vs.New Albany, etc. Railroad Co. as reported in 21 Law. ed., 361, the
rule is that "Conditions attached to subcriptions, which, if valid, lessen the
capital of the company, are a fraud upon the grantor of the franchise, and
upon those who may become creditors of the corporation, and upon
unconditional stockholders."
In the appellant's brief attention is called to the third headnote to
Bank vs. Cook (125 Iowa, 111), where it is stated that a collateral agreement
with a subcriber to stock that his subcription shall not be collectible except
from dividends on the stock, is valid as between the parties and a complete
defense to a suit on notes given for the amount of the subscription. A careful
persual of the decision will show that the rule thus broadly stated in the
headnote is not justified by anything in the reported decision; for what the
court really held was that the making of such promise by the agent of the
corporation who sold the stock is admissible in evidence in support of the
defense of fraud and failure of consideration. Moreover, even if the decision
had been to the effect supposed, the relu announced in the headnote could
have no weight in a jurisdiction like this were there is a statutory provision
prohibiting such agreements.
We may add that the law in force in this jurisdiction makes no
distinction, in respect to the liability of the subcriber, between shares
subscribed before incorporation is effected and shares subscribed thereafter.
All like are bound to pay full value in cash or its equivalent, and any attempt
to discriminate in favor of one subscriber by relieving him of this liability
wholly or in part is forbidden. In what is here said we have reference of
course primarily to subcriptions to shares that have not been previously
issued. It is conceivable that the power of the corporation to make terms
with the purchaser would be greater where the shares which are the subject
of the transaction have been acquired by the corporation in course of
commerce, after they have already been once issued. But the shares with
which are here concerned are not of this sort.

Page 636 of 1072

TOPIC: CONSIDERATION FOR STOCKS


NIELSON & COMPANY, INC., plaintiff-appellant,
vs.
LEPANTO CONSOLIDATED MINING COMPANY, defendant-appellee.
G.R. No. L-21601. December 28, 1968
26 SCRA 541
FACTS:
It appears that the suit involves an operating agreement executed
before World War II between the plaintiff and the defendant whereby the
former operated and managed the mining properties owned by the latter for
a management fee of P2,500.00 a month and a 10% participation in the net
profits resulting from the operation of the mining properties. For brevity and
convenience, hereafter the plaintiff shall be referred to as NIELSON and the
defendant, LEPANTO.
The antecedents of the case are: The contract in question (Exhibit `C')
was made by the parties on January 30, 1937 for a period of five (5) years. In
the latter part of 1941, the parties agreed to renew the contract for another
period of five (5) years, but in the meantime, the Pacific War broke out in
December, 1941.
In January, 1942 operation of the mining properties was disrupted on
account of the war. In February of 1942, the mill, power plant, supplies on
hand, equipment, concentrates on hand and mines, were destroyed upon
orders of the United States Army, to prevent their utilization by the invading
Japanese Army. The Japanese forces thereafter occupied the mining
properties, operated the mines during the continuance of the war, and who
were ousted from the mining properties only in August of 1945.
After the mining properties were liberated from the Japanese forces,
LEPANTO took possession thereof and embarked in rebuilding and
reconstructing the mines and mill; setting up new organization; clearing the
mill site; repairing the mines; erecting staff quarters and bodegas and
repairing existing structures; installing new machinery and equipment;
repairing roads and maintaining the same; salvaging equipment and storing
the same within the bodegas; doing police work necessary to take care of the
materials and equipment recovered; repairing and renewing the water
system; and remembering (Exhibits "D" and "E"). The rehabilitation and
reconstruction of the mine and mill was not completed until 1948 (Exhibit
"F"). On June 26, 1948 the mines resumed operation under the exclusive
management of LEPANTO (Exhibit "F-l").

Page 637 of 1072

Shortly after the mines were liberated from the Japanese invaders in
1945, a disagreement arose between NIELSON and LEPANTO over the status
of the operating contract in question which as renewed expired in 1947.
Under the terms thereof, the management contract shall remain in suspense
in case fortuitous event or force majeure, such as war or civil commotion,
adversely affects the work of mining and milling.
NIELSON held the view that, on account of the war, the contract was
suspended during the war; hence the life of the contract should be
considered extended for such time of the period of suspension. On the other
hand, LEPANTO contended that the contract should expire in 1947 as
originally agreed upon because the period of suspension accorded by virtue
of the war did not operate to extend further the life of the contract
ISSUE/S:
Whether or not Nielson is entitled of the 10% share in the profits
of operation realized during the period of five (5) years from the
resumption of its post-war operations of the Mankayan mines, in
the total sum of P2,403,053.20 with interest thereon at the rate
of 6% per annum from February 6, 1958 until full payment.
RULING:
The above claim of Nielson refers to four categories, namely:
1. cash dividends;
2. stock dividends;
3. depletion reserves; and
4. amount expended on capital investment.
Anent the first category, Lepanto's report for the calendar year
1954 contains a record of the cash dividends it paid up to the date of said
report, and the post-war dividends paid by it corresponding to the years
included in the period of extension of the management contract.
39

According to the terms of the management contract as modified,


appellant is entitled to 10% of the P14,000,000.00 cash dividends that had
been distributed, as stated in the above-mentioned report, or the sum of
P1,400,000.00.
With regard to the second category, the stock dividends declared by
Lepanto during the period of extension of the contract are: On November 28,
1949, the stock dividend declared was 50% of the outstanding authorized
capital of P2,000,000.00 of the company, or stock dividends worth
P1,000,000.00; and on August 22, 1950, the stock dividends declared was
66-2/3% of the standing authorized capital of P3,000,000.00 of the company,
or stock dividends worth P2,000,000.00.

Page 638 of 1072

Appellant's claim that it should be given 10% of the cash value of said
stock dividends with interest thereon at 6% from February 6, 1958 cannot be
granted for that would not be in accordance with the management contract
which entitles Nielson to 10% of any dividends declared paid, when and as
paid. Nielson, therefore, is entitled to 10% of the stock dividends and to the
fruits that may have accrued to said stock dividends pursuant to Article 1164
of the Civil Code. Hence to Nielson is due shares of stock worth P100,000.00,
as per stock dividends declared on November 28, 1949 and all the fruits
accruing to said shares after said date; and also shares of stock worth
P200,000.00 as per stock dividends declared on August 20, 1950 and all
fruits accruing thereto after said date.
Anent the third category, the depletion reserve appearing in the
statement of income and surplus submitted by Lepanto corresponding to the
years covered by the period of extension of the contract, may be itemized as
follows: In 1948, as per Exh. F, p. 36 and Exh. Q, p. 5, the depletion reserve
set up was P11,602.80. In 1949, as per Exh. G, p. 49 and Exh. Q, p. 5, the
depletion reserve set up was P33,556.07. In 1950, as per Exh. H, p. 37, Exh.
Q, p. 6 and Exh. I, p. 37, the depletion reserve set up was P84,963.30. In
1951, as per Exh. I, p. 45, Exh. Q, p. 6, and Exh. J, p. 45, the depletion
reserve set up was P129,089.88. In 1952, as per Exh. J, p. 45, Exh. Q, p. 6
and Exh. K p. 41, the depletion reserve was P147,141.54. In 1953, as per
Exh. K, p. 41, and Exh. Q, p. 6, the depletion reserve set up as P277,493.25.
Regarding the depletion reserve set up in 1948 it should be noted that the
amount given was for the whole year. Inasmuch as the contract was
extended only for the last half of the year 1948, said amount of P11,602.80
should be divided by two, and so Nielson is only entitled to 10% of the half
amounting to P5,801.40.
Likewise, the amount of depletion reserve for the year 1953 was for
the whole year and since the contract was extended only until the first half of
the year, said amount of P277,493.25 should be divided by two, and so
Nielson is only entitled to 10% of the half amounting to P138,746.62.
Summing up the entire depletion reserves, from the middle of 1948 to the
middle of 1953, we would have a total of P539,298.81, of which Nielson is
entitled to 10%, or to the sum of P53,928.88.
Finally, with regard to the fourth category, there is no figure in the
record representing the value of the fixed assets as of the beginning of the
period of extension on June 27, 1948. It is possible, however, to arrive at the
amount needed by adding to the value of the fixed assets as of December
31, 1947 one-half of the amount spent for capital account in the year 1948.
As of December 31, 1947, the value of the fixed assets was
P1,061,878.88 41and as of December 31, 1948, the value of the fixed assets
was P3,270,408.07. 42 Hence, the increase in the value of the fixed assets for

Page 639 of 1072

the year 1948 was P2,208,529.19, one-half of which is P1,104,264.59, which


amount represents the expenses for capital account for the first half of the
year 1948. If to this amount we add the fixed assets as of December 31,
1947 amounting to P1,061,878.88, we would have a total of P2,166,143.47
which represents the fixed assets at the beginning of the second half of the
year 1948.
There is also no figure representing the value of the fixed assets when
the contract, as extended, ended on June 26, 1953; but this may be
computed by getting one-half of the expenses for capital account made in
1953 and adding the same to the value of the fixed assets as of December
31, 1953 is P9,755,840.41 43 which the value of the fixed assets as of
December 31, 1952 is P8,463,741.82, the difference being P1,292,098.69.
One-half of this amount is P646,049.34 which would represent the expenses
for capital account up to June, 1953. This amount added to the value of the
fixed assets as of December 31, 1952 would give a total of P9,109,791.16
which would be the value of fixed assets at the end of June, 1953.
The increase, therefore, of the value of the fixed assets of Lepanto from June,
1948 to June, 1953 is P6,943,647.69, which amount represents the difference
between the value of the fixed assets of Lepanto in the year 1948 and in the
year 1953, as stated above. On this amount Nielson is entitled to a share of
10% or to the amount of P694,364.76.
Considering that most of the claims of appellant have been
entertained, as pointed out in this decision, We believe that appellant is
entitled to be awarded attorney's fees, especially when, according to the
undisputed testimony of Mr. Mark Nestle, Nielson obliged himself to pay
attorney's fees in connection with the institution of the present case. In this
respect, We believe, considering the intricate nature of the case, an award of
fifty thousand (P50,000.00) pesos for attorney's fees would be reasonable.

Page 640 of 1072

TOPIC: CONSIDERATION FOR STOCKS


NAZARIO TRILLANA, administrator-appellee,
vs.
QUEZON COLLEGE, INC., claimant-appellant.
G.R. No. L-5003. June 27, 1953
FACTS:
Damasa Crisostomo sent letter to the Board of Trustees of the Quezon
College to pay his subscriptions of shares of stock.
Damasa Crisostomo however died on October 26, 1948. As no payment
appears to have been made on the subscription mentioned in the foregoing
letter, the Quezon College, Inc. presented a claim before the Court of First
Instance of Bulacan in her testate proceeding, for the collection of the sum of
P20,000, representing the value of the subscription to the capital stock of the
Quezon College, Inc.
This claim was opposed by the administrator of the estate, and the
Court of First Instance of Bulacan, after hearing issued an order dismissing
the claim of the Quezon College, Inc. on the ground that the subscription in
question was neither registered in nor authorized by the Securities and
Exchange Commission.
From this order the Quezon College, Inc. has appealed.
ISSUE/S:
Whether or not Damasa is obligated to pay the subscription of
the stocks by virtue of his letter.
RULING:
It appears that the application sent by Damasa Crisostomo to the
Quezon College, Inc. was written on a general form indicating that an
applicant will enclose an amount as initial payment and will pay the balance
in accordance with law and the regulations of the College. On the other hand,
in the letter actually sent by Damasa Crisostomo, the latter (who requested
that her subscription for 200 shares be entered) not only did not enclose any

Page 641 of 1072

initial payment but stated that "babayaran kong lahat pagkatapos na ako ay
makapagpahuli ng isda."
There is nothing in the record to show that the Quezon College, Inc.
accepted the term of payment suggested by Damasa Crisostomo, or that if
there was any acceptance the same came to her knowledge during her
lifetime. As the application of Damasa Crisostomo is obviously at variance
with the terms evidenced in the form letter issued by the Quezon College,
Inc., there was absolute necessity on the part of the College to express its
agreement to Damasa's offer in order to bind the latter. Conversely, said
acceptance was essential, because it would be unfair to immediately obligate
the Quezon College, Inc. under Damasa's promise to pay the price of the
subscription after she had caused fish to be caught. In other words, the
relation between Damasa Crisostomo and the Quezon College, Inc. had only
thus reached the preliminary stage whereby the latter offered its stock for
subscription on the terms stated in the form letter, and Damasa applied for
subscription fixing her own plan of payment, a relation, in the absence as
in the present case of acceptance by the Quezon College, Inc. of the counter
offer of Damasa Crisostomo, that had not ripened into an enforceable
contract.
Indeed, the need for express acceptance on the part of the Quezon
College, Inc. becomes the more imperative, in view of the proposal of
Damasa Crisostomo to pay the value of the subscription after she has
harvested fish, a condition obviously dependent upon her sole will and,
therefore, facultative in nature, rendering the obligation void, under article
1115 of the old Civil Code which provides as follows: "If the fulfillment of the
condition should depend upon the exclusive will of the debtor, the
conditional obligation shall be void. If it should depend upon chance, or upon
the will of a third person, the obligation shall produce all its effects in
accordance with the provisions of this code." It cannot be argued that the
condition solely is void, because it would have served to create the
obligation to pay, unlike a case, exemplified by Osmea vs. Rama (14 Phil.,
99), wherein only the potestative condition was held void because it referred
merely to the fulfillment of an already existing indebtedness.

Page 642 of 1072

TOPIC: COLLECTION OF UNPAID SUBSCRIPTION


GERARDO GARCIA, plaintiff-appellee,
vs.
ANGEL SUAREZ, defendant-appellant.
G.R. No. L-45493
April 21, 1939
67 SCRA 441
FACTS:
On October 4, 1924, the appellant subscribed to sixteen shares of the
capital stock of the Compaia Hispano-Filipina, Inc. Of the sixteen subscribed
shares, at the par value of P100 each, the appellant only paid P400, the
value of four shares.
On June 5, 1931, the plaintiff-appellee was appointed by the court
receiver of the Compaia Hispano-Filipina, Inc., to collect all the credits of
said corporation, pay its debts and dispose of the remainder of its assets and
of its properties.
On June 18, 1931, the plaintiff-appellee in vain made demand upon the
defendant-appellant to pay the balance of his subscription. On July 10, 1933,
the plaintiff, as receiver, brought an action in the Court of First Instance of
Manila to recover from the defendant-appellant and other shareholders the
balance of their subscriptions, but the complaint was dismissed for lack of
prosecution.
On October 10, 1935, a similar complaint was filed against the
appellant, and after trial, judgment was rendered therein ordering the said
defendant to pay to the plaintiff, as receiver of Compaia Hispano-Filipina,
Inc., the sum of P1,200, with legal interest thereon from October 4, 1924,
and the costs. The defendant appealed and in this instance contends that the
trial court erred in holding that the action of the plaintiff-appellee has not
prescribed, and that the appellant has not been released from his obligation
to pay the balance of his subscription.
ISSUE/S:

Page 643 of 1072

Whether or not defendant-appellant is still liable for his unpaid


subscription.
Whether or not defendant appellant was released from his obligation.
RULING:
Yes.
The subscription to the capital stock of the corporation, unless
otherwise stipulated, is not payable at the moment of the subscription but on
a subsequent date which may be fixed by the corporation. Hence, section 38
of the Corporation Law, amended by Act No. 3518, provides that:
The board of directors or trustees of any stock corporation
formed, organized, or existing under this Act may at any
time declare due and payable to the corporation unpaid
subscriptions to the capital stock . . . .
The board of directors of the Compaia Hispano-Filipino, Inc., not
having declared due and payable the stock subscribed by the appellant, the
prescriptive period of the action for the collection thereof only commenced to
run from June 18, 1931 when the plaintiff, in his capacity as receiver and in
the exercise of the power conferred upon him by the said section 38 of the
Corporation Law, demanded of the appellant to pay the balance of his
subscription. The present action having been filed on October 10, 1935, the
defense of prescription is entirely without basis.
As the second issue, defendant was not release from his obligation
despite his claim that a letter, allegedly signed by R. Pando, acting president
of the corporation Compaia Hispano-Filipina, Inc., wherein the appellant was
released by Pando from all obligation with respect to the payment of his
subscription in consideration of his transfer of his shares to the corporation
was not in accordance with the requirements of the law.
The agents or officers of the corporation have no such power, however,
unless it is expressly conferred upon them by the charter or statute, or by
the stockholders by a by-law or otherwise.
It has not been established that the stockholders of the Compaia
Hispano-Filipina, Inc., have in any wise consented to release the appellant
from his obligation, or that the acting president, R. Pando, was expressly
authorized by the stockholders, or was authorized by the by-laws of the
corporation, to release the appellant from his obligation.
A stock subscription is a contract between the corporation and the
subscriber, and courts will enforce it for or against either. A corporation has
no legal capacity to release a subscriber to its capital stock from the
obligation to pay for his shares, and any agreement to this effect is invalid.

Page 644 of 1072

TOPIC: COLLECTION OF UNPAID SUBSCRIPTION


PHILIPPINE NATIONAL BANK, plaintiff-appellee,
vs.
BITULOK SAWMILL, INC., DINGALAN LUMBER CO., INC., SIERRA
MADRE LUMBER CO., INC., NASIPIT LUMBER CO., INC., WOODWORKS,
INC., GONZALO PUYAT, TOMAS B. MORATO, FINDLAY MILLAR LUMBER
CO., INC., ET AL., INSULAR LUMBER CO., ANAKAN LUMBER CO., AND
CANTILAN LUMBER CO., INC., defendants-appellees.
G.R. Nos. L-24177-85
June 29, 1968
23 SCRA 1366
FACTS:
The Philippine Lumber Distributing Agency, Inc. was organized
sometime in the early part of 1947 upon the initiative and insistence of the
late President Manuel Roxas. The purpose was to insure a steady supply of
lumber, which could be sold at reasonable prices to enable the war sufferers
to rehabilitate their devastated homes. As an inducement he promised and
agreed to finance the agency by making the Government invest P9.00 by
way of counterpart for every peso that the members would invest therein.
There was no any appropriation by the legislature of the counterpart
fund to be put up by the Government, namely, P9.00 for every peso invested
by defendant lumber producers. Accordingly, "the late President Roxas
instructed the Hon. Emilio Abello, then Executive Secretary and Chairman of
the Board of Directors of the Philippine National Bank, for the latter to grant
said agency an overdraft in the original sum of P250,000.00 which was later
increased to P350,000.00, which was approved by said Board of Directors of
the Philippine National Bank on July 28, 1947, payable on or before April 30,
1958, with interest at the rate of 6% per annum, and secured by the chattel
mortgages on the stock of lumber of said agency.
The Philippine Government did not invest the P9.00 for every peso
coming from defendant lumber producers. The loan extended to the
Philippine Lumber Distributing Agency by the Philippine National Bank was
not paid.

Page 645 of 1072

Hence, these suits.


ISSUE/S:
Whether or not the respondents are liable for their unpaid balance to
their subscription, of capital stocks of the Philippine Lumber
Distributing Agency, Inc.
RULING:
YES.
The defendant corporations are liable to pay for their unpaid
subscription. It is established doctrine that subscriptions to the capital of a
corporation constitute a fund to which creditors have a right to look for
satisfaction of their claims and that the assignee in insolvency can maintain
an action upon any unpaid stock subscription in order to realize assets for
the payment of its debt.... A corporation has no power to release an original
subscriber to its capital stock from the obligation of paying for his shares,
without a valuable consideration for such release; and as against creditors a
reduction of the capital stock can take place only in the manner and under
the conditions prescribed by the statute or the charter or the articles of
incorporation. Moreover, strict compliance with the statutory regulations is
necessary.
It would be unwarranted to ascribe to the late President Roxas the view
that the payment of the stock subscriptions, as thus required by law, could
be condoned in the event that the counterpart fund to be invested by the
Government would not be available. Even if such were the case, however,
and such a promise were in fact made, to further the laudable purpose to
which the proposed corporation would be devoted and the possibility that
the lumber producers would lose money in the process, still the plain and
specific wording of the applicable legal provision as interpreted by this Court
must be controlling. It is a well-settled principle that with all the vast powers
lodged in the Executive, he is still devoid of the prerogative of suspending
the operation of any statute or any of its terms.

Page 646 of 1072

TOPIC: COLLECTION OF UNPAID SUBSCRIPTION


MIGUEL VELASCO, assignee of The Philippine Chemical Product Co.
(Ltd.), plaintiff-appellant,
vs.
JEAN M. POIZAT, defendant-appellee
G.R. No. L-11528
March 15, 1918
37 Phil 802
FACTS:
Plaintiff, as assignee in insolvency of "The Philippine Chemical Product
Company" (Ltd.) is seeking to recover of the defendant, Jean M. Poizat, the
sum of P1,500, upon a subscription made by him to the corporate stock of
said company. The company has capital of P50,000, divided into 500 shares.
The defendant subscribed for 20 shares of the stock of the company, and
paid upon his subscription the sum of P500, the par value of 5 shares. The
defendant was a stock holder in the company from the inception of the
enterprise, and for sometime acted as its treasurer and manager. While
serving in this capacity he called in and collected all subscriptions to the
capital stock of the company, except the aforesaid 15 shares subscribed by
him and another 15 shares owned by Jose R. Infante.
The Board of directors in their meeting adopt a resolution that Juan
[Jean] M. Poizat, who was absent, should be required to pay the amount of
his subscription upon the 15 shares for which he was still indebted to the
company. The resolution further provided that, in case he should refuse to
make such payment, the management of the corporation should be
authorized to undertake judicial proceedings against him.
The company became insolvent while the unpaid balance of Poizat was
not yet paid. The defendant still failed and refused to pay the balance with
defenses.
ISSUE/S:
Whether or not respondent Poizat is liable for his unpaid subscription.

Page 647 of 1072

RULING:
YES.
He is liable to pay. A stock subscription is a contract between the
corporation on one side, and the subscriber on the other, and courts will
enforce it for or against either. It is a rule, accepted by the Supreme Court of
the United States that a subscription for shares of stock does not require an
express promise to pay the amount subscribed, as the law implies a promise
to pay on the part of the subscriber. Section 36 of the Corporation Law
clearly recognizes that a stock subscription is subsisting liability from the
time the subscription is made, since it requires the subscriber to pay interest
quarterly from that date unless he is relieved from such liability by the bylaws of the corporation. The subscriber is as much bound to pay the amount
of the share subscribed by him as he would be to pay any other debt, and
the right of the company to demand payment is no less incontestable.
The provisions of the Corporation Law (Act No. 1459) given recognition
of two remedies for the enforcement of stock subscriptions. The first and
most special remedy given by the statute consists in permitting the
corporation to put up the unpaid stock for sale and dispose of it for the
account of the delinquent subscriber. In this case the provisions of section 38
to 48, inclusive, of the Corporation Law are applicable and must be followed.
The other remedy is by action in court, concerning which we find in section
49 the following provision:
Nothing in this Act shall prevent the directors from collecting, by action in
any court of proper jurisdiction, the amount due on any unpaid subscription,
together with accrued interest and costs and expenses incurred.
It is generally accepted doctrine that the statutory right to sell the
subscriber's stock is merely a remedy in addition to that which proceeds by
action in court; and it has been held that the ordinary legal remedy by action
exists even though no express mention thereof is made in the statute.
By virtue of the first subsection of section 36 of the Insolvency Law
(Act No. 1956) the assignee of the insolvent corporation succeeds to all the
corporate rights of action vested in the corporation prior to its insolvency;
and the assignee therefore has the same freedom with respect to suing upon
the stock subscription as the directors themselves would have had under
section 49.
When insolvency supervenes upon a corporation and the court
assumes jurisdiction to wind up, all unpaid stock subscriptions become
payable on demand, and are at once recoverable in an action instituted by
the assignee or receiver appointed by the court. This rule apparently had
origin in a recognition of the principle that a court of equity, having
jurisdiction of the insolvency proceedings, could, if necessary, make the call

Page 648 of 1072

itself, in its capacity as successor to the powers exercised by the board of


directors of the defunct company. Later a further rule gained recognition to
the effect that the receiver or assignee, in an action instituted by proper
authority, could himself proceed to collect the subscription without the
necessity of any prior call whatever.
It evidently cannot be permitted that a subscriber should escape from
his lawful obligation by reason of the failure of the officers of the corporation
to perform their duty in making a call; and when the original model of
making the call becomes impracticable, the obligation must be treated as
due upon demand. If the corporation must be treated still an active entity
and this action should be dismissed for irregularity in the making of the call,
other steps could be taken by the board to cure the defect and another
action could be brought; but where the company is being wound up, no such
procedure would be practicable. The better doctrine is that when insolvency
supervenes all unpaid subscriptions become at once due and enforceable.

Page 649 of 1072

TOPIC: COLLECTION OF UNPAID SUBSCRIPTION: COURT ACTION


BONIFACIO LUMANLAN, plaintiff-appellee,
vs.
JACINTO R. CURA, ET AL., defendants. DIZON & CO., INC.,
ETC., appellant
G.R. No. L-39861
March 21, 1934
59 Phil. 746
FACTS:
The appellant is a corporation duly organized under the laws of the
Philippine Islands with its central office in the City of Manila. The plaintiffappellee Bonifacio Lumanlan, on July 31, 1922, subscribed for 300 shares of
stock of said corporation at a par value of P50 or a total of P15,000. Julio
Valenzuela, Pedro Santos and Francisco Escoto, creditors of this corporation,
filed suit against it in the Court of First Instance of Manila, case No. 37007,
praying that a receiver be appointed, as it appeared that the corporation at
that time had no assets except credits against those who had subscribed for
shares of stock.
The court named Tayag as receiver for the purpose of collecting, said
subscriptions. As Bonifacio Lumanlan had only paid P1,500 of the P15,000,
par value of the stock for which he subscribed, the receiver on August 30,
1930, filed a suit against him in the Court of First Instance of Manila, civil
case No. 37492, for the collection of P15,109, P13,500 of which was the
amount he owed for unpaid stock and P1,609 for loans and advances by the
corporation to Lumanlan. In that case Lumanlan was sentenced to pay the
corporation the above-mentioned sum of P15,109 with legal interest thereon
from August 30, 1930, and costs. Lumanlan appealed from this decision.
Pending this appeal, with the permission of the court, the creditors,
some of the directors and the majority of the stockholders held several
meetings in which it was agreed in substance that subscribers for the capital
stock who were in default should pay the creditors; Lumanlan was
designated to pay the debt of the corporation to Julio Valenzuela, one of the
petitioners in case No. 37007; at that time the corporation owed Valenzuela
the sum of P8,000 plus interest thereon at the rate of 12 per cent per annum
from March 17, 1928. Lumanlan agreed to assume this obligation and in turn
the corporation agreed that if Lumanlan would dismiss his appeal in case No.
37492 the corporation would collect only 50 per cent of the amount
subscribed by him for stock, provided that in case the 50 percent was

Page 650 of 1072

insufficient to pay Valenzuela he should pay an additional amount which


should not exceed the amount of the judgment against him in that case.
In view of this agreement Lumanlan withdrew his appeal and paid
Valenzuela the sum of P11,840 including interest and thereby was
subrogated in place of Valenzuela. The petitioning creditors having been paid
the amounts owed to them by the corporation asked that the receiver be
dismissed and the court granted this. Disregarding this agreement and
notwithstanding the payment made by Lumanlan to Valenzuela, the
corporation on May 5, 1932, asked for the execution of the sentence in case
No. 37492 and by virtue of an order of execution the provincial sheriff levied
upon two parcels of land belonging to Lumanlan described in certificate of
title No. 901 of the Province of Tarlac. Lumanlan brought this case to collect
from Dizon & Co., Inc., and to prevent the sheriff from selling the two parcels
of land. Pending the result of this case the sheriff was enjoined from
proceeding with the sale.
ISSUE/S:
Whether or not the complaint of the plaintiff will prosper considering
that he has still unpaid stock subscription .
RULING:
It appears from the record that during the trial of the case now under
consideration, the Bank of the Philippine Islands appeared in this case as
assignee in the "Involuntary Insolvency of Dizon & Co., Inc. That bank was
appointed assignee in case No. 43065 of the Court of First Instance of the
City of Manila on November 28, 1932. It is therefore evident that there are
still other creditors of Dizon & Co., Inc. This being the case that corporation
has a right to collect all unpaid stock subscriptions and any other amounts
which may be due it.
It is established doctrine that subscriptions to the capital of a
corporation constitute a fund to which the creditors have a right to look for
satisfaction of their claims and that the assignee in insolvency can maintain
an action upon any unpaid stock subscription in order to realize assets for
the payment of its debts. (Philippine Trust Co. vs. Rivera, 44 Phil., 469, 470.)
. . . the Corporation Law clearly recognizes that a stock
subscription is a subsisting liability from the time the
subscription is made, since it requires the subscriber to pay
interest quarterly from that date unless he is relieved from
such liability by the by-laws of the corporation. The
subscriber is as much bound to pay the amount of the
share subscribed by him as he would be to pay any other
debt, and the right of the company to demand payment is

Page 651 of 1072

no less incontestable. (Velasco vs. Poizat, 37 Phil., 802,


805.)

TOPIC: COLLECTION OF UNPAID SUBSCRIPTION: COURT ACTION


EDWARD A. KELLER & CO., LTD., petitioner-appellant,
vs.
COB GROUP MARKETING, INC., JOSE E. BAX, FRANCISCO C. DE
CASTRO, JOHNNY DE LA FUENTE, SERGIO C. ORDOEZ, TRINIDAD C.
ORDOEZ, MAGNO C. ORDOEZ, ADORACION C. ORDOEZ, TOMAS C.
LORENZO, JR., LUIZ M. AGUILA-ADAO, MOISES P. ADAO, ASUNCION
MANAHAN and INTERMEDIATE APPELLATE COURT, respondentsG.R. No. L-68097
January 16, 1986
FACTS:
Edward A. Keller & Co., Ltd. appointed COB Group Marketing, Inc. as
exclusive distributor of its household products, Brite and Nuvan in Panay and
Negros, as shown in the sales agreement dated March 14, 1970 (32-33 RA).
Under that agreement Keller sold on credit its products to COB Group
Marketing.As security for COB Group Marketing's credit purchases up to the
amount of P35,000, one Asuncion Manahan mortgaged her land to Keller.
Manahan assumed solidarily with COB Group Marketing the faithful
performance of all the terms and conditions of the sales agreement.
In July, 1970 the parties executed a second sales agreement whereby
COB Group Marketing's territory was extended to Northern and Southern
Luzon. As security for the credit purchases up to P25,000 of COB Group
Marketing for that area, Tomas C. Lorenzo, Jr. and his father Tomas, Sr. (now
deceased) executed a mortgage on their land in Nueva Ecija. Like Manahan,
the Lorenzos were solidarily liable with COB Group Marketing for its
obligations under the sales agreement.
The credit purchases of COB Group Marketing, which started on
October 15, 1969, limited up to January 22, 1971. On May 8, the board of
directors of COB Group Marketing were apprised by Jose E. Bax the firm's
president and general manager, that the firm owed Keller about
P179,000. Bax was authorized to negotiate with Keller for the settlement of
his firm's liability.On the same day, May 8, Bax and R. Oefeli of Keller signed
the conditions for the settlement of COB Group Marketing's liability.

Page 652 of 1072

Twelve days later, or on May 20, COB Group Marketing, through Bax
executed two second chattel mortgages over its 12 trucks (already
mortgaged to Northern Motors, Inc.) as security for its obligation to Keller
amounting to P179,185.16 as of April 30, 1971. However, the second
mortgages did not become effective because the first mortgagee, Northern
Motors, did not give its consent. But the second mortgages served the
purpose of being admissions of the liability COB Group Marketing to Keller.
The stockholders of COB Group Marketing, Moises P. Adao and Tomas C.
Lorenzo, Jr., in a letter dated July 24, 1971 to Keller's counsel, proposed to
pay Keller P5,000 on November 30, 1971 and thereafter every thirtieth day
of the month for three years until COB Group Marketing's mortgage
obligation had been fully satisfied. They also proposed to substitute the
Manahan mortgage with a mortgage on Adao's lot at 72 7th Avenue, Cubao,
Quezon City.Keller sued on September 16, 1971 COB Group Marketing, its
stockholders and the mortgagors, Manahan and Lorenzo.COB Group
Marketing, Trinidad C. Ordonez and Johnny de la Fuente were declared in
default.After trial, the lower court dismissed the complaint; The petitioner
appealed. The Appellate Court affirmed said judgment.
ISSUE/S:
Whether or not COB Group marketing can be demanded to pay its
obligation infavor of the petitioner.
RULING:
We find that the lower courts erred in nullifying the admissions of
liability made in 1971 by Bax as president and general manager of COB
Group Marketing and in giving credence to the alleged overpayment
computed by Bax .The lower courts not only allowed Bax to nullify his
admissions as to the liability of COB Group Marketing but they also
erroneously rendered judgment in its favor in the amount of its supposed
overpayment in the sum of P100,596.72, in spite of the fact that COB Group
Marketing was declared in default and did not file any counterclaim for the
supposed overpayment.
As to the liability of the stockholders, it is settled that a stockholder is
personally liable for the financial obligations of a corporation to the extent of
his unpaid subscription (Vda. de Salvatierra vs. Garlitos 103 Phil. 757, 763;
18 CJs 1311-2).
While the evidence shows that the amount due from COB Group
Marketing is P184,509.60 as of July 31, 1971 or P186,354.70 as of August 31,
1971 (Exh. JJ), the amount prayed for in Keller's complaint is P182,994.60 as
of July 31, 1971 (18-19 Record on Appeal). This latter amount should be the

Page 653 of 1072

one awarded to Keller because a judgment entered against a party in default


cannot exceed the amount prayed for (Sec. 5, Rule 18, Rules of Court).

TOPIC: EFFECT OF DELINQUENCY:


VALLEY GOLF & COUNTRY CLUB, INC., Petitioner,
vs.
ROSA O. VDA. DE CARAM, Respondent.
G.R. No. 158805
April 16, 2009
FACTS:
Valley Golf & Country Club (Valley Golf) is a duly constituted non-stock,
non-profit corporation which operates a golf course. The members and their
guests are entitled to play golf on the said course and otherwise avail of the
facilities and privileges provided by Valley Golf. The shareholders are likewise
assessed monthly membership dues.In 1961, the late Congressman Fermin
Z. Caram, Jr. (Caram), the husband of the present respondent, subscribed to
purchased and paid for in full one share (Golf Share) in the capital stock of
Valley Golf. He was issued Stock Certificate No. 389 dated 26 January 1961
for the Golf Share.3 The Stock Certificate likewise indicates a par value
of P9,000.00.
Valley Golf would subsequently allege that beginning 25 January 1980,
Caram stopped paying his monthly dues, which were continually assessed
until 31 June 1987. Valley Golf claims to have sent five (5) letters to Caram
concerning his delinquent account within the period from 27 January 1986
until 3 May 1987.
As it turned out, Caram had died on 6 October 1986. Respondent
initiated intestate proceedings before the Regional Trial Court (RTC) of Iloilo
City, Branch 35, to settle her husbands estate. Unaware of the pending
controversy over the Golf Share, the Caram family and the RTC included the
same as part of Carams estate. The RTC approved a project of partition of
Carams estate on 29 August 1989. The Golf Share was adjudicated to
respondent, who paid the corresponding estate tax due, including that on the
Golf Share.It was only through a letter dated 15 May 1990 that the heirs of
Caram learned of the sale of the Golf Share following their inquiry with Valley
Golf about the share. After a series of correspondence, the Caram heirs were
subsequently informed, in a letter dated 15 October 1990, that they were
entitled to the refund of P11,066.52 out of the proceeds of the sale of the
Golf Share, which amount had been in the custody of Valley Golf since 11
June 1987.

Page 654 of 1072

ISSUE/S:
May a non-stock corporation seize and dispose of the membership
share of a fully-paid member on account of its unpaid debts to the
corporation when it is authorized to do so under the corporate by-laws
but not by the Articles of Incorporation?
RULING:
The by-laws does not provide for a mode of notice to the member
before the board of directors puts up the Golf Share for sale, yet the sale
marks the termination of membership. Whatever semblance of a notice that
is afforded is bare at best, ambiguous at most. The member is entitled to
receive a statement of account every month; however, the mode by which
the member is to receive such notice is not elaborated upon. If the member
fails to pay within 45 days from the due date, Valley Golf is immediately
entitled to have the member "posted as delinquent." While the assignation of
"delinquent status" is evident enough, it is not as clear what the word
"posted" entails. Connotatively, the word could imply the physical posting of
the notice of delinquency within the club premises, such as a bulletin board,
which we recognize is often the case. Still, the actual posting modality is
uncertain from the language of the by-laws.
The moment the member is "posted as delinquent," Valley Golf is
immediately enabled to seize the share and sell the same, thereby
terminating membership in the club. The by-laws does not require any notice
to the member from the time delinquency is posted to the day the sale of the
share is actually held. The setup is to the extreme detriment to the member,
who upon being notified that the lien on his share is due for execution would
be duly motivated to settle his accounts to foreclose such possibility.
Does the Corporation Code permit the termination of membership
without due notice to the member? The Code itself is silent on that matter,
and the argument can be made that if no notice is provided for in the articles
of incorporation or in the by-laws, then termination may be effected without
any notice at all. Support for such an argument can be drawn from our ruling
in Long v. Basa,which pertains to a religious corporation that is also a nonstock corporation. Therein, the Court upheld the expulsion of church
members despite the absence of any provision on prior notice in the by-laws,
stating that the members had "waived such notice by adhering to those bylaws became members of the church voluntarily, entered into its covenant
and subscribed to its rules [and by] doing so, they are bound by their
consent."

Page 655 of 1072

However, a distinction should be made between membership in a


religious corporation, which ordinarily does not involve the purchase of
ownership shares, and membership in a non-stock corporation such as Valley
Golf, where the purchase of an ownership share is a condition sine qua non.
Membership in Valley Golf entails the acquisition of a property right. In turn,
the loss of such property right could also involve the application of aspects of
civil law, in addition to the provisions of the Corporation Code. To put it
simply, when the loss of membership in a non-stock corporation also entails
the loss of property rights, the manner of deprivation of such property right
should also be in accordance with the provisions of the Civil Code.
It has been held that a by-law providing that if a member fails to pay
dues for a year, he shall be deemed to have relinquished his membership
and may be excluded from the rooms of the association and his certificate of
membership shall be sold at auction, and any surplus of the proceeds be
paid over him, does not ipso facto terminate the membership of one whose
dues are a year in arrears; the remedy given for non-payment of dues is not
exclusive because the corporation, so long as he remains a member, may
sue on his agreement and collect them.

Page 656 of 1072

TOPIC: EFFECT OF DELINQUENCY:


CALATAGAN GOLF CLUB, INC. Petitioner,
vs.
SIXTO CLEMENTE, JR., Respondent.
G.R. No. 165443
April 16, 2009
FACTS:
Clemente applied to purchase one share of stock of Calatagan,
indicating in his application for membership his mailing address at "Phimco
Industries, Inc. P.O. Box 240, MCC," complete residential address, office and
residence telephone numbers, as well as the company (Phimco) with which
he was connected, Calatagan issued to him Certificate of Stock No. A-01295
on 2 May 1990 after paying P120,000.00 for the share.
Calatagan charges monthly dues on its members to meet expenses for
general operations, as well as costs for upkeep and improvement of the
grounds and facilities. The provision on monthly dues is incorporated in
Calatagans Articles of Incorporation and By-Laws. It is also reproduced at the
back of each certificate of stock. When Clemente became a member the
monthly charge stood at P400.00. He paid P3,000.00 for his monthly dues on
21 March 1991 and another P5,400.00 on 9 December 1991. Then he ceased
paying the dues. At that point, his balance amounted to P400.00.
Ten (10) months later, Calatagan made the initial step to collect
Clementes back accounts by sending a demand letter dated 21 September
1992. It was followed by a second letter dated 22 October 1992. Both letters
were sent to Clementes mailing address as indicated in his membership
application but were sent back to sender with the postal note that the
address had been closed.
Calatagan declared Clemente delinquent for having failed to pay his
monthly dues for more than sixty (60) days, specifically P5,600.00 as of 31
October 1992. Calatagan also included Clementes name in the list of
delinquent members posted on the clubs bulletin board. On 1 December
1992, Calatagans board of directors adopted a resolution authorizing the
foreclosure of shares of delinquent members, including Clementes; and the
public auction of these shares.

Page 657 of 1072

Clemente learned of the sale of his share only in November of 1997. He


filed a claim with the Securities and Exchange Commission (SEC) seeking the
restoration of his shareholding in Calatagan with damages.
ISSUE/S:
Whether or not the action of Calatagan of subjecting the shares of
stock of clemente to public auction valid due to delinquency of the
latter.
RULING:
Section 69 of the Code provides that an action to recover delinquent
stock sold must be commenced by the filing of a complaint within six (6)
months from the date of sale. As correctly pointed out by the Court of
Appeals, Section 69 is part of Title VIII of the Code entitled "Stocks and
Stockholders" and refers specifically to unpaid subscriptions to capital stock,
the sale of which is governed by the immediately preceding Section 68.
There are fundamental differences that defy equivalence or even
analogy between the sale of delinquent stock under Section 68 and the sale
that occurred in this case. At the root of the sale of delinquent stock is the
non-payment of the subscription price for the share of stock itself. The
stockholder or subscriber has yet to fully pay for the value of the share or
shares subscribed. In this case, Clemente had already fully paid for the share
in Calatagan and no longer had any outstanding obligation to deprive him of
full title to his share. Perhaps the analogy could have been made if Clemente
had not yet fully paid for his share and the non-stock corporation, pursuant
to an article or by-law provision designed to address that situation, decided
to sell such share as a consequence. But that is not the case here, and there
is no purpose for us to apply Section 69 to the case at bar.

Page 658 of 1072

TOPIC:ISSUANCE OF CERTIFICATES OF STOCKS


FUA CUN (alias Tua Cun)
vs.
RICARDO SUMMERS, in his capacity as Sheriff ex-oficio of the City of
Manila, and the CHINA BANKING CORPORATION
G.R. No.L-19441.March 27, 1923
FACTS:
On May 18, 1921, Chua Soco executed a promissory note in favor of
the plaintiff Fua Cun for the sum of P25,000 payable in ninety days and
drawing interest at the rate of 1 per cent per month, securing the note with a
chattel mortgage on the shares of stock subscribed for by Chua Soco, who
also endorsed the receipt above mentioned and delivered it to the
mortgagee. The plaintiff thereupon took the receipt to the manager of the
defendant Bank and informed him of the transaction with Chua Soco, but
was told to await action upon the matter by the Board of Directors.
In the meantime Chua Soco appears to have become indebted to the
China Banking Corporation in the sum of P37,731.68 for dishonored
acceptances of commercial paper and in an action brought against him to
recover this amount, Chua Socos interest in the five hundred shares
subscribed for was attached and the receipt seized by the sheriff. The
attachment was levied after the defendant bank had received notice of the
facts that the receipt had been endorsed over to the plaintiff.
Fua Cun thereupon brought the present action maintaining that by
virtue of the payment of the one-half of the subscription price of five hundred
shares Chua Soco in effect became the owner of two hundred and fifty
shares and praying that his, the plaintiffs, lien on said shares, by virtue of
the chattel mortgage, be declared to hold priority over the claim of the
defendant Banking Corporation; that the defendants be ordered to deliver
the receipt in question to him; and that he be awarded the sum of P5,000 in
damages for wrongful attachment.
ISSUE/S:
Whether or not plaintiff has a right to the stock in question

Page 659 of 1072

RULING:
In the absence of special agreement to the contrary, a subscriber for a
certain number of shares of stock does not, upon payment of one-half of the
subscription price, become entitled to the issuance of certificates for one-half
of the number of shares subscribed for; the subscribers right consists only in
equity entitling him to a certificate for the total number of shares subscribed
for by him upon payment of the remaining portion of the subscription price.
It is argued that the interest held by Chua Soco was merely an equity which
could not be made the subject of a chattel mortgage. Though the courts
have uniformly held that chattel mortgages on shares of stock and other
choses in action are valid as between the parties, there is still much to be
said in favor of the defendants contention that the chattel mortgage here in
question would not prevail over liens of third parties without notice; an
equity in shares of stock is of such an intangible character that it is
somewhat difficult to see how it can be treated as a chattel and mortgaged
in such a manner that the recording of the mortgage will furnish constructive
notice to third parties.
There can be no doubt that an equity in shares of stock may be
assigned and that the assignment is valid as between the parties and as to
persons to whom notice is brought home. Such an assignment exists here,
though it was made for the purpose of securing a debt.

Page 660 of 1072

TOPIC:ISSUANCE OF CERTIFICATES OF STOCKS


IRINEO S. BALTAZAR, plaintiff-appellee,
vs.
LINGAYEN GULF ELECTRIC POWER, CO., INC., DOMINADOR C.
UNGSON, BRIGIDO G. ESTRADA, MANUEL L. FERNANDEZ, BENEDICTO
C. YUSON and BERNARDO ACENA, defendants-appellants.
G.R. No.L-16237.June 30, 1965
14 SCRA 522
FACTS:
The Lingayen Gulf Electric Power Co., Inc., was doing business in the
Philippines, with principal offices at Lingayen, Pangasinan, and with an
authorized capital stock of P300.000.00 divided into 3,000 shares of voting
stock at P100.00 par value, per share. Plaintiffs Baltazar and Rose were
among the incorporators, having subscribed to 600 and 400 shares of the
capital stock, or a total par value of P60,000.00 and P40.000.00, respectively.
It is alleged that it has always been the practice and procedure of the
Corporation to issue certificates of stock to its individual subscribers for
unpaid shares of stock. Of the 600 shares of capital stock subscribed by
Baltazar, he had fully paid 535 shares of stock, and the Corporation issued to
him several fully paid up and non-assessable certificates of stock,
corresponding to the 535 shares. After having made transfers to third
persons and acquired new ones, Baltazar had to his credit, on the filing of the
complaint 341 shares fully paid and non-assessable. He had also 65 shares
with par value of P6,500.00, for which no certificate was issued to him. Of
the 400 shares of stock subscribed by Rose, he had 375 shares of fully paid
stock, duly covered by certificates of stock issued to him.
The date of the annual stockholders' meeting of the Corporation had
been fixed, under its by-laws, on the first Tuesday of February of every year,
but for one reason or another, the meeting was to be held on May 1, 1955,
principally for the purpose of electing new officers and Board of Directors for
the calendar year 1955. In connection with said meeting since January 1,
1955, there was a realignment effected, and the fight for control of the
management and property of the corporation was close and keen. The total

Page 661 of 1072

number of fully paid-up shares held by stockholders of one group, was


almost equal the number of fully paid-up shares held by the other group.
The Ungson group (specially defendant Acena), which had been in
complete control of the management and property of the Corporation since
January 1, 1955, in order to continue retaining such control, over the
objection oil three majority members of the Board, in the regular meeting of
the Board of Directors, held on January 30, 1955, passed three (3) resolutions
On the authority of these resolutions, the Ungson group was
threatening and procuring to expel and oust the plaintiffs and their
companion stockholders, for the ultimate purpose of depriving them of their
right to vote in the said annual stockholders' meeting scheduled for May 1,
1955.
ISSUE/S:
Whether or not a corporation may validly condone interest on unpaid
subscriptions to its capital stock
RULING:
In the present case, the defendant-corporation had applied the
payments made by the stockholders to the full par value of the shares of
stock subscribed by them, instead of the accepted interest, as shown by the
capital stock shares certificate issued for the payments made, and the
stockholders had accepted such certificates issued for such payments. This
being the case, the said application of payments must be deemed to have
been agreed upon by the Corporation and the stockholders, and the same
cannot now be changed without the consent of the stockholders concerned.
The Corporation Law and the by-laws of the defendant Corporation do not
contain any provision, prohibiting the application of stockholders' payments
to the full par value of a corporation's capital stock, ahead of the payment of
accrued interest for unpaid subscriptions. It would, therefore, result that a
corporation may, upon request of an interested stockholder, as his option,
apply payment by them to the full par value of shares of capital leaving its
collection later of the accrued interest on unpaid subscriptions, and that once
such option has been exercised and the corresponding stock certificates
have been issued, the corporation cannot, by a unilateral act, legally nullify
and cancel the capital stock certificates so issued.

Page 662 of 1072

TOPIC:ISSUANCE OF CERTIFICATES OF STOCKS


ALFONSO S. TAN
vs.
SECURITIES AND EXCHANGE COMMISSION, VISAYAN EDUCATIONAL
SUPPLY CORP., TAN SU CHING, ALFREDO B. UY, ANGEL S. TAN and
PATRICIA AGUILAR
G.R. No. 95696. March 3, 1992
FACTS:
Respondent corporation was registered on October 1, 1979. As
incorporator, petitioner had four hundred (400) shares of the capital stock
standing in his name at the par value of P100.00 per share, evidenced by
Certificate of Stock No. 2. He was elected as President and subsequently
reelected, holding the position as such until 1982 but remained in the Board
of Directors until April 19, 1983 as director.
On January 31, 1981, while petitioner was still the president of the
respondent corporation, two other incorporators, namely, Antonia Y. Young
and Teresita Y. Ong, assigned to the corporation their shares, represented by
certificate of stock No. 4 and 5 after which, they were paid the corresponding
40% corporate stock-in-trade.
Petitioner's certificate of stock No. 2 was cancelled by the corporate
secretary and respondent Patricia Aguilar by virtue of Resolution No. 1981
(b), which was passed and approved while petitioner was still a member of
the Board of Directors of the respondent corporation.
Due to the withdrawal of the aforesaid incorporators and in order to
complete the membership of the five (5) directors of the board, petitioner
sold fifty (50) shares out of his 400 shares of capital stock to his brother
Angel S. Tan. Another incorporator, Alfredo B. Uy, also sold fifty (50) of his
400 shares of capital stock to Teodora S. Tan and both new stockholders
attended the special meeting, Angel Tan was elected director and on March
27, 1981, the minutes of said meeting was filed with the SEC. These facts
stand unchallenged.

Page 663 of 1072

With the cancellation of Certificate of stock No. 2 and the subsequent


issuance of Stock Certificate No. 6 in the name of Angel S. Tan and for the
remaining 350 shares, Stock Certificate No. 8 was issued in the name of
petitioner Alfonso S. Tan, Mr. Buzon, submitted an Affidavit.
On January 29, 1983, during the annual meeting of the corporation,
respondent Tan Su Ching was elected as President while petitioner was
elected as Vice-president. He, however, did not sign the minutes of said
meeting which was submitted to the SEC on March 30, 1983.
When petitioner was dislodged from his position as president, he withdrew
from the corporation on February 27, 1983, on condition that he be paid with
stocks-in-trade equivalent to 33.3% in lieu of the stock value of his shares in
the amount of P35,000.00.
ISSUE/S:
Whether or not the conversion of the 350 shares with a par value of
only P35,000.00 at P100.00 per share into treasury stocks after
petitioner exchanged them with P2,000,000.00 worth of stocks-in-trade
of the corporation, is valid and lawful.
RULING:
The court holds that the conversion of the 350 shares with a par value
of only P35,000.00 at P100.00 per share into treasury stocks after petitioner
exchanged them with P2,000,000.00 worth of stocks-in-trade of the
corporation, is valid and lawful. Under the instant case, the fact of the matter
is, the new holder, Angel S. Tan has already exercised his rights and
prerogatives as stockholder and was even elected as member of the board of
directors in the respondent corporation with the full knowledge and
acquiescence of petitioner. Due to the transfer of fifty (50) shares, Angel S.
Tan was clothed with rights and responsibilities in the board of the
respondent corporation when he was elected as officer thereof.
Besides, in Philippine jurisprudence, a certificate of stock is not a
negotiable instrument. "Although it is sometime regarded as quasinegotiable, in the sense that it may be transferred by endorsement, coupled
with delivery, it is well-settled that it is non-negotiable, because the holder
thereof takes it without prejudice to such rights or defenses as the registered
owner/s or transferror's creditor may have under the law, except insofar as
such rights or defenses are subject to the limitations imposed by the
principles governing estoppel."
Furthermore, there is a necessity to delineate the function of the stock
itself from the actual delivery or endorsement of the certificate of stock itself
as is the question in the instant case. A certificate of stock is not necessary
to render one a stockholder in corporation.

Page 664 of 1072

Nevertheless, a certificate of stock is the paper representative or


tangible evidence of the stock itself and of the various interests therein. The
certificate is not stock in the corporation but is merely evidence of the
holder's interest and status in the corporation, his ownership of the share
represented thereby, but is not in law the equivalent of such ownership. It
expresses the contract between the corporation and the stockholder, but is
not essential to the existence of a share in stock or the nation of the relation
of shareholder to the corporation.

Page 665 of 1072

TOPIC:ISSUANCE OF CERTIFICATES OF STOCKS


EMBASSY FARMS, INC.
vs.
HON. COURT OF APPEALS (INTERMEDIATE APPELLATE COURT), HON.
ZENAIDA S. BALTAZAR, Judge of the Regional Trial Court, Branch
CLVIII, (158), Pasig, Metro Manila, VOLTAIRE B. CRUZ, Deputy Sheriff,
Branch CLVIII, Regional Trial Court, Pasig, Metro Manila and
EDUARDO B. EVANGELISTA
G.R. No. 80682. August 13, 1990
188 S 492
FACTS:
On August 2, 1984, Alexander G. Asuncion and Eduardo B. Evangelists
entered into a Memorandum of Agreement. Under said agreement EBE
obligated himself to transfer to AGA 19 parcels of agricultural land registered
in his name with an aggregate area of 104,447 square meters, together with
the stocks, equipment and facilities of a piggery farm owned by Embassy
Farms, Inc., a registered corporation wherein ninety) per cent of its shares of
stock is owned by EBE. EBE also obligated himself to cede, transfer and
convey "in a manner absolute and irrevocable any and all of his shares of
stocks" in Embassy Farins Inc. to AGA or his nominees "until the total of said
shares of stock so transferred shall constitute 90% of the paid-in-equity of
said corporation" within a reasonable time from signing of the document.
Likewise, EBE obligated to turnover to AGA the effective control and
management of the piggery upon the signing of the agreement.
On the other hand, AGA obligated himself, upon signing of the
agreement to pay to EBE the total sum of close to P8,630,000.00. Within
reasonable time from signing of the agreement AGA obligated himself to
organize and register a new corporation with an authorized capital stock of
P10,000,000.00 which upon registration will take over all the rights and
liabilities of AGA.
For failure to comply with his obligations, EBE intimated the institution
of appropriate legal action.
ISSUE/S:
Whether or not the shares of stock AGA can be effectively transfer to
other person or his nominees the undelivered shares of stock.

Page 666 of 1072

RULING:
From the pleadings submitted by the parties it is clear that although
EBE has indorsed in blank the shares outstanding in his name he has not
delivered the certificate of stocks to AGA because the latter has not fully
complied with his obligations under the memorandum of agreement. There
being no delivery of the indorsed shares of stock AGA cannot therefore
effectively transfer to other person or his nominees the undelivered shares of
stock. For an effective transfer of shares of stock the mode and manner of
transfer as prescribed by law must be followed. As provided under Section 3
of Batas Pambansa Bilang 68, otherwise known as the Corporation Code of
the Philippines, shares of stock may be transferred by delivery to the
transferree of the certificate properly indorsed. Title may be vested in the
transferree by the delivery of the duly indorsed certificate of stock. However,
no transfer shall be valid, except as between the parties until the transfer is
properly recorded in the books of the corporation.
In the case at bar the indorsed certificate of stock was not actually
delivered to AGA so that EBE is still the controlling stockholder of Embassy
Farms despite the execution of the memorandum of agreement and the turn
over of control and management of the Embassy Farms to AGA on August 2,
1984.
When AGA filed on April 10, 1986 an action for the rescission of
contracts with damages the Pasig Court merely restored and established the
status quo prior to the execution of the memorandum of agreement by the
issuance of a restraining order on July 10, 1987 and the writ of preliminary
injunction on July 30, 1987. It would be unjust and unfair to allow AGA and
his nominees to control and manage the Embassy Farms despite the fact that
AGA who is the source of their supposed shares of stock in the corporation is
not asking for the delivery of the indorsed certificate of stock but for the
rescission of the memorandum of agreement. Rescission would result in
mutual restitution so it is but proper to allow EBE to manage the farm.
Compared to AGA or his nominees EBE would be more interested in the
preservation of the assets, equipment and facilities of Embassy Farms during
the pendency of the main case.

Page 667 of 1072

TOPIC: RIGHT TO TRANSFER OF SHARES/VALIDITY OF RESTRICTIONS ON


RIGHT
HENRY FLEISCHER, plaintiff-appellee,
vs.
BOTICA NOLASCO CO., INC., defendant-appellant.
G.R. No. L-23241
March 14, 1925
FACTS:
On November 15, 1923, the plaintiff filed an amended complaint
against the Botica Nolasco, Inc., alleging that he became the owner of five
shares of stock of said corporation, by purchase from their original owner,
one Manuel Gonzalez; that the said shares were fully paid; and that the
defendant refused to register said shares in his name in the books of the
corporation in spite of repeated demands to that effect made by him upon
said corporation, which refusal caused him damages amounting to P500.
Plaintiff prayed for a judgment ordering the Botica Nolasco, Inc. to register in
his name in the books of the corporation the five shares of stock recorded in
said books in the name of Manuel Gonzalez, and to indemnify him in the sum
of P500 as damages, and to pay the costs.
The defendant again filed a demurrer on the ground that the amended
complaint did not state facts sufficient to constitute a cause of action, and
that said amended complaint was ambiguous, unintelligible, uncertain, which
demurrer was overruled by the court. The defendant answered the amended
complaint denying generally and specifically each and every one of the
material allegations thereof, and, as a special defense, alleged that the
defendant, pursuant to article 12 of its by-laws, had preferential right to buy
from the plaintiff said shares at the par value of P100 a share, plus P90 as
dividends corresponding to the year 1922, and that said offer was refused by
the plaintiff. The defendant prayed for a judgment absolving it from all
liability under the complaint and directing the plaintiff to deliver to the
defendant the five shares of stock in question, and to pay damages in the
sum of P500, and the costs.
ISSUE/S:
Whether or not the defendant can claim a preferential right to buy the
shares of stock subject of the case.

Page 668 of 1072

RULING:
Section 13, paragraph 7, empowers a corporation to make by-laws, not
inconsistent with any existing law, for the transferring of its stock. It follows
from said provision that a by-law adopted by a corporation relating to
transfer of stock should be in harmony with the law on the subject of transfer
of stock. The law on this subject is found in section 35 of Act No. 1459 above
quoted. Said section specifically provides that the shares of stock "are
personal property and may be transferred by delivery of the certificate
indorsed by the owner, etc." Said section 35 defines the nature, character
and transferability of shares of stock. Under said section they are personal
property and may be transferred as therein provided. Said section
contemplates no restriction as to whom they may be transferred or sold. It
does not suggest that any discrimination may be created by the corporation
in favor or against a certain purchaser. The holder of shares, as owner of
personal property, is at liberty, under said section, to dispose of them in
favor of whomsoever he pleases, without any other limitation in this respect,
than the general provisions of law. Therefore, a stock corporation in adopting
a by-law governing transfer of shares of stock should take into consideration
the specific provisions of section 35 of Act No. 1459, and said by-law should
be made to harmonize with said provisions. It should not be inconsistent
therewith.
The by-law now in question was adopted under the power conferred
upon the corporation by section 13, paragraph 7, above quoted; but in
adopting said by-law the corporation has transcended the limits fixed by law
in the same section, and has not taken into consideration the provisions of
section 35 of Act No. 1459.
As a general rule, the by-laws of a corporation are valid if they are
reasonable and calculated to carry into effect the objects of the corporation,
and are not contradictory to the general policy of the laws of the land.
(Supreme Commandery of the Knights of the Golden Rule vs. Ainsworth, 71
Ala., 436; 46 Am. Rep., 332.)
The only restraint imposed by the Corporation Law upon transfer of
shares is found in section 35 of Act No. 1459, quoted above, as follows: "No
transfer, however, shall be valid, except as between the parties, until the
transfer is entered and noted upon the books of the corporation so as to
show the names of the parties to the transaction, the date of the transfer,
the number of the certificate, and the number of shares transferred." This
restriction is necessary in order that the officers of the corporation may know
who are the stockholders, which is essential in conducting elections of
officers, in calling meeting of stockholders, and for other purposes. but any
restriction of the nature of that imposed in the by-law now in question,

Page 669 of 1072

is ultra vires, violative of the property rights of shareholders, and in restraint


of trade.
And moreover, the by-laws now in question cannot have any effect on
the appellee. He had no knowledge of such by-law when the shares were
assigned to him. He obtained them in good faith and for a valuable
consideration. He was not a privy to the contract created by said by-law
between the shareholder Manuel Gonzalez and the Botica Nolasco, Inc. Said
by-law cannot operate to defeat his rights as a purchaser.

Page 670 of 1072

TOPIC: RIGHT TO TRANSFER OF SHARES/VALIDITY OF RESTRICTIONS ON


RIGHT
CYRUS PADGETT, plaintiff-appellee,
vs.
BABCOCK & TEMPLETON, INC., and W. R. BABCOCK, defendantsappellants.
G.R. No. L-38684
December 21, 1933
FACTS:
The appellee was an employee of the appellant corporation and
rendered services as such from January 1, 1923, to April 15, 1929. During
that period he bought 35 shares thereof at P100 a share at the suggestion of
the president of said corporation. He was also the recipient of 9 shares by
way of bonus during Christmas seasons. In this way the said appellee
became the owner of 44 shares for which the 12 certificates were issued in
his favor.
The word "nontransferable" appears on each and every one of these
certificates. Before severing his connections with the said corporation, the
appellee proposed to the president that the said corporation buy his 44
shares at par value plus the interest thereon, or that he be authorized to sell
them to other persons. The corporation bought similar shares belonging to
other employees, at par value. Sometime later, the said president offered to
buy the appellee's shares first at P85 each and then at P80. The appellee did
not agree thereto.
ISSUE/S:
Whether or not the defendant are obliged to buy his shares of stock
value at par value.
RULING:
There is no existing law nor authority in support of the plaintiff's claim
to the effect that the defendants are obliged to buy his shares of stock value

Page 671 of 1072

at par value, plus the interest demanded thereon. In this respect, we hold
that there has been no such contract, either express or implied, between the
plaintiff and the defendants. In the absence of a similar contractual
obligation and of a legal provision applicable thereto, it is logical to conclude
that it would be unjust and unreasonable to compel the said defendants to
comply with a non-existent or imaginary obligation. Whereupon, we are
likewise compelled to conclude that the judgment originally rendered to that
effect is untenable and should be set aside.

Page 672 of 1072

TOPIC: RIGHT TO TRANSFER OF SHARES/VALIDITY OF RESTRICTIONS ON


RIGHT
RURAL BANK OF SALINAS, INC., MANUEL SALUD, LUZVIMINDA TRIAS
and FRANCISCO TRIAS, petitioners,
vs.
COURT OF APPEALS*, SECURITIES AND EXCHANGE COMMISSION,
MELANIA A. GUERRERO, LUZ ANDICO, WILHEMINA G. ROSALES,
FRANCISCO M. GUERRERO, JR., and FRANCISCO GUERRERO ,
SR.,respondents.
G.R. No. 96674 June 26, 1992
FACTS:
On June 10, 1979, Clemente G. Guerrero, President of the Rural Bank of
Salinas, Inc., executed a Special Power of Attorney in favor of his wife,
private respondent Melania Guerrero, giving and granting the latter full
power and authority to sell or otherwise dispose of and/or mortgage 473
shares of stock of the Bank registered in his name (represented by the
Bank's stock certificates nos. 26, 49 and 65), to execute the proper
documents therefore, and to receive and sign receipts for the dispositions.
On February 27, 1980, and pursuant to said Special Power of Attorney,
private respondent Melania Guerrero, as Attorney-in-Fact, executed a Deed
of Assignment for 472 shares out of the 473 shares, in favor of private
respondents Luz Andico (457 shares), Wilhelmina Rosales (10 shares) and
Francisco Guerrero, Jr. (5 shares).Almost four months later, or two (2) days
before the death of Clemente Guerrero on June 24, 1980, private respondent
Melania Guerrero, pursuant to the same Special Power of Attorney, executed
a Deed of Assignment for the remaining one (1) share of stock in favor of
private respondent Francisco Guerrero, Sr.
Subsequently, private respondent Melania Guerrero presented to
petitioner Rural Bank of Salinas the two (2) Deeds of Assignment for
registration with a request for the transfer in the Bank's stock and transfer
book of the 473 shares of stock so assigned, the cancellation of stock
certificates in the name of Clemente G. Guerrero, and the issuance of new
stock certificates covering the transferred shares of stocks in the name of
the new owners thereof. However, petitioner Bank denied the request of
respondent Melania Guerrero.

Page 673 of 1072

ISSUE/S:
Whether or not the respondent court erred in sustaining the Securities
and Exchange Commission when it compelled by Mandamus the Rural
Bank of Salinas to register in its stock and transfer book the transfer of
473 shares of stock to private respondents.
RULING:
The Supreme Court rules in favor of the respondents. Section 5 (b) of
P.D. No. 902-A grants to the SEC the original and exclusive jurisdiction to
hear and decide cases involving intracorporate controversies. An
intracorporate controversy has been defined as one which arises between a
stockholder and the corporation. There is no distinction, qualification, nor
any exception whatsoever (Rivera vs. Florendo, 144 SCRA 643 [1986]). The
case at bar involves shares of stock, their registration, cancellation and
issuances thereof by petitioner Rural Bank of Salinas. It is therefore within
the power of respondent SEC to adjudicate.
Respondent SEC correctly ruled in favor of the registering of the shares
of stock in question in private respondent's names. Such ruling finds support
under Section 63 of the Corporation Code, to wit: Sec. 63. . . . Shares of stock
so issued are personal property and may be transferred by delivery of the
certificate or certificates indorsed by the owner or his attorney-in-fact or
other person legally authorized to make the transfer. No transfer, however,
shall be valid, except as between the parties, until the transfer is recorded in
the books of the corporation . . .
A corporation, either by its board, its by-laws, or the act of
its officers, cannot create restrictions in stock transfers,
because:. . . Restrictions in the traffic of stock must have
their source in legislative enactment, as the corporation
itself cannot create such impediment. By-laws are intended
merely for the protection of the corporation, and prescribe
regulation, not restriction; they are always subject to the
charter of the corporation. The corporation, in the absence
of such power, cannot ordinarily inquire into or pass upon
the legality of the transactions by which its stock passes
from one person to another, nor can it question the
consideration upon which a sale is based. . . . (Tomson on
Corporation Sec. 4137, citedin Fleisher vs. Nolasco, Supra).
The right of a transferee/assignee to have stocks transferred to his
name is an inherent right flowing from his ownership of the stocks. Thus:
Whenever a corporation refuses to transfer and register stock in cases like
the present, mandamus will lie to compel the officers of the corporation to
transfer said stock in the books of the corporation" (26, Cyc. 347, Hyer vs.
Bryan, 19 Phil. 138; Fleisher vs. Botica Nolasco, 47 Phil. 583, 594).

Page 674 of 1072

WHEREFORE, the petition is DISMISSED for lack of merit.

TOPIC: RIGHT TO TRANSFER OF SHARES/VALIDITY OF RESTRICTIONS ON


RIGHT
MARSH THOMSON, petitioner,
vs.
COURT OF APPEALS and THE AMERICAN CHAMPER OF COMMERCE OF
THE PHILIPPINES, INC,respondents.
G.R. No. 116631 October 28, 1998
FACTS:
While petitioner was still working with private respondent, his superior,
A. Lewis Burridge, retired as AmCham's President. Before Burridge decided to
return to his home country, he wanted to transfer his proprietary share in the
Manila Polo Club (MPC) to petitioner. However, through the intercession of
Burridge, private respondent paid for the share but had it listed in
petitioner's name. This was made clear in an employment advice dated
January 13, 1986, wherein petitioner was informed by private respondent.
On April 25, 1986, Burridge transferred said proprietary share to
petitioner, as confirmed in a letter 3 of notification to the Manila Polo Club.
Upon his admission as a new member of the MPC, petitioner paid the transfer
fee of P40,000.00 from his own funds; but private respondent subsequently
reimbursed this amount. On November 19, 1986, MPC issued Proprietary
Membership Certificate Number 3398 in favor of petitioner. But petitioner,
however, failed to execute a document recognizing private respondent's
beneficial ownership over said share.
When petitioner's contract of employment was up for renewal in 1989,
he notified private respondent that he would no longer be available as
Executive Vice President after September 30, 1989. Still, the private
respondent asked the petitioner to stay on for another six (6) months.
Petitioner indicated his acceptance of the consultancy arrangement with a
counter-proposal in his letter dated October 8, 1989. Private respondent
rejected petitioner's counter-proposal.

Page 675 of 1072

On April 5, 1990, private respondent, through counsel sent a letter to


the petitioner demanding the return and delivery of the MPC share which "it
(AmCham) owns and placed in your (Thomson's) name." 11
Failing to get a favorable response, private respondent filed on May 15,
1990, a complaint against petitioner praying, inter alia, that the Makati
Regional Trial Court render judgment ordering Thomson "to return the Manila
Polo Club share to the plaintiff and transfer said share to the nominee of
plaintiff."
ISSUE/S:
Did respondent court err in holding that private respondent is the
beneficial owner of the disputed share?
RULING:
The beneficiary of a trust has beneficial interest in the trust property,
while a creditor has merely a personal claim against the debtor. In trust,
there is a fiduciary relation between a trustee and a beneficiary, but there is
no such relation between a debtor and creditor. While a debt implies merely
an obligation to pay a certain sum of money, a trust refers to a duty to deal
with a specific property for the benefit of another. If a creditor-debtor
relationship exists, but not a fiduciary relationship between the parties, there
is no express trust. However, it is understood that when the purported
trustee of funds is entitled to use them as his or her own (and commingle
them with his or her own money), a debtor-creditor relationship exists, not a
trust.
In the present case, as the Executive Vice-President of AmCham,
petitioner occupied a fiduciary position in the business of AmCham. AmCham
released the funds to acquire a share in the Club for the use of petitioner but
obliged him to "execute such document as necessary to acknowledge
beneficial ownership thereof by the Chamber". A trust relationship is,
therefore, manifestly indicated.
Moreover, petitioner failed to present evidence to support his
allegation of being merely a debtor when the private respondent paid the
purchase price of the MPC share. Applicable here is the rule that a trust
arises in favor of one who pays the purchase money of property in the name
of another, because of the presumption that he who pays for a thing intends
a beneficial interest therein for himself.
Although petitioner initiated the acquisition of the share, evidence on
record shows that private respondent acquired said share with its funds.

Page 676 of 1072

Petitioner did not pay for said share, although he later wanted to, but
according to his own terms, particularly the price thereof.

Page 677 of 1072

TOPIC: RIGHT TO TRANSFER OF SHARES/VALIDITY OF RESTRICTIONS ON


RIGHT
ENRIQUE T. YUCHENGCO, INC., A. T. YUCHENGCO, INC., ANNABELLE Y.
PUEY and MONA LISA Y. ABAYA,plaintiffs-appellees,
vs.
CONRADO M. VELAYO, defendant-appellant.
G.R. No. L-50439 July 20, 1982
FACTS:
The records show that sometime in June 1974, defendant-appellant
Conrado M. Velayo offered to sell to the plaintiffs-appellees 2,265 shares of
common stock of the RIC Tours Philippines, Inc. ("Ric Tours Phil., for short) a
Philippine Corporation then duly licensed as a tourist operator, constituting
70% of the subscribed and outstanding capital stock of the said corporation.
Appellees paid the entire purchase price of P367,500.00 to appellant Velayo,
and the latter, on his part, delivered to the former all the 2,265 shares of
stock of Ric Tours Phil. On July 5, 1974, appellees took possession of all the
assets of Ric Tours Phil.
On September 3, 1974, appellees wrote a letter to appellant
demanding rescission of the contract, the restitution of the sum of
P367,500.00 representing the purchase price of 2,265 shares of Ric Tours
Phil. and offering delivery of the certificates of stock representing the 2,265
shares upon receipt of the P367,500.00.
On April 4, 1975, appellees brought a complaint before the Court of
First Instance of Rizal (Pasig Branch) praying for the annulment of contract,
the return of P367,500.00 plus interest and the payment of damages. On
October 4, 1976, the parties agreed to submit the case for decision on the
basis of their Joint Stipulation of Facts wherein they admitted the facts set
forth above, and the pleadings on record without the necessity of oral
evidence. On March 17, 1977, the trial court rendered its decision favorable
to appellees. Hence, this appeal.
ISSUE/S:

Page 678 of 1072

whether or not the "Stock Purchase Agreement" entered into by the


appellees and appellant Velayo should be annulled, or in the
alternative, declared void ab initio.
RULING:
Appellant contends that appellees have no right to rescind the contract
since the ownership of Ric Tours Phil. and its license to operate as tour
operator were transferred to appellees upon delivery to them of all the
shares of stock together with all the other assets of the corporation:
consequently under the doctrine of res suo domino perit, appellees as buyers
in a consummated sale should suffer the loss when the license was
cancelled.
Appellant's contention is devoid of merit. The provision governing the
Agreement sought to be annuled is Sec 4, Part IV of the Rules and
Regulations Governing the Business of Tour Operators and Tour Guides, which
recites as follows: Sec. 4. No transfer of rights to a license of a tour operator
or ownership of shares or interests in the agency shall be valid unless made
with the prior approval of the Department. (Emphasis supplied).
The above-quoted rule is clear and mandatory. It requires the prior
approval of the Department of Tourism for the validity of any transfer of
rights to a license of a tour operator or ownership of shares or interests in
any tour agency. In the case at bar, it was admitted by both parties, that the
Stock Purchase Agreement was made without the prior approval of the
Department of Tourism. Pursuant to paragraph 7, article 1409 of the New
Civil Code, such agreement would be inexistent and null and void from the
beginning. For it is well-settled that any contract entered into must be in
accordance with, and not repugnant to, an applicable statute whose terms
are deemed embodied therein 2 and without the need for the parties of
expressly making reference to it. 3 Inasmuch as the agreement between the
parties is null and void from the beginning, it produces no legal effect. No
valid transfer of ownership of Ric Tours Phil., to the appellees, therefore, took
place upon delivery to them by the appellant of the shares of stock of said
corporation as to make them suffer the consequence of the subsequent
revocation by the Department of Tourism of the license of Ric Tours Phil., as
they would indeed suffer much loss after parting with their money for which
they would receive nothing. The doctrine of res suo domino perit advanced
by the defendant cannot, therefore, be applied.
From the foregoing, We hold that the transfer to plantiffs-appellees of
rights of ownership of shares of stock of Ric Tours Phil. is void, and since the
agreement concerning the sale or transfer legally did not exist, appellant has
no title whatsoever to the money received by him by virtue thereof, which he

Page 679 of 1072

and should accordingly refund to appellees, with interest thereon at the legal
rate until filly paid. 8

TOPIC: RIGHT TO TRANSFER OF SHARES/VALIDITY OF RESTRICTIONS ON


RIGHT
LIM TAY, petitioner,
vs.
COURT OF APPEALS, GO FAY AND CO. INC., SY GUIOK, and THE
ESTATE OF ALFONSO LIM, respondents.
G.R. No. 126891 August 5, 1998
FACTS:
On January 8, 1980, Respondent-Appellee Sy Guiok secured a loan from
the [p]petitioner in the amount of P40,000 payable within six (6) months. To
secure the payment of the aforesaid loan and interest thereon, Respondent
Guiok executed a Contract of Pledge in favor of the [p]petitioner whereby he
pledged his three hundred (300) shares of stock in the Go Fay & Company
Inc., Respondent Corporation, for brevitys sake. Respondent Guiok obliged
himself to pay interest on said loan at the rate of 10% per annum from the
date of said contract of pledge. On the same date, Alfonso Sy Lim secured a
loan from the [p]petitioner in the amount of P40,000 payable in six (6)
months. To secure the payment of his loan, Sy Lim executed a Contract of
Pledge covering his three hundred (300) shares of stock in Respondent
Corporation. Under said contract, Sy Lim obliged himself to pay interest on
his loan at the rate of 10% per annum from the date of the execution of said
contract.
However, Respondent Guiok and Sy Lim failed to pay their respective
loans and the accrued interests thereon to the [p]petitioner. In October,
1990, the [p]petitioner filed a Petition for Mandamus against Respondent
Corporation, with the SEC entitled Lim Tay versus Go Fay & Company. Inc.,
SEC Case No. 03894.

ISSUE/S:

Page 680 of 1072

Whether or not Petitioners claims that there was dacion en pago, in


which the shares of stock were deemed sold to petitioner, the
consideration for which was the extinguishment of the loans and the
interests thereon is ]correct.
RULING:
Petitioner initially argued that ownership of the shares pledged had
passed to him, upon Respondents Sy Guiok and Sy Lims failure to pay their
respective loans. But on appeal, petitioner claimed that ownership over the
shares had passed to him, not via the contracts of pledge, but by virtue of
prescription and by respondents subsequent acts which amounted to a
novation of the contracts of pledge. We do not agree.
At the outset, it must be underscored that petitioner did not acquire
ownership of the shares by virtue of the contracts of pledge. Article 2112 of
the Civil Code states: The creditor to whom the credit has not been satisfied
in due time, may proceed before a Notary Public to the sale of the thing
pledged. This sale shall be made at a public auction, and with notification to
the debtor and the owner of the thing pledged in a proper case, stating the
amount for which the public sale is to be held. If at the first auction the thing
is not sold, a second one with the same formalities shall be held; and if at the
second auction there is no sale either, the creditor may appropriate the thing
pledged. In this case he shall be obliged to give an acquittance for his entire
claim.
There is no showing that petitioner made any attempt to foreclose or
sell the shares through public or private auction, as stipulated in the
contracts of pledge and as required by Article 2112 of the Civil Code.
Therefore, ownership of the shares could not have passed to him. The
pledgor remains the owner during the pendency of the pledge and prior to
foreclosure and sale, as explicitly provided by Article 2103 of the same Code:
Unless the thing pledged is expropriated, the debtor continues to be the
owner thereof.
Neither did petitioner acquire the shares by virtue of a novation of the
contract of pledge. Novation is defined as the extinguishment of an
obligation by a subsequent one which terminates it, either by changing its
object or principal conditions, by substituting a new debtor in place of the old
one, or by subrogating a third person to the rights of the creditor. Novation
of a contract must not be presumed. In the absence of an express
agreement, novation takes place only when the old and the new obligations
are incompatible on every point.

Page 681 of 1072

TOPIC:TRANSFER OF SHARES OF STOCK AND REGISTRATION


IMELDA O. COJUANGCO, PRIME HOLDINGS, INC., AND THE ESTATE OF
RAMON U. COJUANGCO Petitioners,
vs.
SANDIGANBAYAN, REPUBLIC OF THE PHILIPPINES, AND THE SHERIFF
OF SANDIGANBAYAN, Respondents.
G.R. No. 183278.April 24, 2009
FACTS:
On July 16, 1987, respondent Republic of the Philippines (Republic)
filed before the Sandiganbayan a "Complaint for Reconveyance, Reversion,
Accounting, Restitution and Damages," docketed as Civil Case 0002, praying
for the recovery of alleged ill-gotten wealth from the late President Marcos
and former First Lady Imelda Marcos and their cronies, including some 2.4
million shares of stock in the Philippine Long Distance Telephone Company
(PLDT).
The complaint, which was later amended to implead herein petitioners
Ramon and Imelda Cojuangco (the Cojuangcos), alleged that the Marcoses
ill-gotten wealth included shares in the PLDT covered by shares of stock in
the Philippine Telecommunications Investment Corporation (PTIC), registered
in the name of Prime Holdings, Inc. (Prime Holdings).
ISSUE/S:
Whether the Republic, having transferred the shares to a third party, is
entitled to the dividends, interests, and earnings thereof.
RULING:

Page 682 of 1072

This Court, in directing the reconveyance to the Republic of the


111,415 shares of PLDT stock owned by PTIC in the name of Prime Holdings,
declared the Republic as theowner of said shares and, necessarily, the
dividends and interests accruing thereto.
Dividends are payable to the stockholders of record as of the date of
the declaration of dividends or holders of record on a certain future date, as
the case may be, unless the parties have agreed otherwise. And a transfer of
shares which is not recorded in the books of the corporation is valid only as
between the parties, hence, the transferor has the right to dividends as
against the corporation without notice of transfer but it serves as trustee of
the real owner of the dividends, subject to the contract between the
transferor and transferee as to who is entitled to receive the dividends.
It is thus clear that the Republic is entitled to the dividends accruing
from the subject 111,415 shares since 1986 when they were sequestered up
to the time they were transferred to Metro Pacific via the Sale and Purchase
Agreement of February 28, 2007; and that the Republic has since the latter
date been serving as trustee of those dividends for the Metro Pacific up to
the present, subject to the terms and conditions of the said agreement they
entered into.
WHEREFORE, the petition is DENIED. The challenged Resolutions dated
November 7, 2007 and June 13, 2008 of the Sandiganbayan in Civil Case No.
0002 are, in light of the foregoing, AFFIRMED.

Page 683 of 1072

TOPIC:TRANSFER OF SHARES OF STOCK AND REGISTRATION

Republic of the Philippines (Presidential Commission on Good


Government)
vs.
Sandiganbayan
[GR 107789, 30 April 2003]
FACTS:
On
7
August
1991,
the
PCGG
conducted
an
Eastern
Telecommunications, Philippines, Inc. (ETPI) stockholders meeting during
which a PCGG controlled board of directors was elected. A special
stockholders meeting was later convened by the registered ETPI stockholders
wherein another set of board of directors was elected, as a result of which
two sets of such board and officers were elected. Victor Africa, a stockholder
of ETPI, alleging that the PCGG had since been "illegally 'exercising' the
rights of stockholders of ETPI," especially in the election of the members of
the board of directors, filed a motion before the Sandiganbayan, prayed that
said court order the "calling and holding of the ETPI annual stockholders
meeting for 1992 under the court's control and supervision and prescribed
guidelines."
The PCGG did not object to Africa's motion provided that "(1) An Order
be issued upholding the right of PCGG to vote all the Class "A" shares of ETPI;
(2) In the alternative, in the remote event that PCGG's right to vote the
sequestered shares be not upheld, an Order be issued (a) disregarding the
Stock and Transfer Book and Booklet of Stock Certificates of ETPI in
determining who can vote the shares in an Annual Stockholders Meeting of
ETPI, (b) allowing PCGG to vote 23.9% of the total subscription in ETPI, and
(c) directing the amendment of the Articles of Incorporation and By-laws of
ETPI providing for the minimum safeguards for the conservation of assets

Page 684 of 1072

prior to the calling of a stockholders meeting. The Sandiganbayan resolved


Africa's motion, ordering the conduct of an annual stockholders meeting of
ETPI, for 1992. Assailing the foregoing resolution, the PCGG filed before the
Supreme Court a petition for Certiorari, Mandamus and Prohibition.
The PCGG-controlled ETPI board of directors thus authorized the ETPI
Chair and Corporate Secretary to call the special stockholders meeting.
Notices were sent to those entitled to vote for a meeting. The meeting was
held as scheduled and the increase in ETPI's authorized capital stock from
P250 Million to P2.6 Billion was "unanimously approved." On 1 April 1997,
Africa filed before the Supreme Court a motion to cite the PCGG "and its
accomplices" in contempt and "to nullify the 'stockholders meeting'
called/conducted by PCGG and its accomplices," he contending that only this
Court, and not the Sandiganbayan, has the power to authorize the PCGG to
call a stockholders meeting and vote the sequestered shares. Further, Africa
alleged that he was not given notice of the meeting, and the PCGG had no
right to vote the sequestered Class "A" shares. A motion for leave to
intervene relative to Africa's "Motion to Cite the PCGG and its Accomplices in
Contempt" was filed by ETPI.
The Supreme Court granted the motion for leave but ETPI never filed
any pleading relative to Africa's motion to cite the PCGG in contempt. By
Resolution of 16 February 2001, the Sandiganbayan finally resolved to deny
the motions for reconsideration of its Resolution, prompting Africa to file
before the Supreme Court a petition for Review on Certiorari, challenging the
Sandiganbayan Resolutions (authorizing the holding of a stockholders
meeting to increase ETPI's authorized capital stock and to vote therein the
sequestered Class "A" shares of stock) and (denying reconsideration of the
December 13, 1996 Resolution). The petitions were consolidated.

ISSUE/S:
Whether the PCGG can vote the sequestered ETPI Class "A" shares in the
stockholders meeting for the election of the board of directors.
RULING :
The PCGG cannot vote sequestered shares to elect the ETPI Board of
Directors or to amend the Articles of Incorporation for the purpose of
increasing the authorized capital stock unless there is a prima facie evidence
showing that said shares are ill-gotten and there is an imminent danger of
dissipation. The ETPI Stock and Transfer Book should be the basis for
determining which persons have the right to vote in the stockholders
meeting for the election of the ETPI Board of Directors. The PCGG is entitled
to vote the shares ceded to it by Roberto S. Benedicto and his controlled
corporations under the Compromise Agreement, provided that the shares are

Page 685 of 1072

first registered in the name of the PCGG. The PCGG may not register the
transfer of the Malacaang and the Nieto shares in the ETPI Stock and
Transfer Book; however, it may vote the same as conservator provided that
the PCGG satisfies the two-tiered test devised by the Court in Cojuangco v.
Calpo. The safeguards laid down in the case of Cojuangco v. Roxas shall be
incorporated in the ETPI Articles of Incorporation substantially
contemporaneous to, but not before, the election of the ETPI Board of
Directors. Members of the Sandiganbayan shall not participate in the
stockholders meeting for the election of the ETPI Board of Directors.

TOPIC:TRANSFER OF SHARES OF STOCK AND REGISTRATION


RURAL BANK OF LIPA CITY, INC., THE OFFICERS AND DIRECTORS, et
al.
vs.
CA, SECURITIES AND EXCHANGE COMMISSION, et al.
G.R. No. 124535; September 28, 2001
FACTS:
Private respondent R. Villanueva, Sr., stockholder of the Rural Bank of
Lipa City (BANK) executed a Deed of Assignment, wherein he assigned his
shares, as well as those of 8 other shareholders under his control in favor of
the stockholders of the Bank represented by its directors. Thereafter,
Villanueva, Sr. and his wife, Avelina, executed an Agreement wherein they
acknowledged their indebtedness to the Bank in the amount of P4M, and
stipulated that said debt will be paid out of the proceeds of the sale of their
real
property
described
in
the
Agreement.
At a meeting of the BODs of the Bank on Nov. 15, 1993, Villanueva
spouses assured the Board that their debt would be paid on or before Dec.
31 of that year; otherwise, the Bank would be entitled to liquidate their
shareholdings, including those under their control. In such an event, should
the proceeds of the sale of said shares fail to satisfy in full the obligation, the
unpaid balance shall be secured by other collateral sufficient therefore.

Page 686 of 1072

When the Villanueva spouses failed to settle their obligation to the


Bank on due date, the Board sent them a letter demanding the surrender of
all the stock certificates issued to them; and the delivery of sufficient
collateral to secure the balance of their debt. The Villanuevas ignored the
bank's demands, hence, their shares of stock were converted into Treasury
Stocks. Later, the Villanuevas, through their counsel, questioned the legality
of
the
conversion
of
their
shares.
On Jan. 15, 1994, stockholders of the Bank met to elect the new
directors and set of officers for the year 1994. The Villanuevas were not
notified of said meeting. Thus, Atty. A. Ignacio in a letter, counsel for the
Villanuevas, questioned the legality of the said meeting and the validity of
the proceedings therein. In reply, the new set of officers held that the
Villanuevas were no longer entitled to notice of the said meeting since they
had relinquished their rights as stockholders in favor of the Bank by reason
of the execution of the Deed of Assignment in favor of the Banks
Stockholders.
ISSUE/S:
Whether the transfer of title to shares in favor of petitioners by
execution of the deed of assignment is ineffective until and unless the
duly indorsed certificate of stock is delivered to them.
RULING:
SECTION 63: Certificate of stock and transfer of shares:
The capital stock of stock corporations shall be divided into
shares for which certificates signed by the president or vice
president, countersigned by the secretary or assistant
secretary, and sealed with the seal of the corporation shall
be issued in accordance with the by-laws. Shares of stocks
so issued are personal property and may be transferred by
delivery of the certificate or certificates indorsed by the
owner or his attorney-in-fact or other person legally
authorized to make the transfer. No transfer, however,
shall be valid, except as between the parties, until the
transfer is recorded in the books of the corporation so as to
show the names of the parties to the transaction, the date
of the transfer, the number of the certificate or certificates
and
the
number
of
shares
transferred.
In the case at bar, Petitioners argument that by virtue of the Deed of
Assignment, private respondents had relinquished to them any and all rights
they may have had as stockholders of the Bank is not correct. While it may

Page 687 of 1072

be true that there was an assignment of private respondents' shares to the


petitioners, said assignment was not sufficient to effect the transfer of shares
since there was no endorsement of the certificates of stock by the owners,
their attorneys-in-fact or any other person legally authorized to make the
transfer. Moreover, petitioners admit that the assignment of shares was not
coupled with delivery, the absence of which is a fatal defect. The rule is that
the delivery of the stock certificate duly endorsed by the owner is the
operative act of transfer of shares from the lawful owner to the transferee.
Thus, title may be vested in the transferee only by delivery of the duly
indorsed
certificate
of
stock.
The Supreme Court has uniformly held that for a valid transfer of
stocks, there must be strict compliance with the mode of transfer prescribed
by law. The requirements are:
a. There must be delivery of the stock certificate:
b. The certificate must be endorsed by the owner or his
attorney-in-fact or other persons legally authorized to
make the transfer; and
c. To be valid against third parties, the transfer must be
recorded in the books of the corporation.
As it is, compliance with any of these requisites has not been clearly
and sufficiently shown. It may be argued that despite non-compliance with
the requisite endorsement and delivery, the assignment was valid between
the parties, meaning the private respondents as assignors and the
petitioners as assignees. While the assignment may be valid and binding on
the petitioners and private respondents, it does not necessarily make the
transfer effective. Consequently, the petitioners, as mere assignees,
cannot enjoy the status of a stockholder, cannot vote nor be voted for, and
will not be entitled to dividends, insofar as the assigned shares are
concerned Parenthetically, the private respondents cannot, as yet, be
deprived of their rights as stockholders, until and unless the issue of
ownership and transfer of the shares in question is resolved with finality.

Page 688 of 1072

TOPIC:TRANSFER OF SHARES OF STOCK AND REGISTRATION


BATANGAS LAGUNA TAYABAS BUS COMPANY, INC.,
vs.
BITANGA
G.R. No. 137936. August 10, 2001
FACTS:
These cases involve the Batangas Laguna Tayabas Bus Company, Inc.,
which has been owned by four generations of the Potenciano family.
Immediately prior to the events leading to this controversy, the Potencianos
owned 87.5% of the outstanding capital stock of BLTB.On October 28, 1997,
Dolores A. Potenciano, Max Joseph A. Potenciano, Mercedelin A. Potenciano,
Delfin C. Yorro, and Maya Industries, Inc., entered into a Sale and Purchase
Agreement, whereby they sold to BMB Property Holdings, Inc., represented
by its President, Benjamin Bitanga, their 21,071,114 shares of stock in BLTB.
The said shares represented 47.98% of the total outstanding capital stock of
BLTB.
The purchase price for the shares of stock was P72,076,425.00, the
down payment of which, in the sum of P44,354,723.00, was made payable
upon signing of Agreement, while the balance of P27,721,702.00 was
payable on November 26, 1997. Furthermore, the buyer guaranteed that it
shall take over the management and operations of BLTB but shall

Page 689 of 1072

immediately surrender the same to the sellers in case it fails to pay the
balance of the purchase price on November 26, 1997.
Barely a month after the Agreement was executed, at a meeting of the
stockholders of BLTB, Benjamin Bitanga and Monina Grace Lim were elected
as directors of the corporation, replacing Dolores and Max Joseph
Potenciano. Subsequently, on November 28, 1997, another stockholders
meeting was held, wherein Laureano A. Siy and Renato L. Leveriza were
elected as directors, replacing Candido Potenciano and Delfin Yorro who had
both resigned as such. At the same meeting, the Board of Directors of BLTB
elected the following officers: Benjamin Bitanga as Chairman of the Board,
President and Chief Executive Officer; Monina Grace Lim as Vice President for
Finance and Supply and Treasurer; James Olayvar as Vice President for
Operations and Maintenance; Eduardo Azucena as Vice President for
Administration; Evelio Custodia as Corporate Secretary; and Gemma Santos
as Assistant Corporate Secretary.
However, the Bitanga group refused to relinquish their positions and
continued to act as directors and officers of BLTB. The conflict between the
Potencianos and the Bitanga group escalated to levels of unrest and even
violence among laborers and employees of the bus company.

ISSUE/S:
Whether or not a transfer of stock will ensue even if not recorded in the
books of the corporation.
RULING:
We find that the petitions are impressed with merit. Contrary to the
findings of the Court of Appeals, the Bitanga group was not deprived of due
process when the SEC En Banc issued its Order dated July 21, 1998.
It is not disputed that the transfer of the shares of the group of Dolores
Potenciano to the Bitanga group has not yet been recorded in the books of
the corporation. Hence, the group of Dolores Potenciano, in whose names
those shares still stand, were the ones entitled to attend and vote at the
stockholders meeting of the BLTB on 19 May 1998. This being the case, the
Hearing Panel committed grave abuse of discretion in holding otherwise and
in concluding that there was no quorum in said meeting.
We find no error either in jurisdiction or judgment on the part of the
SEC En Banc, since its conclusions of law were anchored on established
principles and jurisprudence.
In light of all the foregoing, we find that the Court of Appeals erred in
granting the extraordinary remedy of certiorari to the Bitanga group. It is

Page 690 of 1072

elementary that a special civil action for certiorari is limited to correcting


errors of jurisdiction or grave abuse of discretion. None of these have been
found to obtain in the petition before the Court of Appeals. What is more, it
is also settled that the issuance of the writ of preliminary injunction as an
ancillary or preventive remedy to secure the rights of a party in a pending
case is entirely within the discretion of the court taking cognizance of the
case, the only limitation being that this discretion should be exercised based
upon the grounds and in the manner provided by law. The exercise of sound
judicial discretion by the lower court in injunctive matters should not be
interfered with except in cases of manifest abuse.

TOPIC: TRANSFER OF SHARES OF STOCK AND REGISTRATION


SPOUSES JOSE ABEJO AND AURORA ABEJO, TELEC. TRONIC SYSTEMS,
INC., petitioners,
vs.
HON. RAFAEL DE LA CRUZ, JUDGE OF THE REGIONAL TRIAL COURT
(NATIONAL CAPITAL JUDICIAL REGION, BRANCH CLX-PASIG),
SPOUSES AGAPITO BRAGA AND VIRGINIA BRAGA, VIRGILIO BRAGA
AND NORBERTO BRAGA, respondents.
G.R. No.L-63558. May 19, 1987
FACTS:
The principal stockholders of Pocket Bell Philippines, Inc. (Pocket Bell)
namely, the Sps. Jose Abejo and Aurora Abejo (Abejos) sold to Telectronic
Systems, Inc. (Telectronics) their 133,000 minority shareholdings (for P5
million) and of 63,000 shares registered in the name of Virginia Braga and
covered by five stock certificates endorsed in blank by her (for
P1,674,450.00), and the Sps. Agapito Braga and Virginia Braga (Bragas),
erstwhile majority stockholders.
Telectronics requested the corporate secretary of the corporation to
register and transfer to its name, and those of its nominees the total 196,000
Pocket Bell shares in the corporation's transfer book, cancel the surrendered
certificates of stock and issue the corresponding new certificates of stock in
its name and those of its nominees.

Page 691 of 1072

The corporate secretary of Pocket Bell who is also the son of the
Bragas, refused to register the aforesaid transfer of shares in the corporate
books, asserting that the Bragas claim pre-emptive rights over the 133,000
Abejo shares and that Virginia Braga never transferred her 63,000 shares to
Telectronics but had lost the five stock certificates representing those shares.
The Abejos and Telectronics and the latter's nominees, as new majority
shareholders, filed cases with the SEC against the Bragas to compel the
corporate secretary of Pocket Bell to register in their names the transfer and
sale of the aforesaid 196,000 Pocket Bell shares.
On the other hand, Bragas filed a complaint against the Abejos and
Telectronics in the CFI for rescission and annulment of the sale of the shares
of stock in Pocket Bell made by the Abejos in favor of Telectronics on the
ground that it violated the Bragas' alleged pre-emptive right over the Abejos'
shareholdings.
ISSUE/S:
Whether the transfer and endorsement of the certificates of stock
alleged to have been violated pre-emptive be considered an intracorporate dispute.
RULING:
Directing the SEC through its Hearing Committee to proceed
immediately with hearing and resolving the pending mandamus petition for
recording in the corporate books the transfer to Telectronics and its
nominees of the majority (56%) shares of stock of the corporation Pocket Bell
pertaining to the Abejos and Virginia Braga and all related issues, taking into
consideration, without need of resubmittal to it, the pleadings, annexes and
exhibits filed by the contending parties in the cases at bar.
Such dispute clearly involves controversies "between and among
stockholders," as to the Abejos' right to sell and dispose of their shares to
Telectronics, the validity of the latter's acquisition of Virginia Braga's shares,
who between the Bragas and the Abejos' transferee should be recognized as
the controlling shareholders of the corporation, with the right to elect the
corporate officers and the management and control of its operations.

Page 692 of 1072

TOPIC: TRANSFER OF SHARES OF STOCK AND REGISTRATION

BATONG BUHAY GOLD MINES, INC., petitioner,


vs.
THE COURT OF APPEALS and INCORPORATED MINING CORPORATION,
respondents.
G.R. No. L-45048. January 7, 1987
FACTS:
Herein petitioner issued Stock Certificate No. 16807 covering 62,495
shares with a par value of P0.01 per share to Francisco Aguac who was then
legally married to Paula G. Aguac, but said spouses had lived separately for
more than fourteen (14) years prior to the said date. However, Francisco,
without the knowledge or consent of his wife, sold said shares for the sum of
P9, 374.70 in favor of herein private respondent Corporation.
On the same date of the sale, Paula Aguac wrote a letter to the
president of petitioner asking that the transfer of the shares sold by
Francisco be withheld, inasmuch as the same constituted conjugal property
and her share of proceeds of the sale was not given to her.
Later, private respondents counsel presented Stock Certificate No.
16807 duly endorsed by Francisco for registration and transfer of the said
stock in the name of private respondent. However, petitioner refused to

Page 693 of 1072

make such request of transfer on the basis of the letter sent to them by
Paula Aguac and in view of their apprehension that they might be held liable
for damages.
Consequently, private respondent commenced this action for herein
petitioner to release the transfer of the stock certificate. The CFI rendered a
decision in favor of private respondent, thus Stock Certificate No. 16807 was
cancelled and Stock Certificate No. 27650 was issued.
On appeal filed by private respondent anchored on the lower court's
alleged failure to award damages for the wrongful refusal of petitioner to
transfer the subject shares of stock and alleged failure to award attorney's
fees, cost of injunction bond and expenses of litigation, the Court of Appeals
modified the decision of the CFI by ordering petitioner to pay private
respondent sum of P5, 625.55, with interest at the legal rate from March 5,
1970 until full payment.
Hence this petition for review.
ISSUE/S:
May the Court of Appeals award damages by way of unrealized
profits despite the absence of supporting evidence, or merely on
the basis of pure assumption, speculation or conjecture; or can
the respondent recover damages by way of unrealized profits
when it has not shown that it was damaged in any manner by
the act of petitioner?
RULING:
No.
The stipulation of facts of the parties does not at all show that private
respondent intended to sell, or would sell or would have sold the stocks in
question on specified dates. While it is true that shares of stock may go up or
down in value (as in fact the concerned shares here really rose from fifteen
(15) centavos to twenty three or twenty four (23/24) centavos per share and
then fell to about two (2) centavos per share, still whatever profits could
have been made are purely SPECULATIVE, for it was difficult to predict with
any degree of certainty the rise and fall in the value of the shares. Thus this
Court has ruled that speculative damages cannot be recovered.
It is easy to say now that had private respondent gained legal title to
the shares, it could have sold the same and reaped a profit of P5,624.95 but
it could not do so because of petitioner's refusal to transfer the stocks in the
former's name at the time demand was made, but then it is also true that
human nature, being what it is, private respondent's officials could also have
refused to sell and instead wait for expected further increases in value.

Page 694 of 1072

TOPIC: TRANSFER OF SHARES OF STOCK AND REGISTRATION


CHEMPHIL EXPORT & IMPORT CORPORATION (CEIC), petitioner,
vs.
THE HONORABLE COURT OF APPEALS JAIME Y. GONZALES, as
Assignee of the Bank of the Philippine Islands (BPI), RIZAL
COMMERCIAL BANKING CORPORATION (RCBC), LAND BANK OF THE
PHILIPPINES (LBP), PHILIPPINE COMMERCIAL & INTERNATIONAL
BANK (PCIB) and THE PHILIPPINE INVESTMENT SYSTEM
ORGANIZATION (PISO),
G.R. Nos. 112438-39. December 12, 1995
251 SCRA 257
FACTS:
This case involves a claim between CEIC and The Consortium over
shares of stock in the Chemical Industries of the Philippines.
Antonio Garcia under a Deed of Sale transferred to Ferro Chemicals,
Inc. (FCI) shares in Chemical Industries (Chemphil) and other properties. It
was agreed upon that part of the purchase price shall be paid by FCI directly
to SBTC for whatever judgment credits that may be adjudged in the latter's
favor and against Antonio Garcia in a case. FCI assigned its shares in
Chemphil to CEIC. The shares were registered and recorded in the corporate

Page 695 of 1072

books of Chemphil in CEIC's name and the corresponding stock certificates


were issued to it.
Antonio Garcia failed to comply with the terms of the compromise
agreement he entered into with the consortium. Among Garcia's properties
that were levied upon on execution were his in Chemphil. The consortium
acquired the disputed shares of stock at the public auction sale conducted by
the sheriff. A Certificate of Sale covering the disputed shares was issued to it.
ISSUE/S:
Are attachments of shares of stock included in the term
"transfer" as provided in Sec. 63 of the Corporation Code?
RULING:
No.
As succinctly declared in the case of Monserrat v. Ceron, "chattel
mortgage over shares of stock need not be registered in the corporation's
stock and transfer book inasmuch as chattel mortgage over shares of stock
does not involve a "transfer of shares," and that only absolute transfers of
shares of stock are required to be recorded in the corporation's stock and
transfer book in order to have "force and effect as against third persons."
xxx xxx xxx
The word "transferencia" (transfer) is defined by the "Diccionario
de la Academia de la Lengua Castellana" as "accion y efecto de
transfeir" (the act and effect of transferring); and the verb
"transferir", as "ceder or renunciar en otro el derecho o dominio
que se tiene sobre una cosa, haciendole dueno de ella" (to assign
or waive the right in, or absolute ownership of, a thing in favor of
another, making him the owner thereof).
In the Law Dictionary of "Words and Phrases", third series,
volume 7, p. 5867, the word "transfer" is defined as follows:
"Transfer" means any act by which property of one
person is vested in another, and "transfer of shares",
as used in Uniform Stock Transfer Act (Comp. St.
Supp. 690), implies any means whereby one may be
divested of and another acquire ownership of stock.
(Wallach vs. Stein [N.J.], 136 A., 209, 210.)
xxx xxx xxx
In the case of Noble vs. Ft. Smith Wholesale Grocery Co. (127
Pac., 14, 17; 34 Okl., 662; 46 L.R.A. [N.S.], 455), cited in Words
and Phrases, second series, vol. 4, p. 978, the following appears:

Page 696 of 1072

A "transfer" is the act by which the owner of a thing


delivers it to another with the intent of passing the
rights which he has in it to the latter, and a chattel
mortgage is not within the meaning of such term.
xxx xxx xxx.
Although the Monserrat case refers to a chattel mortgage over shares
of stock, the same may be applied to the attachment of the disputed shares
of stock in the present controversy since an attachment does not constitute
an absolute conveyance of property but is primarily used as a means "to
seize the debtor's property in order to secure the debt or claim of the
creditor in the event that a judgment is rendered."
Known commentators on the Corporation Code expound, thus:
xxx xxx xxx
Shares of stock being personal property, may be the subject
matter of pledge and chattel mortgage. Such collateral transfers
are however not covered by the registration requirement of
Section 63, since our Supreme Court has held that such provision
applies only to absolute transfers thus, the registration in the
corporate books of pledges and chattel mortgages of shares
cannot have any legal effect.
xxx xxx xxx
The requirement that the transfer shall be recorded in the books
of the corporation to be valid as against third persons has
reference only to absolute transfers or absolute conveyance of
the ownership or title to a share.
Consequently, the entry or notation on the books of the
corporation of pledges and chattel mortgages on shares is not
necessary to their validity (although it is advisable to do so)
since they do not involve absolute alienation of ownership of
stock (Monserrat vs. Ceron, 58 Phil. 469 [1933]; Chua Guan vs.
Samahang Magsasaka, Inc., 62 Phil. 472 [1935].) To affect third
persons, it is enough that the date and description of the shares
pledged appear in a public instrument. (Art. 2096, Civil Code.)
With respect to a chattel mortgage constituted on shares of
stock, what is necessary is its registration in the Chattel
Mortgage Registry. (Act No. 1508 and Art. 2140, Civil Code.)
CEIC's reliance on the Samahang Magsasaka case is misplaced. Nowhere in
the said decision was it categorically stated that annotation of the

Page 697 of 1072

attachment in the corporate books is mandatory for its validity and for the
purpose of giving notice to third persons.
The only basis, then, for petitioner CEIC's claim is the Deed of Sale under
which it purchased the disputed shares. It is, however, a settled rule that a
purchaser of attached property acquires it subject to an attachment legally
and validly levied thereon.

TOPIC: TRANSFER OF SHARES OF STOCK AND REGISTRATION


GONZALO CHUA GUAN, plaintiff-appellant,
vs.
SAMAHANG MAGSASAKA, INC., and SIMPLICIO OCAMPO, ADRIANO G.
SOTTO, and EMILIO VERGARA, as president, secretary and treasurer
respectively of the same, defendants-appellees.
G.R. No. L-42091. November 2, 1935
FACTS:
Respondent Samahang Magsasaka, Inc., is a corporation duly
organized under the laws of the Philippines with principal office in
Cabanatuan, Nueva Ecija. Gonzalo Co Toco was the owner of 5,894 shares of
the capital stock of the said corporation represented by nine certificates
having a par value of P5 per share. Said shares were mortgaged by Co Toco

Page 698 of 1072

to Chua Chiu to guarantee the payment of his debt of P20, 000. Thus, the
stock certificates were delivered to Chua Chiu and the mortgage was duly
registered in the office of the register of deeds of Manila and in the office of
Respondent Corporation.
Thereafter, Chua Chiu assigned his right and interest in the said
mortgage to the plaintiff, which was duly registered in the office of the
register of deeds of Manila and in the office of Respondent Corporation.
For Co Tocos failure to pay the debt at maturity, plaintiff foreclosed
said mortgage and delivered the certificates of stock and copies of the
mortgage and assignment to the sheriff of the City of Manila in order to sell
the said shares at public auction. After the auction sale of said 5, 894 shares
of stock and the plaintiff being the highest bidder, the sheriff executed in his
favor a certificate of sale of said shares.
Consequently, plaintiff requested from the officers of the corporation
that new certificates be issued in his name. However, respondent officers
refused to issue said new shares in plaintiffs name. They claimed that prior
to said demand, nine attachments had been issued and served and noted on
the books of the corporation against the shares of Co Toco and the plaintiff
objected to having these attachments noted on the new certificates which he
demanded.
ISSUE/S:
Whether or not shares of a corporation could be hypothecated by
placing a chattel mortgage on the certificate representing such
shares.
RULING:
Apart from the cumbersome and unusual method of hypothecating
shares of stock by chattel mortgage, it appears that in the present state of
our law, the only safe way to accomplish the hypothecation of share of stock
of a Philippine corporation is for the creditor to insist on the assignment and
delivery of the certificate and to obtain the transfer of the legal title to him
on the books of the corporation by the cancellation of the certificate and the
issuance of a new one to him. From the standpoint of the debtor this may be
unsatisfactory because it leaves the creditor as the ostensible owner of the
shares and the debtor is forced to rely upon the honesty and solvency of the
creditor. Of course, the mere possession and retention of the debtor's
certificate by the creditor gives some security to the creditor against an
attempted voluntary transfer by the debtor, provided the by-laws of the
corporation expressly enact that transfers may be made only upon the
surrender of the certificate. It is to be noted, however, that Section 35 of the
Corporation Law (Act No. 1459) enacts that shares of stock "may be
transferred by delivery of the certificate endorsed by the owner or his

Page 699 of 1072

attorney in fact or other person legally authorized to make the transfer." The
use of the verb "may" does not exclude the possibility that a transfer may be
made in a different manner, thus leaving the creditor in an insecure position
even though he has the certificate in his possession. Moreover, the shares
still standing in the name of the debtor on the books of the corporation will
be liable to seizure by attachment or levy on execution at the instance of
other creditors. (Cf. Uy Piaoco vs. McMicking, 10 Phil., 286, and Uson vs.
Diosomito, 61 Phil., 535.) This unsatisfactory state of our law is well known to
the bench and bar. (Cf. Fisher, The Philippine Law of Stock Corporations,
pages 163-168.) Loans upon stock securities should be facilitated in order to
foster economic development. The transfer by endorsement and delivery of a
certificate with intention to pledge the shares covered thereby should be
sufficient to give legal effect to that intention and to consummate the juristic
act without necessity for registration.
The Supreme Court is fully conscious of the fact that our decisions in
the case of Monserrat vs. Ceron,supra, and in the present case have done
little perhaps to ameliorate the present uncertain and unsatisfactory state of
our law applicable to pledges and chattel mortgages of shares of stock of
Philippine corporations. The remedy lies with the legislature.
In view of the premises, the attaching creditors are entitled to priority
over the defectively registered mortgage of the appellant and the judgment
appealed from must be affirmed without special pronouncement as to costs
in this instance

TOPIC: TRANSFER OF SHARES OF STOCK AND REGISTRATION


COLLECTOR OF INTERNAL REVENUE, petitioner,
vs.
ANGLO CALIFORNIA NATIONAL BANK (CROCKER-ANGLO NATIONAL
BANK), as Treasurer for CALAMBA SUGAR ESTATE, INC., respondent.
G.R. No. L-12476. January 29, 1960

Page 700 of 1072

FACTS:
Respondent Calamba Sugar, represented by respondent Anglo, is a
foreign corporation organized and existing under the laws of the State of
California, U.S.A., and duly licensed to do business in the Philippines. In
petitioners assessment, the corporation was assessed its alleged deficiency
income taxes for the year 1953, 1954, and 1955 in the amounts ofP138,
855.00, P131,759.00 and P393,459.00 respectively. Said deficiencies were
based upon capital again derived from the respondent's sale to the Pasumil
Planters, Inc., of P250, 000 shares of the capital stock of the Pampanga
Sugar Mills (a domestic corporation) and of a promissory note, dated January
1, 1950, executed by the Pampanga Sugar Mills in the sum of $500,000.00.
The CTA reversed said ruling of petitioner and absolved respondent
from liability.
ISSUE/S:
When and where title to the goods passes from the seller to the
buyer?
RULING:
In this case, it is admitted that the negotiation, perfection and
consummation of the contract of sale were all done in California, U.S.A. It
follows that title to the shares of stock passed from the vendor to the vendee
at said place, from which time the incidents of ownership vested on the
buyer.
The Collector argues that the sit us of shares of stock of a corporation
is considered to be at the domicile of the latter, as held in some cases cited
by him; but in the instant problem, we are not concerned with the imposition
of taxes upon the shares themselves, but on a sale effected abroad that
resulted in capital gains, for which there is a specific provision of law (Sec. 37
[e] N.I.R.C.). As stated by the Tax Court, there is a distinction between the
situs of personal properties and the situs of the income derived from the sale
or exchange of such properties.
As to the contention that section 35 of the Corporation Law (Act No.
1459) requires the transfer to be noted and entered not invalidate the
transfer between the parties nor is it essential to vest title upon the vendee.
The capital gains, now sought to be taxed, arose from the severance of gain,
from the investment occasioned by the transfer of title abroad and not on
account of any registration that might be effected later.
Wherefore, the judgment under view is hereby affirmed.

Page 701 of 1072

TOPIC: TRANSFER OF SHARES OF STOCK AND REGISTRATION


APOLINARIO G. DE LOS SANTOS and ISABELO ASTRAQUILLO,
plaintiffs-appellees, vs.
J. HOWARD MCGRATH ATTORNEY GENERAL OF THE UNITED STATES,
SUCCESSOR TO THE PHILIPPINE ALIEN PROPERTY ADMINISTRATION
OF THE UNITED STATES, defendant-appellant.
G.R. No. L-4818. February 28, 1955
FACTS:

Page 702 of 1072

This case involves the title to 1, 600, 000 shares of stock of the
Lepanto Consolidated Mining Co., Inc. The Shares of stock are covered by
several stock certificates issued in favor of Vicente Madrigal, who is
registered in the books of Lepanto as owner of said stocks.
Claims of Plaintiffs
Apolinario De Los Santos claims of said shares while the other half
was claimed by Isabelo Astraquillo. They claimed that De los Santos bought
55, 000 shares from Juan Campos, 300,000 shares from Carl Hess and 800,
000 shares from Hess for the account and benefit of Astraquillo. However,
by virtue of vesting P-12 Title to the 1, 600,000 shares of stock were vested
in the Alien Property Custodian of the U.S. as Japanese Property. Hence,
plaintiffs filed their respective claims with the Property Custodian. Upon
personal review, the Philippine Alien property Administration decreed that
title to the shares shall remain in the name of the Philippine Alien
Administrator.
Claims of Defendant
As the Attorney General of the U.S, successor to the Administrator,
he claims that prior to the outbreak of the war in the pacific, said shares of
stock were bought by Vicente Madrigal, in trust for, and for the benefit of, the
Mitsui Bussan Kaisha, a corporation organized in accordance with the laws of
Japan, the true owner thereof, with branch office in the Philippines. Madrigal
later on delivered the corresponding stock certificates, with his blank
indorsement thereon, to the Mitsuis, which kept the same, in the files of its
office in Manila. The Mitsuis never sold, or otherwise disposed of, said shares
of stock; and that the stock certificates aforementioned must have been
stolen or looted.
Pursuant to the Philippine Property Act, all property vested in the US, or
any of its officials, located in the Philippines at that time of such vesting, or
the proceeds thereof, shall be transferred to the Republic of the Philippines.
Decision of the CFI
Judgment is rendered in favor of the plaintiffs, declaring that they are
the absolute owners of the disputed shares of stocks. That the transfer of
said shares in favor of the Alien Property Custodian of the US, now Philippine
Alien Property Administration, is declared null and void.
ISSUE/S:
Whether or not plaintiffs had purchased the shares of stock in
question.
RULING:

Page 703 of 1072

Even, however, if Juan Campos and Carl Hess had sold the shares of
stock in question, as testified to by De los Santos, the result, insofar as
plaintiffs are concerned, would be the same. It is not disputed that said
shares of stock were registered, in the records of the Lepanto, in the name of
Vicente Madrigal. Neither is it denied that the latter was, as regards said
shares of stock, a mere trustee for the benefit of the Mitsuis. The record
shows and there is no evidence to the contrary that Madrigal had never
disposed of said shares of stock in any manner whatsoever, except by
turning over the corresponding stock certificates, late in 1941, to the Mitsuis,
the beneficial and true owners thereof. It has, moreover, been established,
by the uncontradicted testimony of Kitajima and Miwa, the managers of the
Mitsuis in the Philippines, from 1941 to 1945, that the Mitsuis had neither
sold, conveyed, or alienated said shares of stock, nor delivered the
aforementioned stock certificates, to anybody during said period. Section 35
of the Corporation Law reads:
The capital stock corporations shall be divided into shares for
which certificates signed by the president or the vice-president,
countersigned by the secretary or clerk and sealed with the seal
of the corporation, shall be issued in accordance with the bylaws. Shares of stock so issued are personal property and may be
transferred by delivery of the certificate endorsed by the owner
or his attorney in fact or other person legally authorized to make
the transfer. No transfer, however, shall be valid, except as
between the parties, until the transfer is entered and noted upon
the books of the corporation so as to show the names of the
parties to the transaction, the date of the transfer, the number of
the certificate, and the number of shares transferred.
No shares of stock against which the corporation holds any
unpaid claim shall be transferable on the books of the
corporation.
Pursuant to this provision, a share of stock may be transferred by
endorsement of the corresponding stock certificate, coupled with its delivery.
However, the transfer shall "not be valid, except as between the parties,"
until it is "entered and noted upon the books of the corporation." no such
entry in the name of the plaintiffs herein having been made, it follows that
the transfer allegedly effected by Juan Campos and Carl Hess in their favor is
"not valid, except as between" themselves. It does not bind either Madrigal
or the Mitsuis, who are not parties to said alleged transaction. What is more,
the same is "not valid," or, in the words of the Supreme Court of Wisconsin
(Re Murphy, 51 Wisc. 519, 8 N. W. 419) which were quoted approval in
Uson vs. Diosomito (61 Phil., 535) "absolutely void" and, hence, as good
as non-existent, insofar as Madrigal and the Mitsuis are concerned. For this
reason, although a stock certificate is sometimes regarded as quasinegotiable, in the sense that it may be transferred by endorsement, coupled

Page 704 of 1072

with delivery, it is well settled that the instrument is non-negotiable, because


the holder thereof takes it without prejudice to such rights or defenses as the
registered owner or creditor may have under the law, except insofar as such
rights or defenses are subject to the limitations imposed by the principles
governing estoppel.

In the case at bar, neither Madrigal nor the Mitsuis had alienated
shares of stock in question. It is not even claimed that either had, through
negligence, given occasion for an improper or irregular disposition of the
corresponding stock certificates. Plaintiffs merely argue without any
evidence whatsoever thereon that Kitajima might have, or must have,
assigned the certificates on or before December 1942, although, as above
stated, this is, not only, improbable, under the conditions, then obtaining,
but, also., impossible, considering that, in April 1943, Kitajima delivered the
instruments to Miwa, who kept them in its possession until 1945. At any rate,
such assignment by Miwa granting for the sake of argument the accuracy
of the surmise of plaintiffs herein was unauthorized by the Mitsuis, who, in
the light of the precedents cited above, are not chargeable with negligence.

TOPIC: TRANSFER OF SHARES OF STOCK AND REGISTRATION


GLORIA M. DE ERQUIAGA, administratrix of the estate of the late
SANTIAGO DE ERQUIAGA & HON. FELICIANO S. GONZALES,
petitioners,
vs.
HON. COURT OF APPEALS, AFRICA VALDEZ VDA. DE REYNOSO, JOSES
V. REYNOSO, JR., ERNESTO , SYLVIA REYNOSO, LOURDES REYNOSO,

Page 705 of 1072

CECILE REYNOSO, EDNA REYNOSO, ERLINDA REYNOSO & EMILY


REYNOSO, respondents.
G.R. No. 47206. September 27, 1989
FACTS:
Santiago de Erquiaga was the owner of 100% or 3, 100 paid-up shares
of stock of the Erquiaga Devt Corp. which owns the Hacienda San Jose in
Irosin, Sorsogon. Said shares of stock was to be sold to Jose Reynoso in the
amount of P 900,00 payable in instalment on definite dates fixed in the
contract. For Reynosos failure to pay the 2 nd and 3rd instalments on time, the
total price was increased to P971, 000.00. However, Reynoso was able to pay
only P 410, 00.00 to Erquiaga. Despite the incomplete payment, Erquiaga
transferred all his shares to Reynoso, as well as the possession of Hacienda
San Jose with a condition that 1,500 shares is pledged in favour of Erquiaga
as security for the unpaid balance of P 561, 321.70 as provided in a
promissory note. Reynoso failed to pay the balance, thus, Erquiaga formally
informed the later that he was rescinding the sale of the shares.
Thereafter, Erquiaga filed a complaint for rescission with the CFI. The
CFI rendered a decision in favor of Erquiaga. Said decision became final and
executor as the parties failed to file an appeal. However, the court ordered
that the return of the initial payment of P410, 000.00 to Reynoso be in
abeyance until the rendition of an accounting by latter of the fruits received
by him on account of 3,100 shares. The court also appointed a receiver upon
the filing of a P100, 000.00 bonds by Erquiaga.
Upon the death of Reynoso, he was substituted by his heirs, herein
private respondents who filed a petition for review for the annulment of the
above mentioned order and appointment. However, the CA dismissed said
petition.
Upon motion of Erquiaga, CFI issued an order receivership and ordering
the delivery of the possession of the Hacienda San Jose to Erquiaga, the filing
of bond by said Erquiaga in the amount of P410, 000.00 conditioned to the
payment of whatever may be due to the substituted heirs of deceased
defendant Reynoso. Another order was issued by the CFI, to wit:
(a) To call and hold a special meeting of the stockholders of the
Erquiaga Development Corporation to elect the members of the
Board of Directors;
(b) In the said meeting the plaintiff is authorized to vote not only
the 1,500 shares of stock in his name but also the 1,600 shares
in the name and possession of the defendants;
Said portion of the order was set aside by the CA.

Page 706 of 1072

ISSUE/S:
Whether the Court
Corporation Law.

of

Appeals

erroneously

applied

the

RULING:
No.
After deliberating on the petition for review, the Supreme Court finds
no reversible error in the Court of Appeals' decision directing the clerk of
court of the trial court to execute a deed of conveyance to Erquiaga of the
1,600 shares of stock of the Erquiaga Development Corporation still in
Reynoso's name and/or possession, in accordance with the procedure in
Section 10, Rule 39 of the Rules of Court. Neither did it err in annulling the
trial court's order: (1) allowing Erquiaga to vote the 3,100 shares of Erquiaga
Development Corporation without having effected the transfer of those
shares in his name in the corporate books; and (2) authorizing Erquiaga to
call a special meeting of the stockholders of the Erquiaga Development
Corporation and to vote the 3,100 shares, without the pre-requisite
registration of the shares in his name. It is a fundamental rule in Corporation
Law (Section 35) that a stockholder acquires voting rights only when the
shares of stock to be voted are registered in his name in the corporate
books.
Until registration is accomplished, the transfer, though valid between
the parties, cannot be effective as against the corporation. Thus, the
unrecorded transferee cannot enjoy the status of a stockholder; he cannot
vote nor be voted for, and he will not be entitled to dividends. The
Corporation will be protected when it pays dividend to the registered owner
despite a previous transfer of which it had no knowledge. The purpose of
registration therefore is two-fold; to enable the transferee to exercise all the
rights of a stockholder and to inform the corporation of any change in share
ownership so that it can ascertain the persons entitled to the rights and
subject to the liabilities of a stockholder.

TOPIC: TRANSFER OF SHARES OF STOCK AND REGISTRATION


NEMESIO GARCIA, petitioner,
vs.
NICOLAS JOMOUAD, Ex-officio Provincial Sheriff of Cebu and
SPOUSES JOSE ATINON & SALLY ATINON, respondents.

Page 707 of 1072

G.R. NO. 133969, JANUARY 26, 2000


FACTS:
This case stemmed from the decision of the trail court in its judgment
ordering Dico to pay the spouses Atinon the sum of P900,000.00 plus
interests for which respondent sheriff proceeded with its execution. In the
course thereof, the Proprietary Ownership Certificate in the name of Dico,
was levied on and scheduled for public auction and petitioner opposed the
same claiming ownership thereof.
Petitioner avers that Dico, was employed as manager of his Young
Auto Supply. In order to assist him in entertaining clients, petitioner "lent" his
POC, to Dico so the latter could enjoy the "signing" privileges of its members.
The Club issued POC No. 0668 in the name of Dico. Thereafter, Dico resigned
as manager of petitioner's business. Upon demand of petitioner, Dico
returned POC No. 0668 to him and Dico then executed a Deed of Transfer,
covering the subject certificate in favor of petitioner. The Club was furnished
with a copy of said deed but the transfer was not recorded in the books of
the Club because petitioner failed to present proof of payment of the
requisite capital gains tax.
ISSUE/S:
Whether or not a bona fide transfer of the shares of a
corporation, not registered or noted in the books of the
corporation, is valid as against a subsequent lawful attachment
of said shares, regardless of whether the attaching creditor had
actual notice of said transfer or not.
RULING:
The transfer of the subject certificate made by Dico to petitioner was
not valid as to the spouses Atinon, the judgment creditors, as the same still
stood in the name of Dico, the judgment debtor, at the time of the levy on
execution. In addition, as correctly ruled by the CA, the entry in the minutes
of the meeting of the Club's board of directors noting the resignation of Dico
as proprietary member thereof does not constitute compliance with Section
63 of the Corporation Code, which provides that No transfer, however, shall
be valid, except as between the parties, until the transfer is recorded in the
books of the corporation showing the names of the parties to the transaction,
the date of the transfer, the number of the certificate or certificates and the
number of shares transferred.
All transfers not so entered on the books of the corporation are
absolutely void; not because they are without notice or fraudulent in law or
fact, but because they are made so void by statute.

Page 708 of 1072

TOPIC: TRANSFER OF SHARES OF STOCK AND REGISTRATION


BENITO H. LOPEZ, petitioner,
vs.
THE COURT OF APPEALS and THE PHILIPPINE AMERICAN GENERAL
INSURANCE CO., INC., respondents.
G.R. NO. L-33157, JUNE 29, 1982

Page 709 of 1072

FACTS:
Petitioner Benito H. Lopez obtained a loan in the amount of P20,000.00
from the Prudential Bank and Trust Company for which he executed a
promissory note for the same amount and a Surety Bond in which he, as
principal, and Philippine American General Insurance Co., Inc. (PHILAMGEN)
as surety, bound themselves jointly and severally in favor of Prudential Bank
for the payment of the sum of P20,000.00. At the same time, Lopez executed
a deed of assignment of 4,000 shares of the Baguio Military Institution
entitled in favour of Philamgen.
Lopez' obligation matured without it being settled thus, the Prudential
Bank made demands for payment both upon Lopez and Philamgen. In turn,
Philamgen sent Lopez several written demands for the latter to pay his note
but Lopez did not comply with said demands. Hence, the Prudential Bank
filed a case against them to enforce payment on the promissory note plus
interest.
The transfer of Stock Certificate for 4,000 shares to Philamgen in the
name of Lopez was issued by the Baguio Military Institute in the name of
Philamgen but when no payment was still made by the principal debtor or
by the surety, the Prudential Bank filed another complaint for the recovery of
the P20,000.00. Philamgen was forced to pay the Prudential Bank thus
Philamgen brought an action in the Court for reimbursement of the said
amount.
ISSUE/S:
Whether or not there was transfer of shares o stocks when
petitioner "sells, assigns and transfers" and delivers shares of
stock to respondent, duly endorsed in blank, in consideration of a
contingent obligation of the former to the latter.
RULING:
There was no transfer of shares but only a contract of pledge.
This is evident from the indemnity agreement which connotes a
continuing obligation of Lopez towards Philamgen to pay a premium of
P1,000.00 for a period of one year and agreed at all times to indemnify
Philamgen of any and all kinds of losses which the latter might sustain by
reason of it becoming a surety. A contract of pledge have been satisfied: (1)
that it be constituted to secure the fulfillment of a principal obligation; (2)
that the pledgor be the absolute owner of the thing pledged; and (3) that the
person constituting the pledge has the free disposal of the property, and in

Page 710 of 1072

the absence thereof, that he be legally authorized for the purpose. (Article
2085, New Civil Code).
The transaction entered into by and between petitioner and
respondent under the Stock Assignment Separate From Certificate in relation
to the Surety Bond and the Indemnity Agreement, constitutes a pledge of the
40,000 shares of stock by the petitioner-pledgor in favor of the private
respondent-pledgee, and not a dacion en pago. True, the stock certificate of
the appellee had been in the name of the appellant but the transfer was
merely nominal, and was not intended to make the plaintiff the owner
thereof. No offer had been made for the return of the stocks to the
defendant. As the appellant had stated, the appellee could have the stocks
transferred to him anytime as long as he reimburses the plaintiff the amount
it had paid to the Prudential Bank. Pending payment, plaintiff is merely
holding the certificates as a pledge or security for the payment of
defendant's obligation.

TOPIC: TRANSFER OF SHARES OF STOCK AND REGISTRATION


ENRIQUE MONSERRAT, plaintiff-appellee,
vs. C
ARLOS G. CERON, ET AL., defendants. ERMA, INC., and, THE SHERIFF
OF MANILA, respondents.
G.R. No. 37078, September 27, 1933

Page 711 of 1072

FACTS:
The plaintiff Monserrat, was the president and manager of the Manila
Yellow Taxicab Co., Inc., and the owner of P1, 200 common shares of stock
thereof. He assigned to defendant Ceron the usufruct of half of the aforesaid
common shares of stock, the transferor having reserved for himself and his
heirs the right to vote derived from said shares of stock and to recover the
ownership thereof at the termination of the usufruct.
Ceron mortgaged to Eduardo R. Matute some shares of stock of the
Manila Yellow Taxicab Co., Inc., among which were the 600 common shares of
stock in question, for the sum of P30,000. Ceron endorsed to Matute the
certificate of stock, of which Matute has been in possession ever since. When
Ceron mortgaged the shares in question to Matute, he did not inform Matute
of the existence of the document, and the latter never had any knowledge
thereof.
ISSUE/S:
Whether or not it is necessary to enter upon the books of the
corporation a mortgage constituted on common shares of stock in
order that such mortgage may be valid and may have force and effect
as against third persons
Whether or not a mortgage constituted on certain shares of stock in
accordance with Act No. 1508, as amended by Act No. 2496, is a
transfer of such shares
RULING:
Yes.
Sec. 35 of the Corporation Code provides xxx No transfer, however,
shall be valid, except as between the parties, until the transfer is entered
and noted upon the books of the corporation so as to show the names of the
parties to the transaction, the date of the transfer the number of the
certificate, and the number of shares transferred. No share of stock against
which the corporation hold, any unpaid claim shall be transferable on the
books of the corporation.
Although a chattel mortgage, accompanied by delivery of the
mortgaged thing, transfers the title and ownership thereof to the mortgage
creditor, such transfer is not absolute but constitutes a mere security for the
payment of the mortgage debt, the transfer in question becoming null and
void from the time the mortgage debtor complies with his obligation to pay
his debt. A "transfer" is the act by which owner of a thing delivers it to

Page 712 of 1072

another with the intent of passing the rights which he has in it to the latter,
and a chattel mortgage is not within the meaning of such term.

TOPIC: TRANSFER OF SHARES OF STOCK AND REGISTRATION


EUGENIO J. PUYAT, ERWIN L. CHIONGBIAN, EDGARDO P. REYES,
ANTONIO G. PUYAT, JAIME R. BLANCO, RAFAEL R. RECTO and

Page 713 of 1072

REYNALDO L. LARDIZABAL, petitioners,


vs. HON. SIXTO T. J. DE GUZMAN, JR., as Associate Commissioner of
the Securities & Exchange Commission, et. Al. respondents.
G.R. NO. L-51122, MARCH 25, 1982
FACTS:
An election for the eleven Directors of the International Pipe Industries
Corporation (IPI) a private corporation. Assemblyman Estanislao A. Fernandez
had purchased from Augusto A. Morales ten (10) shares of stock of IPI for
P200.00 upon request of respondent Acero to qualify him to run for election
as a Director.
Assemblyman Fernandez had acquired a mere P200.00 worth of stock
in IPI, representing ten shares out of 262,843 outstanding shares. He
acquired them after the contested election of Directors, after the quo
warranto suit had been filed before SEC and one day before the scheduled
hearing of the case before the SEC.
ISSUE/S:
Whether or not the transfer of shares of stock to Fernandez was valid.
RULING:
NO.
A ruling upholding the intervention would make the constitutional
provision ineffective. All an Assemblyman need do, if he wants to influence
an administrative body is to acquire a minimal participation in the interest
of the client and then intervene in the proceedings. That which the
Constitution directly prohibits may not be done by indirection or by a general
legislative act which is intended to accomplish the objects specifically or
impliedly prohibited. The intervention of Assemblyman Fernandez falls within
the ambit of the prohibition contained in Section 11, Article VIII of the
Constitution.

TOPIC: TRANSFER OF SHARES OF STOCK AND REGISTRATION


ENRIQUE RAZON, petitioner,
vs.

Page 714 of 1072

INTERMEDIATE APPELLATE COURT and VICENTE B. CHUIDIAN, in his


capacity as Administrator of the Estate of the Deceased JUAN T.
CHUIDIAN, respondents.
G.R. No. 74306 March 16, 1992
FACTS:
E. Razon, Inc. was organized in 1962 by petitioner Enrique Razon for
the purpose of participating in the bidding for the arrastre services in South
Harbor, Manila. The incorporators withdrew from the said corporation so
petitioner then distributed the stocks previously placed in the names of the
withdrawing nominal incorporators to some friends, among them the late
Juan T. Chuidian to whom he gave 1,500 shares of stock.
The shares of stock were registered in the name of Chuidian only as
nominal stockholder and with the agreement that the said shares of stock
were owned and held by the petitioner but Chuidian was given the option to
buy the same. In view of this arrangement, Chuidian in 1966 delivered to the
petitioner the stock certificate covering the 1,500 shares of stock of E.
Razon, Inc. Since then, the Petitioner had in his possession the certificate of
stock until the time, he delivered it for deposit with the Philippine Bank of
Commerce under the parties' joint custody pursuant to their agreement as
embodied in the trial court's order.
ISSUE/S:
Whether or not the oral testimony as regards the true nature of his
agreement with the late Juan Chuidian on the 1,500 shares of stock is
sufficient to prove the transfer of the said shares of stock to him.
RULING:
No.
For an effective, transfer of shares of stock, the mode and manner of
transfer is prescribed by law under Section 3 of Batas Pambansa Bilang, 68
otherwise known as the Corporation Code of the Philippines. It provides that
shares of stock may be transferred by delivery to the transferee of the
certificate properly indorsed. However, no transfer shall be valid, except as
between the parties until the transfer is properly recorded in the books of the
corporation (Sec. 63, Corporation Code of the Philippines; Section 35 of the
Corporation Law)
In the instant case, there is no dispute that the questioned 1,500
shares of stock of E. Razon, Inc. are in the name of the late Juan Chuidian in
the books of the corporation. In such a case, the petitioner who claims
ownership over the questioned shares of stock must show that the same

Page 715 of 1072

were transferred to him by proving that all the requirements for the effective
transfer of shares of stock in accordance with the corporation's by laws, if
any, were followed.
The petitioner did not present any by-laws which could show that the
1,500 shares of stock were effectively transferred to him. In the absence of
the corporation's by-laws or rules governing effective transfer of shares of
stock, the provisions of the Corporation Law are made applicable to the
instant case. Since the certificate of stock covering the questioned 1,500
shares of stock registered in the name of the late Juan Chuidian was never
indorsed to the petitioner, the inevitable conclusion is that the questioned
shares of stock belong to Chuidian. To reiterate, indorsement of the
certificate of stock is a mandatory requirement of law for an effective
transfer of a certificate of stock.

TOPIC: TRANSFER OF SHARES OF STOCK AND REGISTRATION


AQUILINO RIVERA, ISAMU AKASAKO and FUJIYAMA HOTEL &
RESTAURANT, INC., petitioners,
vs. T

Page 716 of 1072

HE HON. ALFREDO C. FLORENDO, as Judge of the Court of First


Instance of Manila (Branch XXXVI), LOURDES JUREIDINI and
MILAGROS TSUCHIYA, respondents.
G.R. No. L-57586. October 8, 1986
144 SCRA 643
FACTS:
Petitioner corporation was organized and register under Philippine laws
with a capital stock of P1,000,000.00 divided into 10,000 shares of P100.00
par value each by the herein petitioner Rivera and four (4) other
incorporators. Sometime thereafter petitioner Rivera increased his
subscription from the original 1,250 to a total of 4899 shares.
Subsequently, Isamu Akasako, a Japanese national and co-petitioner
who is allegedly the real owner of the shares of stock in the name of
petitioner Aquilino Rivera, sold 2550 shares of the same to private
respondent Milagros Tsuchiya for a consideration of P440,000.00 with the
assurance that Milagros Tsuchiya will be made the President and Lourdes
Jureidini a director after the purchase. Aquilino Rivera who was in Japan also
assured private respondents by overseas call that he will sign the stock
certificates because Isamu Akasako is the real owner. However, after the sale
was consummated and the consideration was paid with a receipt of payment
therefore shown, Aquilino Rivera refused to make the indorsement unless he
is also paid.
It also appears that the other incorporators sold their shares to both
respondent Jureidini and Tsuchiya such that both respondents became the
owners of a total of 3300 shares or the majority out of 5,649 outstanding
subscribed shares of the corporation, and that there was no dispute as to the
legality of the transfer of the stock certificate to Jureidini, all of which bear
the signatures of the president and the secretary as required by the
Corporation Law with the proper indorsements of the respective owners
appearing thereon. Exhibits "B-1" to "B-4" are specifically indorsed to her
while Exhibits "B-2" and "B-3" are indorsed in blank. Aquilino Rivera admitted
the genuineness of the signatures of the officers of the corporation and of an
indorsee therein.
Nonetheless, private respondents attempted several times to register
their stock certificates with the corporation but the latter refused to register
the same. Thus, private respondents filed a special civil action for mandamus
and damages with preliminary mandatory injunction and/or receivership
naming herein petitioners as respondents.

ISSUE/S:

Page 717 of 1072

Whether it is the regular court or the Securities and Exchange


Commission that has jurisdiction over the present controversy.
RULING:
It has already been settled that an intra-corporate controversy would
call for the jurisdiction of the Securities and Exchange Commission. On the
other hand, an intra-corporate controversy has been defined as "one which
arises between a stockholder and the corporate. This Court has also ruled
that cases of private respondents who are not shareholders of the
corporation, cannot be a "controversy arising out of intra-corporate or
partnership relations between and among stockholders, members or
associates.
Under Batas Pambansa Blg. 68 otherwise known as "The Corporation
Code of the Philippines," shares of stock are transferred as by delivery to the
transferee of the certificate properly indorsed. 'Title may be vested in the
transferee by delivery of the certificate with a written assignment or
indorsement thereof.
The refusal of petitioner Rivera to indorse the shares of stock in
question and the refusal of the Corporation to register private respondents'
shares in its books shows that the present controversy is not an intracorporate controversy; private respondents are not yet stockholders; they
are only seeking to be registered as stockholders because of an alleged sale
of shares of stock to them. Therefore, as the petition is filed by outsiders not
yet members of the corporation, jurisdiction properly belongs to the regular
courts.

Page 718 of 1072

TOPIC: TRANSFER OF SHARES OF STOCK AND REGISTRATION


JOSEFA SANTAMARIA, assisted by her husband, FRANCISCO
SANTAMARIA, Jr., plaintiff-appellee,
vs.
THE HONGKONG AND SHANGHAI BANKING CORPORATION and R. W.
TAPLIN, defendants-appellant.
G.R. No. L-2808. August 31, 1951
89 PHIL 781
FACTS:
Sometime in February, 1937, Mrs. Josefa T. Santamaria bought 10,000
shares of the Batangas Minerals, Inc., through the offices of Woo, Uy-Tioco &
Naftaly, a stock brokerage firm and pay therefore the sum of P8,041.20. The
buyer received Stock Certificate No. 517, issued in the name of Woo, UyTioco & Naftaly and indorsed in bank by this firm.
On March 9, 1937, Mrs. Santamaria placed an order for the purchase of
10,000 shares of the Crown Mines, Inc. with R.J. Campos & Co., a brokerage
firm, and delivered Certificate No. 517 to the latter as security therefore with
the understanding that said certificate would be returned to her upon
payment of the 10,000 Crown Mines, Inc. shares. The receipt of the
certificate in question signed by one Mr. Cosculluela, Manager of the R.J.
Campos & Co., Inc. According to certificate issued to, R. J. Campos & Co., Inc.
bought for Mrs. Josefa Santamaria 10,000 shares of the Crown Mines, Inc. at .
225 a share, or the total amount of P2,250.
At the time of the delivery of a stock Certificate No. 517 to R.J. Campos
& Co., Inc. this certificate was in the same condition as that when Mrs.
Santamaria received from Woo, Uy-Tioco & Naftaly, with the sole difference
that her name was later written in lead pencil on the upper right hand corner
thereof.Two days later, on March 11, Mrs. Santamaria went to R.J. Campos &
Co., Inc. to pay for her order of 10,000 Crown Mines shares and to get back
Certificate No. 517. Cosculluela then informed her that R.J. Campos & Co.,
Inc. was no longer allowed to transact business due to a prohibition order
from Securities and Exchange Commission. She was also informed that her
Stock certificate was in the possession of the Hongkong and Shanghai
Banking Corporation.
Certificate No. 517 came into possession of the Hongkong and
Shanghai Banking Corporation because R.J. Campos & Co., Inc. had opened
an overdraft account with this bank and to this effect it had executed on April
16, 1936 a document of hypothecation, by the term of which R.J. Campos &

Page 719 of 1072

Co., Inc. pledged to the said bank "all stocks, shares and securities which
I/we may hereafter come into their possession of my/our account and
whether originally deposited for safe custody only or for any other purpose
whatever or which may hereinafter be deposited by me/us in lieu of or in
addition to the Stocks Shares and Securities now deposited or for any other
purposes whatsoever."
Certificate No. 517 already indorsed by R.J. Campos Co. Inc. to the
Hongkong & Shanghai Banking Corporation, was sent by the latter to the
office of the Batangas Minerals, Inc. with the request that the same be
cancelled and a new certificate be issued in the name of R.W. Taplin as
trustee and nominee of the banking corporation. Robert W. Taplin was an
officer of this institution in charge of the securities belonging to or claimed
by the bank. As per this request the Batangas Minerals, Inc. on March 12,
1937, issued Certificate No. 715 in lieu of Certificate No. 517, in the name of
Robert W. Taplin as trustee and nominee of the Hongkong & Shanghai
Banking Corporation.
According to Mrs. Santamaria, she made the claim to the bank for her
certificate, though she did not remember the exact date, but it was most
likely on the following day of that when she went to Cosculluela for the
purpose of paying her order for 10,000 shares of the Crown Mines, Inc., or
else on March 13, 1937. In her interview with Taplin, the bank's
representative, she informed him that the certificate belonged to her, and
she demanded that it be returned to her. Taplin then replied that the bank
did not know anything about the transaction had between her and R.J.
Campos & Co., Inc., and that he could not do anything until the case of the
bank with Campos shall have been terminated. This declaration was not
contradicted by the adverse party.
In Civil Case No. 51224, R.J. Campos & Co., Inc. was declared insolvent,
and on July 12, 1937, the Hongkong & Shanghai Banking Corporation asked
permission in the insolvency court to sell the R.J. Campos & Co., Inc.,
securities listed in its motion by virtue of the document of hypothecation. In
an order dated July 15, 1937, the insolvency court granted this motion.
ISSUE/S:
Whether or not Santamaria is estopped from claiming her right over
the transferred certificate of stocks to the bank.
RULING:
Certificate of stock No. 517 was made out in the name of Wo, Uy-Tioco
& Naftaly, brokers, and was duly indorsed in bank by said brokers. This
certificate of stock was delivered by petitioner to R.J. Campos & Co., Inc. to
comply with a requirement that she deposit something on account if she
wanted to buy 10,000 shares of Crown Mines Inc. In making said deposit,

Page 720 of 1072

petitioner did not take any precaution to protect herself against the possible
misuse of the shares represented by the certificate of stock. She could have
asked the corporation that had issued said certificate to cancel it and issue
another in lieu thereof in her name to apprise the holder that she was the
owner of said certificate. This she failed to do, and instead she delivered said
certificate, as it was, to R.J. Campos & Co., Inc., thereby clothing the latter
with apparent title to the shares represented by said certificate including
apparent authority to negotiate it by delivering it to said company while it
was indorsed in blank by the person or firm appearing on its face as the
owner thereof.
The defendant Bank had no knowledge of the circumstances under
which the certificate of stock was delivered to R.J. Campos & Co., Inc., and
had a perfect right to assume that R.J. Campos & Co., Inc. was lawfully in
possession of the certificate in view of the fact that it was a street certificate,
and was in such form as would entitle any possessor thereof to a transfer of
the stock on the books of the corporation concerned. There is no question
that, in this case, plaintiff made the negotiation of the certificate of stock to
other parties possible and the confidence she placed in R.J. Campos & Co.,
Inc. made the wrong done possible. This was the proximate cause of the
damage suffered by her. She is, therefore, estopped from claiming further
title to or interest therein as against a bona fide pledge or transferee thereof,
for it is a well-known rule that a bona fide pledgee or transferee of a stock
from the apparent owner is not chargeable with knowledge of the limitations
placed on it by the real owner, or of any secret agreement relating to the use
which might be made of the stock by the holder.

Page 721 of 1072

TOPIC: TRANSFER OF SHARES OF STOCK AND REGISTRATION


MANUEL A. TORRES, JR., (Deceased), GRACIANO J. TOBIAS, RODOLFO
L. JOCSON, JR., MELVIN S. JURISPRUDENCIA, AUGUSTUS CESAR
AZURA and EDGARDO D. PABALAN, petitioners,
vs. C
OURT OF APPEALS, SECURITIES AND EXCHANGE COMMISSION,
TORMIL REALTY & DEVELOPMENT CORPORATION, ANTONIO P.
TORRES, JR., MA. CRISTINA T. CARLOS, MA. LUISA T. MORALES and
DANTE D. MORALES, respondents.
G.R. No. 120138. September 5, 1997
278 SCRA 643
FACTS:
The late Manuel A. Torres, Jr. (Judge Torres for brevity) was the majority
stockholder of Tormil Realty & Development Corporation while private
respondents who are the children of Judge Torres' deceased brother Antonio
A. Torres, constituted the minority stockholders.
In 1984, Judge Torres, in order to make substantial savings in taxes,
adopted an "estate planning" scheme under which he assigned to Tormil
Realty & Development Corporation (Tormil for brevity) various real properties
he owned and his shares of stock in other corporations in exchange for
225,972 Tormil Realty shares.
The 1987 annual stockholders meeting and election of directors of
Tormil corporation was scheduled on 25 March 1987 in compliance with the
provisions of its by-laws.
Pursuant thereto, Judge Torres assigned from his own shares, one (l)
share each to petitioners Tobias, Jocson, Jurisprudencia, Azura and Pabalan.
These assigned shares were in the nature of "qualifying shares," for the sole
purpose of meeting the legal requirement to be able to elect them (Tobias
and company) to the Board of Directors as Torres' nominees.

ISSUE/S:
Whether or not the assignment of "qualifying shares" to the nominees
of the late Judge Torres (herein petitioners) was made in accordance

Page 722 of 1072

with the stringent requirements of the law with respect to the


recording of the transfer of said shares.
RULING:
Contrary to the generally accepted corporate practice, the stock and
transfer book of TORMIL was not kept by Ms. Maria Cristina T. Carlos, the
corporate secretary but by respondent Torres, the President and Chairman of
the Board of Directors of TORMIL. Further, the stock and transfer book was
not kept at the principal office of the corporation either but at the place of
respondent Torres.
The fact that respondent Torres holds 81.28% of the outstanding
capital stock of TORMIL is of no moment and is not a license for him to
arrogate unto himself a duty lodged to (sic) the corporate secretary.
All corporations, big or small, must abide by the provisions of the
Corporation Code. Being a simple family corporation is not an exemption.
Such corporations cannot have rules and practices other than those
established by law.

Page 723 of 1072

TOPIC: TRANSFER OF SHARES OF STOCK AND REGISTRATION


LEE E. WON alias RAMON LEE, plaintiff-appellant,
vs.
WACK WACK GOLF and COUNTRY CLUB, INC., defendant-appellee.
G.R. No. L-10122. August 30, 1958
104 PHIL 466
FACTS:
On December 2, 1942, the defendant (a non-stock corporation) issued
to Iwao Teruyama Membership Certificate No. 201 which was assigned to M.
T. Reyes on April 22, 1944. Subsequently in the same year 1944, M. T. Reyes
transferred and assigned said certificate to the plaintiff. On April 26, 1955,
the plaintiff filed an action in the Court of First Instance of Manila against the
defendant, alleging that shortly after the rehabilitation of the defendant after
the war, the plaintiff asked the defendant to register in its books the
assignment in favor of the plaintiff and to issue to the latter a new
certificate, but that the defendant had refused and still refuses to do so
unlawfully; and praying that the plaintiff be declared the owner of one share
of stock of the defendant and that the latter be ordered to issue a
correspondent new certificate.
On June 6, 1955, the defendant filed a motion to dismiss, alleging that
from 1944, when the plaintiff's right of action had accrued, to April 26, 1955,
when the complaint was filed, eleven years have elapsed, and that therefore
the complaint was filed beyond the 5-year period fixed in Article 1149 of the
Civil Code. On July 30, 1955, the Court of First Instance of Manila issued an
order dismissing the complaint. As plaintiff's motion for reconsideration filed
on August 27, 1955 and second motion for reconsideration filed on
September 13, 1955, were both denied, the plaintiff has taken the present
appeal.
ISSUE/S:
Whether the corporations complaint constitute a valid ground for an
action of interpleader.
RULING:
It is the general rule that a bill of interpleader comes too late when it is
filed after judgment has been rendered in favor of one of the claimants of the

Page 724 of 1072

fund, this being especially true when the holder of the funds had notice of
the conflicting claims prior to the rendition of the judgment and had an
opportunity to implead the adverse claimants in the suit in which the
judgment was rendered. Moreover, A stockholder should use reasonable
diligence to hale the contending claimants to court. He need not await
actual institution of independent suits against him before filing a bill of
interpleader. He should file an action of interpleader within a reasonable
time after a dispute has arisen without waiting to be sued by either of the
contending claimants. Otherwise, he may be barred by laches or undue
delay. But where he acts with reasonable diligence in view of the
environmental circumstances, the remedy is not barred.
In the case at bar, the Corporation did not act with diligence, in view of
all the circumstances, such that it may properly invoke the remedy of
interpleader. It was aware of the conflicting claims of the appellees with
respect to the membership fee certificate 201 long before it filed the present
interpleader suit. It had been recognizing Tan as the lawful owner thereof. It
was sued by Lee who also claimed the same membership fee certificate. Yet
it did not interplead Tan. It preferred to proceed with the litigation (civil case
26044) and to defend itself therein. As a matter of fact, final judgment was
rendered against it and said judgment has already been executed. It is not
therefore too late for it to invoke the remedy of interpleader. Moreover, The
Corporation has not shown any justifiable reason why it did not file an
application for interpleader in civil case 26044 to compel the appellees
herein to litigate between themselves their conflicting claims of ownership. It
was only after adverse final judgment was rendered against it that the
remedy of interpleader was invoked by it. By then it was too late, because he
is entitled to this remedy the applicant must be able to show that lie has not
been made independently liable to any of the claimants. And since the
Corporation is already liable to Lee under a final judgment, the present
interpleader suit is clearly improper and unavailing.
Under section 120 of the Code of Civil Procedure, The action of
interpleader is a remedy whereby a person who has personal property in his
possession, or an obligation to render wholly or partially, without claiming
any right to either, comes to court and asks that the persons who claim the
said personal property or who consider themselves entitled to demand
compliance with the obligation, be required to litigate among themselves in
order to determine finally who is entitled to tone or the one thing. The
remedy is afforded to protect a person not against double liability but against
double vexation in respect of one liability. The procedure under the Rules of
Court is the same as that under the Code of Civil Procedure, except that
under the former the remedy of interpleader is available regardless of the
nature of the subject-matter of the controversy, whereas under the latter an
interpleader suit is proper only if the subject-matter of the controversy is
personal property or relates to the performance of an obligation.

Page 725 of 1072

TOPIC: LOST OR DESTROYED CERTIFICATES


PHILEX MINING CORPORATION, petitioner,
vs.
HON. DOMINGO CORONEL REYES, Presiding Judge, Court of First
Instance of Albay, 10th Judicial District, Branch IV, and RICHARD
HUENEFELD, respondents.
G.R. No. L-57707. November 19, 1982
118 SCRA 602
FACTS:
Private respondent, Richard Huenefeld, is a stockholder of petitioner
Philex Mining Corporation (Philex, for short). He originally owned 800,000
shares of stock. Philex declared a 10% stock dividend. Stock Certificate No.
190579 for 80,000 shares was issued by Philex in favor of Huenefeld. Philex
sent the stock certificate to Huenefeld through its transfer agent, First Asian,
Stock Transfer, Inc. (First Asian, for brevity). Huenefeld claims that he never
received the stock certificate.
First Asian wrote Huenefeld informing him that the stock certificate had
been delivered to him at his address at Michelle Apartment, 2030 A. Mabini
Street, Manila; and that if the certificate could not be located that Huenefeld
execute an Affidavit of Loss, with the notice of loss to be published once a
week for three (3) consecutive weeks in a newspaper of general circulation in
accordance with the procedure prescribed BY Republic Act No. 201 (now
Section 73, Corporation Code).
Huenefeld, through counsel, replied that RA 201 is not applicable
because the stock certificate was not lost in the possession of the
stockholder; that assuming it was, the expenses of publication and premiums
for the bond should be at Philex's expense; and demanded the issuance of a
replacement stock certificate. Huenefeld also submitted an Affidavit of Loss
but did not comply with the other requirements on publication.
Thus, Huenefeld commenced suit for Specific Performance with Damages
against Philex, First Asian and/or the latter's General Manager, before the
Court of First Instance of Albay, Branch IV, Legaspi City (Civil Case No. 6400),

Page 726 of 1072

presided by respondent Judge, to compel the issuance of a replacement for


Stock Certificate No. 190579, plus damages.

ISSUE/S:
Whether or not a corporation is bound to replace a stockholder's lost
certificate of stock.
RULING:
Section 5 of Presidential Decree No. 902-A provides:
Sec. 5. In addition to the regulatory and adjudicative
functions of the Securities and Exchange Commission over
corporations, partnerships and other forms of associations
registered with it as expressly granted under existing laws and
decrees; it shall have original and exclusive jurisdiction to hear
and decide cases involving:
a) ...
b) Controversies arising out of intra-corporate or
partnership
relations,
between
and
among
stockholders, members, or associates; between any
or all of them and the corporation, partnership or
association of which they are stockholders, members,
or associates, respectively and between such
corporation, partnership or association and the state
insofar as it concerns their individual franchise or
right to exist as such entity
Evident from the foregoing is that an intra-corporate controversy is one
which arises between a stockholder and the corporation. There is no
distinction, qualification, nor any exemption whatsoever. The provision is
broad and covers all kinds of controversies between stockholders and
corporations. The issue of whether or not a corporation is bound to replace a
stockholder's lost certificate of stock is a matter purely between a
stockholder and the corporation. It is a typical intra-corporate dispute. The
question of damages raised is merely incidental to that main issue.
The foregoing interpretation of Huenefeld does not square with the
intent of the law, which is to segregate from the general jurisdiction of
regular Courts controversies involving corporations and their stockholders
and to bring them to the SEC for exclusive resolution, in much the same way
that labor disputes are now brought to the Ministry of Labor and Employment
(MOLE) and the National Labor Relations Commission (NLRC), and not to the
Courts.

Page 727 of 1072

TOPIC: CORPORATE BOOKS AND RECORDS(STOCK AND TRANSFER BOOK)


NORA A. BITONG, petitioner,
vs.
COURT OF APPEALS (FIFTH DIVISION), EUGENIA D. APOSTOL, JOSE A.
APOSTOL, MR. & MS. PUBLISHING CO., LETTY J. MAGSANOC, AND
ADORACION G. NUYDA, respondents.
G.R. No. 123553. July 13, 1998
292 SCRA 503
FACTS:
Petitioner Nora A. Bitong is the Treasurer and a Member of the Board of
Directors of Mr. & Ms. from the time it was incorporated on 29 October 1976
to 11 April 1989, and was the registered owner of 1,000 shares of stock out
of the 4,088 total outstanding shares, petitioner complained of irregularities
committed from 1983 to 1987 by Eugenia D. Apostol, President and
Chairperson of the Board of Directors. Petitioner claimed that except for the
sale of the name Philippine Inquirer to Philippine Daily Inquirer (PDI
hereafter) all other transactions and agreements entered into byMr. & Ms.
with PDI were not supported by any bond and/or stockholders' resolution.
And, upon instructions of Eugenia D. Apostol, Mr. & Ms. made several cash
advances to PDI on various occasions amounting to P3.276 million. On some
of these borrowings PDI paid no interest whatsoever. Despite the fact that
the advances made by Mr. & Ms. to PDI were booked as advances to an
affiliate, there existed no board or stockholders' resolution, contract nor any
other document which could legally authorize the creation of and support to
an affiliate.
Respondents Eugenia and Jose Apostol were stockholders, directors
and officers in both Mr. & Ms. and PDI. In fact on 2 May 1986 respondents
Eugenia D. Apostol, Leticia J. Magsanoc and Adoracion G. Nuyda subscribed
to PDI shares of stock at P50,000.00 each or a total of P150,000.00. The
stock subscriptions were paid for by Mr. & Ms. and initially treated, as

Page 728 of 1072

receivables from officers and employees. But, no payments were ever


received from respondents, Magsanoc and Nuyda.
The petition principally sought to
a. enjoin respondents Eugenia D. Apostol and Jose A. Apostol
from further acting as president-director and director,
respectively, of Mr. & Ms. and disbursing any money or
funds except for the payment of salaries and similar
expenses in the ordinary course of business, and from
disposing of their Mr. & Ms. shares;
b. enjoin respondents Apostol spouses, Magsanoc and Nuyda
from disposing of the PDI shares of stock registered in their
names;
c. compel respondents Eugenia and Jose Apostol to account
for and reconvey all profits and benefits accruing to them
as a result of their improper and fraudulent acts;
d. compel respondents Magsanoc and Nuyda to account for
and reconvey to Mr. & Ms. all shares of stock paid from
cash advances from it and all accessions or fruits thereof;
e. hold respondents Eugenia and Jose Apostol liable for
damages suffered by Mr. & Ms. and the other stockholders,
including petitioner, by reason of their improper and
fraudulent acts;
f. appoint a management committee for Mr. & Ms. during the
pendency of the suit to prevent further dissipation and loss
of its assets and funds as well as paralyzation of business
operations; and,
g. direct the management committee for Mr. & Ms. to file the
necessary action to enforce its rights against PDI and other
third parties.
ISSUE/S:
Whether petitioner was a holder of the proper certificates of shares of
stock and that the transfer was recorded in the Stock and Transfer
Book.
HELD:
In this case, the Certificate of Stock No. 008 was only legally issued on
17 March 1989 when it was actually signed by the President of the
corporation, and not before that date. While a certificate of stock is not
necessary to make one a stockholder, e.g., where he is an incorporator and
listed as stockholder in the articles of incorporation although no certificate of
stock has yet been issued, it is supposed to serve as paper representative of
the stock itself and of the owner's interest therein. Hence, when Certificate of

Page 729 of 1072

Stock No. 008 was admittedly signed and issued only on 17 March 1989 and
not on 25 July 1983, even as it indicates that petitioner owns 997 shares of
stock of Mr. & Ms., the certificate has no evidentiary value for the purpose of
proving that petitioner was a stockholder since 1983 up to 1989.
And even the factual antecedents of the alleged ownership by
petitioner in 1983 of shares of stock of Mr. & Ms. are indistinctive if not
enshrouded in inconsistencies. In her testimony before the Hearing Panel,
petitioner said that early in 1983, to relieve Mr. & Ms. from political pressure,
Senator Enrile decided to divest the family holdings in Mr. & Ms. as he was
then part of the government and Mr. & Ms. was evolving to be an opposition
newspaper. The JAKA shares numbering 1,000 covered by Certificate of Stock
No. 001 were thus transferred to respondent Eugenia D. Apostol in trust or in
blank.
TOPIC:INSPECTION OF CORPORATE BOOKS AND RECORDS

SY TIONG SHIOU, JUANITA TAN SY, JOLIE ROSS TAN, ROMER TAN,
CHARLIE TAN, AND JESSIE JAMES TAN,
VS. S
Y CHIM AND FELICIDAD CHAN SY
G.R. NO. 174168.MARCH 30, 2009
FACTS:
Four criminal complaints were filed by Spouses Sy against Sy Tiong
Shiou, et al. for alleged violation of Section 74 in relation to Section 144 of
the Corporation Code. In these complaints, the Spouses Sy averred that they
are stockholders and directors of Sy Siy Ho & Sons, Inc. who asked Sy Tiong
Shiou, et al., officers of the corporation, to allow them to inspect the books
and records of the business on three occasions to no avail .Sy Tiong Shiou, et
al. denied the request, citing civil and intra-corporate cases pending in court.
In the two other complaints, Sy Tiong Shiou was charged with falsification
under Article 172, in relation to Article 171 of RPC, and perjury. d any
conveyance of their shares.
The investigating prosecutor issued a resolution recommending the
suspension of the criminal complaints for violation of the Corporation Code
and the dismissal of the criminal complaints for falsification and perjury
against Sy Tiong Shiou. The reviewing prosecutor approved the resolution.
ISSUE/S:
Whether or not the denial of inspection of corporate books and records
is proper.

Page 730 of 1072

RULING:
In the recent case of Ang-Abaya, et al. v. Ang, et al., the Court had the
occasion to enumerate the requisites before the penal provision under
Section 144 of the Corporation Code may be applied in a case of violation of
a stockholder or members right to inspect the corporate books/records as
provided for under Section 74 of the Corporation Code. The elements of the
offense, as laid down in the case, are:
First. A director, trustee, stockholder or member has made a
prior demand in writing for a copy of excerpts from the
corporations records or minutes;
Second. Any officer or agent of the concerned corporation shall
refuse to allow the said director, trustee, stockholder or member
of the corporation to examine and copy said excerpts;
Third. If such refusal is made pursuant to a resolution or order of
the board of directors or trustees, the liability under this section
for such action shall be imposed upon the directors or trustees
who voted for such refusal; and,
Fourth. Where the officer or agent of the corporation sets up the
defense that the person demanding to examine and copy
excerpts from the corporations records and minutes has
improperly used any information secured through any prior
examination of the records or minutes of such corporation or of
any other corporation, or was not acting in good faith or for a
legitimate purpose in making his demand, the contrary must be
shown or proved.
Thus, in a criminal complaint for violation of Section 74 of the
Corporation Code, the defense of improper use or motive is in the nature of a
justifying circumstance that would exonerate those who raise and are able to
prove the same. Accordingly, where the corporation denies inspection on the
ground of improper motive or purpose, the burden of proof is taken from the
shareholder and placed on the corporation. However, where no such
improper motive or purpose is alleged, and even though so alleged, it is not
proved by the corporation, then there is no valid reason to deny the
requested inspection.
In the instant case, however, the Court finds that the denial of
inspection was predicated on the pending civil case against the Spouses Sy.
Even in their Joint Counter-Affidavit dated 23 September 2003, 33 Sy Tiong
Shiou, et al. did not make any allegation that "the person demanding to
examine and copy excerpts from the corporations records and minutes has

Page 731 of 1072

improperly used any information secured through any prior examination of


the records or minutes of such corporation or of any other corporation, or
was not acting in good faith or for a legitimate purpose in making his
demand." Instead, they merely reiterated the pendency of the civil case.
There being no allegation of improper motive, and it being undisputed that
Sy Tiong Shiou, et al. denied Sy Chim and Felicidad Chan Sys request for
inspection, the Court rules and so holds that the DOJ erred in dismissing the
criminal charge for violation of Section 74 in relation to Section 144 of the
Corporation Code.

TOPIC:INSPECTION OF CORPORATE BOOKS AND RECORDS


VICTOR AFRICA
VS.
PRESIDENTIAL COMMISSION ON GOOD GOVERNMENT, JOSE LAURETA,
MELQUIADES GUTIERREZ, EDUARDO M. VILLANUEVA, EDUARDO DE
LOS ANGELES and ROMAN MABANTA, JR.
G.R. No. 83831. January 9, 1992
FACTS:
ETPI was sequestered by the PCGG. A partial lifting of the
sequestration was granted wherein 40% of the shares of stock owned by
Cable and Wireless, Ltd were freed from the effects of sequestration. The
PCGG then called for an annual SH meeting wherein the PCGG nominee and
the Cable and Wireless nominee and Jose Africa were elected as members
of the BOD. Petitioners in their capacity as erstwhile members of the Board
of Directors of ETPI instituted before the Sandiganbayan on September 23,
1988 Civil Case No. 0050, another action for injunction and damages
perpetually restraining the PCGG from electing, designating and supporting
the defendants in their ETPI roles. During the pendency of the case the Clerk
issued a subpoena duces tecum and adtestificandum ordering the PCGG or
its representatives to appear and testify before the Sandiganbayan during
the hearing on November 3, 1988 at 2:00 p.m. and to produce the stock and
transfer book and all stubs of the outstanding stock certificates of ETPI.
The PCGG moved to quash but it was denied.
ISSUE/S:
May the SHAREHOLDERS inspect the books of EPTI?

Page 732 of 1072

RULING:
The issue raised in the original petition in G.R. No. 85594 relating to
the validity of the issuance by the Sandiganbayan of the subpoena duces
tecum and ad testificandum ordering the PCGG or its representative to
testify and produce the stock and transfer book was laid to rest by the joint
resolution in two cases, both entitled Republic vs. Sandiganbayanand
Eduardo Cojuangco, Jr., which applies squarely in the instant petitions.
In
those cases, the SC ruled that sequestration does not deprive a stockholders
right to inspect the books of the corporation.
In upholding therein the right of a stockholder of a sequestered
company to inspect and/or examine the records of a corporation pursuant to
Section 74 of the Corporation Code, the Court found nothing in Executive
Orders Nos. 1, 2 and 14, as well as in BASECO, to indicate an implied
amendment of the Corporation Code, much less an implied modification of a
stockholder's right of inspection as guaranteed by Section 74 thereof. The
only express limitation on the right of inspection, according to the Court, is
that
(1) the right of inspection should be exercised at
reasonable hours on business days;
(2) the person demanding the right to examine and
copy excerpts from the corporate records and
minutes has not improperly used any information
secured through any previous examination of the
records of such corporation; and
(3) the demand is made in good faith or for a
legitimate
(4) purpose.

Page 733 of 1072

TOPIC:INSPECTION OF CORPORATE BOOKS AND RECORDS


REPUBLIC OF THE PHILIPPINES, (PRESIDENTIAL COMMISSION ON
GOOD GOVERNMENT)
Vs.
THE HONORABLE SANDIGANBAYAN (FIRST DIVISION) AND EDUARDO
COJUANGCO, JR.
G.R. No. 88809. July 10, 1991
FACTS:
Private respondent-stockholder requested the SMC and its corporate
secretary the production, inspection, examination/verification and/or
photocopying of the SMC corporate records to inform him of the decisions,
policies, acts and performance of the management of the SMC under the
PCGG-Board.
Since the shares of private respondent in the SMC have been
sequestered by the PCGG, the former (SMC) sought advice from the latter on
the effect of such sequestration. Subsequently, private respondent was
informed by the SMC that all requests for the examination, inspection and
photocopying of its corporate records should be coursed through the PCGG.
The request of private respondent for the inspection/examination of SMC's
corporate records was denied by the PCGG. As regards the corporate records
of URPB, private respondent was likewise advised to course his request
through the PCGG.
ISSUE/S:
Whether or not the PCGG may validly refuse private respondent's right
to inspection
RULING:

Page 734 of 1072

While it may be true that the right of inspection granted by Sec. 74 of


the Corporation Code is not absolute, as when the stockholder is not acting
in good faith and for a legitimate purpose (Gonzales v. PNB, 122 SCRA 489
[1983]); or when the demand is purely speculative or merely to satisfy
curiosity (Grey v. Insular Lumber Co., 40 O.G., No. 31st Supp. 1 [1939]; See
also State ex rel. Thiele v. Cities Service Co. (115 A. 773 [1922]), the same
may not be said in the case of private respondent. This is because:
. . . the "impropriety of purpose such as will defeat
enforcement must be set up (by) the corporation
defensively if the Court is to take cognizance of it as a
qualification. In other words, the specific provisions take
from the stockholder the burden of showing impropriety of
purpose or motive. (Gokongwei, Jr., v. Securities and
Exhange Commission, supra; citing State v. Monida&
Yellowstone Stage Co., 110 Minn. 193, 124 NW 791; State
v. Cities Service Co., 114 A 463.)
In the case at bar, petitioner failed to discharge the burden of proof to
show that private respondent's action in seeking examination of the
corporate records was moved by unlawful or ill-motivated designs which
could appropriately call for a judicial protection against the exercise of such
right. Save for its unsubstantiated allegations, petitioner could offer no proof,
nay, not even a scintilla of evidence that respondent Cojuangco, Jr., was
motivated by bad faith; that the demand was for an illegitimate purpose or
that the demand was impelled by speculation or idle curiosity. Surely,
respondent's substantial share

Page 735 of 1072

TOPIC:INSPECTION OF CORPORATE BOOKS AND RECORDS


JOHN GOKONGWEI, JR,.
VS.
SECURITIES AND EXCHANGE COMMISSION, ANDRES M. SORIANO,
JOSE M. SORIANO, ENRIQUE ZOBEL, ANTONIO ROXAS, EMETERIO
BUNAO, WALTHRODE B. CONDE, MIGUEL ORTIGAS, ANTONIO PRIETO,
SAN MIGUEL CORPORATION, EMIGDIO TANJUATCO, SR., and
EDUARDO R. VISAYA
G.R. No. L-45911, April 11, 1979
FACTS:
Petitioner filed with SEC an "Urgent Motion for Production and
Inspection of Documents", alleging that the Secretary of respondent
corporation refused to allow him to inspect its records despite request made
by petitioner for production of certain documents enumerated in the request,
and that respondent corporation had been attempting to suppress
information from its stockholders despite a negative reply by the SEC to its
query regarding their authority to do so. Among the documents requested to
be copied were
a. minutes of the stockholder's meeting held on March 13,
1961;
b. copy of the management contract between San Miguel
Corporation and A. Soriano Corporation (ANSCOR);
c. latest balance sheet of San Miguel International, Inc.;
d. authority of the stockholders to invest the funds of
respondent corporation in San Miguel International, Inc.;
and
e. lists of salaries, allowances, bonuses, and other
compensation, if any, received by Andres M. Soriano, Jr.
and/or its successor-in-interest.

Page 736 of 1072

SEC resolved the motion for production and inspection of documents


by ordering the respondents to produce and permit the inspection, copying
and photographing, by or on behalf of the petitioner-movant, John
Gokongwei, Jr., of the minutes of the stockholders' meeting of the respondent
San Miguel Corporation held on March 13, 1961. However, As to the Balance
Sheet of San Miguel International, Inc. as well as the list of salaries,
allowances, bonuses, compensation and/or remuneration received by
respondent Jose M. Soriano, Jr. and Andres Soriano from San Miguel
International, Inc. and/or its successors-in-interest, the Petition to produce
and inspect the same is hereby DENIED, as petitioner-movant is not a
stockholder of San Miguel International, Inc. and has, therefore, no inherent
right to inspect said documents.
ISSUE/S:
Whether or not respondent SEC gravely abused its discretion in
denying petitioner's request for an examination of the records of San
Miguel International Inc.
RULING:
Pursuant to the second paragraph of section 51 of the Corporation Law,
"(t)he record of all business transactions of the corporation and minutes of
any meeting shall be open to the inspection of any director, member or
stockholder of the corporation at reasonable hours."
In the case at bar, considering that the foreign subsidiary is wholly
owned by respondent San Miguel Corporation and, therefore, under its
control, it would be more in accord with equity, good faith and fair dealing to
construe the statutory right of petitioner as stockholder to inspect the books
and records of the corporation as extending to books and records of such
wholly subsidiary which are in respondent corporation's possession and
control.

Page 737 of 1072

TOPIC:INSPECTION OF CORPORATE BOOKS AND RECORDS


RAMON A. GONZALES
VS.
THE PHILIPPINE NATIONAL BANK
G.R. No.L-33320. May 30, 1983
FACTS:
Ramon A. Gonzales instituted a special civil action for mandamus
against the herein respondent praying that the latter be ordered to allow him
to look into the books and records of the respondent bank in order to satisfy
himself as to the truth of the published reports that the respondent has
guaranteed the obligation of Southern Negros Development Corporation in
the purchase of a US$ 23 million sugar-mill to be financed by Japanese
suppliers and financiers; that the respondent is financing the construction of
the P 21 million Cebu-Mactan Bridge to be constructed by V.C. Ponce, Inc.,
and the construction of Passi Sugar Mill at Iloilo by the Honiron Philippines,
Inc., as well as to inquire into the validity of Id transactions. The petitioner
has alleged that his written request for such examination was denied by the
respondent.
The trial court having dismissed the petition for mandamus, the instant
appeal to review the said dismissal was filed.
ISSUE/S:
Whether or not the petitioner should be allowed to look into the
records of its transactions.

Page 738 of 1072

RULING:
The right of inspection granted to a stockholder under Section 51 of
Act No. 1459 has been retained, but with some modifications. The second
and third paragraphs of Section 74 of Batas Pambansa Blg. 68 provide the
following:
The records of all business transactions of the corporation and the minutes of
any meeting shag be open to inspection by any director, trustee, stockholder
or member of the corporation at reasonable hours on business days and he
may demand, in writing, for a copy of excerpts from said records or minutes,
at his expense.
Any officer or agent of the corporation who shall refuse to allow any
director, trustee, stockholder or member of the corporation to examine and
copy excerpts from its records or minutes, in accordance with the provisions
of this Code, shall be liable to such director, trustee, stockholder or member
for damages, and in addition, shall be guilty of an offense which shall be
punishable under Section 144 of this Code: Provided, That if such refusal is
made pursuant to a resolution or order of the board of directors or trustees,
the liability under this section for such action shall be imposed upon the
directors or trustees who voted for such refusal; and Provided, further, That it
shall be a defense to any action under this section that the person
demanding to examine and copy excerpts from the corporation's records and
minutes has improperly used any information secured through any prior
examination of the records or minutes of such corporation or of any other
corporation, or was not acting in good faith or for a legitimate purpose in
making his demand.
Although the petitioner has claimed that he has justifiable motives in
seeking the inspection of the books of the respondent bank, he has not set
forth the reasons and the purposes for which he desires such inspection,
except to satisfy himself as to the truth of published reports regarding
certain transactions entered into by the respondent bank and to inquire into
their validity. The circumstances under which he acquired one share of stock
in the respondent bank purposely to exercise the right of inspection do not
argue in favor of his good faith and proper motivation. Admittedly he sought
to be a stockholder in order to pry into transactions entered into by the
respondent bank even before he became a stockholder. His obvious purpose
was to arm himself with materials which he can use against the respondent
bank for acts done by the latter when the petitioner was a total stranger to
the same. He could have been impelled by a laudable sense of civic
consciousness, but it could not be said that his purpose is germane to his
interest as a stockholder.

Page 739 of 1072

TOPIC: INSPECTION OF CORPORATE BOOKS AND RECORDS


ANTONIO PARDO
VS.
THE HERCULES LUMBER CO., INC., and IGNACIO FERRER
G.R. NO. L-22442, August 1, 1924
47 Phil 964
FACTS:
Antonio Pardo is a stockholder in the Hercules Lumber Company, Inc..
On the other hand, respondent, Ignacio Ferrer, is the acting secretary of the
said company. The latter has refused to permit the petitioner or his agent to
inspect the records and business transactions of the said Hercules Lumber
Company, Inc., at times desired by the petitioner because according to
Ferrer, the petitioner has not availed himself of the permission to inspect the
books and transactions of the company within the ten days thus defined in a
board resolution. Thus, his right to inspection and examination is lost, at
least for that year.
Hence, Pardo filed a petition for mandamus to compel the respondents
to permit the plaintiff and his duly authorized agent and representative to
examine the records and business transactions of said company.
ISSUE:
Whether or not the refusal of the acting corporate secretary for
Pardo to inspect and examine corporate books is valid.

Page 740 of 1072

RULING:
The general right given by the statute may not be lawfully abridged to
the extent attempted in this resolution. It may be admitted that the officials
in charge of a corporation may deny inspection when sought at unusual
hours or under other improper conditions; but neither the executive officers
nor the board of directors have the power to deprive a stockholder of the
right altogether. A by-law unduly restricting the right of inspection is
undoubtedly invalid. Authorities to this effect are too numerous and direct to
require extended comment. (14 C.J., 859; 7 R.C.L., 325; 4 Thompson on
Corporations, 2nd ed., sec. 4517; Harkness vs. Guthrie, 27 Utah, 248; 107
Am., St. Rep., 664. 681.) Under a statute similar to our own it has been held
that the statutory right of inspection is not affected by the adoption by the
board of directors of a resolution providing for the closing of transfer books
thirty days before an election. (State vs. St. Louis Railroad Co., 29 Mo., Ap.,
301.)
It will be noted that our statute declares that the right of inspection
can be exercised "at reasonable hours." This means at reasonable hours on
business days throughout the year, and not merely during some arbitrary
period of a few days chosen by the directors.

Page 741 of 1072

TOPIC: INSPECTION OF CORPORATE BOOKS AND RECORDS


W. G. PHILPOTTS
VS.
PHILIPPINE MANUFACTURING COMPANY and F. N. BERRY
G.R. NO. L-15568, November 8, 1919
40 Phil 471
FACTS:
The petitioner, W. G. Philpotts, a stockholder in the Philippine
Manufacturing Company, filed an action to obtain a writ of mandamus to
compel the respondents to permit the plaintiff, in person or by some
authorized agent or attorney, to inspect and examine the records of the
business transacted by said company since January 1, 1918.
ISSUE:
Whether or not the right which the law concedes to a stockholder
to inspect the records can be exercised by a proper agent or
attorney of the stockholder as well as by the stockholder in
person
RULING:
The pertinent provision of our law is found in the second paragraph of
section 51 of Act No. 1459, which reads as follows: "The record of all
business transactions of the corporation and the minutes of any meeting
shall be open to the inspection of any director, member or stockholder of the
corporation at reasonable hours."

Page 742 of 1072

This provision is to be read of course in connecting with the related


provisions of sections 51 and 52, defining the duty of the corporation in
respect to the keeping of its records.
Now it is our opinion, and we accordingly hold, that the right of
inspection given to a stockholder in the provision above quoted can be
exercised either by himself or by any proper representative or attorney in
fact, and either with or without the attendance of the stockholder. This is in
conformity with the general rule that what a man may do in person he may
do through another; and we find nothing in the statute that would justify us
in qualifying the right in the manner suggested by the respondents.
In order that the rule above stated may not be taken in too sweeping a
sense, we deem it advisable to say that there are some things which a
corporation may undoubtedly keep secret, notwithstanding the right of
inspection given by law to the stockholder; as for instance, where a
corporation, engaged in the business of manufacture, has acquired a formula
or process, not generally known, which has proved of utility to it in the
manufacture of its products. It is not our intention to declare that the
authorities of the corporation, and more particularly the Board of Directors,
might not adopt measures for the protection of such process form publicity.
There is, however, nothing in the petition which would indicate that the
petitioner in this case is seeking to discover anything which the corporation
is entitled to keep secret; and if anything of the sort is involved in the case it
may be brought out at a more advanced stage of the proceedings.

Page 743 of 1072

TOPIC: INSPECTION OF CORPORATE BOOKS AND RECORDS


REPUBLIC OF THE PHILIPPINES, (PRESIDENTIAL COMMISSION ON
GOOD GOVERNMENT)
VS.
THE HONORABLE SANDIGANBAYAN (FIRST DIVISION) AND EDUARDO
COJUANGCO, JR.
G.R. No. 88809 July 10, 1991
199 SCRA 39
FACTS:
On December 26, 1988, private respondent-stockholder requested the
San Miguel Corporation (SMC) and its corporate secretary the production,
inspection, examination/verification and/or photocopying of the SMC
corporate records to inform him of the decisions, policies, acts and
performance of the management of the SMC under the PCGG-Board.
Since the shares of private respondent in the SMC have been
sequestered by the PCGG, the former (SMC) sought advice from the latter on
the effect of such sequestration. Subsequently, private respondent was
informed by the SMC that all requests for the examination, inspection and
photocopying of its corporate records should be coursed through the PCGG.
The facts set forth in G.R. No. 88809 are substantially similar in G.R.
No. 88858 except that in the latter case, private respondent as stockholder
of record seeks authority to inspect and examine the corporate records of
United Coconut Planters Bank.
The request of private respondent for the inspection/examination of SMC's
corporate records was denied by the PCGG. As regards the corporate records
of URPB, private respondent was likewise advised to course his request
through the PCGG.
Thereafter, private respondent filed two separate petitions for
prohibition and mandamus before the Sandiganbayan seeking to enforce his
stockholder's right to inspect the corporate records of SMC and the UCPB.
Subsequently, respondent Sandiganbayan rendered the assailed resolutions
aforequoted.
ISSUE:
Whether or not sequestration automatically
stockholder of his right of inspection.

deprive

RULING:

Page 744 of 1072

The right of a stockholder to inspect and/or examine the records of a


corporation is explicitly provided in Section 74 of the Corporation Code, the
pertinent portion of which reads:
Sec. 74. Books to be kept; stock transfer agent.
xxx xxx xxx
The records of all business transactions of the corporation and
the minutes of any meeting shall be open to the inspection of
any director, trustee, stockholder or member of the corporation
at reasonable hours on business days and he may demand, in
writing, for a copy of excerpts from said records or minutes, at
his expense.
The PCGG does not become, ipso facto, the owner of the shares just
because the same have been sequestered; nor does it become the
stockholder of record by virtue of such sequestration.
Just recently, we ruled that the PCGG cannot vote the sequestered
shares of respondent Cojuangco, Jr., in San Miguel Corporation (Cojuangco,
Jr., et al., v. Roxas, et al., GR No. 91925, April 16, 1991; Cojuangco, Jr., et al.,
v. Azcuna, et al., GR No. 93005, April 16, 1991). If the PCGG cannot vote the
sequestered shares of private respondent, with much more reason it cannot
restrain or prevent private respondent, as stockholder from inspecting the
corporate records of the SMC and the UCPB at reasonable hours on business
days. The law grants respondent/stockholder such authority.
Petitioner, in seeking to bar private respondent from exercising his
statutory right of inspection, lays emphasis on the argument that
respondent's express purpose is to "supervise" PCGG's management, if not
to gratify his curiosity regarding the performance of the SMC and the UCPB.
Again, the argument is devoid of merit. Records indicate that private
respondent is the ostensible owner of a substantial number of shares and is a
stockholder of record in SMC and UCPB. * Being a stockholder beyond doubt,
there is therefore no reason why private respondent may not exercise his
statutory right of inspection in accordance with Sec. 74 of the Corporation
Code, the only express limitation being that the right of inspection should be
exercised at reasonable hours on business days; 2) the person demanding to
examine and copy excerpts from the corporation's records and minutes has
not improperly used any information secured through any previous
examination of the records of such corporation; and 3) the demand is made
in good faith or for a legitimate purpose. The latter two limitations, however,
must be set up as a defense by the corporation if it is to merit judicial
cognizance. As such, and in the absence of evidence, the PCGG cannot
unilaterally deny a stockholder from exercising his statutory right of
inspection based on an unsupported and naked assertion that private

Page 745 of 1072

respondent's motive is improper or merely for curiosity or on the ground that


the stockholder is not in friendly terms with the corporation's officers.

TOPIC: EFFECTS OF MERGER AND CONSOLIDATION

Page 746 of 1072

BANK OF THE PHILIPPINE ISLANDS, PETITIONER,


VS.
BPI EMPLOYEES UNION-DAVAO CHAPTER-FEDERATION OF UNIONS IN
BPI UNIBANK, RESPONDENT
[G.R. No. 164301: August 10, 2010]
FACTS:
On March 23, 2000, the Bangko Sentral ng Pilipinas approved the
Articles of Merger executed on January 20, 2000 by and between BPI, herein
petitioner, and FEBTC. This Article and Plan of Merger was approved by the
Securities and Exchange Commission.
Pursuant to the Article and Plan of Merger, all the assets and liabilities
of FEBTC were transferred to and absorbed by BPI as the surviving
corporation. FEBTC employees, including those in its different branches
across the country, were hired by petitioner as its own employees, with their
status and tenure recognized and salaries and benefits maintained.
Respondent BPI Employees Union-Davao Chapter - Federation of
Unions in BPI Unibank ("Union,") is the exclusive bargaining agent of BPI's
rank and file employees in Davao City. The former FEBTC rank-and-file
employees in Davao City did not belong to any labor union at the time of the
merger. Prior to the effectivity of the merger, or on March 31, 2000,
respondent Union invited said FEBTC employees to a meeting regarding the
Union Shop Clause (Article II, Section 2) of the existing CBA between
petitioner BPI and respondent Union.
The parties both advert to certain provisions of the existing CBA, which
are quoted below:
ARTICLE II
Section 1. Maintenance of Membership - All employees within
the bargaining unit who are members of the Union on the date of
the effectivity of this Agreement as well as employees within the
bargaining unit who subsequently join or become members of
the Union during the lifetime of this Agreement shall as a
condition of their continued employment with the Bank, maintain
their
membership
in
the
Union
in
good
standing.
Section 2. Union Shop - New employees falling within the
bargaining unit as defined in Article I of this Agreement, who
may hereafter be regularly employed by the Bank shall,
within thirty (30) days after they become regular employees, join
the Union as a condition of their continued employment. It is

Page 747 of 1072

understood that membership in good standing in the Union is a


condition of their continued employment with the Bank.

Some of the former FEBTC employees joined the Union, while others
refused. Later, however, some of those who initially joined retracted their
membership. Respondent Union then sent notices to the former FEBTC
employees who refused to join, as well as those who retracted their
membership, and called them to a hearing regarding the matter. When
refused to attend the hearing, the president of the Union requested BPI to
implement the Union Shop Clause of the CBA and to terminate their
employment pursuant thereto.
The company did not yield to the request of the union and it was
subsequently submitted for voluntary arbitration. Voluntary Arbitrator
Rosalina Letrondo-Montejo, in a Decision dated November 23, 2001, ruled in
favor of petitioner BPI's interpretation that the former FEBTC employees were
not covered by the Union Security Clause of the CBA between the Union and
the Bank on the ground that the said employees were not new employees
who were hired and subsequently regularized, but were absorbed employees
"by operation of law" because the "former employees of FEBTC can be
considered assets and liabilities of the absorbed corporation." The
Voluntary Arbitrator concluded that the former FEBTC employees could not
be compelled to join the Union, as it was their constitutional right to join or
not to join any organization.

ISSUE/S:
Whether or not the former FEBTC employees that were absorbed by
petitioner upon the merger between FEBTC and BPI should be covered
by the Union Shop Clause found in the existing CBA between petitioner
and
respondent
Union.
RULING:
The provision of the Article 248(e) of the Labor Code in point mandates
that nothing in the said Code or any other law should stop the parties from
requiring membership in a recognized collective bargaining agent as a
condition of employment.
However, under law and jurisprudence, the following kinds of
employees are exempted from its coverage, namely, employees who at the
time the union shop agreement takes effect are bona fide members of a
religious organization which prohibits its members from joining labor unions

Page 748 of 1072

on religious grounds;employees already in the service and already members


of a union other than the majority at the time the union shop agreement took
effect; confidential employees who are excluded from the rank and file
bargaining unit;and employees excluded from the union shop by express
terms of the agreement.
Indeed, the situation of the former FEBTC employees in this case
clearly does not fall within the first three exceptions to the application of the
Union Shop Clause discussed earlier. No allegation or evidence of religious
exemption or prior membership in another union or engagement as a
confidential employee was presented by both parties. The sole category
therefore in which petitioner may prove its claim is the fourth recognized
exception or whether the former FEBTC employees are excluded by the
express terms of the existing CBA between petitioner and respondent.
Petitioner insists that the term "new employees," as the same is used
in the Union Shop Clause of the CBA at issue, refers only to employees hired
by BPI as non-regular employees who later qualify for regular
employment and become regular employees, and not those who, as a legal
consequence of a merger, are allegedly automatically deemed regular
employees of BPI. However, the CBA does not make a distinction as to how a
regular employee attains such a status. Moreover, there is nothing in the
Corporation Law and the merger agreement mandating the automatic
employment as regular employees by the surviving corporation in the
merger. Section 80 of the Corporation Code provides:
SEC. 80. Effects of merger or consolidation. - The merger or
consolidation, as provided in the preceding sections shall
have the following effects:
1. The constituent corporations shall become a
single corporation which, in case of merger, shall be
the surviving corporation designated in the plan of
merger; and, in case of consolidation, shall be the
consolidated corporation designated in the plan of
consolidation;
2. The separate existence of the constituent
corporations shall cease, except that of the surviving
or
the
consolidated
corporation;
3. The surviving or the consolidated corporation
shall possess all the rights, privileges, immunities
and powers and shall be subject to all the duties and
liabilities of a corporation organized under this Code;

Page 749 of 1072

4. The surviving or the consolidated corporation


shall thereupon and thereafter possess all the rights,
privileges, immunities and franchises of each of the
constituent corporations; and all property, real or
personal, and all receivables due on whatever
account, including subscriptions to shares and other
choses in action, and all and every other interest of,
or belonging to, or due to each constituent
corporation, shall be taken and deemed to be
transferred to and vested in such surviving or
consolidated corporation without further act or deed;
and
5. The surviving or the consolidated corporation
shall be responsible and liable for all the liabilities
and obligations of each of the constituent
corporations in the same manner as if such surviving
or consolidated corporation had itself incurred such
liabilities or obligations; and any claim, action or
proceeding pending by or against any of such
constituent corporations may be prosecuted by or
against the surviving or consolidated corporation, as
the case may be. Neither the rights of creditors nor
any lien upon the property of any of such constituent
corporations shall be impaired by such merger or
consolidated.

It is apparent that petitioner hinges its argument that the former FEBTC
employees were absorbed by BPI merely as a legal consequence of a merger
based on the characterization by the Voluntary Arbiter of these absorbed
employees as included in the "assets and liabilities" of the dissolved
corporation - assets because they help the Bank in its operation and
liabilities because redundant employees may be terminated and company
benefits will be paid to them, thus reducing the Bank's financial status.
In legal parlance, however, human beings are never embraced in the
term "assets and liabilities." Moreover, BPI's absorption of former FEBTC
employees was neither by operation of law nor by legal consequence of
contract. There was no government regulation or law that compelled the
merger of the two banks or the absorption of the employees of the dissolved
corporation by the surviving corporation. Had there been such law or
regulation, the absorption of employees of the non-surviving entities of the
merger would have been mandatory on the surviving corporation.In the
present case, the merger was voluntarily entered into by both banks
presumably for some mutually acceptable consideration. In fact, the

Page 750 of 1072

Corporation Code does not also mandate the absorption of the employees of
the non-surviving corporation by the surviving corporation in the case of a
merger.
In the case at bar, since the former FEBTC employees are deemed
covered by the Union Shop Clause, they are required to join the certified
bargaining agent, which supposedly has gathered the support of the majority
of workers within the bargaining unit in the appropriate certification
proceeding. Their joining the certified union would, in fact, be in the best
interests of the former FEBTC employees for it unites their interests with the
majority of employees in the bargaining unit. It encourages employee
solidarity and affords sufficient protection to the majority status of the union
during the life of the CBA which are the precisely the objectives of union
security clauses, such as the Union Shop Clause involved herein. We are
indeed not being called to balance the interests of individual employees as
against the State policy of promoting unionism, since the employees, who
were parties in the court below, no longer contested the adverse Court of
Appeals' decision. Nonetheless, settled jurisprudence has already swung the
balance in favor of unionism, in recognition that ultimately the individual
employee will be benefited by that policy. In the hierarchy of constitutional
values, this Court has repeatedly held that the right to abstain from joining a
labor organization is subordinate to the policy of encouraging unionism as an
instrument of social justice.

Page 751 of 1072

TOPIC: EFFECTS OF MERGER AND CONSOLIDATION


PHILIPPINE NATIONAL BANK & NATIONAL SUGAR DEVELOPMENT
CORPORATION, petitioners,
vs.
ANDRADA ELECTRIC & ENGINEERING COMPANY, respondent.
G.R. No. 142936
April 17, 2002
381 S 244
FACTS:
Andrada Electric and Engineering Company (AEEC) is a partnership
duly organized, existing and operating under the laws of the Philippines.
PNB acquired the assets of Pampanga Sugar Mills Inc. (PASUMIL) that
were earlier foreclosed by the Development Bank of the Philippines under LOI
No. 13. That PNB organized the NASUDECO in September 1975, to take
ownership and possession of assets and ultimately nationalize and
consolidate its interest in other PNB controlled sugar mills.
Prior to October 29, 1971, PASUMIL engaged the service of AEEC for
electrical rewinding and repair most of which were partially paid by the
defendant PASUMIL, leaving several unpaid accounts with the AEEC, that
finally on October 29, 1971 AEEC and PASUMIL entered into a contract to be
performed by AEEC.
Total obligation was P777, 263.80, PASUMIL paid only P250, 000.00
leaving a balance of P527, 263.80 and another payment of P14, 000.00, thus
total balance of P513, 263.80. The defendant PNB, PASUMIL and now
NASUDECO failed and refused to pay; that the President of the NASUDECO is
also the Vice-President of the PNB, and this official holds office at the 10 th
Floor of the PNB, Escolta, Manila, and plaintiff besought this official to pay
the outstanding obligation of the defendant PASUMIL, inasmuch as the
defendant PNB and NASUDECO now owned and possessed the assets of the
defendant PASUMIL, and these defendants all benefited from the works, and
the electrical, as well as the engineering and repairs, performed by the
plaintiff; that because of the failure and refusal of the defendants to pay their
just, valid, and demandable obligations, plaintiff suffered actual damages in
the total amount of P513,263.80; and that in order to recover these sums,
the plaintiff was compelled to engage the professional services of counsel, to
whom the plaintiff agreed to pay a sum equivalent to 25% of the amount of
the obligation due by way of attorneys fees. . Accordingly, the plaintiff

Page 752 of 1072

prayed that judgment be rendered against the defendants PNB, NASUDECO,


and PASUMIL to be jointly and severally liable.
PNB and NASUDECO filed a joint motion to dismiss on the ground that
they were not part to the contract, but the motion was denied and decision
was in favor of AEEC. The decision of the lower court was affirmed by the
Court of Appeals.
ISSUE/S:
Whether or not there was a merger or consolidation?
RULING:
NO.
Petitioners contend that their takeover of the operations of PASUMIL
did not involve any corporate merger or consolidation, because the latter
had never lost its separate identity as a corporation.
A consolidation is the union of two or more existing entities to form a
new entity called the consolidated corporation. A merger, on the other hand,
is a union whereby one or more existing corporations are absorbed by
another corporation that survives and continues the combined business.
The merger, however, does not become effective upon the mere
agreement of the constituent corporations. Since a merger or consolidation
involves fundamental changes in the corporation, as well as in the rights of
stockholders and creditors, there must be an express provision of law
authorizing them. For a valid merger or consolidation, the approval by the
Securities and Exchange Commission (SEC) of the articles of merger or
consolidation is required. These articles must likewise be duly approved by a
majority of the respective stockholders of the constituent corporations.
In the case at bar, we hold that there is no merger or consolidation
with respect to PASUMIL and PNB. The procedure prescribed under Title IX of
the Corporation Code was not followed.
In fact, PASUMILs corporate existence, as correctly found by the CA,
had not been legally extinguished or terminated. Further, prior to PNBs
acquisition of the foreclosed assets, PASUMIL had previously made partial
payments to respondent for the formers obligation in the amount of
P777,263.80. As of June 27, 1973, PASUMIL had paid P250,000 to respondent
and, from January 5, 1974 to May 23, 1974, another P14,000.
Neither did petitioner expressly or impliedly agree to assume the debt
of PASUMIL to respondent. LOI No. 11 explicitly provides that PNB shall study
and submit recommendations on the claims of PASUMILs creditors. 62Clearly,

Page 753 of 1072

the corporate separateness between PASUMIL and PNB remains, despite


respondents insistence to the contrary.

TOPIC: EFFECTS OF MERGER AND CONSOLIDATION


CHESTER BABST, petitioner,
vs.
COURT OF APPEALS, BANK OF THE PHILIPPINE ISLANDS, ELIZALDE
STEEL CONSOLIDATED, INC., and PACIFIC MULTI-COMMERCIAL
CORPORATION, respondents.
G.R. No. 99398 & 104625
January 26, 2001
FACTS:
On June 8, 1973, Elizalde Steel Consolidated, Inc. (ELISCON) obtained
from Commercial Bank and Trust Company (CBTC) a loan in the amount of P
8,015,900.84, with interest at the rate of 14% per annum, evidenced by a
promissory note. ELISCON defaulted in its payments, leaving an outstanding
indebtedness in the amount of P2,795,240.67 as of October 31, 1982.
The letters of credit, on the other hand, were opened for ELISCON by
CBTC using the credit facilities of Pacific Multi-Commercial Corporation
(MULTI) with the said bank, pursuant to the Resolution of the Board of
Directors of MULTI.
Subsequently, on September 26, 1978, Antonio Roxas Chua and
Chester G. Babst executed a Continuing Suretyship,whereby they bound
themselves jointly and severally liable to pay any existing indebtedness of
MULTI to CBTC to the extent of P8,000,000.00 each.
Sometime in October 1978, CBTC opened for ELISCON in favor of
National Steel Corporation three (3) domestic letters of credit, which
ELISCON used to purchase tin black plates from National Steel Corporation.
ELISCON defaulted in its obligation to pay the amounts of the letters of
credit, leaving an outstanding account, as of October 31, 1982, in the total
amount of P3, 963,372.08.
On December 22, 1980, the Bank of the Philippine Islands (BPI) and
CBTC entered into a merger, wherein BPI, as the surviving corporation,
acquired all the assets and assumed all the liabilities of CBTC.

Page 754 of 1072

Meanwhile, ELISCON encountered financial difficulties and became


heavily indebted to the Development Bank of the Philippines (DBP). In order
to settle its obligations, ELISCON proposed to convey to DBP by way
of dacion en pago all its fixed assets mortgaged with DBP, as payment for its
total indebtedness in the amount of P201, 181,833.16. On December 28,
1978, ELISCON and DBP executed a Deed of Cession of Property in Payment
of Debt.
In October 1981, DBP formally took over the assets of ELISCON,
including its indebtedness to BPI. Thereafter, DBP proposed formulas for the
settlement of all of ELISCON's obligations to its creditors, but BPI expressly
rejected the formula submitted to it for not being acceptable.
Consequently, BPI, as successor-in-interest of CBTC, a complaint for
sum of money against ELISCON, MULTI and Babst.
ISSUE/S:
Whether or not ELISCON,MULTI and Babst are liable to BPI.
RULING:
NO
It should be DBP who will be accountable to the debts of ELISCON by
reason of a valid novation.
Novation, in its broad concept, may either be extinctive or modificatory
.It is extinctive when an old obligation is terminated by the creation of a new
obligation that takes the place of the former; it is merely modificatory when
the old obligation subsists to the extent it remains compatible with the
amendatory agreement. An extinctive novation results either by changing
the object or principal conditions (objective or real), or by substituting the
person of the debtor or subrogating a third person in the rights of the
creditor (subjective or personal). Under this mode, novation would have dual
functions one to extinguish an existing obligation, the other to substitute a
new one in its place requiring a conflux of four essential requisites, (1) a
previous valid obligation; (2) an agreement of all parties concerned to a new
contract; (3) the extinguishment of the old obligation; and (4) the birth of a
valid new obligation.
BPI contends that in order to have a valid novation, there must be an
express consent of the creditor. But it was explained and clarified by the
court by way of jurisprudence that:
The aforecited article 1205 [now 1293] of the Civil
Code does not state that the creditor's consent to
the substitution of the new debtor for the old be
express, or given at the time of the substitution, and the

Page 755 of 1072

Supreme Court of Spain, in its judgment of June 16, 1908,


construing said article, laid down the doctrine that "article
1205 of the Civil Code does not mean or require that the
creditor's consent to the change of debtors must be given
simultaneously with the debtor's consent to the
substitution, its evident purpose being to preserve the
creditor's full right, it is sufficient that the latter's consent
be given at any time and in any form whatever, while the
agreement of the debtors subsists. The same rule is stated
in theEnciclopedia Juridica Espaola, volume 23, page 503,
which reads: "'The rule that this kind of novation, like all
others, must be express, is not absolute; for the
existence of the consent may well be inferred from
the act of the creditor, since volition may as well be
expressed by deeds as by words."
In the case at bar, Babst, MULTI and ELISCON all maintain that due to
the failure of BPI to register its objection to the take-over by DBP of
ELISCON's assets, at the creditors' meeting held in June 1981 and thereafter,
it is deemed to have consented to the substitution of DBP for ELISCON as
debtor. There exist clear indications that BPI was aware of the assumption by
DBP of the obligations of ELISCON. In fact, BPI admits that ---DBP had
proposed a payment but it was rejected by BPI.
Indeed, the authority granted by BPI to its account officer to attend the
creditors' meeting was an authority to represent the bank, such that when he
failed to object to the substitution of debtors, he did so on behalf of and for
the bank. Even granting arguendo that the said account officer was not so
empowered, BPI could have subsequently registered its objection to the
substitution, especially after it had already learned that DBP had taken over
the assets and assumed the liabilities of ELISCON. Its failure to do so can
only mean acquiescence in the assumption by DBP of ELISCON's obligations.
As repeatedly pointed out by ELISCON and MULTI, BPI's objection was to the
proposed payment formula, not to the substitution itself.
The original obligation having been extinguished, the contracts of
suretyship executed separately by Babst and MULTI, being accessory
obligations, are likewise extinguished. Hence, BPI should enforce its cause of
action against DBP.

Page 756 of 1072

TOPIC: EFFECTS OF MERGER AND CONSOLIDATION


ASSOCIATED BANK, Petitioner,
vs.
COURT OF APPEALS and LORENZO SARMIENTO JR., Respondents.
G.R. No. 123793. June 29, 1998
290 SCRA 639
FACTS:
On or about September 16, 1975 Associated Banking Corporation and
Citizens Bank and Trust Company merged to form just one banking
corporation known as Associated Citizens Bank, the surviving bank. On or
about March 10, 1981, the Associated Citizens Bank changed its corporate
name to Associated Bank by virtue of the Amended Articles of Incorporation.
On September 7, 1977, the defendant executed in favor of Associated Bank a
promissory note whereby the former undertook to pay the latter the sum
of P2, 500,000.00 payable on or before March 6, 1978. As per said
promissory note, the defendant agreed to pay interest at 14% per annum,
3% per annum in the form of liquidated damages, compounded interests,
and attorneys fees, in case of litigation equivalent to 10% of the amount due.
The defendant, to date, still owes plaintiff bank the amount of P2,
250,000.00 exclusive of interest and other charges. Despite repeated
demands the defendant failed to pay the amount due.
The defendant denied all the allegations and averred that the plaintiff
is not the proper party in interest because the promissory note was executed
in favor of Citizens Bank and Trust Company and the several partial
payments were not properly applied.
ISSUE/S:

Page 757 of 1072

Whether or not the plaintiff as a surviving corporation has the


personality to demand payment for contract executed prior to the
merger by the absorbed company?
RULING:
Ordinarily, in the merger of two or more existing corporations, one of
the combining corporations survives and continues the combined business,
while the rest are dissolved and all their rights, properties and liabilities are
acquired by the surviving corporation. Although there is dissolution of the
absorbed corporations, there is no winding up of their affairs or liquidation of
their assets, because the surviving corporation automatically acquires all
their rights, privileges and powers, as well as their liabilities.
The merger, however, does not become effective upon the mere
agreement of the constituent corporations. The procedure to be followed is
prescribed under the Corporation Code. Section 79 of said Code requires the
approval by the Securities and Exchange Commission (SEC) of the articles of
merger which, in turn, must have been duly approved by a majority of the
respective stockholders of the constituent corporations. The same provision
further states that the merger shall be effective only upon the issuance by
the SEC of a certificate of merger. The effectivity date of the merger is crucial
for determining when the merged or absorbed corporation ceases to exist;
and when its rights, privileges, properties as well as liabilities pass on to the
surviving corporation.
The records do not show when the SEC approved the merger. Private
respondents theory is that it took effect on the date of the execution of the
agreement itself, which was September 16, 1975. The merger does not
become effective by the mere agreement of the constituent corporation but
shall be effective only upon the issuance of the SEC of the Certificate of
merger.
However, a closer perusal of the merger agreement leads to a different
conclusion. The provision quoted earlier has this other clause:
Upon the effective date of the merger, all references to CBTC in
any deed, documents, or other papers of whatever kind or nature
and wherever found shall be deemed for all intents and
purposes, references to ABC, the SURVIVING BANK, as if such
references were direct references to [ABC]. x x x
Thus, the fact that the promissory note was executed after the
effectivity date of the merger does not militate against petitioner. The
agreement itself clearly provides that all contracts -- irrespective of the date
of execution -- entered into in the name of CBTC shall be understood as
pertaining to the surviving bank, herein petitioner. Since, in contrast to the
earlier aforequoted provision, the latter clause no longer specifically refers

Page 758 of 1072

only to contracts existing at the time of the merger, no distinction should be


made. The clause must have been deliberately included in the agreement in
order to protect the interests of the combining banks; specifically, to avoid
giving the merger agreement a farcical interpretation aimed at evading
fulfillment of a due obligation.
Thus, although the subject promissory note names CBTC as the payee,
the reference to CBTC in the note shall be construed, under the very
provisions of the merger agreement, as a reference to petitioner bank, as if
such reference was a direct reference to the latter for all intents and
purposes.
In light of the foregoing, the Court holds that petitioner has a valid
cause of action against private respondent. Clearly, the failure of private
respondent to honor his obligation under the promissory note constitutes a
violation of petitioners right to collect the proceeds of the loan it extended
to the former.

TOPIC: EFFECTS OF MERGER AND CONSOLIDATION


ALGER ELECTRIC, INC., petitioner
vs.
COURT OF APPEALS and NORTHERN CEMENT
CORPORATION, Respondents.
G.R. No. L-34298 February 28, 1985
135 SCRA 37
FACTS:
Petitioner Alger Electric, Inc., was granted a legislative franchise for a
period of fifty (50) years from June 22, 1963 with the right, privilege, and
authority to construct, maintain and operate an electric light, heat, and
power system for the generation and/or distribution of electric light, heat,
and/or power for sale within the municipalities of Sto. Tomas, Damortis and
Rosario, province of La Union, and in the municipality of Sison, province of
Pangasinan.
On August 16, 1968, respondent Northern Cement Corporation
(Northern) and the National Power Corporation (NPC) executed a contract for
NPC to directly supply electric power to Northern's cement plant located in
Labayog, Sison, Pangasinan. As a result, the petitioner filed a petition for
prohibition with preliminary injunction against Northern and NPC in the Court
of First Instance of Manila. The petition alleged that the contract was patently
illegal in view of Section 2, Republic Act No. 3826, which provides:

Page 759 of 1072

Section 2. In the event that the National Power Corporation


should have established its lines in the areas adjacent to or
near the territory covered by this franchise, the National
Power Corporation may make available its powers and heat
only after registration with and through the Alger Electric,
Inc., or with the authority and consent of the grantee.
The petitioner prayed that the contract be declared null and void and
while the case is pending, a writ of preliminary injunction be immediately
issued, enjoining the respondents, their managers, attorneys, agents and/or
representatives acting for and in their behalf from enforcing said contract.

ISSUE/S:
Whether or not the contract is null and void.

RULING:
NO, the contract between NPC and Northern is valid.
Section 2 of Rep. Act No. 3826 is patently unconstitutional for
being exclusive in character. It violates the constitutional mandate that no
franchise for the operation of a public utility shall be exclusive in character
(Article XIV, Section 8 of the 1935 Constitution, now Article XIV, Section 5 of
the Constitution as amended).
It is a established principle that the exclusive nature of any public
franchise is not favored. We may interpret in favor of exclusiveness only
when the statute grants it in express, clear, and unmistakable terms. In all
grants by the government to private corporations, the interpretation of
rights, privileges, or franchises is taken against the grantee. Whatever is not
clearly and expressly granted is withheld.
Section 2 of Republic Act No. 3826 was obviously enacted to prevent
the NPC from distributing or selling electric power where petitioner Alger is
already selling or is able to sell its own self-generated electricity. In this case,
Northern is a bulk purchaser of power. It had never purchase's Alger's
electricity before the suit was filed. It is not the usual consumer - residential
or commercial - for whom retail sales are Ideal. Exclusivity is given by law
with the understanding that the company enjoying it is self-sufficient and
capable of supplying the needed service or product at moderate or
reasonable prices. It would be against public interest where the firm granted
a monopoly is merely ail unnecessary conduit of electric power, jacking up

Page 760 of 1072

prices as a superfluous middleman or an inefficient producer which cannot


supply cheap electricity to power intensive industries. It is in the public
interest when industries dependent on the heavy use of electricity are given
reliable and direct power at the lowest costs thus enabling the sale of
nationally marketed products at prices within the reach of the masses.
Applying the above principles to the specific facts of this case, Northern
cannot be said to have committed an act void ab initio when it concluded the
questioned contract with NPC. Accordingly, the respondent corporation is not
liable for damages to the petitioner.

TOPIC: EFFECTS OF MERGER AND CONSOLIDATION


COMMISSIONER OF INTERNAL REVENUE, Petitioner,
vs.
NORTON and HARRISON COMPANY, Respondent.
G.R. No. L-17618
August 31, 1964
11 SCRA 714
FACTS:
Norton and Harrison is a corporation organized in 1911, (1) to buy and
sell at wholesale and retail, all kinds of goods, wares, and merchandise; (2)
to act as agents of manufacturers in the United States and foreign countries;
and (3) to carry on and conduct a general wholesale and retail mercantile
establishment in the Philippines.
Jackbilt is, likewise, a corporation organized primarily for the purpose
of making, producing and manufacturing concrete blocks.
Norton and Jackbilt entered into an agreement whereby Norton was
made the sole and exclusive distributor of concrete blocks manufactured by
Jackbilt in exchange for a commission.
During the existence of the distribution or agency agreement, or on
June 10, 1949, Norton & Harrison acquired by purchase all the outstanding
shares of stock of Jackbilt.

Page 761 of 1072

Commissioner of Internal Revenue, after conducting an investigation,


assessed the respondent Norton & Harrison for deficiency sales tax and
surcharges in the amount of P32,662.90, making as basis thereof the sales of
Norton to the Public. In other words, the Commissioner considered the sale of
Norton to the public as the original sale and not the transaction from Jackbilt.
The Commissioner of Internal Revenue contends that since Jackbilt was
owned and controlled by Norton & Harrison, the corporate personality of the
former (Jackbilt) should be disregarded for sales tax purposes, and the sale
of Jackbilt blocks by petitioner to the public must be considered as
the original sales from which the sales tax should be computed.
ISSUE/S:
Whether or not Norton and Jackbilt were merged.
RULING:
General Rule provides that ownership of all the stocks of a corporation
does not necessarily breed an identity of a corporate interest between the
companies and be considered as sufficient ground for disregarding the
distinct personalities.
However in the case at bar, there are sufficient grounds to establish
that the identities of the two companies must be disregarded. Among these
circumstances, which we find not successfully refuted by appellee Norton
are:
a. Norton and Harrison owned all the outstanding stocks of
Jackbilt; of the 15,000 authorized shares of Jackbilt on
March 31, 1958, 14,993 shares belonged to Norton and
Harrison and one each to seven others;
b. Norton constituted Jackbilt's board of directors in such a
way as to enable it to actually direct and manage the
other's affairs by making the same officers of the board for
both companies. For instance, James E. Norton is the
President, Treasurer, Director and Stockholder of Norton.
He also occupies the same positions in Jackbilt corporation,
the only change being, in the Jackbilt, he is merely a
nominal stockholder. The same is true with Mr. Jordan, F. M.
Domingo, Mr. Mantaring, Gilbert Golden and Gerardo
Garcia, while they are merely employees of the North they
are Directors and nominal stockholders of the Jackbilt
c. Norton financed the operations of the Jackbilt, and this is
shown by the fact that the loans obtained from the RFC
and Bank of America were used in the expansion program
of Jackbilt, to pay advances for the purchase of equipment,
materials rations and salaries of employees of Jackbilt and

Page 762 of 1072

other sundry expenses. There was no limit to the advances


given to Jackbilt so much so that as of May 31, 1956, the
unpaid advances amounted to P757,652.45, which were
not paid in cash by Jackbilt, but was offset by shares of
stock issued to Norton, the absolute and sole owner of
Jackbilt;
d. Norton treats Jackbilt employees as its own. Evidence
shows that Norton paid the salaries of Jackbilt employees
and gave the same privileges as Norton employees, an
indication that Jackbilt employees were also Norton's
employees. Furthermore service rendered in any one of the
two companies were taken into account for purposes of
promotion;
e. Compensation given to board members of Jackbilt, indicate
that Jackbilt is merely a department of Norton.
The income tax return of Norton for 1954 shows that as President and
Treasurer of Norton and Jackbilt, he received from Norton P56,929.95, but
received from Jackbilt the measly amount of P150.00, a circumstance which
points out that remuneration of purported officials of Jackbilt are deemed
included in the salaries they received from Norton. The same is true in the
case of Eduardo Garcia, an employee of Norton but a member of the Board of
Jackbilt. His Income tax return for 1956 reveals that he received from Norton
in salaries and bonuses P4,220.00, but received from Jackbilt, by way of
entertainment, representation, travelling and transportation allowances
P3,000.00. However, in the withholding statement (Exh. 28-A), it was shown
that the total of P4,200.00 and P3,000.00 (P7,220.00) was received by Garcia
from Norton, thus portraying the oneness of the two companies. The Income
Tax Returns of Albert Golden and Dioscoro Ramos both employees of Norton
but board members of Jackbilt, also disclose the game method of payment of
compensation and allowances. The offices of Norton and Jackbilt are located
in the same compound. Payments were effected by Norton of accounts for
Jackbilt and vice versa. Payments were also made to Norton of accounts due
or payable to Jackbilt and vice versa.

Page 763 of 1072

TOPIC: EFFECTS OF MERGER AND CONSOLIDATION


COMMISSIONER OF INTERNAL REVENUE, petitioner,
vs.
VICENTE A. RUFINO and REMEDIOS S. RUFINO, ERNESTO D. RUFINO
and ELVIRA B. RUFINO, RAFAEL R. RUFINO and JULIETA A. RUFINO,
MANUEL S. GALVEZ and ESTER R. GALVEZ, and COURT OF TAX
APPEALS, respondents.
G.R. Nos. L-33665-68 February 27, 1987
FACTS:
The private respondents were the majority stockholders of the defunct
Eastern Theatrical Co., Inc., a corporation organized in 1934, for a period of
twenty-five years terminating on January 25, 1959. It was organized to
engage in the business of operating theaters, opera houses, places of
amusement and other related business enterprises, more particularly the
Lyric and Capitol Theaters in Manila. The President of this corporation
(hereinafter referred to as the Old Corporation) during the year in question
was Ernesto D. Rufino.

Page 764 of 1072

The private respondents are also the majority and controlling


stockholders of another corporation, the Eastern Theatrical Co Inc. This
corporation is engaged in the same kind of business as the Old Corporation.
The General-Manager of this corporation (hereinafter referred to as the New
Corporation) at the time was Vicente A. Rufino.
In a special meeting of stockholders of the Old Corporation on
December 17, 1958, to provide for the continuation of its business after the
end of its corporate life, and upon the recommendation of its board of
directors, a resolution was passed authorizing the Old Corporation to merge
with the New Corporation by transferring its business, assets, goodwill, and
liabilities to the latter, which in exchange would issue and distribute to the
shareholders of the Old Corporation one share for each share held by them in
the said Corporation. Pursuant to the said resolution, the Old Corporation,
represented by Ernesto D. Rufino as President, and the New Corporation,
represented by Vicente A. Rufino as General Manager, signed on January 9,
1959, a Deed of Assignment providing for the conveyance and transfer of all
the business, property, assets and goodwill of the Old Corporation to the New
Corporation in exchange for the latter's shares of stock to be distributed
among the shareholders on the basis of one stock for each stock held in the
Old Corporation except that no new and unissued shares would be issued to
the shareholders of the Old Corporation; the delivery by the New Corporation
to the Old Corporation of 125,005-3/4 shares to be distributed to the
shareholders of the Old Corporation as their corresponding shares of stock in
the New Corporation; the assumption by the New Corporation of all
obligations and liabilities of the Old Corporation under its bargaining
agreement with the Cinema Stage & Radio Entertainment Free Workers
(FFW) which included the retention of all personnel in the latter's employ;
and the increase of the capitalization of the New Corporation in compliance
with their agreement. This agreement was made retroactive to January 1,
1959.
Pursuant to the said resolution, the Old Corporation, represented by
Ernesto D. Rufino as President, and the New Corporation, represented by
Vicente A. Rufino as General Manager, signed on January 9, 1959, a Deed of
Assignment providing for the conveyance and transfer of all the business,
property, assets and goodwill of the Old Corporation to the New Corporation
in exchange for the latter's shares of stock to be distributed among the
shareholders on the basis of one stock for each stock held in the Old
Corporation except that no new and unissued shares would be issued to the
shareholders of the Old Corporation; the delivery by the New Corporation to
the Old Corporation of 125,005-3/4 shares to be distributed to the
shareholders of the Old Corporation as their corresponding shares of stock in
the New Corporation; the assumption by the New Corporation of all
obligations and liabilities of the Old Corporation under its bargaining
agreement with the Cinema Stage & Radio Entertainment Free Workers

Page 765 of 1072

(FFW) which included the retention of all personnel in the latter's employ;
and the increase of the capitalization of the New Corporation in compliance
with their agreement. This agreement was made retroactive to January 1,
1959.
The resolution of the Old Corporation of December 17, 1958, and the
Deed of Assignment of January 9, 1959, were approved in a resolution by the
stockholders of the New Corporation in their special meeting on January 12,
1959. In the same meeting, the increased capitalization of the New
Corporation to P2,000,000.00 was also divided into 200,000 shares at P10.00
par value each share, and the said increase was registered on March 5, 1959,
with the Securities and Exchange Commission, which approved the same on
August 20,1959.
As agreed, and in exchange for the properties, and other assets of the
Old Corporation, the New Corporation issued to the stockholders of the
former stocks in the New Corporation equal to the stocks each one held in
the Old Corporation, as follows:
Mr. & Mrs. Vicente A. Rufino............... 17,083 shares
Mr. & Mrs. Rafael R. Rufino ................. 16,881 shares
Mr. & Mrs. Ernesto D. Rufino .............. 18,347 shares
Mr. & Mrs. Manuel S. Galvez ............... 16,882 shares
The BIR declared base on these transaction that the merger of the
aforesaid corporations was not undertaken for a bona fide business purpose
but merely to avoid liability for the capital gains tax on the exchange of the
old for the new shares of stock.
ISSUE/S:
Whether or not the merger was valid.
RULING:
There was a valid merger although the actual transfer of the properties
subject of the Deed of Assignment was not made on the date of the merger.
In the nature of things, this was not possible. Obviously, it was necessary for
the Old Corporation to surrender its net assets first to the New Corporation
before the latter could issue its own stock to the shareholders of the Old
Corporation because the New Corporation had to increase its capitalization
for this purpose. This required the adoption of the resolution to this effect at
the special stockholders meeting of the New Corporation on January 12,
1959, the registration of such issuance with the SEC on March 5, 1959, and
its approval by that body on August 20, 1959. All these took place after the
date of the merger but they were deemed part and parcel of, and
indispensable to the validity and enforceability of, the Deed of Assignment.

Page 766 of 1072

The Court finds no impediment to the exchange of property for stock


between the two corporations being considered to have been effected on the
date of the merger. That, in fact, was the intention, and the reason why the
Deed of Assignment was made retroactive to January 1, 1959. Such
retroaction provided in effect that all transactions set forth in the merger
agreement shall be deemed to be taking place simultaneously on January 1,
1959, when the Deed of Assignment became operative.
The certificates of stock subsequently delivered by the New
Corporation to the private respondents were only evidence of the ownership
of such stocks. Although these certificates could be issued to them only after
the approval by the SEC of the increase in capitalization of the New
Corporation, the title thereto, legally speaking, was transferred to them on
the date the merger took effect, in accordance with the Deed of Assignment.
The basic consideration, of course, is the purpose of the merger, as
this would determine whether the exchange of properties involved therein
shall be subject or not to the capital gains tax. The criterion laid down by the
law is that the merger" must be undertaken for a bona fide business purpose
and not solely for the purpose of escaping the burden of taxation." We must
therefore seek and ascertain the intention of the parties in the light of their
conduct contemporaneously with, and especially after, the questioned
merger pursuant to the Deed of Assignment of January 9, 1959.
It has been suggested that one certain indication of a scheme to evade
the capital gains tax is the subsequent dissolution of the new corporation
after the transfer to it of the properties of the old corporation and the
liquidation of the former soon thereafter. This highly suspect development is
likely to be a mere subterfuge aimed at circumventing the requirements of
Section 35 of the Tax Code while seeming to be a valid corporate
combination.
We see no such furtive intention in the instant case. It is clear, in fact,
that the purpose of the merger was to continue the business of the Old
Corporation, whose corporate life was about to expire, through the New
Corporation to which all the assets and obligations of the former had been
transferred. What argues strongly, indeed, for the New Corporation is that it
was not dissolved after the merger agreement in 1959. On the contrary, it
continued to operate the places of amusement originally owned by the Old
Corporation and transfered to the New Corporation, particularly the Capitol
and Lyric Theaters, in accordance with the Deed of Assignment. The New
Corporation, in fact, continues to do so today after taking over the business
of the Old Corporation twenty-seven years ago.

Page 767 of 1072

TOPIC: EFFECTS OF MERGER AND CONSOLIDATION


SOLID MANILA CORPORATION, petitioner,
vs.
BIO HONG TRADING CO., INC. and COURT OF APPEALS, respondents.
G.R. No. 90596 April 8, 1991
FACTS:
Petitioner is the owner of a parcel of land located in Ermita, Manila,
covered by Transfer Certificate of Title No. 157750 of the Register of Deeds of
Manila. The same lies in the vicinity of another parcel, registered in the name

Page 768 of 1072

of the private respondent corporation under Transfer Certificate of Title No.


128784.
The private respondent's title came from a prior owner, and in their
deed of sale, the parties thereto reserved as an easement of way: a portion
thereof measuring NINE HUNDRED FOURTEEN SQUARE METERS, more or
less, had been converted into a private alley for the benefit of neighboring
estates, and that the three meter wide portion of said parcel along the Pasig
River, with an area of ONE HUNDRED SEVENTY NINE (179) SQUARE METERS,
more or less, had actually been expropriated by the City Government, and
developed pursuant to the beautification drive of the Metro Manila Governor.
The petitioner claims that ever since, it had (as well as other residents
of neighboring estates) made use of the above private alley and maintained
and contributed to its upkeep, until sometime in 1983, when, and over its
protests, the private respondent constructed steel gates that precluded
unhampered use.
The petitioner commenced suit for injunction against the private
respondent, to have the gates removed and to allow full access to the
easement.
ISSUE/S:
Whether or not the easement was extinguished by way of merger.
RULING:
NO, since there was no even merger that took place.
No genuine merger took place as a consequence of the sale in favor of
the private respondent corporation. According to the Civil Code, a merger
exists when ownership of the dominant and servient estates is consolidated
in the same person. Merger then, as can be seen, requires full ownership of
both estates.
The servitude in question is a personal servitude, that is to say, one
constituted not in favor of a particular tenement (a real servitude) but rather,
for the benefit of the general public. Art. 614. Servitudes may also be
established for the benefit of a community, or of one or more persons to
whom the encumbered estate does not belong.
In a personal servitude, there is therefore no "owner of a dominant
tenement" to speak of, and the easement pertains to persons without a
dominant estate, in this case, the public at large.
Merger, as we said, presupposes the existence of a prior servientdominant owner relationship, and the termination of that relation leaves the
easement of no use. Unless the owner conveys the property in favor of the

Page 769 of 1072

public if that is possible no genuine merger can take place that would
terminate a personal easement.
Private respondent did acquire ownership over the property including the
disputed alley as a result of the conveyance, it did not acquire the right to
close that alley or otherwise put up obstructions thereon and thus prevent
the public from using it, because as a servitude, the alley is supposed to be
open to the public because the law specifically provides that:
Art. 617. Easements are inseparable from the estate to which
they actively or
passively belong.
Servitudes are merely accessories to the tenements of which they form
part. Although they are possessed of a separate juridical existence, as mere
accessories, they cannot, however, be alienated from the tenement or
mortgaged separately.
The fact, however, that the alley in question, as an easement, is
inseparable from the main lot is no argument to defeat the petitioner's
claims, because as an easement precisely, it operates as a limitation on the
title of the owner of the servient estate, specifically, his right to use (jus
utendi).
The deed itself stipulated that "a portion thereof of the tenement
measuring NINE HUNDRED FOURTEEN SQUARE METERS, more or less, had
been converted into a private alley for the benefit of the neighboring estates.
. ." and precisely, the former owner, in conveying the property, gave the
private owner a discount on account of the easement.

TOPIC: PURPOSES OF NON- STOCK CORPORATIONS


CHINESE YOUNG MENS CHRISTIAN ASSOCIATION OF THE PHILIPPINE
ISLANDS, ET AL.
VS.
VICTOR CHING AND COURT OF APPEALS
G.R. NO. L-36929, JUNE 18, 1976
71 SCRA 460
FACTS:

Page 770 of 1072

Respondent Victor Ching filed an action for mandamus with preliminary


injunction against the herein petitioners. He anchored his action upon the
claim that the Membership Campaign of the Chinese YMCA for 1966 held,
only 175 applications for membership were submitted, canvassed and
accepted on the last day of the membership campaign. Not more than 240
membership applications, as reported, issue of the Chinese Commercial
News, were filed.
It is to be noted that respondent Victor Ching is a member of the Board
of Directors of the Chinese YMCA, while herein petitioners, William Golangco
and Juanito K. Tan, are its president and recording secretary, respectively. In
the campaign for membership for the year 1966, a rivalry had developed
between two groups in the association, one headed by respondent Ching and
the other by petitioner Golangco.
On the last day of the membership campaign, respondent Ching and
herein petitioner Golangco were in the office of the Chinese YMCA.
Respondent Ching, after it was agreed upon that there was going to be no
extension of the membership campaign. After trial, a decision was rendered
annulling the 1966 annual membership campaign of the respondent. On
appeal, the appealed decision was affirmed.

ISSUE/S:
Whether or not membership campaign is valid.

RULING:
The applications and the receipts for payment of the membership fees
show that they were filed and paid not later than the November 26, 1965
deadline, and this was further supported by the bank statement of the
petitioner YMCA deposit account with the China Banking Corporation and the
checks paid by certain members to the YMCA which show that the
application fees corresponding to the questioned 74 applications were
already paid to petitioner YMCA as the time of the said deadline. No evidence
could be cited to rebut this well nigh conclusive documentary evidence. If
there were indeed any applications filed after the deadline, they certainly
should have been positively pin-pointed and specifically annulled.
What is worse, 175 membership applications were undisputedly filed
within the deadline (including the 75 withdrawn by respondent) and yet the
100 remaining unquestioned memberships were nullified by the questioned
decision without the individuals concerned ever having been impleaded or
heard (except the individual petitioners president and secretary).

Page 771 of 1072

The appealed decision thus contravened the established principle that


the courts cannot strip a member of a non-stock non-profit corporation of his
membership therein without cause. Otherwise, that would be an
unwarranted and undue interference with the well established right of a
corporation to determine its membership. In order that membership may be
acquired in a non-stock corporation and valid by-laws must be complied with,
except in so far as they may be and are waived. But provisions in the by-laws
as to formal steps to be taken to acquire membership may be waived by the
corporation, or it may be estopped to assert that they have not been taken.
Finally, the appealed decision did not give due importance to the
undisputed fact therein stated that "at the board meeting of the association
held on December 7, 1965, a list of 174 applications for membership, old and
new, was submitted to the board and approved by the latter, over the
objection of the petitioner who was present at said meeting." Such action of
the petitioner association's board of directors approving the 174 membership
applications of old and new members constituting its active membership as
duly processed and screened by the authorized committee just be deemed a
waiver on its part of any technicality or requirement of form, since otherwise
the association would be practically paralyzed and deprived of the
substantial revenues from the membership dues of P17, 400.00.

TOPIC: PURPOSES OF NON- STOCK CORPORATIONS


THE COLLECTOR OF INTERNAL REVENUE
vs.
THE CLUB FILIPINO, INC. DE CEBU
G.R. No. L-12719 May 31, 1962
FACTS:

Page 772 of 1072

The "Club Filipino, Inc. de Cebu," is a civic corporation organized under


the laws of the Philippines with an original authorized capital stock of
P22,000.00, which was subsequently increased to P200,000.00. Neither in
the articles or by-laws is there a provision relative to dividends and their
distribution, although it is covenanted that upon its dissolution, the Club's
remaining assets, after paying debts, shall be donated to a charitable
Philippine Institution in Cebu.
The Club owns and operates a club house, a bowling alley, a golf
course (on a lot leased from the government), and a bar-restaurant where it
sells wines and liquors, soft drinks, meals and short orders to its members
and their guests. The bar-restaurant was a necessary incident to the
operation of the club and its golf-course. The club is operated mainly with
funds derived from membership fees and dues. Whatever profits it had, were
used to defray its overhead expenses and to improve its golf-course. In 1951.
as a result of a capital surplus, arising from the re-valuation of its real
properties, the value or price of which increased, the Club declared stock
dividends; but no actual cash dividends were distributed to the stockholders.
In 1952, a BIR agent discovered that the Club has never paid percentage tax
on the gross receipts of its bar and restaurant, although it secured B-4, B9(a) and B-7 licenses. In a letter dated December 22, 1852, the Collector of
Internal Revenue assessed against and demanded from the Club certain
amount of tax. The Club wrote the Collector, requesting for the cancellation
of the assessment. The request having been denied, the Club filed the
instant petition for review.
ISSUE/S:
Whether the respondent Club is liable for the payment of the sum of
12,068.84, as fixed and percentage taxes and surcharges prescribed in
sections 182, 183 and 191 of the Tax Code, under which the
assessment was made, in connection with the operation of its bar and
restaurant
RULING:
Section 182, of the Tax Code states, "Unless otherwise provided, every
person engaging in a business on which the percentage tax is imposed shall
pay in full a fixed annual tax of ten pesos for each calendar year or fraction
thereof in which such person shall engage in said business." Section 183
provides in general that "the percentage taxes on business shall be payable
at the end of each calendar quarter in the amount lawfully due on the
business transacted during each quarter; etc." And section 191, same Tax
Code, provides "Percentage tax . . . Keepers of restaurants, refreshment
parlors and other eating places shall pay a tax three per centum, and
keepers of bar and cafes where wines or liquors are served five per centum
of their gross receipts . . .". It has been held that the liability for fixed and

Page 773 of 1072

percentage taxes, as provided by these sections, does not ipso facto attach
by mere reason of the operation of a bar and restaurant. For the liability to
attach, the operator thereof must be engaged in the business as a barkeeper
and restaurateur. The plain and ordinary meaning of business is restricted to
activities or affairs where profit is the purpose or livelihood is the motive, and
the term business when used without qualification, should be construed in its
plain and ordinary meaning, restricted to activities for profit or livelihood.
Having found as a fact that the Club was organized to develop and
cultivate sports of all class and denomination, for the healthful recreation
and entertainment of its stockholders and members; that upon its
dissolution, its remaining assets, after paying debts, shall be donated to a
charitable Philippine Institution in Cebu; that it is operated mainly with funds
derived from membership fees and dues; that the Club's bar and restaurant
catered only to its members and their guests; that there was in fact no cash
dividend distribution to its stockholders and that whatever was derived on
retail from its bar and restaurant was used to defray its overall overhead
expenses and to improve its golf-course (cost-plus-expenses-basis), it stands
to reason that the Club is not engaged in the business of an operator of bar
and restaurant.
The facts that the capital stock of the respondent Club is divided into
shares, does not detract from the finding of the trial court that it is not
engaged in the business of operator of bar and restaurant. What is
determinative of whether or not the Club is engaged in such business is its
object or purpose, as stated in its articles and by-laws. It is a familiar rule
that the actual purpose is not controlled by the corporate form or by the
commercial aspect of the business prosecuted, but may be shown by
extrinsic evidence, including the by-laws and the method of operation.

TOPIC: NON STOCK CORPORATIONS: VOTING


ANTONIO LITONJUA and ARNOLD LITONJUA

Page 774 of 1072

vs.
THE HON. COURT OF APPEALS, ET. AL.
G.R. No. 120294 February 10, 1998
FACTS:
Respondent Wack Wack Golf and Country Club is a non-profit
corporation which offers sports, recreational and social activities to its
members. Petitioner Antonio Litonjua is an Associate Member of said
corporation and his son, co-petitioner Arnold Litonjua, is a Junior Member
thereof. The individual respondents are the members of the Board of
Directors and Membership Committee of Wack Wack. On 10 January 1985,
pursuant to its by-laws, respondent club posted the monthly list of
delinquent members on its premises. Included therein was petitioner Antonio
Litonjua.
On 13 January 1985, after Antonio Litonjua discovered that his name
was on the January 1985 delinquent list, he proceeded to the Cashier's Office
of the club and was informed therein that the reason behind his delinquency
was his failure to pay his November 1984 dues (which should have been paid
before the end of December 1984 as provided in the corporate by-laws).
Antonio Litonjua alleged that he was not able to pay his monthly bill on time
because he has not received his statement of account for November 1984.
As proof, he presented a sealed envelope which he allegedly presumed to be
the November 1984 bill (but was actually the December 1984 statement of
account) and explained that he received it only on 12 January 1985.
A check with the accounting office, however, revealed that the
November 1984 statement of account had already been delivered to Antonio
Litonjua's office and was received by his employee allegedly named
"Aquino." Petitioner asserted that, he did not receive said account and had
no employee by the name of "Aquino." Based on the foregoing, Antonio
Litonjua was able to convince the auxiliary clerks in the Cashier's Office to
delete his name from the list of delinquent members. Consequently, Antonio
Litonjua continued to avail of the club facilities. Later, Antonio Litonjua was
advised of another outstanding balance in the amount of P9,414.00. Again,
he issued a check in payment thereof. As a result, his name was deleted from
the February 1985 list of delinquent members.
ISSUE/S:
Whether or not the statement of account for November 1984 was duly
delivered to and received by Antonio Litonjua's office on 12 December
1984.
RULING:
The Court held that SEC committed an error in apprehending the facts.
We have judiciously studied Mr. Limbo's testimony on record and we failed to

Page 775 of 1072

find therein any statement that he delivered the November 1984 account to
Antonio Litonjua himself. Mr. Limbo was consistent in his testimony to the
effect that on 12 December 1984 he delivered the November 1984
statement of account at the office of Antonio Litonjua and it was received by
an employee of the latter who signed the Special Delivery Receipt. On crossexamination, Mr. Limbo did not waver from his testimony that Antonio
Litonjua's November 1984 bill was duly received by the latter's
employee.Against the testimony of Mr. Victor Limbo, coupled with
documentary evidence in the form of the signed Special Delivery Receipt,
petitioners presented no proof other than the bare denial of Antonio Litonjua
that he never received his statement of account for November 1984 and that
he has no "Aquino" in his employ. Petitioners could have readily offered in
evidence a record or list of Antonio Litonjua's employees to prove that he has
no employee by the name of "Aquino" but, strangely, beyond his mere say-so
no such evidence was adduced.

Page 776 of 1072

TOPIC: NON STOCK CORPORATIONS: VOTING


THE PHILIPPINE PUBLIC SCHOOL TEACHERS ASSOCIATION (PPSTA)
COMMISSION ON ELECTIONS
vs.
Honorable SERGIO A. F. APOSTOL
G.R. No. L-36966 February 28, 1974
FACTS:
On July 20, 1972, private respondent Eufemia M. San Luis as a member
of the Philippine Public School Teachers Association (PPSTA for short), a
fraternal non-stock association of public school teachers throughout the
country, filed with respondent court of first instance at Quezon City a
complaint with preliminary injunction for the annulment of the 1972 annual
elections of the PPSTA board of directors held on June 26-28, 1972 at
Teachers Camp in Baguio City for having been held outside its principal office
at Quezon City against herein petitioners as defendants.
ISSUE/S:
Whether or not the elections of the Board of Directors are null and
void.
RULING:
The Court finds it unnecessary to rule upon the parties' above
conflicting contentions, since it finds to be decisive petitioners' contention
that respondent has no personality and standing as a single individual
member out of thousands of members of the PPSTA to bring the action below
for annulment of the PPSTA 1972 annual convention and elections, as she
was not even a chapter delegate to the said convention and she was duly
represented thereat in accordance with the PPSTA's by-laws by her duly
authorized chapter delegates who have raised no question as to the
proceedings.Article IX, section 5 of the by-laws expressly provides that "only
official delegates to the representative assembly are entitled to take part in
the discussions and to vote."
Respondent's action below was in essence one of quo warranto which
is governed by Rule 66 of the Rules of Court Section 6 thereof provides that
in order that an individual may directly bring the action, he or she must claim
to entitled to the public office or position allegedly unlawfully held or
usurped.Otherwise, the action must be brought by the Solicitor General or
fiscal with leave of the court upon the complaint of the realtor under section
4 of the Rule.
The general rule is that actions for quo warranto should be brought by
the Solicitor General or a fiscal in cases of usurpation of an office established
by law or by the Constitution under color of an executive appointment, or the

Page 777 of 1072

abuse of a public franchise under color of a legislative grant, for these are
public wrongs and not private injuries. Since, under our system all power
emanates from the people, who constitute the sovereignty, the right to
inquire into the authority by which a person assumes to exercise the
functions of a public office or franchise is regarded as inherent in the people
on the right their sovereignty. Hence, the action should be brought by the
Solicitor General or the fiscal who represents the sovereign power.
Respondent manifestly lays no claim herself to the office of PPSTA
director nor has the present action been filed with leave of court by the
Solicitor General or fiscal upon her relation as a party having an interest
injuriously affected, as required by the cited Rule. Her action must therefore
fail on this score and the judgment erroneously rendered by respondent
court shall be set aside.

Page 778 of 1072

TOPIC: CLOSE CORPORATIONS: REQUIREMENTS FOR FORMATION


MANUEL R. DULAY ENTERPRISES, INC.,
vs.
THE HONORABLE COURT OF APPEALS
G.R. No. 91889
August 27, 1993
FACTS:
Petitioner Manuel R. Dulay Enterprises, Inc, a domestic corporation
with the following as members of its Board of Directors: Manuel R. Dulay with
19,960 shares and designated as president, treasurer and general manager,
Atty. Virgilio E. Dulay with 10 shares and designated as vice-president; Linda
E. Dulay with 10 shares; Celia Dulay-Mendoza with 10 shares; and Atty.
Plaridel C. Jose with 10 shares and designated as secretary, owned a
property covered by TCT No. 17880 and known as Dulay Apartment
consisting of sixteen (16) apartment units on a six hundred eighty-nine (689)
square meters lot, more or less, located at Seventh Street (now Buendia
Extension) and F.B. Harrison Street, Pasay City.
Petitioner Corporation through its president, Manuel Dulay, obtained
various loans for the construction of its hotel project, Dulay Continental Hotel
(now Frederick Hotel). It even had to borrow money from petitioner Virgilio
Dulay to be able to continue the hotel project. As a result of said loan,
petitioner Virgilio Dulay occupied one of the unit apartments of the subject
property since property since 1973 while at the same time managing the
Dulay Apartment at his shareholdings in the corporation was subsequently
increased by his father.
On December 23, 1976, Manuel Dulay by virtue of Board Resolution
petitioner corporation sold the subject property to private respondents
spouses Maria Theresa and Castrense Veloso in the amount of P300,000.00
as evidenced by the Deed of Absolute Sale. Subsequently, private
respondent Maria Veloso, without the knowledge of Manuel Dulay,
mortgaged the subject property to private respondent Manuel A. Torres for a
loan of P250,000.00 which was duly annotated. Upon the failure of private
respondent Maria Veloso to pay private respondent Torres, the subject
property was sold on April 5, 1978 to private respondent Torres as the
highest bidder in an extrajudicial foreclosure sale as evidenced by the
Certificate of Sheriff's Sale issued on April 20, 1978.
ISSUE/S:
Whether or not the doctrine of piercing the veil of corporate entity is
applicable?

Page 779 of 1072

RULING:
Petitioner corporation is classified as a close corporation and
consequently a board resolution authorizing the sale or mortgage of the
subject property is not necessary to bind the corporation for the action of its
president. At any rate, corporate action taken at a board meeting without
proper call or notice in a close corporation is deemed ratified by the absent
director unless the latter promptly files his written objection with the
secretary of the corporation after having knowledge of the meeting which, in
his case, petitioner Virgilio Dulay failed to do.
It is relevant to note that although a corporation is an entity which has
a personality distinct and separate from its individual stockholders or
members, the veil of corporate fiction may be pierced when it is used to
defeat public convenience justify wrong, protect fraud or defend crime. The
privilege of being treated as an entity distinct and separate from its
stockholder or members is therefore confined to its legitimate uses and is
subject to certain limitations to prevent the commission of fraud or other
illegal or unfair act. When the corporation is used merely as an alter ego or
business conduit of a person, the law will regard the corporation as the act of
that person. The Supreme Court had repeatedly disregarded the separate
personality of the corporation where the corporate entity was used to annul a
valid contract executed by one of its members.
Petitioners' claim that the sale of the subject property by its president,
Manuel Dulay, to private respondents spouses Veloso is null and void as the
alleged Board Resolution No. 18 was passed without the knowledge and
consent of the other members of the board of directors cannot be sustained
the fact that petitioner Virgilio Dulay on June 24, 1975 executed an affidavit
that he was a signatory witness to the execution of the post-dated Deed of
Absolute Sale of the subject property in favor of private respondent Torres
indicates that he was aware of the transaction executed between his father
and private respondents and had, therefore, adequate knowledge about the
sale of the subject property to private respondents. Consequently, petitioner
corporation is liable for the act of Manuel Dulay and the sale of the subject
property to private respondents by Manuel Dulay is valid and binding.

Page 780 of 1072

TOPIC: CLOSE CORPORATIONS: REQUIREMENTS FOR FORMATION

SAN JUAN STRUCTURAL AND STEEL FABRICATORS, INC.,


vs.
COURT OF APPEALS, et.al.
G.R. No. 129459
September 29, 1998
FACTS:
Plaintiff-appellant San Juan Structural and Steel Fabricators, Inc.'s
amended complaint alleged that on 14 February 1989, plaintiff-appellant
entered into an agreement with defendant-appellee Motorich Sales
Corporation for the transfer to it of a parcel of land. On March 1, 1989. Mr.
Andres T. Co, president of plaintiff-appellant corporation, wrote a letter to
defendant-appellee Motorich Sales Corporation requesting for a computation
of the balance to be paid: that said letter was coursed through defendantappellee's broker. Linda Aduca, who wrote the computation of the balance:
that on March 2, 1989, plaintiff-appellant was ready with the amount
corresponding to the balance, covered by Metrobank Cashier's Check No.
004223, payable to defendant-appellee Motorich Sales Corporation; that
plaintiff-appellant and defendant-appellee Motorich Sales Corporation were
supposed to meet in the office of plaintiff-appellant but defendant-appellee's
treasurer, Nenita Lee Gruenberg, did not appear; that defendant-appellee
Motorich Sales Corporation despite repeated demands and in utter disregard
of its commitments had refused to execute the Transfer of Rights/Deed of
Assignment which is necessary to transfer the certificate of title.

ISSUE/S:
May the doctrine of piercing the veil of corporate fiction be applied to
Motorich?

RULING:
Piercing the Corporate Veil is Not Justified.

Page 781 of 1072

Petitioner also argues that the veil of corporate fiction of Motorich


should be pierced, because the latter is a close corporation. Since "Spouses
Reynaldo L. Gruenberg and Nenita R. Gruenberg owned all or almost all or
99.866% to be accurate, of the subscribed capital stock" of Motorich,
petitioner argues that Gruenberg needed no authorization from the board to
enter into the subject contract. It adds that, being solely owned by the
Spouses Gruenberg, the company can treated as a close corporation which
can be bound by the acts of its principal stockholder who needs no specific
authority.
The Court is not persuaded. First, petitioner itself concedes having
raised the issue belatedly, not having done so during the trial, but only when
it filed its sur-rejoinder before the Court of Appeals. Thus, this Court cannot
entertain said issue at this late stage of the proceedings. It is well-settled the
points of law, theories and arguments not brought to the attention of the trial
court need not be, and ordinarily will not be, considered by a reviewing court,
as they cannot be raised for the first time on appeal. Allowing petitioner to
change horses in midstream, as it were, is to run roughshod over the basic
principles of fair play, justice and due process.
Second, even if the above mentioned argument were to be addressed
at this time, the Court still finds no reason to uphold it. True, one of the
advantages of a corporate form of business organization is the limitation of
an investor's liability to the amount of the investment. This feature flows
from the legal theory that a corporate entity is separate and distinct from its
stockholders. However, the statutorily granted privilege of a corporate veil
may be used only for legitimate purposes. On equitable considerations, the
veil can be disregarded when it is utilized as a shield to commit fraud,
illegality or inequity; defeat public convenience; confuse legitimate issues; or
serve as a mere alter ego or business conduit of a person or an
instrumentality, agency or adjunct of another corporation.

Page 782 of 1072

TOPIC: CLOSE CORPORATIONS: REQUIREMENTS FOR FORMATION


SERGIO F. NAGUIAT
vs.
NATIONAL LABOR RELATIONS COMMISSION
G.R. No. 116123
March 13, 1997
FACTS:
Petitioner CFTI held a concessionaire's contract with the Army Air Force
Exchange Services ("AAFES") for the operation of taxi services within Clark
Air Base. Sergio F. Naguiat was CFTI's president, while Antolin T. Naguiat was
its vice-president. Like Sergio F. Naguiat Enterprises, Incorporated a trading
firm, it was a family-owned corporation.
Individual respondents were previously employed by CFTI as taxicab
drivers. During their employment, they were required to pay a daily
"boundary fee" in the amount of US$26.50 for those working from 1:00 a.m.
to 12:00 noon, and US$27.00 for those working from 12:00 noon to 12:00
midnight. All incidental expenses for the maintenance of the vehicles they
were driving were accounted against them, including gasoline expenses. The
drivers worked at least three to four times a week, depending on the
availability of taxicabs. They earned not less than US$15.00 daily. Due to the
phase-out of the US military bases in the Philippines, from which Clark Air
Base was not spared, the AAFES was dissolved, and the services of individual
respondents were officially terminated on November 26, 1991.
ISSUE/S:
Whether or not Sergio F. Naguiat Enterprises, Inc. is a separate and
distinct juridical entity which cannot be held jointly and severally liable
for the obligations of CFTI?
RULING:
From the evidence proffered by both parties, there is no substantial
basis to hold that Naguiat Enterprises is an indirect employer of individual
respondents much less a labor only contractor. On the contrary, petitioners
submitted documents such as the drivers' applications for employment with
CFTI, and social security remittances and payroll of Naguiat Enterprises
showing that none of the individual respondents were its employees.

Page 783 of 1072

Moreover, in the contract between CFTI and AAFES, the former, as


concessionaire, agreed to purchase from AAFES for a certain amount within a
specified period a fleet of vehicles to be "kept on the road" by CFTI, pursuant
to their concessionaire's contract. This indicates that CFTI became the owner
of the taxicabs which became the principal investment and asset of the
company.
Private respondents failed to substantiate their claim that Naguiat
Enterprises managed, supervised and controlled their employment. It
appears that they were confused on the personalities of Sergio F. Naguiat as
an individual who was the president of CFTI, and Sergio F. Naguiat
Enterprises, Inc., as a separate corporate entity with a separate business.
They presumed that Sergio F. Naguiat, who was at the same time a
stockholder and director of Sergio F. Naguiat Enterprises, Inc., was managing
and controlling the taxi business on behalf of the latter. A closer scrutiny and
analysis of the records, however, evince the truth of the matter: that Sergio
F. Naguiat, in supervising the taxi drivers and determining their employment
terms, was rather carrying out his responsibilities as president of CFTI.
Hence, Naguiat Enterprises as a separate corporation does not appear to be
involved at all in the taxi business. From the foregoing, the ineludible
conclusion is that CFTI was the actual and direct employer of individual
respondents, and that Naguiat Enterprises was neither their indirect
employer nor labor-only contractor. It was not involved at all in the taxi
business.

Page 784 of 1072

TOPIC:SPECIAL CORPORATIONS: RELIGIOUS CORPORATIONS


REPUBLIC OF THE PHILIPPINES
vs.
THE HONORABLE INTERMEDIATE APPELLATE COURT (now Court of
Appeals), and PRINCESS EMME ATIK KIRAM
G.R. No. L-68303. January 15, 1988
FACTS:
The properties in dispute number three undivided lots altogether
consisting of a total of 1,024 hectares of rice lands. The title thereto stood
allegedly in the name of Sultan Jamalul Kiram, who died in 1936. The private
respondent, a niece of the late Sultan, now claims that the original certificate
of title thereto was destroyed as a consequence of a fire that gutted the
office of the Register of Deeds of Sulu sometime in February, 1974. She
likewise alleges that the owner's copy thereof was lost on account of the
same misfortune.
On October 18,1979, she went to the then Court of First Instance of
Sulu, Branch I, at Jolo, now Regional Trial Court, the Honorable Jainal D.
Rasul, District Judge, presiding, for reconstitution.
ISSUE/S:
Whether or not the CA erred in granting the reconstitution
RULING:
The Supreme Court rules for the Republic.
It is not disputed, to begin with, that the notices (of hearing) were not
posted on the main entrances of the provincial and municipal halls of the
locality in which the lands are located. We have held that such a mode of
publication is a jurisdictional requirement. The failure on the part of the
applicant to comply with it confers no jurisdiction upon the court.
Neither is there any showing that the adjacent owners or other interested
parties were actually notified of the pending application. This too taints the
petition with a jurisdictional defect. It is not enough that there is publication
in the Official Gazette. Publication of the notice in the Official Gazette is but

Page 785 of 1072

one requirement. In addition, Republic Act No. 26 decrees that such a notice
be posted "on the main entrance" of the corresponding provincial capitol and
municipal building, as well as served actually upon the owners of adjacent
lands. Failure to comply with such requisites will nullify the decree of
reconstitution.
It shall be noted that a judicial reconstitution of title partakes of a land
registration proceeding. Thus, notice of the proceedings must be done in the
manner set forth by the letter of the law.
The Republic cannot be faulted for nursing doubts about the private
respondent's assertions. In the first place, the private respondent claims that
two deeds have been lost, the original and the duplicate certificates of title.
She furthermore relies on quite doubtful sources as bases for the
reconstitution sought, i.e., certain statutes making references to the
properties. In such a case, the courts are admonished to take utmost caution
that the petition and the evidence presented to support it can stand judicial
scrutiny.
It is not sufficient, as in the case at bar, that the Solicitor General failed
to interpose an opposition to the application. The court must nonetheless
convince itself that the petitioner's evidence is substantial enough to warrant
reconstitution.
This Court agrees with the Republic that the private respondent, based
on the evidence, has not sufficiently shown her right to a reconstitution.

Page 786 of 1072

TOPIC:SPECIAL CORPORATIONS: RELIGIOUS CORPORATIONS


THE DIRECTOR OF LANDS vs .THE HONORABLE COURT OF APPEALS
and IGLESIA NI CRISTO
G.R. No.L-56613. March 14, 1988
FACTS:
On November 28, 1973, private respondent Iglesiani Cristo filed an
application with the then Court of First Instance of Cavite for registration in
its name of a parcel of land with an area of 379 square meters located at
Poblacion, Municipality of Amadeo, Cavite. In said application, private
respondent alleged inter alia that it was the owner in fee simple of the land
afore-described, having acquired title thereto by virtue of a Deed of Absolute
Sale executed in 1947 by Aquelina de la Cruz in its favor and that applicant
and its predecessors-in-interest had been in actual, continuous, public,
peaceful and adverse possession and occupation of said land in the concept
of owner for more than thirty [30] years. Private respondent prayed that
should the Land Registration Act not be applicable, the provisions of Chapter
VIII of Commonwealth Act No. 141, as amended by Republic Act No. 6236 be
applied as applicant and its predecessors-in-interest had been in possession
of the land for more than thirty [30] years and had introduced improvements
thereon, including the fencing thereof on all sides.
The Republic of the Philippines, represented by the Director of Lands,
opposed the application.

ISSUE/S:
Whether or not private respondent was disqualified from holding,
except by lease, alienable lands of the public domain under Section 11,
Article XIV of the 1973 Constitution.
RULING:
No reversible error was committed by the appellate court in ruling that
the true certified copy of the white paper plan was sufficient for the purpose
of identifying the land in question. It bore the approval of the Land

Page 787 of 1072

Registration Commission, and was reverified and approved by the Bureau of


Lands on April 25, 1974 pursuant to the provisions of P.D. No. 239
withdrawing from the Land Registration Commission the authority to approve
original survey plans. It contained the following material data: the barrio
[poblacion], municipality [Amadeo] and province [Cavite] where the subject
land is located, its area of 379 square meters, the land as plotted, its
technical descriptions and its natural boundaries. It was further supported
by the Technical Descriptions signed by a geodetic surveyor and attested by
the Land Registration Commission.
Identification of the land sought to be registered, thereby serving the
purpose for which the original tracing cloth plan is required. The fact
therefore that the original survey plan was recorded on white paper instead
of a tracing cloth should not detract from the probative value thereof.
In the case at bar, such Identity can be well-established by the white
paper plan. To us, it would not matter if the plan introduced to establish the
Identity of the land is made of cloth or is made of paper. For one thing, a
tracing cloth of the plan is required to be submitted to the Bureau of Lands. It
must have a file copy of the same.
Taking the year 1936 as the reckoning point, there being no showing as
to when the Ramoses first took possession and occupation of the land in
question, the 30-year period of open, continuous, exclusive and notorious
possession and occupation required by law was completed in 1966. The
completion by private respondent of this statutory 30-year period has dual
significance in the light of Section 48[b] of Commonwealth Act No. 141, as
amended and prevailing jurisprudence:
1) at this point, the land in question ceased by operation of
law to be part of the public domain; and
2) private respondent could have its title thereto confirmed
through the appropriate proceedings as under the
Constitution then in force, private corporations or
associations were not prohibited from acquiring public
lands, but merely prohibited from acquiring, holding or
leasing such type of land in excess of 1,024 hectares.

Page 788 of 1072

TOPIC:SPECIAL CORPORATIONS: RELIGIOUS CORPORATIONS


IGLESIA EVANGELICA METODISTA EN LAS ISLAS FILIPINAS (IEMELIF)
(Corporation Sole), INC., REV. NESTOR PINEDA, REV. ROBERTO
BACANI, BENJAMIN BORLONGAN, JR., DANILO SAUR, RICHARD PONTI,
ALFREDO MATABANG and all the other members of the IEMELIF
TONDO CONGREGATION of the IEMELIF CORPORATION SOLE,
Petitioners,
vs.
BISHOP NATHANAEL LAZARO, REVERENDS HONORIO RIVERA, DANIEL
MADUCDOC, FERDINAND MERCADO, ARCADIO CABILDO, DOMINGO
GONZALES, ARTURO LAPUZ, ADORABLE MANGALINDAN, DANIEL
VICTORIA and DAKILA CRUZ, and LAY LEADER LINGKOD MADUCDOC
and CESAR DOMINGO
G.R. No. 184088.July 6, 2010
FACTS:
In 1909, Bishop Nicolas Zamora established the petitioner Iglesia
Evangelica Metodista En Las Islas Filipinas, Inc. (IEMELIF) as a corporation
sole with Bishop Zamora acting as its "General Superintendent." Thirty-nine
years later in 1948, the IEMELIF enacted and registered a by-laws that
established a Supreme Consistory of Elders (the Consistory), made up of
church ministers, who were to serve for four years. The by-laws empowered
the Consistory to elect a General Superintendent, a General Secretary, a
General Evangelist, and a Treasurer General who would manage the affairs of
the organization. For all intents and purposes, the Consistory served as the
IEMELIFs board of directors.
Apparently, although the IEMELIF remained a corporation sole on paper
(with all corporate powers theoretically lodged in the hands of one member,
the General Superintendent), it had always acted like a corporation
aggregate. The Consistory exercised IEMELIFs decision-making powers
without ever being challenged. Subsequently, during its 1973 General
Conference, the general membership voted to put things right by changing
IEMELIFs organizational structure from a corporation sole to a corporation
aggregate. On May 7, 1973 the Securities and Exchange Commission (SEC)

Page 789 of 1072

approved the vote. For some reasons, however, the corporate papers of the
IEMELIF remained unaltered as a corporation sole.
Only in 2001, about 28 years later, did the issue reemerge. In answer
to a query from the IEMELIF, the SEC replied on April 3, 2001 that, although
the SEC Commissioner did not in 1948 object to the conversion of the
IEMELIF into a corporation aggregate, that conversion was not properly
carried out and documented. The SEC said that the IEMELIF needed to
amend its articles of incorporation for that purpose.
ISSUE/S:
Whether or not a corporation may change its character as a
corporation sole into a corporation aggregate by mere amendment of
its articles of incorporation without first going through the process of
dissolution.
RULING:
True, the Corporation Code provides no specific mechanism for
amending the articles of incorporation of a corporation sole. But, as the RTC
correctly held, Section 109 of the Corporation Code allows the application to
religious corporations of the general provisions governing non-stock
corporations.
For non-stock corporations, the power to amend its articles of
incorporation lies in its members. The code requires two-thirds of their votes
for the approval of such an amendment. Although a non-stock corporation
has a personality that is distinct from those of its members who established
it, its articles of incorporation cannot be amended solely through the action
of its board of trustees. The amendment needs the concurrence of at least
two-thirds of its membership. If such approval mechanism is made to operate
in a corporation sole, its one member in whom all the powers of the
corporation technically belongs, needs to get the concurrence of two-thirds
of its membership. The one member, here the General Superintendent, is but
a trustee, according to Section 110 of the Corporation Code, of its
membership.
There is no point to dissolving the corporation sole of one member to
enable the corporation aggregate to emerge from it. Whether it is a nonstock corporation or a corporation sole, the corporate being remains distinct
from its members, whatever be their number. The increase in the number of
its corporate membership does not change the complexion of its corporate
responsibility to third parties. The one member, with the concurrence of twothirds of the membership of the organization for whom he acts as trustee,
can self-will the amendment. He can, with membership concurrence,
increase the technical number of the members of the corporation from "sole"
or one to the greater number authorized by its amended articles.

Page 790 of 1072

Here, the evidence shows that the IEMELIFs General Superintendent,


respondent Bishop Lazaro, who embodied the corporation sole, had obtained,
not only the approval of the Consistory that drew up corporate policies, but
also that of the required two-thirds vote of its membership.

TOPIC:DISSOLUTION OF CORPORATIONS: WHERE CREDITORS ARE NOT


AFFECTED
TEODORO B. VESAGAS, and WILFRED D. ASIS,
vs.
The Honorable COURT OF APPEALS and DELFINO RANIEL and
HELENDA RANIEL
G.R. No. 142924.December 5, 2001
FACTS:
The respondent spouses Delfino and Helenda Raniel are members in
good standing of the Luz Villaga Tennis Clud, Inc. (club). They alleged that
petitioner Teodoro B. Vesagas, who claims to be the club's duly elected
president, in conspiracy with petitioner Wilfred D. Asis, who, in turn, claims to
be its duly elected vice-president and legal counsel, summarily stripped
them of their lawful membership, without due process of law. Thereafter,
respondent spouses filed a Complaint with the Securities and Exchange
Commission (SEC) on March 26, 1997 against the petitioners. It was
docketed as SEC Case No. 03-97-5598.1 In this case, respondents asked the
Commission to declare as illegal their expulsion from the club as it was
allegedly done in utter disregard of the provisions of its by-laws as well as
the requirements of due process. They likewise sought the annulment of the
amendments to the by-laws made on December 8, 1996, changing the
annual meeting of the club from the last Sunday of January to November and
increasing the number of trustees from nine to fifteen. Finally, they prayed
for the issuance of a Temporary Restraining Order and Writ of Preliminary
Injunction.
ISSUE/S:
Whether or not the Corporation is dissolved at the time of the
institution of this case with the SEC
RULING:

Page 791 of 1072

Petitioners' claim in gratia argumenti that while the club may have
been considered a corporation during a brief spell, still, at the time of the
institution of this case with the SEC, the club was already dissolved by virtue
of a Board resolution.
Again, the argument will not carry the day for the petitioner. The
Corporation Code establishes the procedure and other formal requirements a
corporation needs to follow in case it elects to dissolve and terminate its
structure voluntarily and where no rights of creditors may possibly be
prejudiced, thus:
"Sec. 118.Voluntary dissolution where no creditors are affected. If dissolution of a corporation does not prejudice the rights of any
creditor having a claim against it, the dissolution may be
effected by majority vote of the board of directors or trustees
and by a resolution duly adopted by the affirmative vote of the
stockholders owning at least two-thirds (2/3) of the outstanding
capital stock or at least two-thirds (2/3) of the members at a
meeting to be held upon call of the directors or trustees after
publication of the notice of time, place and object of the meeting
for three (3) consecutive weeks in a newspaper published in the
place where the principal office of said corporation is located;
and if no newspaper is published in such place, then in a
newspaper of general circulation in the Philippines, after sending
such notice to each stockholder or member either by registered
mail or by personal delivery at least 30 days prior to said
meeting. A copy of the resolution authorizing the dissolution shall
be certified by a majority of the board of directors or trustees
and countersigned by the secretary of the corporation. The
Securities and Exchange Commission shall thereupon issue the
certificate of dissolution."
We note that to substantiate their claim of dissolution, petitioners
submitted only two relevant documents: the Minutes of the First Board
Meeting held on January 5, 1997, and the board resolution issued on April 14,
1997 which declared "to continue to consider the club as a non-registered or
a non-corporate entity and just a social association of respectable and
respecting individual members who have associated themselves, since the
1970's, for the purpose of playing the sports of tennis." Obviously, these two
documents will not suffice. The requirements mandated by the Corporation
Code should have been strictly complied with by the members of the club.
The records reveal that no proof was offered by the petitioners with regard to
the notice and publication requirements. Similarly wanting is the proof of the
board members' certification. Lastly, and most important of all, the SEC
Order of Dissolution was never submitted as evidence.

Page 792 of 1072

TOPIC: DISSOLUTION
AFFECTED

OF

CORPORATIONS:

WHERE

CREDITORS

ARE

AVON DALE GARMENTS, INC., vs. NATIONAL LABOR RELATIONS


COMMISSION, LILIA DUMANTAY, ET AL.
G.R. No. 117932. July 20, 1995
246 SCRA 733
FACTS:
Private respondents were employees of petitioner Avon Dale Garments,
Inc. and its predecessor-in-interest, Avon Dale Shirt Factory. Following a
dispute brought about by the rotation of workers, a compromise agreement
was entered into between petitioner and private respondents wherein the
latter were terminated from service and given their corresponding separation
pay.
However, upon refusal of the petitioner to include in the computation
of private respondents' separation pay the period during which the latter
were employed by Avon Dale Shirt Factory, private respondents filed a
complaint with the labor arbiter claiming a deficiency in their separation pay.
According to private respondents, their previous employment with
petitioner's predecessor-in-interest, Avon Dale Shirt Factory, should be
credited in computing their separation pay considering that Avon Dale Shirt
factory was not dissolved and they were not in turn hired as new employees
by Avon Dale Garments, Inc.
ISSUE/S:
Whether or not a dissolution actually took place
RULING:
Petitioner failed to establish that Avon Dale Garments, Inc. is a
separate and distinct entity from Avon Dale Shirt Factory, absent any

Page 793 of 1072

showing that there was indeed an actual closure and cessation of the
operations of the latter. The mere filing of the Articles of Dissolution with the
Securities and Exchange Commission, without more, is not enough to
support the conclusion that actual dissolution of an entity in fact took place.
On the contrary, the prevailing circumstances in this case indicated
that petitioner company is not distinct from its predecessor Avon Dale Shirt
Factory, but in fact merely continued the operations of the latter under the
same owners, the same business venture, at same address 6, and even
continued to hire the same employees.
Thus, conformably with established jurisprudence, the two entities
cannot be deemed as separate and distinct where there is a showing that
one is merely the continuation of the other. 7 In fact, even a change in the
corporate name does not make a new corporation, whether effected by a
special act or under a general law, it has no effect on the identity of the
corporation, or on its property, rights, or liabilities. Respondent NLRC
therefore, did not commit any grave abuse of discretion in holding that
petitioner should likewise include private respondents' employment with
Avon Dale Shirt Factory in computing private respondents' separation pay as
petitioner failed to substantiate its claim that it is a distinct entity.

Page 794 of 1072

TOPIC: DISSOLUTION
AFFECTED

OF

CORPORATIONS:

WHERE

CREDITORS

ARE

DAGUHOY ENTERPRISES, INC.,


vs.
RITA L. PONCE, with whom is joined her husband, DOMINGO PONCE
G.R. No.L-6515.October 18, 1954
96 PHIL 15
FACTS:
The Daguhoy Enterprises, Inc., a local corporation, with principal office
in the City of Manila filed in the Court of First Instance of the City Civil Case
No. 15923 against Rita L. Ponce and her husband Domingo Ponce, for the
collection of a loan of P6,190 with interest at 12 per cent per annum from
June 24, 1950, plus P2,500 as attorney's fees and P34 as expenses of
litigation. Defendant filed an answer admitting practically all the allegations
of the complaint, set up affirmative defenses, and a counterclaim asking for
the cancellation of the mortgage which secured the payment of the loan of
P6,190. They also filed a petition for the inclusion of Potenciano Gapol as a
third party litigant, at the same time filing a third party complaint against
him asking for damages in the amount of P25,000. The plaintiff corporation
answered the counterclaim and opposed the petition for the inclusion of a
third party litigant. Thereafter, plaintiff corporation filed a motion for
judgment on the pleadings which petition was opposed by the defendants.
ISSUE:
Whether plaintiff corporation had no legal capacity to sue for the
reason that as a corporation it no longer was in existence
RULING:

Page 795 of 1072

One of the affirmative defenses set up by the defendants is that the


plaintiff corporation had no legal capacity to sue for the reason that as a
corporation it no longer was in existence because on April 16, 1951, at a
meeting held by the stockholders and attended by Potenciano Gapol, the
majority stockholder, a resolution was adopted dissolving the said
corporation, and that as a matter of fact, Gapol was designated Assignee.
However, as contended by counsel for the appellee, a mere resolution by the
stockholders or by the Board of Directors of a corporation to dissolve the
same does not effect the dissolution but that some other step, administrative
or judicial, is necessary. Furthermore, as stated by the trial court in its
decision, under section 77 of the Corporation Law, a corporation dissolved
will continue in existence as a judicial entity for a period of three years after
the declaration of its dissolution, to windup its affairs and protect its interests
during the period of liquidation.
TOPIC: DISSOLUTION OF CORPRATIONS: INVOLUNTARY
PHILIPPINE NATIONAL BANK & NATIONAL SUGAR DEVELOPMENT
CORPORATION, vs.
ANDRADA ELECTRIC & ENGINEERING COMPANY
G.R. No. 142936. April 17, 2002
209 SCRA 294
FACTS:
In its complaint, the respondent alleged that it is a partnership duly
organized, existing, and operating under the laws of the Philippines, while
the petitioner Philippine National Bank, is a semi-government corporation
duly organized, existing and operating under the laws of the Philippines the
other defendant, the National Sugar Development Corporation (NASUDECO
in brief), is also a semi-government corporation and the sugar arm of the
PNB,; and the defendant Pampanga Sugar Mills (PASUMIL in short), is a
corporation organized, existing and operating under the 1975 laws of the
Philippines, and had its business office before 1975 at Del Carmen,
Floridablanca, Pampanga.
On August 26, 1975, the defendant PNB acquired the assets of the
defendant PASUMIL that were earlier foreclosed by the Development Bank of
the Philippines (DBP) under LOI No. 311; that the defendant PNB organized
the defendant NASUDECO in September, 1975, to take ownership and
possession of the assets and ultimately to nationalize and consolidate its
interest in other PNB controlled sugar mills; that prior to October 29, 1971,
the defendant PASUMIL engaged the services of plaintiff for electrical
rewinding and repair, most of which were partially paid by the defendant
PASUMIL, leaving several unpaid accounts with the plaintiff; that finally, on
October 29, 1971, the plaintiff and the defendant PASUMIL entered into a
contract.

Page 796 of 1072

Out of the total obligation of P777,263.80, the defendant PASUMIL had


paid only P250,000.00, leaving an unpaid balance, as of June 27, 1973.
ISSUE/S:
Whether the Court of Appeals gravely erred in law in holding the herein
petitioners liable for the unpaid corporate debts of PASUMIL, a
corporation whose corporate existence has not been legally
extinguished or terminated
RULING:
Petitioners posit that they should not be held liable for the corporate
debts of PASUMIL, because their takeover of the latters foreclosed assets did
not make them assignees. On the other hand, respondent asserts that
petitioners and PASUMIL should be treated as one entity and, as such, jointly
and severally held liable for PASUMILs unpaid obligation.
As a rule, a corporation that purchases the assets of another will not be
liable for the debts of the selling corporation, provided the former acted in
good faith and paid adequate consideration for such assets, except when any
of the following circumstances is present:
1) where the purchaser expressly or impliedly agrees to assume the
debts,
2) where the transaction amounts to a consolidation or merger of
the corporations,
3) where the purchasing corporation is merely a continuation of the
selling corporation, and
4) where the transaction is fraudulently entered into in order to
escape liability for those debts.

Page 797 of 1072

TOPIC: LIQUIDATION: METHODS


METROPOLITAN BANK & TRUST COMPANY, INC.
vs.
THE BOARD OF TRUSTEES OF RIVERSIDE MILLS CORPORATION
PROVIDENT AND RETIREMENT FUND
G.R. No. 176959 September 8, 2010
FACTS:
RMC established a Provident and Retirement Plan (Plan) for its regular
employees. Under the Plan, RMC and its employees shall each contribute 2%
of the employees current basic monthly salary, with RMCs contribution to
increase by 1% every five (5) years up to a maximum of 5%. The
contributions shall form part of the provident fund (the Fund) which shall be
held, invested and distributed by the Commercial Bank and Trust Company.
On October 15, 1979, the Board of Trustees of RMCPRF (the Board) entered
into an Investment Management Agreement(Agreement) with Philbank (now,
petitioner Metropolitan Bank and Trust Company). Pursuant to the
Agreement, petitioner shall act as an agent of the Board and shall hold,
manage, invest and reinvest the Fund in Trust Account No. 1797 in its behalf.
The Agreement shall be in force for one (1) year and shall be deemed
automatically renewed unless sooner terminated either by petitioner bank or
by the Board.
In 1984, RMC ceased business operations. Nonetheless, petitioner
continued to render investment services to respondent Board. In a letter
dated September 27, 1995, petitioner informed respondent Board that
Philbanks Board of Directors had decided to apply the remaining trust assets
held by it in the name of RMCPRF against part of the outstanding obligations
of RMC. Subsequently, respondent RMC Unpaid Employees Association, Inc.
(Association), representing the terminated employees of RMC, learned of
Trust Account No. 1797. Through counsel, they demanded payment of their

Page 798 of 1072

share in a letter dated February 4, 1997. When such demand went


unheeded, the Association, along with the individual members of RMCPRF,
filed a complaint for accounting against the Board and its officers.
ISSUE/S:
Whether or not the petitioner had the power to effect reversion of the
Fund to RMC;
Whether or not the functions of the Board of Trustees ceased upon with
RMCs closure
RULING:
1. No.
Under Paragraph 6 of the Agreement, petitioners function shall
be limited to the liquidation and return of the Fund to the Board upon
the termination of the Agreement. Paragraph 14 of said Agreement
further states that "it shall be the duty of the Investment Manager to
assign, transfer, and pay over to its successor or successors all cash,
securities, and other properties held by it constituting the fund less any
amounts constituting the charges and expenses which are authorized
[under the Agreement] to be payable from the Fund." Clearly,
petitioner had no power to effect reversion of the Fund to RMC.
The reversion petitioner effected also could hardly be said to
have been done in good faith and with due regard to the rights of the
employee-beneficiaries. The restriction imposed under Paragraph 13 of
the Plan stating that "in no event shall any part of the assets of the
Fund revert to the Company before all liabilities of the Plan have been
satisfied," demands more than a passive stance as that adopted by
petitioner in locating claims against the Fund. Besides, the
beneficiaries of the Fund are readily identifiable the regular or
permanent employees of RMC who were qualified retirees and those
who were terminated as a result of its closure. Petitioner needed only
to secure a list of the employees concerned from the Board of Trustees
which was its principal under the Agreement and the trustee of the
Plan or from RMC which was the trustor of the Fund under the
Retirement Plan. Yet, petitioner notified respondent Board of Trustees
only after Philbanks Board of Directors had decided to apply the
remaining trust assets of RMCPRF to the liabilities of the company.
2. No.
Under Section 122 of the Corporation Code, a dissolved
corporation shall nevertheless continue as a body corporate for three
(3) years for the purpose of prosecuting and defending suits by or
against it and enabling it to settle and close its affairs, to dispose and
convey its property and to distribute its assets, but not for the purpose

Page 799 of 1072

of continuing the business for which it was established. Within those


three (3) years, the corporation may appoint a trustee or receiver who
shall carry out the said purposes beyond the three (3)-year winding-up
period. Thus, a trustee of a dissolved corporation may commence a
suit which can proceed to final judgment even beyond the three (3)year period of liquidation.
In the same manner, during and beyond the three (3)-year
winding-up period of RMC, the Board of Trustees of RMCPRF may do no
more than settle and close the affairs of the Fund. The Board retains its
authority to act on behalf of its members, albeit, in a limited capacity.
It may commence suits on behalf of its members but not continue
managing the Fund for purposes of maximizing profits. Here, the
Boards act of issuing the Resolution authorizing petitioner to release
the Fund to its beneficiaries is still part of the liquidation process,
which is, satisfaction of the liabilities of the Plan, and does not amount
to doing business. Hence, it was properly within the Boards power to
promulgate.
TOPIC: LIQUIDATION: METHODS
VICTOR YAM & YEK SUN LENT, doing business under the name and
style of Philippine Printing Works; petitioners,
vs.
THE COURT OF APPEALS and MANPHIL INVESTMENT CORPORATION,
respondents.
GR No. 104726, 11 February 1999
FACTS:
Parties entered into several laon agreements, the petitioners, Yam and
Lent, being the borrower while the private respondent, Manphil Investment
Corporaton, the lender. After payment of the first loan, the latter was placed
under a receivership by the Central Bank, appointing Lirio and Destejo as
receiver and in-house examiner, respectively. A check was sent to
respondent as partial payment of the second loan which was marked as full
payment in the vouchers. Demands were made for the balance of the same,
however, it was unheeded prompting respondent to file a case against the
petitioner for collection of the balance. The trial court ruled in favor of
respondents which the Court of Appeals affirmed.
ISSUE/S:
Whether or not petitioner is liable to the penalties and service charges
of the loan.

Page 800 of 1072

RULING:
The alleged condonation of the penalties and service charges by
Sobrepeas, president of respondent, must be in writing to be binding
between and among the parties. Since it was not reduced in writing, the
same is not effective. Further, the alleged condonation happened after the
resopondent corporation was placed under receivership. As held in
Villanueva v. Court of Appeals the appointment of a receiver operates to
suspend the authority of a [corporation] and of its directors and officers over
its property and effects, such authority being reposed in the receiver. Thus,
Sobrepeas had no authority to condone the debt. Petition denied.

TOPIC: LIQUIDATION: METHODS


ALHAMBRA CIGAR & CIGARETTE MANUFACTURING COMPANY, INC.,
petitioner,
vs.
SECURITIES & EXCHANGE COMMISSION, respondent.
G.R. NO. L-23606 JULY 29, 1968
FACTS:
Petitioner Alhambra Cigar and Cigarette Manufacturing Company, Inc.
was duly incorporated under Philippine laws on January 15, 1912. By its
corporate articles it was to exist for fifty (50) years from incorporation. Its
term of existence expired on January 15, 1962. On that date, it ceased
transacting business, entered into a state of liquidation. Thereafter, a new
corporation. Alhambra Industries, Inc. was formed to carry on the business of
Alhambra.
On June 20, 1963 within Alhambra's three-year statutory period for
liquidation - Republic Act 3531 was enacted into law. It amended Section 18
of the Corporation Law; it empowered domestic private corporations to
extend their corporate life beyond the period fixed by the articles of
incorporation for a term not to exceed fifty years in any one instance.
Previous to Republic Act 3531, the maximum non-extendible term of such
corporations was fifty years.
On July 15, 1963, at a special meeting, Alhambra's board of directors
resolved to amend paragraph "Fourth" of its articles of incorporation to
extend its corporate life for an additional fifty years, or a total of 100 years
from its incorporation. FOURTH. That the term for which said corporation is

Page 801 of 1072

to exist is fifty (50) years from and after the date of incorporation, and for an
additional period of fifty (50) years thereafter.On October 28, 1963,
Alhambra's articles of incorporation as so amended certified correct by its
president and secretary and a majority of its board of directors, were filed
with respondent Securities and Exchange Commission (SEC).
ISSUE/S:
Whether or not the corporation can still extend its corporate term
within the three-year statutory period for liquidation.
RULING:
A corporation cannot extend its life by amendment of its articles of
incorporation effected during the three-year period for liquidation when its
original term of existence had already expired. Since the privilege of
extension is purely statutory, all of the statutory conditions precedent must
be complied with in order that the extension may be effectuated. And,
generally these conditions must be complied with, and the steps necessary
to effect the extension must be taken, during the life of the corporation, and
before the expiration of the term of existence as original fixed by its charter
or the general law, since, as a rule, the corporation is ipso facto dissolved as
soon as that time expires. So where the extension is by amendment of the
articles of incorporation, the amendment must be adopted before that time.
And, similarly, the filing and recording of a certificate of extension after that
time cannot relate back to the date of the passage of a resolution by the
stockholders in favor of the extension so as to save the life of the
corporation. The contrary is true, however, and the doctrine of relation will
apply, where the delay is due to the neglect of the officer with whom the
certificate is required to be filed, or to a wrongful refusal on his part to
receive it. And statutes in some states specifically provide that a renewal
may be had within a specified time before or after the time fixed for the
termination of the corporate existence".

Page 802 of 1072

TOPIC: LIQUIDATION: METHODS


CHUNG KA BIO, WELLINGTON CHUNG, CHUNG SIONG PEK,
VICTORIANO CHUNG, and MANUEL CHUNG TONG OH, petitioners,
vs.
INTERMEDIATE APPELLATE COURT (2nd Special Cases Division),
SECURITIES and EXCHANGE COMMISSION EN BANC, HON. ANTONIO
R. MANABAT, HON. JAMES K. ABUGAN, HON. ANTERO F.L. VILLAFLOR,
JR., HON. SIXTO T.J. DE GUZMAN, JR., ALFREDO CHING, CHING TAN,
CHIONG TIONG TAY, CHUNG KIAT HUA, CHENG LU KUN, EMILIO
TAEDO, ROBERTO G. CENON and PHILIPPINE BLOOMING MILLS
COMPANY, INC., respondents.
G.R. NO. 71837 JULY 26, 1988
FACTS:
The Philippine Blooming Mills Company, Inc. was incorporated on
January 19, 1952, for a term of 25 years which expired on January 19,1977.
On May 14, 1977, the members of its board of directors executed a deed of
assignment of all of the accounts receivables, properties, obligations and
liabilities of the old PBM in favor of Chung Siong Pek in his capacity as
treasurer of the new PBM, then in the process of reincorporation. On June 14,
1977, the new PMB was issued a certificate of incorporation by the Securities
and Exchange Commission.

Page 803 of 1072

On May 5, 1981, Chung Ka Bio and the other petitioners herein, all
stockholders of the old PBM, filed with the SEC a petition for liquidation (but
not for dissolution) of both the old PBM and the new PBM. The allegation was
that the former had become legally non-existent for failure to extend its
corporate life and that the latter had likewise been ipso facto dissolved for
non-use of the charter and continuous failure to operate within 2 years from
incorporation.

ISSUE/S:
Whether or not the board of directors of an already dissolved
corporation does not have the inherent power, without the express
consent of the stockholders, to convey all its assets to a new
corporation.
Whether or not the new corporation has not substantially complied
with the two-year requirement of Section 22 of the new Corporation
Code on non-user because its stockholders never adopted a set of bylaws.
RULING:
1. There is the presumption of regularity which must operate in favor
of the private respondents, who insist that the proper authorization as
required by the Corporation Law was duly obtained at a meeting called for
the purpose. Otherwise, the new PBM would not have been issued a
certificate of incorporation, which should also be presumed to have been
done regularly. It must also be noted that under Section 28-1/2, "any
stockholder who did not vote to authorize the action of the board of directors
may, within forty days after the date upon which such action was authorized,
object thereto in writing and demand payment for his shares." The record
does not show, nor have the petitioners alleged or proven, that they filed a
written objection and demanded payment of their shares during the
reglementary forty-day period. This circumstance should bolster the private
respondents' claim that the authorization was unanimous.
While we agree that the board of directors is not normally permitted to
undertake any activity outside of the usual liquidation of the business of the
dissolved corporation, there is nothing to prevent the stockholders from
conveying their respective shareholdings toward the creation of a new
corporation to continue the business of the old. Winding up is the sole
activity of a dissolved corporation that does not intend to incorporate anew.
If it does, however, it is not unlawful for the old board of directors to
negotiate and transfer the assets of the dissolved corporation to the new
corporation intended to be created as long as the stockholders have given

Page 804 of 1072

their consent. This was not prohibited by the Corporation Act. In fact, it was
expressly allowed by Section 28-1/2.
2. Non-filing of the by-laws will not result in automatic dissolution of
the corporation. Under Section 6(i) of PD 902-A, the SEC is empowered to
"suspend or revoked, after proper notice and hearing, the franchise or
certificate of registration of a corporation" on the ground inter alia of "failure
to file by-laws within the required period." It is clear from this provision that
there must first of all be a hearing to determine the existence of the ground,
and secondly, assuming such finding, the penalty is not necessarily
revocation but may be only suspension of the charter. In fact, under the rules
and regulations of the SEC, failure to file the by-laws on time may be
penalized merely with the imposition of an administrative fine without
affecting the corporate existence of the erring firm. In any case, the
deficiency claimed by the petitioners was corrected when the new PBM
adopted and filed its by-laws on September 6, 1981, thus rendering the third
issue also moot and academic. It should be stressed in this connection that
substantial compliance with conditions subsequent will suffice to perfect
corporate personality.

TOPIC: LIQUIDATION: METHODS


REPUBLIC OF THE PHILIPPINES
VS.
MARSMAN DEVELOPMENT COMPANY
G.R. NO. L-18956 APRIL 27, 1972
FACTS:
Sometime before October 15, 1953 an investigation was conducted on
the business operation and activities of the corporation leading to the
discovery that certain taxes were due (from) it on logs produced from its
concession. The Bureau of Internal Revenue made three assessments,
totalling P59,133.78, and demanded payment thereof. Defendants however
failed to pay the taxes hence the filing of charges in court. The defendants
contend that the present action is already barred under section 77 of the
Corporation Law, Act No. 1459, as amended, which allows the corporate
existence of a corporation to continue only for three years after its
dissolution, for the purpose of presenting or defending suits by or against it,
and to settle and close its affairs. They point out that inasmuch as the
Marsman Development Co. was extra-judicially dissolved on April 23, 1954, a
fact admitted in the amended complaint, the filing of both the original

Page 805 of 1072

complaint on September 8, 1958 and the amended complaint on August 26,


1956 was beyond the aforesaid three-year period.
ISSUE/S:
Whether or not the right of the government to collect the sums
has already prescribed.
RULING:
The stress given by appellants to the extinction of the corporate and
juridical personality as such of Appellant Corporation by virtue of its extrajudicial dissolution which admittedly took place on April 23, 1954 is
misdirected. While Section 77 of the Corporation Law does provide that:
Every corporation whose charter expires by its own limitation or is
annulled by forfeiture or otherwise, or whose corporate existence for other
purposes is terminated in any other manner, shall nevertheless be continued
as a body corporate for three years after the time when it would have been
so dissolved, for the purpose of prosecuting and defending suits by or
against it and of enabling it gradually to settle and close its affairs, to
dispose of and convey its property and to divide its capital stock, but not for
the purpose of continuing the business for which it was established.
Further, at any time during said three years said corporation is
authorized and empowered to convey all of its property to trustees for the
benefit of members, stock-holders, creditors, and others interested. From
and after any such conveyance by the corporation of its property in trust for
the benefit of its members, stockholders, creditors, and others in interest, all
interest which the corporation had in the property terminates, the legal
interest vests in the trustee, and the beneficial interest in the members,
stockholders, creditors, or other persons in interest.
It is to be recalled that the assessments against appellant corporation
for deficiency taxes due for its operations since 1947 were made by the
Bureau of Internal Revenue on October 15, 1953, September 13, 1954 and
November 8, 1954, such that the first was before its dissolution and the last
two not later than six months after such dissolution. Thus, in whatever way
the matter may be viewed, the Government became the creditor of the
corporation before the completion of its dissolution by the liquidation of its
assets. Appellant F.H. Burgess, whom it chose as liquidator, became in law
the trustee of all its assets for the benefit of all persons enumerated in
Section 78, including its creditors, among whom is the Government, for the
taxes herein involved. To assume otherwise would render the extra-judicial
dissolution illegal and void, since, according to Section 62 of the Corporation
Law, such kind of dissolution is permitted only when it "does not affect the
rights of any creditor having a claim against the corporation." It is immaterial

Page 806 of 1072

that the present action was filed after the expiration of three years after April
23, 1954, for at the very least, and assuming that judicial enforcement of
taxes may not be initiated after said three years despite the fact that the
actual liquidation has not been terminated and the one in charge thereof is
still holding the assets of the corporation, obviously for the benefit of all the
creditors thereof, the assessment aforementioned, made within the three
years, definitely established the Government as a creditor of the corporation
for whom the liquidator is supposed to hold assets of the corporation. And
since the suit at bar is only for the collection of taxes finally assessed against
the corporation within the three years invoked by appellants, their fourth
assignment of error cannot be sustained. As to the allegation that appellant
Burgess has not in fact received any property or asset of the corporation,
that is a matter that can well be taken care of in the execution of the
judgment which may be rendered herein, albeit it seems some kind of fraud
would be perceptible, if the corporation had been dissolved without leaving
any assets whatsoever with the liquidator.

TOPIC: LIQUIDATION: METHODS


TAN TIONG BIO, ET AL., petitioners,
vs.
COMMISSIONER OF INTERNAL REVENUE, respondent.
G.R. NO. L-15778 APRIL 23, 1962
FACTS:
On October 19, 1946, the Central Syndicate, a corporation organized
under the laws of the Philippines, thru its General Manager, David Sycip, sent
a letter to the Collector of Internal Revenue advising the latter that it
purchased from Dee Hong Lue the entire stock of surplus properties which
the said Dee Hong Lue had bought from the Foreign Liquidation Commission
and that as it assumed Dee Hong Lue's obligation to pay the 3-1/2% sales
tax on said surplus goods, it was remitting the sum of P43,750.00 in his
behalf as deposit to answer for the payment of said sales tax with the
understanding that it would later be adjusted after the determination of the
exact consideration of the sale.

Page 807 of 1072

On January 31, 1948, the syndicate again wrote the Collector


requesting the refund of P1,103.28 representing alleged excess payment of
sales tax due to the adjustment and reduction of the purchase price in the
amount of P31,522.18. The Collector decided after a thorough investigation
of the facts that the Central Syndicate was the importer and original seller of
the surplus goods in question and, therefore, the one liable to pay the sales
tax. Accordingly, on January 4, 1952, the Collector assessed against the
syndicate the amount of P33,797.88 and P300.00 as deficiency sales tax,
inclusive of the 25% surcharge and compromise penalty, respectively, and
on the same date, in a separate letter, he denied the request of the
syndicate for the refund of the sum of P1,103.28.
On September 8, 1954, the Central Syndicate elevated the case to the
Court of Tax Appeals questioning the ruling of the Collector which denies its
claim for refund as well as the assessment made against it of the sum of
P33,797.88, plus the sum of P300.00 as compromise penalty. On November
5, 1954, the Collector filed a motion requiring the syndicate to file a bond to
guarantee the payment of the tax assessed against it on the ground that
cannot be legally done it appearing that the syndicate is already a nonexisting entity due to the expiration of its corporate existence.
ISSUE/S:
Whether the sales tax in question can be enforced against its
successors-in-interest who are the present petitioners.
RULING:
The creditor of a dissolved corporation may follow its assets once they
passed into the hands of the stockholders. Thus, recognized are the following
rules in American jurisprudence: The dissolution of a corporation does not
extinguish the debts due or owing to it. A creditor of a dissolve corporation
may follow its assets, as in the nature of a trust fund, into the hands of its
stockholders. An indebtedness of a corporation to the federal government for
income and excess profit taxes is not extinguished by the dissolution of the
corporation. And it has been stated, with reference to the effect of
dissolution upon taxes due from a corporation, "that the hands of the
government cannot, of course, collect taxes from a defunct corporation, it
loses thereby none of its rights to assess taxes which had been due from the
corporation, and to collect them from persons, who by reason of transactions
with the corporation, hold property against which the tax can be enforced
and that the legal death of the corporation no more prevents such action
than would the physical death of an individual prevent the government from
assessing taxes against him and collecting them from his administrator, who
holds the property which the decedent had formerly possessed". Bearing in
mind that our corporation law is of American origin, the foregoing authorities

Page 808 of 1072

have persuasive effect in considering similar cases in this jurisdiction. This


must have been taken into account when in G.R. No. L-8800 this Court said
that petitioners could be held personally liable for the taxes in question as
successors-in-interest of the defunct corporation.

TOPIC: LIQUIDATION: DURATION


REYNOLDS PHILIPPINE CORPORATION, petitioner,
vs.
HON. COURT OF APPEALS and SERG'S PRODUCTS, INC., respondents.
G.R. No.L-36187. January 17, 1989
FACTS:
On June 2, 1966, the petitioner sought to recover from the private
respondent Serg's Products, Inc. the sum of P32,565.62 representing the
unpaid price of aluminum foils and cores sold and delivered by it to the
latter.
The private respondent denied liability for payment of the account on
the ground that the aluminum foils and cores were ordered or purchased by
Serg's Chocolate Products, a partnership of Antonio Goquiolay and Luis
Sequia Mendoza, not Serg's Products, Inc., a corporation managed and
controlled by Antonio Goquiolay and his wife Conchita Goquiolay, as majority
stockholders and principal officers. The trial court rendered judgment finding
the private respondent liable.

Page 809 of 1072

Upon private respondent's appeal to the Court of Appeals reversed the


trial court and dismissed the complaint.
ISSUE/S:
Who is the real debtor of the petitioner?
RULING:
In this case, the trial court which heard the witnesses testify, hence
was in a superior position to assess the probative worth of their evidence,
found that although the commercial documents were indeed in the name of
"Serg's Chocolate Products," the following facts proved that the true
purchaser of the aluminum foils and cores from the petitioner, was "Serg's
Products, Inc." not the partnership denominated "Serg's Chocolate
Products".
The following evidence supported the findings of the court:
1) The rolls of aluminum foil were ordered and signed for by Antonio
Goquiolay president of Serg's Products, Inc.
2) Antonio Goquiolay did not appear in court to shed light on
whether he signed the purchase orders and delivery receipts as
managing partner of "Serg's Chocolate Products," or as president
and general manager of "Serg's Products, Inc."
3) The error in identifying the customer as "Serg's Chocolate
Products," instead of Serg's Products, Inc." in the sales orders,
delivery receipts and invoices was caused by Antonio Goquiolay
himself who placed the orders.
4) The trial court noted that "Serg's Products, Inc." "Acted in such a
manner that third persons dealing with it were led to believe that
'Serg's Products, Inc.' and 'Serg's Chocolate Products' were one
and the same party.
5) Serg's Chocolate Products ceased to exist in 1959 for under the
partnership Agreement between Goquiolay and Mendoza had a
term of five (5) years, or up to 1959 only.
While that term was renewable for the same period upon agreement of
the parties, no evidence was adduced that it was renewed after it expired in
1959. Having ceased to exist since 1959, the partnership has no more
juridical personality nor capacity to sue and be sued. "Serg's Chocolate
Products" is nothing but a name now which the manager of Serg's Products,
Inc. appears to have used to confuse, deceive, and delay, if not completely
evade, the payment of the corporation's just debt to the petitioner.

Page 810 of 1072

TOPIC: LIQUIDATION: DURATION


MAMBULAO LUMBER COMPANY, plaintiff-appellant,
vs.
PHILIPPINE NATIONAL BANK and ANACLETO HERALDO Deputy
Provincial Sheriff of Camarines Norte, defendants-appellees.
G.R. No.L-22973.January 30, 1968
FACTS:
On May 5, 1956 the plaintiff applied for an industrial loan of P155,000
with the Naga Branch of defendant PNB and the former offered real estate,
machinery, logging and transportation equipments as collaterals. The
application, however, was approved for a loan of P100,000 only. To secure
the payment of the loan, the plaintiff mortgaged to defendant PNB a parcel
of land, together with the buildings and improvements existing thereon,
situated in the poblacion of Jose Panganiban (formerly Mambulao), province
of Camarines Norte, and covered by Transfer Certificate of Title No. 381 of
the land records of said province, as well as various sawmill equipment,

Page 811 of 1072

rolling unit and other fixed assets of the plaintiff, all situated in its compound
in the aforementioned municipality.
The plaintiff failed to pay the amortization on the amounts released to
and received by it. Repeated demands were made upon the plaintiff to pay
its obligation but it failed or otherwise refused to do so. Upon inspection and
verification made by employees of the PNB, it was found that the plaintiff
had already stopped operation about the end of 1957 or early part of 1958.
On November 8, 1961, Deputy Provincial Sheriff Anacleto Heraldo took
possession of the chattels mortgaged by the plaintiff and made an inventory
thereof in the presence of a PC Sergeant and a policeman of the municipality
of Jose Panganiban. On November 9, 1961, the said Deputy Sheriff issued the
corresponding notice of public auction sale of the mortgaged chattels to be
held on November 21, 1961, at 10:00 a.m., at the plaintiff's compound
situated in the municipality of Jose Panganiban, Province of Camarines Norte.
On December 21, 1961, the foreclosure sale of the mortgaged chattels
was held at 10:00 a.m. and they were awarded to the PNB for the sum of P4,
200 and the corresponding bill of sale was issued in its favor by Deputy
Provincial Sheriff Heraldo.
Upon the foregoing facts, the trial court rendered the decision
appealed from which, as stated in the first paragraph of this opinion,
sentenced the Mambulao Lumber Company to pay to the defendant PNB the
sum of P3, 582.52 with interest thereon at the rate of 6% per annum from
December 22, 1961 (day following the date of the questioned foreclosure of
plaintiff's chattels) until fully paid, and the costs. Mambulao Lumber
Company interposed the instant appeal.
ISSUE/S:
Whether the claim that the proceeds of the sale of the real
properties alone together with the amount it remitted to the PNB
later was more than sufficient to liquidate its total obligation to
herein appellee bank.
RULING:
It is clear that there was no further necessity to foreclose the mortgage
of herein appellant's chattels on December 21, 1961; and on this ground
alone, we may declare the sale of appellant's chattels on the said date,
illegal and void. But we take into consideration the fact that the PNB must
have been led to believe that the stipulated 10% of the unpaid loan for
attorney's fees in the real estate mortgage was legally maintainable, and in
accordance with such belief, herein appellee bank insisted that the proceeds
of the sale of appellant's real property was deficient to liquidate the latter's

Page 812 of 1072

total indebtedness. Be that as it may, however, we still find the subsequent


sale of herein appellant's chattels illegal and objectionable on other grounds.
While the law grants power and authority to the mortgagee to sell the
mortgaged property at a public place in the municipality where the
mortgagor resides or where the property is situated, this Court has held that
the sale of a mortgaged chattel may be made in a place other than that
where it is found, provided that the owner thereof consents thereto; or that
there is an agreement to this effect between the mortgagor and the
mortgagee.But when, as in this case, the parties agreed to have the sale of
the mortgaged chattels in the City of Manila, which, any way, is the
residence of the mortgagor, it cannot be rightly said that mortgagee still
retained the power and authority to select from among the places provided
for in the law and the place designated in their agreement over the objection
of the mortgagor. In providing that the mortgaged chattel may be sold at the
place of residence of the mortgagor or the place where it is situated, at the
option of the mortgagee, the law clearly contemplated benefits not only to
the mortgagor but to the mortgagee as well.
WHEREFORE AND CONSIDERING ALL THE FOREGOING, the decision
appealed from should be, as hereby, it is set aside. The Philippine National
Bank and the Deputy Sheriff of the province of Camarines Norte are ordered
to pay, jointly and severally, to Mambulao Lumber Company the total
amount of P56, 000.73, broken as follows: P150.73 overpaid by the latter to
the PNB, P42, 850.00 the value of the chattels at the time of the sale with
interest at the rate of 6% per annum from December 21, 1961, until fully
paid, P10, 000.00 in exemplary damages, and P3, 000.00 as attorney's fees.
Costs against both appellees
TOPIC: POWERS OF CORPORATION AT LIQUIDATION
CATMON SALES INTERNATIONAL CORPORATION, Petitioner,
vs.
ATTY. MANUEL D. YNGSON, JR., as Liquidator of Catmon Sales
International Corporation, Respondent.
G.R. No. 179761.January 15, 2010
FACTS:
On February 8, 1999, petitioner Catmon Sales International Corporation
filed a Petition for Declaration in a State of Suspension of Payments with the
SEC. The case was docketed as SEC Case No. 02-99-6204.
On May 10, 2000, the SEC declared petitioner technically insolvent
considering that there was no settlement reached with its creditors and that
its inability to pay its creditors had lasted for a period longer than one year
from the filing of the petition. In an Order dated August 28, 2000, the SEC

Page 813 of 1072

denied petitioners motion for reconsideration. In the same Order, the SEC
appointed respondent Manuel D. Yngson, Jr. of Receivers and Liquidators, Inc.
as petitioners liquidator.
On May 31, 2001, the SEC terminated the services of respondent.
Respondent, in turn, submitted his Accomplishment Report summarizing all
the activities he had undertaken and billed the SEC the total sum of
P623,214.35, representing his liquidators fee and reimbursement of out-ofpocket expenses. On December 18, 2001, the SEC ordered that an audit be
conducted to determine the proper amount to be paid to respondent. The
Corporation Finance Department noted a slight difference in the liquidators
computation. On September 23, 2004, respondent manifested to the SEC
that he was willing to reduce his liquidators fee provided that his request for
administrative expenses be settled in full.
On June 23, 2005, the SEC, through its General Counsel, ordered the
members of the Board of Directors of petitioner to pay respondent his claim
for reimbursement of the expenses incurred in the performance of his duties
as liquidator, together with his liquidators fee, for a total amount of P398,
284. 40. Petitioners motion for reconsideration was denied on October 11,
2005.
ISSUE/S:
Whether the Board of Directors of Catmon should be held liable
for the claim of the liquidator.
RULING:
The Court notes that respondent initially demanded P623, 214.35,
representing his liquidators fee of P450, 000.00 and out-of-pocket expenses
of P173, 214.35. Respondent later manifested that he was amenable to
reduce by one-half his liquidators fee. Before fixing the amount due the
respondent, the SEC, in fact, ordered that an audit be conducted to
determine the proper amount to be paid. Clearly, the fee fixed by the SEC
was not without basis. Besides, as correctly held by the CA, "respondent
actually rendered services in accordance with his oath of office as liquidator
for which he is entitled to be compensated by petitioner.
WHEREFORE, premises considered, the petition is DENIED for lack of merit.
The Court of Appeals Decision dated April 24, 2007 and Resolution dated
September 14, 2007 in CA-G.R. SP No. 95938 is AFFIRMED.

Page 814 of 1072

TOPIC: POWERS OF CORPORATION AT LIQUIDATION


RENE KNECHT and KNECHT, INC., petitioners,
vs.
UNITED CIGARETTE CORP., represented by ENCARNACION GONZALES
WONG, and EDUARDO BOLIMA, Sheriff, Regional Trial Court, Branch
151, Pasig City,respondents.
G.R. No. 139370. July 4, 2002
FACTS:
Rose Packing Company, Inc. (Rose Packing), a domestic corporation,
owns three (3) parcels of land with a total area of 31, 842 square meters
situated in Sto. Domingo, Cainta, Rizal. The largest among these parcels has
an area of 31,447 square meters covered by Transfer Certificate of Title (TCT)
No. 73620 of the Registry of Deeds of Rizal. The other two remaining parcels
are unregistered. The area covered by TCT No. 73620 is mortgaged with the
Philippine Commercial and Industrial Bank (PCIB).

Page 815 of 1072

On October 26, 1965, Rose Packing, through its President Rene Knecht,
sold to the United Cigarette Corporation (UCC), a domestic corporation, the
said parcels of land, with all the buildings and improvements thereon, for
P800,000.00. Rose Packing made a warranty that the lots are free from all
liens and encumbrances, except the real estate mortgage constituted over
the area covered by TCT No. 73620. For its part, UCC promised to pay the
purchase price under the following terms and conditions: (a) a P250,000.00
down payment must be made upon signing of the deed of sale with
mortgage; (b) it will assume Rose Packings P250,000.00 overdraft line
obligation with the PCIB, subject to the latters approval; and (c) the balance
of P300,000.00 shall be paid in two annual installments at P150,000.00 each
(within 12 and 14 months) from the date of sale, with 10% annual interest.
To secure the deal, UCC initially paid Rose Packing P80,000.00 as earnest
money.
Before the deed of sale could be executed, the parties found that Rose
Packings actual obligation with the PCIB far exceeded the P250,000.00
which UCC assumed to pay under their agreement. So the PCIB demanded
additional collateral from UCC as a condition precedent for the approval of
the sale of the mortgaged property. However, UCC did not comply.
Meanwhile, Rose Packing again offered to sell the same lots to other
prospective buyers without the knowledge of UCC and without returning to
the latter the earnest money it earlier paid.
Aggrieved, UCC, on March 2, 1966, filed with the then Court of First
Instance (CFI) of Rizal, Branch I, a complaint against Rose Packing and Rene
Knecht for specific performance and recovery of damages.
ISSUE/S:
Whether or not a corporation liquidated pending litigation can be
accorded with the protection an ordinary corporation enjoys.
RULING:
Petitioners basis in filing these multiple petitions is the expiration of
UCCs corporate existence.Indeed, the rights of a corporation (dissolved
pending litigation) are accorded protection by law. This is clear from Section
145 of the Corporation Code, thus:Section 145. Amendment or repeal. No
right or remedy in favor of or against any corporation, its stockholders,
members, directors, trustees, or officers, nor any liability incurred by any
such corporation, stockholders, members, directors, trustees, or officers,
shall be removed or impaired either by the subsequent dissolution of said
corporation or by any subsequent amendment or repeal of this Code or of
any part thereof.

Page 816 of 1072

The dissolution of UCC itself, or the expiration of its three-year


liquidation period, should not be a bar to the enforcement of its rights as a
corporation. One of these rights, to be sure, includes the UCCs right to seek
from the court the execution of a valid and final judgment in Civil Case No.
9165 through its trustee/liquidator Encarnacion Gonzales Wong for the
benefit of its stockholders, creditors and any other person who may have
legal claims against it. To hold otherwise would be to allow petitioners to
unjustly enrich themselves at the expense of UCC. This, in effect, renders
nugatory all the efforts and expenses of UCC in its quest to secure justice,
not to mention the undue delay in disposing of this case prejudicial to the
administration of justice.
Next, petitioners aver that the November 8, 1995 second alias writ of
execution, implemented in the June 27, 1997 Order of the trial court, varied
the judgment in Civil Case No. 9165 resulting in the deprivation of their
property without due process.

TOPIC: POWERS OF CORPORATION AT LIQUIDATION


CELIA B. CHUA, et. al vs. NATIONAL LABOR RELATIONS COMMISSION
LABOR ARBITER DOMINADOR M. CRUZ, Public Respondents, and
FERNANDO R. GUMABON, et. al
G.R. Nos. 89971-75 October 17 1990
FACTS:
Stanford Microsystems, Inc. filed a petition for suspension of payments
and appointment of rehabilitation receiver with the SEC, which declared
Stanford to be in a state of suspension of payments. It issued an order
appointing Sycip Gorres & Velayo & Co. as the rehabilitation receiver. In view
of these developments, the former employees of Stanford filed with the
DOLE cases for money claims. The petitioners maintained that the SEC has
jurisdiction over the case and not the NLRC. The seven secured creditor

Page 817 of 1072

banks of Stanford and 6,341 former employees executed a Memorandum of


Agreement to speed up the orderly liquidation of Stanford.
In January, 1987, the SEC disapproved the Rehabilitation Plan
submitted by SGV and dismissed Stanford's Petition for Suspension of
Payments and Appointment of a Rehabilitation Receiver. Subsequently, the
SEC ordered Stanford's liquidation.
ISSUE/S:
Whether the SEC or the NLRC has jurisdiction nad whether or not
the MOA is valid.
RULING:
The SEC has jurisdiction. Jurisdiction over liquidation proceedings of
insolvent corporations is vested in the Securities and Exchange Commission
(SEC) pursuant to Presidential Decree No. 902-A, as amended. On the other
hand, jurisdiction over money claims of employees against their employers is
vested in the Labor Arbiter whose decision may be appealed to the National
Labor Relations Commission (NLRC) pursuant to Article 217 of the Labor
Code.
In this case, however, since a MOA was validly entered into by the
parties to speed up the litigation, the NLRC is thus prohibited from interfering
with the interfering with the implementation of the MOA.
WHEREFORE, the instant petition is GRANTED. The questioned
resolutions dated October 6, 1988, November 3, 1988 and January 3, 1989 of
the National Labor Relations Commission are declared NULL and VOID and
are hereby SET ASIDE.
TOPIC:POWERS OF CORPORATION AT LIQUIDATION
LUIS C. CLEMENTE, LEONOR CLEMENTE DE ELEPAO, HEIRS OF
ARCADIO C. OCHOA, represented by FE O. OCHOA-BAYBAY,
CONCEPCION, MARIANO, ARTEMIO, VICENTE, ANGELITA, ROBERTO,
HERNANDO AND LOURDES, all surnamed ELEPAO, petitioners,
vs.
THE HON. COURT OF APPEALS, ELVIRA PANDINCO-CASTRO AND
VICTOR CASTRO, respondents.
G.R. No. 82407. March 27, 1995
FACTS:
In an action entitled entitled "Declaration of Ownership with
Receivership," instituted before the Regional Trial Court, herein petitioners

Page 818 of 1072

and herein private respondents both claimed ownership over a parcel of land
located in the Bario of Lecheria, Calamba, and Laguna. However, only herein
petitioners who were able to prove their allegations as private respondents
did not present any evidence.
Herein petitioners claims that Sociedad Anonima was organized and
actually did business and held itself out as a corporation from Nov., 1909, up
to September 24, 1932. Its principal business was cockfighting or the
operation and management of a cockpit. During its existence it acquired by
installment the subject parcel of land. Thus, Patent No. 38994 was issued in
its name. Petitioners also claim that Mariano Elepao and Pablo Clemente,
now both deceased, were the original stockholders of the aforesaid
"sociedad." Pablo Clemente's shares of stocks were however later distributed
and apportioned to his heirs in accordance with a Project of Partition. Thus,
"sociedad" issued stock certificates to the aforesaid heirs of Pablo Clemente.
Consequently, herein petitioners claim ownership over the subject property
on the basis that their fathers were the only known stockholders of the
sociedad.
The trial court dismissed the complaint not merely on what it
apparently perceived to be an insufficiency of the evidence that firmly could
establish herein petitioners claim of ownership over the property in dispute
but also on its thesis that, absent a corporate liquidation, it is the
corporation, not the stockholders, which can assert, if at all, any title to the
corporate assets.
Said decision was affirmed by the CA.

ISSUE/S:
Whether or not petitioners can be held, given their submissions,
to have succeeded in establishing for themselves a firm title to
the property in question.
RULING:
Like the courts below, the Supreme Court find petitioners' evidence to
be direly wanting; all that appear to be certain are that the "Sociedad
Popular Calambea," believed to be a "sociedad anonima" and for a while
engaged in the operation and management of a cockpit, has existed
sometime in the past; that it has acquired the parcel of land here involved;
and that the plaintiffs' predecessors, Mariano Elepao and Pablo Clemente,
had been original stockholders of the sociedad. Except in showing that they
are the successors-in-interest of Elepao and Clemente, petitioners have
been unable to come up with any evidence to substantiate their claim of
ownership of the corporate asset.

Page 819 of 1072

If, indeed, the sociedad has long become defunct, it should behoove
petitioners, or anyone else who may have any interest in the corporation, to
take appropriate measures before a proper forum for a peremptory
settlement of its affairs. We might invite attention to the various modes
provided by the Corporation Code (see Sees. 117-122) for dissolving,
liquidating or winding up, and terminating the life of the corporation. Among
the causes for such dissolution are when the corporate term has expired or
when, upon a verified complaint and after notice and hearing, the Securities
and Exchange Commission orders the dissolution of a corporation for its
continuous inactivity for at least five (5) years. The corporation continues to
be a body corporate for three (3) years after its dissolution for purposes of
prosecuting and defending suits by and against it and for enabling it to settle
and close its affairs, culminating in the disposition and distribution of its
remaining assets. It may, during the three-year term, appoint a trustee or a
receiver who may act beyond that period. The termination of the life of a
juridical entity does not by itself cause the extinction or diminution of the
rights and liabilities of such entity (see Gonzales vs. Sugar Regulatory
Administration, 174 SCRA 377) nor those of its owners and creditors. If the
three-year extended life has expired without a trustee or receiver. having
been expressly designated by the corporation within that period, the board of
directors (or trustees) itself, following the rationale of the Supreme Court's
decision in Gelano vs. Court of Appeals (103 SCRA 90) may be permitted to
so continue as "trustees" by legal implication to complete the corporate
liquidation. Still in the absence of a board of directors or trustees, those
having any pecuniary interest in the assets, including not only the
shareholders but likewise the creditors of the corporation, acting for and in
its behalf, might make proper representations with the Securities and
Exchange commission, which has primary and sufficiently broad jurisdiction
in matters of this nature, for working out a final settlement of the corporate
concerns.

TOPIC:POWERS OF CORPORATION AT LIQUIDATION


CARLOS GELANO and GUILLERMINA MENDOZA DE GELANO,
petitioners,
vs.
THE HONORABLE COURT OF APPEALS and INSULAR SAWMILL, INC.,
respondents.
G.R. No. L-39050. February 24, 1981
FACTS:

Page 820 of 1072

Private respondent Insular Sawmill, Inc. is a corporation organized on


September 17, 1945 with a corporate life of 50 years or up to September 17,
1995, with primary purpose of carrying on a general lumber and sawmill
business. To carry on its business, it leased the paraphernal property of
petitioner-wife Guillerma Gelano at the corner of Canonigo and Otis, Paco,
Manila for P1, 200.00 a month. On account of rentals to be paid by the
corporation on the land, petitioner-husband obtained cash advances in the
amount of P25, 000.00 from Insular. Said amount is to be deducted from the
monthly rentals of the leased premises until fully paid. However, only P 5,
590.00 was paid by Carlos leaving an unpaid balance of P 20, 000 which
Guillerma refused to pay on the ground that such amounts were obtained
without her knowledge and consent.
Petitioners also made credit purchases of lumber materials from Insular
with the total price of P 1,120.46 wherein a balance of P 946.46 remained
unpaid. Thereafter, private respondent, through Joseph Tan Yoc Su, executed
a joint and several promissory with Carlos Gelano in favor of China Banking
Corp. in the amount of P 8 000.00. Due to Carlos failure to pay the note
upon maturity, the bank collected the amount of P 9, 106.00 from the
corporations current account with the bank.
Consequently, Insular filed a collection complaint against petitioners.
In the meantime, the corporation amended its Articles of Incorporation
to shorten its term of existence up to December 31, 1960 only. Said
amended Article was filed and approved by the SEC, but the trial court was
not notified of such amendment.
Almost four years thereafter, the trial court rendered a decision in
favor of private respondent.
On appeal by both parties, the CA rendered a decision modifying the
RTC decision by holding petitioners jointly and severally liable on Insulars
claim.
After receipt of the copy of the decision, petitioners came to know that
the corporation was dissolved way back on Dec. 31, 1960. Thus, they filed a
Motion to Dismiss and/ or reconsideration of the CA decision. Said motion
was denied by the CA.
Hence this petition for review.

ISSUE/S:
Whether a corporation, whose corporate life had ceased by the
expiration of its term of existence, could still continue
prosecuting and defending suits after its dissolution and beyond

Page 821 of 1072

the period of three years provided for under Act No. 1459,
otherwise known as the Corporation law, to wind up its affairs,
without having undertaken any step to transfer its assets to a
trustee or assignee.
RULING:
Yes.
The complaint in this case was filed on May 29, 1959 when private
respondent Insular Sawmill, Inc. was still existing. While the case was being
tried, the stockholders amended its Articles of Incorporation by shortening
the term of its existence from December 31, 1995 to December 31, 1960,
which was approved by the Securities and Exchange Commission.
In American corporate law, upon which our Corporation Law was
patterned, it is well settled that, unless the statutes otherwise provide, all
pending suits and actions by and against a corporation are abated by a
dissolution of the corporation. Section 77 of the Corporation Law provides
that the corporation shall "be continued as a body corporate for three (3)
years after the time when it would have been ... dissolved, for the purpose of
prosecuting and defending suits By or against it ...," so that, thereafter, it
shall no longer enjoy corporate existence for such purpose. For this reason,
Section 78 of the same law authorizes the corporation, "at any time during
said three years ... to convey all of its property to trustees for the benefit of
members, Stockholders, creditors and other interested," evidently for the
purpose, among others, of enabling said trustees to prosecute and defend
suits by or against the corporation begun before the expiration of said
period. Commenting on said sections, Justice Fisher said:
It is to be noted that the time during which the corporation,
through its own officers, may conduct the liquidation of its assets
and sue and be sued as a corporation is limited to three years
from the time the period of dissolution commences; but that
there is no time limited within which the trustees must complete
a liquidation placed in their hands. It is provided only (Corp. Law,
Sec. 78) that the conveyance to the trustees must be made
within the three-year period. It may be found impossible to
complete the work of liquidation within the three-year period or
to reduce disputed claims to judgment. The authorities are to the
effect that suits by or against a corporation abate when it ceased
to be an entity capable of suing or being sued (7 R.C.L. Corps.,
Par. 750); but trustees to whom the corporate assets have been
conveyed pursuant to the authority of Section 78 may sue and
be sued as such in all matters connected with the liquidation. By
the terms of the statute the effect of the conveyance is to make
the trustees the legal owners of the property conveyed, subject
to the beneficial interest therein of creditors and stockholders.

Page 822 of 1072

When Insular Sawmill, Inc. was dissolved on December 31, 1960, under
Section 77 of the Corporation Law, it still has the right until December 31,
1963 to prosecute in its name the present case. After the expiration of said
period, the corporation ceased to exist for all purposes and it can no longer
sue or be sued.
However, a corporation that has a pending action and which cannot be
terminated within the three-year period after its dissolution is authorized
under Section 78 to convey all its property to trustees to enable it to
prosecute and defend suits by or against the corporation beyond the Threeyear period although private respondent (did not appoint any trustee, yet the
counsel who prosecuted and defended the interest of the corporation in the
instant case and who in fact appeared in behalf of the corporation may be
considered a trustee of the corporation at least with respect to the matter in
litigation only. Said counsel had been handling the case when the same was
pending before the trial court until it was appealed before the Court of
Appeals and finally to this Court. We therefore hold that there was a
substantial compliance with Section 78 of the Corporation Law and as such,
private respondent Insular Sawmill, Inc. could still continue prosecuting the
present case even beyond the period of three (3) years from the time of its
dissolution.
From the above quoted commentary of Justice Fisher, the trustee may
commence a suit which can proceed to final judgment even beyond the
three-year period. No reason can be conceived why a suit already
commenced By the corporation itself during its existence, not by a mere
trustee who, by fiction, merely continues the legal personality of the
dissolved corporation should not be accorded similar treatment allowed to
proceed to final judgment and execution thereof.
The word "trustee" as used in the corporation statute must be
understood in its general concept which could include the counsel to whom
was entrusted in the instant case, the prosecution of the suit filed by the
corporation. The purpose in the transfer of the assets of the corporation to a
trustee upon its dissolution is more for the protection of its creditor and
stockholders. Debtors like the petitioners herein may not take advantage of
the failure of the corporation to transfer its assets to a trustee, assuming it
has any to transfer which petitioner has failed to show, in the first place. To
sustain petitioners' contention would be to allow them to enrich themselves
at the expense of another, which all enlightened legal systems condemn.

Page 823 of 1072

TOPIC:POWERS OF CORPORATION AT LIQUIDATION


JAMES REBURIANO and URBANO REBURIANO, petitioners,
vs.
HONORABLE COURT OF APPEALS AND PEPSI COLA BOTTLING
COMPANY OF THE PHILIPPINES INC., respondents.
G.R. No. 102965. January 21, 1999
FACTS:
On June 1, 1987, the RTC rendered a decision in civil case No. Q-35598,
entitled "Pepsi Cola Bottling Company of the Philippines Inc. v. Urbano (Ben)
Reburiano and James Reburiano, The RTC ordered herein petitioners to pay
jointly and severally Pepsi Cola the sum of P 55, 000.00 less whatever
empties (cases and bottles) may be returned by said defendants valued at
the rate of P55.00 per empty case with bottles.
Said decision was appealed for modification to the CA by Pepsi Cola.
Thus, on June 26, 1990 the CA set aside the RTC decision and ordered herein
petitioners to pay jointly and severally the plaintiff-appellant the sum of
P55,000.00 with interest at the legal rate from January 1982.
Said decision became final and executor, thus trial court issued a writ
of execution on February 5, 1991.
Meanwhile, prior to the promulgation of the decision of the trial court,
Pepsi Cola amended its Articles of Incorporation to shorten its term of
existence to July 8, 1983. Said amendment was approved by the SEC on
March 2, 1984. The trial court was not notified of this fact.
Consequently, on February 13, 1991, petitioners moved to quash the
writ of execution alleging that when the above- mentioned decisions were
rendered, private respondent Pepsi Cola was no longer in existence and had
no more juridical personality and so, as such, it no longer had the capacity to
sue and be sued. Moreover, as a corporation, lost its existence and juridical
personality, Atty. Romualdo M. Jubay had no more clients in this case and so
his appearance in this case was no longer possible and tenable.
The RTC denied the motion to quash.

ISSUE/S:
Whether or not petitioners are correct in contending that "a
dissolved and non-existing corporation could no longer be
represented by a lawyer and concomitantly a lawyer could not
appear as counsel for a non-existing judicial person.
RULING:

Page 824 of 1072

NO.
The Court thinks that petitioners are in error in contending that "a
dissolved and non-existing corporation could no longer be represented by a
lawyer and concomitantly a lawyer could not appear as counsel for a nonexisting judicial person."
Sec. 122 of the Corporation Code provides in part:
122. Corporate Liquidation. Every Corporation whose
charter expires by its own limitation or is annulled by
forfeiture or otherwise, or whose corporate existence for
other purposes is terminated in any other manner, shall
nevertheless be continued as a body corporate for three
(3) years after the time when it would have been so
dissolved, for the purpose of prosecuting and defending
suits by or against it and enabling it to settle and close its
affairs, to dispose of and convey its property and to
distribute its assets, but not for the purpose of continuing
the business for which it was established.
At any time during said three (3) years, said
corporation is authorized the empowered to convey all of
its property to trustees for the benefit of stockholders,
members, creditors, and other persons in interest. From
and after any such conveyance by the corporation of its
property in trust for the benefit of its stockholders,
members, creditors and others in interests, all interests
which the corporation had in the property in terminates,
the legal interest vests in the trustees, and the beneficial
interest in the stockholders, members, creditors or other
persons in interest.
In Board of Liquidators v. Kalaw, this Court stated:
. . . The legal interest became vested in the trustee
the Board of Liquidators. The beneficial interest
remained with the sole stockholder the government. At
no time had the government withdrawn the property, or
the authority to continue the present suit, from the Board
of Liquidators. If for this reason alone, we cannot stay the
hand of the Board of Liquidators from prosecuting this case
to its final conclusion. The provision of Section 78 (now
Section 122) of the Corporation Law the third method of
winding up corporate affairs finds application.
Indeed, in Gelano vs. Court of Appeals, a case having substantially
similar facts as the instant case, this Court held:

Page 825 of 1072

However, a corporation that has a pending action


and which cannot be terminated within the three-year
period after its dissolution is authorized under Sec. 78 [now
122] of the Corporation Law to convey all its property to
trustees to enable it to prosecute and defend suits by or
against the corporation beyond the three-year period.
Although private respondent did not appoint any trustee,
yet the counsel who prosecuted and defended the interest
of the corporation in the instant case and who in fact
appeared in behalf of the may be considered a trustee of
the corporation at least with respect to the matter in
litigation only. Said counsel had been handling the case
when the same was pending before the trial court until it
was appealed before the Court of Appeals and finally to
this Court. We therefore hold that there was substantial
compliance with Sec. 78 [now 122] of the Corporation Law
and such private respondent Insular Sawmill, Inc. could still
continue prosecuting the present case even beyond the
period of three (3) years from the time of dissolution.
. . . [T]he trustee may commence a suit which can
proceed to final judgment even beyond the three-year
period. No reason can be conceived why a suit already
commenced by the corporation itself during its existence,
not by a mere trustee who, by fiction, merely continues the
legal personality of the dissolved corporation should not be
accorded similar treatment allowed to proceed to final
judgment and execution thereof.
In the Gelano case, the counsel of the dissolved corporation was
considered a trustee. In the later case of Clemente v. Court of Appeals, we
held that the board of directors may be permitted to complete the corporate
liquidation by continuing as "trustees" by legal implication. For, indeed, as
early as 1939, in the case of Sumera v. Valencia, this Court held:
It is to be noted that the time during which the corporation,
through its own officers, may conduct the liquidation of its assets
and sue and be sued as a corporation is limited to three years
from the time the period of dissolution commences: but there is
no time limit within which the trustees must complete a
liquidation placed in their hands. It is provided only (Corp. Law,
Sec. 78 [now Sec. 122]) that the conveyance to the trustees
must be made within the three-year period. It may be found
impossible to complete the work of liquidation within the threeyear period or to reduce disputed claims to judgment. The
authorities are to the effect that suits by or against a corporation
abate when it ceased to be an entity capable of suing or being

Page 826 of 1072

sued (7 R.C.L., Corps., par. 750); but trustees to whom the


corporate assets have been conveyed pursuant to the authority
of Sec. 78 [now Sec. 122] may sue and be sued as such in all
matters connected with the liquidation. . . .
Furthermore, the Corporation Law provides:
145. Amendment or repeal. No right or remedy in favor
of or against any corporation, its stockholders, members,
directors, trustees, or officers, nor any liability incurred by any
such corporation, stockholders, members, directors, trustees, or
officers, shall be removed or impaired either by the subsequent
dissolution of said corporation or by any subsequent amendment
or repeal of this Code or of any part thereof.
This provision safeguards the rights of a corporation which is dissolved
pending litigation.
There is, therefore, no reason why the suit filed by private respondent
should not be allowed to proceed to execution. It is conceded by petitioners
that the judgment against them and in favor of private respondent in C.A.
G.R. No. 16070 had become final and executory. The only reason for their
refusal to execute the same is that there is no existing corporation to which
they are indebted. Such argument is fallacious. As previously mentioned, the
law specifically allows a trustee to manage the affairs of the corporation in
liquidation. Consequently, any supervening fact, such as the dissolution of
the corporation, repeal of a law, or any other fact of similar nature would not
serve as an effective bar to the enforcement of such right.

Page 827 of 1072

TOPIC:POWERS OF CORPORATION AT LIQUIDATION


REPUBLIC PLANTERS BANK, petitioner,
vs.
COURT OF APPEALS and FERMIN CANLAS, respondents.
G.R. No. 93073. December 21, 1992
FACTS:
Private respondent Canlas as treasurer of Worldwide Garment
Manufacturing, Inc., and Shozo Yamaguchi, the companys President/Chief
Operating Officer, by virtue of a Board Resolution applied for credit facilities
with petitioner Bank in the forms of export advances and letters of credit/
trust receipts accommodations. Said transactions were evidenced by 9
promissory notes signed by Canlas and Yamaguchi.
Meanwhile, Worldwide Garment Manufacturing, Inc. noted to change its
corporate name to Pinch Manufacturing Corporation.
Subsequently, petitioner filed a complaint for the recovery of sums of
money covered by the 9 promissory notes with interest thereon. The
complaint was originally brought against Worldwide but late on amended to
drop Worldwide as defendant and substitute Pinch Manufacturing in its place.
Only Fermin Canlas filed an amended answer denying having issued
the promissory notes in question since according to him, he was not an
officer of Pinch Manufacturing Corporation, but instead of Worldwide
Garment Manufacturing, Inc.,
The RTC rendered a decision in favor of petitioner, ordering Pinch,
Yamaguchi and Canlas to pay, jointly and severally petitioner.
The CA affirmed the decision of the RTC but absolved Canlas from
liability.
ISSUE/S:
Whether an amendment in a corporation's Articles of
Incorporation effecting a change of corporate name extinguished
the personality of the original corporation.
RULING:
No.
The respondent Court made a grave error in holding that an
amendment in a corporation's Articles of Incorporation effecting a change of

Page 828 of 1072

corporate name, in this case from Worldwide Garment manufacturing Inc to


Pinch Manufacturing Corporation extinguished the personality of the original
corporation.
The corporation, upon such change in its name, is in no sense a new
corporation, nor the successor of the original corporation. It is the same
corporation with a different name, and its character is in no respect changed.
A change in the corporate name does not make a new corporation, and
whether effected by special act or under a general law, has no affect on the
identity of the corporation, or on its property, rights, or liabilities.
The corporation continues, as before, responsible in its new name for
all debts or other liabilities which it had previously contracted or incurred.
As a general rule, officers or directors under the old corporate name
bear no personal liability for acts done or contracts entered into by officers of
the corporation, if duly authorized. Inasmuch as such officers acted in their
capacity as agent of the old corporation and the change of name meant only
the continuation of the old juridical entity, the corporation bearing the same
name is still bound by the acts of its agents if authorized by the Board.

TOPIC: FOREIGN CORPORATIONS: DEFINITIONS & RIGHTS

Page 829 of 1072

AVON INSURANCE PLC. BRITISH RESERVE INSURANCE CO. LTD., et al


petitioners,
vs.
COURT OF APPEALS, REGIONAL TRIAL COURT OF MANILA, BRANCH
51. YUPANGCO COTTON MILLS. WORLDWIDE SURETY & INSURANCE
CO., INC., respondents.
G.R. NO. 97642, AUGUST 29, 1997
FACTS:
Petitioners claim that the trial court's jurisdiction does not extend to
them, since they are foreign reinsurance companies that are not doing
business in the Philippines. Having entered into reinsurance contracts
abroad, petitioners are beyond the jurisdictional ambit of our courts and
cannot be served summons through extraterritorial service, as under Section
17, Rule 14 of the Rules of Court, nor through the Insurance Commissioner,
under Section 14. Private respondent Yupangco Cotton Mills contend on the
other hand that petitioners are within our courts' cognitive powers, having
submitted voluntarily to their jurisdiction by filing motions to dismiss.
ISSUE/S:
Whether or not the suit over the foreign corporation will prosper.
RULING:
NO.
There is no sufficient basis in the records which would merit the
institution of this collection suit in the Philippines. There is nothing to
substantiate the private respondent's submission that the petitioners had
engaged in business activities in this country. This is not an instance where
the erroneous service of summons upon the defendant can be cured by the
issuance and service of alias summons, as in the absence of showing that
petitioners had been doing business in the country, they cannot be
summoned to answer for the charges leveled against them.
Private respondent has made no allegation or demonstration of the
existence of petitioners' domestic agent, but avers simply that they are
doing business not only abroad but in the Philippines as well. The
reinsurance treaties between the petitioners and Worldwide Surety and
Insurance were made through an international insurance broker, and not
through any entity or means remotely connected with the Philippines.
Moreover, there is authority to the effect that a reinsurance company is not
doing business in a certain state merely because the property or lives which
are insured by the original insurer company are located in that state. The
reason for this is that a contract of reinsurance is generally a separate and

Page 830 of 1072

distinct arrangement from the original contract of insurance, whose


contracted risk is insured in the reinsurance agreement. Hence, the original
insured has generally no interest in the contract of reinsurance.
TOPIC: FOREIGN CORPORATIONS: DEFINITIONS & RIGHTS
PEDRO R. PALTING, petitioner,
vs.
SAN JOSE PETROLEUM INCORPORATED, respondent.
G.R. No. L-14441. December 17, 1966
FACTS:
SAN JOSE OIL, is a domestic mining corporation, 90% of the
outstanding capital stock of which is owned by respondent SAN JOSE
PETROLEUM, a foreign (Panamanian) corporation, the majority interest of
which is owned by OIL INVESTMENTS, INC., another foreign (Panamanian)
company. This latter corporation in turn is wholly (100%) owned by
PANTEPEC OIL COMPANY, C. A., and PANCOASTAL PETROLEUM COMPANY, C.
A., both organized and existing under the laws of Venezuela. As of
September 30, 1956, there were 9,979 stockholders of PANCOASTAL
PETROLEUM found in 49 American states and U.S. territories, holding
3,476,988 shares of stock; whereas, as of November 30, 1956, PANTEPEC OIL
COMPANY was said to have 3,077,916 shares held by 12,373 stockholders
scattered in 49 American states. In the two lists of stockholders, there is no
indication of the citizenship of these stockholders, or of the total number of
authorized stocks of each corporation for the purpose of determining the
corresponding percentage of these listed stockholders in relation to the
respective capital stock of said corporation.

ISSUE/S:
Whether or not the "tie-up" between the respondent SAN JOSE
PETROLEUM, a foreign corporation, and SAN JOSE OIL COMPANY,
INC., a domestic mining corporation, is violative of the
Constitution, the Laurel-Langley Agreement, the Petroleum Act of
1949, and the Corporation Law
RULING:
Yes.
The privilege to utilize, exploit, and develop the natural resources of
this country was granted, by Article III of the Constitution, to Filipino citizens
or to corporations or associations 60% of the capital of which is owned by
such citizens. With the Parity Amendment to the Constitution, the same right

Page 831 of 1072

was extended to citizens of the United States and business enterprises


owned or controlled, directly or indirectly, by citizens of the United States.
There could be no serious doubt as to the meaning of the word
"citizens" used in the aforementioned provisions of the Constitution. The
right was granted to 2 types of persons: natural persons (Filipino or American
citizens) and juridical persons (corporations 60% of which capital is owned by
Filipinos and business enterprises owned or controlled directly or indirectly,
by citizens of the United States). In American law, "citizen" has been defined
as "one who, under the constitution and laws of the United States, has a
right to vote for representatives in congress and other public officers, and
who is qualified to fill offices in the gift of the people."
These concepts clarified, is herein respondent SAN JOSE PETROLEUM
an American business enterprise entitled to parity rights in the Philippines?
The answer must be in the negative for the following reasons:
Firstly It is not owned or controlled directly by citizens of the United
States, because it is owned and controlled by a corporation, the OIL
INVESTMENTS, another foreign (Panamanian) corporation.
Secondly Neither can it be said that it is indirectly owned and
controlled by American citizens through the OIL INVESTMENTS, for this latter
corporation is in turn owned and controlled, not by citizens of the United
States, but still by two foreign (Venezuelan) corporations, the PANTEPEC OIL
COMPANY and PANCOASTAL PETROLEUM.
Thirdly Although it is claimed that these two last corporations are
owned and controlled respectively by 12,373 and 9,979 stockholders residing
in the different American states, there is no showing in the certification
furnished by respondent that the stockholders of PANCOASTAL or those of
them holding the controlling stock, are citizens of the United States.

Page 832 of 1072

TOPIC:REQUIREMENTS FOR THE ESTABLISHMENT OF A BRANCH/TO DO


BUSINESS IN THE
PHILS - DOCUMENTARY
GEORG GROTJAHN GMBH & CO., petitioner,
vs.
HON. LUCIA VIOLAGO ISNANI, Presiding Judge, RTC, Makati, Br. 59;
ROMANA R. LANCHINEBRE; and TEOFILO A.
LANCHINEBRE, respondents.
G.R. NO. 109272 AUGUST 10, 1994
235 SCRA 216
FACTS:
Petitioner is a multinational company organized and existing under the
laws of the Federal Republic of Germany. It filed with the SEC for the
establishment of a regional or area headquarters in the Philippines, pursuant
to Presidential Decree No. 218, which was approved by the Board of
Investments (BOI).
Private respondent Lanchinebre was a sales representative of
petitioner from 1983 to mid-1992. she secured a loan and additional cash
advances which she failed to settle. Thus, was sued for collection of sum of
money and damages.
The Certificate of Registration and License issued to the petitioner was
merely "for the establishment of a regional or area headquarters in the
Philippines and does not include a license to do business in the Philippines.
There is no allegation in the complaint moreover that it is suing under an
isolated transaction. There is no averment in the complaint regarding
petitioner's capacity to sue or be sued.
ISSUE/S:
Whether or not petitioner, a foreign corporation, has capacity to
sue.
RULING:
YES.
There is no general rule or governing principle as to what constitutes
"doing" or "engaging in" or "transacting" business in the Philippines. Each
case must be judged in the light of its peculiar circumstances. In the case at
bench, petitioner does not engage in commercial dealings or activities in the
country because it is precluded from doing so by P.D. No. 218, under which it
was established. Nonetheless, it has been continuously, since 1983, acting
as a supervision, communications and coordination center for its home
office's affiliates in Singapore, and in the process has named its local agent

Page 833 of 1072

and has employed Philippine nationals like private respondent Romana


Lanchinebre. From this uninterrupted performance by petitioner of acts
pursuant to its primary purposes and functions as a regional/area
headquarters for its home office, it is clear that petitioner is doing business
in the country. Moreover, private respondents are estopped from assailing
the personality of petitioner. The rule is that a party is estopped to challenge
the personality of a corporation after having acknowledged the same by
entering into a contract with it.

Page 834 of 1072

TOPIC:REQUIREMENTS FOR THE ESTABLISHMENT OF A BRANCH/TO DO


BUSINESS IN THE
PHILS - APPOINTMENT OF A RESIDENT AGENT
NEW YORK MARINE MANAGERS, INC., petitioner,
vs.
COURT OF APPEALS and VLASONS SHIPPPING INC., respondents.
G.R. No. 111837, October 24, 1995
247 SCRA 417
FACTS:
Petitioner is a foreign corporation organized under the laws of the
United States. On 25 July 1990 American Natural Soda Ash Corporation
(ANSAC) loaded in Portland, U.S.A., a shipment of soda ash on board the
vessel "MS Abu Hanna" for delivery to Manila. The supplier/shipper insured
the shipment with petitioner. Upon arrival in Manila the shipment was
unloaded and transferred to the vessel "MV Biyayang Ginto" owned by
private respondent. Since the shipment allegedly sustained wettage,
hardening and contamination, it was rejected as total loss by the consignees.
When the supplier sought to recover the value of the cargo loss from
petitioner the latter paid the claim in the amount of US$58,323.96.
Thus petitioner as subrogee filed a complaint for damages against
private respondent alleging among others that plaintiff is a non-life foreign
insurance corporation organized under the laws of the State of New York with
offices at 123 William Street, New York, N.Y. 10038 and engaged in an
isolated transaction in this case.
ISSUE/S:
Whether or not the complaint is fatally defective for failure to
allege the duly authorized representative or resident agent of
petitioner in the Philippines.
RULING:
YES.
Petitioner's complaint is fatally defective for failing to allege its duly
authorized representative or resident agent in this jurisdiction. The pleadings
filed by counsel for petitioner do not suffice. True, a lawyer is generally
presumed to be properly authorized to represent any cause in which he
appears, and no written power of attorney is required to authorize him to
appear in court for his client. But this presumption is disputable. Where said
authority has been challenged or attacked by the adverse party the lawyer is
required to show proof of such authority or representation in order to bind his

Page 835 of 1072

client. The requirement of the production of authority is essential because


the client will be bound by his acquiescence resulting from his knowledge
that he was being represented by said attorney. In the instant case, the
extent of authority of counsel for petitioner has been expressly and
continuously assailed but he has failed to show competent proof that he was
indeed duly authorized to represent petitioner.

TOPIC: AMENDMENT OF LICENSE

Page 836 of 1072

AETNA CASUALTY & SURETY COMPANY, plaintiff-appellant,


vs.
PACIFIC STAR LINE, THE BRADMAN CO. INC., MANILA PORT SERVICE
and/or MANILA RAILROAD COMPANY, INC., defendants-appellees.
G.R. No. L-26809. December 29, 1977
FACTS:
On February 11, 1963, Smith Bell & Co. (Philippines), Inc. and Aetna
Surety Casualty & Surety Co. Inc., as subrogee, instituted Civil Case No.
53074 in the Court of First Instance of Manila against Pacific Star Line, The
Bradman Co. Inc., Manila Port Service and/or Manila Railroad Company, Inc.
to recover the amount of US $2,300.00 representing the value of the stolen
and damaged cargo plus litigation expenses and exemplary damages in the
amounts of P1,000.00 and P2,000.00, respectively, with legal interest
thereon from the filing of the suit and costs.
Defendant Pacific Star Line, as a common carrier, was operating the
vessel SS Ampal on a commercial run between United States and Philippine
Ports including Manila; that the defendant, The Bradman Co. Inc., was the
ship agent in the Philippines for the SS Ampal and/or Pacific Star Line; that
the Manila Railroad Co. Inc. and Manila Port Service were the arrastre
operators in the port of Manila and were authorized to delivery cargoes
discharged into their custody on presentation of release papers from the
Bureau of Customs and the steamship carrier and/or its agents; that on
December 2, 1961, the SS Ampal took on board at New York, N.Y., U.S.A., a
consignment or cargo including 33 packages of Linen & Cotton Piece Goods
for shipment to Manila for which defendant Pacific Star Line issued Bill of
Lading No. 18 in the name of I. Shalom & Co., Inc., as shipper, consigned to
the order of Judy Philippines, Inc., Manila; that the SS Ampal arrived in Manila
on February 10, 1962 and in due course, discharged her cargo into the
custody of Manila Port Service; that due to the negligence of the defendants,
the shipment sustained damages valued at US $2,300.00 representing
pilferage and seawater damage; that I. Shalom & Co., Inc. immediately filed
claim for the undelivered land damaged cargo with defendant Pacific Star
Line in New York, N.Y., but said defendant refused and still refuses to pay the
said claim; that the cargo was insured by I. Shalom & Co., Inc. with plaintiff
Aetna Casualty & Surety Company for loss and/or damage; that upon
demand, plaintiff Aetna Casualty & Surety Company indemnified I. Shalom &
Co., Inc. the amount of US $2,300.00; that in addition to this, the plaintiffs
had obligated themselves to pay attorney's fees and they further anticipated
incurring litigation expenses which may be assessed at P1,000.00; that
plaintiffs and/or their predecessor-in-interest sustained losses due to the
negligence of Pacific Star Line prior to delivery of the cargo to Manila or, in
the alternative, due to the negligence of Manila Port Service after delivery of
the cargo to it by the SS Ampal; that despite repeated demands, none of the

Page 837 of 1072

defendants has been willing to accept liability for the claim of the plaintiffs
and/or I. Shalom & Co., Inc.; and that by reason of defendants' evident bad
faith, they should consequently be liable to pay exemplary damages in the
amount of P2,000.00.
ISSUE/S:
Whether or notAetna Casualty & Surety Company must have
amended its license to transact business of insurance in the
Philippines.
RULING:
It cannot be said that the Aetna Casualty & Surety Company is
transacting business of insurance in the Philippines for which it must have a
license. The contract of insurance was entered into in New York, U.S.A., and
payment was made to the consignee in its New York branch. It appears from
the list of cases issued by the Clerk of Court of the Court of First Instance of
Manila that all the actions, except two (2) cases filed by Smith, Bell & Co.,
Inc. against the Aetna Casualty & Surety Company, are claims against the
shipper and the arrastre operators just like the case at bar.
Consequently, since the appellant Aetna Casualty & Surety Company is
not engaged in the business of insurance in the Philippines but is merely
collecting a claim assigned to it by the consignee, it is not barred from filing
the instant case although it has not secured a license to transact insurance
business in the Philippines.

Page 838 of 1072

TOPIC: AMENDMENT OF LICENSE


HATHIBHAI BULAKHIDAS, petitioner,
vs.
THE HONORABLE PEDRO L. NAVARRO, as Presiding Judge of the
Court of First Instance of Rizal, Seventh Judicial District, Pasig,
Metro Manila, Branch 11 and DIAMOND SHIPPING CORPORATION,
respondent.
G.R. No. L-49695. April 7, 1986
FACTS:
Petitioner, a foreign partnership, filed a complaint against a domestic
corporation, Diamond Shipping Corporation, before the Court of First Instance
of Rizal for the recovery of damages allegedly caused by the failure of the
said shipping corporation to deliver the goods shipped to it by petitioner to
their proper destination. Paragraph 1 of said complaint alleged that plaintiff
is "a foreign partnership firm not doing business in the Philippines" and that
it is "suing under an isolated transaction." Defendant filed a motion to
dismiss the complaint on the ground that plaintiff has no capacity to sue and
that the complaint does not state a valid cause of action against defendant.
Acting on said motion to dismiss, the CFI dismissed the complaint on
the ground that plaintiff being "a foreign corporation or partnership not doing
business in the Philippines it cannot exercise the right to maintain suits
before our Courts."
Hence, this petition.
ISSUE/S:
Whether a foreign corporation not engaged in business in the
Philippines can institute an action before Philippine courts.
RULING:
In Aetna Casualty and Surety Co. vs. Pacific Star Lines, 80
SCRA 635, a case similar to the case at bar, the SC said:
It is settled that if a foreign corporation is not engaged in
business in the Philippines, it may not be denied the right
to file an action in Philippine courts for isolated
transactions. The object of Sections 68 and 69 of the
Corporation law was not to prevent the foreign corporation
from performing single acts, but to prevent it from
acquiring a domicile for the purpose of business without

Page 839 of 1072

taking the steps necessary to render it amenable to suit in


the local courts. It was never the purpose of the Legislature
to exclude a foreign corporation which happens to obtain
an isolated order for business from the Philippines, from
securing redress in the Philippine courts.
In Mentholatum Co. Inc. et al. vs. Mangaliman, et al., this Court
ruled that:
No general rule or governing principle can be laid down as
to what constitutes 'doing' or 'engaging' in or 'transacting'
business. Indeed, each case must be judged in the light of
its peculiar environmental circumstances. The true test,
however, seems to be whether the foreign corporation is
continuing the body or substance of the business or
enterprise for which it was organized or whether it has
substantially retired from it and turned it over to another.
The term implies a continuity of commercial dealings and
arrangements, and contemplates, to that extent, the
performance of acts or works or the exercise of some of the
functions normally incident to, and in progressive
prosecution of, the purpose and object of its organization.
And in Eastboard Navigation, Ltd. et al vs. Juan Ysmael & Co.,
Inc., this Court held that:
(d) While plaintiff is a foreign corporation without license to
transact business in the Philippines, it does not follow that
it has no capacity to bring the present action. Such license
is not necessary because it is not engaged in business in
the Philippines. In fact, the transaction herein involved is
the first business undertaken by plaintiff in the Philippines,
although on a previous occasion plaintiff's vessel was
chartered by the National Rice and Corn Corporation to
carry rice cargo from abroad to the Philippines. These two
isolated transactions do not constitute engaging in
business in the Philippines within the purview of Sections
68 and 69 of the Corporation Law so as to bar plaintiff from
seeking redress in our courts.
Again, in Facilities Management Corporation vs. De la Osa 89
SCRA 131, 139, following Aetna Casualty & Surety Co. vs. Pacific Star Line,
supra, held a foreign corporation not engaged in business in the Philippines
is not barred from seeking redress from the courts of the Philippines.
Thus, the case of Atlantic Mutual Insurance Co. vs. Cebu
Stevedoring Co., 17 SCRA 1037, cited by respondent finds no
application to the case at bar. It must be observed in the Atlantic case

Page 840 of 1072

that there was no allegation in the complaint that the two foreign
corporations involved therein were not engaged in business in the
Philippines. All that was averred in the complaint was that they were both
foreign corporations existing under the laws of the United States. Thus, the
qualifying circumstance of the said foreign corporations' capacity to sue is
wanting. Contrary to the Atlantic case, the complaint filed by petitioner
herein sufficiently alleged that it is a foreign partnership (or corporation) not
engaged in business in the Philippines and that it was suing under an
isolated transaction.

Page 841 of 1072

TOPIC: AMENDMENT OF LICENSE


SCHMID & OBERLY, INC., petitioner,
vs.
RJL MARTINEZ FISHING CORPORATION, respondent.
G.R. No. 75198. October 18, 1988
FACTS:
RJL MARTINEZ is engaged in the business of deep-sea fishing. As RJL
MARTINEZ needed electric generators for some of its boats and SCHMIID sold
electric generators of different brands, negotiations between them for the
acquisition thereof took place. The parties had two separate transactions
over "Nagata"-brand generators.The first transaction was the sale of three
(3) generators. In this transaction, it is not disputed that SCHMID was the
vendor of the generators. The company supplied the generators from its
stockroom; it was also SCHMID which invoiced the sale.
The second transaction, which gave rise to the present controversy,
involves twelve (12) "Nagata"-brand generators. As RJL MARTINEZ was
canvassing for generators, SC gave RJL MARTINEZ its Quotation dated August
19, 1975 [Exhibit 'A"] for twelve (12) "Nagata'-brand generators with the
specifications "NAGATA" Single phase AC Alternators, 110/220 V, 60 cycles,
1800 rpm, unity power factor, rectifier type and radio suppressor,, 5KVA
(5KW) $546.75.
It was stipulated that payment would be made by confirming an
irrevocable letter of credit in favor of NAGATA CO. Furthermore, among the
General Conditions of Sale appearing on the dorsal side of the Quotation.
Agreeing with the terms of the Quotation, RJL MARTINEZ opened a letter of
credit in favor of NAGATA CO. Accordingly, on November 20,1975, SCHMID
transmitted to NAGATA CO. an order for the twelve (12) generators to be
shipped directly to RJL MARTINEZ. NAGATA CO. thereafter sent RJL MARTINEZ
the bill of lading and its own invoice and, in accordance with the order,
shipped the generators directly to RJL MARTINEZ. The invoice states that
"one (1) case of 'NAGATA' AC Generators" consisting of twelve sets was
bought by order and for account risk of Messrs. RJL Martinez Fishing
Corporation.
For its efforts, SCHMID received from NAGATA CO. a commission of
$1,752.00 for the sale of the twelve generators to RJL MARTINEZ. All fifteen
(15) generators subject of the two transactions burned out after continuous
use. RJL MARTINEZ informed SCHMID about this development. In turn,
SCHMID brought the matter to the attention of NAGATA CO. In July 1976,
NAGATA CO. sent two technical representatives who made an ocular

Page 842 of 1072

inspection and conducted tests on some of the burned out generators, which
by then had been delivered to the premises of SCHMID.
The tests revealed that the generators were overrated. As indicated
both in the quotation and in the invoice, the capacity of a generator was
supposed to be 5 KVA (kilovolt amperes). However, it turned out that the
actual capacity was only 4 KVA. SCHMID replaced the three (3) generators
subject of the first sale with generators of a different brand.
As for the twelve (12) generators subject of the second transaction, the
Japanese technicians advised RJL MARTINEZ to ship three (3) generators to
Japan, which the company did. These three (3) generators were repaired by
NAGATA CO. itself and thereafter returned to RJL MARTINEZ; the remaining
nine (9) were neither repaired nor replaced. NAGATA CO., however, wrote
SCHMID suggesting that the latter check the generators, request for spare
parts for replacement free of charge, and send to NAGATA CO. SCHMID's
warranty claim including the labor cost for repairs. In its reply letter, SCHMID
indicated that it was not agreeable to these terms.
As not all of the generators were replaced or repaired, RJL MARTINEZ
formally demanded that it be refunded the cost of the generators and paid
damages. SCHMID in its reply maintained that it was not the seller of the
twelve (12) generators and thus refused to refund the purchase price
therefore. Hence, RJL MARTINEZ brought suit against SCHMID on the theory
that the latter was the vendor of the twelve (12) generators and, as such
vendor, was liable under its warranty against hidden defects.
ISSUE/S:
Whether the transaction between the parties was a sale or an
indent transaction. SCHMID maintains that it was the latter; RJL
MARTINEZ claims that it was a sale.
RULING:
There is no statutory definition of "indent" in this jurisdiction. However,
the Rules and Regulations to Implement Presidential Decree No. 1789 (the
Omnibus Investments Code) lumps "indentors" together with "commercial
brokers" and "commission merchants" in this manner:
... A foreign firm which does business through the middlemen
acting in their own names, such as indentors, commercial
brokers or commission merchants, shall not be deemed doing
business in the Philippines . But such indentors, commercial
brokers or commission merchants shall be the ones deemed to
be doing business in the Philippines [Part I, Rule I, Section 1, par.
g (1).]
Therefore, an indentor is a middlemen in the same class as commercial
brokers and commission merchants. Webster defines an indent as "a
purchase order for goods especially when sent from a foreign country." It

Page 843 of 1072

would appear that there are three parties to an indent transaction, namely,
the buyer, the indentor, and the supplier who is usually a non-resident
manufacturer residing in the country where the goods are to be bought
[Commissioner of Internal Revenue v. Cadwallader Pacific Company, G.R. No.
L-20343, September 29, 1976, 73 SCRA 59.] An indentor may therefore be
best described as one who, for compensation, acts as a middleman in
bringing about a purchase and sale of goods between a foreign supplier and
a local purchaser. In the case at bar, the admissions of the parties and the
facts appearing on record more than suffice to warrant the conclusion that
SCHMID was not a vendor, but was merely an indentor, in the questioned
transaction.
It is argued that if SCHMID is considered as a mere agent of NAGATA
CO., a foreign corporation not licensed to do business in the Philippines ,
then the officers and employees of the former may be penalized for violation
of the old Corporation Law which provided:
Sec. 69 ... Any officer or agent of the corporation or any person transacting
business for any foreign corporation not having the license prescribed shall
be punished by imprisonment for not less than six months nor more than two
years or by a fine 'of not less than two hundred pesos nor more than one
thousand pesos or both such imprisonment and fine, in the discretion of the
Court.
The afore-quoted penal provision in the Corporation Law finds no
application to SCHMID and its officers and employees relative to the
transactions in the instant case. What the law seeks to prevent, through said
provision, is the circumvention by foreign corporations of licensing
requirements through the device of employing local representatives. An
indentor, acting in his own name, is not, however, covered by the abovequoted provision. In fact, the provision of the Rules and Regulations
implementing the Omnibus Investments Code quoted above, which was
copied from the Rules implementing Republic Act No. 5455, recognizes the
distinct role of an indentor, such that when a foreign corporation does
business through such indentor, the foreign corporation is not deemed doing
business in the Philippines.
Not being the vendor, SCHMID cannot be held liable for the implied
warranty for hidden defects under the Civil Code [Art. 1561, et seq.]

Page 844 of 1072

TOPIC:DOING BUSINESS WITH/WITHOUT LICENSE:SUITS BY OR AGAINST


FOREIGN
CORPORATION
SEHWANI, INCORPORATED and/or BENITAS FRITES, INC., Petitioner,
vs.
IN-N-OUT BURGER, INC., Respondent.
G.R. No. 171053. October 15, 2007
536 SCRA 225
FACTS:
Respondent IN-N-OUT Burger, Inc., a foreign corporation organized
under the laws of California, U.S.A., and not doing business in the Philippines,
filed before the Bureau of Legal Affairs of the IPO (BLA-IPO), an
administrative complaint against petitioners Sehwani, Inc. and Benitas
Frites, Inc. for violation of intellectual property rights, attorneys fees and
damages with prayer for the issuance of a restraining order or writ of
preliminary injunction.4
Respondent alleged that it is the owner of the tradename "IN-N-OUT" and
trademarks "IN-N-OUT," "IN-N-OUT Burger & Arrow Design" and "IN-N-OUT
Burger Logo," which are used in its business since 1948 up to the present.
These tradename and trademarks were registered in the United States as
well as in other parts of the world.5
On June 2, 1997, respondent applied with the IPO for the registration of its
trademark "IN-N-OUT Burger & Arrow Design" and servicemark "IN-N-OUT." In
the course of its application, respondent discovered that petitioner Sehwani,
Inc. had obtained Trademark Registration No. 56666 for the mark "IN N OUT"
(THE INSIDE OF THE LETTER "O" FORMED LIKE A STAR) on December 17,
1993 without its authority.6 Respondent thus demanded that petitioner
Sehwani, Inc. desist from claiming ownership of the mark "IN-N-OUT" and to
voluntarily cancel its Trademark Registration No. 56666. Petitioner Sehwani,
Inc. however refused to accede to the demand and even entered into a
Licensing Agreement granting its co-petitioner Benitas Frites, Inc. license to
use for a period of five years the trademark "IN-N-OUT BURGER" in its
restaurant in Pasig City.7 Hence, respondent filed a complaint for violation of
intellectual property rights.
ISSUE/S:
Whether respondent can established business in the Philippines.

Page 845 of 1072

RULING:
The fact that respondents marks are neither registered nor used in the
Philippines is of no moment. The scope of protection initially afforded by
Article 6bis of the Paris Convention has been expanded in the 1999 Joint
Recommendation Concerning Provisions on the Protection of Well-Known
Marks, wherein the World Intellectual Property Organization (WIPO) General
Assembly and the Paris Union agreed to a nonbinding recommendation that
a well-known mark should be protected in a country even if the mark is
neither registered nor used in that country. Part I, Article 2(3) thereof
provides:
(3) [Factors Which Shall Not Be Required]
(a) A Member State shall not require, as a condition for
determining whether a mark is a well-known mark:
i.

ii.

iii.

that the mark has been used in, or that the


mark has been registered or that an application
for registration of the mark has been filed in or
in respect of, the Member State;
that the mark is well known in, or that the mark
has been registered or that an application for
registration of the mark has been filed in or in
respect of, any jurisdiction other than the
Member State; or
that the mark is well known by the public at
large in the Member State. (Underscoring
supplied)

Moreover, petitioners claim that no ground exists for the cancellation


of their registration lacks merit.The evidence on record shows that not only
did the petitioners use the IN-N-OUT Burger trademark for the name of their
restaurant, but they also used identical or confusingly similar mark for their
hamburger wrappers and french-fries receptacles, thereby effectively
misrepresenting the source of the goods and services.

Page 846 of 1072

TOPIC:DOING BUSINESS WITH/WITHOUT LICENSE:SUITS BY OR AGAINST


FOREIGN
CORPORATION
LORENZO SHIPPING CORP., petitioner,
vs.
CHUBB and SONS, Inc., GEARBULK, Ltd. and PHILIPPINE
TRANSMARINE CARRIERS, INC., respondents.
G.R. No. 147724. June 8, 2004
431 SCRA 266
FACTS:
Lorenzo Shipping Corporation is a domestic corporation engaged in
coastwise shipping. Gearbulk ltd. is a foreign corporation licensed as a
common carrier under the laws of Norway and doing business in the
Philippines through its agent Philippine Transmarine Carrier Inc. Philippine
Transmarine Carrier (PTC ) is the agent of Gearbulk in the Philippines .
Sumitomo Corporation (SC), a foreign corporation organized under the laws
of the USA is the consignee. It insured the shipment with Chubb and Sons,
Inc., a foreign corporation organized and licensed to engage in insurance
business under the laws of the United States of America .
On November 21, 1987, Mayer Steel Pipe Corporation of Binondo,
Manila, loaded 581 bundles of black steel on board one of the vessels of LS
for shipment to Davao City from Manila. Upon reaching the Sasa Wharf in
Davao on Dec.2, PTC discovered that the steels were submerged in seawater.
An inspector made a finding that the steel pipes were no longer in good
condition for rust were already forming. Gearbulk noted the damage and
shipped the pipes on its vessel to the USA . Due to its heavily rusted
condition, the consignee Sumitomo rejected the damaged steel pipes and
declared them unfit for the purpose they were intended. It then filed a
marine insurance claim with respondent Chubb and Sons, Inc.
On December 2, 1988, Chubb and Sons, Inc. filed a complaint for
collection of a sum of money against respondents Lorenzo Shipping,
Gearbulk, and Transmarine alleging that it is not doing business in the
Philippines , and that it is suing under an isolated transaction. Gearbulk filed
a cross-claim against LS and a MD against Chubb stating that it has no
capacity to sue before Philippine Courts and it should be dismissed on forum
non conveniens. The RTC ruled in favor of Chubb and found that it has the

Page 847 of 1072

right to institute the action and Lorenzo to be negligent. The CA affirmed the
decision of the RTC.
ISSUE/S:
Whether or not Chubb and Sons, as a mere assignee of a foreign
corporation which has no authority to sue in the Philippines , has
capacity to sue before the Philippine courts
RULING:
Subrogration contemplates full substitution such that it places the
party subrogated in the shoes of the creditor, and he may use all means
which the creditor could employ to enforce payment. The rights to which the
subrogee succeeds are the same as, but not greater than, those of the
person for whom he is substituted.
The law on corporations is clear in depriving foreign corporations which
are doing business in the Philippines without a license from bringing or
maintaining actions before, or intervening in Philippine courts but a foreign
corporation needs no license to sue before Philippine courts on an isolated
transaction. The Supreme Court rejected the claim of petitioner Lorenzo
Shipping that respondent Chubb and Sons is not suing under an isolated
transaction because the steel pipes, subject of this case, are covered by two
(2) bills of lading; hence, two transactions. The stubborn fact remains that
these two (2) bills of lading spawned from the single marine insurance policy
that respondent Chubb and Sons issued in favor of the consignee Sumitomo,
covering the damaged steel pipes. The execution of the policy is a single
act, an isolated transaction. For "doing business" is not really the number or
the quantity of the transactions, but more importantly, the intention of an
entity to continue the body of its business in the country. The phrase
"isolated transaction" has a definite and fixed meaning, i.e. a transaction or
series of transactions set apart from the common business of a foreign
enterprise in the sense that there is no intention to engage in a progressive
pursuit of the purpose and object of the business organization. Whether a
foreign corporation is "doing business" does not necessarily depend upon the
frequency of its transactions, but more upon the nature and character of the
transactions.
Furthermore, respondent insurer Chubb and Sons, by virtue of the right
of subrogation provided for in the policy of insurance, is the real party in
interest in the action for damages before the court a quo against the carrier
Lorenzo Shipping to recover for the loss sustained by its insured. It then,
thus possesses the right to enforce the claim and the significant interest in
the litigation. In the case at bar, it is clear that respondent insurer was suing
on its own behalf in order to enforce its right of subrogation.

Page 848 of 1072

TOPIC:DOING BUSINESS WITH/WITHOUT LICENSE:SUITS BY OR AGAINST


FOREIGN
CORPORATION
MR HOLDINGS, LTD.
VS.
SHERIFF CARLOS P. BAJAR,
SHERIFF FERDINAND M. JANDUSAY, SOLIDBANK CORPORATION, AND
MARCOPPER MINING CORPORATION
G.R. No. 138104.April 11, 2002
FACTS:
Under a "Principal Loan Agreement" and "Complementary Loan
Agreement," ADB agreed to Marcopper a loan in the aggregate amount of
US$40,000,000.00 to finance the latters mining project at Sta. Cruz,
Marinduque. The principal loan of US$ 15,000,000.00 was sourced from
ADBs ordinary capital resources, while the complementary loan of US$
25,000,000.00 was funded by the Bank of Nova Scotia, a participating
finance institution.
On even date, ADB and Placer Dome, a foreign corporation which owns
40% of Marcopper, executed a "Support and Standby Credit Agreement"
whereby the latter agreed to provide Marcopper with cash flow support for
the payment of its obligations to ADB. To secure the loan, Marcopper
executed in favor of ADB a "Deed of Real Estate and Chattel Mortgage",
covering substantially all of its properties and assets in Marinduque.
When Marcopper defaulted in the payment of its loan obligation, Placer
Dome, in fulfillment of its undertaking under the "Support and Standby Credit
Agreement," and presumably to preserve its international credit standing,
agreed to have its subsidiary corporation, petitioner MR Holding, Ltd.,
assumed Marcoppers obligation to ADB in the amount of US$ 18,453,450.02.
Consequently, in an "Assignment Agreement, ADB assigned to petitioner all
its rights, interests and obligations under the principal and complementary
loan agreements. Marcopper likewise executed a "Deed of Assignment" 8 in
favor of petitioner. Under its provisions, Marcopper assigns, transfers, cedes
and conveys to petitioner, its assigns and/or successors-in-interest all of its
(Marcoppers) properties, mining equipment and facilities.
Meanwhile Solidbank obtained a Partial Judgment against Marcopper
ordering Marcopper to pay Solidbank the sum of PHP 52,970,756.89. RTC of
Manila issued a writ of execution pending appeal directing Carlos P. Bajar,
respondent sheriff, to require Marcopper "to pay the sums of money to
satisfy the Partial Judgment. Bajar issued two notices of levy on Marcoppers

Page 849 of 1072

personal and real properties, and over all its stocks of scrap iron and
unserviceable mining equipment. Having learned of the scheduled auction
sale, petitioner served an "Affidavit of Third-Party Claim" upon respondent
sheriffs, asserting its ownership over all Marcoppers mining properties,
equipment and facilities by virtue of the "Deed of Assignment."
ISSUE/S:
Does petitioner have the legal capacity to sue?
RULING:
The traditional case law definition has metamorphosed into a statutory
definition, having been adopted with some qualifications in various pieces of
legislation in our jurisdiction. For instance, Republic Act No. 7042, otherwise
known as the "Foreign Investment Act of 1991," defines "doing business" as
follows:
"d) The phrase doing business shall include soliciting
orders, service contracts, opening offices, whether called
liaison offices or branches; appointing representatives or
distributors domiciled in the Philippines or who in any
calendar year stay in the country for a period or periods
totalling one hundred eight(y) (180) days or more;
participating in the management, supervision or control of
any domestic business, firm, entity, or corporation in the
Philippines; and any other act or acts that imply a
continuity of commercial dealings or arrangements,
and contemplate to that extent the performance of
acts or works; or the exercise of some of the
functions normally incident to, and in progressive
prosecution of, commercial gain or of the purpose
and object of the business organization; Provided,
however, That the phrase doing business shall not be
deemed to include mere investment as a shareholder by a
foreign entity in domestic corporations duly registered to
do business, and/or the exercise of rights as such investor,
nor having a nominee director or officer to represent its
interests in such corporation, nor appointing a
representative or distributor domiciled in the Philippines
which transacts business in its own name and for its own
account." (Emphasis supplied).
Likewise, Section 1 of Republic Act No. 5455, provides that:
"SECTION. 1. Definition and scope of this Act. - (1) x x x
the phrase doing business shall include soliciting orders,
purchases, service contracts, opening offices, whether
called
liaison
offices
or
branches;
appointing
representatives or distributors who are domiciled in the
Philippines or who in any calendar year stay in the

Page 850 of 1072

Philippines for a period or periods totaling one hundred


eighty days or more; participating in the management,
supervision or control of any domestic business firm, entity
or corporation in the Philippines; and any other act or acts
that imply a continuity of commercial dealings or
arrangements, and contemplate to that extent the
performance of acts or works, or the exercise of some of
the functions normally incident to, and in progressive
prosecution of, commercial gain or of the purpose and
object of the business organization."
In the case at bar, the Court of Appeals categorized as "doing
business" petitioners participation under the "Assignment Agreement" and
the "Deed of Assignment." This is simply untenable. The expression "doing
business" should not be given such a strict and literal construction as to
make it apply to any corporate dealing whatever. At this early stage and with
petitioners acts or transactions limited to the assignment contracts, it
cannot be said that it had performed acts intended to continue the business
for which it was organized. It may not be amiss to point out that the purpose
or business for which petitioner was organized is not discernible in the
records. No effort was exerted by the Court of Appeals to establish the nexus
between petitioners business and the acts supposed to constitute "doing
business." Thus, whether the assignment contracts were incidental to
petitioners business or were continuation thereof is beyond determination.
We cannot apply the case cited by the Court of Appeals, Far East Intl Import
and Export Corp. vs. Nankai Kogyo Co., Ltd., which held that a single act may
still constitute "doing business" if "it is not merely incidental or casual, but is
of such character as distinctly to indicate a purpose on the part of the foreign
corporation to do other business in the state." In said case, there was an
express admission from an official of the foreign corporation that he was sent
to the Philippines to look into the operation of mines, thereby revealing the
foreign corporations desire to continue engaging in business here. But in the
case at bar, there is no evidence of similar desire or intent. Unarguably,
petitioner may, as the Court of Appeals suggested, decide to operate
Marcoppers mining business, but, of course, at this stage, that is a mere
speculation. Or it may decide to sell the credit secured by the mining
properties to an offshore investor, in which case the acts will still be isolated
transactions. To see through the present facts an intention on the part of
petitioner to start a series of business transaction is to rest on assumptions
or probabilities falling short of actual proof. Courts should never base its
judgments on a state of facts so inadequately developed that it cannot be
determined where inference ends and conjecture begins.

Page 851 of 1072

TOPIC:DOING BUSINESS WITH/WITHOUT LICENSE:SUITS BY OR AGAINST


FOREIGN
CORPORATION
THE COMMISSIONER OF CUSTOMS
VS.
K.M.K. GANI, INDRAPAL & CO.,
AND THE HONORABLE COURT OF TAX APPEALS
G.R. NO. 73722. FEBRUARY 26, 1990
FACTS:
Two containers loaded with 103 cartons of merchandise covered by 11
airway bills of several supposedly Singapore-based consignees arrived at the
Manila International Airport from Hongkong. The cargoes were consigned to
these different entities: K.M.K. and Indrapal and Sin Hong Lee Trading Co.,
Ltd., AAR TEE Enterprises, and C. Ratilal all purportedly based in Singapore.
While the cargoes were at the Manila International Airport, a "reliable
source" tipped off the BOC that the said cargoes were going to be unloaded
in Manila. Forthwith, the Bureau's agency on such matters, the Suspected
Cargo and Anti-Narcotics (SCAN), dispatched an agent to verify the
information. Upon arriving at the airport, the SCAN agent saw an empty PAL
van parked directly alongside the plane's belly from which cargoes were
being unloaded. When the SCAN agent asked the van's driver why he was at
the site, the driver drove away in his vehicle. The SCAN agent then
sequestered the unloaded cargoes. These goods were transferred to the
International Cargo Terminal under Warrant of Seizure and Detention and
thereafter subjected to Seizure and Forfeiture proceedings for "technical
smuggling."
ISSUE/S:
Whether or not the private respondents failed to establish their
personality to sue in a representative capacity, hence making
their action dismissable,
RULING:

Page 852 of 1072

The fact that a foreign corporation is not doing business in the


Philippines must be disclosed if it desires to sue in Philippine courts under
the "isolated transaction rule." Without this disclosure, the court may choose
to deny it the right to sue.
In the case at bar, the private respondents K.M.K. and INDRAPAL aver
that they are "suing upon a singular and isolated transaction." But they failed
to prove their legal existence or juridical personality as foreign corporations.
We are cognizant of the fact that under the "isolated transaction rule," only
foreign corporations and not just any business organization or entity can
avail themselves of the privilege of suing before Philippine courts even
without a license. Counsel Armando S. Padilla stated before the respondent
Court of Tax Appeals that his clients are "suing upon a singular and isolated
transaction." But there is no proof to show that K.M.K. and INDRAPAL are
indeed what they are represented to be. It has been simply stated by
Attorney Padilla that K.M.K. Gani is "a single proprietorship," while INDRAPAL
is "a firm," and both are "doing business in accordance with the laws of
Singapore ... ," with specified addresses in Singapore. In cases of this nature,
these allegations are not sufficient to clothe a claimant of suspected
smuggled goods of juridical personality and existence. The "isolated
transaction rule" refers only to foreign corporations. Here the petitioners are
not foreign corporations. They do not even pretend to be so. The first
paragraph of their petition before the Court, containing the allegation of their
identities, does not even aver their corporate character. On the contrary,
K.M.K. alleges that it is a "single proprietorship" while INDRAPAL hides under
the vague identification as a "firm," although both describe themselves with
the phrase "doing business in accordance with the laws of Singapore."
Absent such proof that the private respondents are corporations
(foreign or not), the respondent Court of Tax Appeals should have barred
their invocation of the right to sue within Philippine jurisdiction under the
"isolated transaction rule" since they do not qualify for the availment of such
right.

Page 853 of 1072

TOPIC:DOING BUSINESS WITH/WITHOUT LICENSE:SUITS BY OR AGAINST


FOREIGN
CORPORATION
COMMUNICATION MATERIALS AND DESIGN, INC., ASPAC MULTITRADE, INC., (FORMERLY ASPAC-ITEC PHILIPPINES, INC.) AND
FRANCISCO S. AGUIRRE
VS.
THE COURT OF APPEALS, ITEC INTERNATIONAL, INC., AND ITEC, INC.
G.R. NO. 102223. AUGUST 22, 1996
FACTS:
CMDI and ASPAC are both domestic corporations, while Francisco S.
Aguirre is their President and majority stockholder. ITEC is a foreign
corporation not licensed to do business in the Philippines. ITEC entered into a
contract with petitioner ASPAC referred to as "Representative Agreement".
Pursuant to the contract, ITEC engaged ASPAC as its "exclusive
representative" in the Philippines for the sale of ITEC's products, in
consideration of which, ASPAC was paid a stipulated commission. The
agreement was signed by G.A. Clark and Francisco S. Aguirre, presidents of
ITEC and ASPAC respectively, for and in behalf of their companies.
Through a "License Agreement" entered into by the same parties,
ASPAC was able to incorporate and use the name "ITEC" in its own name.
Thus, ASPAC Multi-Trade, Inc. became legally and publicly known as ASPACITEC (Philippines).
One year into the second term of the parties' Representative
Agreement, ITEC decided to terminate the same, because petitioner ASPAC
allegedly violated its contractual commitment as stipulated in their
agreements.ITEC charges the petitioners and another Philippine Corporation,
DIGITAL BASE COMMUNICATIONS, INC. (DIGITAL, for brevity), the President of
which is likewise petitioner Aguirre, of using knowledge and information of
ITEC's products specifications to develop their own line of equipment and
product support, which are similar, if not identical to ITEC's own, and offering
them to ITEC's former customer.

Page 854 of 1072

ISSUE/S:
Whether or not private respondent has capacity to bring suit
here in the Philippines.
RULING:
A foreign corporation doing business in the Philippines may sue in
Philippine Courts although not authorized to do business here against a
Philippine citizen or entity who had contracted with and benefited by said
corporation.To put it in another way, a party is estopped to challenge the
personality of a corporation after having acknowledged the same by entering
into a contract with it. And the doctrine of estoppel to deny corporate
existence applies to a foreign as well as to domestic corporations.One who
has dealt with a corporation of foreign origin as a corporate entity is
estopped to deny its corporate existence and capacity: The principle will be
applied to prevent a person contracting with a foreign corporation from later
taking advantage of its noncompliance with the statutes chiefly in cases
where such person has received the benefits of the contract.
The rule is deeply rooted in the time-honored axiom of Commodum ex
injuriasua non haberedebet no person ought to derive any advantage of
his own wrong. This is as it should be for as mandated by law, "every person
must in the exercise of his rights and in the performance of his duties, act
with justice, give everyone his due, and observe honesty and good faith."

Page 855 of 1072

TOPIC:DOING BUSINESS WITH/WITHOUT LICENSE:SUITS BY OR AGAINST


FOREIGN
CORPORATION
COLUMBIA PICTURES, INC., ORION PICTURES CORPORATION,
PARAMOUNT PICTURES CORPORATION, TWENTIETH CENTURY FOX
FILM CORPORATION, UNITED ARTISTS CORPORATION, UNIVERSAL
CITY STUDIOS, INC., THE WALT DISNEY COMPANY, AND WARNER
BROTHERS, INC.
VS.
COURT OF APPEALS, SUNSHINE HOME VIDEO, INC. AND DANILO A.
PELINDARIO,
G.R. NO. 110318. AUGUST 28, 1996
FACTS:
Complainants lodged a formal complaint with the NBI for violation of
PD No. 49, as amended, and sought its assistance in their anti-film piracy
drive. Agents of the NBI and private researchers made discreet surveillance
on various video establishments in Metro Manila including Sunshine
NBI Senior Agent Lauro C. Reyes applied for a search warrant with the
court a quo against Sunshine seeking the seizure of pirated video tapes of
copyrighted films and television sets, video cassettes and/or laser disc
recordings equipment and other machines and paraphernalia used or
intended to be used in the unlawful exhibition, showing, reproduction, sale,
lease or disposition of videograms tapes in the premises above described.
The search warrant was served to Sunshine and/or their representatives. In
the course of the search of the premises indicated in the search warrant, the
NBI Agents found and seized various video tapes of duly copyrighted motion
pictures/films owned or exclusively distributed by private complainants, and
machines, equipment, television sets, paraphernalia, materials, accessories
all of which were included in the receipt for properties accomplished by the
raiding team. Copy of the receipt was furnished and/or tendered to Mr. Danilo
A. Pelindario, registered owner-proprietor of Sunshine Home Video.
ISSUE/S:
Whether or not foreign corporation can maintain a suit in
Philippine courts in the absence of a license to do business

Page 856 of 1072

RULING:
Presidential Decree No. 1789, in Article 65 thereof, defines "doing
business" to include soliciting orders, purchases, service contracts, opening
offices, whether called "liaison" offices or branches; appointing
representatives or distributors who are domiciled in the Philippines or who in
any calendar year stay in the Philippines for a period or periods totalling one
hundred eighty days or more; participating in the management, supervision
or control of any domestic business firm, entity or corporation in the
Philippines, and any other act or acts that imply a continuity of commercial
dealings or arrangements and contemplate to that extent the performance of
acts or works, or the exercise of some of the functions normally incident to,
and in progressive prosecution of, commercial gain or of the purpose and
object of the business organization.
The implementing rules and regulations of said presidential decree
conclude the enumeration of acts constituting "doing business" with a catchall definition, thus:
Sec. 1(g). "Doing Business" shall be any act or combination of acts
enumerated in Article 65 of the Code. In particular "doing business" includes:
xxx xxx xxx
(10) Any other act or acts which imply a continuity of
commercial dealings or arrangements, and contemplate to
that extent the performance of acts or works, or the
exercise of some of the functions normally incident to, or in
the progressive prosecution of, commercial gain or of the
purpose and object of the business organization.
Finally, Republic Act No. 7042 19 embodies such concept in this wise:
Sec. 3. Definitions. As used in this Act:
xxx xxx xxx
(d) the phrase "doing business shall include soliciting
orders, service contracts, opening offices, whether called
"liaison" offices or branches; appointing representatives or
distributors domiciled in the Philippines or who in any
calendar year stay in the country for a period or periods
totalling one hundred eight(y) (180) days or more;
participating in the management, supervision or control of
any domestic business, firm, entity or corporation in the
Philippines; and any other act or acts that imply a
continuity of commercial dealings or arrangements, and
contemplate to that extent the performance of acts or
works, or the exercise of some of the functions normally
incident to, and in progressive prosecution of, commercial
gain or of the purpose and object of the business

Page 857 of 1072

organization: Provided, however, That the phrase "doing


business" shall not be deemed to include mere investment
as a shareholder by a foreign entity in domestic
corporations duly registered to do business, and/or the
exercise of rights as such investor; nor having a nominee
director or officer to represent its interests in such
corporation; nor appointing a representative or distributor
domiciled in the Philippines which transacts business in its
own name and for its own account.
Accordingly, the certification issued by the Securities and Exchange
Commission stating that its records do not show the registration of petitioner
film companies either as corporations or partnerships or that they have been
licensed to transact business in the Philippines, while undeniably true, is of
no consequence to petitioners' right to bring action in the Philippines. Verily,
no record of such registration by petitioners can be expected to be found for,
as aforestated, said foreign film corporations do not transact or do business
in the Philippines and, therefore, do not need to be licensed in order to take
recourse to our courts.

Page 858 of 1072

TOPIC:DOING BUISINESS WITH/WITHOUT LICENSE: SUIT BY OR AGAINST


FOREIGN CORPORATION
ERIKS PTE. LTD.
VS.
COURT OF APPEALS AND DELFIN F. ENRIQUEZ, JR.
G.R. NO. 118843, FEBRUARY 6, 1997
FACTS:
Eriks Pte. Ltd. is a non-resident foreign corporation engaged in the
manufacture and sale of elements used in sealing pumps, valves and pipes
for industrial purposes, valves and control equipment used for industrial fluid
control and PVC pipes and fittings for industrial uses. In its complaint, it
alleged that it is a corporation duly organized and existing under the laws of
the Republic of Singapore and not licensed to do business in the Philippines
and is not so engaged and is suing on an isolated transaction for which it has
capacity to sue.
On various dates, Delfin Enriquez, Jr., doing business under the name
and style of Delrene EB Controls Center and/or EB Karmine Commercial,
ordered and received from petitioner various elements used in sealing
pumps, valves, pipes and control equipment, PVC pipes and fittings. The
transfers of goods were perfected in Singapore, for private respondent's
account, F.O.B. Singapore, with a 90-day credit term. Subsequently, demands
were made by petitioner upon private respondent to settle his account, but
the latter failed/refused to do so.
Petitioner corporation filed with the RTC for the recovery of
S$41,939.63 or its equivalent in Philippine currency, plus interest thereon
and damages. Private respondent responded with a Motion to Dismiss,
contending that petitioner corporation had no legal capacity to sue.
ISSUE/S:
Whether Petitioner Corporation may maintain an action in
Philippine courts considering that it has no license to do business
in the country.
RULING:
The Corporation Code provides:
Sec. 133. Doing business without a license. No foreign
corporation transacting business in the Philippines without

Page 859 of 1072

a license, or its successors or assigns, shall be permitted to


maintain or intervene in any action, suit or proceeding in
any court or administrative agency of the Philippines; but
such corporation may be sued or proceeded against before
Philippine courts or administrative tribunals on any valid
cause of action recognized under Philippine laws.
However, there is no definitive rule on what constitutes "doing,"
"engaging in," or "transacting" business. The Corporation Code itself does
not define such terms. To fill the gap, the evolution of its statutory definition
has produced a rather all-encompassing concept in Republic Act No. 7042 in
this wise:
Sec. 3. Definitions. As used in this Act:
xxx xxx xxx
(d) the phrase "doing business" shall include soliciting
orders, service contracts, opening offices, whether called
"liaison" offices or branches; appointing representatives or
distributors domiciled in the Philippines or who in any
calendar year stay in the country for a period or periods
totalling one hundred eight(y) (180) days or more;
participating in the management, supervision or control of
any domestic business, firm, entity or corporation in the
Philippines; and any other act or acts that imply a
continuity of commercial dealings or arrangements, and
contemplate to that extent the performance of acts or
works,or the exercise of some of the functions normally
incident to, and in progressive prosecution of, commercial
gain or of the purpose and object of the business
organization: Provided, however, That the phrase "doing
business" shall not be deemed to include mere investment
as a shareholder by a foreign entity in domestic
corporations duly registered to do business, and/or the
exercise of rights as such investor; nor having a nominee
director or officer to represent its interests in such
corporation; nor appointing a representative or distributor
domiciled in the Philippines which transacts business in its
own name and for its own account. (emphasis supplied)
We find no reason to disagree with both lower courts. More than the
sheer number of transactions entered into, a clear and unmistakable
intention on the part of petitioner to continue the body of its business in the
Philippines is more than apparent. As alleged in its complaint, it is engaged
in the manufacture and sale of elements used in sealing pumps, valves, and
pipes for industrial purposes, valves and control equipment used for
industrial fluid control and PVC pipes and fittings for industrial use. Thus, the
sale by petitioner of the items covered by the receipts, which are part and

Page 860 of 1072

parcel of its main product line, was actually carried out in the progressive
prosecution of commercial gain and the pursuit of the purpose and object of
its business, pure and simple. Further, its grant and extension of 90-day
credit terms to private respondent for every purchase made, unarguably
shows an intention to continue transacting with private respondent, since in
the usual course of commercial transactions, credit is extended only to
customers in good standing or to those on whom there is an intention to
maintain long-term relationship. This being so, the existence of a
distributorship agreement between the parties, as alleged but not proven by
private respondent, would, if duly established by competent evidence, be
merely corroborative, and failure to sufficiently prove said allegation will not
significantly affect the finding of the courts below. Nor our own ruling. It is
precisely upon the set of facts above detailed that we concur with
respondent Court that petitioner corporation was doing business in the
country.

Page 861 of 1072

TOPIC:DOING BUSINESS WITH OR WITHOUT A LICENSE: SUITS BY OR


AGAINST
FOREIGN CORPORATIONS
FAR EAST INTERNATIONAL IMPORT and EXPORT CORPORATION
VS.
NANKAI KOGYO CO. LTD., ET AL.
G.R. No. L-13525, November 30, 1962
6 SCRA 725
FACTS:
On December 26, 1956, the Far East International Import & Export
Corporation, organized under Philippine Laws, entered into a Contract of
Sale of Steel Scrap with the Nankai Kogyo Co., Ltd., a foreign corporation
organized under Japanese Laws with address at Osaka, Japan. The buyer sign
in Japan and the seller in Manila, Philippines.
Upon perfection of the contract and after having been informed of the
readiness to ship and that the Export License was to expire on March 18,
1957,Nankai opened a letter for credit (No. 38/80049) with the China
Banking Corporation, issued by the Nippon Kangyo, Ltd., Tokyo, Japan, in the
amount of $312,500.00 on January 30, 1957.
On March 15, 1957, only 4 days before the expiration of the Far East
license, three boats sent by Nankai arrived in the Philippines, one to load in
Manila, the other two at Poro Point, San Fernando, La Union, and Tacloban,
Leyte, respectively. However, the loading of the said boats was stopped due
to the expiration of the said license. An agreement was reached whereby the
Far East would seek an extension of the license. However, the untimely death
of President Magsaysay and the taking over by President Garcia changed the
picture, for the latter and/or his agents refused to extend the license. The
two boats sailed to Japan without any cargo, the third (SS Mina) only 1,058.6
metric tons.
On April 27, 1957, Nankai confirmed and acknowledged delivery of the
1,058.6 metric tons of steel scrap, but asked for damages amounting to
$148,135.00 consisting of dead freight charges, damages, bank charges,
phone and cable expenses.
On May 4, 1957, Far East wrote the Everett Steamship Corporation,
requesting the issuance of a complete set of the Bill of Lading for the
shipment, in order that payment thereof be effected against the Letter of
Credit. Under date of May 7, 1957, the Everett informed Far East that they

Page 862 of 1072

were not in a position to comply because the Bill of Lading was issued and
signed in Tokyo by the Master of the boat, upon request of the Charterer,
defendant herein.
As repeated requests, both against the shipping agent and the buyers
(Nankai), for the issuance of the of Bill Lading were ignored, Far East filed on
May 16, 1957, the present complaint for Specific Performance, damages, a
writ of preliminary mandatory injunction directed against Nankai and the
shipping company, to issue and deliver to the plaintiff, a complete set of
negotiable of Lading for the 1,058.6 metric tons of scrap and a writ of
preliminary injunction against the China Banking Corporation and the Nankai
to maintain the Letter Credit.
The appellant alleges that the lower court did not acquire jurisdiction,
because it was not doing business in the Philippines and the requirement of
summons had not been fulfilled.
ISSUE/S:
Whether or not Nankai is doing business in the Philippines for
Philippine Courts to acquire jurisdiction
RULING:
It is difficult to lay down any rule of universal application to determine
when a foreign corporation is doing business. Each case must turn upon its
own peculiar facts and upon the language of the statute applicable. But from
the proven facts obtaining in this particular case, the appellant's defense of
lack of jurisdiction appears unavailing.
The doing of a single act does not constitute business within the
meaning of statutes prescribing the conditions to be complied with the
foreign corporations must be qualified to this extent, that a single act may
bring the corporation. In such a case, the single act of transaction is not
merely incidental or casual, but is of such character as distinctly to indicate
a purpose on the part of the foreign corporation to do other business in the
state, and to make the state a basis of operations for the conduct of a part of
corporation's ordinary business.
It is finally noted that when defendant's motion to dismiss in the
Micronesian case was denied, it immediately brought the matter to this Court
on Prohibition seeking to restrain the Workmen's Compensation mission from
exercising jurisdiction over the controversy. In the present case, the
defendant, while entering a Special Appearance to contest the jurisdiction of
the Court, pursued its defense further by filing its Answer and going into trial.
There is no appeal on the lower court's findings that the failure of the
appellee herein to make full shipment of the scrap was due, not to the fault

Page 863 of 1072

of said appellee, but to the action and intervention of the Philippine


Government, which was beyond the control of the plaintiff. This aspect of the
case is particularly covered by paragraph 13 of the contract, heretofore
reproduced.

TOPIC:DOING BUSINESS WITH OR WITHOUT A LICENSE: SUITS BY OR


AGAINST
FOREIGN CORPORATIONS
FACILITIES MANAGEMENT CORPORATION
VS.
LEONARDO DE LA OSA AND THE HONORABLE COURT OF INDUSTRIAL
RELATIONS
G.R. NO. L-38649 MARCH 26, 1979
89 SCRA 131
FACTS:
De la Osa alleged that he was employed by respondents painter,
houseboy and cashier sucessively. He further averred that from December,
1965 to August, 1966, inclusive, he rendered overtime services daily and
that this entire period was divided into swing and graveyard shifts to which
he was assigned, but he was not paid both overtime and night shift
premiums despite his repeated demands from respondents.
In a petition filed on July 1, 1967, Leonardo dela Osa sought his
reinstatement with full backwages, as well as the recovery of his overtime
compensation, swing shift and graveyard shift differentials.
Respondents filed on August 7, 1967 their letter- answer without
substantially denying the material allegations of the basic petition but
interposed the following special defenses, namely: That respondents
Facilities Management Corporation and J. S. Dreyer are domiciled in Wake
Island which is beyond the territorial jurisdiction of the Philippine
Government
ISSUE/S:
Whether or not the mere act by a non-resident foreign
corporation of recruiting Filipino workers for its own use abroad,
in law is considered doing business in the Philippines
RULING:
The principal issue presented in this special civil action is whether
petitioner has been 'doing business in the Philippines' so that the service of

Page 864 of 1072

summons upon its agent in the Philippines vested the Court of First Instance
of Manila with jurisdiction.
From the facts of record, the petitioner may be considered as doing
business in the Philippines within the scope of Section 14, Rule 14 of the
Rules of the Court which provide:
SEC 14. Service upon private foreign corporations. If the
defendant is a foreign corporation or a non-resident joint stock
company or association: doing business in the Philippines,
service may be made on its resident agent designated in
accordance with law for that purpose or, if there be no such
agent, on the government official designated by law to that
effect, or on any of its officers or agents within the Philippines.
Indeed, the petitioner, in compliance with Act 2486 as implemented by
Department of Labor Order No. IV dated May 20, 1968 had to appoint Jaime
V. Catuira, 1322 A. Mabini, Ermita, Manila as agent for FMC with authority to
execute Employment Contracts and receive, in behalf of that corporation,
legal services from and be bound by processes of the Philippine Courts of
Justice, for as long as he remains an employee of FMC. It is a fact that when
the summons for the petitioner was served on Jaime V. Catuira he was still in
the employ of the FMC.
In his motion to dismiss, petitioner admits that Mr. Catuira represented
it in this country 'for the purpose of making arrangements for the approval
by the Department of Labor of the employment of Filipinos who are recruited
by the Company as its own employees for assignment abroad.' In effect, Mr.
Catuira was an officer representing petitioner in the Philippines.
Under the rules and regulations promulgated by the Board of
Investments which took effect Feb. 3, 1969, implementing Rep. Act No. 5455,
which took effect Sept. 30, 1968, the phrase 'doing business' has been
exemption with illustrations, among them being as follows:
xxx xxx xxx
(f) the performance within the Philippines of any act or
combination of acts enumerated in section l(l) of the Act shall
constitute 'doing business' therein. in particular, 'doing business
includes:
(1) Soliciting orders, purchases (sales) or service
contracts. Concrete and specific solicitations by a
foreign firm, not acting independently of the foreign
firm amounting to negotiation or fixing of the terms
and conditions of sales or service contracts,
regardless of whether the contracts are actually

Page 865 of 1072

reduced to writing, shall constitute doing business


even if the enterprise has no office or fixed place of
business in the Philippines. xxx
(2) Appointing a representative or distributor who is
dociled in the Philippines, unless said representative
or distributor has an independent status, i.e., it
transacts business in its name and for its own
account, and not in the name or for the account of
the principal.
xxx xxx xxx
(4) Opening offices, whether called 'liaison'offices,
agencies or branches, unless proved otherwise.
xxx xxx xxx
(10) Any other act or acts that imply a continuity of
commercial
dealings
or
arrangements,
and
contemplate to that extent the performance of acts
or works, or the exercise of some of the functions
normally incident to, or in the progressive
prosecution of, commercial gain or of the purpose
and objective of the business organization.

Page 866 of 1072

TOPIC:DOING BUSINESS WITH OR WITHOUT A LICENSE: SUITS BY OR


AGAINST
FOREIGN CORPORATIONS
FRENCH OIL MILL MACHINERY CO., INC.
VS.
COURT OF APPEALS
G.R. NO. 126477 SEPTEMBER 11, 1998
295 SCRA 462
FACTS:
Private Respondent Ludo & Luym Oleochemical Co Filed a complaint for
breach of contract with damages against petitioner foreign corporation
French Oil Mill and the latter's alleged Philippine agent Trans-World Trading
Company. The complaint alleges that
FOMMCO is a corporation with
principal office at, Piqua, Ohio, United States of America, engaged in
business in the Philippines through its agent Trans-World Trading Company.
FOMMCO may be served with summons and other court processes through
its agent, Trans-World Trading Company.
Summons was served on Trans-World which moved to dismiss the
complaint arguing that it is not petitioner's agent. Petitioner itself filed a
special appearance with motion to dismiss contending that the court had no
jurisdiction over its person due to improper service of summons. It argued
that
a. it is not doing business in the Philippines and
b. Trans-World is not its agent, therefore the procedure in Sections
14 and 17, Rule 14 should have been observed.
The court a quo initially dismissed the complaint for lack of jurisdiction
over petitioner but on private respondent's motion for reconsideration, said
court reversed the order of dismissal and ruled that summons was properly
served on petitioner whom it found doing business in the Philippines and
Trans-World as its agent. Petitioner elevated the case to the Court of Appeals
(CA) via petition for certiorari and prohibition but to no avail. Not satisfied,
petitioner filed this petition under Rule 45 which was initially dismissed for
being filed late but on petitioner's motion for reconsideration was reinstated
by the Court.
Petitioner contends that it is not doing business in the Philippines and
that Trans-World is not its agent, and thus, the summons served on the latter
has no effect on the former.
ISSUE/S:

Page 867 of 1072

Whether or not petitioner foreign corporation is doing business in


the Philippines.
RULING:
It is not enough to merely allege in the complaint that a defendant
foreign corporation is doing business. For purposes of the rule on summons,
the fact of doing business must first be "established by appropriate
allegations in the complaint and the court in determining such fact need not
go beyond the allegations therein.
In this case, the allegations that
petitioner entered into a contract with private respondent to supply and
install various machineries and equipments for the use of the latter's oil mill
factory and that the first shipment of machineries from petitioner was
received by private respondent are sufficient allegations that petitioner is
doing business for purposes of Section 14, Rule 14. In any case, the
determination that a foreign corporation is doing business is merely tentative
and only to enable the local court to acquire jurisdiction over the person of
the foreign corporation through service of summons. It does not foreclose a
subsequent finding to the contrary depending on the evidence.

Page 868 of 1072

TOPIC:DOING BUSINESS WITH OR WITHOUT A LICENSE: SUITS BY OR


AGAINST
FOREIGN CORPORATIONS
H.B. ZACHRY COMPANY INTERNATIONAL
VS.
HON. COURT OF and VINNEL-BELVOIR CORPORATION
G.R. No. 106989 May 10, 1994
232 SCRA 29
FACTS:
On 17 July 1987, VBC entered into a written Subcontract Agreement
with Zachry, a foreign corporation. The latter had been engaged by the
United States Navy to design and construct 264 Family Housing Units at the
US Naval Base at Subic, Zambales. Under the agreement, specifically under
Section 3 on Payment, VBC was to perform all the construction work on the
housing project and would be paid "for the performance of the work the sum
of Six Million Four Hundred Sixty-eight Thousand U.S. Dollars (U.S.
$6,468,000.00), subject to additions and deductions for changes as
hereinafter provided." On 20 March 1990, VBC filed a Complaint with the RTC
of Makati against Zachry for the collection of the payments due it with a
prayer for a writ of preliminary attachment over Zachry's bank account in
Subic Base and over the remaining thirty-one undelivered housing units
which were to be turned over to the US Navy by Zachry on 30 March 1990.
The trial court granted the application for the issuance of the writ of
preliminary attachment and fixing the attachment bond at P24, 266,000.00.
On 6 April 1990, Zachry filed a motion to dismiss the complaint on the
ground of lack of jurisdiction over its person because the summons was not
validly served on it. It alleges that it is a foreign corporation duly licensed on
13 November 1989 by the Securities and Exchange Commission to do
business in the Philippines and, pursuant to Section 128 of the Corporation
Code of the Philippines, had appointed Atty. Lucas Nunag as its resident
agent on whom any summons and legal processes against it. Summons and
a copy of the Amended Complaint were served on 24 April 1990 on Zachry
through Atty. Nunag as shown in the sheriff's return dated 24 April 1990.
On 24 May 1990, Zachry filed an Omnibus Motion (a) to dismiss the
complaint for lack of jurisdiction over its person since the subsequent service
of summons did not cure the jurisdictional defect it earlier pointed out and, in
the alternative, to dismiss the case or suspend the proceedings therein for
failure of the plaintiff to submit the controversy in question to arbitration as
provided for in its contract with Zachry; and (b) to dissolve the writ of
attachment of 26 March 1990 "for having been issued without jurisdiction,
having been issued prior to the service of summons. may be served. Atty.

Page 869 of 1072

Nunag's address is at the 10th Floor, Shell House, 156 Valero St., Makati,
Metro Manila.
ISSUE:
Whether or not the writ of attachment may be validly issued
against a foreign corporation
RULING:
It was error for the Court of Appeals to declare, on the ground of grave
abuse of discretion, the nullity of the writ of attachment issued by the trial
court on 21 March 1990. In the first place, the writ was in fact issued only on
26 March 1990 and served, together with the summons, copy of the
complaint, the Order of 21 March 1990, and the bond, on 27 March 1990 on
Zachry at its field office in Subic Bay, Zambales, through one Ruby Apostol.
What the Court of Appeals referred to as having been issued on 21 March
1990 is the order granting the application for the issuance of a writ of
preliminary attachment upon the posting of a bond of P24,266,000.00. 41 In
the second place, even granting arguendo that the Court of Appeals had
indeed in mind the 26 March 1990 writ of attachment, its issuance, as well as
the issuance of the 21 March 1990 Order, did not suffer from any procedural
or jurisdictional defect; the trial court could validly issue both.
However, the writ of attachment cannot be validly enforced through
the levy of Zachry's property before the court had acquired jurisdiction over
Zachry's person either through its voluntary appearance or the valid service
of summons upon it. 42 To put it in another way, a distinction should be made
between the issuance and the enforcement of the writ. The trial court has
unlimited power to issue the writ upon the commencement of the action
even before it acquires jurisdiction over the person of the defendant, but
enforcement thereof can only be validly done after it shall have acquired
such jurisdiction.
The validity then of the order granting the application for a writ of
preliminary attachment on 21 March 1990 and of the issuance of the writ of
preliminary attachment on 26 March 1990 is beyond dispute. However, the
enforcement of the preliminary attachment on 27 March 1990, although
simultaneous with the service of the summons and a copy of the complaint,
did not bind Zachry because the service of the summons was not validly
made. When a foreign corporation has designated a person to receive
service of summons pursuant to the Corporation Code, that designation is
exclusive and service of summons on any other person is inefficacious. 49 The
valid service of summons and a copy of the amended complaint was only
made upon it on 24 April 1990, and it was only then that the trial court
acquired jurisdiction over Zachry's person. Accordingly, the levy on
attachment made by the sheriff on 27 April 1990 was invalid. However, the
writ of preliminary attachment may be validly served anew.

Page 870 of 1072

TOPIC:DOING BUSINESS WITH OR WITHOUT A LICENSE: SUITS BY OR


AGAINST
FOREIGN CORPORATIONS
THE HOME INSURANCE COMPANY
VS.
EASTERN SHIPPING LINES and/or ANGEL JOSE TRANSPORTATION,
INC.
G.R. NO. L-34382, JULY 20, 1983
123 SCRA 424
FACTS:
The plaintiff is a foreign insurance company duly authorized to do
business in the Philippines through its agent, Mr. Victor H. Bello, of legal age
and with office address at Oledan Building, Ayala Avenue, Makati, Rizal.
On or about January 13, 1967, S. Kajita & Co., on behalf of Atlas
Consolidated Mining & Development Corporation, shipped on board the SS
"Eastern Jupiter' from Osaka, Japan, 2,361 coils of "Black Hot Rolled Copper
Wire Rods." The said vessel is owned and operated by defendant Eastern
Shipping Lines. The shipment was covered by Bill of Lading No. O-MA-9, with
arrival notice to Phelps Dodge Copper Products Corporation of the Philippines
(CONSIGNEE) at Manila. The shipment was insured with plaintiff against all
risks in the amount of P1,580,105.06. However, when the goods arrived,
some of them were in bad order and that there is a shortage in its actual
weight as against its invoiced weight. Thus, for the loss/damage suffered by
the cargo, plaintiff paid the consignee under its insurance policy the amount
of P3, 260.44, by virtue of which plaintiff became subrogated to the rights
and actions of the Consignee. Plaintiff made demands for payment against
the CARRIER and The Transportation Company for reimbursement of the
aforesaid amount but each refused to pay the same.
Petitioner herein thus filed civil suit against Eastern Shipping. However,
its petition was dismissed for its failure to prove its capacity to sue in
Philippine courts.
ISSUE/S:
Whether or not the Honorable Trial Court erred in dismissing the
complaint on the finding that plaintiff-appellant has no capacity
to sue.
RULING:
On the basis of factual and equitable considerations, there is no
question that the private respondents should pay the obligations found by

Page 871 of 1072

the trial court as owing to the petitioner. Only the question of validity of the
contracts in relation to lack of capacity to sue stands in the way of the
petitioner being given the affirmative relief it seeks. Whether or not the
petitioner was engaged in single acts or solitary transactions and not
engaged in business is likewise not in issue. The petitioner was engaged in
business without a license. The private respondents' obligation to pay under
the terms of the contracts has been proved.
When the complaints in these two cases were filed, the petitioner had
already secured the necessary license to conduct its insurance business in
the Philippines. It could already filed suits.
Petitioner was, therefore, telling the truth when it averred in its
complaints that it was a foreign insurance company duly authorized to do
business in the Philippines through its agent Mr. Victor H. Bello. However,
when the insurance contracts which formed the basis of these cases were
executed, the petitioner had not yet secured the necessary licenses and
authority. The lower court, therefore, declared that pursuant to the basic
public policy reflected in the Corporation Law, the insurance contracts
executed before a license was secured must be held null and void. The court
ruled that the contracts could not be validated by the subsequent
procurement of the license.
To repeat, the objective of the law was to subject the foreign
corporation to the jurisdiction of our courts. The Corporation Law must be
given a reasonable, not an unduly harsh, interpretation which does not
hamper the development of trade relations and which fosters friendly
commercial intercourse among countries.
Section 133 of the present Corporation Code provides:
SEC. 133. Doing business without a license.-No foreign
corporation transacting business in the Philippines without a
license, or its successors or assigns, shag be permitted to
maintain or intervene in any action, suit or proceeding in any
court or administrative agency in the Philippines; but such
corporation may be sued or proceeded against before Philippine
courts or administrative tribunals on any valid cause of action
recognized under Philippine laws.
The old Section 69 has been reworded in terms of non-access to courts
and administrative agencies in order to maintain or intervene in any action
or proceeding.
The prohibition against doing business without first securing a license is now
given penal sanction which is also applicable to other violations of the
Corporation Code under the general provisions of Section 144 of the Code.

Page 872 of 1072

It is, therefore, not necessary to declare the contract nun and void
even as against the erring foreign corporation. The penal sanction for the
violation and the denial of access to our courts and administrative bodies are
sufficient from the viewpoint of legislative policy.
Our ruling that the lack of capacity at the time of the execution of the
contracts was cured by the subsequent registration is also strengthened by
the procedural aspects of these cases.
The petitioner averred in its complaints that it is a foreign insurance
company, that it is authorized to do business in the Philippines, that its agent
is Mr. Victor H. Bello, and that its office address is the Oledan Building at
Ayala Avenue, Makati. These are all the averments required by Section 4,
Rule 8 of the Rules of Court. The petitioner sufficiently alleged its capacity to
sue. The private respondents countered either with an admission of the
plaintiff's jurisdictional averments or with a general denial based on lack of
knowledge or information sufficient to form a belief as to the truth of the
averments.
We find the general denials inadequate to attack the foreign
corporations lack of capacity to sue in the light of its positive averment that
it is authorized to do so. Section 4, Rule 8 requires that "a party desiring to
raise an issue as to the legal existence of any party or the capacity of any
party to sue or be sued in a representative capacity shall do so by specific
denial, which shag include such supporting particulars as are particularly
within the pleader's knowledge. At the very least, the private respondents
should have stated particulars in their answers upon which a specific denial
of the petitioner's capacity to sue could have been based or which could
have supported its denial for lack of knowledge. And yet, even if the
plaintiff's lack of capacity to sue was not properly raised as an issue by the
answers, the petitioner introduced documentary evidence that it had the
authority to engage in the insurance business at the time it filed the
complaints.

Page 873 of 1072

TOPIC:DOING BUSINESS WITH OR WITHOUT A LICENSE: SUITS BY OR


AGAINST
FOREIGN CORPORATIONS
HUTCHISON PORTS PHILIPPINES LIMITED
VS.
SUBIC BAY METROPOLITAN AUTHORITY
G.R. No. 131367, August 31, 2000
FACTS:
Herein petitioner was selected by SBMA together with ICTSI and RPSI,
as one of the three bidders for pre-qualification in connection with SBMAs
newspaper advertisement inviting the private sector to develop and operate
a modern marine container terminal within the Subic Bay Freeport Zone.
However, even before the opening of the sealed bids, RPSI formally protested
that ICTSI is legally barred from operating a second port in the Philippines
based on Executive Order No. 212 and Department of Transportation and
Communication (DOTC) Order 95-863. RPSI thus requested that the financial
bid of ICTSI should be set aside.
Nevertheless, the opening of the sealed financial bids, more
particularly the royalty fee proceeded under advisement relative to the
protest signified by RPSI. ICTSI emerged as the highest bidder.
On August 15, 1996, the SBMA-PBAC issued a resolution rejecting the
bid of ICTSI because said bid does not comply with the requirements of the
tender documents and the laws of the Philippines, and eventually awarded
the project to HPPL.
Notwithstanding the SBMA Boards recommendations and action awarding
the project to HPPL, then Executive Secretary Ruben Torres submitted a
memorandum to the Office of the President recommending that another
rebidding be conducted. Consequently, the Office of the President issued a
Memorandum directing the SBMA Board of Directors to refrain from signing
the Concession Contract with HPPL and to conduct a rebidding of the project.
On July 7, 1997, the HPPL, feeling aggrieved by the SBMAs failure and
refusal to commence negotiations and to execute the Concession Agreement
despite its earlier pronouncements that HPPL was the winning bidder, filed a
complaint against SBMA before the Regional Trial Court (RTC) of Olongapo
City, Branch 75, for specific performance, mandatory injunction and
damages.
ISSUES:

Page 874 of 1072

Whether or not petitioner HPPL has the legal capacity to even


seek redress from Philippine courts, it being a foreign
corporation.
RULING:
Admittedly, petitioner HPPL is a foreign corporation, organized and
existing under the laws of the British Virgin Islands. While the actual bidder
was a consortium composed of petitioner, and two other corporations,
namely, Guoco Holdings (Phils.) Inc. and Unicol Management Services, Inc., it
is only petitioner HPPL that has brought the controversy before the Court,
arguing that it is suing only on an isolated transaction to evade the legal
requirement that foreign corporations must be licensed to do business in the
Philippines to be able to file and prosecute an action before Philippines
courts.
The maelstrom of this issue is whether participating in the bidding is a mere
isolated transaction, or did it constitute engaging in or transacting
business in the Philippines such that petitioner HPPL needed a license to do
business in the Philippines before it could come to court.
There is no general rule or governing principle laid down as to what
constitutes doing or engaging in or transacting business in the
Philippines.
Each case must be judged in the light of its peculiar
circumstances. Thus, it has often been held that a single act or transaction
may be considered as doing business when a corporation performs acts for
which it was created or exercises some of the functions for which it was
organized. The amount or volume of the business is of no moment, for even
a singular act cannot be merely incidental or casual if it indicates the foreign
corporations intention to do business.
Participating in the bidding process constitutes doing business
because it shows the foreign corporations intention to engage in business
here. The bidding for the concession contract is but an exercise of the
corporations reason for creation or existence. Thus, it has been held that a
foreign company invited to bid for IBRD and ADB international projects in the
Philippines will be considered as doing business in the Philippines for which a
license is required. In this regard, it is the performance by a foreign
corporation of the acts for which it was created, regardless of volume of
business, that determines whether a foreign corporation needs a license or
not.
The primary purpose of the license requirement is to compel a foreign
corporation desiring to do business within the Philippines to submit itself to
the jurisdiction of the courts of the state and to enable the government to
exercise jurisdiction over them for the regulation of their activities in this
country. If a foreign corporation operates a business in the Philippines
without a license, and thus does not submit itself to Philippine laws, it is only

Page 875 of 1072

just that said foreign corporation be not allowed to invoke them in our courts
when the need arises. While foreign investors are always welcome in this
land to collaborate with us for our mutual benefit, they must be prepared as
an indispensable condition to respect and be bound by Philippine law in
proper cases, as in the one at bar. The requirement of a license is not
intended to put foreign corporations at a disadvantage, for the doctrine of
lack of capacity to sue is based on considerations of sound public policy.
Accordingly, petitioner HPPL must be held to be incapacitated to bring this
petition for injunction before this Court for it is a foreign corporation doing
business in the Philippines without the requisite license.

Page 876 of 1072

TOPIC:DOING BUSINESS WITH OR WITHOUT A LICENSE: SUITS BY OR


AGAINST
FOREIGN CORPORATIONS
LA CHEMISE LACOSTE, S. A., Petitioner,
vs.
HON. OSCAR C. FERNANDEZ, Presiding Judge of Branch XLIX,
Regional Trial Court, National Capital Judicial Region, Manila and
GOBINDRAM HEMANDAS, Respondents.
G.R. No. L-65659 May 2l, 1984
GOBINDRAM HEMANDAS SUJANANI, Petitioner,
vs.
HON. ROBERTO V. ONGPIN, in his capacity as Minister of Trade and
Industry, and HON. CESAR SAN DIEGO, in his capacity as Director of
Patents, Respondents.
G.R. No. L-63796-97 May 2, 1984
129 S 373
FACTS:
In 1975, Hemandas & Co., a duly licensed domestic firm applied for
and was issued Reg. No. SR-2225 (SR stands for Supplemental Register) for
the trademark "CHEMISE LACOSTE & CROCODILE DEVICE" by the Philippine
Patent Office for use on T-shirts, sportswear and other garment products of
the company. Two years later, it applied for the registration of the same
trademark under the Principal Register. The Patent Office eventually issued
an order dated March 3, 1977 which states that:
... Considering that the mark was already registered in the
Supplemental Register in favor of herein applicant, the Office has
no other recourse but to allow the application, however, Reg. No.
SR-2225 is now being contested in a Petition for Cancellation
docketed as IPC No. 1046, still registrant is presumed to be the
owner of the mark until after the registration is declared
cancelled.
Thereafter, Hemandas & Co. assigned to respondent Gobindram
Hemandas all rights, title, and interest in the trademark "CHEMISE LACOSTE
& DEVICE".

Page 877 of 1072

Petitioner, on the other hand is a foreign corporation, organized and


existing under the laws of France and not doing business in the Philippines, It
is undeniable from the records that it is the actual owner of the
abovementioned trademarks used on clothing and other goods specifically
sporting apparels sold in many parts of the world and which have been
marketed in the Philippines since 1964, The main basis of the private
respondent's case is its claim of alleged prior registration.
Petitioner filed its application for registration of the trademark
"Crocodile Device" (Application Serial No. 43242) and "Lacoste" (Application
Serial No. 43241).The former was approved for publication while the latter
was opposed by Games and Garments in Inter Partes Case No. 1658. In
1982, the petitioner filed a Petition for the Cancellation of Reg. No. SR-2225
docketed as Inter Partes Case No. 1689.

ISSUE/S:
Whether or not the petitioner company has the capacity to sue despite
the fact that it was not doing business in the Philippines.
RULING:
YES.
The present cause of action by the company is not here seeking to
enforce any legal or control rights arising from, or growing out of, any
business which it has transacted in the Philippine Islands. The sole purpose
of the action: Is to protect its reputation, its corporate name, its goodwill,
whenever that reputation, corporate name or goodwill have, through the
natural development of its trade, established themselves.' And it contends
that its rights to the use of its corporate and trade name: Is a property right,
a right in rem, which it may assert and protect against all the world, in any of
the courts of the world-even in jurisdictions where it does not transact
business-just the same as it may protect its tangible property, real or
personal, against trespass, or conversion. Since it is the trade and not the
mark that is to be protected, a trade-mark acknowledges no territorial
boundaries of municipalities or states or nations, but extends to every
market where the trader's goods have become known and Identified by the
use of the mark.
In upholding the right of the petitioner to maintain the present suit
before our courts for unfair competition or infringement of trademarks of a
foreign corporation, we are moreover recognizing our duties and the rights of
foreign states under the Paris Convention for the Protection of Industrial
Property to which the Philippines and France are parties. We are simply
interpreting and enforcing a solemn international commitment of the

Page 878 of 1072

Philippines embodied in a multilateral treaty to which we are a party and


which we entered into because it is in our national interest to do so.

TOPIC:DOING BUSINESS WITH OR WITHOUT A LICENSE: SUITS BY OR


AGAINST
FOREIGN CORPORATIONS
MARUBENI NEDERLAND B.V., petitioner,
vs.
THE HONORABLE JUDGE RICARDO P. TENSUAN, Presiding Judge of
the Court of First Instance of Rizal, Branch IV, Quezon City and
ARTEMIO GATCHALIAN, respondents.
G.R. No. 61950 September 28, 1990
FACTS:
Petitioner Marubeni Nederland B.V. and D.B. Teodoro Development
Corporation (DBT for short) entered into a contract whereby petitioner
agreed to supply all the necessary equipment, machinery, materials,
technical know-how and the general design of the construction of DBT's lime
plant at the Guimaras Islands in Iloilo for a total contract price of
US$5,400,000.00 on a deferred payment basis. Simultaneously with the
supply contract, the parties entered into two financing contracts, namely a
construction loan agreement in the amount of US$1,600,000.00 and a cash
loan agreement for US$1,500,000.00. The obligation of DBT to pay the loan
amortizations on their due dates under the three (3) contracts were
absolutely and unconditionally guaranteed by the National Investment and
Development Corporation (NIDC).
But before the first installment became due, DBT wrote a letter to the
NIDC interposing certain claims against the petitioner and at the same time
requesting NIDC for a revision of the repayment schedule and of the amounts
due under the contracts on account of petitioners delay in the performance
of its contractual commitments. In due time, the problems regarding the
lime plant were ironed out and the parties signed a "Settlement Agreement"
on July 2, 1981.
However, DBT through counsel, informed petitioner that it was
rejecting the lime plant on the ground that it has not been constructed in
accordance with their agreement. DBT made a formal demand for

Page 879 of 1072

indemnification. In its letter dated June 1, 1982, petitioner refused to accept


DBT's unilateral rejection of the plant and reasoned that the alleged
operation and technical problems were "totally unrelated to the guaranteed
capacity and specifications of the plant and definitely are not attributable to
any fault or omission on the part of Marubeni."
Before the first installment under the "Settlement Agreement" could be paid,
private respondent Artemio Gatchalian, a stockholder of DBT sued petitioner
Marubeni for contractual breach.
ISSUE/S:
Whether or not the court has no jurisdiction over the person of
petitioner since it is a foreign corporation neither doing nor licensed to
do business in the Philippines.
RULING:
Petitioner can be sued in the regular courts because it is doing
business in the Philippines. The applicable law is Republic Act No. 5455 as
implemented by the following rules and regulations of the Board of
Investments which took effect on February 3, 1969. Thus:
xxx xxx xxx
(f) the performance within the Philippines of any act or
combination of acts enumerated in Section 1 (1) of the Act
shall constitute "doing business" therein. In particular,
"doing business" includes:
1) Soliciting orders, purchases (sales) or service contracts.
Concrete and specific solicitations by a foreign firm
amounting to negotiation or fixing of the terms and
conditions of sales or service contracts, regardless of
whether the contracts are actually reduced to writing, shall
constitute doing business even if the enterprise has no
office or fixed place of business in the Philippines. . . .
2) Appointing a representative or distributor who is
domiciled in the Philippines, unless said representative or
distributor has an independent status, i.e., it transacts
business in its name and for its own account, and not in
the name or for the account of the principal.
xxx xxx xxx
4) Opening offices whether called "liaison" offices, agencies
or branches, unless proved otherwise.

Page 880 of 1072

xxx xxx xxx


10) Any other act or acts that imply a continuity of
commercial dealings or arrangements, and contemplate to
that extent the performance of acts or works, or the
exercise of some of the functions normally incident to, or in
the progressive prosecution of, commercial gain or of the
purpose and objective of the business organization.
It cannot be denied that petitioner had solicited the lime plant business
from DBT through the Marubeni Manila branch. Records show that the "turnkey proposal for the . . . 300 T/D Lime Plant" was initiated by the Manila
office through its Mr. T. Hojo. In a follow-up letter dated August 3, 1976, Hojo
committed the firm to a price reduction of $200,000.00 and submitted the
proposed contract forms. As reflected in the letterhead used, it was Marubeni
Corporation, Tokyo, Japan which assumed an active role in the initial stages
of the negotiation. Petitioner Marubeni Nederland B.V. had no visible
participation until the actual signing of the October 28, 1976 agreement in
Tokyo and even there, in the space reserved for petitioner, it was the
signature of "S. Adachi as General Manager of Marubeni Corporation, Tokyo
on behalf of Marubeni Nederland B.V." which appeared.
Even assuming for the sake of argument that Marubeni Nederland B.V.
is a different and separate business entity from Marubeni Japan and its
Manila branch, in this particular transaction, at least, Marubeni Nederland
B.V. through the foregoing acts, had effectively solicited "orders, purchases
(sales) or service contracts" as well as constituted Marubeni Corporation,
Tokyo, Japan and its Manila Branch as its representative in the Philippines to
transact business for its account as principal. These circumstances, taken
singly or in combination, constitute "doing business in the Philippines" within
the contemplation of the law.
At this juncture it must be emphasized that a foreign corporation doing
business in the Philippines with or without license is subject to process and
jurisdiction of the local courts. If such corporation is properly licensed, well
and good. But it shall not be allowed, under any circumstances, to invoke its
lack of license to impugn the jurisdiction of our courts.

Page 881 of 1072

TOPIC:DOING BUSINESS WITH/WITHOUT LICENSE:SUITS BY OR AGAINST


FOREIGN
CORPORATION
PHILIPPINE COLUMBIA ENTERPRISES CO., RUFINO DY CHIN, and
FERMIN DY, petitioners,
vs.
HON. GREGORIO T. LANTIN in his capacity as Judge of the Court of
First Instance of Manila, Br. VII, and KATOH & Co., LTD.,
respondents.
G.R. No. L-29072. June 7, 1971
39 SCRA 376
FACTS:
Private respondent Katoh & Co.,Ltd, alleged in its civil complaint that it
is a corporation duly organized under the laws of Japan, with head office in
Tokyo, Japan. The complaint alleged ten (10) causes of action against the
defendants Philippines Columbia Enterprises Co., with principal place of
business in Manila, and the general partners, thereof, Rufino Dy Chin and
Fermin Sy, who reside in Manila.
These ten(10) causes of action are for the collection of payment of
ten(10) different shipments of angle bars, mild steel bars, and cold rolled
steel sheets allegedly ordered in May, July, October and November, 1966 by
the defendants from the plaintiff which plaintiff had duly shipped and
defendants duly received but which defendant refused to pay.
The complaint does not allege that plaintiff has secured a license to
transact business in the Philippines but its alleges that it has not been and
is not engaged in business in the Philippines and that the transactions
averred in this complaint were exports made and consummated in Tokyo,
Japan in pursuance of international trade.

Page 882 of 1072

ISSUE/S:
Whether plaintiffs allegations in its complaint, particularly in its ten
(10) causes of action, constitute by themselves an admission that it is
transacting business in the Philippines.
RULING:
An examination of complaint will show that the same expressly avers
that the transactions upon which respondent plaintiff is suing were
consummated in Tokyo and hence, not in the Philippines. Petitionersdefendants assertion that the contracts were made in the Philippines
squarely contradicts the averments in the complaint. And the basic and wellknown rule is that whether a cause of action is pleaded or not must be
ascertained solely upon the face of the complaint.
Since the petitioners averment that the plaintiffs transactions were
made in the Philippines, being contradictory of the complaint, can not be set
up in motion to dismiss for lack of cause of action, but must be pleaded in an
answer, any reception of evidence on the point would merely duplicate the
trial on the merits, and should be deferred.
Therefore, the court below committed no abuse of discretion
amounting to excess of jurisdiction in resolving to defer action on motion to
dismiss. The last objection of the petitioners to the deferment order is that if
they file a counterclaim in their answer against respondent foreign
corporation, they would be recognizing the legal capacity of said corporation
which they are precisely questioning. This fear is without legal basis, for
actions by foreign corporations are governed by rules different from those in
actions against them.
A counterclaim partakes of the nature of a complaint and/or cause of
action against the plaintiff, so that if the petitioners-defendants should file a
counterclaim, the private respondent-plaintiff Katoh & Co.,Ltd., would not be
maintaining a suit and, consequently, Section 69 of the Corporation Law
would not apply.

Page 883 of 1072

TOPIC:DOING BUSINESS WITH OR WITHOUT A LICENSE: SUITS BY OR


AGAINST
FOREIGN CORPORATIONS
PHILIP MORRIS, INC.
VS.
FORTUNE TOBACCO
GR NO. 158589, JUNE 27,2006
FACTS:
Petitioner Philip Morris, Inc., a corporation organized under the laws of
the State of Virginia, United States of America, is, per Certificate of
Registration No. 18723 issued on April 26, 1973 by the Philippine Patents
Office (PPO), the registered owner of the trademark MARK VII for
cigarettes. Similarly, petitioner Benson & Hedges (Canada), Inc., a subsidiary
of Philip Morris, Inc., is the registered owner of the trademark MARK TEN
for cigarettes as evidenced by PPO Certificate of Registration No. 11147. And
as can be seen in Trademark Certificate of Registration No. 19053, another
subsidiary of Philip Morris, Inc., the Swiss company Fabriques de Tabac
Reunies, S.A., is the assignee of the trademark LARK, which was originally
registered in 1964 by Ligget and Myers Tobacco Company. On the other
hand, respondent Fortune Tobacco Corporation, a company organized in the
Philippines, manufactures and sells cigarettes using the trademark MARK.
Petitioners then filed a Complaint for Infringement of Trademark and
Damages against respondent Fortune Tobacco Corporation. In the Complaint
with prayer for the issuance of a preliminary injunction, petitioners alleged
that they are foreign corporations not doing business in the Philippines and
are suing on an isolated transaction. They averred that the countries in which
they are domiciled grant to corporate or juristic persons of the Philippines
the privilege to bring action for infringement, without need of a license to do
business in those countries. Petitioners likewise manifested being registered
owners of the trademark MARK VII and MARK TEN
for cigarettes as

Page 884 of 1072

evidenced by the corresponding certificates of registration and an applicant


for the registration of the trademark LARK MILDS. Petitioners claimed that
they have registered the aforementioned trademarks in their respective
countries of origin and that, by virtue of the long and extensive usage of the
same; these trademarks have already gained international fame and
acceptance. Imputing bad faith on the part of the respondent, petitioners
claimed that the respondent, without any previous consent from any of the
petitioners, manufactured and sold cigarettes bearing the identical and/or
confusingly similar trademark MARK Accordingly, they argued that
respondents use of the trademark MARK in its cigarette products have
caused and is likely to cause confusion or mistake, or would deceive
purchasers and the public in general into buying these products under the
impression and mistaken belief that they are buying petitioners products.
ISSUE/S:
Whether or not Petitioner has the legal capacity to sue the respondent
for alleged infringement; and
RULING:
A trademark is any distinctive word, name, symbol, emblem, sign, or
device, or any combination thereof adopted and used by a manufacturer or
merchant on his goods to identify and distinguish them from those
manufactured, sold, or dealt in by others. Inarguably, a trademark deserves
protection.
The Court stated that petitioners standing to sue in Philippine courts
had been recognized. However, that the appellate court qualified its holding
with a statement, following G.R. No. 91332, entitled Philip Morris, Inc., et al.
v. The Court of Appeals and Fortune Tobacco Corporation, that such right to
sue does not necessarily mean protection of their registered marks in the
absence of actual use in the Philippines.
Thus clarified, what petitioners
now harp about is their entitlement to protection on the strength of
registration of their trademarks in the Philippines.
A foreign corporation may have the capacity to sue for infringement
but the question of whether they have an exclusive right over their symbol
as to justify issuance of the controversial writ will depend on actual use of
their trademarks in the Philippines in line with Sections 2 and 2-A of RA116.
It is thus incongruous for petitioners to claim that when a foreign corporation
not licensed to do business in the Philippines files a complaint for
infringement, the entity need not be actually using its trademark in
commerce in the Philippines. Such a foreign corporation may have the
personality to file a suit for infringement but it may not necessarily be
entitled to protection due to absence of actual use of the emblem in the local
market.

Page 885 of 1072

Hence, it may be stated right off that the registration of a trademark


unaccompanied by actual use thereof in the country accords the registrant
only the standing to sue for infringement in Philippine courts. Entitlement to
protection of such trademark in the country is entirely a different matter.

TOPIC:DOING BUSINESS WITH OR WITHOUT A LICENSE: SUITS BY OR


AGAINST
FOREIGN CORPORATIONS
PUMA SPORTSCHUFABRIKEN RUDOLF DASSLER, K.G.
vs.
INTERMEDIATE APPELLATE COURT
G.R. NO. 75067 FEBRUARY 26, 1988
FACTS:
On July 25, 1985, petitioner PUMA SPORTSSCHUFABRIKEN, a German
corporation manufacturing PUMA PRODUCTS, filed a complaint for
infringement of patent or trademark with a prayer for the issuance of a writ
preliminary injunction against the private respondent Mil-Oro Manufacturing
Corp. before the RTC of Makati. On August 19, 1985, the trial court denied
the motion to dismiss and granted the petitioners application for a writ of
injunction. Mil- Oro appealed to the Court of Appeals. On June 23,1986,Court
of Appeals reversed the order of the trial court and ordered the respondent
judge to dismiss the civil case files by the petitioner.
ISSUE/S:
Whether the Court of Appeals erred in holding that petitioner had no
legal capacity to sue.
RULING:

Page 886 of 1072

Petitioner maintains that it has substantially complied with the


requirements of Section 21-A of Republic Act (RA) No. 166, as amended.
According to the petitioner, its complaint specifically alleged that it is not
doing business in the Philippines and is suing under the said Republic Act.
Said Section 21-A requires that the country of which the said corporation or
juristic person is a citizen, or in which it is domiciled, by treaty, convention or
law, grants a similar privilege to corporate or juristic persons of the
Philippines but does not mandatorily require that such reciprocity between
the Federal Republic of Germany and the Philippines be pleaded.
Such reciprocity arrangement is embodied in and supplied by the
Union Convention for the Protection of industrial Property (Paris Convention)
to which both the Philippines and Federal Republic of Germany are
signatories and that since the Paris Convention is a treaty which, pursuant to
our constitution, forms part of the law of the land, our courts are bound to
take judicial notice of such treaty, and, consequently, this fact need not be
averred in the complaint.
The Convention of the Union of Paris for the Protection of Industrial
Property to which the Philippines became a party on September 27, 1965
provides in its Article 8 thereof that a trade name (corporation name) shall
be protected in all the countries of the Union without the obligation of filing
or registration, whether or not it forms part of the trademark.
Thus, petitioner had the legal capacity to file the action below.

Page 887 of 1072

TOPIC:DOING BUSINESS WITH OR WITHOUT A LICENSE: SUITS BY OR


AGAINST
FOREIGN CORPORATIONS
SUBIC BAY METROPOLITAN AUTHORITY
VS.
UNIVERSAL INTERNATIONAL GROUP OF TAIWAN
GR. NO. 131680 SEPTEMBER 14, 2000
FACTS:
In 1995, a Lease and Development Agreement was executed by
respondent UIG and petitioner SBMA under which respondent UIG shall lease
from petitioner SBMA the Binictican Golf Course and appurtenant facilities
thereto to be transformed into a world class 18-hole golf course, golf
club/resort, commercial tourism and residential center. The contract in
pertinent part contains pre-termination clauses, which provides in its Section
22 thereof the acts which constitute what is default.
In 1997, Petitioner SBMA sent a letter to private respondent UIG
calling its attention to its alleged several contractual violations in view of
private respondent UIGs failure to deliver its various contractual obligations,
primarily its failure to complete the rehabilitation of the Golf Course in time
for the APEC Leaders Summit, and to pay accumulated lease rentals and
utilities, and to post the required performance bond. Respondent UIG, in its
letter, interposed as an excuse the alleged default of its main contractor FF
Cruz, resulting in their filing of suit against the latter, and committed itself to
comply with its obligations within a few days. Private respondent UIG,
however, failed to comply with its undertakings. Petitioner SBMA sent a letter
to private respondent UIG declaring the latter in default of its contractual
obligations to SBMA under Section 22.1 of the Lease and Development

Page 888 of 1072

Agreement and required it to show cause why petitioner SBMA should not
pre-terminate the agreement. Private respondents paid the rental arrearages
but the other obligations remained unsatisfied.
ISSUE/S:
Whether or not respondent court committed a reversible error in ruling
that private respondents had the capacity to sue and possess material
interest to institute an action against petitioners.

RULING:
As a general rule, unlicensed foreign non-resident corporations cannot
file suits in the Philippines as provided in Section 133 of the Corporation
Code. A corporation has legal status only within the state or territory in which
it was organized. For this reason, a corporation organized in another country
has no personality to file suits in the Philippines. In order to subject a foreign
corporation doing business in the country to the jurisdiction of our courts, it
must acquire a license from the SEC and appoint an agent for service of
process. Without such license, it cannot institute a suit in the Philippines.
However, that the licensing requirement was never intended to favor
domestic corporations who enter into solitary transactions with unwary
foreign firms and then repudiate their obligations simply because the latter
are not licensed to do business in this country. After contracting with a
foreign corporation, a domestic firm is estopped from denying the formers
capacity to sue. In this case, SBMA is estopped from questioning the
capacity to sue of UIG. In entering into the LDA with UIG, SBMA effectively
recognized its personality and capacity to institute the suit before the trial
court.

Page 889 of 1072

TOPIC:DOING BUSINESS WITH OR WITHOUT A LICENSE: SUITS BY OR


AGAINST
FOREIGN CORPORATIONS
TIME, INC., petitioner,
vs.
HON. ANDRES REYES, as Judge of the Court of First Instance of Rizal,
ELISEO S. ZARI, as Deputy Clerk of Court, Branch VI, Court of First
Instance of Rizal, ANTONIO J. VILLEGAS and JUAN PONCE ENRILE,
respondents.
G.R. No. L-28882. May 31, 1971
39 SCRA 303
FACTS:
This is a petition by Time, Inc. for certiorari and prohibition, with
preliminary injunctions, to annul certain orders of the respondent CFI of Rizal,
issued in its Civil Case No. 10403, entitled Antonio J. Villegas and Juan
Ponce Enrile vs. Time, Inc., and to prohibit the said Rizal court from further
proceeding with the said civil case contending that it is the Manila CFI which
has the jurisdiction.
The petition alleges that the petitioner time, Inc., is an American
Corporation with principal offices at Rockefeller Center, New York City, N.Y.,
and is the publisher of Time, a weekly magazine; the petition, however,
does not alleged the petitioners legal capacity to sue in the courts of the
Philippines.
In said civil case, therein plaintiffs Antonio J. Villegas and Juan Ponce
Enrile seek to recover from the therein petitioner damages upon an alleged
of libel arising from a publication of time(Asia Edition) magazine, in its issue
of 18 August 1967, of an essay, entitled Corruption in Asia.

Page 890 of 1072

ISSUE/S:
Whether the petition will prosper.
RULING:
The dismissal of the present petition is asked on the ground that the
petitioner foreign corporation failed to allege its capacity to sue in the courts
of the Philippines. Respondents rely on Section 69 of the Corporation law,
which provides:
SEC. 69. No foreign corporation formed, organized, or
existing under any laws other than those of the Philippines
shall be permitted to. Maintain by itself or assignee any
suit for the recovery of any debt, claim, or demand
prescribed in the section immediately preceding
They also invoke the ruling in Marshall- Wells Co. vs. Elser and Co., Inc.
46 Phil. 70, that no foreign corporation may be permitted to maintain a suit
in the local courts unless it shall have the license required by the law, and
the ruling in Atlantic Mutual Ins. Co., Inc. vs. Cebu Stevedoring corporation
Inc. 17 SCRA 1037, that where the law denies to a foreign corporation the
right to maintain suit unless it has previously complied with a certain
requirement, then such compliance or the fact that the suing corporation is
exempt there from, becomes a necessary averment in the complaint.
The Court failed to see how these doctrines can be a propos in the case at
bar, since the petitioner is not maintaining any suit but is merely defending
one against itself; it did not file any complaint but only a corollary defensive
petition to prohibit the lower court from further proceeding with a suit that it
had no jurisdiction to entertain.
Petitioners failure to aver its legal capacity to institute the present
petition is not fatal, for
A foreign corporation may by writ of prohibition, seek relief against
the wrongful assumption of jurisdiction. And a foreign corporation seeking a
writ of prohibition against further maintenance a suit, on the ground of want
jurisdiction, is not bound by the ruling of the court in which the suit was
brought, on the motion to quash service of summons, that it has jurisdiction.

Page 891 of 1072

TOPIC:DOING BUSINESS WITH OR WITHOUT A LICENSE: SUITS BY OR


AGAINST
FOREIGN CORPORATIONS
UNIVERSAL RUBBER PRODUCTS, INC., petitioner,
vs.
HON. COURT OF APPEALS, CONVERSE RUBBER CORPORARION,
EDWARDSON MANUFACTURING CO., INC. AND HON. PEDRO C.
NAVARRO, respondents.
G.R. No. L-30266. June 29, 1984
130 SCRA 104
FACTS:
Respondent corporations, Converse Rubber Corp. and Edwardson
Manufacturing Co., sued petitioner Universal Rubber products, Inc. before the
CFI of Rizal for unfair competition with damages and attorneys fees.
Petitioner Universal Rubber Products, Inc. contends that private
respondent Converse Rubber Corp. is a foreign corporation not licensed to do
business in the Philippines and that respondent Edwardson is merely its
licensee; that respondent Converse has no goodwill to speak of and that it
has no registration right over its own name.

ISSUE/S:
Whether respondent Converse Rubber Corp. has capacity to sue.

Page 892 of 1072

RULING:
The Court held that the disability of a foreign corporation from suing
in the Philippines is limited to suits to enforce any legal or contracts rights
arising from, or growing out, of any business which it has transacted in the
Philippine Islands. On the other hand, where the purpose of the suit is to
protect its reputation, its corporate name, its goodwill, whenever that
reputation, its corporate name or goodwill have, through the natural
development of its trade, established themselves, an unlicensed foreign
corporation may sue in the Philippines.
Hence, it is clear that Section 29 of the Corporation Law does not disqualify
plaintiff- appellee Converse Rubber, which does not have a branch office in
any part of the Philippines and is not doing business in the Philippines,
from filing and prosecuting this action of unfair competition. Therefore
Converse Rubber Corp. can file and prosecute the action of unfair
competition.
TOPIC:DOING BUSINESS WITH OR WITHOUT A LICENSE: SUITS BY OR
AGAINST
FOREIGN CORPORATIONS
B. VAN ZUIDEN BROS., LTD.,
VS.
GTVL MANUFACTURING INDUSTRIES, INC.
G.R. NO. 147905 MAY 28, 2007
FACTS:
Petitioner Zuiden, is a corporation, incorporated under the laws of
Hong Kong. Zuiden is not engaged in business in the Philippines, but is suing
before the Philippine Courts, for the reasons hereinafter stated. It is engaged
in the importation and exportation of several products, including lace
products. On several occasions, GTVL purchased lace products from
petitioner.
The procedure for these purchases, as per the instructions of GTVL,
was that Zuiden delivers the products purchased by GTVL, to a certain Hong
Kong corporation, known as Kenzar Ltd. (Kenzar)and the products are then
considered as sold, upon receipt by Kenzar of the goods purchased by GTVL.
Kenzar had the obligation to deliver the products to the Philippines and/or to
follow whatever instructions GTVL had on the matter.
Insofar as Zuiden is concerned, upon delivery of the goods to KENZAR
in Hong Kong, the transaction is concluded; and GTVL became obligated to
pay the agreed purchase price. However, commencing October 31, 1994 up
to the present, GTVL has failed and refused to pay the agreed purchase price
for several deliveries ordered by it and delivered by Zuiden.

Page 893 of 1072

ISSUE/S:
Whether or not petitioner, an unlicensed foreign corporation, has legal
capacity to sue before Philippine courts.
RULING:
Section 133 of the Corporation Code provides that No foreign
corporation transacting business in the Philippines without a license, or its
successors or assigns, shall be permitted to maintain or intervene in any
action, suit or proceeding in any court or administrative agency of the
Philippines; but such corporation may be sued or proceeded against before
Philippine courts or administrative tribunals on any valid cause of action
recognized under Philippine laws. An unlicensed foreign corporation doing
business in the Philippines cannot sue before Philippine courts. On the other
hand, an unlicensed foreign corporation not doing business in the Philippines
can sue before Philippine courts.
In the present case, the series of transactions between petitioner and
respondent cannot be classified as "doing business" in the Philippines under
Section 3(d) of RA 7042. An essential condition to be considered as "doing
business" in the Philippines is the actual performance of specific commercial
acts within the territory of the Philippines for the plain reason that the
Philippines has no jurisdiction over commercial acts performed in foreign
territories. Here, there is no showing that petitioner performed within the
Philippine territory the specific acts of doing business mentioned in Section
3(d) of RA 7042. Petitioner did not also open an office here in the Philippines,
appoint a representative or distributor, or manage, supervise or control a
local business. While petitioner and respondent entered into a series of
transactions implying a continuity of commercial dealings, the perfection and
consummation of these transactions were done outside the Philippines.
Further, the series of transactions between petitioner and respondent
transpired and were consummated in Hong Kong. There was no single
activity which petitioner performed here in the Philippines pursuant to its
purpose and object as a business organization. Moreover, petitioners desire
to do business within the Philippines is not discernible from the allegations of
the complaint or from its attachments. Therefore, there is no basis for ruling
that petitioner is doing business in the Philippines.
In resume, to be doing or "transacting business in the Philippines" for
purposes of Section 133 of the Corporation Code, the foreign corporation
must actually transact business in the Philippines, that is, perform specific
business transactions within the Philippine territory on a continuing basis in
its own name and for its own account. Actual transaction of business within
the Philippine territory is an essential requisite for the Philippines to acquire

Page 894 of 1072

jurisdiction over a foreign corporation and thus require the foreign


corporation to secure a Philippine business license. If a foreign corporation
does not transact such kind of business in the Philippines, even if it exports
its products to the Philippines, the Philippines has no jurisdiction to require
such foreign corporation to secure a Philippine business license.
Considering that petitioner is not doing business in the Philippines, it
does not need a license in order to initiate and maintain a collection suit
against respondent for the unpaid balance of respondents purchases.

TOPIC: POWERS AND FUNCTIONS OF SEC


SECURITIES AND EXCHANGE COMMISSION
VS.
PERFORMANCE FOREIGN EXCHANGE CORPORATION
G.R. NO. 154131
JULY 20, 2006
FACTS:
Respondent Performance Foreign Exchange Corporation(PFEC) is a
domestic corporation duly registered under Securities and Exchange
Commission (SEC) with its primary purpose to operate as a broker/agent
between market participants in transactions involving, but not limited to,
foreign exchange, deposits, interest rate instruments, fixed income
securities, bonds/bills, repurchased agreements of fixed income securities,
certificate of deposits, bankers acceptances, bills of exchange, over-thecounter option of the aforementioned instruments, Lesser Developed
Countrys (L.D.C.) debt, energy and stock indexes and all related, similar or
derivative products, other than acting as a broker for the trading of securities
pursuant to the Revised Securities Act of the Philippines. Its secondary
purpose is to engage in money changer or exchanging foreign currencies into
domestic currency, Philippine currency or other foreign currencies into
another currency.
After two years of operation, respondent received a letter from the
SEC, requiring it to appear before the Compliance and Enforcement
Department (CED) on December 14, 2000 for a clarificatory conference
regarding its business operations. Respondents officers complied and
explained before the CED the nature of their business. In 2001, Emilio B.
Aquino, Director of CED, issued a Cease and Desist Order in CED Case No.
99-2297, stating that his department conducted an inquiry on respondents

Page 895 of 1072

business operations for possible violation of Republic Act (R.A.) No.


8799,otherwise known as The Securities Regulation Code; that the outcome
of the inquiry shows that respondent is engaged in the trading of foreign
currency futures contracts in behalf of its clients without the necessary
license; that such transaction can be deemed as a direct violation of Section
11 of R.A. No. 87994 and the related provisions of its Implementing Rules and
Regulations; and that it is imperative to enjoin respondent from further
operating as such to protect the interest of the public.

ISSUE/S:
Whether or not petitioner SEC has acted with grave abuse of discretion
in issuing the Cease and Desist Order and its subsequent Order making
it permanent.

RULING:
Under Section 64 of R.A. No. 8799, it provides that upon complaint by
any aggrieved party, may issue a cease and desist order without the
necessity of a prior hearing if in its judgment the act or practice, unless
restrained, will operate as a fraud on investors or is otherwise likely to cause
grave or irreparable injury or prejudice to the investing public.
Under the above provision, there are two essential requirements that
must be complied with by the SEC before it may issue a cease and desist
order: First, it must conduct proper investigation or verification; and Second,
there must be a finding that the act or practice, unless restrained, will
operate as a fraud on investors or is otherwise likely to cause grave or
irreparable injury or prejudice to the investing public.

In the present case, the first requirement is not present. Petitioner did
not conduct proper investigation or verification before it issued the
challenged orders. The clarificatory conference undertaken by petitioner
regarding respondents business operations cannot be considered a proper
investigation or verification process to justify the issuance of the Cease and
Desist Order. It was merely an initial stage of such process, considering that
after it issued the said order following the clarificatory conference, petitioner
still sought verification from the BSP on the nature of respondents business
activity.
Petitioners act of referring the matter to the BSP is an essential part of
the investigation and verification process. In fact, such referral indicates that
petitioner concedes to the BSPs expertise in determining the nature of

Page 896 of 1072

respondents business. It bears stressing, however, that such investigation


and verification, to be proper, must be conducted by petitioner before, not
after, issuing the Cease and Desist Order in question. This, petitioner utterly
failed to do. The issuance of such order even before it could finish its
investigation and verification on respondents business activity obviously
contravenes Section 64 of R.A. No. 8799 earlier quoted.
Thus, the cease and desist order stays against the corporation until
the latter shall be able to submit the appropriate endorsement from the
Bangko Sentral ng Pilipinas that it can engage in financial derivative
transactions. Moreover, before a cease and desist order may be issued by
the SEC, there must be a showing that the act or practice sought to be
restrained will operate as a fraud on investors or is likely to cause grave,
irreparable injury or prejudice to the investing public. Such requirement
implies that the act to be restrained has been determined after conducting
the proper investigation/verification. In this case, the nature of the act to be
restrained can only be determined after the BSP shall have submitted its
findings to petitioner. However, there is nothing in the questioned Orders
that shows how the public is greatly prejudiced or damaged by respondents
business operation.

TOPIC: POWERS AND FUNCTIONS OF SEC


JESUS LIM ARRANZA ET.AL
VS.
B. F. HOMES, INC
G.R. NO. 131683
JUNE 19, 2000
FACTS:
Respondent BF Homes, Inc. (BFHI), is a domestic corporation engaged
in developingsubdivisions and selling residential lots. One of the subdivisions
that respondent developed was the BF Homes Paraaque Subdivision, which
now sprawls across not only a portion of the City of Paraaque but also those
of the adjoining cities of Las Pias and Muntinlupa.
When the Central Bank ordered the closure of Banco Filipino, which had
substantial investments in respondent BFHI, respondent filed with the SEC a
petition for rehabilitation and a declaration that it was in a state of
suspension of payments. On 18 March 1985, the SEC placed respondent
under a management committee. Upon that committees dissolution on 2
February 1988, the SEC appointed Atty. Florencio B. Orendain as a Receiver,
and approved a Revised Rehabilitation Plan.
As a Receiver, Orendain instituted a central security system and
unified the sixty~five homeowners associations into an umbrella
homeowners association called United BF Homeowners Associations, Inc.

Page 897 of 1072

(UBFHAI), which was thereafter incorporated with the Home Insurance and
Guaranty Corporation (HIGC). In 1989, respondent, through Orendain, turned
over to UBFHAI control and administration of security in the subdivision, the
Clubhouse and the open spaces along Concha Cruz Drive. Through the
Philippine Waterworks and Construction Corporation (PWCC), respondents
managing company for waterworks in the various BF Homes subdivisions,
respondent entered into an agreement with UBFHAI for the annual collection
of community assessment fund and for the purchase of eight new pumps to
replace the over~capacitated pumps in the old wells.
In 1994, Orendain was relieved by the SEC of his duties as a Receiver,
and a new Board of Receivers consisting of eleven members of respondents
Board of Directors was appointed for the implementation of Phases II and III
of respondents rehabilitation. The new Board, through its Chairman, Albert
C. Aguirre, revoked the authority given by Orendain to use the open spaces
at Concha Cruz Drive and to collect community assessment funds; deferred
the purchase of new pumps; recognized BF Paraaque Homeowners
Association, Inc., (BFPHAI) as the representative of all homeowners in the
subdivision; took over the management of the Clubhouse; and deployed its
own security guards in the subdivision.
Consequently, in 1995, petitioners filed with the HLURB a class suit "for
and in behalf of the more than 7,000 homeowners in the subdivision" against
respondent BFHI, BF Citiland Corporation, PWCC and A.C. Aguirre
Management Corporation "to enforce the rights of purchasers of lots" in BF
Homes Paraaque.
Petitioners raised "issues" on the following basic needs of the
homeowners: rights~of~way; water; open spaces; road and perimeter wall
repairs; security; and the interlocking corporations that allegedly made it
convenient for respondent "to compartmentalize its obligations as general
developer, even if all of these are hooked into the water, roads, drainage and
sewer systems of the subdivision."

ISSUE/S:
Whether or not the Securities and Exchange Commission has
jurisdiction to resolve the present controversy.
RULING:
The fact that respondent is under receivership does not divest the
HLURB of that jurisdiction. A receiver is a person appointed by the court, or in
this instance, by a quasi~judicial administrative agency, in behalf of all the
parties for the purpose of preserving and conserving the property and
preventing its possible destruction or dissipation, if it were left in the

Page 898 of 1072

possession of any of the parties. The appointment of a receiver does not


dissolve a corporation, nor does it interfere with the exercise of its corporate
rights. In this case where there appears to be no restraints imposed upon
respondent as it undergoes rehabilitation receivership, respondent continues
to exist as a corporation and hence, continues or should continue to perform
its contractual and statutory responsibilities to petitioners as homeowners.
No violation of the SEC order suspending payments to creditors would
result as far as petitioners complaint before the HLURB is concerned. Since
what petitioners seek to enforce are respondents obligations as a
subdivision developer. Such claims are basically not pecuniary in nature
although it could incidentally involve monetary considerations. All that
petitioners claims entail is the exercise of proper subdivision management
on the part of the SEC~appointed Board of Receivers towards the end that
homeowners shall enjoy the ideal community living that respondent
portrayed they would have when they bought real estate from it. It can not
also be considered as heving claims against respondent.
On the other hand, the jurisdiction of the SEC is defined by P.D. No.
902~A, as amended and for the SEC to acquire jurisdiction over any
controversy under these provisions, two elements must be considered:
(1) the status or relationship of the parties; and
(2) the nature of the question that is the subject of their
controversy.
The first element requires that the controversy must arise "out of
intra~corporate or partnership relations between and among stockholders,
members or associates; between any or all of them and the corporation,
partnership or association of which they are stockholders, members or
associates, respectively; and between such corporation, partnership or
association and the State in so far as it concerns their individual franchises."
Petitioners are not stockholders, members or associates of respondent. They
are lot buyers and now homeowners in the subdivision developed by the
respondent.
The second element requires that the dispute among the parties be
intrinsically connected with the regulation or the internal affairs of the
corporation, partnership or association. The controversy in this case is
remotely related to the "regulation" of Respondent Corporation or to
respondents "internal affairs."

Page 899 of 1072

TOPIC: POWERS AND FUNCTIONS OF SEC


WILLIAM H. QUASHA
VS.
SECURITIES AND EXCHANGE COMMISSION
G.R. NO. L-47536
MAY 31, 1978
FACTS:
Petitioner had filed on October 10, 1977 and October 17, 1977 his
complaints and continuing opposition with respondent commission against
the filing of respondent Manila Polo Club, Inc.'s Amended Articles of
Incorporation and Amended By-Laws which would convert said club into a
proprietary club, assailing the amendments as illegal, inequitable and
immoral, alleging inter alia that the amendments have the effect of enabling
the members to appropriate the club's property and to use it as their
contribution to the new club; the real estate assets of Manila Polo Club
consist of 25 hectares, more or less, of prime real estate in 'he middle of
Forbes lark Makati, Metro Manila. which are conservatively valued at present
market valuation of P200 Million and its buildings, improvements, facilities
and other equipments at about P20 Million.

Page 900 of 1072

The more or less 2,000 actual members who will become proprietary
owners of the Club's assets under the proposed conversion will inequitably
enrich themselves if this Honorable Commission will allow the comparatively
paltry of P12,500.00 to be paid for each proprietary membership;" "the value
which the Club now commands results from The accrued contribution of past
(and present) members' money, time, effort and foresight; and the
conversion plan does not in any way compensate the predecessors of the
present membership (and also those of the present membership who do not
opt for conversion) who substantially contributed to making the Club what it
is today" and further claiming that the amendments had not been duly
adopted by the required two-thirds vote. Petitioner prayed for the
disapproval and cancellation of respondent club's amended articles and bylaws and denial of its application to register its proprietary shares, and
prayed for a restraining order meanwhile enjoining it from selling and/or
accepting any payments for the questioned proprietary shares.
Respondent club on the other hand issued notices dated October 25, 1977
fixing December 28, 1977 as the deadline for members to purchase a
proprietary share for P12,500.00 cash in one lump sum or in 24 monthly
installments with 14% interest per annum and a P5,000 downpayment and
giving the non-buyers members the choice of remaining in the club as
Associate Members by informing the club to this effect.
ISSUE/S:
Whether or not the hearing officer of the SEC is empowered to issue
the questioned order denying the injunctive relief.
RULING:
The Court ruled that in view of the extremely limited time, with the
commission's hearing officer having issued his questioned order denying
injunctive relief only on December 22, 1977 at the height of the Christmas
holiday with just a few days before The scheduled deadline of December 28,
1977, petitioner properly filed the present petition directly with this Court
without going through the prescribed procedure of filing an appeal with
respondent Commission en banc within the 30-day reglementary period
since such recourse was obviously not a plain, speedy and adequate remedy.
The questions raised by petitioner in his pending complaints with
respondent commission warrant a full-blowing trial' on the merits" after
which the main issues may be duly adjudicated as contended by him, and
since respondents likewise concur in this stand, the case will be remanded to
respondent commission for such trial and determination on the merits.

Page 901 of 1072

Finally, petitioner has not satisfactorily established his right to the


restraining order prayed for. Considering that petitioner submitted the
incident on the basis of his memorandum without presentation of evidence,
the Court holds that respondent commission did not act with grave abuse of
discretion in denying the restraining order prayed for.

TOPIC: POWERS AND FUNCTIONS OF SEC


TRADERS ROYAL BANK
VS.
COURT OF APPEALS
G.R. No. L-78412
September 26, 1989
FACTS:
On March 30,1982, the Philippine Blooming Mills, Inc. (PBM) and
Alfredo Ching jointly submitted to the Securities and Exchange Commission a
petition for suspension of payments (SEC No. 2250) where Alfredo Ching was
joined as co-petitioner because under the law, he was allegedly entitled, as
surety, to avail of the defenses of PBM and he was expected to raise most of
the stockholders' equity of Pl00 million being required under the plan for the
rehabilitation of PBM. Traders Royal Bank was included among PBM's
creditors named in Schedule A accompanying PBM's petition for suspension
of payments.

Page 902 of 1072

On May 13, 1983, the petitioner bank filed a case against PBM and
Alfredo Ching, to collect P22,227,794.05 exclusive of interests, penalties and
other bank charges representing PBM's outstanding obligation to the bank.
Alfredo Ching, a stockholder of PBM, was impleaded as co-defendant for
having signed as a surety for PBM's obligations to the extent of ten million
pesos (Pl0,000,000) under a Deed of Suretyship dated July 21, 1977.
In its en banc decision, the SEC declared that it had assumed
jurisdiction over petitioner Alfredo Ching pursuant to Section 6, Rule 3 of the
new Rules of Procedure of the SEC providing that "parties in interest without
whom no final determination can be had of an action shall be joined either as
complainant, petitioner or respondent" to prevent multiplicity of suits.
On July 9, 1982, the SEC issued an Order placing PBM's business,
including its assets and liabilities, under rehabilitation receivership, and
ordered that "all actions for claims listed in Schedule A of the petition
pending before any court or tribunal are hereby suspended in whatever
stage the same may be, until further orders from the Commission". As
directed by the SEC, said order was published once a week for three
consecutive weeks in the Bulletin Today, Philippine Daily Express and Times
Journal at the expense of PBM and Alfredo Ching.
PBM and Ching jointly filed a motion to dismiss Civil Case No. 1028-P in
the RTC, Pasay City, invoking the pendency in the SEC of PBM's application
for suspension of payments (which Ching co-signed) and over which the SEC
had already assumed jurisdiction. Before the motion to dismiss could be
resolved, the court dropped PBM from the complaint, on motion of the
plaintiff bank, for the reason that the SEC had already placed PBM under
rehabilitation receivership. On August 15, 1983, the trial court denied Ching's
motion to dismiss the complaint against himself.
ISSUE/S:
Whether the court a quo could acquire jurisdiction over Ching in his
personal and individual capacity as a surety of PBM in the collection
suit filed by the bank, despite the fact that PBM's obligation to the
bank had been placed under receivership by the SEC?
RULING:
Although Ching was impleaded in SEC Case No. 2250, as a copetitioner of PBM, the SEC could not assume jurisdiction over his person and
properties. The Securities and Exchange Commission was empowered, as
rehabilitation receiver, to take custody and control of the assets and
properties of PBM only, for the SEC has jurisdiction over corporations only not
over private individuals, except stockholders in an intra-corporate dispute

Page 903 of 1072

(Sec. 5, P.D. 902-A and Sec. 2 of P.D. 1758). Being a nominal party in SEC
Case No. 2250, Ching's properties were not included in the rehabilitation
receivership that the SEC constituted to take custody of PBM's assets.
Therefore, the petitioner bank was not barred from filing a suit against Ching,
as a surety for PBM. An anomalous situation would arise if individual sureties
for debtor corporations may escape liability by simply co- filing with the
corporation a petition for suspension of payments in the SEC whose
jurisdiction is limited only to corporations and their corporate assets.

TOPIC: POWERS AND FUNCTIONS OF SEC


VICMAR DEVELOPMENT
VS.
COURT OF APPEALS
G.R. No. 81547
May 21, 1990
FACTS:
Sometime in August, 1982, a conflict arose between petitioner Vicente
Angliongto and private respondent Rufino T. Nasser on the matter of
exclusive control and management of Petitioner Corporation. On July 7, 1983,
petitioner Corporation by petitioner Angliongto, filed a verified petition in the
public respondent SEC against private respondent Nasser, alleging, that
private respondent Nasser was a Director, Executive Vice-President and
General Manager of petitioner Corporation from 1974 to August 26, 1982 but
during the annual meeting of stockholders of petitioner corporation held on

Page 904 of 1072

August 26, 1982, private respondent Nasser was not re-elected as member
of the Board of Directors or to his previous management positions.
In view of the result of the annual stockholders' meeting, private
respondent Nasser was then advised by the incoming president, herein
petitioner Angliongto that the latter would actively manage the corporate
affairs of Petitioner Corporation. In view thereof, private respondent Nasser
was asked to turn over all corporate books and records in his possession to
the duly elected officers, among others, which demand remained (un)heeded
by private respondent Nasser as the latter continued to hold office as
Executive Vice-President and General Manager of petitioner Corporation,
performing acts and entering into transactions inimical to the interests of the
petitioner Corporation and its stockholders. Said petition also prayed for the
issuance of a restraining order and thereafter, a permanent injunction to
enjoin private respondent Nasser from representing himself as an officer of
petitioner Corporation, among other things, and for him, to surrender all
corporate books and records to the duly elected officers of said Corporation.
On September 19, 1983, after due hearing in the aforesaid case
respondent SEC, through respondent Hearing Officer Alberto Atas, issued a
Writ of Preliminary Injunction, enjoining private respondent Nasser from
acting as, and/or representing himself to be, the Executive Vice-President
and/or General Manager and/or officer in any capacity of petitioner
Corporation.

ISSUE/S:
Whether or not the Securities and Exchange Commission has abused
its discretion in recalling its Order to enforce a writ of preliminary
injunction?

RULING:
The dispute between petitioner Vicmar Development Corporation and
Vicente Angliongto on the one hand and private respondent Rufino Nasser on
the other, for the exclusive control and management of petitioner
Corporation, triggered off the filing of SEC Case No. 2490, an intracorporate
controversy over which the Securities and Exchange Commission has original
and exclusive jurisdiction under Presidential Decree No. 902-A.
The facts reveal that the writ of preliminary injunction issued on
September 19, 1983 enjoined private respondent Nasser from acting as,
and/or representing himself to be, the Executive Vice-President and/or
General Manager and/or officer in any capacity of petitioner Corporation.
Upon presentment of the Agreement dated November 10, 1983 showing a

Page 905 of 1072

transfer of ownership, control and management of Vicmar Corporation by


Vicente Angliongto unto Nasser, the SEC correctly recalled the order of March
5, 1986 directing the implementation of the aforesaid writ, pending hearing
on the motion dated March 17, 1986. To allow execution of the writ of
preliminary injunction in favor of the petitioners despite having transferred
their rights of ownership, control and management over said corporation to
respondent Nasser would be baseless, the contract having shown prima
facie that the latter is entitled to remain as Vice-President and General
Manager of petitioner Corporation.
Thus, no grave abuse of discretion can be attributed to the SEC in
recalling the order to enforce a writ of preliminary injunction in this wise.
After all, the issuance or recall of preliminary writ of injunction is an
interlocutory matter that remains at all times within the control of the court
(Alvaro v. Zapata, 118 SCRA 728 [1982]). The grant or denial of an injunction
rests upon the sound discretion of the lower tribunal, in the exercise of which
the Supreme Court will not interfere except in a clear case of abuse.

TOPIC: ORIGINAL ANG EXCLUSIVE JURISDICTION OF THE RTC


FLORENCIO ORENDAIN, petitioner,
vs.
BF HOMES, INC., respondent.
G.R. No. 146313. October 31, 2006
FACTS:
BF Homes, Inc. availed financial assistance from various sources to buy
properties and convert them into residential subdivisions. Despite its solvent
status, it filed a Petition for Rehabilitation and for Declaration in a State of
Suspension of Payments. SEC ordered the appointment of a rehabilitation
receiver, FBO Management Networks, Inc., with Orendain as Chairman to
prevent paralyzation of BF Homes business operations. Deed of Absolute
Sale was executed by and between BF Homesrepresented by Orendainas

Page 906 of 1072

absolute and registered owner, and the Local Superior of the Franciscan
Sisters of the Immaculate Phils., Inc. (LSFSIPI) over a parcel of land.
BF Homes filed a Complaint with the RTC against LSFSIPI and Orendain
for reconveyance of the property alleging that the LSFSIPI transacted with
Orendain in his individual capacity and therefore, neither FBO Management,
Inc. nor Orendain had title to the property transferred. Moreover, it averred
that the selling price was grossly inadequate or insufficient amounting to
fraud and conspiracy with the LSFSIPI.
ISSUE/S:
Whether a simple reconveyance suit is within the jurisdiction of the RTC
or SEC?
RULING:
It is the RTC which has jurisdiction. Clearly, the controversy involves
matters purely civil in character and is beyond the ambit of the limited
jurisdiction of the SEC. The better policy in determining which body has
jurisdiction over a case would be to consider not only [1] the status or
relationship of the parties but also [2] the nature of the question that is the
subject of their controversy.
More so, the first element requires that the controversy must arise out
of intra-corporate or partnership relations between any or all of the parties
and the corporation, partnership or association of which they are
stockholders, members or associates; between any or all of them and the
corporation, partnership or association of which they are stockholders,
members or associates, respectively; and between such corporation,
partnership or association and the State insofar as it concerns their
individual franchises. The second element requires that the dispute among
the parties be intrinsically connected with the regulation of the corporation.
If the nature of the controversy involves matters that are purely civil in
character, necessarily, the case does not involve an intra-corporate
controversy. The determination of whether a contract is simulated or not is
an issue that could be resolved by applying pertinent provisions of the Civil
Code. Section 5 of PD No. 902-A does not apply in the instant case. The
LSFSIPI is neither an officer nor a stockholder of BF Homes, and this case
does not involve intra-corporate proceedings. In addition, the seller
Orendain, is being sued in his individual capacity for the unauthorized sale of
the property in controversy. In addition, jurisdiction over the case for
reconveyance is clearly vested in the RTC as provided in paragraph (2),
Section 19, B.P. Blg. 129.

Page 907 of 1072

TOPIC: ORIGINAL ANG EXCLUSIVE JURISDICTION OF THE RTC


ALFREDO P. PASCUAL and LORETA S. PASCUAL, petitioners,
vs.
COURT OF APPEALS (former Seventh Division), ERNESTO P. PASCUAL
and HON. ADORACION ANGELES, in her capacity as Presiding Judge,
RTC, Kaloocan City, Branch 121, respondents.
G.R. No. 138542. August 25, 2000
FACTS:
Defendant Ernesto and Plaintiff Alfredo Pascual are full blood brothers.
Petitioner Loreta Pascual is the wife of Alfredo. Between 1963 to 1975,
Luciano R. Pascual, Sr. acquired substantial shares in Phillens Manufacturing
Corp. Luciano, Sr. parceled out and assigned a good number of these shares

Page 908 of 1072

in the names of his children. With Lucianos substantial shareholdings, his


eldest son, Alfredo became President, General Manager, and Vice-Chairman
of the Board of Phillens. Defendant was only 20 years old then. Alfredo was
also president of L.R. Pascual & Sons, Inc. which held substantial shares in
Phillens. (Defendant is a stockholder of L. R. Pascual & Sons, Inc.) Although
during and after the lifetime of the parties parents, Alfredo held family
property in trust for Luciano Sr. and Consolacion, and for his brothers and
sisters, Alfredo gave the latter no accounting at any point in time contrary to
what their father intended. Because from 1969 to 1990, Alfredo turned over
zero profit to Ernesto as far as his share was concerned, defendant tried to
arrange a meeting between them about the matter of accounting -- without
any success during a 5-year period (1990-1995). Alfredo would each time be
sensitive, evasive, and drunk, so nothing became of those efforts. Several
fraudulent transactions were then entered into by Alfredo by virtue of his
office.
On March 21, 1996, petitioners filed a motion to dismiss on the ground
that the complaint raises an intra-corporate controversy between the parties
over which original and exclusive jurisdiction is vested in the Securities and
Exchange Commission (SEC). At first, the trial court granted petitioners
motion and dismissed the complaint on the ground that the complaint
stemmed from alleged fraudulent acts and misrepresentations of petitioner
Alfredo P. Pascual as a corporate officer of Phillens Manufacturing Corp.
(Phillens) and thus the SEC had jurisdiction over the case. However, on
respondents motion, the trial court reconsidered its order and reinstated
respondents action. In an order, dated September 29, 1997, the trial court
held that, since the corporation had been dissolved in 1990 and its corporate
affairs terminated in 1993, there were no more corporate affairs to speak of
at the time of the filing of the complaint.
ISSUE/S:
What court has the proper jurisdiction over the case?
RULING:
The trial court and not the SEC has jurisdiction over the case. Sec. 5(b)
does not define what an intra-corporate controversy is, but case law has
fashioned out two tests for determining what suit is cognizable by the SEC or
the regular courts, and sometimes by the National Labor Relations
Commission. The first test uses the enumeration in Sec. 5(b) of the
relationships to determine jurisdiction, to wit:
1) Those between and among stockholders and members;
2) Those between and among stockholders and members, on
one hand, and the corporation, on the other hand; and

Page 909 of 1072

3) Those between the corporation and the State but only


insofar as its franchise or right to exist as an entity is
concerned.
The second test, on the other hand, focuses on the nature of the
controversy itself. Recent decisions of this Court consider not only the
subject of their controversy but also the status of the parties.
The Court of Appeals correctly ruled that the regular courts, not the
SEC, have jurisdiction over this case. Petitioners and private respondent
never had any corporate relations in Phillens. It appears that private
respondent was never a stockholder in Phillens, of which the parties
predecessor-in-interest, Luciano Pascual, Sr., was a stockholder and whose
properties are being litigated. Private respondents allegation is that, upon
the death of their father, he became co-owner in the estate left by him, and
part of this estate includes the corporate interests in Phillens. He also
alleges that petitioners repudiated the trust relationship created between
them and appropriated to themselves even the property that should have
belonged to respondent. It is thus clear that there is no corporate
relationship involved here. That petitioner Alfredo Pascual was a corporate
officer holding in trust for his brother their fathers corporate interests did
not create an intra-corporate relationship between them.
Nor is the controversy corporate in nature. As we have stated before,
the grant of jurisdiction must be viewed in the light of the nature and
function of the SEC under the law. P.D. No. 902-A, Sec. 3 gives the SEC
jurisdiction, supervision, and control over all corporations, partnerships or
associations, who are the grantees of primary franchise and/or a license or
permit issued by the government to operate in the Philippines. From this, it
can be deduced that the regulatory and adjudicatory functions of the SEC,
insofar as intra-corporate controversies are concerned, comes into play only
if a corporation still exists.
In the case at bar, the corporation whose properties are being
contested no longer exists, it having been completely dissolved in 1993;
consequently, the supervisory authority of the SEC over the corporation has
likewise come to an end. It is true that a complaint for accounting,
reconveyance, etc. of corporate properties has previously been held to be
within the jurisdiction of the SEC. There is no question that assessing the
financial status of an existing corporation, for purposes of an action for
accounting, requires the expertise of the SEC. But in the case of a dissolved
corporation, no such expertise is required, for all its business has been
properly accounted for already, and what is left to be determined is properly
within the competence of regular courts.
It may be noted in this connection that pursuant to R.A. No. 8799, Sec.
5.2, which took effect on August 8, 2000, the jurisdiction of the SEC to decide

Page 910 of 1072

cases involving intra-corporate dispute was transferred to courts of general


jurisdiction and, in accordance therewith, all cases of this nature, with the
exception only of those submitted for decision, were transferred to the
regular courts. Hence, the question whether this case should be filed in the
SEC is now only of academic interest. For even if it involves an intracorporate dispute, it would be remanded to the Regional
Trial Court just the same.

TOPIC:DEVICES
OR
MISREPRESENTATION

SCHEMES

AMOUNTING

TO

FRAUD

OR

HERNANI N. FABIA,
vs.
COURT OF APPEALS, DEPARTMENT OF JUSTICE, OFFICE OF THE CITY
PROSECUTOR OF MANILA, REGIONAL TRIAL COURT OF MANILA-Br.
22, and THE MARITIME TRAINING CENTER OF THE PHILIPPINES
(MTCP)
G.R. No. 132684.August 20, 2001

Page 911 of 1072

FACTS:
Petitioner Hernani N. Fabia, until his resignation on 10 August 1994,
was the President of private respondent MTCP, a domestic corporation
engaged in providing maritime courses and seminars to prospective
overseas contract workers and seamen. He was likewise a Director and
stockholder thereof.
On 3 January 1996 MTCP through its new President Exequiel B. Tamayo
filed an affidavit-complaint for estafa against Hernani N. Fabia with the Office
of the City Prosecutor of Manila alleging that on various occasions from
January to July 1994 Fabia drew cash advances from MTCP, covered by cash
vouchers, amounting to P1,291,376.61 which he failed to liquidate despite
repeated demands.
Petitioner Fabia in his 20 March 1996 Reply-Affidavit and Motion to
Dismiss admitted having received the various amounts covered by the cash
vouchers but reasoned that they were in the nature of simple loans that had
already been liquidated and paid as shown by the receipts and vouchers
which he had attached to his pleadings.

ISSUE/S:
Whether or not the instant case involves
controversy primarily cognizable by the SEC

an

intra-corporate

RULING:
Section 6, PD 902-A confines the jurisdiction of the SEC to "intracorporate disputes" defined as any act or omission of the Board of
Directors/Trustees of corporations, or of partnerships, or of other
associations, or of their stockholders, officers, or partners, including any
fraudulent devices, schemes or representations, in violation of any law or
rules and regulations administered and enforced by the Commission.9 This
underscores the relationship of the party-litigants with each other, and
indicates that the nature of the cause of action should be limited to
fraudulent devices, schemes or representations, in violation of any law, rules
and/or regulations administered and enforced by the Commission for the
cause of action to fall within the ambit of authority of the SEC elements
that are both present in the instant case. Petitioner was the President as well
as a Director and stockholder in private respondent MTCP, who was charged
with the misappropriation or diversion of corporate funds after having failed
to liquidate the amount of P21,291,376.61 he had received as cash advances
from the company.
Indeed, the charge against petitioner is for estafa, an offense
punishable under The Revised Penal Code (RPC), and prosecution for the

Page 912 of 1072

offense is presently before the regular courts. However, as correctly pointed


out by private respondent MTCP, jurisdiction is determined not from the law
upon which the cause of action is based, nor the type of proceedings
initiated, but rather, it is gleaned from the allegations stated in the
complaint. It is evident from the complaint that the acts charged are in the
nature of an intra-corporate dispute as they involve fraud committed by
virtue of the office assumed by petitioner as President, Director, and
stockholder in MTCP, and committed against the MTCP corporation. This
sufficiently removes the action from the jurisdiction of the regular courts, and
transposes it into an intra-corporate controversy within the jurisdiction of the
SEC. The fact that a complaint for estafa, a felony punishable under the RPC,
has been filed against petitioner does not negate and nullify the intracorporate nature of the cause of action, nor does it transform the
controversy from intra-corporate to a criminal one.
Accordingly, as the matter involves an intra-corporate dispute within
the jurisdiction of the SEC, the issue of whether prior non-accounting
precludes a finding of probable cause for the charge of estafa no longer finds
relevance.

TOPIC:DEVICES
OR
MISREPRESENTATION

SCHEMES

AMOUNTING

TO

FRAUD

OR

A & A CONTINENTAL COMMODITIES PHILIPPINES, INC.,


vs.
SECURITIES AND EXCHANGE COMMISSION and ROLANDO G. AGUILA

Page 913 of 1072

G.R. No.L-55343. August 16, 1993


225 S 314
FACTS:
Petitioner is a domestic corporation duly organized and existing under
Philippine laws, engaged in the commodities brokerage business. On August
3, 1979, petitioner and private respondent entered into a contract for the
purchase or sale of commodities. On January 21, 1980, private respondent
bought, through petitioner, seven contracts of copper. The margin
requirement: for the seven contracts was P18,750 per contract or a total
amount of P131,250.00, which amount was earmarked from private
respondent's cash deposit with petitioner of P306,326.46. On January 23,
1980. petitioner, allegedly "without valid and justifiable cause, maliciously,
arbitrarily, wantonly, fraudulently, and recklessly" ordered private
respondent to increase his margin requirements per contract from P18,750 to
P75,000 and gave private respondent up to 5 P.M. of the same day within
which to deposit with petitioner the amount of P344,771.05. Private
respondent requested additional time within which to raise the amount, but
petitioner informed him that it would immediately sell, his seven copper
contracts should he fail to deposit the additional amount by5 P.M. that same
day. Private respondent then requested that should petitioner proceed with
the sale, the same be not effected immediately upon the opening of trading
if prices were low but at a later time. However, petitioner did not accede to
the request and sold five contracts for $1.2150 a pound immediately upon
the opening of trading on January 24, 1980 and the other two at $1.25 a
pound at a latter time.
Claiming that he had incurred a loss of P199,646.68 due to the
premature sale of the copper contracts and that he would have made a profit
of P100,000 had the sale been made at a later date, private respondent filed
with the SEC on March 19, 1980 a complaint.
ISSUE/S:
Whether or not the SEC has jurisdiction over SEC Case No. 1886.
RULING:
Considering that Petitioners' Complaints sufficiently allege acts
amounting to fraud and misrepresentation committed by Respondent
Corporation, the SEC must be held to retain its original and exclusive
jurisdiction over these five (5) cases notwithstanding the revocation by the
Central Bank of Respondent Corporation's license or permit to operate as a

Page 914 of 1072

financing company and despite the fact that the suits involve collections of
sums of money paid to said corporation, the recovery of which would
ordinarily fall within the jurisdiction of regular Courts. The fraud committed is
detrimental to the interest of the public and, therefore, encompasses a
category of relationship within the SEC jurisdiction.
Otherwise stated, in order that the SEC can take cognizance of a case,
the controversy must pertain to any of the following relationships:
a. between the corporation, partnership or association and
the public;
b. between the corporation, partnership or association and its
stockholders, partners, members or officers;
c. between the corporation, partnership or association and
the state in so far as its franchise, permit or license to
operate is concerned; and
d. among
the
stockholders,
partners
or
associates
themselves.
Withal, the complaint alleged fraud on the part of petitioner which
supposedly resulted in monetary losses to private respondent for which
reason the conclusion, on the basis of the Orosa and Magalad cases, is that
the SEC has original and exclusive jurisdiction over SEC Case No. 1886.

TOPIC:DEVICES
OR
MISREPRESENTATION

SCHEMES

AMOUNTING

TO

FRAUD

OR

MANUEL M. ALLEJE
vs.

Page 915 of 1072

COURT OF APPEALS, SPORTS HEALTH AND PHYSICAL EDUCATION


(SHAPE) CENTRE, INC., and/or ARMIE E. ELMA, Presiding Judge, RTC
Pasig, Br. 153
G.R. No. 107152. January 25, 1995
240 S 606
FACTS:
Private respondent SHAPE is a duly registered non-stock, non-profit
corporation the primary purpose of which is to foster and promote health,
conduct physical education and fitness exercises as well as pleasure and
recreation activities by establishing and maintaining facilities, sports centers,
and the like. SHAPE owns and operates a main sports and fitness center at
its principal office at the University of Life Complex in Pasig, Metro Manila,
with a branch at the Philamlife Homes Clubhouse in Quezon City.
Petitioner Manuel M. Alegre was the Executive Vice President of SHAPE until
his termination of 6 June 1991.
On 16 October 1991 SHAPE filed a complaint for injunction and
damages with application for preliminary injunction and/or temporary
restraining order with preliminary attachment against petitioner Alleje.
ISSUE/S:
Whether or not partakes of an intracorporate controversy solely
cognizable by the SEC.
RULING:
The Supreme Court grants the petition.
The applicable law is PD 902-A.
It is axiomatic that jurisdiction over the subject matter of a case is
conferred by law and is determined by the allegations of the complaint
irrespective of whether plaintiff is entitled to some or all of the claims
asserted therein.
Among the questionable acts imputed to petitioner are:
a. withdrawal of funds of SHAPE purportedly as either
"personal advances" or "advances for liquidation" which he
could not have accomplished if he were not an "EVP";
b. failure to remit funds belonging to SHAPE;
c. unauthorized removal of "nautilus" equipment from
SHAPE's gym for use in his own health and fitness centers;
and
d. naming his health and fitness centers as "SHAPE CAMP,"
making it appear that they had some business connection
with SHAPE.

Page 916 of 1072

Clearly, the complaint alleges that as an officer of SHAPE Alleje


employed devises or schemes tantamount to fraud and misrepresentation in
order to divert corporate funds and assets for his personal use. This has
transposed an otherwise ordinary action for recovery of certain properties
and sum of money with damages into an intracorporate controversy which
calls for the adjudicative powers of the SEC pursuant to Sec. 5, par. (a), of PD
902-A. In other words, the complaint filed SHAPE before the Pasig trial court
imputes unmistakable acts of fraud to Alleje as an officer of SHAPE which
have supposedly resulted in its heavy financial losses. The fraud committed
is detrimental to the interest not only of the corporation itself but also of its
members who have unselfishly agreed among themselves that no part of the
net income of the corporation shall inure to any of them.

TOPIC:DEVICES
OR
MISREPRESENTATION

SCHEMES

AMOUNTING

TO

FRAUD

OR

MARIA LUISA FLOR C. BAEZ, SPOUSES PETRONILO ESTEVES AND


LUISA ESTEVES AND CONSUELO M. CAULBOY,
vs.

Page 917 of 1072

DIMENSIONAL CONSTRUCTION TRADE AND DEVELOPMENT


CORPORATION AND THE REGIONAL TRIAL COURT, BRANCH XLV,
URDANETA, PANGASINAN
G.R. No.L-62648. November 22, 1985
140 S 249
FACTS:
The facts of this case disclose that on October 13, 1980, petitioners
herein filed with the aforementioned court below a complaint, praying
therein that the defendant Dimensional Construction Trade and Development
Corporation, the private respondent in this case, be ordered to pay them the
sums of money which were already due to them under the various
promissory notes issued by said defendant. Defendant Corporation was
served copy of the summons and complaint on November 18, 1980, but it
failed to file any responsive pleading.
On August 20, 1981, or nine (9) months thereafter, petitioners moved to
have private respondents declared in default. On October 14, 1981, the trial
court declared private respondent in default and set the date for the
reception of plaintiff's evidence. According to petitioners herein, they
submitted their evidence ex-parte.
ISSUE/S:
Whether or not it is the Securities and Exchange Commission which
has jurisdiction to hear and decide the plaintiff's complaint
RULING:
Plaintiffs cause of action is merely for the collection of the various
sums of money that have already become payable to petitioners due to the
promissory notes executed by defendant corporation which have already
matured. There is neither allegation nor any mention whatsoever in plaintiff's
complaint that a device or scheme was resorted to by private respondent
corporation amounting to fraud and misrepresentation. It is, therefore,
difficult to consider that petitioners' case falls within the jurisdiction of the
Securities and Exchange Commission pursuant to PD 902-A. Paradoxically,
despite the absence of imputation of fraud and misrepresentation being
alleged by plaintiff, it is the defendant corporation itself which insinuates the
existence of fraud and misrepresentation on its part.
In the promissory notes issued by private respondent corporation, it is
clearly indicated therein that the sums of money received by private
respondent were in the nature of investments of the petitioners, agreed upon
by the parties to be returned by the corporation upon the maturity of said
promissory notes. As the money received by private respondent do not
constitute payment of subscription of shares, the petitioners herein did not

Page 918 of 1072

become members of respondent Dimensional Trade and Development


Corporation.
From the practical standpoint it would even be a useless exercise to
refer to the Securities and Exchange Commission the subject case which has
been pending in court for over five (5) years considering that private
respondents herein did not even elect to file any answer to the complaint
filed against it in the court below nor has it made any mention in its
pleadings submitted to this court that it has a good and meritorious defense
to the petitioners' cause of action. The efforts of the private respondent to
promote unwarranted delay should not be allowed to succeed any further.

TOPIC:DEVICES
OR
MISREPRESENTATION

SCHEMES

AMOUNTING

TO

FRAUD

OR

RAUL H. SESBRENO,
vs.
HONORABLE COURT OF APPEALS and HERMILO RODIS, SR.
G.R. No. 84096. January 26, 1995

Page 919 of 1072

240 S 606
FACTS:
Private respondents Hermilo Rodis, Sr., together with Douglas Sandiego
and Ricardo Silverio, Sr., was charged with estafa.
Respondent Rodis moved to quash the information on the ground that
the Securities and Exchange Commission (SEC), not the regular courts, had
jurisdiction over the offense charged and that the facts stated herein did not
constitute an offense. The trial court denied the motion and private
respondent elevated the case to the then Intermediate Appellate Court on a
petition for certiorari.

ISSUE/S:
Whether or not private respondent may be held liable for estafa under
the facts obtaining in the trial court.
RULING:
The respondent court held that private respondent's liability, if any, is
only civil.
The Court of Appeals, correctly ruled that a money market transaction
partakes of the nature of a loan and therefore "nonpayment thereof would
not give rise to criminal liability for estafa through misappropriation or
conversion.
In money market placement, the investor is a lender who loans his
money to a borrower through a middleman or dealer. Petitioner here loaned
his money to a borrower through Philfinance. When the latter failed to deliver
back petitioner's placement with the corresponding interest earned at the
maturity date, the liability incurred by Philfinance was a civil one. As such,
petitioner could have instituted against Philfinance before the ordinary courts
a simple action for recovery of the amount he had invested and he could
have prayed therein for damages.
It appears, however, that petitioner did not even implead Philfinance in
the complaint for damages arising from the nonreturn of investment with
respect to the same money market placement involved herein, which he
eventually filed against Delta Motors Corporation and Pilipinas Bank.
The conclusion we have here reached is, of course, without to such
right of reimbursement as Pilipinas may have vis-a-vis Philfinance.
Petitioner's recovery of his investment and the dismissal of the criminal
aspect of the case he had filed against private respondent as a consequence

Page 920 of 1072

of this decision notwithstanding, he still has an opportunity to hold private


respondent liable in Criminal Case No. CU-10568.

TOPIC: CONTROVERSIES ARISING OUT OF INTRA-CORPORATE OR


PARTNERSHIP
RELATIONS
IGLESIA EVANGELICA METODISTA EN LAS ISLAS FILIPINAS (IEMELIF),
INC., petitioner
vs.

Page 921 of 1072

NATANAEL B. JUANE, respondent


G.R. No. 172447 September 18, 2009
FACTS:
Complaint, captioned Unlawful Detainer, filed by Iglesia Evangelica
Metodista en las Islas Filipinas (IEMELIF), Inc. Against Reverend Natanael B.
Juane. IEMELIF is a religious corporation existing and duly organized under
Philippine laws. Juane is a former minister or pastor of IEMELIF. He was
elected as one of the members of the Highest Consistory of Elders (or Board
of Trustees) of IEMELIF in the February 2000 IEMELIF General Conference.
Bishop Lazaro, acting in his capacity as the General Superintendent of
IEMELIF Chursh as well as the General Administrator of the IEMELIF Cathedral
in Tondo, removed Juane as Resident pastor of the Tondo Cathedral
Congregation and assigned him as Resident Pastor of the Sta. Mesa (Banal na
Hapag) Congregation.
However, Juane defied said re-assignment and continued to arrogate
upon himself the position of Resident Pastor of the Cathedral. In accordance
with the Discipline of the Church approved the expulsion of herein Juane as a
pastor of the IEMELIF Church for various acts of defiance and rebellion.
Juane filed a Motion to Dismiss Civil Case contending that the
complaint therein actually involved intra-corporate controversies, under
Securities Regulation Code.
ISSUE/S:
Whether the complaint filed by IEMELIF against Juane constitutes an
intra-corporate dispute beyond the jurisdiction of the MeTC.
RULING:
Even if the transformation of IEMELIF from a corporation sole to a
corporation aggregate was legally defective, its head or governing body , i.e.
Bishop Lazaro, whose acts were approved by the Highest Consistory of
Elders, still did not change.
If the transformation did materialize, the corporation aggregate would
be composed of the Highest Consistory of Elders, which nevertheless
approved the very same acts. As either Bishop Lazaro or the Highest
Consistory of Elders had the authority to appoint Juane as Resident Pastor of
the IEMELIF Tondo Congregation, it also had the power to remove him as
such or transfer him to another congregation.

Page 922 of 1072

TOPIC: CONTROVERSIES ARISING OUT OF INTRA-CORPORATE OR


PARTNERSHIP
RELATIONS
GD EXPRESS WORLDWIDE N.V.
VS.
COURT OF APPEALS
G.R. No. 136978
May 8, 2009

Page 923 of 1072

FACTS:
Petitioner GD Express Worldwide N.V. (GD Express) is a corporation
duly organized and existing under the laws of the Netherlands. Its
predecessor-in-interest, TNT Limited (TNT) entered into a joint venture
agreement with Philippine Aerospace Development Corporation (PADC) for
the establishment of a domestic corporation as their corporate vehicle to
operate as an international air freight carrier. The joint venture agreements
stipulated that PADC would own 80% of the shares of stock of the corporate
vehicle while TNT would own the remaining 20%.
The agreements essentially laid down the relationship between TNT
and PADC and the management, control and existence of the corporation.
Also, pursuant to the joint venture agreements, PADC and TNT registered
with the SEC a corporation to be known as Air Philippines Corporation (APC).
APC amended its articles of incorporation to change its corporate name
to Pacific East Asia Cargo Airlines, Inc. (PEAC). TNT transferred all its shares
in PEAC to petitioner GD Express. PEAC immediately commenced operations.
Herein petitioner Amihan Management Services, Inc. (Amihan), a domestic
corporation, was contracted to undertake the daily operations in PEAC
pursuant to the joint venture agreement.
The Office of the President mandated the Committee on Privatization to
require the Asset Privatization Trust (APT) to dispose of PADCs 80% share in
PEAC. Thus, petitioner GD Express and PADC executed the Terms of
Reference that would govern the disposition of PADCs equity comprising
12,800 subscribed shares of stock in PEAC.
The APT issued the Asset Specific Bidding Rules (ASBR) incorporating
the Terms of Reference for the sale of PADCs shares of stock in PEAC. The
ASBR required prospective bidders, among others, to comply with the
obligations and undertakings/warranties enumerated therein. At the bidding,
respondent Filchart, also a domestic corporation, emerged as the highest
bidder of the 12,800 shares of stock owned by PADC in PEAC.
Alleging that respondent Filchart was bent on reneging on its
obligations and warranties under the ASBR and Terms of Reference,
petitioner GD Express instituted a case for specific performance to compel
PADC and APT to faithfully comply with the joint venture agreements.
During the pendency of the case, PADC and respondent Filchart
executed the corresponding deed of absolute sale, by virtue of which PADC
sold to respondent Filchart its shares of stock in PEAC.
This prompted petitioner GD Express to file an amended complaint 9 to
introduce another cause of action for the nullification of the said transfer and

Page 924 of 1072

to implead the Committee on Privatization, the PEAC and respondent Filchart


as additional defendants.
ISSUE/S:
Whether the SEC Hearing Officer was not authorized to assume
jurisdiction over SEC

RULING:
There is no question that the prayers for the appointment of a
management receiver, the nullification and amendment of certain provisions
of PEACs articles of incorporation and by-laws, the recognition of the
election of respondent Filcharts directors, as well as the inspection of the
corporate books, are intra-corporate in nature as they pertain to the
regulation of corporate affairs.
In view of the transfer of jurisdiction over intra-corporate disputes from
the SEC to the SCCs, which are the same RTCs exercising general jurisdiction,
the question of jurisdiction is no longer decisive to the resolution of the
instant case.
It should be noted that the SCCs are still considered courts of general
jurisdiction. Section 5.2 of R.A. No. 8799 directs merely the Supreme Courts
designation of RTC branches that shall exercise jurisdiction over intracorporate disputes. Nothing in the language of the law suggests the
diminution of jurisdiction of those RTCs to be designated as SCCs. The
assignment of intra-corporate disputes to SCCs is only for the purpose of
streamlining the workload of the RTCs so that certain branches thereof like
the SCCs can focus only on a particular subject matter.
The designation of certain RTC branches to handle specific cases is
nothing new. For instance, pursuant to the provisions of the R.A. No. 6657 or
the Comprehensive Agrarian Reform Law, the Supreme Court has assigned
certain RTC branches to hear and decide cases under Sections 56 and 57 of
R.A. No. 6657.
The RTC exercising jurisdiction over an intra-corporate dispute can be
likened to an RTC exercising its probate jurisdiction or sitting as a special
agrarian court. The designation of the SCCs as such has not in any way
limited their jurisdiction to hear and decide cases of all nature, whether civil,
criminal or special proceedings.
There is no jurisdictional infirmity for either court the only question that
remains is whether Civil Case No. 96-17-675 and SEC Case No. 08-97-5746,
now transferred to the proper SCC, may proceed concurrently or should be

Page 925 of 1072

consolidated or whether SEC Case No. 08-97-5746 should be suspended to


await the outcome of Civil Case No. 96-17-675.
Thus, the SCC to which SEC Case No. 08-97-5746 was transferred has
sufficient discretion to determine whether under the circumstances of the
case, it should await the outcome of Civil Case No. 96-17-675.

TOPIC: CONTROVERSIES ARISING OUT OF INTRA-CORPORATE OR


PARTNERSHIP
RELATIONS
THE INTESTATE ESTATE OF ALEXANDER T. TY, represented by the
Administratrix, SYLVIA S. TY, petitioner,
vs.
COURT OF APPEALS, HON. ILDEFONSO E.GASCON, and ALEJANDRO B.
TY, respondents.

Page 926 of 1072

G.R. No. 112872 April 19, 2001


FACTS:
Petitioner Sylvia S. Ty was married to Alexander T. Ty. Son of private
respondent Alejandro B. Ty. When Alexander died, petitioner was appointed
administratrix of her late husbands intestate estate. Thereafter, petitioner
filed a motion for leave to sell or mortgage estate property in order to
generate funds for the payment of deficiency estate taxes. Private
respondent Alejandro Ty then filed two complaints for the recovery of the
above-mentioned property, praying for the declaration for nullity of the deed
of absolute sale of the shares of stock executed by private respondent in
favor of the deceased Alexander, and for the recovery of the pieces of
property that were placed in the name of deceased Alexander by private
respondent, the same property being sought to be sold out, mortgaged, or
disposed of by petitioner. Private respondent claimed in both cases that
even if said property were placed in the name of deceased Alexander, they
were acquired through private respondents money, without any cause or
consideration from deceased Alexander. Motions to dismiss were filed by
petitioner. Both motions alleged lack of jurisdiction for the trial court,
claiming that the cases involved intra-corporate disputes cognizable by the
Securities and Exchange Commission (SEC).
ISSUE/S:
Whether or not the case involves an intra-corporate dispute.
RULING:
In the cases at bar, the relationship of private respondent when he sold
his shares of stock to his son was one of vendor and vendee, nothing else.
The question raised in the complaints is whether or not there was indeed a
sale in the absence of cause or consideration. The proper forum for such a
dispute is a regular trial court. The Court agrees with the ruling of the Court
of appeals that no special corporate skill is necessary in resolving the issue
of the validity of the transfer of shares from one stockholder to another of
the same corporation. Both actions, although involving different property,
sought to declare the nullity of the transfers of said property to the decedent
on the ground that they were not supported by any cause or consideration,
and thus, are considered void ab initio for being absolutely simulated or
fictitious. The determination whether a contract is simulated or not is an
issue that could be resolved by applying pertinent provisions of the Civil
Code particularly those relative to obligations and contracts. Disputes
concerning the application of the Civil Code are properly cognizable by courts
of general jurisdiction. It should also be noted that under the newly enacted
Securities Regulation Code (Republic Act No. 8799), this issue is now moot
and academic because whether or not the issue is intra-corporate, it is the
regional trial court and no longer the SEC that takes cognizance of the
controversy. Under Section 5.2 of Republic Act No. 8799, original and

Page 927 of 1072

exclusive jurisdiction to hear and decide cases involving intra-corporate


controversies have been transferred to courts of general jurisdiction or the
appropriate regional trial court.

TOPIC: CONTROVERSIES
PARTNERSHIP
RELATIONS

ARISING

OUT

OF

INTRA-CORPORATE

OR

HERNANI N. FABIA, petitioner,


vs.
COURT OF APPEALS, DEPARTMENT OF JUSTICE, OFFICE OF THE CITY
PROSECUTOR OF MANILA, RTC-Br. 22, MANILA and THE MARITIME
TRAINING CENTER OF THE PHILIPPINES
G.R. No. 132684. September 11, 2002

Page 928 of 1072

FACTS:
Petitioner Hernani N. Fabia, until his resignation on 10 August 1994,
was the President of private respondent MTCP, a domestic corporation
engaged in providing maritime courses and seminars to prospective
overseas contract workers and seamen. He was likewise a Director and
stockholder thereof. On 3 January 1996 MTCP through its new President
Exequiel B. Tamayo filed an affidavit-complaint for estafa against Hernani N.
Fabia with the Office of the City Prosecutor of Manila alleging that on various
occasions from January to July 1994 Fabia drew cash advances from MTCP,
covered by cash vouchers, which he failed to liquidate despite repeated
demands. Petitioner Fabia in his 20 March 1996 Reply-Affidavit and Motion to
Dismiss admitted having received the various amounts covered by the cash
vouchers but reasoned that they were in the nature of simple loans that had
already been liquidated and paid as shown by the receipts and vouchers
which he had attached to his pleadings.
On 8 April 1996 the Office of the City Prosecutor of Manila dismissed
the complaint for lack of jurisdiction for the reason that the controversy
pertained to the relationship between a corporation and a former officer
thereof, hence, it was the SEC which had original and exclusive jurisdiction
over the case. MTCP moved to reconsider the resolution but the same was
denied with the additional ground that "the charge involves accounting and
liquidation of cash advances which receipts and vouchers had not been
examined by an independent certified public accountant for a conclusive
determination as to the actual amount stashed by the officer," hence, the
evidence was insufficient to show probable cause. Thereafter, on 13
September 1996, MTCP filed a petition for review before the DOJ questioning
the 2 resolutions issued by the Office of the City Prosecutor. The petition was
however dismissed by the DOJ on 2 December 1996 as it found no reversible
error committed by the Office of the City Prosecutor. MR was likewise denied.
Consequently, on 9 May 1997 MTCP filed a petition for certiorari before
the CA raising as sole issue whether the defense of lack of accounting
precludes a finding of probable cause, with prayer that the DOJ Resolutions
be annulled.
The Court of Appeals granted the petition and in its assailed Decision
of 12 November 1997 held that the amount subject of the estafa charge had
in fact been determined by an independent certified public accountant as
shown by the report from the accounting firm of Mendoza, Ignacio, Corvera
and Co., containing an itemized account of the unliquidated cash advances
made by petitioner, which fact he admitted in his Reply-Affidavit.
On 27 November 1997 petitioner moved for a reconsideration of the
Decision but it was denied. Hence, on 23 January 1998 the Office of the City

Page 929 of 1072

Prosecutor as directed caused the filing of an Information for estafa against


petitioner before the Manila RTC.

ISSUE/S:
Whether the RTC has jurisdiction over the case.

RULING:
NO... Section 6, PD 902-A confines the jurisdiction of the SEC to "intracorporate disputes" defined as any act or omission of the Board of
Directors/Trustees of corporations, or of partnerships, or of other
associations, or of their stockholders, officers, or partners, including any
fraudulent devices, schemes or representations, in violation of any law or
rules and regulations administered and enforced by the Commission. This
underscores the relationship of the party-litigants with each other, and
indicates that the nature of the cause of action should be limited to
fraudulent devices, schemes or representations, in violation of any law, rules
and/or regulations administered and enforced by the Commission for the
cause of action to fall within the ambit of authority of the SEC, elements that
are both present in the instant case. Petitioner was the President as well as a
Director and stockholder in private respondent MTCP, who was charged with
the misappropriation or diversion of corporate funds after having failed to
liquidate the amount he had received as cash advances from the company.
Indeed, the charge against petitioner is for estafa, an offense
punishable under The RPC, and prosecution for the offense is presently
before the regular courts. However, as correctly pointed out by private
respondent MTCP, jurisdiction is determined not from the law upon which the
cause of action is based, nor the type of proceedings initiated, but rather, it
is gleaned from the allegations stated in the complaint. It is evident from the
complaint that the acts charged are in the nature of an intra-corporate
dispute as they involve fraud committed by virtue of the office assumed by
petitioner as President, Director, and stockholder in MTCP, and committed
against the MTCP Corporation. This sufficiently removes the action from the
jurisdiction of the regular courts, and transposes it into an intra-corporate
controversy within the jurisdiction of the SEC. The fact that a complaint for
estafa, a felony punishable under the RPC, has been filed against petitioner
does not negate and nullify the intra-corporate nature of the cause of action,
nor does it transform the controversy from intra-corporate to a criminal one.

Page 930 of 1072

Accordingly, as the matter involves an intra-corporate dispute within


the jurisdiction of the SEC, the issue of whether prior non-accounting
precludes a finding of probable cause for the charge of estafa no longer finds
relevance.
The doctrine of primary jurisdiction exhorts us to refer the instant case
to the SEC for its resolution of the matter in dispute. However, it should be
noted that RA 8799, The Securities Regulation Code, has amended PD 902-A,
and transferred the jurisdiction of the SEC over intra-corporate cases to the
courts of general jurisdiction or the appropriate Regional Trial Courts. To
transfer the present case to the SEC would only result in a circuitous
administration of justice. Thus, the Regional Trial Court of Manila should
dismiss the Crim. Case without prejudice to the filing of the proper action
which shall then be raffled off to the appropriate branch of the court
pursuant to A.M. No. 00-11-03-SC.
However, in conformity with RA 8799, The Securities Regulation Code,
amending PD 902-A, which has effectively transferred the jurisdiction of the
Securities and Exchange Commission over all cases enumerated under Sec.
5 of PD 902-A to the courts of general jurisdiction or the appropriate Regional
Trial Courts, the Crim. case is ordered TRANSFERRED to the Regional Trial
Court of Manila to be raffled among the designated branches empowered to
try and decide cases formerly cognizable by the SEC pursuant to A.M. No. 001143-SC. No costs.

TOPIC: CONTROVERSIES
PARTNERSHIP
RELATIONS

ARISING

OUT

OF

INTRA-CORPORATE

OR

TEODORO B. VESAGAS, and WILFRED D. ASIS, petitioners, vs. The


Honorable COURT OF APPEALS and DELFINO RANIEL and HELENDA
RANIEL, respondents.

Page 931 of 1072

G.R. No. 142924. December 5, 2001


523 SCRA 233
FACTS:
Herein private respondents, Delfino and Helenda Raniel, were
members in good standing of Luz VillageTennis Club alleging that club
president Vesagas summarily stripped them of their membership without due
process of law. Thus, plaintiffs moved to declare as illegal their expulsion
from the club in utter disregard of the provisions of the clubs by laws.
Respondents, on the other hand, moved to dismiss the complaint on the
ground that the SEC lacked the jurisdiction over the subject matter. They
contend that since its inception in the 1970s, the club in practice has not
been a corporation, and that it was only the plaintiffs who surreptitiously
caused the clubs registration with the SEC. Further, they argued that the
club has ceased to be a corporate body at any rate. Thus, no intra-corporate
relationship can arise as between the parties.
ISSUE/S:
Whether or not the SEC had jurisdiction over the case filed by the
plaintiffs.
RULING:
The Supreme Court agreed with the findings of the Commission that the club
has been duly registered and that a certificate of incorporation was issued in
its favor. Thus, the SEC has jurisdiction over the case.
Petitioners are estopped from denying the personality of the
corporation by their very own acts. The dispute was considered as intracorporate in character because:
1. the parties involved are officers and members of the club;
2. the conflict arose from this relation between the parties;
3. and the subject of complaint involved expulsions from the club
membership, validity of amendments of by laws.
Thus, the same falls within the Securities and Exchange Commission
jurisdiction.
TOPIC: CONTROVERSIES ARISING OUT OF INTRA-CORPORATE OR
PARTNERSHIP
RELATIONS

Page 932 of 1072

SPOUSES JOSE ABEJO AND AURORA ABEJO, TELEC. TRONIC SYSTEMS,


INC., petitioners,
vs.
HON. RAFAEL DE LA CRUZ, JUDGE OF THE REGIONAL TRIAL COURT
(NATIONAL CAPITAL JUDICIAL REGION, BRANCH CLX-PASIG),
SPOUSES AGAPITO BRAGA AND VIRGINIA BRAGA, VIRGILIO BRAGA
AND NORBERTO BRAGA, respondents.
G.R. No. L-63558
May 19, 1987
FACTS:
The Spouses Abejo sold their minority shareholdings in PBPI to TSI as
well as the shares of Virginia Braga in PBPI by virtue of the stock certificates
covering the latters shares endorsed in blank by Braga. TSI therefore asked
that the transfer be recorded in the books of the corporation.
The
corporations treasurer, Norberto Braga, son of the Spouses Braga, refused to
enter the same contending that it has pre-emptive rights over the Abejo
shares and that the certificates covering the shares of his parent were lost.
Abejo and TSI prayed for Mandamus before the SEC for Braga to enter the
name of TSI in the transfer book. Norberto Braga sought to dismiss
contending that the SEC has no jurisdiction over the nature of the action
since it does not involve an intra-corporate controversy between
stockholders, the principal petitioners, TSI, not being a stockholder of record
of Pocket Bell. Later, SEC ordered that the name of TSI be entered in the
books. Meanwhile, pending SEC hearing, Braga filed a complaint before the
CFI prating for nullity and rescission of the sale of the Abejos to TSI invoking
his pre-emptive right and a complaint for nullity of the sale of Virginias
shares. Abejos sought to dismiss the case before the CFI invoking lack of
jurisdiction.
ISSUE/S:
Whether or not the SEC has jurisdiction over the controversy.
RULING:
Supreme Court held that the Securities and Exchange Commission has
original and exclusive jurisdiction over the dispute between the principal
stockholders of the corporation PBPI, namely, the Abejos and Telectronics,
the purchasers of the 56% majority stock on the one hand, and the Bragas,
erstwhile majority stockholders, on the other, and that the SEC, through its
en banc Resolution of May 15, 1984 correctly ruled in dismissing the Bragas'
petition questioning its jurisdiction, that "the issue is not the ownership of
shares but rather the nonperformance by the Corporate Secretary of the
ministerial duty of recording transfers of shares of stock of the Corporation of
which he is secretary." The very complaint of the Bragas for annulment of the
sales and transfers as filed by them in the regular court questions the
validity of the transfer and endorsement of the certificates of stock, claiming
alleged preemptive rights in the case of the Abejos' shares and alleged loss

Page 933 of 1072

of the certificates and lack of consent and consideration in the case of


Virginia Braga's shares. Such dispute clearly involves controversies "between
and among stockholders," as to the Abejos' right to sell and dispose of their
shares to Telectronics, the validity of the latter's acquisition of Virginia
Braga's shares, who between the Bragas and the Abejos' transferee should
be recognized as the controlling shareholders of the corporation, with the
right to elect the corporate officers and the management and control of its
operations. Such a dispute and case clearly fall within the original and
exclusive jurisdiction of the SEC to decide.

TOPIC: CONTROVERSIES ARISING OUT OF INTRA-CORPORATE OR


PARTNERSHIP
RELATIONS

Page 934 of 1072

DANIEL R. AGUINALDO, DOMINADOR R. AYTONA AND ROMEO H.


BORSOTO
VS.
S.E.C., JOSE G. RICAFORT, CONRADO T. CALALANG, BENJAMIN V.
ARITAO, SALVADOR O. RIVERA, EDGARDO DE CASTRO, ARMANDO O.
ONGSIOKO AND
NATIONWIDE DEVELOPMENT CORPORATION (NADECOR)
G.R. NO. 102965; JANUARY 21, 1999
FACTS:
Private respondents claim that NADECOR has a total outstanding
capital stock of 30,000 shares. Out of these 30,000 shares, 7,000 shares,
representing 23% of the outstanding capital stock of NADECOR is owned by
a U.S. Corporation, the Sawyer Adecor International, Inc. (SAICOR). Aytona,
Aguinaldo, Calalang, Ricafort, and five others were elected as directors of
NADECOR by the stockholders at a meeting held on August 18, 1980, and
during the organization meeting held on the same day, petitioners Aytona
and Aguinaldo, and one R.H. Borsoto were elected Chairman of the Board,
President, and Corporate Secretary, respectively, of the NADECOR.
Private respondents claim that petitioners did not comply with their
fiduciary duties of loyalty, diligence and care to NADECOR and, worse,
committed fraudulent machinations to exclude private respondents from
their rightful participation in the management of the NADECOR, which
culminated in the unlawful and malicious refusal to perform their ministerial
duty to issue notices of the annual stockholders' meeting for the year 1981
in breach of the law as set forth in the Corporation Code and the Amended
By-Laws of the NADECOR.
Despite the election of the above-named new set of directors and the
appointment of new corporate officers, private respondents claim that
petitioners continued to exclude them from the valid exercise of their rights
by refusing to honor and respect the said election, and fraudulently continue
to represent themselves as officers of the NADECOR and illegally usurp the
functions of the officers of the NADECOR which now rightfully pertain to
herein private respondents and the other new corporate officers.
ISSUE/S:
Whether or not the SEC committed grave abuse of secretion in the
issuance and continued enforcement of the TRO, and the delay of its
en banc division in resolving the petitions which also pray for the lifting
of the questioned TRO.
RULING:

Page 935 of 1072

Section 6 of PD 902-A grants the SEC in order to effectively exercise


such jurisdiction, the power to issue preliminary or permanent injunctions,
whether prohibitory or mandatory, in all cases in which it has jurisdiction,
and in which cases the pertinent provisions of the Rules of Court shall apply.
Since the SEC is at least a co-equal body of the Regional Trial Court
when it adjudicates controversies over which it has jurisdiction, it follows that
the temporary restraining order issued by SEC must have the same life-span
as that issued by the trial court. It is a well-settled rule that a temporary
restraining order issued by a trial court has a life of only 20 days.
To the extent, therefore, that the enforcement of the temporary
restraining order issued by the respondent SEC exceeded 20 days, the SEC
committed grave abuse of discretion. However, although the questioned
order no longer has any force and effect, the respondent SEC still has the
jurisdiction and obligation to proceed with the hearing of the case on the
merits and to issue the appropriate orders pursuant thereto subject to review
by the CA and eventually by the SC.

TOPIC: CONTROVERSIES ARISING OUT OF INTRA-CORPORATE OR


PARTNERSHIP
RELATIONS
JOSE PENEYRA and MILAGROS CALDERON, petitioners,
vs.
HON. INTERMEDIATE APPELLATE COURT and HONORABLE
GODOFREDO RILLORAZA, respondents.

Page 936 of 1072

G.R. No.L-68935. January 22, 1990


FACTS:
On May 7, 1976, the Board of Trustees of the Corregidor College Inc.
awarded the management and operation of its canteen at a monthly rental of
P80.00 to petitioners herein who are stockholders of the said College.
Subsequently, upon instructions of Eulogio Dizon, Chairman of the Board of
Trustees of Corregidor College, Inc., the rental payments of petitioners were
refused, and on August 6, 1980, partial demolition of the canteen was
effected. Consequently, on September 9, 1980, petitioners filed in the then
Court of First Instance of Nueva Ecija an action against Eulogio R. Dizon for
damages with preliminary mandatory injunction.
After Dizon filed his answer and counterclaim, pre-trial took place. On
February 20, 1981, petitioners presented their evidence and rested their
case. The judge to whom the case was assigned having inhibited himself, the
case was re-assigned to herein respondent judge, the Hon. Godofredo
Rilloraza.
On September 2, 1983, petitioners filed a motion for leave to amend
the complaint so as to include Corregidor College, Inc. as additional
defendant. Dizon opposed the motion since petitioners had already
presented their evidence.
ISSUE/S:
Does jurisdiction over Civil Case No. 774-G pertain to the Securities
and Exchange Commission?
RULING:
Certainly, the present controversy cannot qualify as an intracontroversy, its root being a contractual breach separate and distinct from
the corporate relationship between petitioners and Corregidor College, Inc.,
which, it must be noted, was not even named as a defendant in the original
complaint. It was therefore patent error for the Court of Appeals to
immediately rule that the present case belongs to the SEC just because
petitioners alleged that they are stockholders of Corregidor College, Inc.
Under Section 3 of Presidential Decree 902-A, the jurisdiction of the
SEC is limited to matters intrinsically connected with the regulation of
corporations, partnerships and associations and those dealing with the
internal affairs of such entities. P.D. 902-A does not confer in the SEC
absolute jurisdiction and control over all matters affecting corporations. To
uphold the appellate court's ruling would remove without legal imprimatur
from the regular courts all controversies over matters involving or affecting

Page 937 of 1072

corporations regardless of the nature of the transactions which give rise to


such disputes.

TOPIC: CONTROVERSIES ARISING OUT OF INTRA-CORPORATE OR


PARTNERSHIP
RELATIONS
MAINLAND CONSTRUCTION, CO., INC., and/or LUCITA LU
CARABUENA, ROBERT L. CARABUENA, ELLEN LU CARABUENA, and
MARTIN LU, petitioners,
vs.

Page 938 of 1072

MILA MOVILLA, ERNESTO MOVILLA, JR., MILA JUDITH C. MOVILLA,


JUDE BRIX C. MOVILLA, JONARD ELLERY C. MOVILLA, AND MAILA
JONAH M. QUIMBO, surviving heirs of ERNESTO MOVILLA, and THE
HONORABLE COMMISSIONER of the NATIONAL LABOR RELATIONS
COMMISSION-5TH DIVISION, respondents.
G.R. No. 118088. November 23, 1995
FACTS:
Mainland Construction Co., Inc. is a domestic corporation, duly
organized and existing under Philippine laws, having been issued a
certificate of registration by the Securities and Exchange Commission (SEC)
on July 26, 1977, under Registry Number 74691. Its principal line of business
is the general construction of roads and bridges and the operation of a
service shop for the maintenance of equipment. Respondents on the other
hand, are the surviving heirs of complainant, Ernesto Movilla, who died
during the pendency of the action with the Labor Arbiter.
Ernesto Movilla, who was a Certified Public Accountant during his
lifetime, was hired as such by Mainland in 1977. Thereafter, he was
promoted to the position of Administrative Officer with a monthly salary of
P4,700.00. On April 12, 1987, during petitioner corporation's annual meeting
of stockholders, the following were elected members of the Board of
Directors, viz.: Robert L. Carabuena, Ellen L. Carabuena, Lucita Lu
Carabuena, Martin G. Lu and Ernesto L. Movilla. On the same day, an
organizational meeting was held and the Board of Directors elected Ernesto
Movilla as Administrative Manager. 3He occupied the said position up to the
time of his death.
On April 2, 1991, the Department of Labor and Employment (DOLE)
conducted a routine inspection on petitioner corporation and found that it
committed such irregularities in the conduct of its business.
On the basis of this finding, petitioner corporation was ordered by
DOLE to pay to its thirteen employees, which included Movilla, the total
amount of P309,435.89, representing their salaries, holiday pay, service
incentive leave pay differentials, unpaid wages and 13th month pay. All the
employees listed in the DOLE's order were paid by petitioner corporation,
except Ernesto Movilla.
On October 8, 1991, Ernesto Movilla filed a case against petitioner
corporation and/or Lucita, Robert, and Ellen, all surnamed Carabuena, for
unpaid wages, separation pay and attorney's fees, with the Department of
Labor and Employment, Regional Arbitration, Branch XI, Davao City.

ISSUE/S:

Page 939 of 1072

Which of the two agencies of the government the NLRC or the SEC has
jurisdiction over the controversy?

RULING:
The Supreme Court finds for the respondents, it appearing that
petitioners' contention is bereft of merit.
In order that the SEC can take cognizance of a case, the controversy
must pertain to any of the following relationships: a) between the
corporation, partnership or association and the public; b) between the
corporation, partnership or association and its stockholders, partners,
members or officers;c) between the corporation, partnership or association
and the State as far as its franchise, permit or license to operate is
concerned; and d) among the stockholders, partners or associates
themselves.The fact that the parties involved in the controversy are all
stockholders or that the parties involved are the stockholders and the
corporation does not necessarily place the dispute within the ambit of the
jurisdiction of SEC. The better policy to be followed in determining
jurisdiction over a case should be to consider concurrent factors such as the
status or relationship of the parties or the nature of the question that is the
subject of their controversy.
In the case at bench, the claim for unpaid wages and separation pay
filed by the complainant against petitioner corporation involves a labor
dispute. It does not involve an intra-corporate matter, even when it is
between a stockholder and a corporation. It relates to an employer-employee
relationship which is distinct from the corporate relationship of one with the
other. The existence of an employer-employee relationship is a factual
question and public respondent's findings are accorded great weight and
respect as the same are supported by substantial evidence. Hence, we
uphold the conclusion of public respondent that Ernesto Movilla was an
employee of petitioner corporation.
It is pertinent to note that petitioner corporation is not prohibited from
hiring its corporate officers to perform services under a circumstance which
will make him an employee. Moreover, although a director of a corporation is
not, merely by virtue of his position, its employee, said director may act as
an employee or accept duties that make him also an employee.
Since Ernesto Movilla's complaint involves a labor dispute, it is the
NLRC, under Article 217 of the Labor Code of the Philippines, which has
jurisdiction over the case at bench.

Page 940 of 1072

WHEREFORE, the petition is DISMISSED for lack of showing of any


grave abuse of discretion on the part of public respondent NLRC. The
assailed decision of public respondent is thus AFFIRMED.

TOPIC: CONTROVERSIES ARISING OUT OF INTRA-CORPORATE OR


PARTNERSHIP
RELATIONS
SECURITIES AND EXCHANGE COMMISSION and HEARING OFFICER
JOSEFINA PASAY-PAZ, petitioners,
vs.

Page 941 of 1072

COURT OF APPEALS, BAN HUA-UY FLORES, BAN HA UY-CHUA,


ROLAND KING and SOON KEE COMMERCIAL, INC., respondents.
G.R. No. 93832. August 23, 1991
FACTS:
Petitioner Johnny K.H. Uy and private respondents Ban Hua Uy-Flores
and Ban Ha Uy-Chua are brother and sisters. They belong to the Uy family of
Bacolod City which owns several corporations, including LTBS Marketing
Corporation and the Soon Kee Commercial, Inc. All the three (3) abovenamed individuals, including other members of the Uy family, were
interlocking stockholders and officers of the two (2) aforementioned
corporations. Thus, private respondents Ban Hua Uy-Flores and Ban Ha UyChua were the managing directors of the said corporations and were in
custody of the corporate accounting and tax records as well as the funds of
UBS Marketing Corporation and Soon Kee Commercial, Inc. Private
respondent Roland King is the accountant of the said corporations and other
allied Uy family enterprises.
Due to serious disagreements and conflicts, the members of the Uy family,
through several conciliation meetings held before their selected Board of
Mediators, agreed to divide the family business so that the UBS Marketing
Corporation would go to petitioner Johnny K.H. Uy while the Soon Kee
Commercial, Inc. would go to the rest of the Uy family, including herein
private respondents Ban Hua Uy-Flores and Ban Ha Uy-Chua.
Accordingly, on 5 June 1987, several deeds of assignments were
executed by the parties wherein all the stockholdings of petitioner Johnny
K.H. Uy and his wife, Magdalena Uy in Soon Kee Commercial, Inc. were
assigned either to private respondents Ban Hua Uy-Flores, Ban Ha Uy- Chua
or other members of the Uy family while all the stockholdings of private
respondents Ban Hua Uy-Flores and Ban Ha Uy-Chua in LTBS Marketing
Corporation were assigned to petitioner Johnny K.H. Uy or the latter's wife.
On 1 July 1987 the parties formalized this division of the family business as
well as the other terms of the settlement.
On 6 April 1988, petitioners Johnny K.H. Uy and UBS Marketing Corporation
filed with the Securities and Exchange Commission a complaint (petition)
against the private respondents Ban Hua Uy-Flores, Ban Ha Uy-Chua, Roland
King and Soon Kee Commercial, Inc. for the recovery of UBS Marketing
Corporation's corporate books, books of account, and the accounting and
turn over of the funds and properties belonging to UBS Marketing
Corporation, docketed therein as SEC Case No. 03328.
ISSUE/S:
Whether or not the SEC has jurisdiction over the dispute subject of SEC
Case No.03328.

Page 942 of 1072

RULING:
In the case at bar, at the time of the execution of the Deed of
Assignment wherein the petitioner Johnny K-H. Uy and his wife, Magdalena
Uy, assigned all their stockholdings in Soon Kee Commercial, Inc. to the
private respondents Ban Hua Uy Flores and Ban Ha Uy-Chua and other
members of the UY family, and the Deed of Assignment wherein the private
respondents Ban Hua Uy-Flores and Ban Ha Uy-Chua, assigned all their
stockholdings in UBS Marketing Corporation to the petitioner Johnny K.H. Uy
or to his wife, the petitioner Johnny KH. Uy and the private respondents Ban
Hua Uy-Flores and Ban Ha Uy-Chua were all interlocking stockholders and
officers of the two (2) corporations owned by the Uy family. Hence, the deeds
of assignment were intra-corporate transactions which arose from intracorporate relations or between and among the stockholders of the two (2)
family corporations. The controversy subject of SEC Case No. 03328 is,
therefore, an intra-corporate controversy which falls within the original and
exclusive jurisdiction of the SEC under Section 5(b) of PD No. 902-A, as
amended.
The fact that when the complaint in SEC Case No. 03328 was filed with
the SEC, the private respondents Ban Hua Uy-Flores and Ban Ha Uy-Chua
were no longer stockholders of the UBS Marketing Corporation did not divest
the SEC of its jurisdiction over the case.

TOPIC: CONTROVERSIES ARISING OUT OF INTRA-CORPORATE OR


PARTNERSHIP
RELATIONS
SUNSET VIEW CONDOMINIUM CORPORATION, petitioner,
vs.

Page 943 of 1072

THE HON. JOSE C. CAMPOS, JR. OF THE COURT OF FIRST INSTANCE,


BRANCH XXX, PASAY CITY and AGUILAR-BERNARES REALTY,
respondents.
G.R. No.L-52361. April 27, 1981
FACTS:
The petitioner, Sunset View Condominium Corporation, in both cases,
is a condominium corporation within the meaning of Republic Act No. 4726 in
relation to a duly registered Amended Master Deed with Declaration of
Restrictions of the Sunset View Condominium Project located at 2230 Roxas
Boulevard, Pasay City of which said petitioner is the Management Body
holding title to all the common and limited common areas.
The private respondent, Aguilar-Bernares Realty, a sole proprietorship
with business name registered with the Bureau of Commerce, owned and
operated by the spouses Emmanuel G. Aguilar and Zenaida B. Aguilar, is the
assignee of a unit, "Solana", in the Sunset View Condominium Project with La
Perla Commercial, Incorporated, as assignor.
The La Perla Commercial, Incorporated bought the "Solana" unit on
installment from the Tower Builders, Inc. 4 The petitioner, Sunset View
Condominium Corporation, filed for the collection of assessments levied on
the unit against Aguilar-Bernares Realty, private respondent herein, a
complaint dated June 22, 1979 docketed as Civil Case No. 7303-P of the
Court of First Instance of Pasay City.

ISSUE/S:
Is it the regular court or the Securities & Exchange Commission that
has jurisdiction over cases for collection of assessments assessed by
the Condominium Corporation on condominium units the full purchase
price of which has not been paid?
RULING:
The private respondents, therefore, who have not fully paid the
purchase price of their units and are consequently not owners of their units
are not members or shareholders of the petitioner condominium corporation,
Inasmuch as the private respondents are not shareholders of the
petitioner condominium corporation, the instant case for collection cannot be
a "controversy arising out of intracorporate or partnership relations between
and among stockholders, members or associates; between any or all of them
and the corporation, partnership or association of which they are

Page 944 of 1072

stockholders, members or associates, respectively" which controversies are


under the original and exclusive jurisdiction of the Securities & Exchange
Commission, pursuant to Section 5 (b) of P.D. No. 902- A. The subject matters
of the instant cases according to the allegations of the complaints are under
the jurisdiction of the regular courts: that of G.R. NO. 52361, which is for the
collection of P8,335.38 with interest plus attorney's fees equivalent to the
principal or a total of more than P10,000.00 is under the jurisdiction of the
Court of First Instance; and that of G.R. NO. 52524, which is for the collection
of P6,168-06 is within the jurisdiction of the City Court.
In view of the foregoing, it is no longer necessary to resolve the issue
raised in G.R. NO. 52524 of whether an order of the City Court denying a
motion to dismiss on the ground of lack of jurisdiction can be appealed to the
Court of First Instance.
WHEREFORE, the questioned orders of the respondent Judge dated
December 11, 1979 and January 4, 1980 in Civil Case No. 7303-P, subject
matter of the Petition in G.R. No. 52361, are set aside and said Judge is
ordered to try the case on the merits. The orders dated December 14, 1979
and January 14, 1980 in Civil Case No. 7530-P, subject matter of the petition
in G.R. No. 52524 are set aside and the case is ordered remanded to the
court a quo, City Court of Pasay City, for trial on the merits, with costs
against the private respondents.

TOPIC: CONTROVERSIES ARISING OUT OF INTRA-CORPORATE OR


PARTNERSHIP
RELATIONS
WESTERN INSTITUTE OF TECHNOLOGY, INC., HOMERO L. VILLASIS,
DIMAS ENRIQUEZ, PRESTON F. VILLASIS & REGINALD F. VILLASIS,
petitioner,
vs.

Page 945 of 1072

RICARDO T. SALAS, SALVADOR T. SALAS, SOLEDAD SALAS-TUBILLEJA,


ANTONIO S. SALAS, RICHARD S. SALAS & HON. JUDGE PORFIRIO
PARIAN, respondents.
G.R. No. 113032. August 21, 1997
FACTS:
Private respondents Ricardo T. Salas, Salvador T. Salas, Soledad SalasTubilleja, Antonio S. Salas, and Richard S. Salas, belonging to the same
family, are the majority and controlling members of the Board of Trustees of
Western Institute of Technology, Inc. (WIT, for short), a stock corporation
engaged in the operation, among others, of an educational institution.
According to petitioners, the minority stockholders of WIT, sometime on June
1, 1986 in the principal office of WIT at La Paz, Iloilo City, a Special Board
Meeting was held. In attendance were other members of the Board including
one of the petitioners Reginald Villasis. Prior to aforesaid Special Board
Meeting, copies of notice thereof, dated May 24, 1986, were distributed to all
Board Members. In said meeting, the Board of Trustees passed Resolution No.
48, s. 1986, granting monthly compensation to the private respondents as
corporate officers retroactive June 1, 1985.
A few years later, that is, on March 13, 1991, petitioners Homero
Villasis, Prestod Villasis, Reginald Villasis and Dimas Enriquez filed an
affidavit-complaint against private respondents before the Office of the City
Prosecutor of Iloilo, as a result of which two (2) separate criminal
informations, one for falsification of a public document under Article 171 of
the Revised Penal Code and the other for estafa under Article 315, par. 1(b)
of the RPC, were filed before Branch 33 of the Regional Trial Court of Iloilo
City.
ISSUE/S:
Whether or not the SEC has jurisdiction over the case filed.
RULING:
Once the case is decided by the SEC, the losing party may file a
petition for review before the Court of Appeals raising questions of fact, of
law, or mixed questions of fact and law. It is only after the case has ran this
course, and not earlier, can it be brought to us via a petition for review on
certiorari under Rule 45 raising only pure questions of law.
This Court is reluctant to accept this claim of falsification. The
prosecution omitted to submit the complete minutes of the regular meeting
of the Board of Trustees on March 30, 1986. It is the perception of this Court
that the grant of compensation or salary to the accused in their capacity as

Page 946 of 1072

officers of the corporation, through Resolution No. 48, enacted on March 30,
1986 by the Board of Trustees, is authorized by both the Articles of
Incorporation and the By-Laws of the corporation. To state otherwise is to
depart from the clear terms of the said articles and by-laws. In their defense
the accused have properly and rightly asserted that the grant of salary is not
for directors, but for their being officers of the corporation who oversee the
day to day activities and operations of the school.

TOPIC: CONTROVERSIES IN THE ELECTION OR APPOINTMENT/ DISMISSAL OF


CORPORATE OFFICERS
RENATO REAL, Petitioner,
vs.
SANGU PHILIPPINES, INC. and/ or KIICHI ABE, Respondents.
G.R. No. 168757. January 19, 2011

Page 947 of 1072

FACTS:
Petitioner was removed from his position as Manager of Respondent
Corporation, a corporation engaged in the business of providing manpower
for general services, like janitors, janitresses and other maintenance
personnel, to various clients, through a Board Resolution adopted by the
Corporations Board of Directors. His dismissal was allegedly due to the
following:
(1) Continuous absences at his post at Ogino Philippines Inc. for
several months which was detrimental to the corporations
operation;
(2) Loss of trust and confidence; and,
(3) To cut down operational expenses to reduce further losses
being experienced by respondent corporation.
Consequently, petitioner together with 29 others filed a complaint for
illegal dismissal against the corporation and Kiichi Abe, the corporations
Vice-President and General Manager.
The Labor Arbiter declared petitioner and his co-complainants as
having illegally dismissed and thus, ordered reinstatements.
On appeal by respondents, the NLRC modified the LAs decision.
Accordingly, the complaint of petitioner was dismissed for lack of jurisdiction
and as to the rest of the complaints, they are hereby ordered to immediately
report back to work but without the payment of backwages. Said decision
was affirmed by the CA.
Hence, this Petition for Review on Certiorari.

ISSUE/S:
Whether petitioners complaint for illegal dismissal constitutes an
intra-corporate controversy and thus, beyond the jurisdiction of
the Labor Arbiter.
RULING:
No.
The Supreme Court declared that jurisdiction should be determined by
considering not only the status or relationship of the parties, but also the
nature of the question under controversy. This two-tier test was adopted in
the recent case of Speed Distribution Inc. v. Court of Appeals:
To determine whether a case involves an intra-corporate controversy,
and is to be heard and decided by the branches of the RTC specifically

Page 948 of 1072

designated by the Court to try and decide such cases, two elements
must concur:
(1) the status or relationship of the parties, and
(2) the nature of the question that is the subject of their
controversy.
The first element requires that the controversy must arise out of intracorporate or partnership relations between any or all of the parties and the
corporation, partnership, or association of which they are not stockholders,
members or associates, between any or all of them and the corporation,
partnership or association of which they are stockholders, members or
associates, respectively; and between such corporation, partnership, or
association and the State insofar as it concerns the individual franchises.
The second element requires that the dispute among the parties be
intrinsically connected with the regulation of the corporation. If the nature of
the controversy involves matters that are purely civil in character,
necessarily, the case does not involve an intra-corporate controversy.
Guided by this recent jurisprudence, the Court thus finds no merit in
respondents contention that the fact alone that petitioner is a stockholder
and director of Respondent Corporation automatically classifies this case as
an intra-corporate controversy. To reiterate, not all conflicts between the
stockholders and the corporation are classified as intra-corporate. There are
other factors to consider in determining whether the dispute involves
corporate matters as to consider them as intra-corporate controversies.
"Corporate officers in the context of Presidential Decree No. 902-A are
those officers of the corporation who are given that character by the
Corporation Code or by the corporations by-laws. There are three specific
officers whom a corporation must have under Section 25 of the Corporation
Code. These are the president, secretary and the treasurer. The number of
officers is not limited to these three. A corporation may have such other
officers as may be provided for by its by-laws like, but not limited to, the
vice-president, cashier, auditor or general manager. The number of corporate
officers is thus limited by law and by the corporations by-laws."
The Supreme Court however examined the records of this case and
found nothing to prove that petitioners appointment was made pursuant to
the provision of respondent corporations By-Laws. No copy of board
resolution appointing petitioner as Manager or any other document showing
that he was appointed to said position by action of the board was submitted
by respondents. What was found instead were mere allegations of
respondents in their various pleadings that petitioner was appointed as
Manager of respondent corporation and nothing more. "The Court has

Page 949 of 1072

stressed time and again that allegations must be proven by sufficient


evidence because mere allegation is definitely not evidence."
Thus, that there is no intra-corporate relationship between the parties
insofar as petitioners complaint for illegal dismissal is concerned and that
same does not satisfy the relationship test. Petitioners complaint likewise
does not satisfy the nature of controversy test.
Therefore, With the elements of intra-corporate controversy being
absent in this case, the Court thus holds that petitioners complaint for illegal
dismissal against respondents is not intra-corporate. Rather, it is a
termination dispute and, consequently, falls under the jurisdiction of the
Labor Arbiter pursuant to Section 217 of the Labor Code.

TOPIC: CONTROVERSIES IN THE ELECTION OR APPOINTMENT/ DISMISSAL OF


CORPORATE OFFICERS
MATLING INDUSTRIAL AND COMMERCIAL CORPORATION, RICHARD K.
SPENCER, CATHERINE SPENCER, AND ALEX MANCILLA, Petitioners,

Page 950 of 1072

vs.
RICARDO R. COROS, Respondent.
G.R. No. 157802. October 13, 2010
FACTS:
A complaint for illegal dismissal was filed with the NLRC, Iligan City, by
respondent, the Vice President for Finance and Administration of petitioner
Matling, against Matling and some of its corporate officers.
Petitioners moved to dismiss the complaint on the ground that the
complaint pertains to the jurisdiction of the SEC as it involves an intracorporate controversy in as much as respondent was a member of Matlings
Board of Directors aside from being its Vice-President for Finance and
Administration prior to his termination.
The Labor Arbiter granted the motion to dismiss, ruling that
respondent was a corporate officer and that his removal was under the
jurisdiction of the SEC, pursuant to Sec. 5 (c), of PD No. 902.
On appeal by respondent, the NLRC set aside the dismissal, concluding
that the complaint was properly cognizable by the Labor Arbiter, not the SEC
because petitioner was not a corporate officer as his position was not among
those listed in Matlings By-laws. Said decision was affirmed by the CA.
Hence, this petition.

ISSUE/S:
Whether the respondent was a corporate officer of Matling or not.
Whether the LA or the RTC had jurisdiction over his complaint for
illegal dismissal.
RULING:
Conformably with Section 25 of the Corporation Code, a position must
be expressly mentioned in the By-Laws in order to be considered as a
corporate office. Thus, the creation of an office pursuant to or under a ByLaw enabling provision is not enough to make a position a corporate office.
Guerrea v. Lezama, the first ruling on the matter, held that the only officers
of a corporation were those given that character either by the Corporation
Code or by the By-Laws; the rest of the corporate officers could be
considered only as employees or subordinate officials. Thus, it was held in
Easycall Communications Phils., Inc. v. King:
An "office" is created by the charter of the corporation and the
officer is elected by the directors or stockholders. On the other
hand, an employee occupies no office and generally is employed
not by the action of the directors or stockholders but by the

Page 951 of 1072

managing officer of the corporation who also determines the


compensation to be paid to such employee.

In the case at bar, Respondent was appointed vice president for


nationwide expansion by Malonzo, petitioner's general manager, not by the
board of directors of petitioner. It was also Malonzo who determined the
compensation package of respondent. Thus, respondent was an employee,
not a "corporate officer." The CA was therefore correct in ruling that
jurisdiction over the case was properly with the NLRC, not the SEC
(now the RTC).

As a rule, the illegal dismissal of an officer or other employee of a


private employer is properly cognizable by the LA. This is pursuant to Article
217 (a) 2 of the Labor Code, as amended. However, Where the complaint for
illegal dismissal concerns a corporate officer, the controversy falls under the
jurisdiction of the Securities and Exchange Commission (SEC), because the
controversy arises out of intra-corporate or partnership relations between
and among stockholders, members, or associates, or between any or all of
them and the corporation, partnership, or association of which they are
stockholders, members, or associates, respectively; and between such
corporation, partnership, or association and the State insofar as the
controversy concerns their individual franchise or right to exist as such
entity; or because the controversy involves the election or appointment of a
director, trustee, officer, or manager of such corporation, partnership, or
association. Such controversy, among others, is known as an intra-corporate
dispute.
Effective on August 8, 2000, upon the passage of Republic Act No.
8799, otherwise known as The Securities Regulation Code, the SECs
jurisdiction over all intra-corporate disputes was transferred to the RTC,
pursuant to Section 5.2 of RA No. 8799, to wit:
5.2. The Commissions jurisdiction over all cases enumerated
under Section 5 of Presidential Decree No. 902-A is hereby
transferred to the Courts of general jurisdiction or the
appropriate Regional Trial Court: Provided, that the Supreme
Court in the exercise of its authority may designate the Regional
Trial Court branches that shall exercise jurisdiction over these
cases. The Commission shall retain jurisdiction over pending
cases involving intra-corporate disputes submitted for final
resolution which should be resolved within one (1) year from the
enactment of this Code. The Commission shall retain jurisdiction
over pending suspension of payments/rehabilitation cases filed
as of 30 June 2000 until finally disposed.

Page 952 of 1072

TOPIC: CONTROVERSIES IN THE ELECTION OR APPOINTMENT/ DISMISSAL OF


CORPORATE OFFICERS
ATTY. VIRGILIO R. GARCIA, Petitioner,
vs.
EASTERN TELECOMMUNICATIONS PHILIPPINES, INC. and ATTY.
SALVADOR C. HIZON, Respondents.

Page 953 of 1072

G.R. No. 173115. April 16, 2009


FACTS:
Petitioner was the Vice President and Head of business Support
Services and Human Resource Departments of respondent ETPI. Respondent
Hizon is the President and Chief Executive Officer of ETPI.
Three complaints for sexual harassment were filed against petitioner
by Atty. Maria Larrie Alinsunurin, former manager of ETPIs Office of the Legal
Counsel; Ms. Emma Valeros-Cruz, Assistant Vice President of ETPI and former
secretary of Atty. Garcia; and Dr. Mercedita M. Macalintal, medical
retainer/company physician of ETPI. By reason of said complaints, a
Committee on Decorum was constituted to investigate the complaints. While
investigation was on going, petitioner was placed under preventive
suspension. After said investigation, the Committee recommended
petitioners dismissal. Consequently, Atty. Hizon advised petitioner that his
employment was terminated.
Thereafter, petitioner filed a complaint for illegal dismissal against ETPI
and Atty. Hizon.
The Labor Arbiter rendered
suspension and the dismissal illegal.

decision

finding

the

preventive

On appeal filed by respondents, the NLRC reversed the decision of the


Labor arbiter and dismissed the case for lack of jurisdiction.
ISSUE/S:
Whether The Question Of Legality Or Illegality Of The Removal Or
Termination Of Employment Of An Officer Of A Corporation Is An
Intra-Corporate Controversy That Falls Under The Original
Exclusive Jurisdiction Of The Regional Trial Courts?
RULING:
Yes.
The Supreme Court, in a long line of cases, has decreed that a
corporate officers dismissal or removal is always a corporate act and/or an
intra-corporate controversy, over which the Securities and Exchange
Commission [SEC] (now the Regional Trial Court) has original and exclusive
jurisdiction.
We have ruled that an intra-corporate controversy is one which
pertains to any of the following relationships:
(1) between the corporation, partnership or association and the
public;

Page 954 of 1072

(2) between the corporation, partnership or association and the


State insofar as the formers franchise, permit or license to
operate is concerned;
(3) between the corporation, partnership or association and its
stockholders, partners, members or officers; and
(4) among the stockholders, partners or associates themselves.
In Lozon v. National Labor Relations Commission, we declared
that Presidential Decree No. 902-A confers on the SEC original
and exclusive jurisdiction to hear and decide controversies and
cases involving intra-corporate and partnership relations
between or among the corporation, officers and stockholders and
partners, including their elections or appointments x x x.
Before a dismissal or removal could properly fall within the jurisdiction
of the SEC, it has to be first established that the person removed or
dismissed was a corporate officer."Corporate officers" in the context of
Presidential Decree No. 902-A are those officers of the corporation who are
given that character by the Corporation Code or by the corporations bylaws. There are three specific officers whom a corporation must have under
Section 25 of the Corporation Code. These are the president, secretary and
the treasurer. The number of officers is not limited to these three. A
corporation may have such other officers as may be provided for by its bylaws like, but not limited to, the vice-president, cashier, auditor or general
manager. The number of corporate officers is thus limited by law and by the
corporations by-laws.
In the case before us, the by-laws of ETPI provide:
ARTICLE V: Officers
Section 1. Number. The officers of the Company shall be a
Chairman of the Board, a President, one or more VicePresidents, a Treasurer, a Secretary, an Assistant Secretary,
and such other officers as may be from time to time be
elected or appointed by the Board of Directors. One person
may hold any two compatible offices.
One of the corporate officers provided for in the by-laws of ETPI is the
Vice-President. It can be gathered from Atty. Garcias complaint-affidavit that
he was Vice President for Business Support Services and Human Resource
Departments of ETPI when his employment was terminated effective 16 April
2000. It is therefore clear from the by-laws and from Atty. Garcia himself that
he is a corporate officer. One who is included in the by-laws of a corporation
in its roster of corporate officers is an officer of said corporation and not a
mere employee. Being a corporate officer, his removal is deemed to be an
intra-corporate dispute cognizable by the SEC and not by the Labor Arbiter.

Page 955 of 1072

The Court agrees with both the NLRC and the Court of Appeals that
Atty. Garcias ouster as Vice-President, who is a corporate officer of ETPI,
partakes of the nature of an intra-corporate controversy, jurisdiction over
which is vested in the SEC (now the RTC). The Labor Arbiter thus erred in
assuming jurisdiction over the case filed by Atty. Garcia, because he had no
jurisdiction over the subject matter of the controversy.

TOPIC: CONTROVERSIES IN THE ELECTION OR APPOINTMENT/ DISMISSAL OF


CORPORATE OFFICERS
ARMANDO T. DE ROSSI, petitioner,
vs.
NATIONAL LABOR RELATIONS COMMISSION (First Division), MATLING
INDUSTRIAL AND COMMERCIAL CORPORATION AND RICHARD K.
SPENCER, respondents.

Page 956 of 1072

G.R. No. 108710. September 14, 1999


FACTS:
Petitioner, an Italian Citizen, was the Executive Vice President and
General Manager of private respondent Matling. His employment was
terminated on the following grounds:

Failure to secure his employment permit;


Gross mismanagement of the business affairs of the
company; and
Misused of corporate funds.

Consequently, he filed a complaint with the NLRC for illegal dismissal


with corresponding damages.
The Labor Arbiter rendered a decision in favor of petitioner by ordering
his reinstatement and awarding him backwages. Thus, a writ of execution
was issued to execute said order.
On appeal by Matling, the NLRC dismissed the case by virtue of Section
5, Par. (c), of P.D. No. 902-A.
Hence, this petition for certiorari.

ISSUE/S:
Whether or not the National Labor Relations Commission
Committed Grave Abuse Of Discretion Amounting To Lack Of
Jurisdiction Or Acted In Excess Of Its Jurisdiction In Holding That
The Securities And Exchange Commission Has Jurisdiction Over
The Complaint For Illegal Dismissal Filed By Petitioner.

RULING:
No.
The Supreme Court agrees with the public respondent's submission
through the Solicitor General. In a string of cases this Court has consistently
held that the SEC, and not the NLRC, has original and exclusive jurisdiction
over cases involving the removal of corporate officers. Section 5, paragraph
(c) of P.D. 902-A unequivocally provides that SEC has jurisdiction over intracorporate affairs regarding the election or appointment of officers of a
corporation, to wit:

Page 957 of 1072

Sec. 5. In addition to the regulatory and adjudicative


functions of the Securities and Exchange Commission over
corporations, partnerships and other forms of associations
registered with it as expressly granted under existing laws and
decrees, it shall have original and exclusive jurisdiction to hear
and decide cases involving:
xxx xxx xxx
(c) Controversies in the election or appointments of
directors, trustees, officers or managers of such
corporation, partnership or association.
We have earlier pronounced that an "office" is created by the charter of
the corporation under which a corporation is organized, and the officer is
elected by the directors or stockholders. In the present case, private
respondents aver that the officers and their terms of office are prescribed by
the corporation's by-laws, which provide as follows:
BY-LAW NO. III Directors and Officers
xxx xxx xxx
The officers of the corporation shall be the President, Executive
Vice President, Secretary and Treasurer, each of whom may hold his
office until his successor is elected and qualified, unless sooner
removed by the Board of Directors; Provided, That for the convenience
of the corporation the office of the Secretary and Treasurer may be
held by one and the same person. Officers shall be designated by the
stockholders' meeting at the time they elect the members of the Board
of Directors. Any vacancy occurring among the officers of the
Corporation on account of removal or resignation shall be filled by a
stockholder's meeting. Stockholders holding one half, or more of the
subscribed capital stock of the corporation may demand and compel
the resignation of any officer at any time.
The by-laws being in force, clearly petitioner is considered an officer of
MICC, elected and/or designated by its board of directors. Following Section
5(c) of P.D. No. 902-A, the SEC exercises exclusive jurisdiction over
controversies regarding the election and/or designation of directors, trustees,
officers or managers of a corporation, partnership or association. This
provision is indubitably applicable to the petitioner's case. Jurisdiction here is
not with the Labor Arbiter nor the NLRC, but with the SEC.
Note that a corporate officer's removal from his office is a corporate
act. If such removal occasions an intra-corporate controversy, its nature is
not altered by the reason or wisdom, or lack thereof, with which the Board of
Directors might have in taking such action. When petitioner, as Executive

Page 958 of 1072

Vice-President allegedly diverted company funds for his personal use


resulting in heavy financial losses to the company, this matter would amount
to fraud. Such fraud would be detrimental to the interest not only of the
corporation but also of its members. This type of fraud encompasses
controversies in a relationship within the corporation covered by SEC
jurisdiction. Perforce, the matter would come within the area of corporate
affairs and management, and such a corporate controversy would call for the
adjudicative expertise of the SEC, not the Labor Arbiter or the NLRC.

TOPIC: CONTROVERSIES IN THE ELECTION OR APPOINTMENT/ DISMISSAL OF


CORPORATE OFFICERS
LESLIE W. ESPINO, petitioner,

Page 959 of 1072

vs.
HON. NATIONAL LABOR RELATIONS COMMISSION and PHILIPPINE AIR
LINES, respondents.
G.R. Nos. 109642-43. January 5, 1995
FACTS:
Petitioner was the Executive Vice President- Chief Operating Officer of
private respondent PAL when his services were terminated by the Board of
Directors of PAL. His dismissal was based on the findings of the panels
created by then Pres. Cory Aquino to investigate petitioners involvement in
four cases denominated as GOLDAIR, ROBELLE, KASBAH/LA
PRIMAVERA, and MIDDLE EAST which allegedly prejudiced the interests of
both PAL and the Philippine Government.
Consequently, petitioner filed a complaint of illegal dismissal against
PAL with the NLRC. The Labor Arbiter rendered a decision finding that
petitioner was dismissed without just and valid cause and accordingly
ordered his reinstatement.
On appeal filed by PAL, the NLRC promulgated a resolution dismissing
the complaint for illegal dismissal for lack of jurisdiction.
Hence, this petition for certiorari.

ISSUE/S:
Whether or not the National Labor Relations Commission (NLRC)
has jurisdiction over a complaint filed by a corporate Executive
Vice President-Chief Operating Officer for illegal dismissal
resulting from the termination of his services as such officer by
virtue of four (4) separate resolutions of the Board of Directors
Air Lines (PAL).

RULING:
No.
The Court, citing Presidential Decree No. 902-A, laid down the rule in
the case of Philippine School of Business Administration v. Leano, and
consequently reiterated in three (3) other casesthat it is the Securities and
Exchange Commission (SEC) and not the NLRC which has original and
exclusive jurisdiction over cases involving the removal from employment of
corporate officers.

Page 960 of 1072

Sec. 5. of Presidential Decree No. 902-A regarding the jurisdiction of


the Securities and Exchange Commission provides, as follows:
Sec. 5. In addition to the regulatory and adjudicative functions of
the Securities and Exchange Commission over corporations,
partnerships and other forms of associations registered with it as
expressly granted under existing laws and decrees, it shall have
original and exclusive jurisdiction to hear and decide cases
involving:
xxx xxx xxx
(b) Controversies arising out of intracorporate or
partnership relations, between and among stockholders,
members, or associates; between any or all of them and
the corporation, partnership or association of which they
are stockholders, members, or associates, respectively;
and between such corporation, partnership or association
and the state insofar as it concerns their individual
franchise or right to exist as such entity.
(c) Controversies in the election or appointments of
directors, trustees, officers or managers of such
corporations, partnerships or associations.
xxx xxx xxx
In intra-corporate concerning the election or appointment of officers of
a corporation, Section 5, PD 902-A specifically provides:
Sec. 5. In addition to the regulatory and adjudicative functions of
the Securities and Exchange Commission over corporations,
partnerships and other forms of associations registered with it as
expressly granted under existing laws and decrees, it shall have
original and exclusive jurisdiction to hear and decide cases
involving:
xxx xxx xxx
(c) Controversies in the election or appointments of
directors, trustees, officers or managers of such
corporations, partnerships or associations.
Indisputably, the position of Executive Vice President-Chief Operating
Officer from which petitioner Espino claims to have been illegally dismissed,
is an elective office under Section 7, Article III is an elective corporate office
under Section 1, Article IV of the Amended by-Laws of PAL. The said
corporate office has a fixed term of one (1) year and the one elected shall
hold office until a successor shall have been elected and qualified. He lost

Page 961 of 1072

that position when his appointment or election as Executive Vice PresidentChief Operating Officer, together with other senior officers who were similarly
charged administratively, were deferred by the Board of Directors in its
organizational meeting on October 19, 1990. He was later considered by the
Board as resigned from the service, for reasons earlier stated, and the said
position was later abolished.
The matter of petitioner's not being elected to the office of Executive
Vice-President-Chief Operating Officer thus falls squarely within the purview
of Section 5 par. (c) of P.D. 902-A. In the case of PSBA v. Leano, supra, which
involved an Executive Vice President who was not re-elected to the said
position during the election of officers on September 5, 1981 by the PSBA's
newly elected Board of Directors, the Court emphatically stated:
This is not a case of dismissal. The situation is that of a corporate office
having been declared vacant, and that of TAN's not having been elected
thereafter. The matter of whom to elect is a prerogative that belongs to the
Board, and involves the exercise of deliberate choice and the faculty of
discriminative selection. Generally speaking, the relationship of a person to a
corporation, whether as officer or as agent or employee, is not determined
by the nature of the services performed, but by the incidents of the
relationship as they actually exists.
A corporate officer's dismissal is always a corporate act and/or an
intra-corporate controversy and that nature is not altered by the reason or
wisdom which the Board of Directors may have in taking such action.
Furthermore, it must be noted that the reason behind the non-election
of petitioner to the position of Executive Vice President-Chief Operating
Officer arose from, or is closely connected with, his involvement in the
alleged irregularities in the aforementioned cases which, upon investigation
and recommendation, were resolved by the PAL Board of Directors against
him and other senior officers. Evidently, this intra-corporate ruling places the
instant case under the specialized competence and expertise of the SEC.
The jurisdiction of the SEC has likewise been clarified by this Court in
the case of Union Glass and Container Corporation, et al. v. SEC, et al., thus:
This grant of jurisdiction must be viewed in the light of the nature and
function of the SEC under the law. Section 3 of PD No. 902-A confers upon
the latter "absolute jurisdiction, supervision, and control over all
corporations, partnerships or associations, who are grantees of primary
franchise and/or license or permit issued by the government to operate in
the Philippines . . . ." The principal function of the SEC is the supervision and
control over corporations, partnerships and associations with the end in view
that investment in these entities may be encouraged and protected, and
their activities pursued for the promotion of economic development.

Page 962 of 1072

It is in aid of this office that the adjudicative power of the SEC must be
exercised. Thus the law explicitly specified and delimited its jurisdiction to
matters intrinsically connected with the regulations of corporations,
partnerships and associations and those dealing with the internal affairs of
such corporations, partnerships or associations.
Otherwise stated, in order that the SEC can take cognizance of a case,
the controversy must pertain to any of the following relationships:
(a) between the corporation, partnership or association and the
public;
(b) between the corporation, partnership or association and its
stockholders, partners, members, or officers;
(c) between the corporation, partnership or association and the
state in so far as its franchise, permit or license to operate is
concerned, and
(d) among the stockholders, partners or associates themselves.
The fact that petitioner sought payment of his backwages, other
benefits, as well as moral and exemplary damages and attorney's fees in his
complaint for illegal dismissal will not operate to prevent the SEC from
exercising its jurisdiction under PD 902-A. While the affirmative reliefs and
monetary claims sought by petitioner in his complaint may, at first glance,
mislead one into placing the case under the jurisdiction of the Labor Arbiter,
a closer examination reveals that they are actually part of the perquisites of
his elective position; hence, intimately linked with his relations with the
corporation. In Dy v. NLRC, et al.,the Court, confronted with the same issue
ruled, thus:
The question of remuneration, involving as it does, a person who
is not a mere employee but a stockholder and officer, an integral
part, it might be said, of the corporation, is not a simple labor
problem but a matter that comes within the area of corporate
affairs and management, and is in fact a corporate controversy in
contemplation of the Corporation Code.
The Court has likewise ruled in the case of Andaya v. Abadia that in
intra-corporate matters, such as those affecting the corporation, its directors,
trustees, officers and shareholders, the issue of consequential damages may
just as well be resolved and adjudicated by the SEC. Undoubtedly, it is still
within the competence and expertise of the SEC to resolve all matters arising
from or closely connected with all intra-corporate disputes.
Petitioner's reliance on the principle of estoppel to justify the exercise
or jurisdiction by the NLRC over the instant complaint is misplaced. it is not
accurate for petitioner to conclude that PAL did not raise the issue of
jurisdiction at the initial stages of the case, for, while it may be predicated on
a different ground, i.e., that appeal from the resolution of the Board of

Page 963 of 1072

Directors of PAL as regards termination of his services, is to the Office of the


President, PAL did in fact question the jurisdiction of the Labor Arbiter. An
error of this nature, under the circumstances, could not justify petitioner's
insistence that PAL did not raise the issue of jurisdiction at the outset, but
only before the NLRC.
It is well-settled that jurisdiction over the subject matter is conferred
by law and the question of lack of jurisdiction may be raised at anytime even
on appeal. 11 The principle of estoppel cannot be invoked to prevent this
Court from taking up the question, which has been apparent on the face of
the pleadings since the start of the litigation before the Labor Arbiter. In the
case of Dy v. NLRC, supra, the Court, citing the case of Calimlim v. Ramirez
reiterated that the decision of a tribunal not vested with appropriate
jurisdiction is null and void. Again, the Court in Southeast Asian Fisheries
Development Center-Aquaculture Department v. NLRC restated the rule that
the invocation of estoppel with respect to the issue of jurisdiction is
unavailing because estoppel does not apply to confer jurisdiction upon a
tribunal that has none over the cause of action. The instant case does not
provide an exception to the said rule.
In fine, the issue of the SEC's jurisdiction is settled and the Court finds
it unnecessary to dwell further on other questions raised by petitioner. Thus,
finding no grave abuse of discretion on the part of NLRC in dismissing the
complaint for illegal dismissal, the instant petition must be dismissed.

Page 964 of 1072

TOPIC: CONTROVERSIES IN THE ELECTION OR APPOINTMENT/ DISMISSAL OF


CORPORATE OFFICERS
JOSEMARIA G. ESTRADA, petitioner,
vs.
THE HONORABLE NATIONAL LABOR RELATIONS COMMISSION and
PHILIPPINE AIRLINES, INC., respondents.
G.R. No. 106722. October 4, 1996
FACTS:
Petitioner was the Senior Vice-President of the Marketing Group of
private respondent PAL and who was administratively charged and
preventively suspended after the then Solicitor General Francisco Chavez
implicated him in the 2 Billion anomaly in PAL.
After investigation, the investigating committee recommended
petitioners dismissal from service. Consequently, PALs Board of Directors
passed a resolution declaring petitioner resigned from service effective
immediately for loss of confidence and acts inimical to the interest of the
company.
Thereafter, petitioner filed a complaint for illegal dismissal. In its
decision, the Labor Arbiter found that petitioner was illegally dismissed and
thus, ordered PAL to reinstate him in his previous position and to pay him
backwages and other benefits.
On appeal by PAL, the NLRC ordered the dismissal of the petitioners
complaint holding that jurisdiction over the case lies with the SEC.
Hence, this petition for certiorari.

ISSUE/S:
Whether or not the NLRC has jurisdiction over the case for illegal
termination filed by petitioner
RULING:
The Supreme Court notes that the issues raised herein have already
been passed upon in Lozon v. National Labor Relations Commission, et. al.
and Espino v. National Labor Relations Commission, et. al. In fact, in those

Page 965 of 1072

cases Lozon and Espino, together with herein petitioner Estrada, were among
the several Executive Vice-Presidents of PAL who were dismissed by the
Board for their involvement in the same P2 billion PAL -anomaly. Lozon and
Espino, just like herein petitioner, sued PAL for illegal dismissal. The Labor
Arbiter's decision in their favor was reversed and ordered dismissed by the
NLRC on appeal for lack of jurisdiction. On certiorari, the Court ruled as
follows:
In Fortune Cement Corporation v. NLRC, the Court has quoted
with approval the Solicitor General's contention that "a corporate
officer's dismissal is always a corporate act and/or intracorporate controversy and that nature is not altered by the
reason or wisdom which the Board of Directors may have in
taking such action." Not the least insignificant in the case at
bench is that petitioner's dismissal is intertwined with still
another intra-corporate affair, earlier so ascribed as the "twobillion-peso PAL scam," that inevitably places the case under the
specialized competence of the SEC and well beyond the ambit of
a labor arbiter's normal jurisdiction under the general provisions
of Article 217 of the Labor Code.
xxx xxx xxx
The fact that petitioner sought payment of his backwages,
other benefits, as well as moral and exemplary damages and
attorney's fees in his complainant for illegal dismissal will not
operate to prevent the SEC from exercising its jurisdiction under
PD 902-A. While the affirmative reliefs and monetary claims
sought by petitioner in his complaint may, at first glance,
mislead one into placing the case under the jurisdiction of the
Labor Arbiter, a closer examination reveals that they are actually
part of the perquisites of his elective position; hence, intimately
linked with his relations with the corporation.
The Court fails to see any cogent reason, and none was persuasively
presented, why the above ruling should not be applied to the case at bench.

Page 966 of 1072

TOPIC: CONTROVERSIES IN THE ELECTION OR APPOINTMENT/ DISMISSAL OF


CORPORATE OFFICERS
ISLAMIC DIRECTORATE OF THE PHILIPPINES, MANUEL F. PEREA and
SECURITIES & EXCHANGE COMMISSION, petitioners,
vs.
COURT OF APPEALS and IGLESIA NI CRISTO, respondents.
G.R. No. 117897. May 14, 1997
FACTS:
Islamic Directorate of the Philippines (IDP) was allegedly organized and
incorporated in 1971 by Islamic leaders of all Muslim Major tribal Groups in
the Philippines headed by Dean Cesar Adib Majul. The Primary Purpose of
which is to establish an Islamic Center in Quezon City for the construction of
a "Mosque (prayer place), Madrasah (Arabic School), and other religious
infrastructures" so as to facilitate the effective practice of Islamic faith in the
area. Moreover, the Board of Trustees of the IDP was composed of the
following per Article 6 of its Articles of Incorporation:
Senator Mamintal Tamano
Congressman Ali Dimaporo
Congressman Salipada Pendatun
Dean Cesar Adib Majul
Sultan Harun Al-Rashid Lucman
Delegate Ahmad Alonto
Commissioner Datu Mama Sinsuat
Mayor Aminkadra Abubakar
As such, the Libyan Government donated money to IDP to purchase a
land, covered by two titles, at Culiat, Tandang Sora, Quezon City. After the
purchase, Martial Law was declared in the Philippines. Thus, most of the
1971 Board of Trustees flew to the Middle East to escape political
persecution.
Thereafter, the IDP-Carpizo Group and the IDP-Abbas group sprung.
Both groups claimed to be the legitimate IDP.
In a decision rendered by the SEC, the elections of the Board members
by both groups were declared NULL and VOID. However, the Carpizo Group,
in an alleged Board resolution of the IDP, authorized the sale of the two

Page 967 of 1072

parcels of land owned by IDP to the private respondent INC as evidenced by


a Deed of Sale.
The SEC declared said sale Null and Void. Such appeal was ordered set
aside by the CA.
Hence, this appeal.

ISSUE/S:
Did the Court of Appeals commit reversible error in setting aside
that portion of the SEC's Decision in SEC Case No. 4012 which
declared the sale of two (2) parcels of land in Quezon City
between the IDP-Carpizo Group and private respondent INC null
and void?
RULING:
Yes.
There can be no question as to the authority of the SEC to pass upon
the issue as to who among the different contending groups is the legitimate
Board of Trustees of the IDP since this is a matter properly falling within the
original and exclusive jurisdiction of the SEC by virtue of Sections 3 and 5(c)
of Presidential Decree No. 902-A:
Sec. 3. The Commission shall have absolute jurisdiction,
supervision and control over all corporations, partnership or
associations, who are the grantees of primary franchises and/or a
license or permit issued by the government to operate in the
Philippines . . . .
xxx xxx xxx
Sec. 5. In addition to the regulatory and adjudicative functions of
the Securities and Exchange Commission over corporations,
partnerships and other forms of associations registered with it as
expressly granted under existing laws and decrees, it shall have
original and exclusive jurisdiction to hear and decide cases
involving:
xxx xxx xxx
c) Controversies in the selection or appointment of
directors, trustees, officers, or managers of such
corporations, partnerships or associations. . . . .
If the SEC can declare who is the legitimate IDP Board, then by parity
of reasoning, it can also declare who is not the legitimate IDP Board. This is
precisely what the SEC did in SEC Case No. 4012 when it adjudged the
election of the Carpizo Group to the IDP Board of Trustees to be null and void.
By this ruling, the SEC in effect made the unequivocal finding that the IDPCarpizo Group is a bogus Board of Trustees. Consequently, the Carpizo Group

Page 968 of 1072

is bereft of any authority whatsoever to bind IDP in any kind of transaction


including the sale or disposition of ID property.
It must be noted that SEC Case No. 4012 is not the first case wherein
the SEC had the opportunity to pass upon the status of the Carpizo Group. As
far back as October 3, 1986, the SEC, in Case No. 2687, in a suit between the
Carpizo Group and the Abbas Group, already declared the election of the
Carpizo Group (as well as the Abbas Group) to the IDP Board as null and void
for being violative of the Articles of Incorporation. Nothing thus becomes
more settled than that the IDP-Carpizo Group with whom private respondent
INC contracted is a fake Board.
Premises considered, all acts carried out by the Carpizo Board, particularly
the sale of the Tandang Sora property, allegedly in the name of the IDP, have
to be struck down for having been done without the consent of the IDP thru a
legitimate Board of Trustees. Article 1318 of the New Civil Code lays down
the essential requisites of contracts:
There is no contract unless the following requisites concur:
(1) Consent of the contracting parties;
(2) Object certain which is the subject matter of the
contract;
(3) Cause of the obligation which is established.
All these elements must be present to constitute a valid contract. For,
where even one is absent, the contract is void. As succinctly put by Tolentino,
consent is essential for the existence of a contract, and where it is wanting,
the contract is non-existent. 38 In this case, the IDP, owner of the subject
parcels of land, never gave its consent, thru a legitimate Board of Trustees,
to the disputed Deed of Absolute Sale executed in favor of INC. This is,
therefore, a case not only of vitiated consent, but one where consent on the
part of one of the supposed contracting parties is totally wanting.
Ineluctably, the subject sale is void and produces no effect whatsoever.

Page 969 of 1072

TOPIC: CONTROVERSIES IN THE ELECTION OR APPOINTMENT/ DISMISSAL OF


CORPORATE OFFICERS
BIENVENIDO ONGKINGCO, as President and GALERIA DE
MAGALLANES CONDOMINIUM ASSOCIATION, INC., petitioners,
vs.
NATIONAL LABOR RELATIONS COMMISSION and FEDERICO B.
GUILAS, respondents.
G.R. NO. 119877, MARCH 31, 1997
270 SCRA 613
FACTS:
Petitioner Galeria de Magallanes Condominium Association, Inc.
(Galeria for brevity) is a non-stock, non-profit corporation formed in
accordance with R.A. No. 4726, otherwise known as the Condominium Act,
with Bienvenido Ongkingco as its president.
On 1 September 1990, Galeria's Board of Directors appointed private
respondent Federico B. Guilas as Administrator/Superintendent subject to
review after five (5) months and subsequently thereafter as Galeria's
finances improved. Later a resolution was passed by the Board of Directors of
Galeria, private respondent was not re-appointed as Administrator. Thus,
private respondent instituted a complaint against petitioners for illegal
dismissal and non-payment of salaries with the NLRC.
Private respondent was a mere employee of petitioner corporation
being tasked mainly, as administrator/superintendent, with the upkeep of the
condominium's common areas, thus she cannot be deemed a corporate
officer covered by the SEC.
.
ISSUE/s:

Page 970 of 1072

Whether or not the NLRC is the proper authority to decide over


the case.
RULING:
No.
Private respondent is an officer of petitioner corporation by reason of
its by-laws specifically including the Superintendent/Administrator in its
roster of corporate officers. A corporate officer's dismissal is always a
corporate act and/or an intra-corporate controversy and that nature is not
altered by the reason or wisdom which the Board of Directors may have in
taking such action. Perforce, Section 5(c) of Presidential Decree No. 902-A,
which provides that the SEC exercises exclusive jurisdiction over
controversies in the election or appointment of directors, trustees, officers or
managers of corporations, partnerships or associations. Having thus
determined, his dismissal or non-appointment is clearly an intra-corporate
matter and jurisdiction, therefore, properly belongs to the SEC and not the
NLRC.

Page 971 of 1072

TOPIC: CONTROVERSIES IN THE ELECTION OR APPOINTMENT/ DISMISSAL OF


CORPORATE OFFICERS
EFREN P. PAGUIO, petitioner,
vs.
NATIONAL LABOR RELATIONS COMMISSION, METROMEDIA TIMES
CORPORATION, ROBINA Y. GOKONGWEI, LIBERATO GOMEZ, JR.,
YOLANDA E. ARAGON, FREDERICK D. GO and ALDA
IGLESIA,respondents
G.R. No. 147816
May 9, 2003
FACTS:
Respondent Metromedia Times Corporation entered, for the fifth time,
into an agreement with petitioner Efren P. Paguio, appointing the latter to be
an account executive of the firm. Petitioner was to solicit advertisements for
"The Manila Times," a newspaper of general circulation, published by
respondent company for a compensation consisting of a 15% commission on
direct advertisements less withholding tax and a 10% commission on agency
advertisements based on gross revenues less agency commission and the
corresponding withholding tax. The commissions were to be given to
petitioner only after the clients would have paid for the advertisements.
Apart from commissions, petitioner was also entitled to a monthly allowance
of P2,000.00 as long as he met the P30,000.00-monthly quota.
The issue arose when petitioner was issued notice of his termination
two months after his renewal and the statement that he was not an
employee of the respondent on ground of alleged misconduct i.e. pirating
clients from his co-executives and failing to produce results.
ISSUE/S:
Whether or not the dismissal of petitioner was valid.

Page 972 of 1072

RULING:
No.
Petitioner is not a corporate officer but a regular employee and thus his
case is recognizable by the NLRC, despite the written contract defining their
relationship.
Evident from the fact is the presence of control over the results to be
achieved but likewise the manner and the means used in reaching that
end. Metromedia Times Corporation exercised such control by requiring
petitioner, among other things, to submit a daily sales activity report and
also a monthly sales report as well. Petitioner also performed activities which
were necessary and desirable to the business of the employer. The notice of
termination recites no valid or just cause for the dismissal of petitioner nor
does it appear that he has been given an opportunity to be heard in his
defense, thus the dismissal was illegal.

Page 973 of 1072

TOPIC: CONTROVERSIES IN THE ELECTION OR APPOINTMENT/ DISMISSAL OF


CORPORATE OFFICERS
PEARSON & GEORGE, (S.E. ASIA), INC. petitioner,
vs.
NATIONAL LABOR RELATIONS COMMISSION and LEOPOLDO
LLORENTE, respondents.
G.R. No. 113928, February 1, 1996
253 SCRA 166
FACTS:
Private respondent Leopoldo Llorente was a member of the Board of
Directors of the petitioner and was elected as Vice-Chairman of the Board
and as Managing Director. On 29 January 1990, Llorente was preventively
suspended, with pay, by reason of alleged anomalous transactions entered
into by him, which were prejudicial to the interest of the petitioner. A reelection was conducted and Llorente was not re-elected.
ISSUE/S:
Whether or not the removal from the office by non-re-election of
private respondent is one of illegal dismissal case wherein NLRC
has jurisdiction to try and decide.
RULING:
The Court ruled in the negative. The removal of Llorente as Managing
Director is purely an intra-corporate dispute which falls within the exclusive
jurisdiction of the SEC and not of the NLRC, pursuant to its authority under
paragraphs (b) and (c), Section 5 of P.D. No. 902-A. The office of Managing
Director presupposes that its occupant is a Director; hence, one who is not a
Director of the petitioner or who has ceased to be a Director cannot be
elected or appointed as a Managing Director. Elsewise stated, the holding of

Page 974 of 1072

the position of Director is a prerequisite for the election, appointment, or


designation of Managing Director. If a Managing Director should lose his
position because he ceased to be a Director for any reason, such as nonreelection as in the case of Llorente, such loss is not dismissal but failure to
qualify or to maintain a prerequisite for that position. Then too, the position
of Managing Director was abolished.

TOPIC: CONTROVERSIES IN THE ELECTION OR APPOINTMENT/ DISMISSAL OF


CORPORATE OFFICERS
ERNESTO M. APODACA, petitioner,
vs.
NATIONAL LABOR RELATIONS COMMISSION, JOSE M. MIRASOL and
INTRANS PHILS., INC., respondents.
G.R. NO. 80039 APRIL 18, 1989
172 SCRA 442
FACTS:
Petitioner was employed in respondent corporation for which he
subscribed to 1,500 shares of respondent corporation at P100.00 per share
or a total of P150,000.00. Petitioner was later appointed President and
General Manager of the respondent corporation but resigned on January 2,
1986.
On December 19, 1986, petitioner instituted with the NLRC a complaint
against private respondents for the payment of his unpaid wages, his cost of
living allowance, the balance of his gasoline and representation expenses
and his bonus compensation for 1986. Private respondents admitted that
there is due to petitioner the amount of P17,060.07 but this was applied to
the unpaid balance of his subscription in the amount of P95,439.93.
Petitioner questioned the set-off alleging that there was no call or notice for
the payment of the unpaid subscription and that, accordingly, the alleged
obligation is not enforceable.
ISSUE/S:
Whether or not the NLRC has jurisdiction to resolve a claim for
non-payment of stock subscriptions to a corporation
RULING:
NO.

Page 975 of 1072

NLRC has no jurisdiction to determine such intra-corporate dispute


between the stockholder and the corporation as in the matter of unpaid
subscriptions. This controversy is within the exclusive jurisdiction of the
Securities and Exchange Commission.
Assuming arguendo that the NLRC may exercise jurisdiction over the
said subject matter under the circumstances of this case, the unpaid
subscriptions are not due and payable until a call is made by the corporation
for payment. Private respondents have not presented a resolution of the
board of directors of respondent corporation calling for the payment of the
unpaid subscriptions. It does not even appear that a notice of such call has
been sent to petitioner by the respondent corporation. As there was no
notice or call for the payment of unpaid subscriptions, the same is not yet
due and payable.
Assuming further that there was a call for payment of the unpaid
subscription, Article 113 of the Labor Code allows such a deduction from the
wages of the employees by the employer, only in three instances:
a. In cases where the worker is insured with his consent by
the employer, and the deduction is to recompense the
employer for the amount paid by him as premium on the
insurance;
b. For union dues, in cases where the right of the worker or
his union to checkoff has been recognized by the employer
or authorized in writing by the individual worker concerned;
and
c. In cases where the employer is authorized by law or
regulations issued by the Secretary of Labor

Page 976 of 1072

TOPIC: CONTROVERSIES IN THE ELECTION OR APPOINTMENT/ DISMISSAL OF


CORPORATE OFFICERS
PHIL. SCHOOL OF BUSINESS & ADMINISTRATION (PSBA) petitioner,
vs.
LACANDOLA LEANO OF NLRC & RUFINO TAN, respondents
G.R. NO. L 58468, FEBRUARY 24, 1984
FACTS:
Tan is one of the principal stockholders of PSBA who became its
director and executive vice president. In a meeting called for the re-election
of board of directors to fill the vacancies of three board seat, he was not reelected. He thus filed for illegal dismissal before the NLRC alleging lack of
valid cause and notice. He also filed similar case before the SEC questioning
the validity of the PSBA elections.
ISSUE/S:
Whether or not the SEC has jurisdiction over the case as it
involves an intra corporate controversy.
RULING:
Yes.
PSBA is a domestic corporation and whose general management s
exercised by the members of the Board elected annually by the stockholders
in an annual general meeting. Any vacancy is filled by a majority vote of the
submitted capital stock entitled to vote in a meeting specially called for the
purpose and the director so chosen hold the office for the unexpired term.
This is not a case of dismissal. The situation is that of a corporation
office having been declared vacant and of Tans not having been elected
thereafter. The matter of who to elect is a prerogative that belong to the
members and involves the exercise of deliberate choice ad the faculty of

Page 977 of 1072

discriminative selection. Generally speaking, the relationship of a person to a


corporation whether as officers or agent/employee is not determined by the
nature of the services performed but by the incidents of the relationship as
they actually exist.

TOPIC: CONTROVERSIES IN THE ELECTION OR APPOINTMENT/ DISMISSAL OF


CORPORATE OFFICERS
PURIFICACION G. TABANG, petitioner,
vs.
NATIONAL LABOR RELATIONS COMMISSION and PAMANA GOLDEN
CARE MEDICAL CENTER FOUNDATION, INC., respondents.
G.R. No. 121143 January 21, 1997
266 SCRA 462
FACTS:
Petitioner Purificacion Tabang was a founding member, a member of
the Board of Trustees, and the corporate secretary of private respondent
Pamana Golden Care Medical Center Foundation, Inc., a non-stock
corporation engaged in extending medical and surgical services.
On October 30, 1990, the Board of Trustees issued a memorandum
appointing petitioner as Medical Director and Hospital Administrator of
private respondent's Pamana Golden Care Medical Center in Calamba,
Laguna.
On May 1, 1993, petitioner was allegedly informed personally by Dr.
Ernesto Naval that the Board of Trustees passed a resolution relieving her of
her position as Medical Director and Hospital Administrator. Thus, petitioner
filled a complaint for illegal dismissal and non-payment of wages, allowances
and 13th month pay before the labor arbiter.
Respondent corporation moved for the dismissal of the complaint on
the ground of lack of jurisdiction over the subject matter. It argued that
petitioner's position as Medical Director and Hospital Administrator was
interlinked with her position as member of the Board of Trustees, hence, her
dismissal is an intra-corporate controversy which falls within the exclusive
jurisdiction of the Securities and Exchange Commission (SEC).

Page 978 of 1072

Petitioner opposed the motion to dismiss, contending that her position


as Medical Director and Hospital Administrator was separate and distinct
from her position as member of the Board of Trustees. She claimed that there
is no intra-corporate controversy involved since she filed the complaint in her
capacity as Medical Director and Hospital Administrator, or as an employee
of private respondent.
ISSUE/S:
Whether or not the NLRC has jurisdiction over the instant case.
RULING:
No.
It is the SEC which has jurisdiction over the case at bar. Contrary to the
contention of petitioner, a medical director and a hospital administrator are
considered as corporate officers under the by-laws of respondent
corporation. Section 2(i), Article I thereof states that one of the powers of the
Board
of
Trustees
is
"(t)o
appoint
a
Medical
Director,
Comptroller/Administrator, Chiefs of Services and such other officers as it
may deem necessary and prescribe their powers and duties." Perforce,
Section 5(c) of Presidential Decree No. 902-A, which provides that the SEC
exercises exclusive jurisdiction over controversies in the election
appointment of directors, trustees, officers or managers of corporations,
partnerships or associations, applies in the present dispute. Accordingly,
jurisdiction over the same is vested in the SEC, and not in the Labor Arbiter
or the NLRC.
A corporate officer's dismissal is always a corporate act, or an intracorporate controversy, and the nature is not altered by the reason or wisdom
with which the Board of Directors may have in taking such action.

Page 979 of 1072

TOPIC: CONTROVERSIES IN THE ELECTION OR APPOINTMENT/ DISMISSAL OF


CORPORATE OFFICERS
UNION MOTOR CORPORATION, petitioner,
vs.
NATIONAL LABOR RELATIONS COMMISSION and ALEJANDRO A.
ETIS, respondents.
G.R. NO. 159738, DECEMBER 9, 2004
314 SCRA 531
FACTS:
Respondent was hired by the petitioner as an automotive mechanic at
the service department in the latters Paco Branch and was transferred to the
Caloocan City Branch. Respondent informed the company that he had to take
a sick leave as he had a painful and unbearable toothache, which continued
on succeeding days as advised by the physician.
Petitioner terminated his services on ground of incurring a total of
more than five (5) consecutive absences without proper notification being an
abandonment of office under Section 6.1.1, Article III of the Company Rules.
ISSUE/S:
Whether or not respondents dismissal is valid.
RULING:
The evidence on record shows that the respondent informed the
petitioner of his illness through the company nurse. The security guard who
was dispatched by the petitioner to verify the information received by the
company nurse, confirmed the respondents illness. We find and so hold that
the respondent complied with the requisite of giving notice of his illness and
the reason for his absences to the petitioner.
Indeed, the physician and the dentist who examined the complainant,
aside from their respective letterheads, had written their respective license
numbers below their names and signatures. These facts have not been

Page 980 of 1072

impugned nor rebutted by respondent-appellee throughout the proceedings


of his case.
To warrant removal from service, the negligence should not merely be
gross but also habitual. Gross negligence implies a want or absence of or
failure to exercise slight care or diligence, or the entire absence of care. It
evinces a thoughtless disregard of consequences without exerting any effort
to avoid them. The petitioner has not sufficiently shown that the respondent
had willfully disobeyed the company rules and regulation. The petitioner also
failed to prove that the respondent abandoned his job. The bare fact that the
respondent incurred excusable and unavoidable absences does not amount
to an abandonment of his employment.

Page 981 of 1072

TOPIC: PETITIONS FOR DECLARATION IN THE STATE OF SUSPENSION


PAYMENT
ADVENT CAPITAL AND FINANCE CORPORATION vs. NICASIO I.
ALCANTARA and EDITHA I. ALCANTARA
G.R. No. 183050

January 25, 2012

FACTS:
On July 16, 2001 petitioner Advent Capital and Finance Corporation
(Advent Capital) filed a petition for rehabilitation1 with the Regional Trial
Court (RTC) of Makati City.2 Subsequently, the RTC named Atty. Danilo L.
Concepcion as rehabilitation receiver.3 Upon audit of Advent Capitals books,
Atty. Concepcion found that respondents Nicasio and Editha Alcantara
(collectively, the Alcantaras) owed Advent Capital P27,398,026.59,
representing trust fees that it supposedly earned for managing their several
trust accounts.4
Prompted by this finding, Atty. Concepcion requested Belson Securities,
Inc. (Belson) to deliver to him, as Advent Capitals rehabilitation receiver, the
P7,635,597.50 in cash dividends that Belson held under the Alcantaras Trust
Account 95-013. Atty. Concepcion claimed that the dividends, as trust fees,
formed part of Advent Capitals assets. Belson refused, however, citing the
Alcantaras objections as well as the absence of an appropriate order from
the rehabilitation court.5
Thus, Atty. Concepcion filed a motion before the rehabilitation court to
direct Belson to release the money to him. He said that, as rehabilitation
receiver, he had the duty to take custody and control of Advent Capitals
assets, such as the sum of money that Belson held on behalf of Advent
Capitals Trust Department.
ISSUE:

Page 982 of 1072

whether or not the cash dividends held by Belson and claimed by both
the Alcantaras and Advent Capital constitute corporate assets of the latter
that the rehabilitation court may, upon motion, require to be conveyed to the
rehabilitation receiver for his disposition.
RULING:
Cash dividends held by Belson and claimed by both the Alcantaras and
Advent Capital does not constitute corporate assets of the latter that the
rehabilitation court may, upon motion, require to be conveyed to the
rehabilitation receiver for his disposition.
Advent Capital asserts that the cash dividends in Belsons possession formed
part of its assets based on paragraph 9 of its Trust Agreement with the
Alcantaras,
According to Advent Capital, it could automatically deduct its management
fees from the Alcantaras portfolio that they entrusted to it. Paragraph 9 of
the Trust Agreement provides that Advent Capital could automatically deduct
its trust fees from the Alcantaras portfolio, at the end of each calendar
quarter, with the corresponding duty to submit to the Alcantaras a quarterly
accounting report within 20 days after.
But the problem is that the trust fees that Advent Capitals receiver was
claiming were for past quarters. Based on the stipulation, these should have
been deducted as they became due. As it happened, at the time Advent
Capital made its move to collect its supposed management fees, it neither
had possession nor control of the money it wanted to apply to its
claim. Belson, a third party, held the money in the Alcantaras
names. Whether it should deliver the same to Advent Capital or to the
Alcantaras is not clear. What is clear is that the issue as to who should get
the same has been seriously contested.
The real owner of the trust property is the trustor-beneficiary. In this case,
the trustors-beneficiaries are the Alcantaras. Thus, Advent Capital could not
dispose of the Alcantaras portfolio on its own. The income and principal of
the portfolio could only be withdrawn upon the Alcantaras written instruction
or order to Advent Capital. The latter could not also assign or encumber the
portfolio or its income without the written consent of the Alcantara. All
these are stipulated in the Trust Agreement.

Page 983 of 1072

TOPIC: PETITIONS FOR DECLARATION IN THE STATE OF SUSPENSION


PAYMENT
SIOCHI FISHERY ENTERPRISES, INC. vs. BANK OF THE PHILIPPINE
ISLANDS
G.R. No. 193872

October 19, 2011

FACTS:
Petitioners Siochi Fishery Enterprises, Inc., Jun-Jun Fishing Corporation,
Dede Fishing Corporation, Blue Crest Aqua-Farms, Inc. and Iloilo Property
Ventures, Inc. (petitioners) are domestic corporations of the Siochi family.
Petitioners are engaged in various businesses and have interlocking
stockholders and directors. Their principal office is located at 31 Don B.
Bautista Boulevard, Dampalit, Malabon City.
In the course of their business, petitioners borrowed from respondent
Bank of the Philippine Islands (BPI) and from Ayala Life Assurance, Inc. As of
30 June 2004, petitioners total obligation amounted to P85,362,262.05.
On 15 July 2004, petitioners filed with the RTC a petition5 for corporate
rehabilitation. Petitioners prayed that the RTC (1) issue a stay order; (2)
declare petitioners in a state of suspension of payments; (3) approve
petitioners proposed rehabilitation plan; and (4) appoint a rehabilitation
receiver.
ISSUE:
Whether or not petitioners are capable of rehabilitation.
RULING:

Page 984 of 1072

The rehabilitation plan is an indispensable requirement in corporate


rehabilitation proceedings. Section 5 of the Rules enumerates the essential
requisites of a rehabilitation plan:
The rehabilitation plan shall include (a) the desired business targets or
goals and the duration and coverage of the rehabilitation; (b) the terms and
conditions of such rehabilitation which shall include the manner of its
implementation, giving due regard to the interests of secured creditors; (c)
the material financial commitments to support the rehabilitation plan; (d) the
means for the execution of the rehabilitation plan, which may include
conversion of the debts or any portion thereof to equity, restructuring of the
debts, dacion en pago, or sale of assets or of the controlling interest; (e) a
liquidation analysis that estimates the proportion of the claims that the
creditors and shareholders would receive if the debtors properties were
liquidated; and (f) such other relevant information to enable a reasonable
investor to make an informed decision on the feasibility of the rehabilitation
plan.
The Court notes that petitioners failed to include a liquidation analysis
in their rehabilitation plan.

Page 985 of 1072

TOPIC: PETITIONS FOR DECLARATION IN THE STATE OF SUSPENSION


PAYMENT
JOSE MARCEL PANLILIO, ERLINDA PANLILIO, NICOLE MORRIS and
MARIO T. CRISTOBAL, Petitioners,
vs.
REGIONAL TRIAL COURT, BRANCH 51, CITY OF MANILA, represented
by HON. PRESIDING JUDGE ANTONIO M. ROSALES; PEOPLE OF THE
PHILIPPINES; and the SOCIAL SECURITY SYSTEM, Respondents.
G.R. No. 173846. February 2, 2011
FACTS:
On October 15, 2004, Jose Marcel Panlilio, Erlinda Panlilio, Nicole Morris
and Marlo Cristobal (petitioners), as corporate officers of Silahis International
Hotel, Inc. (SIHI), filed with the Regional Trial Court (RTC) of Manila, Branch
24, a petition for Suspension of Payments and Rehabilitationin SEC Corp.
Case No. 04-111180.
On October 18, 2004, the RTC of Manila, Branch 24, issued an
Orderstaying all claims against SIHI upon finding the petition sufficient in
form and substance.
At the time, however, of the filing of the petition for rehabilitation,
there were a number of criminal chargespending against petitioners in
Branch 51 of the RTC of Manila. These criminal charges were initiated by
respondent Social Security System (SSS) and involved charges of violations
of Section 28 (h)of Republic Act 8282, or the Social Security Act of 1997 (SSS
law), in relation to Article 315 (1) (b) of the Revised Penal Code, or Estafa.
Consequently, petitioners filed with the RTC of Manila, Branch 51, a
Manifestation and Motion to Suspend Proceedings. Petitioners argued that
the stay order issued by Branch 24 should also apply to the criminal charges
pending in Branch 51. Petitioners, thus, prayed that Branch 51 suspend its
proceedings until the petition for rehabilitation was finally resolved.

Page 986 of 1072

ISSUE/S:
Whether or not during the pendency of rehabilitation
proceedings, criminal charges for violation of Batas Pambansa
Bilang 22 should be suspended.
RULING:
The prosecution of the officers of the corporation has no bearing on the
pending rehabilitation of the corporation, especially since they are charged in
their individual capacities. Such being the case, the purpose of the law for
the issuance of the stay order is not compromised, since the appointed
rehabilitation receiver can still fully discharge his functions as mandated by
law. It bears to stress that the rehabilitation receiver is not charged to defend
the officers of the corporation. If there is anything that the rehabilitation
receiver might be remotely interested in is whether the court also rules that
petitioners are civilly liable. Such a scenario, however, is not a reason to
suspend the criminal proceedings, because as aptly discussed in Rosario,
should the court prosecuting the officers of the corporation find that an
award or indemnification is warranted, such award would fall under the
category of claims, the execution of which would be subject to the stay order
issued by the rehabilitation court. The penal sanctions as a consequence of
violation of the SSS law, in relation to the revised penal code can therefore
be implemented if petitioners are found guilty after trial. However, any civil
indemnity awarded as a result of their conviction would be subject to the
stay order issued by the rehabilitation court. Only to this extent can the order
of suspension be considered obligatory upon any court, tribunal, branch or
body where there are pending actions for claims against the distressed
corporation.
On a final note, this Court would like to point out that Congress has
recently enacted Republic Act No. 10142, or the Financial Rehabilitation and
Insolvency Act of 2010. Section 18 thereof explicitly provides that criminal
actions against the individual officer of a corporation are not subject to the
Stay or Suspension Order in rehabilitation proceedings, to wit:
The Stay or Suspension Order shall not apply.

Page 987 of 1072

TOPIC: PETITIONS FOR DECLARATION IN THE STATE OF SUSPENSION


PAYMENT
RICARDO V. CASTILLO, Petitioner,
vs.
UNIWIDE WAREHOUSE CLUB, INC. and/or JIMMY GOW, Respondents.
G.R. No. 169725. April 30, 2010
FACTS:
The case stems from a Complaintfor illegal dismissal filed on August
26, 2002 by herein petitioner Ricardo V. Castillo against herein respondents
Uniwide Warehouse Club, Inc. and its president, Jimmy N. Gow. The
complaint, docketed as NLRC NCR Case No. 08-06770-2002, contained a
prayer for the payment of worked Saturdays for the year 2001; holiday pay;
separation pay; actual, moral and exemplary damages; and attorneys fees.
However, almost two months from the filing of the Complaint, or on
October 18, 2002, respondents submitted a Motion to Suspend Proceedings
on the ground that in June 1999, the Uniwide Group of Companies had
petitioned the Securities and Exchange Commission (SEC) for suspension of
payments and for approval of its proposed rehabilitation plan. It appears that
on June 29, 1999, the SEC had ruled favorably on the petition and ordered
that all claims, actions and proceedings against herein respondents pending
before any court, tribunal, board, office, body or commission be suspended,
and that following the appointment of an interim receiver, the suspension
order had been extended to until February 7, 2000. On April 11, 2000, the
SEC declared the Uniwide Group of Companies to be in a state of suspension
of payments and approved its rehabilitation plan.
In an Orderdated February 17, 2003, Labor Arbiter Lilia S. Savari
denied the Motion to Suspend Proceedings in the present case. Respondents
lodged an appeal with the NLRC which, on September 30, 2003, sustained
the Labor Arbiter and held that as early as February 7, 2000 the suspension

Page 988 of 1072

order of the SEC should be considered lifted already and that with the
approval of the rehabilitation plan, the suspension of the proceedings in the
instant labor case would no longer be necessary.
ISSUE/S:
Whether respondents can file suspension of pending claims in
order to maintain parity of status among the different creditors of
the distressed corporation at least while the rehabilitation efforts
are ongoing.
RULING:
Jurisprudence is settled that the suspension of proceedings referred to
in the law uniformly applies to "all actions for claims" filed against a
corporation, partnership or association under management or receivership,
without distinction, except only those expenses incurred in the ordinary
course of business. In the oft-cited case of Rubberworld (Phils.) Inc. v.
NLRC,the Court noted that aside from the given exception, the law is clear
and makes no distinction as to the claims that are suspended once a
management committee is created or a rehabilitation receiver is appointed.
Since the law makes no distinction or exemptions, neither should this Court.
Ubi lex non distinguit nec nos distinguere debemos. Philippine Airlines, Inc. v.
Zamoradeclares that the automatic suspension of an action for claims
against a corporation under a rehabilitation receiver or management
committee embraces all phases of the suit, that is, the entire proceedings of
an action or suit and not just the payment of claims.
The reason behind the imperative nature of a suspension or stay order
in relation to the creditors claims cannot be downplayed, for indeed the
indiscriminate suspension of actions for claims intends to expedite the
rehabilitation of the distressed corporation by enabling the management
committee or the rehabilitation receiver to effectively exercise its/his powers
free from any judicial or extrajudicial interference that might unduly hinder
or prevent the rescue of the debtor company. To allow such other actions to
continue would only add to the burden of the management committee or
rehabilitation receiver, whose time, effort and resources would be wasted in
defending claims against the corporation, instead of being directed toward
its restructuring and rehabilitation.
At this juncture, it must be conceded that the date when the claim
arose, or when the action was filed, has no bearing at all in deciding whether
the given action or claim is covered by the stay or suspension order. What
matters is that as long as the corporation is under a management committee
or a rehabilitation receiver, all actions for claims against it, whether for
money or otherwise, must yield to the greater imperative of corporate

Page 989 of 1072

revival, excepting only, as already mentioned, claims for payment of


obligations incurred by the corporation in the ordinary course of business.
It is, thus, not difficult to see why the subject action for illegal dismissal
and damages against respondent corporation ought to have been suspended
at the first instance respondents submitted before the Labor Arbiter their
motion to suspend proceedings in the illegal dismissal case. This, considering
that at the time the labor case was filed on August 26, 2002, respondent
corporation was undergoing proceedings for rehabilitation and was later on
declared to be in a state of suspension of payments.
In fact, a Certification issued by the SEC and signed by its General
Counsel, Vernette G. Umali-Paco, states that as of August 17, 2006, the
petition of Uniwide Sales, Inc. for declaration of suspension of payments and
rehabilitations was still pending with it, and that the company was still under
its rehabilitation proceedings. Hence, since petitioners claim was one for
wages accruing from the time of dismissal, as well as for benefits and
damages, the same should have been suspended pending the rehabilitation
proceedings. In other words, the Labor Arbiter should have abstained from
resolving the illegal dismissal case and, instead, directed petitioner to
present his claim to the rehabilitation receiver duly appointed by the SEC,
inasmuch as the stay or suspension order was effective and it subsisted from
issuance until the dismissal of the petition for rehabilitation or the
termination of the rehabilitation proceedings. The Court of Appeals was thus
correct in directing the suspension of the proceedings in NLRC NCR Case No.
08-06770-2002

Page 990 of 1072

TOPIC: PETITIONS FOR DECLARATION IN THE STATE OF SUSPENSION


PAYMENT
PACIFIC WIDE REALTY AND DEVELOPMENT CORPORATION, Petitioner,
vs. PUERTO AZUL LAND, INC., Respondent.
G.R. No. 178768. November 25, 2009
PACIFIC WIDE REALTY AND DEVELOPMENT CORPORATION, Petitioner,
vs. PUERTO AZUL LAND, INC., Respondent.
G.R. No. 180893
FACTS:
In G.R. No. 180893
Puerto Azul Land, Inc. (PALI) is the owner and developer of the Puerto
Azul Complex situated in Ternate, Cavite. Its business involves the
development of Puerto Azul into a satellite city with residential areas, resort,
tourism and retail commercial centers with recreational areas. In order to
finance its operations, it obtained loans from various banks, the principal
amount of which amounted to Six Hundred Forty Million Two Hundred TwentyFive Thousand Three Hundred Twenty-Four Pesos (P640,225,324.00). PALI
and its accommodation mortgagors, i.e., Ternate Development Corporation
(TDC), Ternate Utilities, Inc. (TUI), and Mrs. Trinidad Diaz-Enriquez, secured
the loans.
In the beginning, PALIs business did very well. However, it started
encountering problems when the Philippine Stock Exchange rejected the
listing of its shares in its initial public offering which sent a bad signal to the
real estate market. This resulted in potential investors and real estate buyers
shying away from the business venture. The situation was aggravated by the

Page 991 of 1072

1997 Asian financial crisis and the decline of the real estate market.
Consequently, PALI was unable to keep up with the payment of its
obligations, both current and those that were about to fall due. One of its
creditors, the Export and Industry Bank (EIB), later substituted by Pacific
Wide Realty and Development Corporation (PWRDC), filed foreclosure
proceedings on PALIs mortgaged properties. Thrust to a corner, PALI filed a
petition for suspension of payments and rehabilitation, accompanied by a
proposed rehabilitation plan and three (3) nominees for the appointment of a
rehabilitation receiver.
In G.R. No. 178768
EIB entered its appearance before the rehabilitation court and moved
for the clarification of the stay order dated September 17, 2004 and/or leave
to continue the extrajudicial foreclosure of the real estates owned by PALIs
accommodation mortgagors. In opposition, PALI argued that the foreclosure
sought would preempt the rehabilitation proceedings and would give EIB
undue preference over PALIs other creditors. On November 10, 2004, the
RTC issued an Order, denying EIBs motion.
EIB filed an urgent motion to order PALI and/or the mortgagor
TUI/rehabilitation receiver to pay all the taxes due on Transfer Certificate of
Title (TCT) No. 133164. EIB claimed that the property covered by TCT No.
133164, registered in the name of TUI, was one of the properties used to
secure PALIs loan from EIB. The said property was subject to a public auction
by the Treasurers Office of Pasay City for non-payment of realty taxes.
Hence, EIB prayed that PALI or TUI be ordered to pay the realty taxes due on
TCT No. 133164.
PALI opposed the motion, arguing that the rehabilitation courts stay
order stopped the enforcement of all claims, whether for money or
otherwise, against a debtor, its guarantors, and its sureties not solidarily
liable to the debtor; thus, TCT No. 133164 was covered by the stay order.
ISSUE/S:
Whether the rehabilitation court erred when it allowed the
foreclosure of the accommodation mortgagees property and
excluded the same from the coverage of the stay order.
RULING:
The governing law concerning rehabilitation and suspension of actions
for claims against corporations is Presidential Decree (P.D.) No. 902-A, as
amended (P.D. No. 902-A). Section 6(c) of P.D. No. 902-A mandates that,
upon appointment of a management committee, rehabilitation receiver,
board, or body, all actions for claims against corporations, partnerships or
associations under management or receivership pending before any court,

Page 992 of 1072

tribunal, board, or body shall be suspended. Stated differently, all actions for
claims against a corporation pending before any court, tribunal or board shall
ipso jure be suspended in whatever stage such actions may be found.
The justification for the suspension of actions or claims pending
rehabilitation proceedings is to enable the management committee or
rehabilitation receiver to effectively exercise its/his powers free from any
judicial or extrajudicial interference that might unduly hinder or prevent the
"rescue" of the debtor company. To allow such other action to continue would
only add to the burden of the management committee or rehabilitation
receiver, whose time, effort and resources would be wasted in defending
claims against the corporation instead of being directed toward its
restructuring and rehabilitation.
In G.R. No. 178768, the rehabilitation court, in its Orders dated March
31, 2005 and August 16, 2005, removed TCT No. 133164 from the coverage
of the stay order. The property covered by TCT No. 133164 is owned by TUI.
TCT No. 133164 was mortgaged to PWRDC by TUI as an accommodation
mortgagor of PALI by virtue of the Mortgage Trust Indenture (MTI) dated
February 1995.
The MTI was executed among TDC, TUI and Mrs. Trinidad DiazEnriquez, as mortgagors; PALI, as borrower; and Urban Bank, as trustee.
Under Section 4.04 thereof, the mortgagors and the borrower guaranteed to
pay and discharge on time all taxes, assessments and governmental charges
levied or assessed on the collateral and immediately surrender to the trustee
copies of the official receipts for such payments. It was also agreed therein
that should the borrower fail to pay such uncontested taxes, assessments
and charges within sixty (60) calendar days from due date thereof, the
trustee, at its option, shall declare the mortgagors and the borrower in
default under Section 6.01(d) of the MTI, or notify all the lenders of such
failure.
In excluding the property from the coverage of the stay order and allow
PWRDC to foreclose on the mortgage and settle the realty tax delinquency of
the property with Pasay City, the rehabilitation court used as justification
Section 12, Rule 4 of the Interim Rules on Corporate Rehabilitation.

Page 993 of 1072

TOPIC: PETITIONS FOR DECLARATION IN THE STATE OF SUSPENSION


PAYMENT
PHILIPPINE NATIONAL BANK and EQUITABLE PCI BANK, Petitioners,
vs.
HONORABLE COURT OF APPEALS, SECURITIES AND EXCHANGE
COMMISSION EN BANC, ASB HOLDINGS, INC., ASB REALTY
CORPORATION, ASB DEVELOPMENT CORPORATION (formerly TIFFANY
TOWER REALTY CORPORATION), ASB LAND INC., ASB FINANCE, INC.,
MAKATI HOPE CHRISTIAN SCHOOL, INC., BEL-AIR HOLDINGS
CORPORATION, WINCHESTER TRADING, INC., VYL DEVELOPMENT
CORPORATION, GERICK HOLDINGS CORPORATION, and
NEIGHBORHOOD HOLDINGS, INC., Respondents.
G.R. No. 165571. January 20, 2009
FACTS:
Petitioners Philippine National Bank (PNB) and Equitable PCI Bank are
members of the consortium of creditor banks constituted pursuant to the
Mortgage Trust Indenture (MTI) 6 dated May 29, 1989, as amended, by and
between Rizal Commercial Banking Corporation-Trust and Investments
Division, acting as trustee for the consortium, and ASB Development
Corporation (ASBDC, formerly Tiffany Tower Realty Corporation). Other
members of the consortium include Metropolitan Bank and Trust Company
(Metrobank), Prudential Bank, Union Bank of the Philippines, and United
Coconut Planters Bank. Private respondents ASB Holdings, Inc., ASBDC, ASB
Land, Inc., ASB Finance, Inc., Makati Hope Christian School, Inc., Bel-Air
Holdings Corporation, Winchester Trading, Inc., VYL Holdings Corporation,
and Neighborhood Holdings, Inc. (ASB Group) are corporations engaged in
real estate development. The ASB Group is owned by Luke C. Roxas. 7 Under

Page 994 of 1072

the MTI, petitioners granted a loan of PhP 1,081,000,000 to ASBDC secured


by a mortgage of five parcels of land with improvements.
On May 2, 2000, private respondents filed with the SEC a verified petition for
rehabilitation with prayer for suspension of actions and proceedings pending
rehabilitation pursuant to Presidential Decree No. (PD) 902-A, as amended.
The case was docketed as SEC Case No. 05-00-6609. Private respondents
stated that they possess sufficient properties to cover their obligations but
foresee inability to pay them within a period of one year. They cited the
sudden non-renewal and/or massive withdrawal by creditors of their loans to
ASB Holdings, the glut in the real estate market, severe drop in the sale of
real properties, peso devaluation, and decreased investor confidence in the
economy which resulted in the non-completion of and failure to sell their
projects and default in the servicing of their credits as they fell due. The ASB
Group had assets worth PhP 19,410,000,000 and liabilities worth PhP
12,700,000,000. Faced with at least 712 creditors, 317 contractors/suppliers,
and 492 condominium unit buyers, and the prospect of having secured and
non-secured creditors press for payments and threaten to initiate foreclosure
proceedings, the ASB Group pleaded for suspension of payments while
working for rehabilitation with the help of the SEC.
ISSUE/S:
Whether a petition for rehabilitation and suspension of payments
cannot be filed without previously filing a petition for suspension
of payment.
RULING:
In the case at bar, the ASB Group filed with the SEC a petition for
rehabilitation with prayer for suspension of actions and proceedings pending
rehabilitation. Contrary to petitioners arguments, the mere fact that the ASB
Group averred that it has sufficient assets to cover its obligations does not
make it "solvent" enough to prevent it from filing a petition for rehabilitation.
A corporation may have considerable assets but if it foresees the
impossibility of meeting its obligations for more than one year, it is
considered as technically insolvent. Thus, at the first instance, a corporation
may file a petition for rehabilitationa remedy provided under Sec. 4-1.
When Sec. 4-1 mentioned technical insolvency under Sec. 3-12, it was
referring to the definition of technical insolvency in the said section; it was
not requiring a previous filing of a petition for suspension of payments which
petitioners would have us believe.
Petitioners harp on the SECs failure to examine whether the ASB
Group is technically insolvent. They contend that the SEC should wait for a
year after the filing of the petition for suspension of payments when
technical insolvency may or may not arise. This is erroneous. The period

Page 995 of 1072

mentioned under Sec. 3-12, "longer than one year from the filing of the
petition," does not refer to a year-long waiting period when the SEC can
finally say that the ailing corporation is technically insolvent to qualify for
rehabilitation. The period referred to the corporations inability to pay its
obligations; when such inability extends beyond one year, the corporation is
considered technically insolvent. Said inability may be established from the
start by way of a petition for rehabilitation, or it may be proved during the
proceedings for suspension of payments, if the latter was the first remedy
chosen by the ailing corporation. If the corporation opts for a direct petition
for rehabilitation on the ground of technical insolvency, it should show in its
petition and later prove during the proceedings that it will not be able to
meet its obligations for longer than one year from the filing of the petition.
As regards the status of the Repayment Schedule required to be
attached to the petition for rehabilitation (Sec. 4-2[g]), this requirement is
conditioned on whether one was approved by the SEC in the first place. If
there is none, as in the case of a petition for rehabilitation due to technical
insolvency directly filed under Rule IV, Sec. 4-1, then there is no status report
to submit with the petition.
TOPIC: PETITIONS FOR DECLARATION IN THE STATE OF SUSPENSION
PAYMENT
PRYCE CORPORATION, petitioner,
vs.
THE COURT OF APPEALS and CHINA BANKING CORPORATION,
respondents.
G.R. No. 172302. February 4, 2008
543 SCRA 657
FACTS:
Pryce Corporation, petitioner, was incorporated under Philippine laws
on September 7, 1989. Its primary purpose was to develop real estate in
Mindanao. It engaged in the development of memorial parks, operated a
major hotel in Cagayan de Oro City, and produced industrial gases.
The 1997 Asian financial crisis, however, badly affected petitioners
operations, resulting in heavy losses. It could not meet its obligations as they
became due. It incurred losses of P943.09 million in 2001, P479.05 million in
2002, and P125.86 million in 2003.
Thus, on July 12, 2004, petitioner filed with the Regional Trial Court
(RTC), Branch 138, Makati City, acting as Commercial Court, a petition for
rehabilitation. Petitioner prayed for the appointment of a Rehabilitation
Receiver from among the nominees named therein and the staying of the

Page 996 of 1072

enforcement of all claims, monetary or otherwise against it. Petitioner also


prayed that after due hearing, its proposed Rehabilitation Plan be approved.

ISSUE/S:
Whether the Court of Appeals erred in denying the petition for
rehabilitation of petitioner Pryce Corporation.

RULING:
The petition for rehabilitation does not allege that there is a clear and
imminent danger that petitioner will lose its corporate assets if a receiver is
not appointed. In other words, the "serious situation test" laid down by
Rizal Commercial Banking Corporation has not been met or at least
substantially complied with. Significantly, the Stay Order dated July 13, 2004
issued by the RTC does not state any serious situation affecting petitioners
corporate assets. We observe that in appointing Mr. Gener T. Mendoza as
Rehabilitation Receiver, the only basis of the lower court was its
finding that "the petition is sufficient in form and substance."
However, it did not specify any reason or ground to sustain such finding.
Clearly, the petition failed to comply with the "serious situation test.

Page 997 of 1072

TOPIC: PETITIONS FOR DECLARATION IN THE STATE OF SUSPENSION


PAYMENT
UNIWIDE HOLDINGS, INC., Petitioner,
vs.
JANDECS TRANSPORTATION CO., INC., Respondent.
G.R. No. 168522. December 19, 2007
541 SCRA 657
FACTS:
In January 1997, petitioner and respondent Jandecs Transportation Co.,
Inc. entered into a contract of "Assignment of Leasehold Rights" under which
the latter was to operate food and snack stalls at petitioner's Uniwide Coastal
Mall in Paraaque City. The contract was for a period of 18 years,
commencing October 1, 1997 up to September 30, 2015, for a consideration
of P2,460,630.15. The parties also agreed that respondent's stalls would be
located near the movie houses and would be the only stalls to sell food and
beverages in that area.
On February 7, 1997, respondent paid the contract price in full.
Petitioner, however, failed to turn over the stall units on October 1, 1997 as
agreed upon. Respondent sought the rescission of the contract and the
refund of its payment. Petitioner refused both.
On July 23, 1999, respondent filed a complaint in the Regional Trial
Court (RTC), Branch 257 of Paraaque City, for breach of contract, rescission
of contract, damages and issuance of a writ of preliminary attachment. In the
complaint, respondent claimed that, despite full payment, petitioner (1)

Page 998 of 1072

failed to deliver the stall units on the stipulated date; (2) opened its own food
and snack stalls near the cinema area and (3) refused to accommodate its
request for the rescission of the contract and the refund of payment.
ISSUE:
Whether the SEC's order of suspension of payments and approval
of its rehabilitation plan is warranted in this case.
RULING:
The relevant law dealing with the suspension of payments for money
claims against corporations under rehabilitation is Presidential Decree (PD)
No. 902-A, as amended. The term "claim" under said law refers to debts or
demands of pecuniary nature. It is the assertion of rights for the payment of
money. The raison d' tre behind the suspension of claims pending
rehabilitation was explained in the case of BF Homes, Inc. v. CA :
...the reason for suspending actions for claims
against the corporation should not be difficult to
discover. It is not really to enable the management
committee or the rehabilitation receiver to substitute
the [corporation] in any pending action against it
before any court, tribunal, board or body. Obviously,
the real justification is to enable the management
committee or the rehabilitation receiver to effectively
exercise its/his powers free from any judicial or extrajudicial interference that might unduly hinder or
prevent the "rescue" of the debtor [corporation]. To
allow such other action to continue would only add to
the burden of the management committee or
rehabilitation receiver, whose time, effort and
resources would be wasted in defending claims
against the corporation instead of being directed
toward its restructuring and rehabilitation.
In Philippine Air Lines [(PAL)], Incorporated v. Zamora, we said that "all
actions for claims against a corporation pending before any court, tribunal or
board shall ipso jure be suspended in whatever stage such actions may be
found upon the appointment by the SEC of a management committee or a
rehabilitation receiver."
However, we would still find no cogent reason to reverse our August
17, 2005 resolution denying petitioner's appeal even if the proceedings here
were to be suspended in the meantime. And such suspension would not at all
affect our position that the MR should be denied as well.

Page 999 of 1072

TOPIC:PETITIONS FOR DECLARATION IN THE STATE OF SUSPENSION OF


PAYMENT
BANK OF THE PHILIPPINEISLANDS, AS SUCCESSOR OF FAREAST
BANK AND TRUST COMPANY,
VS.
SECURITIES AND EXCHANGE COMMISSION, REHABILITATION
RECEIVER, ASB HOLDINGS, INC., ASB DEVELOPMENT CORPORATION,
NACHURA, ASB LAND, INC., ASB FINANCE,INC., MAKATI HOPE
CHRISTIANSCHOOL, INC., BEL-AIR HOLDINGSCORP., WINCHESTER
TRADING,INC., VYL DEVELOPMENT CORP.,GERRICK HOLDINGS CORP.,
NEIGHBORHOOD HOLDINGS, INC.,AND THE COURT OF APPEALS
G.R. NO. 164641.DECEMBER 20, 2007
FACTS:
BPI, through its predecessor-in- interest, FEBTC, extended credit
accommodations to the ASB Group, secured by a real estate mortgage over 2
properties. ASB Group filed a petition for rehabilitation and suspension of
payments before the SEC. The interim receiver submitted its Proposed
Rehabilitation Plan (Rehabilitation Plan) for the ASB Group. The Rehabilitation
Plan provides, among others, a dacion en pago by the ASB Group to BPI of
one of the properties mortgaged to the latter at the ASB Group against the
total amount of the ASB Groups exposure to the bank. In turn, ASB Group
would require the release of the other property mortgaged to BPI, to be
thereafter placed in the asset pool.
BPI opposed the Rehabilitation Planand moved for the dismissal of the
ASB Groups petition for rehabilitation. The SEC hearing panel issued an

Page 1000 of 1072

order approving ASB Groups proposed rehabilitation plan and appointed Mr.
Fortunato Cruz as rehabilitation receiver.
ISSUE/S:
Whether or not the Rehabilitation Plan is violative of BPIs
contractual rights.
RULING:
Rehabilitation proceedings in our jurisdiction, much like the bankruptcy
laws of the United States, have equitable and rehabilitative purposes. On the
one hand, they attempt to provide for the efficient and equitable distribution
of an insolvent debtors remaining assets to its creditors; and on the other, to
provide debtors with a fresh start by relieving them of the weight of their
outstanding debts and permitting them to reorganize their affairs. The
rationale of P.D. No. 902-A, as amended, is to effect a feasible and viable
rehabilitation, by preserving a foundering business as going concern,
because the assets of a business are often more valuable when so
maintained than they would be when liquidated.
The Court reiterates that the SECs approval of the Rehabilitation Plan
did not impair BPIs right to contract. As correctly contended by private
respondents, the non-impairment clause is a limit on the exercise of
legislative power and not of judicial or quasi-judicial power. The SEC, through
the hearing panel that heard the petition for approval of the Rehabilitation
Plan, was acting as a quasi-judicial body and thus, its order approving the
plan cannot constitute an impairment of the right and the freedom to
contract.

Page 1001 of 1072

TOPIC:PETITIONS FOR DECLARATION IN THE STATE OF SUSPENSION OF


PAYMENT
PHILIPPINE AIRLINES, INC.
VS.
HEIRS OF BERNARDIN J. ZAMORA
G.R. NO. 164267. NOVEMBER 23, 2007
FACTS:
Zamora was a cargo representative assigned at the ICO-IOD Philippine
Airlines, Inc. He alleged that his immediate supervisor instructed him to alter
some entries in the Customs Boatnote and Inbound Handling Report to
conceal Abuyuans smuggling and pilferage activities. When he refused to
follow this order, Abuyuan concocted charges of insubordination and neglect
of customers against him.
Zamora received a Memorandum informing him of his temporary
transfer to the DCO. Zamora refused to follow the directive because: first,
there was no valid and legal reason for his transfer; second, the transfer
violated the collective bargaining agreement between the management and
the employees union that no employee shall be transferred without just and
proper cause; and third, the transfer did not comply with the 15-day prior
notice rule. He wrote to the management requesting that an investigation be
conducted on the smuggling and pilferage activities. He disclosed that he
has a telex from Honolulu addressed to Abuyuan to prove Abuyuans illegal
activities. As a result, the management invited Zamora to several
conferences to substantiate his allegations. Zamora claimed that during
these conferences, he was instructed to continue reporting to the ICO-IOD to
observe the activities therein. Even so, his salaries were withheld.

Page 1002 of 1072

Philippine Airlines, Inc. claimed that Zamora had an altercation with


Abuyuan to the point of a fistfight. The management requested Zamora to
explain in writing the incident. It found his explanation unsatisfactory.To
diffuse the tension between the parties, the management decided to
temporarily transfer Zamora to the DCO. Zamora refused to receive these
and continued reporting to the ICO-IOD. Consequently, he was reported
absent at the DCO since November 13, 1995. His salaries were subsequently
withheld. He also ignored the managements directive requiring him to
explain in writing his continued absence. Management served Zamora a
Notice of Administrative Charge for Absence Without Official Leave (AWOL).
He was informed of his termination due to Insubordination/Neglect of
Customer, Disrespect to Authority, and AWOL.
Zamora filed an action for illegal dismissal, unfair labor practice, nonpayment of wages, and damages. LA dismissed the complaint for lack of
merit. The NLRC reversed the Labor Arbiters decision and declared Zamoras
transfer illegal. The Court of Appeals affirmed the decision of the NLRC. In
the instant petition, petitioner Philippine Airlines, Inc. raises the following
issues:
ISSUE/S:
Whether the decision of the NLRC had become final and
executory
RULING:
To reiterate, the suspension of all actions for claims against a
corporation embraces all phases of the suit, be it before the trial court or any
tribunal or before this Court. No other action may be taken, including the
rendition of judgment during the state of suspension. It must be stressed
that what are automatically stayed or suspended are the proceedings of a
suit and not just the payment of claims during the execution stage after the
case had become final and executory. Once the process of rehabilitation,
however, is completed, this Court will proceed to complete the proceedings
on the suspended actions.
Furthermore, the actions that are suspended cover all claims against
the corporation whether for damages founded on a breach of contract of
carriage, labor cases, collection suits or any other claims of a pecuniary
nature. No exception in favor of labor claims is mentioned in the law.
More importantly, as the instant case involves essentially the same
facts, parties, and issues as G.R. No. 166996 entitled Philippine Airlines, Inc.,
et al. v. Bernardin J. Zamora, we find it unnecessary to make further
pronouncements which might otherwise conflict with the disposition made by
the Courts Third Division therein.

Page 1003 of 1072

TOPIC:PETITIONS FOR DECLARATION IN THE STATE OF SUSPENSION OF


PAYMENT
ALEMAR'S SIBAL & SONS, INC.
VS.
HONORABLE JESUS M. ELBINIAS, IN HIS CAPACITY AS THE PRESIDING
JUDGE OF REGIONAL TRIAL COURT, NATIONAL CAPITAL REGION,
BRANCH CXLI (141), MAKATI, AND G.A. YUPANGCO & CO., INC.
G.R. NO. 75414. JUNE 4, 1990
FACTS:
G.A. Yupangco filed an action with respondent trial court for collection
of a sum of money with prayer for damages and preliminary attachment
against Alemar's Bookstore, a business entity owned and managed by
Alemar's. Respondent court rendered its decisionordering defendant,
Alemar's Book Store, to pay G.A. Yupangco& Co., Inc.,
Subsequently, Ledesma, Saludo and Associates, as intervenor-movant,
filed an omnibus motion informing the respondent trial court that the
petitioner Alemar's has been placed under rehabilitation receivership by the
Securities and Exchange Commission and that movant has been appointed
as its receiver. It prayed that it be allowed to intervene, that the decision of
August 30, 1985 be set aside and that further proceedings in this case be
suspended.
G.A. Yupangco maintained that it received notice of the
receivership only after one month after the collection suit. It further averred
that the motion to intervene by the receiver was not seasonably made. G.A.
Yupangco urged the issuance of a writ of execution to implement the default

Page 1004 of 1072

judgment which had become final and executory, there being no motion for
reconsideration or appeal. The corresponding writ was issued.
Alemar's moved for the discharge of the writ on the ground that its
issuance was improper since the proceedings in Civil Case No. 9252 have
been suspended pursuant to the October 29, 1985 order.
ISSUE/S:
Whether or not respondent court can validly proceed with the
execution of a final decision for the payment of a sum of money
despite the fact that the judgment debtor has been placed under
receivership.
RULING:
The cases of Central Bank vs. Morfe, and Lipana vs. Development Bank
of Rizal, are most enlightening on why an execution in this particular
instance could be legally held in abeyance despite a final judgment. In both
cases, there was an attempt by a creditor to enforce payment against a bank
(which was either declared insolvent or placed under receivership) by
obtaining a favorable judgment in the regular court and insisting upon its
execution on the ground that the courts cannot validly obstruct the
enforcement of judgments that have become final and executory.
The rationale behind the Court's imprimatur of the stay of execution in
the aforementioned cases is squarely applicable to the instant petition even
if Alemar's is obviously not a banking institution.
It must be stressed that the SEC had earlier ordered the suspension of
all actions for claims against Alemar's in order that all the assets of said
petitioner could be inventoried and kept intact for the purpose of
ascertaining an equitable scheme of distribution among its creditors.
During rehabilitation receivership, the assets are held in trust for the
equal benefit of all creditors to preclude one from obtaining an advantage or
preference over another by the expediency of an attachment, execution or
otherwise. For what would prevent an alert creditor, upon learning of the
receivership, from rushing posthaste to the courts to secure judgments for
the satisfaction of its claims to the prejudice of the less alert creditors.
As between creditors, the key phrase is "equality is equity." When a
corporation threatened by bankruptcy is taken over by a receiver, all the
creditors should stand on an equal footing. Not anyone of them should be
given any preference by paying one or some of them ahead of the others.
This is precisely the reason for the suspension of all pending claims against
the corporation under receivership. Instead of creditors vexing the courts

Page 1005 of 1072

with suits against the distressed firm, they are directed to file their claims
with the receiver who is a duly appointed officer of the SEC.

TOPIC:PETITIONS FOR DECLARATION IN THE STATE OF SUSPENSION OF


PAYMENT
BAROTAC SUGAR MILLS, INC.
VS.
COURT OF APPEALS and PITTSBURGH TRADE CENTER, CO., INC.
G.R. No. 123379, July 15, 1997
FACTS:
PITTSBURGH filed before the RTC a complaint for a sum of money
against BAROTAC. Instead of filing an answer, BAROTAC filed, a Motion to
Suspend Proceedings on the ground that a Petition for Suspension of
Payments with Prayer for the Appointment of a Management or
Rehabilitation Committee had been filed with the Securities and Exchange
Commission (SEC) pursuant to Presidential Decree 902-A, as amended by
P.D. Nos. 1653 and 1758. This motion met opposition from PITTSBURGH.
The court issued an Order denying petitioner's motion.
ISSUE/S:
Whether the mere filing with the SEC of such petition suspends
the proceedings in the RTC.
RULING:
The conclusion then is inevitable that pursuant to the underscored
proviso in sub-paragraph (c) of the aforementioned Section 6, taken together
with sub-paragraph (d) of Section 5 and sub-paragraph (d) of Section 6, a
court action is ipso jure suspended only upon the appointment of a

Page 1006 of 1072

management committee or a rehabilitation receiver. Since there is no


showing at all that a management committee or a rehabilitation receiver for
BAROTAC has been appointed by the SEC, suspension of the proceedings in
Civil Case No. Q-94-20347 before the RTC of Quezon City is not warranted.

TOPIC:PETITIONS FOR DECLARATION IN THE STATE OF SUSPENSION OF


PAYMENT
BF HOMES, INCORPORATED V. COURT OF APPEALS,
ROSALINDA R. ROA and VICENTE MENDOZA
G.R. No. 76879, October 3, 1990
ROSALINDA ROA and VICENTE MENDOZA V. COURT OF APPEALS and
BF HOMES, INCORPORATED,
G.R.No. 77143, October 3, 1990
FACTS:
BF contracted a loan from Rosalinda R. Roa and Vicente Mendoza. The
obligation was embodied in a promissory note and secured by two postdated checks issued by BF in favor of the lenders. On September 25, 1984,
BF filed a Petition for Rehabilitation and for a Declaration in a State of
Suspension of Payments under Sec. 5(d) of P.D. No. 902-A with a prayer that
upon the filing of the petition and in the meantime, all claims against it for
any and all accounts or indebtedness be suspended, but allowing petitioner
to continue with its normal operations. It also asked for the approval of the
proposed rehabilitation plan.
On October 17, 1984, Roa and Mendoza filed a complaint against BF
with the Regional Trial Court of Quezon City, docketed as Civil Case No. Q43104, for the recovery of the loan of P250,000.00, with interest and
attorney's fees. The complaint also prayed for the issuance of a writ of
preliminary attachment against the properties of BF.
The trial court issued the writ against properties of BF sufficient to satisfy the
principal claim in the amount of P257,333.33. BF moved for the dismissal of

Page 1007 of 1072

the case for lack of jurisdiction, or at least for its suspension in view of the
pendency of SEC Case No. 002693. It also asked for the lifting of the writ of
preliminary attachment.
ISSUE/S:
Whether the mere filing with the SEC of Petition for Rehabilitation
and for a Declaration in a State of Suspension of Payments under
Sec. 5(d) of P.D. No. 902-Asuspends actions for claims against the
corporation
RULING:
The pertinent provision of law dealing with the suspension of actions
for claims against the corporation is Sec. 6(c) of P.D. 902-A, as amended,
which reads:
Sec. 6.n order to effectively exercise such jurisdiction, the Commission shall
possess the following powers:
xxx xxx xxx
(c) To appoint one or more receivers of the property, real
and personal, which is the subject of the action pending
before the Commission in accordance with the pertinent
provisions of the Rules of Court, and in such other cases
whenever necessary in order to preserve the rights of
parties-litigants and/or protect the interest of the investing
public and creditors: Provided, however, That the
Commission may, in appropriate cases, appoint a
rehabilitation receiver of corporations, partnerships or
other associations not supervised or regulated by other
government agencies who shall have, in addition to the
powers of a regular receiver under the provisions of the
Rules of Court, such functions and powers as are provided
for in the succeeding paragraph (d) hereof: Provided,
further, That the Commission may appoint a rehabilitation
receiver of corporations, partnership or other associations
supervised or regulated by other government agencies,
such as banks and insurance companies, upon request of
the government agency concerned: Provided, finally, That
upon appointment of a management committee,
rehabilitation receiver, board or body, pursuant to this
Decree, all actions for claims against corporations,
partnership, or associations under management or
receivership pending before any court, tribunal, board or
body shall be suspended accordingly. (As amended by P.D.
Nos. 1653, 1758 and 1799; Emphasis supplied.)

Page 1008 of 1072

As will be noted, the duration of the suspension is not indicated in the


law itself. And neither is it specified in the SEC order creating the
management committee.
Under Sec. 6(d) of P.D. No. 902-A, the management committee or
rehabilitation receiver is empowered to take custody and control of all
existing assets and properties of such corporations under management; to
evaluate the existing assets and liabilities, earnings and operations of such
corporations; to determine the best way to salvage and protect the interest
of investors and creditors; to study, review and evaluate the feasibility of
continuing operations and restructure and rehabilitate such entities if
determined to be feasible by the SEC.
Consequently, we feel that the trial court cannot at this point
determine the extent of BF's liability, if any, to Roa and Mendoza. This is true
whether it is retained as party defendant or substituted by the management
committee (or the rehabilitation receiver) as directed by the respondent
court. What Roa and Mendoza should do now is file their claims with the
rehabilitation receiver and submit to him such evidence as they would
otherwise have to adduce before the trial court to prove such claims.
TOPIC:PETITIONS FOR DECLARATION IN THE STATE OF SUSPENSION OF
PAYMENT
BANK OF THE PHILIPPINE ISLANDS
VS.
COURT OF APPEALS AND RUBY INDUSTRIAL CORPORATION\
G.R. No. 97178. January 10, 1994
FACTS:
BPI filed with the RTC complaint against RUBY for foreclosure of real
estate mortgage. After filing its answer with counterclaim, RUBY submitted to
the trial court a motion for suspension of the proceedings on the ground that
SEC issued an Order placing RUBY under a rehabilitation plan pursuant to
Sec. 6, par. (c), of P.D. 902-A. In that Order, SEC declared that "(a)ccordingly,
with the creation of the Management Committee all actions or claims against
Ruby Industrial Corporation pending before any court, tribunal, branch or
body are hereby deemed suspended." On 19 December 1984, the trial court
issued an order granting the motion of RUBY and suspended the
proceedings.
BPI filed a motion for reopening of the proceedings, invoking our ruling
in Philippine Commercial International Bank v. Court of Appeals2which states
that "SEC's order of suspension of payments of Philfinance as well as for all
actions or claims against Philfinance could only be applied to claims of

Page 1009 of 1072

unsecured creditors. Such order can not extend to creditors holding a


mortgage, pledge or any lien on the property unless they give up the
property, security or lien in favor of all the creditors of Philfinance."
ISSUE/S:
Whether petitioner, which is a secured creditor of respondent
RUBY, may still judicially enforce its claim against the latter
which has already been placed by SEC under rehabilitation
pursuant to Sec. 5 and Sec. 6, pars. (c) and (d), P.D. 902-A.
RULING:
In the instant case, the action of petitioner for foreclosure of real estate
mortgage had been filed against respondent RUBY and was pending with the
trial court when RUBY was placed by SEC under rehabilitation through the
creation of a management committee pursuant to Sec. 6, par. (d), P.D. 902-A.
In its order of 10 August 1984, SEC directed that all actions or claims against
RUBY pending before any court, tribunal, branch or body be deemed
suspended. On the basis of this order, the jurisdiction of this trial court over
the case was also considered suspended. As a result, SEC acquired
jurisdiction, which is bolstered by the fact that it had already appointed a
rehabilitation receiver for the distressed corporation and had directed that all
proceedings or claims against it be suspended.
More importantly, the doctrine in the PCIB case has since been
abrogated. In Alemar'sSibal& Sons v. Elbinias,BF Homes, Inc. v. Court of
Appeals,Araneta v. Court of Appeals,and RCBC v. Court of Appeals, we
already ruled that whenever a distressed corporation asks SEC for
rehabilitation and suspension of payments, preferred creditors may no longer
assert such preference, but shall stand on equal footing with other creditors.
Foreclosure shall be disallowed so as not to prejudice other creditors or cause
discrimination among them. If foreclosure is undertaken despite the fact that
a petition for rehabilitation has been filed, the certificate of sale shall not be
delivered pending rehabilitation. If this has already been done, no transfer
certificate of title shall likewise be effected within the period of rehabilitation.
The rationale behind PD 902-A, as amended, is to effect a feasible and viable
rehabilitation. This cannot be achieved if one creditor is preferred over the
others.

Page 1010 of 1072

TOPIC: PETITIONS FOR DECLARATION IN THE STATE OF SUSPENSION OF


PAYMENTS
EMILIO CHING
VS.
LAND BANK OF THE PHILIPPINES
G.R. NO. 73123 SEPTEMBER 2, 1991
201 SCRA 191
FACTS:
Private respondents Filand Manufacturing and Estate Development Co.,
Inc. and Emilio Ching obtained from petitioner Land Bank of the Philippines a
loan in the amount of Ten Million Pesos. Private respondents having failed to
pay the loan on its due date, petitioner instituted before the RTC of Manila a
complaint for recovery thereof.
During the pendency of the collection suit, private respondents Filand
Manufacturing, Emilio Ching and his spouse Inai Teh and Top Construction
Enterprises, Inc., thru Emilio Ching, filed before the respondent RTC of Pasay
City a petition for declaration of insolvency citing as ground therefore was
their inability to pay the various debts and liabilities incurred by them, either
jointly or solidarily or guaranteed by one for the other, in the course of their
businesses, such inability being due to business reverses Acting on said
petition, respondent court issued an Order of Adjudication declaring private

Page 1011 of 1072

respondents insolvent pursuant to Section 18 of the Insolvency Law (Act No.


1956).
Petitioner bank moved for a reconsideration of the Order of
Adjudication on grounds that the court has no jurisdiction over the subject
matter of the petition insofar as petitioning corporations are concerned and
the petition is defective in form and substance.
ISSUE/S:
Whether or not the trial court has jurisdiction over insolvency
proceedings and suspension of payments
RULING:
Under Act 1956, otherwise known as the Insolvency Law, jurisdiction
over proceedings for suspension of payments, voluntary and involuntary
insolvency is exclusively vested in the regular courts. However, P.D. No. 1758
issued in 1981 added to the exclusive and original jurisdiction of the SEC
defined and delineated in Section 5 of P.D. 902-A, the following:
d) Petitions of corporations, partnerships or associations to be
declared in the state of suspension of payments in cases where
the corporation, partnership or association possesses sufficient
property to cover all its debts but foresees the impossibility of
meeting them when they respectively fall due or in cases where
the corporation, partnership or association has no sufficient
assets to cover its liabilities, but is under the management of a
Rehabilitation Receiver or Management Committee created
pursuant to this Decree.
The SEC like any other administrative body, is a tribunal of limited
jurisdiction and as such, could wield only such powers as are specifically
granted to it by its enabling statute. Its jurisdiction should be interpreted in
strictissimi juris.
Section 5, par. (d) Should be construed as vesting upon the SEC
original and exclusive jurisdiction only over petitions to be declared in a state
of suspension of payments, which may either be:
a) a simple petition for suspension of payments based on the
provisions of the Insolvency Law, or
b) a similar petition accompanied by a prayer for the
creation/appointment of a management committee and/or
rehabilitation receiver based on the provisions of P.D. No.
902-A. Said provision cannot be stretched to include
petitions for insolvency. The reason is that under said
Section 5, par. (d) above-quoted, the jurisdiction of the SEC
over cases where the corporation, partnership or
association has no sufficient assets to cover its liabilities,

Page 1012 of 1072

(and therefore insolvent) is qualified by the conjunctive


phrase "but is under the management of a Rehabilitation
Receiver or Management Committee created pursuant to
this Decree." This qualification effectively circumscribes
the jurisdiction of the SEC over insolvent corporations,
partnerships and associations, and consequently, over
proceedings for the declaration of insolvency. It
demonstrates beyond doubt that jurisdiction over
insolvency proceedings pertains neither in the first
instance nor exclusively to the SEC but only in continuation
of or as an incident to the exercise of its jurisdiction over
petitions to be declared in a state of suspension of
payments wherein the petitioning corporation, partnership
or association had previously been placed under a
rehabilitation receiver or management committee by the
SEC itself.
Viewed differently, where the petition filed is one for declaration of a
state of suspension of payments due to a recognition of the inability to pay
one's debts and liabilities, and where the petitioning corporation either:
a) has sufficient property to cover all its debts but foresees
the impossibility of meeting them when they fall due
(solvent but illiquid or
b) has no sufficient property (insolvent) but is under the
management of a rehabilitation receiver or a
management committee, the applicable law is P.D. No.
902-A pursuant to Sec. 5 par. (d) thereof.
However, if the petitioning corporation has no sufficient assets to cover
its liabilities and is not under a rehabilitation receiver or a management
committee created under P.D. No. 902-A and does not seek merely to have
the payments of its debts suspended, but seeks a declaration of insolvency,
as in this case, the applicable law is Act 1956 on voluntary insolvency,
specifically section 14 thereof, which provides:
Sec. 14. An insolvent debtor, owing debts exceeding in
amount the sum of one thousand pesos, may apply to be
discharged from his debts and liabilities by petition to the Court
of First Instance of the province or city in which he has resided
for six month next preceding the filing of such petition. In his
petition, he shall set forth his place of residence, the period of his
residence therein immediately prior to filing said petition, his
inability to pay all his debts in full, his willingness to surrender all
his property, estate, and effects not exempt from execution for
the benefit of his creditors, and an application to be adjudged an
insolvent. He shall annex to his petition a schedule and inventory

Page 1013 of 1072

in the form hereinafter provided. The filing of such petition shall


be an act of insolvency.
Neither could the grant of additional powers to SEC under Section 6(c)
and (d) of P.D. No. 902- A, as amended, be construed as vesting upon it
exclusive and original jurisdiction over insolvency proceedings.

Page 1014 of 1072

TOPIC: PETITIONS FOR DECLARATION IN THE STATE OF SUSPENSION OF


PAYMENTS
PHILIPPINE COMMERCIAL INTERNATIONAL BANK and MELCHOR B.
FRANCISCO
VS.
THE HON. COURT OF APPEALS
G.R. No. L-76853, April 18, 1989
FACTS:
On March 3, 1981, Philippine Underwriters Finance Corporation
executed a pledge agreement involving certain shares of stocks and bonds in
favor of Insular Bank of Asia and America (now Philippine Commercial
International Bank, the petitioner herein) as a security for its outstanding
obligation.
On June 18, 1981, the Securities and Exchange Commission placed
Philfinance under suspension of payments upon "the directive of the
President of the Philippines to conserve the assets of the Corporation and
obtain an equitable payment to all its creditors."
On August 7, 1981, SEC appointed a Receivership Committee to
conserve the assets of Phil-finance and determine the best way to protect
the creditors, as well as make the necessary representations with any court
or other body for the consolidation of all claims against Philfinance which are
pending before such court or body in order to forestall the probability of
inequitable disposition/satisfaction of said claims.
On December 19, 1983, upon the basis of the findings of the
Receivership Committee and on its own, SEC ordered the dissolution and
liquidation of Philfinance.
Meanwhile, Philfinance failed to satisfy its outstanding obligation with
PCIB which prompted the latter to post a Notice of Auction Sale of the
pledged shares of stocks and bonds on August 18, 1986 by the other
petitioner, Notary Public Melchor B. Francisco.
On August 15, 1986, the Receiver filed a petition for a writ of
preliminary injunction with the Regional Trial Court to stop the
aforementioned auction sale.
On September 24, 1986, the trial court issued an order denying the
petition.
ISSUE:

Page 1015 of 1072

Whether or not the order of suspension of payments of


Philfinance could be applied to claims of unsecured creditors.
RULING:
SEC's order for suspension of payments of Philfinance as well as for all
actions of claims against Philfinance could only be applied to claims of
unsecured creditors. Such order can not extend to creditors holding a
mortgage, pledge or any lien on the property unless they give up the
property, security or lien in favor of all the creditors of Philfinance
This ruling finds support in Chartered Bank vs. Imperial and National
Bank (48 Phil. 931). where We held:
It is, therefore, clear and evident that the law recognizes
and respects the right of a creditor holding a mortgage, pledge
or lien of any kind, attachment or execution on the property of
the debtor, recorded and not dissolved under said Act, to refrain
from voting at the election of an assignee, and consequently, to
preserve said right; to refrain from taking part or intervening in
the insolvency proceedings and to retain the property mortgaged
to him and the respective security or lien the court having no
power, even if the debtor is adjudged insolvent, to dispose of
said property, security or lien and cede or transfer them to the
sheriff or assignee by virtue of said adjudication ... as long as the
creditor does not voluntarily deliver or assign said property,
security or lien for the benefit of all the creditors of the insolvent.
It is true that the aforequoted ruling deals with insolvency but by
analogy the same could be adopted in this case considering that the rights of
a preferred creditor remain to be respected and recognized in every existing
situation. To hold otherwise would render the said rights inutile and illusory.
Besides, We find no substantial difference between the suspension of actions
in the instant case and that under the Insolvency Law. Consequently, the
herein order of suspension, could not have a different interpretation as
regards secured credits than that already given by this Court. The records
show that PCIB neither surrendered the pledged shares of stock and bonds
nor participated in the proceedings before the SEC regarding the suspension
of payments or actions of claims against Philfinance or in the latter's
subsequent dissolution and liquidation. The pledged properties being still in
PCIB's possession, the Receiver could not possess the same for equitable
distribution to the creditors of Philfinance.

Page 1016 of 1072

TOPIC: PETITIONS FOR DECLARATION IN THE STATE OF SUSPENSION OF


PAYMENTS
RADIOLA-TOSHIBA PHILIPPINES, INC.,
VS.
THE INTERMEDIATE APPELLATE COURT, HON. LEONARDO I. CRUZ
G.R. No. 75222 July 18, 1991
FACTS:
On July 2, 1980, three creditors filed a petition for the involuntary
insolvency of Carlos Gatmaytan and Teresita Gatmaytan, the private
respondents herein, the case docketed as Special Proceeding No. 1548 of the
then Court of First Instance (now Regional Trial Court) of Pampanga and
Angeles City.
On July 9, 1980, the respondent court issued an order taking
cognizance of the said matter. Moreover, the Court forbids the payment of
any debts, and the delivery of any property owing and belonging to said
respondents-debtors from other persons, or, to any other persons for the use
and benefit of the same respondents-debtors and/or the transfer of any
property by and for the said respondents-debtors to another, upon
petitioners' putting up a bond by way of certified and reputable sureties.
On April 12, 1983, petitioners-creditors filed second urgent motion for
issuance of insolvency order and resolution of the case, alleging among other
things, that in November, 1982, they filed an urgent motion to issue
insolvency order; on December 2, 1982, they presented a motion to prohibit
the city sheriff of Angeles City from disposing the personal and real
properties of the insolvent debtors, Carlos Gatmaytan and Teresita
Gatmaytan; on January 18, 1983, they appealed in the Bulletin Today issue of
even date a news item to the effect that Radiola-Toshiba Phil. Inc. has already
shut down its factory, sometime in March 1983, through their representative,
they caused to be investigated the real properties in the names of Carlos
Gatmaytan and Teresita Gatmaytan and they were surprised to find out that
some of the aforesaid properties were already transferred to Radiola-Toshiba
Phil. Inc.; and that in view of such development, it is their submission that
without an insolvency order and a resolution of the case which was ripe for
resolution as early as March 3, 1982, the rights and interest of petitionerscreditors would be injured and jeopardized.
ISSUE/S:

Page 1017 of 1072

whether or not the levy on attachment in favor of the petitioner


is dissolved by the insolvency proceedings against respondent
spouses commenced four months after said attachment.
RULING:
On this issue, Section 32 of the Insolvency Law (Act No. 1956, as
amended), provides:
Sec. 32 As soon as an assignee is elected or appointed and
qualified, the clerk of the court shall, by an instrument under his
hand and seal of the court, assign and convey to the assignee all
the real and personal property, estate, and effects of the debtor
with all his deeds, books, and papers relating thereto, and such
assignment shall relate back to the commencement of the
proceedings in insolvency, and shall relate back to the acts upon
the adjudication was founded, and by operation of law shall vest
the title to all such property, estate, and effects in the assignee,
although the same is then attached on mesne process, as the
property of the debtor. Such assignment shall operate to vest in
the assignee all of the estate of the insolvent debtor not exempts
by law from execution. It shall dissolve any attachment levied
within one month next preceding the commencement of the
insolvency proceedings and vacate and set aside any judgment
entered in any action commenced within thirty days immediately
prior to the commencement of insolvency proceedings and shall
set aside any judgment entered by default or consent of the
debtor within thirty days immediately prior to the
commencement of the insolvency proceedings. (Emphasis
supplied)
The provision of the above-quoted Section 32, of the Insolvency Law is
very clear that attachments dissolved are those levied within one (1)
month next preceding the commencement of the insolvency proceedings
and judgments vacated and set aside are judgments entered in any action,
including judgment entered by default or consent of the debtor, where the
action was filed within thirty (30) days immediately prior to the
commencement of the insolvency proceedings. In short, there is a cut off
period one (1) month in attachment cases and thirty (30) days in
judgments entered in actions commenced prior to the insolvency
proceedings. Section 79, on the other hand, relied upon by private
respondents, provides for the right of the plaintiff if the attachment is not
dissolved before the commencement of proceedings in insolvency, or is
dissolved by an undertaking given by the defendant, if the claim upon which
the attachment suit was commenced is proved against the estate of the
debtor. Therefore, there is no conflict between the two provisions.

Page 1018 of 1072

TOPIC: PETITIONS FOR DECLARATION IN THE STATE OF SUSPENSION OF


PAYMENTS
RIZAL COMMERCIAL BANKING CORPORATION,
VS.
INTERMEDIATE APPELLATE COURT AND BF HOMES INCORPORATED,
G.R. NO. 74851, SEPTEMBER 14, 1992
213 SCRA 223
FACTS:
On September 28, 1984, B.F. Homes filed a "Petition for Rehabilitation
and for Declaration or Suspension of Payment" with the Securities and
Exchange Commission. One of the creditors listed in its inventory of creditors
and liabilities was RCBC.
On October 26, 1984, RCBC requested the Provincial Sheriff of Rizal to extrajudicially foreclose its real estate mortgage on some properties of B.F.
Homes. A notice of extra-judicial foreclosure sale was issued by the Sheriff on
October 29, 1984, scheduled on November 29, 1984, copies furnished both
B.F. Homes (mortgagor) and RCBC (mortgagee).
On motion of B.F. Homes, the SEC issued on November 28, 1984 a
temporary restraining order, enjoining RCBC and the sheriff from proceeding
with the public auction sale. The sale was rescheduled to January 29, 1985.
On January 25, 1985, the SEC ordered the issuance of a writ of
preliminary injunction upon petitioner's filing of a bond. However, petitioner
did not file a bond until January 29, 1985, the very day of the auction sale, so
no writ of preliminary injunction was issued by the SEC. Presumably,
unaware of the filing of the bond, the sheriffs proceeded with the public
auction sale on January 29, 1985, in which RCBC was the highest bidder for
the properties auctioned.
On February 5, 1985, B.F. Homes filed in the SEC a consolidated motion
to annul the auction sale and to cite RCBC end the sheriff for contempt. RCBC
opposed the motion.
Because of the proceedings in the SEC, the sheriff withheld the
delivery to RCBC of a certificate of sale covering the auctioned properties.
On February 13, 1985, the SEC in Case No. 002693 belatedly issued a
writ of preliminary injunction stopping the auction sale which had been
conducted by the sheriff two weeks earlier.
On March 13, 1985, despite SEC Case No. 002693, RCBC filed with the
RTC, Rizal an action for mandamus against the provincial sheriff of Rizal and

Page 1019 of 1072

his deputy to compel them to execute in its favor a certificate of sale of the
auctioned properties.
On April 8, 1986, the IAC rendered a decision, setting aside the decision of
the trial court, dismissing the mandamus case and suspending issuance to
RCBC of new land titles, "until the resolution of case by SEC in Case No.
002693,"
ISSUE/S:
Whether or not the suspension of issuance to RCBC of new land
titles is valid
RULING:
While it is recognized that RCBC is a preferred creditor and likewise the
highest bidder at the auction sale, We have however stated that whenever a
distressed corporation asks the SEC for rehabilitation and suspension of
payments, preferred creditors may no longer assert such preference, but as
earlier stated, stand on equal footing with other creditors. Foreclosure shall
be disallowed so as not to prejudice other creditors, or cause discrimination
among them. If foreclosure is undertaken despite the fact that a petition for
rehabilitation has been filed, the certificate of sale shall not be delivered
pending rehabilitation. Likewise, if this has also been done, no transfer of
title shall be effected also, within the period of rehabilitation. The rationale
behind PD 902-A, as amended, is to effect a feasible and viable
rehabilitation. This cannot be achieved if one creditor is preferred over the
others.
In this connection, the prohibition against foreclosure attaches as soon
as a petition for rehabilitation is filed. Were it otherwise, what is to prevent
the petitioner from delaying the creation of the Management Committee and
in the meantime dissipate all its assets. The sooner the SEC takes over and
imposes a freeze on all the assets, the better for all concerned.

TOPIC: PETITIONS FOR DECLARATION IN THE STATE OF SUSPENSION OF


PAYMENTS

Page 1020 of 1072

RUBBERWORLD (PHILS.), INC.


vs.
NATIONAL LABOR RELATIONS COMMISSION
G.R. NO. 126773, APRIL 14, 1999
305 SCRA 722
FACTS:
Petitioner is a domestic corporation which used to be in the business of
manufacturing footwear, bags and garments. It filed with the Securities and
Exchange Commission on November 24, 1994 a petition for suspension of
payments praying that it be declared in a state of suspension of payments
and that the SEC accordingly issue an order restraining its creditors from
enforcing their claims against petitioner corporation. It further prayed for the
creation of a management committee as well as for the approval of the
proposed rehabilitation plan and memorandum of agreement between
petitioner corporation and its creditors. The SEC favourably granted said
petition for suspension of payments.
Private respondents, who claim to be employees of Petitioner
Corporation, filed against petitioners from April to July 1995 their respective
complaints for illegal dismissal, unfair labor practice, damages and payment
of separation pay, retirement benefits, 13th month pay and service incentive
pay.
Petitioners moved to suspend the proceedings in the above labor cases
on the strength of the SEC Order dated December 28, 1994. However, the
Labor Arbiter denied the aforesaid motion holding that the injunction
contained in the SEC Order applied only to the enforcement of established
rights and did not include the suspension of proceedings involving claims
against petitioner which have yet to be ascertained. The Labor Arbiter
further held that the order of the SEC suspending all actions for claims
against petitioners does not cover the claims of private respondents in the
labor cases because said claims and the concomitant liability of petitioners
still had to be determined, thus carrying no dissipation of the assets of
petitioners. The same was affirmed by the NLRC.
ISSUE/S:
Whether or not the Respondent NLRC acted without or in excess
of Jurisdiction or with grave abuse of discretion amounting to lack
of jurisdiction in affirming the order of the labor arbiter denying
petitioners motion to suspend proceedings despite the SECs
Order directing the suspension of all actions against a company
under the first stages of insolvency proceedings.
RULING:

Page 1021 of 1072

The Supreme Court disagrees.


The lawisclear: upon the creation of a management committee or the
appointment of rehabilitation receiver, all claims for actions "shall be
suspended accordingly." No exception in favor of labor claims is mentioned
in the law. Since the law makes no distinction or exemptions, neither should
this Court. Ubi lex non distinguit nec nos distinguere debemos. Allowing
labor cases to proceed clearly defeats the purpose of the automatic stay
and severely encumbers the management committee's time and resources.
The said committee would need to defend against these suits, to the
detriment of its primary and urgent duty to work towards rehabilitating the
corporation and making it viable again. To rule otherwise would open the
floodgates to other similarly situated claimants and forestall if not defeat the
rescue efforts. Besides, even if the NLRC awards the claims of private
respondents, as it did, its ruling could not be enforced as long as the
petitioner is under the management committee.
In Chua v. National Labor Relation Commission, we ruled that labor
claims cannot proceed independently of a bankruptcy liquidation proceeding,
since these claims "would spawn needless controversy, delays, and
confusion." With more reason, allowing labor claims to continue in spite of a
SEC suspension order in rehabilitation case would merely lead to such
results.
In Chua v. National Labor Relation Commission, we ruled that labor
claims cannot proceed independently of a bankruptcy, liquidation
proceeding, since these claims "would spawn needless controversy, delays,
and confusion". With more reason, allowing labor claims allowing, labor
claims to continue in spite of a SEC suspension order in rehabilitation case
would merely lead to such results.
The preferential right of workers and employees under Article 110 of
the Labor Code may be invoked only upon the institution of insolvency or
judicial liquidation proceeding. Indeed, it is well-settled that "a declaration of
bankruptcy, or a judicial liquidation must be present before preferences over
various money, claims may be enforced But debtors resort to preference of
credit -- giving preferred creditors the right to have their claims paid ahead
of those of other claimants -- only when their assets are insufficient to pay
their debts fully. The purpose of rehabilitation proceedings is precisely to
enable the company to gain a new lease on life and thereby allow creditors
to be paid their claims from its earnings. In insolvency proceedings, on the
other hand, the company stops operating, and the claims of creditors are
satisfied from the assets of the insolvent corporation. The present case
involves the rehabilitation, not the liquidation, of petitioner-corporation.
Hence, the preference of credit granted to workers or employees underArticle 110 of the Labor Code is not applicable.

Page 1022 of 1072

TOPIC: PETITIONS FOR DECLARATION IN THE STATE OF SUSPENSION OF


PAYMENTS
UNION BANK OF THE PHILIPPINES
VS.
THE HONORABLE COURT OF APPEALS
G.R. No. 131729, May 19, 1998
FACTS:
On September 16, 1997, private respondents EYCO Group of
Companies ("EYCO"), and the Yutingcos, all of whom are controlling
stockholders of the aforementioned corporations, jointly filed with the SEC a
Petition for the Declaration of Suspension of Payments, Formation and
Appointment
of
Rehabilitation
Receiver/Committee,
Approval
of
Rehabilitation Plan with Alternative Prayer for Liquidation and Dissolution of
Corporations. Upon finding the above petition to be sufficient in form and
substance, the SEC Hearing Panel dated September 19, 1997 setting its
hearing on October 22, 1997. At the same time, said panel also directed the
suspension of all actions, claims and proceedings against private
respondents pending before any court, tribunal, office, board and/or
commission.
Meanwhile, some of private respondents' creditors, composed mainly
of twenty-two domestic banks including herein petitioner Union Bank of the
Philippines, also convened on September 19, 1997 for the purpose of
deciding their options in the event that private respondents invoke the
provisions of Presidential Decree No. 902-A, as amended. Without notifying
the members of the consortium, Union Bank, however, decided to break
away from the group by suing private respondents in the regular courts.
Aside from commencing suits in the regular courts, petitioner also
vehemently opposed private respondents' petition for suspension of
payments in the SEC by filing a Motion to Dismiss on October 22, 1997. It
contended that the SEC was bereft of jurisdiction over such petition on the
ground that the inclusion of the Yutingcos in the petition "cannot be allowed
since the authority and power of the Commission under the virtue of the law
applies only to corporations, partnerships and other forms of associations,
and not to individual petitioners who are not clearly covered by P.D. 902-A as
amended."
ISSUE/S:
Whether or not the SEC can validly acquire jurisdiction over a
petition for suspension of payments filed pursuant to Section 5

Page 1023 of 1072

(d) of P.D. No. 902 A, as amended, when such petition joins as


co-petitioners the petitioning corporate entities AND individual
stockholders thereof
RULING:
The Supreme Court fully agrees with petitioner in contending that the
SEC's jurisdiction on matters of suspension of payments is confined only to
those initiated by corporations, partnerships or associations. Actually, this is
not the first time that the Court has encountered an issue as the one at bar.
It has made a similar pronouncement in the seminal case of Chung Ka Bio v.
Intermediate Appellate Court, et al., likewise involving a petition for
suspension of payments filed by a corporate entity and an individual
stockholder, where we ruled that:
This section [referring to Section 5 (d) of P.D. No. 902-A, as amended]
clearly does not allow a mere individual to file the petition which is limited to
"corporations, partnerships or associations." Administrative agencies like the
SEC are tribunals of limited jurisdiction and, as such, can exercise only those
powers which are specifically granted to them by their enabling statutes.
Consequently, where no authority is granted to hear petitions of individuals
for suspension of payments, such petitions are beyond the competence of
the SEC. . .
The circumstance that Ching is a co-signer in the corporation's promissory
notes, collateral or guarantee or security agreements, does not make him a
proper party. Jurisdiction over the subject matter must exist as a matter of
law and cannot be fixed by agreement of the parties, acquired through, or
waived, enlarged or diminished by, any act or omission; neither can it be
conferred by acquiescence of the tribunal. Hence, Alfredo Ching, as a mere
individual, cannot be allowed as a co-petitioner in SEC Case No. 2250.
Notwithstanding the foregoing conclusions, this Court, however, does
not subscribe to the theory espoused by petitioner that the case filed by
private respondents should be dismissed outright in its entirety. The reason is
that while it is true that the SEC cannot acquire jurisdiction over an individual
filing a petition for suspension of payments together with a corporate entity,
a closer scrutiny of Chung Ka Bio and MPPI does not in any manner suggest,
even tangentially, that a petition as the one at bar must be dismissed
likewise with respect to the corporate co-petitioner. What Chung Ka Bio and
MPPI respectively declared was that "Alfredo Ching, as a mere individual,
cannot be allowed as a co-petitioner in SEC Case No. 2250" and "respondent
Court of Appeals was correct in ordering the dismissal of the petition for
suspension of payments insofar as the Co spouses were concerned."
In a case of misjoinder of parties which in this case is the co-filing of
the petition for suspension of payments by both the Yutingcos and the EYCO
group the remedy has never been to dismiss the petition in its entirety but

Page 1024 of 1072

to dismiss it only as against the party upon whom the tribunal or body
cannot acquire jurisdiction. The result, therefore, is that the petition with
respect to EYCO shall subsist and may be validly acted upon by the SEC. The
Yutingcos, on the other hand, shall be dropped from the petition and be
required to pursue their remedies in the regular courts of competent
jurisdiction.
We are, of course, aware of the argument advanced by petitioner that the
petition should be entirely dismissed and taken out of the SEC's jurisdiction
on account of the alleged insolvency of private respondents. In this regard,
petitioner theorizes that private respondents have already become insolvent
when they allegedly disposed of a substantial portion of their properties in
fraud of creditors, hence, suspension of payments with the SEC is not the
proper remedy.
Such argument does not persuade us. Petitioner's allegations of
fraudulent dispositions of private respondents' assets and the supposed
insolvency of the latter are hardly of any consequence to the assumption of
jurisdiction by the SEC over the nature or subject matter of the petition for
suspension of payments. Aside from the fact that these allegations are
evidentiary in nature and still remains to be proved, we have likewise
consistently ruled that what determines the nature of an action, as well as
which court or body has jurisdiction over it, are the allegations of the
complaint, or a petition as in this case, and the character of the relief sought.
That the merits of the case after due proceedings are later found to veer
away from the claims asserted by EYCO in its petition, as when it is shown
later that it is actually insolvent and may not be entitled to suspension of
payments, does not divest the SEC at all of its jurisdiction already acquired
at its inception through the allegations made in the petition.

Page 1025 of 1072

TOPIC: ELEMENTS OF AN INVESTMENT CONTRACT


Securities and Exchange Commission
vs.
Howey Co.
No. 843
Argued May 2, 1946
Decided May 27, 1946
328 U.S. 293
FACTS:
The respondents, W. J. Howey Company and Howey-in-the-Hills
Service, Inc., are Florida corporations under direct common control and
management. The Howey Company owns large tracts of citrus acreage in
Lake County, Florida. During the past several years, it has planted about 500
acres annually, keeping half of the groves itself and offering the other half to
the public "to help us finance additional development." Howey-in-the-Hills
Service, Inc., is a service company engaged in cultivating and developing
many of these groves, including the harvesting and marketing of the crops.
Each prospective customer is offered both a land sales contract and a
service contract, after having been told that it is not feasible to invest in a
grove unless service arrangements are made. While the purchaser is free to
make arrangements with other service companies, the superiority of Howeyin-the-Hills Service, Inc., is stressed. Indeed, 85% of the acreage sold during
the 3-year period ending May 31, 1943, was covered by service contracts
with Howey-in-the-Hills Service, Inc.
The land sales contract with the Howey Company provides for a
uniform purchase price per acre or fraction thereof, varying in amount only in
accordance with the number of years the particular plot has been planted
with citrus trees. Upon full payment of the purchase price, the land is
conveyed to the purchaser by warranty deed. Purchases are usually made in
narrow strips of land arranged so that an acre consists of a row of 48 trees.
During the period between February 1, 1941, and May 31, 1943, 31 of the 42
persons making purchases bought less than 5 acres each. The average
holding of these 31 persons was 1.33 acres, and sales of as little as O.65, O.7
and O.73 of an acre were made. These tracts are not separately fenced, and

Page 1026 of 1072

the sole indication of several ownership is found in small land marks


intelligible only through a plat book record.
The service contract, generally of a 10-year duration without option of
cancellation, gives Howey-in-the-Hills Service, Inc., a leasehold interest and
"full and complete" possession of the acreage. For a specified fee plus the
cost of labor and materials, the company is given full discretion and authority
over the cultivation of the groves and the harvest and marketing of the
crops. Without the consent of the company, the landowner or purchaser has
no right of entry to market the crop; thus, there is ordinarily no right to
specific fruit. The company is accountable only for an allocation of the net
profits based upon a check made at the time of picking. All the produce is
pooled by the respondent companies, which do business under their own
names.
The purchasers, for the most part, are nonresidents of Florida. They are
predominantly business and professional people who lack the knowledge,
skill, and equipment necessary for the care and cultivation of citrus trees.
They are attracted by the expectation of substantial profits. It was
represented, for example, that profits during the 1943-1944 season
amounted to 20%, and that even greater profits might be expected during
the 1944-1945 season, although only a 10% annual return was to be
expected over a 10-year period. Many of these purchasers are patrons of a
resort hotel owned and operated by the Howey Company in a scenic section
adjacent to the groves. The hotel's advertising mentions the fine groves in
the vicinity, and the attention of the patrons is drawn to the groves as they
are being escorted about the surrounding countryside. They are told that the
groves are for sale; if they indicate an interest in the matter, they are then
given a sales talk.
ISSUE/S:
Whether or not, under the circumstances, the land sales contract, the
warranty deed and the service contract together constitute an
"investment contract" within the meaning of 2(1).
RULING:
An investment contract mean a contract or scheme for "the placing of
capital or laying out of money in a way intended to secure income or profit
from its employment." Individuals were led to invest money in a common
enterprise with the expectation that they would earn a profit solely through
the efforts of the promoter or of someone other than themselves.
By including an investment contract within the scope of 2(1) of the
Securities Act, Congress was using a term the meaning of which had been
crystalized by this prior judicial interpretation. It is therefore reasonable to
attach that meaning to the term as used by Congress, especially since such a

Page 1027 of 1072

definition is consistent with the statutory aims. In other words, an investment


contract, for purposes of the Securities Act, means a contract, transaction or
scheme whereby a person invests his money in a common enterprise and is
led to expect profits solely from the efforts of the promoter or a third party, it
being immaterial whether the shares in the enterprise are evidenced by
formal certificates or by nominal interests in the physical assets employed in
the enterprise. Such a definition necessarily underlies this Court's decision
in SEC v. Joiner Corp., 320 U. S. 344, and has been enunciated and applied
many times by lower federal courts. [Footnote 5] It permits the fulfillment of
the statutory purpose of compelling full and fair disclosure relative to the
issuance of "the many types of instruments that, in our commercial world,
fall within the ordinary concept of a security." H.Rep. No.85, 73rd Cong., 1st
Sess., p. 11. It embodies a flexible, rather than a static, principle, one that is
capable of adaptation to meet the countless and variable schemes devised
by those who seek the use of the money of others on the promise of profits.
The transactions in this case clearly involve investment contracts, as
so defined. The respondent companies are offering something more than fee
simple interests in land, something different from a farm or orchard coupled
with management services. They are offering an opportunity to contribute
money and to share in the profits of a large citrus fruit enterprise managed
and partly owned by respondents. They are offering this opportunity to
persons who reside in distant localities and who lack the equipment and
experience requisite to the cultivation, harvesting, and marketing of the
citrus products. Such persons have no desire to occupy the land, or to
develop it themselves; they are attracted solely by the prospects of a return
on their investment. Indeed, individual development of the plots of land that
are offered and sold would seldom be economically feasible, due to their
small size. Such tracts gain utility as citrus groves only when cultivated and
developed as component parts of a larger area. A common enterprise
managed by respondents or third parties with adequate personnel and
equipment is therefore essential if the investors are to achieve their
paramount aim of a return on their investments. Their respective shares in
this enterprise are evidenced by land sales contracts and warranty deeds,
which serve as a convenient method of determining the investors' allocable
shares of the profits. The resulting transfer of rights in land is purely
incidental.
Thus, all the elements of a profit-seeking business venture are present
here. The investors provide the capital and share in the earnings and profits;
the promoters manage, control, and operate the enterprise. It follows that
the arrangements whereby the investors' interests are made manifest
involve investment contracts, regardless of the legal terminology in which
such contracts are clothed. The investment contracts in this instance take
the form of land sales contracts, warranty deeds, and service contracts which
respondents offer to prospective investors. And respondents' failure to abide

Page 1028 of 1072

by the statutory and administrative rules in making such offerings, even


though the failure result from a bona fide mistake as to the law, cannot be
sanctioned under the Act.
This conclusion is unaffected by the fact that some purchasers choose
not to accept the full offer of an investment contract by declining to enter
into a service contract with the respondents. The Securities Act prohibits the
offer, as well as the sale, of unregistered, nonexempt securities. Hence, it is
enough that the respondents merely offer the essential ingredients of an
investment contract.

TOPIC: ELEMENTS OF AN INVESTMENT CONTRACT

SECURITIES AND EXCHANGE COMMISSION vs. PROSPERITY.COM, INC.,


G.R. No. 164197

January 25, 2012

FACTS:
Prosperity.Com, Inc. (PCI) sold computer software and hosted websites
without providing internet service. To make a profit, PCI devised a scheme in
which, for the price of US$234.00 (subsequently increased to US$294), a
buyer could acquire from it an internet website of a 15-Mega Byte (MB)
capacity. At the same time, by referring to PCI his own down-line buyers, a
first-time buyer could earn commissions, interest in real estate in the
Philippines and in the United States, and insurance coverage worth
P50,000.00.
To benefit from this scheme, a PCI buyer must enlist and sponsor at
least two other buyers as his own down-lines. These second tier of buyers
could in turn build up their own down-lines. For each pair of down-lines, the
buyer-sponsor received a US$92.00 commission. But referrals in a day by the
buyer-sponsor should not exceed 16 since the commissions due from excess
referrals inure to PCI, not to the buyer-sponsor.
Apparently, PCI patterned its scheme from that of Golconda Ventures,
Inc. (GVI), which company stopped operations after the Securities and
Exchange Commission (SEC) issued a cease and desist order (CDO) against

Page 1029 of 1072

it. As it later on turned out, the same persons who ran the affairs of GVI
directed PCIs actual operations.
In 2001, disgruntled elements of GVI filed a complaint with the SEC
against PCI, alleging that the latter had taken over GVIs operations. After
hearing,1 the SEC, through its Compliance and Enforcement unit, issued a
CDO against PCI. The SEC ruled that PCIs scheme constitutes an Investment
contract and, following the Securities Regulations Code,2 it should have first
registered such contract or securities with the SEC.
ISSUE:
Whether or not PCIs scheme constitutes an investment contract that
requires registration under R.A. 8799.

RULING:
The Securities Regulation Code treats investment contracts as
securities that have to be registered with the SEC before they can be
distributed and sold. An investment contract is a contract, transaction, or
scheme where a person invests his money in a common enterprise and is led
to expect profits primarily from the efforts of others.
Apart from the definition, which the Implementing Rules and
Regulations provide, Philippine jurisprudence has so far not done more to
add to the same. Of course, the United States Supreme Court, grappling
with the problem, has on several occasions discussed the nature of
investment contracts. That courts rulings, while not binding in the
Philippines, enjoy some degree of persuasiveness insofar as they are logical
and consistent with the countrys best interests.
The United States Supreme Court held in Securities and Exchange
Commission v. W.J. Howey Co. that, for an investment contract to exist, the
following elements, referred to as the Howey test must concur: (1) a
contract, transaction, or scheme; (2) an investment of money; (3) investment
is made in a common enterprise; (4) expectation of profits; and (5) profits
arising primarily from the efforts of others. Thus, to sustain the SEC position
in this case, PCIs scheme or contract with its buyers must have all these
elements.

Page 1030 of 1072

An example that comes to mind would be the long-term commercial


papers that large companies, like San Miguel Corporation (SMC), offer to the
public for raising funds that it needs for expansion. When an investor buys
these papers or securities, he invests his money, together with others, in
SMC with an expectation of profits arising from the efforts of those who
manage and operate that company. SMC has to register these commercial
papers with the SEC before offering them to investors.
Here, PCIs clients do not make such investments. They buy a product
of some value to them: an Internet website of a 15-MB capacity. The client
can use this website to enable people to have internet access to what he has
to offer to them, say, some skin cream. The buyers of the website do not
invest money in PCI that it could use for running some business that would
generate profits for the investors. The price of US$234.00 is what the buyer
pays for the use of the website, a tangible asset that PCI creates, using its
computer facilities and technical skills.
TOPIC: ELEMENTS OF AN INVESTMENT CONTRACT

POWER HOMES UNLIMITED CORPORATION, petitioner,


vs.
SECURITIES AND EXCHANGE COMMISSION AND NOEL
MANERO, respondents
G.R. No. 164182
February 26, 2008
495 S579
FACTS:
Petitioner is a domestic corporation duly registered with public
respondent SEC on October 13, 2000. Its primary purpose is: To engage in
the transaction of promoting, acquiring, managing, leasing, obtaining options
on, development, and improvement of real estate properties for subdivision
and allied purposes, and in the purchase, sale and/or exchange of said
subdivision and properties through network marketing.
Respondent Noel Manero requested public respondent SEC to
investigate petitioners business. He claimed that he attended a seminar
conducted by petitioner where the latter claimed to sell properties that were
inexistent and without any brokers license. Romulo E. Munsayac, Jr. inquired
from public respondent SEC whether petitioners business involves
"legitimate network marketing."

Page 1031 of 1072

Respondent SEC, held a conference on December 13, 2000 that was


attended by petitioners incorporators John Lim, Paul Nicolas and Leonito
Nicolas. The attendees were requested to submit copies of petitioners
marketing scheme and list of its members with addresses.
Public respondent SEC visited the business premises of petitioner
wherein it gathered documents such as certificates of accreditation to
several real estate companies, list of members with web sites, sample of
member mail box, webpages of two (2) members, and lists of Business
Center Owners who are qualified to acquire real estate properties and
materials on computer tutorials.
Public respondent SEC found petitioner to be engaged in the sale or
offer for sale or distribution of investment contracts, which are considered
securities under Sec. 3.1 (b) of Republic Act (R.A.) No. 8799 (The Securities
Regulation Code), but failed to register them in violation of Sec. 8.1 of the
same Act, public respondent SEC issued a CDO and ordered the petitioner to
immediately CEASE AND DESIST from further engaging in the sale, offer or
distribution of the securities upon the receipt of this order.
ISSUE/S:
Whether or not petitioners business constitutes an investment
contract which should be registered with public respondent SEC before
its sale or offer for sale or distribution to the public.

RULING:
Section 8. Requirement of Registration of Securities. 8.1. Securities
shall not be sold or offered for sale or distribution within the Philippines,
without a registration statement duly filed with and approved by the
Commission. Prior to such sale, information on the securities, in such form
and with such substance as the Commission may prescribe, shall be made
available to each prospective purchaser.
A test was established to determine whether a transaction falls within
the scope of an "investment contract."Known as the Howey Test, it requires
a transaction, contract, or scheme whereby a person (1) makes an
investment of money, (2) in a common enterprise, (3) with the expectation of
profits, (4) to be derived solely from the efforts of others.Although the
proponents must establish all four elements, the US Supreme Court stressed
that the Howey Test "embodies a flexible rather than a static principle, one
that is capable of adaptation to meet the countless and variable schemes
devised by those who seek the use of the money of others on the promise of
profits."Needless to state, any investment contract covered by the Howey

Page 1032 of 1072

Test must be registered under the Securities Act, regardless of whether its
issuer was engaged in fraudulent practices.
After Howey came the 1973 US case of SEC v. Glenn W. Turner
Enterprises, Inc. et al. In this case, the 9thCircuit of the US Court of Appeals
ruled that the element that profits must come "solely" from the efforts of
others should not be given a strict interpretation. It held that a literal reading
of the requirement "solely" would lead to unrealistic results. It reasoned out
that its flexible reading is in accord with the statutory policy of affording
broad protection to the public. Our R.A. No. 8799 appears to follow this
flexible concept for it defines an investment contract as a contract,
transaction or scheme (collectively "contract") whereby a person invests his
money in a common enterprise and is led to expect profits not solely
but primarily from the efforts of others. Thus, to be a security subject to
regulation by the SEC, an investment contract in our jurisdiction must be
proved to be: (1) an investment of money, (2) in a common enterprise, (3)
with expectation of profits, (4) primarily from efforts of others.
The trainings or seminars are merely designed to enhance petitioners
business of teaching its investors the know-how of its multi-level marketing
business. An investor enrolls under the scheme of petitioner to be entitled to
recruit other investors and to receive commissions from the investments of
those directly recruited by him. Under the scheme, the accumulated amount
received by the investor comes primarily from the efforts of his recruits.
The business operation or the scheme of petitioner constitutes an
investment contract that is a security under R.A. No. 8799. Thus, it must be
registered with public respondent SEC before its sale or offer for sale or
distribution to the public. As petitioner failed to register the same, its offering
to the public was rightfully enjoined by public respondent SEC. The CDO was
proper even without a finding of fraud. As an investment contract that is
security under R.A. No. 8799, it must be registered with public respondent
SEC; otherwise the SEC cannot protect the investing public from fraudulent
securities. The strict regulation of securities is founded on the premise that
the capital markets depend on the investing publics level of confidence in
the system.

Page 1033 of 1072

TOPIC: ELEMENTS OF AN INVESTMENT CONTRACT

HIKOI SUZUKI, RAMON DEL ROSARIO and TAKAYUKI SATO, Petitioners,


vs.
DIANA DE GUZMAN, Respondent.
G.R. No. 146979 : July 27, 2006
496 S CRA 651
FACTS:
On January 10, 1996, the Suzuki Beach Hotel, Inc. (SBHI) was
registered with the Securities and Exchange Commission (SEC), with Diana
de Guzman (respondent) and Editha Taborda (Taborda) as two of the
incorporators. Respondent subscribed to 29,800 shares with a total par value
of P2, 980,000.00 and paid her subscription in the amount of P745, 000.00.
Taborda subscribed to 100 shares with a total par value of P10, 000.00 and
paid P2, 500.00 on her subscription.
Hikoi Suzuki, Ramon del Rosario, Takayuki Sato (petitioners), acting as
Board of Directors of SBHI, issued a Resolution declaring due and

Page 1034 of 1072

demandable all unpaid shares of stock and gave the stockholders until
December 30, 1997 to pay their unpaid subscription. Notice of the call for
payment was sent to respondent and Taborda but they failed to pay their
respective unpaid subscriptions.
On January 9, 1998, petitioners scheduled a meeting of the Board on
January 12, 1998 to discuss the sale of delinquent shares of stocks. On
January 10, 1998, notice of the meeting was sent to respondent and Taborda.
On January 12, 1998, petitioners approved a Resolution to sell all delinquent
shares of stock at a public auction set on January 30, 1998.
On January 30, 1998, the auction sale was conducted. Petitioner
Ramon del Rosario and Agnes Rodriguez (Rodriguez) submitted the winning
bids for the shares of stock of respondent and Taborda, respectively. On the
same day, respondent and Taborda filed an Affidavit of Protest on the auction
sale of their shares of stock.
On March 4, 1998, respondent and Taborda filed with the SEC a Petition
for Calling Special Stockholders Meetings and for Election of Directors and
Officers, Declaration of Nullity of the Call of Sale of Unpaid Stock Subscription
with Writ of Preliminary Injunction and Temporary Restraining Order.
ISSUE/S:
Whether or not the SEC has jurisdiction over the case.
RULING:
The time the CA promulgated the assailed Decision on January 26,
2000, the SEC was still empowered, under Section 5 of P.D. 902-A, to hear
and decide cases involving intra-corporate disputes, thus:
SEC. 5(b) Controversies arising out of intra-corporate or
partnership relations, between and among stockholders,
members or associates; between any or all of them and the
corporation, partnership or association of which they are the
stockholders, members or associates, respectively; and between
such corporation, partnership or association and the state insofar
as it concerns their individual franchise or right to exist as such
entity;
However, on August 9, 2000, during the pendency of petitioners'
Motion for Reconsideration of the CA Decision, R.A. No. 8799 took effect.
Section 5.2 of R.A. No. 8799, provides:
5.2. The
Commission's
jurisdiction
over
all
cases
enumerated under Section 5 of Presidential Decree No.
902-A is hereby transferred to the Courts of general
jurisdiction
or
the
appropriate
Regional
Trial

Page 1035 of 1072

Court:Provided, That the Supreme Court in the exercise of its


authority may designate the Regional Trial Court branches that
shall exercise jurisdiction over these cases. The Commission
shall retain jurisdiction over pending cases involving
intra-corporate disputes submitted for final resolution
which should be resolved within one (1) year from the
enactment of this Code. The Commission shall retain
jurisdiction over pending suspension of payments/rehabilitation
cases filed as of 30 June 2000 until finally disposed.
Thus, original and exclusive jurisdiction to hear and decide cases
involving intra-corporate controversies have been transferred to courts of
general jurisdiction or the appropriate Regional Trial Court. The case
involving herein parties has not been submitted for final resolution on the
merits in the SEC. Only the issue on jurisdiction was dealt with by the SEC
which is the subject of herein petition. Thus, herein case does not fall within
the exception advert to in the aforequoted Section 5.2.
Although the petition filed with the CA was procedurally deficient for
non-compliance with the rules on material date and certification of nonforum shopping, the CA should have reconsidered its Decision on the
question of jurisdiction in view of the advent of R.A. No. 8799 transferring
cases originally cognizable by the SEC to the Regional Trial Courts.
Technicalities must give way to the realities of the situation. It is elementary
that jurisdiction over the subject matter, or the jurisdiction to hear and
decide a case, is conferred by law and it is not within the courts, let alone the
parties, to themselves determine and conveniently set aside. Pursuant to
Section 5.2 of R.A. No. 8799, this Court designated specific branches of the
Regional Trial Courts to try and decide cases formerly cognizable by the SEC.
TOPIC: ELEMENTS OF AN INVESTMENT CONTRACT

MANUEL V. BAVIERA, Petitioner,


vs.
ESPERANZA PAGLINAWAN, in her capacity as Department of Justice
State Prosecutor; LEAH C. TANODRA-ARMAMENTO, In her capacity as
Assistant Chief State Prosecutor and Chairwoman of Task Force on
Business Scam; JOVENCITO R. ZUNO, in his capacity as Department
of Justice Chief State Prosecutor; STANDARD CHARTERED BANK,
PAUL SIMON MORRIS, AJAY KANWAL, SRIDHAR RAMAN, MARIVEL
GONZALES, CHONA REYES, MARIA ELLEN VICTOR, and ZENAIDA
IGLESIAS, Respondents.
G.R. No. 168380: February 8, 2007
515 SCRA 170

Page 1036 of 1072

FACTS:
Manuel Baviera, petitioner in these cases, was the former head of the
HR Service Delivery and Industrial Relations of Standard Chartered BankPhilippines (SCB), one of herein Respondents. SCB is a foreign banking
corporation duly licensed to engage in banking, trust, and other fiduciary
business in the Philippines. Pursuant to Resolution No. 1142 dated December
3, 1992 of the Monetary Board of the Bangko Sentral ng Pilipinas (BSP), the
conduct of SCB's business in this jurisdiction is subject to the following
conditions:
1) At the end of a one-year period from the date the SCB
starts its trust functions, at least 25% of its trust accounts
must be for the account of non-residents of the Philippines
and that actual foreign exchange had been remitted into
the Philippines to fund such accounts or that the
establishment of such accounts had reduced the
indebtedness of residents (individuals or corporations or
government agencies) of the Philippines to non-residents.
At the end of the second year, the above ratio shall be
50%, which ratio must be observed continuously
thereafter;
2) The trust operations of SCB shall be subject to all existing
laws, rules and regulations applicable to trust services,
particularly the creation of a Trust Committee; and
3) The bank shall inform the appropriate supervising and
examining department of the BSP at the start of its
operations.
SCB did not comply with the above conditions. Instead, as early as
1996, it acted as a stock broker, soliciting from local residents foreign
securities called "GLOBAL THIRD PARTY MUTUAL FUNDS" (GTPMF),
denominated in US dollars. These securities were not registered with the
Securities and Exchange Commission (SEC). These were then remitted
outwardly to SCB-Hong Kong and SCB-Singapore.
SCB's counsel, Romulo Mabanta Buenaventura Sayoc and Delos
Angeles Law Office, advised the bank to proceed with the selling of the
foreign securities although unregistered with the SEC, under the guise of a
"custodianship agreement;" and should it be questioned, it shall invoke
Section 72of the General Banking Act (Republic Act No.337).In sum, SCB was
able to sell GTPMF securities worth around P6 billion to some 645 investors.
However, SCB's operations did not remain unchallenged. On July 18,
1997, the Investment Capital Association of the Philippines (ICAP) filed with
the SEC a complaint alleging that SCB violated the Revised Securities Act,
particularly the provision prohibiting the selling of securities without prior

Page 1037 of 1072

registration with the SEC; and that its actions are potentially damaging to the
local mutual fund industry.
SCB denied offering and selling securities, contending that it has been
performing a "purely informational function" without solicitations for any of
its investment outlets abroad; that it has a trust license and the services it
renders under the "Custodianship Agreement" for offshore investments are
authorized by Section 72 of the General Banking Act; that its clients were the
ones who took the initiative to invest in securities; and it has been acting
merely as an agent or "passive order taker" for them.
Notwithstanding its commitment and the BSP directive, SCB continued
to offer and sell GTPMF securities in this country. This prompted petitioner to
enter into an Investment Trust Agreement with SCB wherein he purchased
US$8,000.00 worth of securities upon the bank's promise of 40% return on
his investment and a guarantee that his money is safe. After six (6) months,
however, petitioner learned that the value of his investment went down to
US$7,000.00. He tried to withdraw his investment but was persuaded by
Antonette de los Reyes of SCB to hold on to it for another six (6) months in
view of the possibility that the market would pick up.
The in vestment of the petitioner continued to went down further up to
US S3,000.00 thus he filed for letter-complaint demanding compensation for
his lost investment. But SCB denied his demand on the ground that his
investment is "regular."
ISSUE/S:
Whether or not the DOJ did not commit grave abuse of discretion in
dismissing petitioner's complaint in I.S. 2004-229 for violation of
Securities Regulation Code.

RULING:
For violation of the Securities Regulation Code:
Section 53.1 of the Securities Regulation Code provides:
SEC. 53. Investigations, Injunctions and Prosecution of
Offenses.53. 1. The Commission may, in its discretion, make
such investigation as it deems necessary to
determine whether any person has violated or is
about to violate any provision of this Code, any rule,
regulation or order thereunder, or any rule of an
Exchange, registered securities association, clearing

Page 1038 of 1072

agency, other self-regulatory organization, and may


require or permit any person to file with it a
statement in writing, under oath or otherwise, as the
Commission shall determine, as to all facts and
circumstances concerning the matter to be
investigated.
The
Commission
may
publish
information concerning any such violations and to
investigate any fact, condition, practice or matter
which it may deem necessary or proper to aid in the
enforcement of the provisions of this Code, in the
prescribing of rules and regulations there under, or in
securing information to serve as a basis for
recommending further legislation concerning the
matters to which this Code relates: Provided,
however, That any person requested or subpoenaed
to produce documents or testify in any investigation
shall simultaneously be notified in writing of the
purpose
of
such
investigation: Provided,
further, That
all
criminal
complaints
for
violations of this Code and the implementing
rules and regulations enforced or administered
by the Commission shall be referred to the
Department
of
Justice
for
preliminary
investigation and prosecution before the
proper
court: Provided,
furthermore, That
in
instances where the law allows independent civil or
criminal proceedings of violations arising from the
act, the Commission shall take appropriate action to
implement the same: Provided, finally; That the
investigation, prosecution, and trial of such cases
shall be given priority.
A criminal complaint for violation of any law or rule administered by
the SEC must first be filed with the latter. If the Commission finds that there
is probable cause, then it should refer the case to the DOJ. Since petitioner
failed to comply with the foregoing procedural requirement, the DOJ did not
gravely abuse its discretion in dismissing his complaint in I.S. No. 2004-229.
A criminal charge for violation of the Securities Regulation Code is a
specialized dispute. Hence, it must first be referred to an administrative
agency of special competence, i.e., the SEC. Under the doctrine of primary
jurisdiction, courts will not determine a controversy involving a question
within the jurisdiction of the administrative tribunal, where the question
demands the exercise of sound administrative discretion requiring the
specialized knowledge and expertise of said administrative tribunal to
determine technical and intricate matters of fact. The Securities Regulation

Page 1039 of 1072

Code is a special law. Its enforcement is particularly vested in the SEC.


Hence, all complaints for any violation of the Code and its implementing
rules and regulations should be filed with the SEC. Where the complaint is
criminal in nature, the SEC shall indorse the complaint to the DOJ for
preliminary investigation and prosecution as provided in Section 53.1 earlier
quoted.
Petitioner committed a fatal procedural lapse when he filed his criminal
complaint directly with the DOJ. Verily, no grave abuse of discretion can be
ascribed to the DOJ in dismissing petitioner's complaint.

TOPIC: WHAT SECURITIES ARE REQUIRED TO BE REGISTERED:


TIMESHARE REALTY CORPORATION, Petitioner,
vs.
CESAR LAO and CYNTHIA V. CORTEZ, Respondents.

Page 1040 of 1072

G.R. No. 158941

February 11, 2008

FACTS:
Petitioner sold to Ceasar M. Lao and Cynthia V. Cortez (respondents),
one timeshare of Laguna de Boracay for US$7,500.00 under Contract No.
135000998 payable in eight months and fully paid by the respondents.
Subsequently, SEC issued a resolution to the effect that petitioner was
without authority to sell securities, like timeshares, prior to February 11,
1998. It further stated in the resolution/order that the Registration Statement
of petitioner became effective only on February 11, 1998. It also held that
the 30 days within which a purchaser may exercise the option to unilaterally
rescind the purchase agreement and receive the refund of money paid
applies to all purchase agreements entered into by petitioner prior to the
effectivity of the Registration Statement. Petitioner sought a reconsideration
of the aforesaid order but the SEC denied the same. Respondents wrote
petitioner demanding their right and option to cancel their Contract, as it
appears that Laguna de Boracay is selling said shares without license or
authority from the SEC. But despite repeated demands, petitioner failed and
refused to refund or pay respondents. Respondents directly filed with SEC En
Banca Complaint against petitioner and the Members of its Board of
Directors for violation of Section 4 of Batas Pambansa Bilang (B.P. Blg.) 178.
The SEC En Banc rendered a Decision in favor of respondents, ordering
petitioner, together with Julius S. Strachan, Angel G. Vivar, Jr., and Cecilia R.
Palma, to pay respondents the amount of US$7,500.00.
ISSUE/S:
What securities are required to be registered?
RULING:
The provisions of B.P. Blg. 178 do not support the contention of
petitioner that its mere registration as a corporation already authorizes it to
deal with unregistered timeshares. Corporate registration is just one of
several requirements before it may deal with timeshares:
Section 8. Procedure for registration. - (a) All securities required to be
registered under subsection (a) of Section four of this Act shall be registered
through the filing by the issuer or by any dealer or underwriter interested in
the sale thereof, in the office of the Commission, of a sworn registration
statement with respect to such securities, containing or having attached
thereto, the following:
(36) Unless previously filed and registered with the
Commission and brought up to date:
A copy of its articles of incorporation with all
amendments thereof and its existing by-laws or

Page 1041 of 1072

instruments corresponding thereto, whatever the


name, if the issuer be a corporation.

TOPIC: WHAT SECURITIES ARE REQUIRED TO BE REGISTERED:


MAKATI STOCK EXCHANGE, INC., petitioner,
vs.
SECURITIES AND EXCHANGE COMMISSION and MANILA STOCK
EXCHANGE, respondents.
G.R. No. L-23004. June 30, 1965

Page 1042 of 1072

FACTS:
This is a review of the resolution of the Securities and Exchange
Commission which would deny the Makati Stock Exchange, Inc., permission
to operate a stock exchange unless it agreed not to list for trading on its
board, securities already listed in the Manila Stock Exchange.
Objecting to the requirement, Makati Stock Exchange, Inc. contends
that the Commission has no power to impose it and that, anyway, it is illegal,
discriminatory and unjust.
ISSUE:
Whether or not the order of the Securities and Exchange Commission,
that double or multiple listing of securities should be prohibited for the
protection of the investors, is correct?
RULING:
No.
The objection of Makati Stock Exchange, Inc., to this rule is
understandable. There is actually only one securities exchange, The Manila
Stock Exchange, that has been operating alone for the past 25 years; and all
or presumably all available or worthwhile securities for trading in the market
are now listed there. In effect, the Commission permits the Makati Stock
Exchange, Inc., to deal only with other securities. Which is tantamount to
permitting a store to open provided it sells only those goods not sold in other
stores. And if there's only one existing store, the result is a monopoly.
"Double listing of a security," explains the Commission, "divides the
sellers and the buyers, thus destroying the essence of a stock exchange as a
two-way auction market for the securities, where all the buyers and sellers in
one geographical area converge in one defined place, and the bidders
compete with each other to purchase the security at the lowest possible
price and those seeking to sell it compete with each other to get the highest
price therefore. In this sense, a stock exchange is essentially monopolistic."
As the purpose of the Act is to give adequate and effective protection
to the investing public against fraudulent representations, or false promises
and the imposition of worthless ventures, it is hard to see how the proposed
concentration of the market has a necessary bearing to the prevention of
deceptive devices or unlawful practices. For it is not mere semantics to
declare that acts for the protection of investors are necessarily beneficial to
them; but not everything beneficial to them is necessary for their protection.
And yet, the Commission realizes that if there were two or more exchanges

Page 1043 of 1072

"the same security may sell for more in one exchange and sell for less in the
other. Variance in price of the same security would be the rule ... ." Needless
to add, the brokerage rates will also differ.
The Legislature has specified the conditions under which a stock
exchange may legally obtain a permit (sec. 17, Securities Act); it is not for
the Commission to impose others. If the existence of two competing
exchanges jeopardizes public interest which is doubtful let the Congress
speak. Undoubtedly, the opinion and recommendation of the Commission will
be given weight by the Legislature, in judging whether or not to restrict
individual enterprise and business opportunities. But until otherwise directed
by law, the operation of exchanges should not be so regulated as practically
to create a monopoly by preventing the establishment of other stock
exchanges and thereby contravening:
a. the organizers' (Makati's) Constitutional right to
equality before the law;
b. their guaranteed civil liberty to pursue any lawful
employment or trade; and
c. the investor's right to choose where to buy or to sell,
and his privilege to select the brokers in his
employment.
And no extended elucidation is needed to conclude that for a licensing
officer to deny license solely on the basis of what he believes is best for the
economy of the country may amount to regimentation or, in this instance,
the exercise of undelegated legislative powers and discretion. Thus, it has
been held that where the licensing statute does not expressly or impliedly
authorize the officer in charge, he may not refuse to grant a license simply
on the ground that a sufficient number of licenses to serve the needs of the
public have already been issued.

TOPIC: WHAT SECURITIES ARE REQUIRED TO BE REGISTERED:


LA ORDEN DE PP. BENEDICTINOS DE LAS ISLAS FILIPINAS
vs.
J. A. STIVER and PHILTRUST
G.R. No. L-4568
June 16, 1953

Page 1044 of 1072

FACTS:
"La Orden" floated a bond issue, maturing from two to fifteen years, in
the total sum of P450,000 with the Philippine Trust," as trustee. As security
for the bonds, the former corporation executed a first mortgage and deed of
trust over certain parcels of land in the City of Manila, in favor of the latter
company, for the benefits of the bondholders. In 1941, most of the bonds
having matured and unpaid, the "Philippine Trust" instituted an action in the
Court of First Instance of Manila, for the purpose of selling the property
mortgaged, or portions thereof, and applying the proceeds to the redemption
of the matured bonds plus accrued interests.
The court placed the property mortgaged under receivership, and sold
the same during the Japanese occupation. An amount of Japanese occupation
currency, sufficient to redeem the entire bond issue, together with all
accrued interest, was then turned over by the receiver to the "Philippine
Trust" and the mortgage and deeds of trust was cancelled with the approval
of the court. In its order closing the receivership, the Court also released "la
Orden" from all obligation it contracted with respect to the bonds.
After receipt of the redemption money, the "Philippine Trust" notified
the bondholders, by publication, of its readiness to redeem the bonds. Some
of the bondholders surrendered their bonds and were paid the value thereof,
in the currency then existing. Others failed to do so, however, and as the
redemption money became worthless after the liberation of the Philippines
by the American Forces, their bonds have not been redeemed up to this
date.
ISSUE/S:
Whether or not the Securities and Exchange Commission erred in
ordering it to issue a replacement bond when the obligation
represented by the bond no longer exist because the bond had been
paid and the issuer thereof was discharged from its obligation.
RULING:
The phrases "which have not been fully paid, or for which no payment
whatsoever has been made," refers to subscription not fully paid or for which
no payment whatsoever has been made, and upon the satisfactory proof of
valid claims of ownership to such securities or interest therein, "the issuers
shall recognize all valid claims of ownership thereto or interest therein." The
cessation or extinction of the obligation of the issuer of the securities or
bonds to issue replacement securities or bonds upon proof of their loss or
destruction is not provided therein. The determination as to whether or not
the issuer of the securities or bonds had discharged its obligation, until
otherwise provided by law, lies within the province of the court of Securities
and Exchange Commission is powerless to pass upon it.

Page 1045 of 1072

The bondholder having established his ownership to ten bonds issued


by the petitioner at P500 each which had been lost or destroyed during the
battle for liberation of the City of Manila, the issuer of the bonds, the herein
petitioner, is in duty bound to issue bonds to replace them. If the lost or
destroyed bonds are not to be replaced by the issuance of other bonds, how
may the bondholder go to the trustee who had received the payment of the
mortgage obligation represented by the bonds to claim payment of their
bonds? the payment made by the issuer of the bonds of the whole amount of
the mortgage obligation or bonded indebtedness to the trustee who is still in
possession of part of the said amount has discharged the issuer from its
obligation to pay the bondholders, and whether the trustee after calling upon
the bondholders to receive the amount due to them upon their bonds has
been discharged from liability to the bondholders who have not been paid
because of their failure to call upon and receive from the trustee what is due
them upon their bonds, are matters foreign to the functions of the Securities
and Exchange Commission because they fall within the field of judicial
determination and adjudication.
J. A. Stiver, the bondholder, admits that he was paid P2,000 upon his
lost bonds. The replacement bond to be issued then must be for P3,000 only.
Of course, as clarified by the Securities and Exchange Commission in its
order of 17 May 1951, the directive to the Philippine Trust Company is just to
"certify and deliver" the replacement bond, as stipulated and provided for in
the Deed of Trust.

TOPIC: WHAT SECURITIES ARE REQUIRED TO BE REGISTERED:


PHILIPPINE STOCK EXCHANGE, INC., petitioner,
vs.

Page 1046 of 1072

THE HONORABLE COURT OF APPEALS, SECURITIES AND EXCHANGE


COMMISSION and PUERTO AZUL LAND, INC., respondents.
G.R. No. 125469. October 27, 1997
281 SCRA 232
FACTS:
TMBC acquired Manila Stock Exchange (MSE) Seat No. 97, registered in
the name of Roberto K. Recio (Recio) , through an execution sale which arose
from a levy on execution to satisfy a loan obligation of Recio to TMBC. TMBC
requested MSE to record its ownership of MSE Seat No. 97 in MSE's
membership books. MSE refused to register TMBC in its membership books
and contested the latter's ownership of said seat. According to MSE, its bylaws allow only individuals or corporations engaged primarily in the business
of stocks and bonds brokers and dealers in securities to be a member or to
hold a seat in the MSE.
TMBC settled for a mere acknowledgment from MSE of its legal or
naked ownership of, or proprietary right over, MSE Seat No. 97 which was
done by MSE through its Acknowledgment Letter.Before the aforementioned
acknowledgment of MSE's title, the Philippine Stock Exchange, Inc. (PSEI)
was incorporated unifying the MSE and the Makati Stock Exchange (MKSE)
into one exchange. The PSEI issued a certificate of membership to Recio as
Member No. 29. Believing that MSE Seat No. 97 became PSE Seat No. 29 of
the unified exchanges and that the certificate of membership to PSEI was
issued to Recio on the basis of his previous ownership of MSE Seat No. 97,
TMBC sought to rectify the PSEI's listing of Recio as a member without any
reservation or annotation therein that TMBC owns proprietary rights over PSE
Seat No. 29.
Armed with MSE's acknowledgment of its legal ownership or naked title
over MSE Seat No. 97, TMBC sought PSEI's recognition of its legal ownership
of PSE Seat No. 29. TMBC's efforts were met with PSEI's repeated refusal.
TMBC lodged a Petition for Mandamus with Claim for Damages, at the SEC
SICD, against herein petitioners PSEI and its Board of Governors.
The petition prayed that the SEC order the PSEI to acknowledge
TMBC's proprietary interest or legal or naked ownership of PSE Seat No. 29 to
enable TMBC to register said seat to a qualified nominee or otherwise sell the
same to a qualified vendee. Petitioners filed a motion to dismiss the
aforesaid action on the following grounds: the SEC had no jurisdiction to try
and hear the same. The SEC through its SICD Hearing Panel denied said
motion to dismiss.
ISSUE/S:

Page 1047 of 1072

Whether or not it was not ministerial duty on the part of the petitioners
to acknowledge the proeatry or legal ownweship of TMBC over PSE
Seat No. 29.
RULING:
As to the propriety of mandamus as a remedy, petitioners claim it was
not their ministerial duty to acknowledge the proprietary, legal or naked
ownership of TMBC over PSE Seat No. 29. True, the Court has invariably ruled
that generally, the performance of an official act or duty, which necessarily
involves the exercise of discretion or judgment, cannot be compelled by
mandamus. However, the Court has also declared that the general rule does
not apply in cases where there is gross abuse of discretion, manifest
injustice, or palpable excess of authority. These exceptions apply to the
present case. As aptly observed by the CA and the Court quote: It is beyond
cavil that the MSE had already recognized the legal or naked ownership of
private respondent to MSE Seat No. 97, but for reasons only known to them,
the PSE Board of Governors, who are members of the MSE, adamantly
refused to recognize the corresponding seat in the PSE. In fact, it is not
seriously disputed that MSE Seat No. 97 became PSE Seat No. 29 upon the
latter's incorporation. Petitioners' dubious claim that they could not
acknowledge the proprietary interest of respondent TMBC over the seat since
allegedly even respondent Roberto K. Recio was not a recognized member
due to his failure to so apply is belied by the facts. For one thing Mr. Recio
was issued a Certificate of Membership by the PSE. For another, Mr. Recio's
name has consistently appeared as a member of the PSE in the PSE's
Monthly Report. Given these facts, it cannot be gainsaid that petitioner's
refusal to acknowledge respondent TMBC's proprietary right over PSE Seat
No. 29 was grossly unjust and tyrannical and, therefore controllable by the
extraordinary writ of mandamus.
On a final note, on July 18, 2000, prior to the promulgation of the
assailed CA decision, Republic Act No. 8799 known as The Securities
Regulation Code was enacted and upon its effectivity, the SEC's jurisdiction
over this case was transferred to the courts of general jurisdiction or the
Regional Trial Courts.

TOPIC: EXEMPT TRANSACTIONS

Page 1048 of 1072

TIMESHARE REALTY CORPORATION, petitioner,


vs.
CESAR LAO and CYNTHIA V. CORTEZ, respondents.
G.R. No. 158941
February 11, 2008
544 S 254
FACTS:
On October 6, 1996, herein petitioner sold to Ceasar M. Lao and
Cynthia V. Cortez (respondents), one timeshare of Laguna de Boracay for
US$7,500.00 under Contract No. 135000998 payable in eight months and
fully paid by the respondents.
Sometime in February 1998, the SEC issued a resolution to the effect
that petitioner was without authority to sell securities, like timeshares, prior
to February 11, 1998. It further stated in the resolution/order that the
Registration Statement of petitioner became effective only on February 11,
1998. It also held that the 30 days within which a purchaser may exercise the
option to unilaterally rescind the purchase agreement and receive the refund
of money paid applies to all purchase agreements entered into by petitioner
prior to the effectivity of the Registration Statement.
On March 30, 1998, respondents wrote petitioner demanding their
right and option to cancel their Contract, as it appears that Laguna de
Boracay is selling said shares without license or authority from the SEC. But
despite repeated demands, petitioner failed and refused to refund or pay
respondents.
Respondents directly filed with SEC En Banca Complaintagainst
petitioner and the Members of its Board of Directors - Julius S. Strachan,
Angel G. Vivar, Jr. and Cecilia R. Palma - for violation of Section 4 of Batas
Pambansa Bilang (B.P. Blg.) 178.
ISSUE/S:
Whether or not the eventual approval or issuance of license has
retroactive effect and therefore ratifies all earlier transactions.
RULING:
NO.
The provisions of B.P. Blg. 178 do not support the contention of
petitioner that its mere registration as a corporation already authorizes it to
deal with unregistered timeshares. Corporate registration is just one of
several requirements before it may deal with timeshares:
Section 8. Procedure for registration. - (a) All securities required
to be registered under subsection (a) of Section four of this Act

Page 1049 of 1072

shall be registered through the filing by the issuer or by any


dealer or underwriter interested in the sale thereof, in the office
of the Commission, of a sworn registration statement with
respect to such securities, containing or having attached thereto,
the following:
xxxx
(36) Unless previously filed and registered with the Commission
and brought up to date:
(a) A copy of its articles of incorporation with all
amendments thereof and its existing by-laws or
instruments corresponding thereto, whatever the
name, if the issuer be a corporation.
Prior to fulfillment of all the other requirements of Section 8, petitioner
is absolutely proscribed under Section 4 from dealing with unregistered
timeshares, thus:
Section 4. Requirement of registration of securities. - (a) No
securities, except of a class exempt under any of the provisions
of Section five hereof or unless sold in any transaction exempt
under any of the provisions of Section six hereof, shall be sold or
offered for sale or distribution to the public within the Philippines
unless such securities shall have been registered and
permitted to be sold as hereinafter provided.

Page 1050 of 1072

TOPIC: EXEMPT TRANSACTIONS

NESTLE PHILIPPINES, INC., petitioner,


vs.
COURT OF APPEALS and SECURITIES AND EXCHANGE
COMMISSION, respondents.
G.R. No. 86738 November 13, 1991
203 SCRA 504
FACTS:
Sometime in February 1983, the authorized capital stock of petitioner
Nestle Philippines Inc. ("Nestle") was increased from P300 million to P600
million . Such increase was duly approved by the SEC.
Nestle has only two (2) principal stockholders: San Miguel Corporation
and Nestle S.A. The other stockholders, who are individual natural persons,
own only one (1) share each, for qualifying purposes, i.e., to qualify them as
members of the Board of Directors being elected thereto on the strength of
the votes of one or the other principal shareholder.
On 16 December 1983, the Board of Directors and stockholders of
Nestle approved resolutions authorizing the issuance of 344,500 shares out
of the previously authorized but unissued capital stock of Nestle, exclusively
to San Miguel Corporation and to Nestle S.A. San Miguel Corporation
subscribed to and completely paid up 168,800 shares, while Nestle S.A.
subscribed to and paid up the balance of 175,700 shares of stock.
On 28 March 1985, petitioner Nestle filed a letter, with the SEC seeking
exemption of its proposed issuance of additional shares to its existing
principal shareholders, from the registration requirement of Section 4 of the
Revised Securities Act and from payment of the fee referred to in Section
6(c) of the same Act. In that letter, Nestle requested confirmation of the
correctness of two (2) propositions submitted by it:
1) That there is no need to file a petition for exemption under
Section 6(b) of the Revised Securities Act with respect to
the issuance of the said 344,600 additional shares to our
existing stockholders out of our unissued capital stock; and
2) That the fee provided in Section 6(c) of [the Revised
Securities] Act is not applicable to the said issuance of
additional shares.
In respect of its claimed exemption from the fee provided for in Section
6(c) of the Revised Securities Act, Nestle contended that since Section 6 (a)
(4) of the statute declares (in Nestles view) the proposed issuance of

Page 1051 of 1072

344,500 previously authorized but unissued shares of Nestle's capital stock


to its existing shareholders as an exempt transaction, the SEC could not
collect fees for "the same transaction" twice. Nestle adverted to its payment
back in 21 February 1983 of the amount of P50,000.00 as filing fees to the
SEC when it applied for and eventually received approval of the increase of
its authorized capital stock effected by Board and shareholder action last 16
December 1983.

ISSUE/S:
Whether or not petitioner is entitled to the exemption particularly
under Section 6(a) (4) of the Revised Securities Act.

RULING:
Issuance of previously authorized but theretofore unissued capital
stock by the corporation requires only Board of Directors approval. Neither
notice to nor approval by the shareholders or the SEC is required for such
issuance. There would, accordingly, under the view taken by petitioner
Nestle, no opportunity for the SEC to see to it that shareholders (especially
the small stockholders) have a reasonable opportunity to inform themselves
about the very fact of such issuance and about the condition of the
corporation and the potential value of the shares of stock being offered.
Under the reading urged by petitioner Nestle of the reach and scope of the
third clause of Section 6(a) (4), the issuance of previously authorized but
unissued capital stock would automatically constitute an exempt transaction,
without regard to the length of time which may have intervened between the
last increase in authorized capital stock and the proposed issuance during
which time the condition of the corporation may have substantially changed,
and without regard to whether the existing stockholders to whom the shares
are proposed to be issued are only two giant corporations as in the instant
case, or are individuals numbering in the hundreds or thousands.
In contrast, under the ruling issued by the SEC, an issuance of
previously authorized but still unissued capital stock may, in a particular
instance, be held to be an exempt transaction by the SEC under Section 6(b)
so long as the SEC finds that the requirements of registration under the
Revised Securities Act are "not necessary in the public interest and for the
protection of the investors" by reason, inter alia, of the small amount of stock
that is proposed to be issued or because the potential buyers are very
limited in number and are in a position to protect themselves. In fine,
petitioner Nestle's proposed construction of Section 6(a) (4) would establish
an inflexible rule of automatic exemption of issuances of additional,

Page 1052 of 1072

previously authorized but unissued, capital stock. We must reject an


interpretation which may disable the SEC from rendering protection to
investors, in the public interest, precisely when such protection may be most
needed.
Petitioner Nestle's second claim for exemption is from payment of the
fee provided for in Section 6 (c) of the Revised Securities Act, a claim based
upon petitioner's contention that Section 6 (a) (4) covers both issuance of
stock in the course of complying with the statutory requirements of increase
of authorized capital stock and issuance of previously authorized and
unissued capital stock. Petitioner claims that to require it now to pay onetenth of one percent (1%) of the issued value of the 344,500 shares of stock
proposed to be issued, is to require it to pay a second time for the same
service on the part of the SEC. Since we have above rejected petitioner's
reading of Section 6 (a) (4), last clause, petitioner's claim about the
additional fee of one-tenth of one percent (1%) of the issue value of the
proposed issuance of stock (amounting to P34,450 plus P344.50 for other
fees or a total of P37,794.50) need not detain us for long. We think it clear
that the fee collected in 21 February 1983 by the SEC was assessed in
connection with the examination and approval of the certificate of increase
of authorized capital stock then submitted by petitioner. The fee, upon the
other hand, provided for in Section 6 (c) which petitioner will be required to
pay if it does file an application for exemption under Section 6 (b), is quite
different; this is a fee specifically authorized by the Revised Securities Act,
(not the Corporation Code) in connection with the grant of an exemption
from normal registration requirements imposed by that Act. We do not find
such fee either unreasonable or exorbitant.

Page 1053 of 1072

TOPIC: PUBLIC COMPANIES


PHILIPPINE VETERANS BANK vs. JUSTINA CALLANGAN
G.R. No. 191995

August 3, 2011

FATCS:
The Philippine Veterans Bank (the Bank) argued that it is not a
public company subject to the reportorial requirements under Section 17.1
of the SRC because its shares can be owned only by a specific group of
people, namely, World War II veterans and their widows, orphans and
compulsory heirs, and is not open to the investing public in general. The
Bank also requested the Court to take into consideration the financial impact
to the cause of veteranism; compliance with the reportorial requirements
under the SRC, if the Bank would be considered a public company, would
compel the Bank to spend approximately P40 million just to reproduce and
mail the Information Statement to its 400,000 shareholders nationwide.
ISSUE:
Whether or not that it should not be considered a "public company"?
RULING:
Rule 3(1)(m) of the Amended Implementing Rules and Regulations of
the SRC defines a public company as any corporation with a class of
equity securities listed on an Exchange or with assets in excess of Fifty
Million Pesos (P50,000,000.00) and having two hundred (200) or more
holders, at least two hundred (200) of which are holding at least one hundred
(100) shares of a class of its equity securities.
From these provisions, it is clear that a public company, as
contemplated by the SRC, is not limited to a company whose shares of stock
are publicly listed; even companies like the Bank, whose shares are offered
only to a specific group of people, are considered a public company, provided
they meet the requirements enumerated above.
The records establish, and the Bank does not dispute, that the Bank
has assets exceeding P50,000,000.00 and has 395,998 shareholders. It is

Page 1054 of 1072

thus considered a public company that must comply with the reportorial
requirements set forth in Section 17.1 of the SRC.
TOPIC:
TRADING
IN
SECURITIES:
RESTRICTIONS
ON BORROWING

MARGIN

REQUIREMENTS

AND

CAROLINA INDUSTRIES, INC., Petitioner,


vs.
CMS STOCK BROKERAGE, INC., (Formerly SISON, LUZ & JALBUENA
INC), CARLOS MORAN SISON, LUIS F. SISON, and the HON. COURT OF
APPEALS, Respondents.
G.R. No. L-46908 May 17, 1980
97 SCRA 734
FACTS:
Defendant CMS Stock Brokerage, Inc. (formerly Sison, Luz & Jalbuena,
Inc.), for the calendar year 1969, was a licensed securities broker and dealer
engaged, for compensation, in the business of buying and selling stocks and
securities for and in behalf of investors, such as the plaintiff. The CMS Stock
Brokerage, Inc. is a member firm of the Makati Stock Exchange. Defendant
Carlos Moran Sison is the president and at the same time the major and
controlling stockholder of defendant corporation. Defendants, in admitting
the foregoing facts, made the qualification that during the period from
January 10 to August 29, 1969, Arsenio N. Luz III was the president of
defendant corporation.
On or about June 17, 1969, plaintiff opened a margin account with
defendants for purchasing, carrying and selling stocks and securities listed in
the Makati Stock Exchange, as evidenced by a "Margin Account Agreement"
executed on that date by plaintiff through its treasurer and controlling
stockholder, Mariano T. Lim, and approved by defendant corporation, acting
through its vice-president and general manager, defendant Luis F. Sison.
Agreements of the parties was also included, as to the payment is
made and maintenance of the plaintiffs account and possible recourse of the
defendant in case of failure by the plaintiff to maintain its account.
During the period from June 17, 1969 to July 10, 1969, plaintiff Carolina
Industries, Inc. deposited with defendant CMS Stock Brokerage, Inc. various
cash amounts totaling P586,796.00, as partial payments to the debit
balance, and securities valued at P48,000.00, consisting of 400 shares of
Benguet Consolidated.
As of September 12, 1969, the account of plaintiff Carolina Industries,
Inc. with defendant CMS Stock Brokerage, Inc. had a debit balance of P804,

Page 1055 of 1072

179.69 against a security deposit with a market value of a little over a million
pesos. Its debit balance as of September 12, 1969, was over 70% of its
security deposit, or more than 20% over the 50% ceiling set by Section 18(a)
(1) of the Securities Act.
On September 15, 1969, defendant corporation purchased for
plaintiff's account 4,260 Marinduque shares worth P749, 985.00, and on
September 16, 1969, defendant corporation purchased also for plaintiff's
account 9,975 more Marinduque shares worth P1, 909,536.00.
Plaintiff's Mariano T. Lim drew P100, 000.00 on Manufacturers Bank on
a loose check pre-signed by plaintiff's president, Rafael Alvarez, dated
September 19, 1969. Then, Lim drew an additional P250, 000.00 on
Manufacturers Bank, on a loose check, also pre-signed by Alvarez, dated
September 20, 1969. Defendant Luis Sison, vice-president and general
manager of defendant corporation, issued a receipt for the two checks in
"payment for deposit". Lim drew a third check, also dated September 20,
1969, on Manufacturers Bank for P150, 000.00.
On September 24, 1969, Lim signed his endorsement on a PNB
Cashier's check for P500, 000.00, payable to plaintiff, and delivered it to
defendant Carlos Moran Sison. Defendant Luis Sison returned to Lim the
three Manufacturers Bank checks.
Lim drew another check for P250, 000.00, also pre-signed by plaintiff's
president, on Manila Banking Corporation, dated September 25, 1969. The
receipts issued therefore states, "For deposit into his account". The next day,
Lim obtained from the PNB a Cashier's check for P250,000.00, payable to
plaintiff and deposited it with the Manila Banking Corporation in time to
answer for the September 25, 1969 Manila Banking check which he had
delivered to defendant corporation.
Later, plaintiff stopped payment of both PNB Cashier's check of P500,
000.00 and TMBC check of P250, 000.00.
Defendants liquidated plaintiff's margin account with the sales of the
securities given as collaterals therefore.
ISSUE/S:
Whether or not the defendants extend to plaintiff excessive credit in
violation of Section 18 of the Securities Act?
RULING:
SEC. 18. Margin requirements. - (a) For the purpose of preventing the
excessive use of credit for the purchase or carrying of securities, the
Commission shall prescribe rules and regulations with respect to the amount
of credit that may be initially extended and subsequently maintained on any

Page 1056 of 1072

security (other than an exempted security) registered on a securities


exchange. For the initial extension of credit, such rules and regulations shall
be based upon the following standard:
An amount not greater than whichever is higher of
(1) Fifty per centum of the current market price of the security, or
(2) One hundred per centum of the lowest market price of the security
during the preceding thirty-six calendar months, but not more than
sixty-five per centum of the current market price.
xxx xxx xxx
12. In trading on margin a broker shall not extend credit to his
customers beyond the following maximum:
(a) on securities which are duly registered and/or licensed by the
Insular Treasurer or by the Commission, but not listed on any
exchange, 30 per centum of the current market value of the
securities;
(b) on securities duly registered and/or licensed by the Insular
Treasurer or by the Commission, and listed on an Exchange, an
amount whichever is the higher of:
(1) 40 per centum of the current market price of the
securities; or
(2) 100 per centum of the lowest market price of the
securities during the preceding 36 months but not more
than 50 per centum of the current market price.
It is noteworthy that the foregoing provisions enjoin the over-extension
of credit and not the application for excessive credit. It does not mean that
the customer to whom credit has been extended or for whom it has been
arranged has acted in violation of the Act or any rule or regulation there
under. The nature of the brokerage business is such that it is the broker, not
the client, who is in a position to verify, at any time, the status of the client's
account. It is only the broker, therefore, who can prevent the over-extension
of credit. In the instant case, it should be noted that petitioner did not know
the exact amount of its under margin, and that even after its request for a
statement of account, it was only some three months thereafter that private
respondents were able to comply.
In the case at bar, there was a clear violation of the said requirements
of the law as clearly established by the facts that during a period of three
months, from June 17, 1969 to September 12, 1969, respondent CMS Stock
Brokerage, Inc. extended loans or credit to the petitioner in the purchasing

Page 1057 of 1072

and carrying of securities under an agreement entitled "Margin Account


Agreement" . The loans were secured by collaterals consisting of registered
securities which were purchased thru respondent brokerage firm. It is not
disputed that the margin account of the petitioner with respondent
corporation was consistently under margin at all times during the period
from June 23, to September 12, 1969. In other words, its debit balance was
over the 50% ceiling of its security deposit set by Section 18 (a) (1) of the
Securities Act. Petitioner's total margin deposit amounted to only P634,
796.00 and that from June 23 to September 12, 1969, all credits extended to
it were over the ceiling allowed, as a result of which it was consistently under
margin. Thus, the percentage of petitioner's debit balance to the market
value of its security deposit deteriorated during the period from June 17 to
September 12, 1969, from 49.48% on June 17 to 90.39% on September 8;
90.55% on September 9; 82:93% on September 11; and 78.57 % on
September 12. This was a patent violation of Section 18 (a) of the Securities
Act, and Rule 12 of the Implementing Rules and Regulations.

Page 1058 of 1072

TOPIC: FRAUDULENT TRANSACTIONS


PHILIPPINE ASSOCIATION OF STOCK TRANSFER AND REGISTRY
AGENCIES, INC., Petitioner,
vs.
THE HONORABLE COURT OF APPEALS; THE HONORABLE SECURITIES
AND EXCHANGE COMMISSION; AND SEC CHAIRMAN PERFECTO R.
YASAY, JR., Respondents.
G.R. No. 137321. October 15, 2007

FACTS:
Petitioner is an association of Stock Transfer agents principally
engaged in the registration of stock transfers in the Stock-and-transfer book
of corporations and whose Board of Directors in a resolution unanimously
approved, allows its members to increase the transfer processing fee they
charge their clients from P 45 to P 75 per certificate and eventually to P 100
per certificate. The resolution also authorizes the imposition of a processing
fee for the cancellation of stock certificate at P 20 per certificate.

The SEC allowed petitioner to impose the P 75 per certificate transfer


fee and P 20 per certificate cancellation fee, but the approval of the P 100
transfer fee was withheld until after a public hearing.

Thereafter, the Philippine Association of Securities Brokers and Dealers,


Inc. registered its objection to the measure advanced by petitioner and
requested the SEC to defer its implementation.
Consequently, SEC advised petitioner to hold in abeyance the
implementation of the increases until the matter was cleared with all the
parties concerned. However, petitioner proceeded with the implementation

Page 1059 of 1072

of the increased fees. Thus, another letter from the SEC was issued strongly
urging petitioner to desist from implementing the new rates in the interest of
all participants in the security market.
On petitioners failure to follow the initial orders, the SEC issued Order
NO. 104, series of 1996 enjoining petitioner from imposing the new fees and
further ordered petitioner to pay basic fine of P5,000 and a daily fine of P500
for continuing violations.
On appeal filed by petitioner, the CA dismissed the petition.
Hence this Appeal.

ISSUE/S:
Whether the SEC acted with grave abuse of discretion or lack or
excess of jurisdiction in issuing the controverted Orders of July 8
and 11, 1996.

RULING:
No.
While this case was pending, The Revised Securities Act by authority of
which the assailed orders were issued was repealed by Republic Act No. 8799
or The Securities Regulation Code, which became effective on August 8,
2000. Nonetheless, the Court finds it pertinent to rule on the parties
submissions considering that the effects of the July 11, 1996 Order had not
been obliterated by the repeal of The Revised Securities Act and there is still
present a need to rule on whether petitioner was liable for the fees imposed
upon it.
Furthermore, The Court notes that before its repeal, Section 47 of The
Revised Securities Act clearly gave the SEC the power to enjoin the acts or
practices of securities-related organizations even without first conducting a
hearing if, upon proper investigation or verification, the SEC is of the opinion
that there exists the possibility that the act or practice may cause grave or
irreparable injury to the investing public, if left unrestrained.
Thus, The regulatory and supervisory powers of the Commission under
Section 40 of the then Revised Securities Act, in our view, were broad
enough to include the power to regulate petitioners fees. Indeed, Section 47
gave the Commission the power to enjoin motu proprio any act or practice of
petitioner which could cause grave or irreparable injury or prejudice to the
investing public. The intentional omission in the law of any qualification as to

Page 1060 of 1072

what acts or practices are subject to the control and supervision of the SEC
under Section 47 confirms the broad extent of the SECs regulatory powers
over the operations of securities-related organizations like petitioner.
The SECs authority to issue the cease-and-desist order being
indubitable under Section 47 in relation to Section 40 of the then Revised
Securities Act, and there being no showing that the SEC committed grave
abuse of discretion in finding basis to issue said order, we rule that the Court
of Appeals committed no reversible error in affirming the assailed orders. For
its open and admitted defiance of a lawful cease-and-desist order, petitioner
was held appropriately liable for the payment of the penalty imposed on it in
the SECs July 11, 1996 Order.
TOPIC: FRAUDULENT TRANSACTIONS
SECURITIES AND EXCHANGE COMMISSION, petitioner,
vs.
THE HONORABLE COURT OF APPEALS, CUALOPING SECURITIES
CORPORATION AND FIDELITY STOCK TRANSFERS, INC., respondents
G.R. NOS. 106425 & 106431-32, JULY 21, 1995
246 SCRA 738
FACTS:
CUALOPING is a stockbroker, while Fidelity Stock Transfer, Inc. is the
stock transfer agent of Philex Mining Corporation. The certificates of stock of
PHILEX representing one million four hundred [thousand] (1,400,000) shares
were stolen from the premises of FIDELITY. Said stolen stock certificates
ended in the hands of a certain Agustin Lopez, a messenger of New World
Security Inc., an entirely different stock brokerage firm. Lopez traded the
stolen
stock
certificates
to
CUALOPING
that
bore
the
"apparent" indorsement (signature) in blank of the owners (the stockholders
to whom the stocks were issued by PHILEX) and verification of the usual
initials of the officers of FIDELITY.
CUALOPING stamped each and every certificate with the words
"Indorsement Guaranteed," and thereafter traded the same with the stock
exchange. After the stock exchange awarded and confirmed the sale of the
stocks represented by said certificates to different buyers, the same were
delivered to FIDELITY for the cancellation of the stocks certificates and for
issuance of new certificates in the name of the new buyers. FIDELITY
rejected the issuance of new certificates in favor of the buyers for reasons
that the signatures of the owners of the certificates were allegedly forged.
ISSUE/S:

Page 1061 of 1072

Whether or not Cualoping & Philex are liable for the fraudulent
stocks of certificates.
RULING:
No.
Both FIDELITY and CUALOPING have been guilty of negligence in the
conduct of their affairs involving the questioned certificates of stock. To
constitute, however, a violation of the Revised Securities Act that can
warrant an imposition of a fine under Section 29 (3), in relation to Section 46
of the Act, fraud or deceit, not mere negligence, on the part of the offender
must be established. Fraud here is akin to bad faith which implies a
conscious and intentional design to do a wrongful act for a dishonest purpose
or moral obliquity; it is unlike that of the negative idea of negligence in that
fraud or bad faith contemplates a state of mind affirmatively operating with
furtive objectives. Given the factual circumstances found by the appellate
court, neither FIDELITY nor CUALOPING, albeit indeed remiss in the
observance of due diligence, can be held liable under the above provisions of
the Revised Securities Act.
Specifically, Section 29, in relation to Section 46, of the Revised
Securities Act provides: (a) It shall be unlawful for any person, directly or
indirectly, in connection with the purchase or sale of any securities (3) To
engage in any act, transaction practice, or course of business which operates
or would operate as a fraud or deceit upon any person.

Page 1062 of 1072

TOPIC: FRAUDULENT TRANSACTIONS


ONAPAL PHILIPPINES COMMODITIES, INC., petitioner,
vs.
THE HONORABLE COURT OF APPEALS and SUSAN CHUA, respondents.
G.R. No. 90707. February 1, 1993
281 SCRA 281
FACTS:
The petitioner, ONAPAL Philippines Commodities, Inc. (petitioner), a
duly organized and existing corporation, was licensed as commission
merchant/broker by the SEC, to engage in commodity futures trading in Cebu
City under Certificate of Registration No. CEB-182. On April 27, 1983,
petitioner and private respondent concluded a "Trading Contract". Like all
customers of the petitioner, private respondent was furnished regularly with
"Commodities Daily Quotations" showing daily movements of prices of
commodity futures traded and of market reports indicating the volume of
trade in different future exchanges in Hongkong, Tokyo and other centers.
Every time a customer enters into a trading transaction with petitioner as
broker, the trading order is communicated by telex to its principal, Frankwell
Enterprises of Hongkong. If the transaction, either buying or selling
commodity futures, is consummated by the principal, the petitioner issues a
document known as "Confirmation of Contract and Balance Sheet" to the
customer. An order of a customer of the petitioner is supposed to be
transmitted from Cebu to petitioner's office in Manila. From Manila, it should
be forwarded to Hongkong and from there, transmitted to the Commodity
Futures Exchange in Japan.
There were only two parties involved as far as the transactions covered
by the Trading Contract are concerned the petitioner and the private
respondents.

Page 1063 of 1072

Petitioner now contends that commodity futures trading is a legitimate


business practiced in the United States, recognized by the SEC and permitted
under the Civil Code, specifically Article 1462.
Petitioner further argues that the SEC, in the exercise of its powers,
authorized the operation of commodity exchanges to supervise and regulate
commodity futures trading.

ISSUE/S:
Whether the trading contract executed by the parties is a
fraudulent transaction.
RULING:
The trading contract signed by private respondent and Albert Chiam,
representing petitioner, is a contract for the sale of products for future
delivery, in which either seller or buyer may elect to make or demand
delivery of goods agreed to be bought and sold, but where no such delivery
is actually made. By delivery is meant the act by which the res or subject is
placed in the actual or constructive possession or control of another. It may
be actual as when physical possession is given to the vendee or his
representative; or constructive which takes place without actual transfer of
goods, but includes symbolic delivery or substituted delivery as when the
evidence of title to the goods, the key to the warehouse or bill of
lading/warehouse receipt is delivered. 12 As a contract in printed form,
prepared by petitioner and served on private respondent, for the latter's
signature, the trading contract bears all the indicia of a valid trading contract
because it complies with the Rules and Regulations on Commodity Futures
Trading as prescribed by the SEC. But when the transaction which was
carried out to implement the written contract deviates from the true import
of the agreement as when no such delivery, actual or constructive, of the
commodity or goods is made, and final settlement is made by payment and
receipt of only the difference in prices at the time of delivery from that
prevailing at the time the sale is made, the dealings in futures become mere
speculative contracts in which the parties merely gamble on the rise or fall in
prices. A contract for the sale or purchase of goods/commodity to be
delivered at future time, if entered into without the intention of having any
goods/commodity pass from one party to another, but with an understanding
that at the appointed time, the purchaser is merely to receive or pay the
difference between the contract and the market prices, is a transaction
which the law will not sanction, for being illegal.
The written trading contract in question is not illegal but the
transaction between the petitioner and the private respondent purportedly to
implement the contract is in the nature of a gambling agreement and falls

Page 1064 of 1072

within the ambit of Article 2018 of the New Civil Code, which is quoted
hereunder:
If a contract which purports to be for the delivery of goods,
securities or shares of stock is entered into with the intention
that the difference between the price stipulated and the
exchange or market price at the time of the pretended delivery
shall be paid by the loser to the winner, the transaction is null
and void. The loser may recover what he has paid.
The facts clearly establish that the petitioner is a direct participant in
the transaction, acting through its authorized agents. It received the
customer's orders and private respondent's money. As per terms of the
trading contract, customer's orders shall be directly transmitted by the
petitioner as broker to its principal, Frankwell Enterprises Ltd. of Hongkong,
being a registered member of the International Commodity Clearing House,
which in turn must place the customer's orders with the Tokyo Exchange.
There is no evidence that the orders and money were transmitted to its
principal Frankwell Enterprises Ltd. in Hongkong nor were the orders
forwarded to the Tokyo Exchange. We draw the conclusion that no actual
delivery of goods and commodity was intended and ever made by the
parties. In the realities of the transaction, the parties merely speculated on
the rise and fall in the price of the goods/commodity subject matter of the
transaction. If private respondent's speculation was correct, she would be the
winner and the petitioner, the loser, so petitioner would have to pay private
respondent the "margin". But if private respondent was wrong in her
speculation then she would emerge as the loser and the petitioner, the
winner. The petitioner would keep the money or collect the difference from
the private respondent. This is clearly a form of gambling provided for with
unmistakeable certainty under Article 2018 abovestated. It would thus be
governed by the New Civil Code and not by the Revised Securities Act nor
the Rules and Regulations on Commodity Futures Trading laid down by the
SEC.
Article 1462 of the New Civil Code does not govern this case because
the said provision contemplates a contract of sale of specific goods where
one of the contracting parties binds himself to transfer the ownership of and
deliver a determinate thing and the other to pay therefore a price certain in
money or its equivalent. The said article requires that there be delivery of
goods, actual or constructive, to be applicable. In the transaction in question,
there was no such delivery; neither was there any intention to deliver a
determinate thing.
The transaction is not what the parties call it but what the law defines it to
be.
After considering all the evidence in this case, it appears that
petitioner and private respondent did not intend, in the deals of purchasing

Page 1065 of 1072

and selling for future delivery, the actual or constructive delivery of the
goods/commodity, despite the payment of the full price therefor. The
contract between them falls under the definition of what is called "futures".
The payments made under said contract were payments of difference in
prices arising out of the rise or fall in the market price above or below the
contract price thus making it purely gambling and declared null and void by
law.

TOPIC: INSIDERS DUTY TO DISCLOSE WHEN TRADING: MATERIAL OR


SIGNIFICANT FACTS
ELEANOR ERICA STRONG, ET AL.
VS.
FRANCISCO GUTIERREZ REPIDE
G.R. No. L-7154.February 21, 1912
FACTS:
Eleanor Erica Strong, was the owner of 800 shares of the capital stock
of the Philippine Sugar Estates Development Company, Limited (sociedad
anonima). Francisco Gutierrez Repide, by means subsequently found and
adjudged to have been fraudulent, obtained possession of said shares and
thereafter alleged to be the owner thereof. Eleanor Erica Strong commenced
an action against the defendant asking that the fraudulent sale by means of
which the defendant obtained possession of the said shares be declared null
and void and that they be returned to her.
The court, therefore, declares that the purchase of these shares of
stock by the defendant is fraudulent and void.

Page 1066 of 1072

The satisfaction was effected by means of a stipulation or agreement


entered into between the attorneys for the plaintiff and the defendant, in
which the satisfaction of the judgment was acknowledged by both parties.
After demand upon and refusal by the Francisco Gutierrez, Eleanor
Erica Strong began this action for the recovery of said sum. The court
rendered judgment in favor of the plaintiff. The appellant in this case relies
for the success of this appeal upon the form of the judgment of the court
below in said action No. 2365. He asserts that that judgment is for a sum of
money and not for the rescission of a contract and the return of shares of
stock. Basing himself upon this contention appellant asserts that that
judgment having been satisfied by the payment of the sum adjudged to be
due, a subsequent action for dividends on said stock is in effect an action for
interest on the said sum found to be due, that it affects the subject matter of
a judgment already paid and discharged.
ISSUE/S:
Whether or not the judgement in question was paid by the
stipulation or agreement was entered into between him and the
plaintiff, by virtue of which released him from all responsibility in
connection with the transaction relating to the stock.
RULING:
It is true that plaintiff could have included in her action and recovered
at the most only those dividends which were due at the time judgment in her
favor was entered. It happens in this case that most of the dividends became
payable after the plaintiff had secured her judgment. That being so, they
could not have been included by her in the original complaint, not could they
have been incorporated within the judgment in that action. This, then,
furnishes another reason why the contention of the appellant in this regard
cannot be sustained. Under such circumstances a plea of multiplicity, even if
made, would not have been available as to those dividends which became
payable after the judgment was entered in that action.
The remaining question presented by appellant relates to the interest
which he was entitled to recover or the amount due him from the plaintiff. As
we have already seen, the judgment of the court in the first place gave him
the interest on said amount from the 10th day of October, 1903, to the 27th
day of July, 1909. On motion made by the plaintiff the court amended that
judgment by giving the defendant interest on said sum from the 10th day of
October, 1903, to the 12th day of January, 1904. The reason for the
amendment was the fact, as disclosed by the proofs, that on the latter date
the plaintiff tendered to the defendant said sum of money and the defendant
at that time refused to accept the same. Under such circumstances, the

Page 1067 of 1072

court properly held that the tender of the sum and its refusal by the
defendant stopped the running of interest in favor of the latter and he was
not, therefore, entitled to recover interest from that day forward. The
appellant argues in this connection that he should not be blamed or
punished for the refusal to accept the tender of the plaintiff for the reason
that he was not the owner of the stock at the time of such tender and,
therefore, could not accept it. As we have already seen in touching another
question raised on this appeal, the court, in a judgment now final, found that
the sale of stock afterwards declared fraudulent was executed between the
plaintiff and the defendant. As to this there can be no question. As a
necessary result the plaintiff need look for her redress no further than the
defendant himself and she could produce all of the legal effects possible in
her favor by dealing directly with him, as she did when she made the tender
in question.

TOPIC:DISCLOSURE REGULATIONS FOR PUBLICLY- LISTED SHARES


UNION BANK OF THE PHILIPPINES, petitioner,
vs.
SECURITY AND EXCHANGE COMMISSION, respondent.
G.R. No. 138949.June 6, 2001
FACTS:
On April 4, 1997, petitioner, through its General Counsel and Corporate
Secretary, sought the opinion of Chairman Perfecto Yasay, Jr. of respondent
Commission as to the applicability and coverage of the Full Material
Disclosure Rule on banks, contending that said rules, in effect, amend
Section 5 (a) (3) of the Revised Securities Act which exempts securities
issued or guaranteed by banking institutions from the registration
requirement provided by Section 4 of the same Act.
Chairman Yasay, in a letter dated April 8, 1997, informed petitioner
that while the requirements of registration do not apply to securities of banks
which are exempt under Section 5 (a) (3) of the Revised Securities Act,
however, banks with a class of securities listed for trading on the Philippine

Page 1068 of 1072

Stock Exchange, Inc. are covered by certain Revised Securities Act Rules
governing the filing of various reports with respondent Commission, i.e., (1)
Rule 11 (a)-1 requiring the filing of Annual, Quarterly, Current, Predecessor
and Successor Reports; (2) Rule 34-(a)-1 requiring submission of Proxy
Statements; and (3) Rule 34-(c)-1 requiring submission of Information
Statements, among others.
On May 9, 1997, respondent Commission, through its Money Market
Operations Department Director, wrote petitioner, reiterating its previous
position that petitioner is not exempt from the filing of certain reports. The
letter further stated that the Revised Securities Act Rule 11 (a) requires the
submission of reports necessary for full, fair and accurate disclosure to the
investing public, and not the registration of its shares.
ISSUE/S:
Whether or not petitioner is required to comply with the
respondent SEC's full disclosure rules.
RULING:
It must be emphasized that petitioner is a commercial banking
corporation listed in a stock exchange. Thus, it must adhere not only to
banking and other allied special laws, but also to the rules promulgated by
Respondent SEC, the government entity tasked not only with the
enforcement of the Revised Securities Act, but also the supervision of all
corporations, partnerships or associations which are grantees of governmentissued primary franchises and/or licenses or permits to operate in the
Philippines.
RSA Rules 11 (a)-1, 34 (a)-1 and 34 (c)-1 require the submission of
certain reports to ensure full, fair accurate disclosure of information for the
protection of the investing public. These Rules were issued by the
respondent pursuant to the authority conferred upon it by Section 3 of the
RSA.
The said Rules do not amend Section 5(a)(3) of the Revised Securities
Act, because they do not revoke or amend the exemption from registration of
the securities enumerated thereunder. They are reasonable regulations
imposed upon petitioner as a banking corporation trading its securities in the
stock market.
That petitioner is under the supervision of the Bangko Sentral ng
Pilipinas (BSP) and the Philippine Stock Exchange (PSE) does not exempt it
from complying with the continuing disclosure requirements embodied in the
assailed Rules. Petitioner, as a bank, is primarily subject to the control of the
BSP; and as a corporation trading its securities in the stock market, it is
under the supervision of the SEC. It must be pointed out that even the PSE is

Page 1069 of 1072

under the control and supervision of respondent. 14 There is no oversupervision here. Each regulating authority operates within the sphere of its
powers. That stringent requirements are imposed is understandable,
considering the paramount importance given to the interests of the investing
public.
Otherwise stated, the mere fact that in regard to its banking functions,
petitioner is already subject to the supervision of the BSP does not exempt
the former reasonable disclosure regulations issued by the SEC. These
regulations are meant to assure full, fair and accurate disclosure of
information for the protection of investors in the stock market. Imposing such
regulations is a function within the jurisdiction of the SEC. Since petitioner
opted to trade its shares in the exchange, then it must abide by the
reasonable rules imposed by the SEC.

TOPIC:DEVICES
OR
MISREPRESENTATION

SCHEMES

AMOUNTING

TO

FRAUD

OR

CEMCO HOLDINGS, INC.


vs.
NATIONAL LIFE INSURANCE COMPANY OF THE PHILIPPINES, INC.
G.R. No. 171815. August 7, 2007
FACTS:
Union Cement Corporation , a publicly-listed company, has two
principal stockholders UCHC, a non-listed company, with shares amounting
to 60.51%, and petitioner Cemco with 17.03%.Majority of UCHCs stocks
were owned by BCI with 21.31% and ACC with 29.69%.Cemco, on the other
hand, owned 9% of UCHC stocks.
BCI informed the Philippine Stock Exchange (PSE) that it and its subsidiary
ACC had passed resolutions to sell to CemcoBCIs stocks in UCHC equivalent
to 21.31% and ACCs stocks in UCHC equivalent to 29.69%.

Page 1070 of 1072

It was stated that as a result of petitioner Cemcos acquisition of BCI and


ACCs shares in UCHC, petitioners total beneficial ownership, direct and
indirect, in UCC has increased by 36% and amounted to at least 53% of the
shares of UCC
As a consequence of this disclosure, the PSE, in a letter to the SEC, inquired
as to whether the Tender Offer Rule under Rule 19 of the Implementing Rules
of the Securities Regulation Code is not applicable to the purchase by
petitioner of the majority of shares of UCC.
Director Justina Callangan
responded to the query of
department that the tender
still have to be confirmed by

of the SECs Corporate Finance Department


the PSE that while it was the stance of the
offer rule was not applicable, the matter must
the SEC en banc.

Thereafter, in a subsequent letter dated 27 July 2004, Director Callangan


confirmed that the SEC en banc had resolved that the Cemco transaction
was not covered by the tender offer rule.
On 28 July 2004, feeling aggrieved by the transaction, respondent National
Life Insurance Company of the Philippines, Inc., a minority stockholder of
UCC, sent a letter to Cemco demanding the latter to comply with the rule on
mandatory tender offer. Cemco, however, refused.

ISSUE/S:
Whether or not the SEC has jurisdiction over respondents
complaint and to require Cemco to make a tender offer for
respondents UCC shares.
RULING:
Yes.
SEC was acting pursuant to Rule 19(13) of the Amended Implementing
Rules and Regulations of the Securities Regulation Code. The foregoing rule
emanates from the SECs power and authority to regulate, investigate or
supervise the activities of persons to ensure compliance with the Securities
Regulation Code, more specifically the provision on mandatory tender offer
under Section 19 thereof.
We are nonetheless convinced that the SEC has the competence to
render the particular decision it made in this case. A definite inference may
be drawn from the provisions of the SRC that the SEC has the authority not
only to investigate complaints of violations of the tender offer rule, but to

Page 1071 of 1072

adjudicate certain rights and obligations of the contending parties and grant
appropriate reliefs in the exercise of its regulatory functions under the SR
.Section 5.1 of the SRC allows a general grant of adjudicative powers to the
SEC which may be implied from or are necessary or incidental to the carrying
out of its express powers to achieve the objectives and purposes of the SRC.
We must bear in mind in interpreting the powers and functions of the SEC
that the law has made the SEC primarily a regulatory body with the
incidental power to conduct administrative hearings and make decisions. A
regulatory body like the SEC may conduct hearings in the exercise of its
regulatory powers, and if the case involves violations or conflicts in
connection with the performance of its regulatory functions, it will have the
duty and authority to resolve the dispute for the best interests of the public.
The power conferred upon the SEC to promulgate rules and regulations
is a legislative recognition of the complexity and the constantly-fluctuating
nature of the market and the impossibility of foreseeing all the possible
contingencies that cannot be addressed in advance. Rules and regulations
when promulgated in pursuance of the procedure or authority conferred
upon the administrative agency by law, partake of the nature of a statute,
and compliance therewith may be enforced by a penal sanction provided in
the law.
Moreover, petitioner is barred from questioning the jurisdiction of the
SEC. It must be pointed out that petitioner had participated in all the
proceedings before the SEC and had prayed for affirmative relief.

Page 1072 of 1072

Anda mungkin juga menyukai